2003-2013 March June FCPS part 1 papers

March 24, 2018 | Author: moizzafar999 | Category: Cerebellum, Heart, Atrium (Heart), Phenylalanine, Spinal Cord


Comments



Description

https://www.facebook.com/groups/fcpspastpaper/ https://www.facebook.com/FcpsPart1BcQs?ref=hl Past Papers 2003- June 2013 11200 BCQs Composed and Compiled by Dr Dileep Kumar https://www.facebook.com/groups/fcpspastpaper/ https://www.facebook.com/FcpsPart1BcQs?ref=hl June 2013 1.Scenario: Lady with dry eyes and dry mouth... diagnosis is a.Sjogren(Ans) b.Milkulicz syndrome? 2 Smooth and cordinated movement by a. basal ganglia(Ans) b. spinocerebellar 3.Scenario: elderly,diplopia,granuloma,raised b.p a. giant cell arteritis(Ans) b. takayasu disease c. wegeners Granulomatosis 4. Zinc required for a cellular oxiadtion(Ans) b glucose oxidation c 5. Maxillary artery form a 1st arch(Ans) b 2nd arch 6.Testosterone causes transient bone growth because? a it causes epiphyseal closure(ans) b downgrade its receptors c 17 ketosteroid 7 GFR decreased by a afferent arteriolar contraction(ANS) b.renin increases c.efferent arteriolar constriction 8 Aotosomal recessive a. Achondroplasia b. Marfans syn c. Alpha1 antitrypsin defiecncy(ANS) d. Neurofibramatosis 9 Nucleus a continous with rough endoplasmic reticulum(Ans) b conatin pores 10 basophila in cell due to which organelle? a ribosomes(Ans) b glogi apparatus 11 fatty liver NOT seen in a viral hepatitis(ans) b congestion of liver c pregnancy d obesity e alcohol12 scenario: pt with increased heaptic glycogen with normal structure, no inc in serum glucose after giving fructose,enzyme deficiency a fructokinase b glucose 6 phospahate(Ans) https://www.facebook.com/groups/fcpspastpaper/ https://www.facebook.com/FcpsPart1BcQs?ref=hl c hexokinase 13 scenario 6 yrs old, given anti malarial, hb low, ad coke cloured urine,( inc tlc n platelets were also there may b) a g6pd deficincy(Ans) b black water fever c Paroxysomal nocturnal hemoglobinuria 14 vasodilators a PGI2(ans) b ltc4 c ltd4 d TXA2 15 difference bw kwashiorkor and marsmus a bilateral pitting edema(Ans) b muscle wasting 16 Two point discrimination sensor a mesiners corpuscles b merkels disk c Golgi tendons d free nerve e muscle spindle 17 which is not a direct effect of cushing syndrome? a facial hair growth(ans) b buffalo hump c muscle weakness d thin limbs 18 hormone required for brain developlment in fetal life a thyroid hormone(Ans) b growth hormone c cortisol d androgens 19 scenario, mother rh negative, baby delivered at 34 weeks with anemia palour, died after 6 hrs after transfusion, most likely cause a. basal ganglia staining(Ans) b. cardiac malformations 20 post ganglionic sympathic nerve fibers r present in1.cervical nerve 2.all spinal nerve(ans) 3.thoracic n(ans)?? Thoraco lumbar? 4.lumber 5.sacral 21 insulin increased by a beta blockers b secretin (ans)(its homologous to glucagon) c thiazides 22 virus act carcinogecially thru a oncogenes(Ans) (in virus are oncogenes, in humans are protoncogens) b protooncogenes c promotor 23 radiation causes carcinoma in how many years? a. les than 1 yr https://www.facebook.com/groups/fcpspastpaper/ https://www.facebook.com/FcpsPart1BcQs?ref=hl b. less than 2 yrs. c. 2-5 yrs d > 10 yrz e. 7-10 yrs(ans) (Radiation can cause cancer in most parts of the body, in all animals, and at any age, although radiation-induced solid tumors usually take 10–15 years, and can take up to 40 years, to become clinically manifest, and radiation-induced leukemias typically require 2–10 years to appear) 24 atrial flutter a treated with defibrillation shock therapy(Ans) b heart rate is so rapid above 400 beats / min c heart beat between 200 and 350/min(??) d circus waves are not main reason 25 carcinogen a nitrosamides(Ans) b nicotine c cadmieum 26 u wave a slow reploarization of papillary muscles(ans)(its due to hypokalemia)(1st aid) b atrial repolarization c.SA depolarization d.AV depolarization e septum depolarization 27 purkinje fibes a specialized heart muscle cells b contain numerous myofibrils 28 hb 6gm/dl increased plateled count with hyperplastic bone marrow? a.iron def anemiab.hemolytic anemia c.acute blood loss d.sideroblastic anemia e.leukemia(?) 29 clostridium a are spores forming(Ans) b are facultative anerobes( they are obligate anerobes) 30 pseudomembrane colitis least likey caused by a ceftriaxone b clauvinic ampicllin(Ans) c clindamycin d vancomycin(it’s the treatment of pseudomembrane colitis) 31 virus a altered protien synthesis 32 In right atrium, SA node is located just near the A- Auricle B- Fossa Ovalis C- Superior part of Sulcus Terminale(Ans) D- Lower part of Crista Terminalis 33 which of following DOES NOT drain in internal juglar vein a ext. juglar(ans) b sup. thyroid https://www.facebook.com/groups/fcpspastpaper/ https://www.facebook.com/FcpsPart1BcQs?ref=hl 34 soldeir came from high attitude , 2000 ft , pcv 60 %,malaise,fever, spleenomegaly, cyanosis a sec polycythemia(ans)?? b methmaglobinemia 35 Men from sea to attitude a pul vasoconstriction(Ans) 36 increased diffusion of CO2 through alveolar membrane is due to? a increased diffusion coefficient (20 times more ability to diffuse) b increased solubility c more affinity then o2(Asim shoaib) 37 water intoxication a distended neck veins b pulmonary edema(cerebral edema happens) c slow pulse(Ans)(Asim shoaib) d Increased alertness(guyton) e thirst 38 total water depletion, replaced by? a hypotonic (ans by some ppl.. logically fit) b isotonic c 5% dextrose( ans by most) d 1% glucosee ringer lactate 39 unconcious patient brought to hospital after 1 hour of severe haemoraghe his bp is 50 mmhg,pulse is rapid, cool n calmy skin, which mechanism is most active at this stage a baroreceptors reflex b cerebral ischemic(ans)(its start acting below 60mmHg of BP ) c cushing reaction d chemoreceptors 40 po2 60 mmhg, hgb 15. cause of injury a hypoxic b anemia c overutilization d histotoxic 41 integrins at a leukocytes(Ans) b platelets c endothelial cells 42 primary amylodosis of increase immunoglobulin light chain a systemic(Ans) b reactive systemic 43 superior parathyroid a at the level of middle of thyroid gland on posterior surface at 1st tracheal ring( its actually 3rd ring )(ans nearly to right) b variable in position c in frront of pretracheal fascia d supplied by superior thyroid artery(inferioris the main supply) 44 cricoid level a oesophagus with pharynx(Ans) b common carotid bifurcation level 45 vertical lesion of optic chiasma https://www.facebook.com/groups/fcpspastpaper/ https://www.facebook.com/FcpsPart1BcQs?ref=hl a bitemporal hemianopia 46 patient has done clostomy, its now on parenteral diet a fatty liver b hyperglycemia(Ans) c hypophosphotemia 47 opiods are agonist, one with antagonist a pethidine b buprenorphine(ans) c pentazocine(also correct bt is prototype) d morphine e fentanyl 48 Regarding ANTI-EPILEPTIC DRUGS,which one is INCORRECTa dizipam inhibits gaba channels b ethusuxsimide act via blocking of K channels(ans)(its Ca channels) c. phenobarbitaol… d phenytoin prolongs the inactivated stated on Na Channels e vigabatrin irreversibly block the enzyme 49 Lumbar puncture done through: A- L2 & L3 B- L4 & L5(ans) C- S1- S2 50 subduaral hemaorhage a superoir cerebral veins(ans) b middle meningeal artery 51 most dangerous effect of diphtheria exotoxin on which organ? a heart(Ans) b.Larynx c.Intestine 52 renin secretion is increased by? a decreased atrial strech b macual densa? c plasma osmolarity( Ans)? d inc K+ ? 53 most characteristic symptom of esophageal atresia? a regurgitation of all of first feed b attacks of cough on feed (this one also by Wikipedia) c continuous dribbling(ans) (Asim shoaib) 54 after adrenalectomy, cause of death within few days is coz of absent of a aldosterone (Ans) Addison’s is due to mineralocorticoid deficiency basically b cortisol? 55 which of the following is not neoplasia 1. choriostoma(ans) 2. memgioma 3. lymphoma 56. 40 yrz old lady with swelling n pain in knee joint, synovial fluid shows basophilic rhomboid crystals: a. calcium pyrophosphate(ans)(psedogout) b. monosodium urate c. glycine 57 90 % of anthrax pts have lesions in https://www.facebook.com/groups/fcpspastpaper/ https://www.facebook.com/FcpsPart1BcQs?ref=hl a.skin(ans) b.lungs58. Glucose transport is directly proportional to it's concentration, this phenomenon is known as A -Active Transport B- Diffusion (ans? Mostly say this) C- Facilitated Diffusion(ans)? 59 nutrient which need further digestion to be absorbed a sucrose(ans) b tripeptides 60 high triglyceride content a chylomicrons(Ans) 61 cholesterol with negative coronary risk factor a hdl(Ans) b ldl ? 62 counter current multiplier a requires hypertonic medullary interstitium(Ans) b produce by vasa recta(mainitaineb by it 63 preganlonic nerve fibres contain a B fibres(Ans) 64 muscle tension is decreased to prevent muscle rupture by a golgi tendons??(Ans) b muscle spindle ?? 65 durg to irreversibly impair platelet function a aspirin(Ans) b ibuprofen 66 A patient comes to a doctor with a complain of pain down the medial side of right forearm & hand, & wasting of small muscles of hand. On examination ,circulation of right upper limb was also impaired. The anatomical structure most likely to be involved is? A. Head of right humarus B. Left cervical rib C. Right 1st rib D. Right 2nd rib E. Right cervical rib(Ans) 67 Pt presented with hemoptysis & hematuria. It progressed to renal failure. On biopsy there were focal depositions with linear pattern A- Focal glomerulonephritis B- Rapid Progressive Glomerulonephritis C- Good pasture's (Ans)68 scenario male 30 years splenomagaly fever malaise cardiac murmer and splinter hemorrhages and hematuria . Most appropriate test a.Blood serology b.Blood culture(ans) c.Stool culture 69 scenario some weeks of disease, fever, lethargic, creatinine was 4, pedal edema n anemia hb,6 rbc 3.6, cause a erythropoiten deficiency(Ans)(i.e CRF) b acute renal failure( acute dose not cause anemia) https://www.facebook.com/groups/fcpspastpaper/ https://www.facebook.com/FcpsPart1BcQs?ref=hl 70 sceanrio: pt of acromegaly.. lips n bone enlarged a adenoma of adenohypophsis b adeoma of acidophilic cells(Ans) (acidophilic cells include somatotrohs and mammaotrohs ) c adenoma of chromphobic cells 71 most likely about mammary gland a extend vertically from 2 to 6 rib( its right also) b contain 15 to 20 lactiferous ducts(Ans)? c in pregnancy, contain lactiferous ducts and secretory alveoli d sebaceous glands in aereola 72 scenario of post op. abdominal surgery, thromboembloism cause? a stasis n hypercoagubility(Ans) b stasis c endothelial injury and stasis d endothelial injury and hypercoagulability 73 biceps reflex absent, knee n ankle reflex hyper, ventilation rate 16 / min, cause spinal cord section a L1 l2 l3 b c8 t1 c t8 t9 d c5 c6(ans) e c3 c4 74 ventrical cortico spinal fibres ends on a lumbar b mid thoracic(ans)(Wikipedia)(its small as compared to main lateral corticospinal and ends gradually on mid thoracic) c thoracic d end of spinal cord 75 standing from supine position a incraese heart rate(ans) b decrease cardiac output 76 not of cardiogenic shock a hemaorragheb MI c CHF d dehydration 77. Water enter interstitium a.filtration(Ans) b.diffusion c.cotransport with Na d. decrease hydrostatic pressure 78 most evident clinical feature of sjogrens a enlargement of salivary gland b dry mouth n dry eyes c dry mouth only 79 leprosy a intracellular acid fast b intraceelualar non acid fast c extracelluar acid fast 80 scenario: child with skin eruptions along facial hair line, conjunctivitis, bluish white https://www.facebook.com/groups/fcpspastpaper/ https://www.facebook.com/FcpsPart1BcQs?ref=hl specs with red margins at buccal mucosa (measles) a kopliks spot 81 Not oro fecal a triculais spirailis(Ans) b triculais tachuli 82 hepatits B a by sexual contact(Ans) b oro fecal 83 not a 2nd line drug in Rheumatoid arthritis a antibiotics(Ans) b anti malarials c ?golds e sulphazine 84 from under cook meat a tape worm(ans) b liver fluke 85 smooth muscles a arrange in longitudinal and circular layer(Ans) b striated muscle 86 post traumatic patient, memory loss, area a hippocampus(ans) 87 heart muscle show no tetnus a increase absolute refractory period88. Pancreatic fistula, somatostatin a inhibit other enzymes b dec secretions(ans?) 89 somatosensory different from primary motor in a granular layer(ans?) b at lateral side c has lager homunculus 90 A patient in an operation requiring general anaesthesia has high BP which drugs should be given a glycerine nitride? b halothane c hydralizine(Ans) d metoprolol 91 regarding medulla, correct one a forth ventricle in upper medulla 92 A 40 year Male working as a taxi driver in sharjah presented with C/O fever,weight loss,anorexia. Diarrehea, spleenomegaly,he is emaciated,has jaundice and generalized lymphadenopathy.whats the diagnosis. a AIDS(ans) b leishmaniasis c yellow fever 93 Turner syndrome a. 44XO(ans) b. 44XXX c. 44XXXX 94 not present in AIDS https://www.facebook.com/groups/fcpspastpaper/ https://www.facebook.com/FcpsPart1BcQs?ref=hl a sqaumous cell ca(Ans) b primary brain lyhpoma c kaposis’s 95 scenario: nodule on dorsum of hand,same side axillary lymph nodes are enlarged, happened due to? (melanoma case) a ultra violet radiations(Ans) b old job related asbestos c latex glove allergy 96 coronary sinus co tinuation ofA graet cardiac vein 97 A pt came to gingival hypertrophy, bleeding from gums, petachies, painful sore at lips..you advice investigation to.rule out a.Lyphoma?? b.Lukemia?? c. bachet disease 98 souce of creatinine a muscle(Ans) b heart 99 pt with diabetic nephropathy, investigation of chioce a albumin(ans) b serum creatinine levels c creatinine clearance 100 scenario, pt having swellin on right neck, dicsharging, routine culture shows nothing. now first initial investigation for diagnosis a biopsy(Ans)(it will reveal everything) b zehil stain 101 question regarding ovary’s embryology a all primary ooctyes go in prophase of first meotic in fetal life(ans) b most of oocyte undergo degeneration from birth to puberty c oocyte completes secondary miosis before ovulation something like that 102 which structure seperate maternal and fetal blood at last stages of pregnancy a Cytotrophoblast b.Syncishiotrophoblast c.Cyto and synchio d.Synschiotrophoblast and fetal endothelium(Ans) 103 2nd division of trigeminal nerve NOT supply to a one air sinus b nasal cavity c nasal pharynx? d temporal bone 104 Female having HIV but no AIDS CD count 800 now having middle ear infection SOM (supartive otits media), cause a pneumocytis carinii(below 500 typical infections of aids get start) b streptococci pneumonia c M avium d psedomanas aueriginosa ( cause ext otits media) e staph aureus105 men having abcess at left thigh after injury from thorn, cause a staph aureus https://www.facebook.com/groups/fcpspastpaper/ https://www.facebook.com/FcpsPart1BcQs?ref=hl b strept pyogens 106 19 yr old boy sitting in a well ventilated room of 26'C. severely sweating , His pulse 120 beats/min, Bp 150/90. What's the diagnosis? A- Heat Stroke( surrounding/ambient temp should b raised in this ) B-Mental Stress C-Exercise(Ans) D-Sympatholytic drugs 107 most potent mechansim of preventing heat loss a shivering (potent mechanism for heat production) b vasoconstriction (Ans) (potent mechanism to prevent heat) 108 old lady, shortened limb, rotated laterally, cause a fracture neck of femur(Ans) b dislocation of hip(Anot dislocation, hip fracture can be true ) c supr gluteal d inf gluteal 109 asthma pt, most decrease volume is a fev1 b fev1/fvc ratio c fvc 110 cynosis's cause a carbon monoxide posioning(cherry red coloration) b deoxyhenoglobin less than 5mg/dl(ans) 111 Regarding Aminoacids,which one of the following amino acids is not synthesized by the human body and should be included in the diet. A- Alanine B- Tyrosine C- Phenylalanine(Ans)( (in phenylketonuria,phenylalanine is excluded from human diet). D- tryptophan E112 Hering–Breuer reflex due to a stretch receptors in bronchioles (Ans) b irritants c j receptors 113 protein bound fraction of the drug is: a.active b inactive c more lipid soluble114 frequency distribution a tells distribution of value appear in data b tells distribution of value which is missing c gives percentage of values 115 standard deviation for a variability(ans) 116 you are allowed to breech patient's confidentiality A- For insurance claim B- If parents ask C- if pt authorizes you(ans) D- after pt's death 117 cerebellum is connected to Midbrain by https://www.facebook.com/groups/fcpspastpaper/ https://www.facebook.com/FcpsPart1BcQs?ref=hl A-Superior Cerebellar Peduncle(ans) B-Middle Cerebellar Peduncle C-Inferior Cerebellar Penduncle 118 A 45 yrs female came with abnormal vaginal bleeding. biospy shows simple hyperplasia due to? a.increase stimulation with estrogen(ans) b.decrease stimulation with estrogen c.increase stimulation with LH 119 a patient was lying in spine position ABCESS will enter in lung A.right upr lobe B.left posterior lobe c.right posterior lobe Dright apical superior E 120 arterioles diameter a 0.03 mm with naked eye? b ½ to 1mm c 1mm d 3mm e 5 mm121. Which of the following have the greatest amount of blood 1. Capillaries 2. Resistance vessels 3. Large veins(ans) 4 Arteries 122. A young adult received crush injury to his foot which became infected and necrosed for which BKA (below knee amputation) was carried out. Six months after the amputation, the patient developed a tender Haphazard bundles of nerves(nodule) with pain at the site of amputation. The nodule most likely is: a.neuroma(ans) b.ganglioma c.ganglionueroma d.hamartoma123 adh acts primarily on a.PCT b.Cortical Ct(ans)(late distal n collecting so mostly collecting) c.Medullary CT d.loop of henle e.DCT 124, which cells are not part of CNS A oligodendrocytes B padular astrocytes C ….. astrocytes D schawn cells(ans) 125 Parkinson a peduncular knee b static tremor(Ans) (Static tremor,cogwhel rigdity ,clasp knife in uper neuron lesion) C clasp knife rigidity 126 Pus forming cavities in sinuses type or a patient came to ER with fever n multiple https://www.facebook.com/groups/fcpspastpaper/ https://www.facebook.com/FcpsPart1BcQs?ref=hl abscess in neck region draining outside. which is the likely organism? a.pseudomonas b.staph aureus c.Treponema pallidum d.Actinomyces israelli(ans) 127. last to return after donation of 1 litre blood a rbc(Ans) b 128 scientist want to study about telomerase activity so which cells have rapid turnover a endothelial b.erythrocyets c.leukocytes d.germ cell(ans) 129 apoptosis, most initial a activation of caspases(ans) b phagocytosis c activate bcL2(its deactivated 1st ) 130 sight of lodging of inhaled foreign body a right lower lobe(ans)131 SA node activity decreased by parasympathetic increase in a potassium(Ans) b Na c Na n ca 132 thyroid problem and pt’s ven. rate was 180 with irregular regular pulse A digoxin B propanaol C amiodrone D verapamil (ans) 133 in pregnanacy rhematic fever, detoritation of cardiac status due to a mitral regurgitation b mitral stenosis c aortic regurgitation d aortic stenosis 134 Vagal stimulation on heart or like Acetylcholine increases in heart. a PR interval prolong(Ans) b stroke volume c heart rate 135 long term memory need a antaomical changes in neuronal circuits b Random storage in brain c dont reqire repeatition d 10 mints 136 exopthalamus, cause a increase TSI b increase t4 levels c increase t3 and t4 137 surfactant produced by a type 2 alveloar epithelial cells(Ans) 138. giving many dugs in TB bcoz a delay resistance https://www.facebook.com/groups/fcpspastpaper/ https://www.facebook.com/FcpsPart1BcQs?ref=hl 139 ineffective erythropoises A beta thalasemia B sickle 140 Not contain parasympathetic fibes a acessory spinal nerve??? or acessory nerve?(ans) b glossopharngeal 141 ganglin of lacrimal? or was parotid ?? a pterygopalatine ganglion 142 scenario: parasite causing urinary bladder ca, granuloma present. a. ascaris b shistosomiasis(Ans) 143 preganglioninc neurotransmitter a ACH 144 increase blood volume in vein due to a increase capacity b decrease compliance 145 scenario, eye partially ptois, unable to move up and medially, (downward n outward gaze) a occulomotor nerve palsy 146. Scenario: cataract, deafness and cardiac malformations a mothers Rubella(Ans) 147 chronic liver disese, (G.I bleed and portal HTN mentioned?dont remember)what will do a inj vit k b inj octreotide(Ans) 148 iron absorption a in ferrous fe+2 from b presence of oxalates 149 hco3 absorb at a proximal and distal cortical b proximal and collecting(Ans)? 150 pt passing urine at 1ml/mim.urine's lowest osmalrity level? a cortical collecting tubule(here ADH action make tubule hyperosmolar) b papillary collecting tuble c macula densa(ans)(As dct not mention so its right) d tip of loop of henle(here isotonic fluid) 151 kidney not related and posteriorly to a diaphragm b rectus abdominis c psoas d tranverse abdominis e colinic flexure 152 protocaval anastomosis a inf and sup rectal veins(Ans?)b superor epigastric and inferior epigastric 153 which steriod synthesis step reduce prostrate hypertrophy a inhibition of 5 alpha reductase(Ans) b increase cholesterol desmoloase c 17,20 hydrolase deficiency?? 154 pregnant women with thyrotoxicosis a propylthiouracil(Ans) 155 pt with thyrotoxicosis, treatment a propanaol https://www.facebook.com/groups/fcpspastpaper/ https://www.facebook.com/FcpsPart1BcQs?ref=hl 156 myocardial infarction, at 5th day or week, pt died autopsy showed? a cardiac temponade(ans) 157 most abundant immunoglobulin in plasma a IgG 158 PPD inserted, type of cells along with macrophages a T lymphocytes(Ans) 159 SLE is familial bcoz of a high incidence in monozygotic twins(Ans) b incidence is high in females c if its in family, more chances to occur…sumthing like that 160 transplantation rejected in 10 minutes a antibody mediated(Ans) b cell mediated c graft versus host 161 regarding csf a its byoncy and cushioning effect keep brain safe from injury(Ans) b produce 1000 ml in 24 hours c provide neutrient 162 csf in meningtis, what not correct a pyogens don’t decrease glucose(Ans) b fungal keep glucose normal 163 structure arching over left bronchus? a arch of aorta(Ans) 164. Resting membrane potential of heart? a is more depends on potassium equilibrium(ans) b is more depends on sodium equilibrium165 ketamine is used in repeated burn dressing’s coz? a its does not cause hypotension b.Causes profound analgesia(ans)(past ques) c,it can b given im iv d.it does not causes addiction e it is not narcotic 166 which is NOT leukoplakia precursor a spices (Sipces can cause chronic irritation and lead to leukoplakia) b spirits c syphilis(Ans)??( it can cause leukoplakia rarely) d sexual contact(Ans? 167. P wave a atrial depolarization b atrial contraction? 168 in type 2 mobitz a decrease in stroke volume b increase in cardiac output c ventricular rate slower then atria(Ans) 169 thymus? a has numerous lymphocytes(Ans) b derivative of 4th arch c deveops from thyroid tissue d Largest at puberty e regresses shortly after birth 170 female in er pain in RIF, lmp 3 weeks back, fever n wbc raised https://www.facebook.com/groups/fcpspastpaper/ https://www.facebook.com/FcpsPart1BcQs?ref=hl a appendix b uterus 171 no change to MCV, which condition? a sickle b 2nd post heamorrahge day(ans) 172 sicle cell scenario. boy having hb 5.6 deeply jaundiced, ,plt 260000 nd tlc 10000 having non healing ulcers, abdominal pain after tooth extraction a aplastic crisis b painful crisis c sequestration crisis d hemolytic crisis 173 g6 pd a henz bodies b helmet c etc etc 174 primary cartilaginous jointa costochondral(ans) b.teeth in jaw c joints of skull d. inferior radio ulnar e. pubic symphisis 175 which is exactly 2/3 body water a intracellular b Extracellular c plasma 176 Scenario of azad kashmir lady injured in in earth quake, one of his distant uncle died too???? a family crisis b social crisis c situational crisis(Ans) d economic crisis e developmental crisis 177 scenario: young adult atypical lymphocytes, virus? a ebv virus(Ans) b cytomegalus virus 178 all are wrong except one ?? a hbv and burkitt lyymphoma 179 hanging causes death by a Fracture of odontoid process of axis(Ans) b atlas 180 Which of the arteries does not supply the scalp A- Maxillary B- Verterbral C- Occipital 181 Winging scapula a Long thoracic 182 appendicular artery a br . of posterior cecal artery b https://www.facebook.com/groups/fcpspastpaper/ https://www.facebook.com/FcpsPart1BcQs?ref=hl 183 Anal canal a supplied by sup and inf rectal artery (Ans) b Inf rectal drains in portal circulation 184 Internal sphincter is supplied by A. Pudendal nerve(ans) B. Hypogastric plexus185 basic drugs bind to A albumin (Asim n shoaib) (acidic drug binds to alklotic albumin) B alpha 1 glyco protein (ans) wikipedia 186 diabetic type 2 pt with dyspepsia and consistent diabetic gastroperisis, what shoud given a domperidone b metaclopramide c scrulfate 187 edema in nephrotic syndrome due to a. low colloid osmotic pressure of plasma(Ans) 188. Ascorbic acid vit c causes a.hydroxylation of collagen(ans) b triple helix formation 189 Regarding lower gastroesophageal sphincter a. its a true anatomic sphincter b its pressure increases in pregnancy c.its pressure decreases in achalasia d.it relaxes during swallowing(ans) 190 About retics a. corrected count is good indicator of erythropoietc activity b.contain DNA remanants c with wright stains d include in CBC191 bleeding time increased in a liver failure b vit k deficiency c hemophillia D thrombocytopenia(Ans) E thrombesthenia 192 Glucose transport n kidney a cotransport with Na 193 cause if inc Ca in cardiac cell a adrenaline(ans) b diltiazam c trimethphen 194 An isolated complete rupture of anterior cruciate ligament will result in instability of tibia over femoral condyle. The direction of instability will most likely be? A. Anterior(Ans) B. Antero-lateral C. Antero-medial D. Postero-lateral E. Postero-medial 195 About platelets a. forms clot by binding with collagen fibre(ans) B. has granules C hav no nucleus196 regarding in pregnancy effects of parahormone(not remember https://www.facebook.com/groups/fcpspastpaper/ https://www.facebook.com/FcpsPart1BcQs?ref=hl fully) a. hyperparathroidism due to estrogen b inhibition of bone resorption 197 hypergicemia.. Question like that …or may b I am mixing it with question no 21 (abt insulin) A thiazides diuretics? 198 during formation of nonessential a a( amino gp ???) is donatd by( not sure this came or not) a-oxaloacetate b-glutamate c-pyruvate A thiazides diuretics? 199 incraese uric acid( not sure this question came or not) a prizinamaide? 200 Organs present in the peritoneal cavity ? A fallopian tube b ovary? c proximal part of duodenum(ans) d uterus e gonads https://www.facebook.com/groups/fcpspastpaper/ https://www.facebook.com/FcpsPart1BcQs?ref=hl March 7 2013 1.Scenario: Lady with dry eyes and dry mouth... diagnosis is a.Sjogren(Ans) b.Milkulicz syndrome? 2 Smooth and cordinated movement by a. basal ganglia(Ans) b. spinocerebellar 3.Scenario: elderly,diplopia,granuloma,raised b.p a. giant cell arteritis(Ans) b. takayasu disease c. wegeners Granulomatosis 4. Zinc required for a cellular oxiadtion(Ans) b glucose oxidation c 5. Maxillary artery form a 1st arch(Ans) b 2nd arch 6.Testosterone causes transient bone growth because? a it causes epiphyseal closure(ans) b downgrade its receptors c 17 ketosteroid 7 GFR decreased by a afferent arteriolar contraction(ANS) b.renin increases c.efferent arteriolar constriction 8 Aotosomal recessive a. Achondroplasia b. Marfans syn c. Alpha1 antitrypsin defiecncy(ANS) d. Neurofibramatosis 9 Nucleus a continous with rough endoplasmic reticulum(Ans) b conatin pores 10 basophila in cell due to which organelle? a ribosomes(Ans) b glogi apparatus 11 fatty liver NOT seen in a viral hepatitis(ans) b congestion of liver c pregnancy d obesity e alcohol 12 scenario: pt with increased heaptic glycogen with normal structure, no inc in serum glucose after giving fructose,enzyme deficiency a fructokinase b glucose 6 phospahate(Ans) c hexokinase 13 scenario 6 yrs old, given anti malarial, hb low, ad coke cloured urine,( inc tlc n platelets were also there may b) a g6pd deficincy(Ans) b black water fever c Paroxysomal nocturnal hemoglobinuria https://www.facebook.com/groups/fcpspastpaper/ https://www.facebook.com/FcpsPart1BcQs?ref=hl 14 vasodilators a PGI2(ans) b ltc4 c ltd4 d TXA2 15 difference bw kwashiorkor and marsmus a bilateral pitting edema(Ans) b muscle wasting 16 Two point discrimination sensor a mesiners corpuscles b merkels disk c Golgi tendons d free nerve e muscle spindle 17 which is not a direct effect of cushing syndrome? a facial hair growth(ans) b buffalo hump c muscle weakness d thin limbs 18 hormone required for brain developlment in fetal life a thyroid hormone(Ans) b growth hormone c cortisol d androgens 19 scenario, mother rh negative, baby delivered at 34 weeks with anemia palour, died after 6 hrs after transfusion, most likely cause a. basal ganglia staining(Ans) b. cardiac malformations 20 post ganglionic sympathic nerve fibers r present in1.cervical nerve 2.all spinal nerve(ans) 3.thoracic n(ans)?? Thoraco lumbar? 4.lumber 5.sacral 21 insulin increased by a beta blockers b secretin (ans)(its homologous to glucagon) c thiazides 22 virus act carcinogecially thru a oncogenes(Ans) (in virus are oncogenes, in humans are protoncogens) b protooncogenes c promotor 23 radiation causes carcinoma in how many years? a. les than 1 yr b. less than 2 yrs. c. 2-5 yrs d > 10 yrz e. 7-10 yrs(ans) (Radiation can cause cancer in most parts of the body, in all animals, and at any age, although radiation-induced solid tumors usually take 10–15 years, and can take up to 40 years, to become clinically manifest, and radiation-induced leukemias typically require 2–10 years to appear) 24 atrial flutter a treated with defibrillation shock therapy(Ans) b heart rate is so rapid above 400 beats / min c heart beat between 200 and 350/min(??) d circus waves are not main reason 25 carcinogen a nitrosamides(Ans) https://www.facebook.com/groups/fcpspastpaper/ https://www.facebook.com/FcpsPart1BcQs?ref=hl b nicotine c cadmieum 26 u wave a slow reploarization of papillary muscles(ans)(its due to hypokalemia)(1st aid) b atrial repolarization c.SA depolarization d.AV depolarization e septum depolarization 27 purkinje fibes a specialized heart muscle cells b contain numerous myofibrils 28 hb 6gm/dl increased plateled count with hyperplastic bone marrow? a.iron def anemiab.hemolytic anemia c.acute blood loss d.sideroblastic anemia e.leukemia(?) 29 clostridium a are spores forming(Ans) b are facultative anerobes( they are obligate anerobes) 30 pseudomembrane colitis least likey caused by a ceftriaxone b clauvinic ampicllin(Ans) c clindamycin d vancomycin(it’s the treatment of pseudomembrane colitis) 31 virus a altered protien synthesis 32 In right atrium, SA node is located just near the A- Auricle B- Fossa Ovalis C- Superior part of Sulcus Terminale(Ans) D- Lower part of Crista Terminalis 33 which of following DOES NOT drain in internal juglar vein a ext. juglar(ans) b sup. thyroid 34 soldeir came from high attitude , 2000 ft , pcv 60 %,malaise,fever, spleenomegaly, cyanosis a sec polycythemia(ans)?? b methmaglobinemia 35 Men from sea to attitude a pul vasoconstriction(Ans) 36 increased diffusion of CO2 through alveolar membrane is due to? a increased diffusion coefficient (20 times more ability to diffuse) b increased solubility c more affinity then o2(Asim shoaib) 37 water intoxication a distended neck veins b pulmonary edema(cerebral edema happens) c slow pulse(Ans)(Asim shoaib) d Increased alertness(guyton) e thirst 38 total water depletion, replaced by? a hypotonic (ans by some ppl.. logically fit) b isotonic c 5% dextrose( ans by most) d 1% glucosee ringer lactate 39 unconcious patient brought to hospital after 1 hour of severe haemoraghe his bp is 50 mmhg,pulse is rapid, cool n calmy skin, which mechanism is most active at this stage https://www.facebook.com/groups/fcpspastpaper/ https://www.facebook.com/FcpsPart1BcQs?ref=hl a baroreceptors reflex b cerebral ischemic(ans)(its start acting below 60mmHg of BP ) c cushing reaction d chemoreceptors 40 po2 60 mmhg, hgb 15. cause of injury a hypoxic b anemia c overutilization d histotoxic 41 integrins at a leukocytes(Ans) b platelets c endothelial cells 42 primary amylodosis of increase immunoglobulin light chain a systemic(Ans) b reactive systemic 43 superior parathyroid a at the level of middle of thyroid gland on posterior surface at 1st tracheal ring( its actually 3rd ring )(ans nearly to right) b variable in position c in frront of pretracheal fascia d supplied by superior thyroid artery(inferioris the main supply) 44 cricoid level a oesophagus with pharynx(Ans) b common carotid bifurcation level 45 vertical lesion of optic chiasma a bitemporal hemianopia 46 patient has done clostomy, its now on parenteral diet a fatty liver b hyperglycemia(Ans) c hypophosphotemia 47 opiods are agonist, one with antagonist a pethidine b buprenorphine(ans) c pentazocine(also correct bt is prototype) d morphine e fentanyl 48 Regarding ANTI-EPILEPTIC DRUGS,which one is INCORRECTa dizipam inhibits gaba channels b ethusuxsimide act via blocking of K channels(ans)(its Ca channels) c. phenobarbitaol… d phenytoin prolongs the inactivated stated on Na Channels e vigabatrin irreversibly block the enzyme 49 Lumbar puncture done through: A- L2 & L3 B- L4 & L5(ans) C- S1- S2 50 subduaral hemaorhage a superoir cerebral veins(ans) b middle meningeal artery 51 most dangerous effect of diphtheria exotoxin on which organ? a heart(Ans) b.Larynx c.Intestine 52 renin secretion is increased by? a decreased atrial strech b macual densa? c plasma osmolarity( Ans)? https://www.facebook.com/groups/fcpspastpaper/ https://www.facebook.com/FcpsPart1BcQs?ref=hl d inc K+ ? 53 most characteristic symptom of esophageal atresia? a regurgitation of all of first feed b attacks of cough on feed (this one also by Wikipedia) c continuous dribbling(ans) (Asim shoaib) 54 after adrenalectomy, cause of death within few days is coz of absent of a aldosterone (Ans) Addison’s is due to mineralocorticoid deficiency basically b cortisol? 55 which of the following is not neoplasia 1. choriostoma(ans) 2. memgioma 3. lymphoma 56. 40 yrz old lady with swelling n pain in knee joint, synovial fluid shows basophilic rhomboid crystals: a. calcium pyrophosphate(ans)(psedogout) b. monosodium urate c. glycine 57 90 % of anthrax pts have lesions in a.skin(ans) b.lungs 58. Glucose transport is directly proportional to it's concentration, this phenomenon is known as A -Active Transport B- Diffusion (ans? Mostly say this) C- Facilitated Diffusion(ans)? 59 nutrient which need further digestion to be absorbed a sucrose(ans) b tripeptides 60 high triglyceride content a chylomicrons(Ans) 61 cholesterol with negative coronary risk factor a hdl(Ans) b ldl ? 62 counter current multiplier a requires hypertonic medullary interstitium(Ans) b produce by vasa recta(mainitaineb by it 63 preganlonic nerve fibres contain a B fibres(Ans) 64 muscle tension is decreased to prevent muscle rupture by a golgi tendons??(Ans) b muscle spindle ?? 65 durg to irreversibly impair platelet function a aspirin(Ans) b ibuprofen 66 A patient comes to a doctor with a complain of pain down the medial side of right forearm & hand, & wasting of small muscles of hand. On examination ,circulation of right upper limb was also impaired. The anatomical structure most likely to be involved is? A. Head of right humarus B. Left cervical rib C. Right 1st rib D. Right 2nd rib E. Right cervical rib(Ans) 67 Pt presented with hemoptysis & hematuria. It progressed to renal failure. On biopsy there were focal depositions with linear pattern https://www.facebook.com/groups/fcpspastpaper/ https://www.facebook.com/FcpsPart1BcQs?ref=hl A- Focal glomerulonephritis B- Rapid Progressive Glomerulonephritis C- Good pasture's (Ans) 68 scenario male 30 years splenomagaly fever malaise cardiac murmer and splinter hemorrhages and hematuria . Most appropriate test a.Blood serology b.Blood culture(ans) c.Stool culture 69 scenario some weeks of disease, fever, lethargic, creatinine was 4, pedal edema n anemia hb,6 rbc 3.6, cause a erythropoiten deficiency(Ans)(i.e CRF) b acute renal failure( acute dose not cause anemia) 70 sceanrio: pt of acromegaly.. lips n bone enlarged a adenoma of adenohypophsis b adeoma of acidophilic cells(Ans) (acidophilic cells include somatotrohs and mammaotrohs ) c adenoma of chromphobic cells 71 most likely about mammary gland a extend vertically from 2 to 6 rib( its right also) b contain 15 to 20 lactiferous ducts(Ans)? c in pregnancy, contain lactiferous ducts and secretory alveoli d sebaceous glands in aereola 72 scenario of post op. abdominal surgery, thromboembloism cause? a stasis n hypercoagubility(Ans) b stasis c endothelial injury and stasis d endothelial injury and hypercoagulability 73 biceps reflex absent, knee n ankle reflex hyper, ventilation rate 16 / min, cause spinal cord section a L1 l2 l3 b c8 t1 c t8 t9 d c5 c6(ans) e c3 c4 74 ventrical cortico spinal fibres ends on a lumbar b mid thoracic(ans)(Wikipedia)(its small as compared to main lateral corticospinal and ends gradually on mid thoracic) c thoracic d end of spinal cord 75 standing from supine position a incraese heart rate(ans) b decrease cardiac output 76 not of cardiogenic shock a hemaorragheb MI c CHF d dehydration 77. Water enter interstitium a.filtration(Ans) b.diffusion c.cotransport with Na d. decrease hydrostatic pressure 78 most evident clinical feature of sjogrens a enlargement of salivary gland b dry mouth n dry eyes c dry mouth only 79 leprosy a intracellular acid fast https://www.facebook.com/groups/fcpspastpaper/ https://www.facebook.com/FcpsPart1BcQs?ref=hl b intraceelualar non acid fast c extracelluar acid fast 80 scenario: child with skin eruptions along facial hair line, conjunctivitis, bluish white specs with red margins at buccal mucosa (measles) a kopliks spot 81 Not oro fecal a triculais spirailis(Ans) b triculais tachuli 82 hepatits B a by sexual contact(Ans) b oro fecal 83 not a 2nd line drug in Rheumatoid arthritis a antibiotics(Ans) b anti malarials c ?golds e sulphazine 84 from under cook meat a tape worm(ans) b liver fluke 85 smooth muscles a arrange in longitudinal and circular layer(Ans) b striated muscle 86 post traumatic patient, memory loss, area a hippocampus(ans) 87 heart muscle show no tetnus a increase absolute refractory period 88. Pancreatic fistula, somatostatin a inhibit other enzymes b dec secretions(ans?) 89 somatosensory different from primary motor in a granular layer(ans?) b at lateral side c has lager homunculus 90 A patient in an operation requiring general anaesthesia has high BP which drugs should be given a glycerine nitride? b halothane c hydralizine(Ans) d metoprolol 91 regarding medulla, correct one a forth ventricle in upper medulla 92 A 40 year Male working as a taxi driver in sharjah presented with C/O fever,weight loss,anorexia. Diarrehea, spleenomegaly,he is emaciated,has jaundice and generalized lymphadenopathy.whats the diagnosis. a AIDS(ans) b leishmaniasis c yellow fever 93 Turner syndrome a. 44XO(ans) b. 44XXX c. 44XXXX 94 not present in AIDS a sqaumous cell ca(Ans) b primary brain lyhpoma c kaposis’s https://www.facebook.com/groups/fcpspastpaper/ https://www.facebook.com/FcpsPart1BcQs?ref=hl 95 scenario: nodule on dorsum of hand,same side axillary lymph nodes are enlarged, happened due to? (melanoma case) a ultra violet radiations(Ans) b old job related asbestos c latex glove allergy 96 coronary sinus co tinuation of A graet cardiac vein 97 A pt came to gingival hypertrophy, bleeding from gums, petachies, painful sore at lips..you advice investigation to.rule out a.Lyphoma?? b.Lukemia?? c. bachet disease 98 souce of creatinine a muscle(Ans) b heart 99 pt with diabetic nephropathy, investigation of chioce a albumin(ans) b serum creatinine levels c creatinine clearance 100 scenario, pt having swellin on right neck, dicsharging, routine culture shows nothing. now first initial investigation for diagnosis a biopsy(Ans)(it will reveal everything) b zehil stain 101 question regarding ovary’s embryology a all primary ooctyes go in prophase of first meotic in fetal life(ans) b most of oocyte undergo degeneration from birth to puberty c oocyte completes secondary miosis before ovulation something like that 102 which structure seperate maternal and fetal blood at last stages of pregnancy a Cytotrophoblast b.Syncishiotrophoblast c.Cyto and synchio d.Synschiotrophoblast and fetal endothelium(Ans) 103 2nd division of trigeminal nerve NOT supply to a one air sinus b nasal cavity c nasal pharynx? d temporal bone 104 Female having HIV but no AIDS CD count 800 now having middle ear infection SOM (supartive otits media), cause a pneumocytis carinii(below 500 typical infections of aids get start) b streptococci pneumonia c M avium d psedomanas aueriginosa ( cause ext otits media) e staph aureus 105 men having abcess at left thigh after injury from thorn, cause a staph aureus b strept pyogens 106 19 yr old boy sitting in a well ventilated room of 26'C. severely sweating , His pulse 120 beats/min, Bp 150/90. What's the diagnosis? A- Heat Stroke( surrounding/ambient temp should b raised in this ) B-Mental Stress C-Exercise(Ans) D-Sympatholytic drugs 107 most potent mechansim of preventing heat loss a shivering (potent mechanism for heat production) b vasoconstriction (Ans) (potent mechanism to prevent heat) https://www.facebook.com/groups/fcpspastpaper/ https://www.facebook.com/FcpsPart1BcQs?ref=hl 108 old lady, shortened limb, rotated laterally, cause a fracture neck of femur(Ans) b dislocation of hip(Anot dislocation, hip fracture can be true ) c supr gluteal d inf gluteal 109 asthma pt, most decrease volume is a fev1 b fev1/fvc ratio c fvc 110 cynosis's cause a carbon monoxide posioning(cherry red coloration) b deoxyhenoglobin less than 5mg/dl(ans) 111 Regarding Aminoacids,which one of the following amino acids is not synthesized by the human body and should be included in the diet. A- Alanine B- Tyrosine C- Phenylalanine(Ans)( (in phenylketonuria,phenylalanine is excluded from human diet). D- tryptophan E112 Hering–Breuer reflex due to a stretch receptors in bronchioles (Ans) b irritants c j receptors 113 protein bound fraction of the drug is: a.active b inactive c more lipid soluble 114 frequency distribution a tells distribution of value appear in data b tells distribution of value which is missing c gives percentage of values 115 standard deviation for a variability(ans) 116 you are allowed to breech patient's confidentiality A- For insurance claim B- If parents ask C- if pt authorizes you(ans) D- after pt's death 117 cerebellum is connected to Midbrain by A-Superior Cerebellar Peduncle(ans) B-Middle Cerebellar Peduncle C-Inferior Cerebellar Penduncle 118 A 45 yrs female came with abnormal vaginal bleeding. biospy shows simple hyperplasia due to? a.increase stimulation with estrogen(ans) b.decrease stimulation with estrogen c.increase stimulation with LH 119 a patient was lying in spine position ABCESS will enter in lung A.right upr lobe B.left posterior lobe c.right posterior lobe Dright apical superior E 120 arterioles diameter a 0.03 mm with naked eye? b ½ to 1mm c 1mm https://www.facebook.com/groups/fcpspastpaper/ https://www.facebook.com/FcpsPart1BcQs?ref=hl d 3mm e 5 mm 121. Which of the following have the greatest amount of blood 1. Capillaries 2. Resistance vessels 3. Large veins(ans) 4 Arteries 122. A young adult received crush injury to his foot which became infected and necrosed for which BKA (below knee amputation) was carried out. Six months after the amputation, the patient developed a tender Haphazard bundles of nerves(nodule) with pain at the site of amputation. The nodule most likely is: a.neuroma(ans) b.ganglioma c.ganglionueroma d.hamartoma 123 adh acts primarily on a.PCT b.Cortical Ct(ans)(late distal n collecting so mostly collecting) c.Medullary CT d.loop of henle e.DCT 124, which cells are not part of CNS A oligodendrocytes B padular astrocytes C ….. astrocytes D schawn cells(ans) 125 Parkinson a peduncular knee b static tremor(Ans) (Static tremor,cogwhel rigdity ,clasp knife in uper neuron lesion) C clasp knife rigidity 126 Pus forming cavities in sinuses type or a patient came to ER with fever n multiple abscess in neck region draining outside. which is the likely organism? a.pseudomonas b.staph aureus c.Treponema pallidum d.Actinomyces israelli(ans) 127. last to return after donation of 1 litre blood a rbc(Ans) b 128 scientist want to study about telomerase activity so which cells have rapid turnover a endothelial b.erythrocyets c.leukocytes d.germ cell(ans) 129 apoptosis, most initial a activation of caspases(ans) b phagocytosis c activate bcL2(its deactivated 1st) 130 sight of lodging of inhaled foreign body a right lower lobe(ans)131 SA node activity decreased by parasympathetic increase in a potassium(Ans) b Na c Na n ca 132 thyroid problem and pt’s ven. rate was 180 with irregular regular pulse A digoxin B propanaol C amiodrone ascaris b shistosomiasis(Ans) 143 preganglioninc neurotransmitter a ACH 144 increase blood volume in vein due to a increase capacity b decrease compliance 145 scenario. eye partially ptois. cause a increase TSI b increase t4 levels c increase t3 and t4 137 surfactant produced by a type 2 alveloar epithelial cells(Ans) 138.I bleed and portal HTN mentioned?dont remember)what will do a inj vit k b inj octreotide(Ans) 148 iron absorption a in ferrous fe+2 from b presence of oxalates 149 hco3 absorb at a proximal and distal cortical b proximal and collecting(Ans)? 150 pt passing urine at 1ml/mim. granuloma present. Scenario: cataract.com/FcpsPart1BcQs?ref=hl D verapamil (ans) 133 in pregnanacy rhematic fever. (G. deafness and cardiac malformations a mothers Rubella(Ans) 147 chronic liver disese. giving many dugs in TB bcoz a delay resistance 139 ineffective erythropoises A beta thalasemia B sickle 140 Not contain parasympathetic fibes a acessory spinal nerve??? or acessory nerve?(ans) b glossopharngeal 141 ganglin of lacrimal? or was parotid ?? a pterygopalatine ganglion 142 scenario: parasite causing urinary bladder ca.https://www. a PR interval prolong(Ans) b stroke volume c heart rate 135 long term memory need a antaomical changes in neuronal circuits b Random storage in brain c dont reqire repeatition d 10 mints 136 exopthalamus.facebook. (downward n outward gaze) a occulomotor nerve palsy 146.facebook.com/groups/fcpspastpaper/ https://www.urine's lowest osmalrity level? a cortical collecting tubule(here ADH action make tubule hyperosmolar) b papillary collecting tuble c macula densa(ans)(As dct not mention so its right) . a. unable to move up and medially. detoritation of cardiac status due to a mitral regurgitation b mitral stenosis c aortic regurgitation d aortic stenosis 134 Vagal stimulation on heart or like Acetylcholine increases in heart. facebook. type of cells along with macrophages a T lymphocytes(Ans) 159 SLE is familial bcoz of a high incidence in monozygotic twins(Ans) b incidence is high in females c if its in family. more chances to occur…sumthing like that 160 transplantation rejected in 10 minutes a antibody mediated(Ans) b cell mediated c graft versus host 161 regarding csf a its byoncy and cushioning effect keep brain safe from injury(Ans) b produce 1000 ml in 24 hours c provide neutrient 162 csf in meningtis.it can b given im iv d.Causes profound analgesia(ans)(past ques) c. at 5th day or week. pt died autopsy showed? a cardiac temponade(ans) 157 most abundant immunoglobulin in plasma a IgG 158 PPD inserted. treatment a propanaol 156 myocardial infarction. Resting membrane potential of heart? a is more depends on potassium equilibrium(ans) b is more depends on sodium equilibrium 165 ketamine is used in repeated burn dressing’s coz? a its does not cause hypotension b. what not correct a pyogens don’t decrease glucose(Ans) b fungal keep glucose normal 163 structure arching over left bronchus? a arch of aorta(Ans) 164.facebook.com/FcpsPart1BcQs?ref=hl d tip of loop of henle(here isotonic fluid) 151 kidney not related and posteriorly to a diaphragm b rectus abdominis c psoas d tranverse abdominis e colinic flexure 152 protocaval anastomosis a inf and sup rectal veins(Ans?)b superor epigastric and inferior epigastric 153 which steriod synthesis step reduce prostrate hypertrophy a inhibition of 5 alpha reductase(Ans) b increase cholesterol desmoloase c 17.com/groups/fcpspastpaper/ https://www.20 hydrolase deficiency?? 154 pregnant women with thyrotoxicosis a propylthiouracil(Ans) 155 pt with thyrotoxicosis.https://www.it does not causes addiction e it is not narcotic 166 which is NOT leukoplakia precursor a spices (Sipces can cause chronic irritation and lead to leukoplakia) b spirits c syphilis(Ans)??( it can cause leukoplakia rarely) d sexual contact(Ans? . com/FcpsPart1BcQs?ref=hl 167. boy having hb 5.facebook. inferior radio ulnar e.teeth in jaw c joints of skull d.6 deeply jaundiced. one of his distant uncle died too???? a family crisis b social crisis c situational crisis(Ans) d economic crisis e developmental crisis 177 scenario: young adult atypical lymphocytes. pubic symphisis 175 which is exactly 2/3 body water a intracellular b Extracellular c plasma 176 Scenario of azad kashmir lady injured in in earth quake. lmp 3 weeks back. . fever n wbc raised a appendix b uterus 171 no change to MCV. abdominal pain after tooth extraction a aplastic crisis b painful crisis c sequestration crisis d hemolytic crisis 173 g6 pd a henz bodies b helmet c etc etc 174 primary cartilaginous jointa costochondral(ans) b. virus? a ebv virus(Ans) b cytomegalus virus 178 all are wrong except one ?? a hbv and burkitt lyymphoma 179 hanging causes death by a Fracture of odontoid process of axis(Ans) b atlas 180 Which of the arteries does not supply the scalp A.plt 260000 nd tlc 10000 having non healing ulcers.com/groups/fcpspastpaper/ https://www.Maxillary B. which condition? a sickle b 2nd post heamorrahge day(ans) 172 sicle cell scenario.facebook.Verterbral C. P wave a atrial depolarization b atrial contraction? 168 in type 2 mobitz a decrease in stroke volume b increase in cardiac output c ventricular rate slower then atria(Ans) 169 thymus? a has numerous lymphocytes(Ans) b derivative of 4th arch c deveops from thyroid tissue d Largest at puberty e regresses shortly after birth 170 female in er pain in RIF.Occipital .https://www. Postero-lateral E. what shoud given a domperidone b metaclopramide c scrulfate 187 edema in nephrotic syndrome due to a. Pudendal nerve(ans) B. The direction of instability will most likely be? A. forms clot by binding with collagen fibre(ans) B. corrected count is good indicator of erythropoietc activity b.contain DNA remanants c with wright stains d include in CBC 191 bleeding time increased in a liver failure b vit k deficiency c hemophillia D thrombocytopenia(Ans) E thrombesthenia 192 Glucose transport n kidney a cotransport with Na 193 cause if inc Ca in cardiac cell a adrenaline(ans) b diltiazam c trimethphen 194 An isolated complete rupture of anterior cruciate ligament will result in instability of tibia over femoral condyle.com/FcpsPart1BcQs?ref=hl 181 Winging scapula a Long thoracic 182 appendicular artery a br . Ascorbic acid vit c causes a. Hypogastric plexus 185 basic drugs bind to A albumin (Asim n shoaib) (acidic drug binds to alklotic albumin) B alpha 1 glyco protein (ans) wikipedia 186 diabetic type 2 pt with dyspepsia and consistent diabetic gastroperisis.com/groups/fcpspastpaper/ https://www.facebook. Antero-medial D. has granules C hav no nucleus . of posterior cecal artery b 183 Anal canal a supplied by sup and inf rectal artery (Ans) b Inf rectal drains in portal circulation 184 Internal sphincter is supplied by A. low colloid osmotic pressure of plasma(Ans) 188. Antero-lateral C. Postero-medial 195 About platelets a.it relaxes during swallowing(ans) 190 About retics a. Anterior(Ans) B.hydroxylation of collagen(ans) b triple helix formation 189 Regarding lower gastroesophageal sphincter a.https://www. its a true anatomic sphincter b its pressure increases in pregnancy c.facebook.its pressure decreases in achalasia d. facebook. Question like that …or may b I am mixing it with question no 21 (abt insulin) A thiazides diuretics? 198 during formation of nonessential a a( amino gp ???) is donatd by( not sure this came or not) a-oxaloacetate b-glutamate c-pyruvate A thiazides diuretics? 199 incraese uric acid( not sure this question came or not) a prizinamaide? 200 Organs present in the peritoneal cavity ? A fallopian tube b ovary? c proximal part of duodenum(ans) d uterus e gonads .. hyperparathroidism due to estrogen b inhibition of bone resorption 197 hypergicemia.facebook.com/FcpsPart1BcQs?ref=hl 196 regarding in pregnancy effects of parahormone(not remember fully) a.https://www.com/groups/fcpspastpaper/ https://www. hairy leukoplakia assoc.facebook. phenoromone are a cant detect by human due to low threshhold of olfaction b relase among members of diff. most common cause of fatty liver in our region? a hep a nd b b alcohol c inc fat intake d protien def 9.grade IV encephalopathy caused by a paracetamol b ethylene glycol 2.https://www. ptnt with ATT now develop visual field abnormalty nd eye changes which drug? a ethambutol b inh c rifampacin 5. ptnt on ATT develop joint pain nd have inc. basal ganglia complex have? a high ca b high magnesium c low ca 10.facebook. uric acid level which drug? a pyrazinamide b rifampacin c ethambutol 3.T 9:22 dislocation occurs in a cml b aml c cll 8. mcv a pernicious anemia b anemia of chronic disease . species c elecit a special behaviour response 4. peripheral blood of patnt show marked anisocytosis nd poikilocytosis with inc.com/FcpsPart1BcQs?ref=hl March 7 Paper 2013 1.com/groups/fcpspastpaper/ https://www. with a HIV 7. which nerve not present in carotid sheath? aX b XI c XII d Ix e auditory 6. most potent chemotactic agent a c5a 20.com/FcpsPart1BcQs?ref=hl c thalasemia 11. mcv 126fl serum ferritin 600ng/ml? a IDA b megaloblastic . vit b12 normally absorb in a ileum b duodenum c stomach 12. baby with recurrent cervical lymphadenopathy klebsiella pneumonia is isolated from lesion which immun deficiency? a chronic gran. disease b severe combine agamma c t cell function defect 19. anemic. child has surgery of terminal ileum which vitamin not absorbed? a b12 b vit c 18. a 4 year boy with failure to thrive recurrent sinusitis otitis media nd RTI nd family history of such complains to cnfrm diagnosis? a choloride sweat test b fat absorption test jejunal biopsy 16.ptnt with butterfly rash nd protienuriamost initial screening test? a ana b anti dna 14.most potent protien synthesis in skeletal muscle? a testosterone b dht c progesterone d estradiol 17.facebook.com/groups/fcpspastpaper/ https://www. old male farmer with ascites on ascitic fluid have malignant cell nd on biopsy angiosarcoma cause? a afalatoxin b vinyl chloride c arsenic 15.https://www.a ptnt with fracture of femur died after 2 days cause? a fat embolism 13.40 year male with skin pigmentation nd disturbance in balance.facebook. com/groups/fcpspastpaper/ https://www. boy with loss of apetite vomiting high color urine nd yellow sclera? a bilirubin + alt b hep a nd b c liver biopsy 27.8 tlc 59x10`9 peripheral show mature lymphocyte? a CLL b Hairy cell leukemia c sarcoidosis 26. IHD with or without MI features? a stable angina b unstable angina c MI d prazimental angina 23.klienfelter karyotype? a XO b XX c XXY 29. first symptom of hypopitutrism sec.https://www.edema of nephrotic synd? a hypoalbumin 24. diagnosis of typhoid in 2nd week? a blood culture + widal b widal c blood culture 30.com/FcpsPart1BcQs?ref=hl c anemia of chronic disease 21.facebook. angina worsens by giving? a captopril b vasoopressin c b blocker 22. 60 YEAR OLD MALE 8 MNTH HISTORY OF PROGRESIVe genrelisd lymphadenopathy hb 9.type of TYPE III hypersenstvty? a post streptococal gmnephritis 28.facebook. to pitutiary tumor? a DI .best example of autoimmunity against single organ? a hashimoto thyr b poly arteritis c sle d RA 25. digoxin related question in which toxicity in? a given with quinidine b hyperkalemia c hypermagnesimia d hypocalcemia 36 hepatic pedicle consist of? a hepatic artery b br.facebook. insulin inc entry of glucose in a skeletal muscle b neurons c mucosa of small intestine 33.dopa causes all except a ammenorhea glactorhea syndrome b antiemetic c parkinsonism d antipshycotic 34. of portal vein c hepatic artery portal vein common hepatic duct 37 diff b/w cimetidine nd ranitidine s that ranitidine? a less effective b less CNS side effects c dec gastric motility 38 distance for respiratory droplets to cause infection? a 1-2 feet b 3-6 feet 39 artery of foregut? a celiac b SMA c IMA 40 body first mechanism to produce heat against cold? .https://www. endocarditis c marantic endocard 32. blockade of dopa receptor by l.com/FcpsPart1BcQs?ref=h l b DM c hypogonadism 31.aspirin given to decrease chance of IHD will cause? a dec pltlt agregation 35.com/groups/fcpspastpaper/ https://www.facebook. myxoid degeneration assoc with a mitral prolapse b inf. facebook.com/FcpsPart1BcQs?ref=hl a shivering 41 presence of scanty barr body? a turner b klienfelter c TFS d downsyndrome male 42 regarding thalamus? a control emotion 'b control body water c recieve all somatic nd sensory information 43 structure passing foramen ovale? a maxillary b mandibular c meningeal artery 44 baby prefering bottle milk over breast milk? a cleft soft palat b cleft soft nd hard palate c dec hormones d weak larynx muscle e nose blokade 45 fracture of shaft of humerus nd ptnt cant raise arm? a axilley nerve 46 ptnt cant evert his foot damage to? a tibialis ant b tibialis post c peroneus longus d flexor digitorum' 47 stucture not forming wall of inguinal canal a aponeurosis of EO b IO c lacunar ligament d cojoint tendon e fascia transversalis 48 structure which prevent movement of unwanted material from passing from one epithelium cell to other? a tight junc b gap junc c desmosome d zona adherens e internuclear material .facebook.com/groups/fcpspastpaper/ https://www.https://www. https://www.facebook. rupture to bulbous urethra urine accumulate in? a superfical perineal pouch b rectovesical pouch .com/groups/fcpspastpaper/ https://www.facebook. BT a vwf def b liver disease c vit k def 53 malignancy with autosomal dominant heritance a child retinoblastoma b ovarian ca c breast ca 54 child treated for malaria develop anemia nd dark urine? a g6pd def 55 neurotransmitr in slow wave sleep a Ach b serotonin c enkephalin 56 oxytocin nd adh originate from? a hypothalamus b post pitutiary 57 2nd pharyngeal pouch derv.? a rami of mandibule b stylohyoid ligament c stylomandibular ligament 58 damage to hypothalamus which hormone will inc? a prolactin b GH c ACTH 59 lung xray show multiple opacities on bronchial wash have plasma cells lymphocyte epitheloid macrophage what organism? a mycobacterium t.com/FcpsPart1BcQs?ref=hl 50 regarding metaplasia chose inappropriate? a is irreversible 51 most common thing causing tumor is? a non lethal mutaion in gene b mutation of proto oncogene 52 cause of inc.b 60. which is benign tumor? a adenoma b hepatoma c melanoma 70.com/FcpsPart1BcQs?ref=hl 61.facebook.hypertrophic cardiomyopathy assoc with? a myocyte disarray 64.facebook.com/groups/fcpspastpaper/ https://www. positive predictive value is? a proportion of true positive in all positive b same as senstivity c same as specificity 67. Fast Pain Mediated by which Fibers ? a A alpha b A delta c C fibers d Highly Myelinated Fibers 66.https://www.child with parotid swelling have right testis enlarge other family member also same complain ? a mumps . receptor for vibration sense? a pacinian b ruffini c misener 68. knife injury to mitd axillary area in 4th intercostal space damage to? a internal intercostal memb b internal intercostal muscle c atria 63. breast tumor of upper outer quadrnt lymph drainage? a apical b anterior 62.child treated for 1% permitherin and also other child with same problem most best? a sarcoptes scabie 71. receptor for pressure on skin? a pacinian b ruffini c hair follicle 69.receptor for withdrawl reflex? a nocciceptor b pacinian c miesner 65. regarding midbrain appropriate? a duct of midbrain is cerebral aqueduct b substantia nigra divide cerebral peduncle into tectum nd crus cerebri c have to colliculi posterirly d CN IV originate anteriorly . which is not a bacterial inf.healthy individual Na balance by ? a osmoreceptor 80.facebook. osmotic pressure of interstitial fluid is difr from plasma by?? a.facebook.on ecg PR nd QRS with ratio of 3:1? a mobitz 1 b mobitz 2 c complete block 76. carbidopa given with levodopa to ? a help dec peripheral inactivation b to help in entering brain 74.com/FcpsPart1BcQs?ref=hl 72.5 73. not given for candida infection? a ketoconazole b clotrimazole c amphotericin b d griseofulvin e nystatin 78.1 osmol/kg water b.PTH hormone regulate calcium by? a inc.? a histoplasmosis other options were bacterial 77.5 c.com/groups/fcpspastpaper/ https://www.3 d. its reabsorption in distal tubule 75.1.which dec heart rate? a brain bridge reflex b oculocardiac reflex 81. muscle arising from femur damage to which causes instable knee joint? a vastus lateralis b rectus femoris c sartorius 79.https://www. which organ form peptide which help in bone growth? a kidney b liver c brain d intestine 90.type 2 hypersenstivity example? a erythroblastosis fetalis 86.https://www.facebook. synapse least likely to present in? . in isotonic exercise which doesnot inc? a heart rate b systolic BP 85. high altitude acclimization causes? a dec erythropoitn b inc pulmonary ventilation c pulmonary vasodilation d metabolic acidosis 84.arthus reaction example of ? a type III hypersenstivity 87.facebook. which is least malignant? a FAP coli 'b tubulo villous c villous d metastatic polyp e tubular adenomatous polyp 91.com/groups/fcpspastpaper/ https://www.com/FcpsPart1BcQs?ref=hl 82. blood supply of brain regulated by? a paco2 b pao2 89. enzyme for serotonin nd epinephrine metabolism? a MAO b compt 92.growth hormone most appropriate? a is polypeptide b inc growth of bone nd cartilage thru somatomedins 88. poisioning not relieved by naloxone? a morphine b pheno c pethidine d heroin 83. PDA is remanant of left? a ventral aorta b dorsal aorta c 6 aortic arch d 5 aortic arch 94.facebook. which is not a liver granuloma? a gumma b lipogranuloma c sarcoidosis d silicosis 98.https://www. ? a Tobacco smoke b asbestosis c silicosis 97. which is slow growing thyroid tumor? a follicular b pappilary 99.com/groups/fcpspastpaper/ https://www. woman with 1 year immune disorder develop right cervical lymphadenopathy what is it? a AIDS b SLE 95. ADH effect most is on? a medullary colecting duct b cortical colecting duct c distal tubule 100.com/FcpsPart1BcQs?ref=hl a sympathetic chain ganglia b dorsal root ganglia c dorsal horn d ventral horn e lateral horn 93.facebook. erythropoitn sec by? a macula densa b cell of proximal tubule c mesengial cell d cell of renal tubul(something like this) . Which carcinogen causes bronchogenic ca. Subcapsular afferent lymph vessel is feature of ?? a Spleen b Thymus c Lymph node 96. Eating behaviour a Inhibited by leptin secreted by adipocyte b inhibited by leptin secreted by muscle c increased by stomach distension d increased by destroying hunger centre 7.com/groups/fcpspastpaper/ https://www. aorta chose incorect? a on right side of chisterna chyli b on left to IVC c divide at l4 d give lateral pair branches to parietes 3.P a baroreceptor b renin angiotensin c ADH 8.identification or difference of left and right kidney ? a relation of stucture in hilum b size nd weight c coverings 6. which immunological test is used for the following conditions? a gonnococal inf b hydatid disease c giardiasis 4.facebook. Tractas solitarius a Carries first order neurons for taste sensations b Carries second order neurons for taste sensations c motor nucleus for tounge muscle 9. third trimester pregnant lady dec cardiac output is in left lateral position to avoid compression of which abdominal structure? a IVC B right lung c liver d spleen 2.in asthma allergen attach to? a mast cell on bronchi mucosa 5.facebook. The single most important local factor affecting wound healing a -infection b ischemia c foriegn body .https://www. short term regulation of B. related to abd.com/FcpsPart1BcQs?ref=hl 1. what is the study? a case-control b Cohort c Cross-sectional 13. Which of the following is a poor prognostic lastfactor for sepsis? a dic b vasodilation c diffuse tissue injury 14.. belly of digastric nd mandible .com/FcpsPart1BcQs?ref=hl d vit c 10. what is the diagnosis? a Pul.facebook. About Active transport: a Always involves Sodium-Potassium pump b Uphill transfer of molecules in a few cells. The cells forming the coronata radiata are: a Theca Interna b Theca Externa c Granulosa cells 12. The digastric triangle is formed by: a The anterior Belly of digastric muscle. c Require Glucose d protien carriers are needed 11.Two groups of Smokers & non-smokers.Embolism b MI 15. 500 each to study the effects of Smoking. post.com/groups/fcpspastpaper/ https://www.facebook..A 75 year old man with COPD is bedridden and develop tacyhcardia & chest pain.https://www. Regarding Fibrocartilage a It has a prechrondrium b It is found in the Pinna c It is found in the Interverterbal disc 16. Which of the following haS MOST Prominent prepotential? a S A node b Atrial muscle cells c Bundle of his d Purkinje fibres e Ventricular muscle cells 17. 18. which inc. pitutiary microscopic findings? a 50% corticotroph b somatotroph appear as small basophilic cells other options dont remember 21. relation to roots of brachial plexus d attach sup. metabolic rate? a inc. regarding scaleni ant. regarding vertebra? a c7 has long spine b lumbar vertebra have heart shape body c thoracic have foramen transversarium d ventral nd neural cure same 23. to subclavian vien b phrenic nerve passes in front c give post. to c3-c6 vertebra 22. most probable to relieve exophthalmus do? a hypophysectomy b thyroidectomy c give drugs to reduce t cell activation d testosterone 25.regarding ant. which structure arches over root of right lung? a azygous b hemiazygo us c arch of . ptnt with hemicolectomy no liver metastasis how to monitor? a CEA b alpha fetoprotien 20. thyroxine levels b anxiety state other dnt remember 24. muscle? a ant. Which of the following complications is he at likely to have? a Dehisce b Contracture c Keloid 19.A 30year old fireman got severe burns. aorta . cord.Parietal lobe..Vertebre..Endoderm 4-Memory centre is located in: Insula.Hyperurecimia 8-Nerve supply to extensors of arm is supplied by: Lat.Parotids(correct)..Medial cord(correct). 11-Common site of fracture at rib is: Angle(correct). 18-Autospy done on the patient having CRF.Submandibular.the artery commonly involved in brain is: PICA.most probably suffering from: Bacterial inf.Hypertrophied pituatry.interosseous A.Crystallines..fasiculata..HyperPTH.meatus.AdenoCA.Hypertrophied thyroids .Radial.Annulus fibrosis.Medulla.disloacation of Shoulder joint is: Musculocutaneous. (correct).functioning of Glom.Infections.pituatary gland causes dec.sense of touch and vibartions.Rectus femoris.tubercle 12-Following is not a tumor marker: PLAP.bHCG. 13-Most common histology found in lung tumors is: Squamous(correct).Viral inf.Mixed.Post cord.Costochondral joint.Sup.Shaft.Cerbral A.Nasal septum 17-Patient having increased levels of IgE.Axillary(correct).Middle meatus.Ant.the ideal fluid replacement would be: Packed RBCs.most prabable finding would be: Hypertrophied PTH gland(correct).Whole blood for 3 days(correct).Ulnar A.. 15-Soldier comes with heavy bleeding.Fungal inf.Soleus 2-Adult derivative of notochord is: Nucleus pulposis(correct).Hypertrophied adrenals.Acid Phospatase(correct).Post.deviation of tongue.CEA.AICA..Inferior meatus (correct).Suprascapular 10-Damage to scaphoid bone causes injury to: Radial A.Plantaris.Whole blood for 18 days 16-Maxillary sinus opens into: Sup..Ant.Cerbral.Frontal lobe 5-The most commom type of salivary gland tumor is: Salivary glands.Brachial A..Colloids.Median.Sublingual 6-Distruction of Ant.D.Lateral and medial cord 9-Nerve injured in Ant.Hyper vit.AFP.Temporal lobe.Parasitic inf(correct).1-The muscle involved in unlocking of knee joint is: Poplitus(correct)..Glom.Adrenal cortex(correct) 7-Least chances of renal stones is associated with: Hyperlipidemia(correct).Ectoderm.Dec.granulosa.Vertebral canal 3-Vertebrae is derived from: Myotome.Spinal(correct).Small CA 14-Patients comes with Rt.Dermatome.Sclerotome. (correct).TLC:3.Fallopian tubes(correct) 35-Least common site for ectopic pregnancy would be at: Ovaries(correct).Greater omentum.Parietal lobe 25-Patient with injury to left 8th cervical segment of spinal cord will not show following sign: Dec sense of position..Openning at bladder 33-Common relation of ureter is: Ant.5/ul.Fallopian tubes.Platelets 30-Myelin in CNS is formed by: Microglia.Occipital lobe.Oligodendrocytes(correct).Inc PTH.Damaging nuclear membrane.Aplastic anaemia(correct) 23-Nucleus ambigus supplies to: Teeths.Schawan cells.K(correct) 22-Young pt.Shivering.12th.having Hb:6.Vit D. .Dec sense of pain and temprature below the lesion on same side.Post.Cystic A and Left Hepatic.IX.Invloving protien synthesis(correct).Dec.to gonadal vessels and ant.vibration below lesion on same side.Lyrnx(correct) 24-Olfactory area is present in: Ant.Stress.21st 21-Patient having dec levels of factorII.Extensor plantar on left side.antithrombin III.Karyolysis.Hunger pangs 20-Foetal period starts after which week: 11th.at PUJ.19-Excitation of post.Vit B12 def.to gonadal vessels and post.Inc.Pouch of douglus.Thyroid.Jaw.Plts:1lac is suffering from: Iron def.perforating material.Only Cystic A..to iliac A.Folate def.Karyorrehxis.to iliac A(correct). 29-Most sensitive cells to hypoxia are: RBCs.0.bone density(correct) 27-Most common site of malignancy in pts suffering from nuclear outbreak Haematopoitic.Ovaries.Cervix 36-Gall bladder is supplied by: Cystic A and Left gastrcA.Neurons(correct).VII. (correct) 26-Feature of Rickets and Osteomalacia is: Inc.Inf.Mitochondrial shrinkage 32-Ureter isnot constricted at: Pelvic brim.Tongue.Psoas muscle(correct).Nephrons.8th(correct).X.WBCs.Astrocytes 31-Feature not associated with irreversible cell injury is: Mylein figures(correct).Ca.Lung(correct)...Dec power of muscles below the leson on same side. 34-Patient feels pain after she underwent surgery for fibroids..Breast..temporal gyrus.most likely have: Dec..pituatary would cause: Sweeting.Bones 28-Virus acts on cells by: Damaging cell membrane.16th.Dec Vit.structure injured would be: Ureters.. cheif.Goblet cells.T4 and TSH levels(correct) 39-Patient having hyperplastic bone marrow.T4.Megaloblastic.22yrs female pt.Colon.Sickle cell anaemia 47-Type of defect in Heredetary spherocytosis is: Enzymatic defect.most likely cause of death is: Amniotic fluid embolism.Ureteric crest.Fat embolism(correct) 53-Patient with old history of adenoCA of colon is operated for polypectomy.Hausstral folds.TSH.Structural defect(correct). 44-Fat tags attached to the walls of large intestine are known as: Taenia coli(correct)..Internal and external V.Prostatic urethera(correct) 50-Terminal part of CBD in relation to pancrease is: Embeded into pancrease.Thalasemia..Inc Platelets is probably suffering from: Aplastic anaemia.anaemia.Bladder..Commom Iliac V.T3.parietal(correct).Ileum(correct).Embolus.Metaplastic polyp 54-Tyrosine derivateve doesnt include: .Hereditary spherocytosis(correct).Primary polycythemia(correct).T3 and T4.lie anteriorly 51-Following would cause massive infarction and destruction leading to patient death: Thrombus. 38-Hypothyroid patient in on thyroxine.Villous adenoma.Fatty thrombus.Gonadal.with 13th week of pregnancy after having crush fractures in RSA dies after 3 days. 48-Patient with jaundice.lie posteriorly..Thromboembolism(correct) 52.VitB12 def..the cells lost in the gastrectomy are: Mucous.epigastric..Iron def.Myelofibrosis.Sec.Leaukemia(correct) 40-Stimulus for Eryrthropoiten secreation is: Hypoxia 41-Erytropoiten is always raised in: Polycythemia vera.polycythemia (Polycythemia vera is only type of polycythemia in which there is Dec..Rectum 46-Following is an autosomal dominant disease is: Cystic fibrosis.on histologic evaluation pathologist labelled it as benign growth with no chances into malignant transformation. 42-Following vein would be dilated in portal HTN: Inf.Appendeces epiplocea 45-Bile salts are reabsorbed from the: Duedenum.Air embolism..External Iliac V.Tubular adenoma.Hemolytic anaemia(correct) 49-Ejaculatory duct opens into: Ureter.high reticulocyte count is having: Folate def.37-Venous drainage of urinary bladder is into: Internal Iliac V(correct). 43-Patient after gastrectomy is on VitB12.Renal vein.Left colic V(correct).anaemia.Tubulovillous(correct).best marker to monitor his thyroid status is: T3. EPO.it would be: FAP.Jejunum.. Intercostal N.Heart.Internal and Innermost layers(correct).Fasica trans.inc Alk.Nor adrnaline.Infrahyoid.the genotype would be: XX. (correct) 62-Femoral sheath is formed by: Fascia transversalis.Axillary. 72-Man is suffering from testicular ca.Inferolateral.Glissons(correct).lymphocyctic infiltartion 69-Thyroid gland moves with swallowing because it is enclosed in: Pretracheal fasica(correct).XO(correct).Investing fasica.and Fasica iliaca(correct).XYY 66-Organ having least chances of infarction: Lungs.most likely suffering from: Pagets..loss of sensation over thumb and index finger.TSH.Cholesterol(correct) 56-Capsule of liver in known as: Disse.+ve AFB sputum..Hyper VitD.Vertebral fascia.Traumatic 68-Pt.Kidneys.with cervical lymphadenopaty.internal oblique.Phrenic N(correct).External oblique.HyperPTH.Gram positive inf.Liquefactive(correct).Gram negative inf.transversus.Spleen 67-Type of necrosis in brain infarction: Couglative..Internal oblique.XY.sign to look for TB on microscopy is: Chronic inflammation.Ant surface..Bone mets(correct).Radial..Osteomalacia 64-External spermatic fasica is formed by: External oblique(correct)..phosp..Autonomic N.Internal oblique 63-Patient with bone pains having normal Ca.Beneath Endothoracic fascia 71-Fibrous pericardium and medicatinal pleura is supplied by: Vagus N. 57-Most common site of thyroglossal cyst is: Suprahyoid.Fat..Transversus muslce 65-In Turner syndrome.Superantigen infection 61-Following doesnt form the layer of inguinal canal: External oblique.Dopamine 55-Precursor of steriod hormone is: Progesteron.Liver(correct).Pregnenolone.rectus abdominus.caseous necrosis(correct).Carotid fascia 70-Neurovascular bundle in chest wall lies between: External and Internal intercostal mucles.Prolactin.XXY.Ulnar 59-The base of urinary bladder is made by: Post surface(correct). Innermost and enothoracic fasica.nerve involved is: Median(correct).Superior surface 60-Shock without having vasodilation is likely associated with: Burns.Cremastric muscle.Hyoid(correct) 58-Wasting of thenar eminence.Adrenaline.the lympahtic drainage of testicle is into: . IgA nephropathy.nodes 73-In exercise.post streptococcal GN(correct)..cause would be: Minimal change disease.he is suffering from: Folate def.ANF 85-Tumor compressing optic chiasma would cause: Binasal haemianopia.Local mechanism(correct) 75-Fisherman is presented with gingival bleeding and ecchymosis.Inflammatory cells 82-Child with sore throat after 3 weeks developed oedema... 83-Microscopic feature of polyarterites nodusa is: Granuloma..Anxiety 3.Metaplasaia(correct).Iron def..internal iliac nodes B.Vit C def(correct) 76-Facor affecting collagen synthesis during healing is: Infections(correct).Copper dust.Dec oestrogen 80-Presence of pancratic tissue in gastric mucosa is termed as: Hamartoma.Vit B12 def.Complete blindness 86-Growth hormone is maximally rasies in: Sleeping(correct).Carbon 78-Child having meckels diverticulum is having bleeding per rectum.it is due to: Volvulus.Neoplasia.haemturia.Lymph drainage of perianal skin is from one of the following A..Parasymphthetic sys.the venous blood returns to the heart by: Muscle pump in calves 74-Circulation in heart is maintained by: Symphthatic sys.Dec.Focal segmental GS.lateral group of horizontal superficail nodes D.Silica.Asbestos(correct).Amenorrhea is due to: Inhibition of LH and FSH thru Prolactin(correct).Hunger.Wound contraction(correct)..medial group of horizontal superficail nodes(correct) E.Bitemporal haemianopia(correct).Fibrinoid necrosis(correct) 84-Angiotension II exerts its effects by activating: Renin.Excercise.Aldosterone(correct).Choristoma.Internal iliac.Lesser omentum .Superficial inguinal.Para aortic L..Ectopic gastric tissue(correct) 79-During lactation.Dysplasia 81-Main difference between primary and secondary intention wound healing is: Granulation tissue.external iliac nodes C.Pre aortic.External iliac L.nodes(correct).Angiotension I.Vit C.Bilateral haemianopia.prolactin.vertical group of nodes 4.Foriegn body 77-Mesothelioma is associated with: Vinyl chloride.Interssusption. 46XY (correct) B.Thymus develops from .autonomic nerves E.ammonia C.urea a B.absent uterus.A normal looking Girl came to you with primary amenorrhea.forms falciform ligament C.related to left kidney E.46XY C.tachycardia E.ilioinguinal nerver B. short stature diagnosed as Turner syndrome.45XO (correct) B. wht is her karyotype? A.on examination she was having short blind vagina wid normal vulva.47XY C.her karyotype is? A.related to rt kidney b 9.wat is true about DNA A.A. scanty pubic n axillary hairs.46XX E 10.connects with deudenum B.Double stranded in which two strands are antiparallel to each other B.Which is most imp indicator of muscle protein loss A.Can exist as helix in single stranded form as well as in Double stranded form D. E.45XO 45.sacral splanchnic nerves D.normal breasts.Pudendal nerves (correct) 15.pelvis splanchnic nerves C.supply of sphincter urethra comes from A.related to left adrenal gland D. a single turn of DNA is 2nm C. ? 40.restlessness 39.47XXY D. 41.Which of the following is not a feature of shock A.A pt came to you with primary amenorrhoea.muslce weakness C.decrease cardiac output B.uric acid E. webbed neck.increased urine output(correct) D.47XXY D.none of above D. mesoderm of 3rd brachial pouch 16.A node E. E.2nd pharyngeal pouch (correct) E.Diaphragmatic hernia occurs due to A.In cervical carcinoma.A. supplies S. D.4th brachial pouch (correct).ectoderm of 3rd brachial pouch D. B. C. absence of pleuro-pericardial membrane absence of septum transversum absence of pleuro-peritoneal membrane(correct) absence of central tendon absence of arch? 18.both internal n external iliac D.3rd brachial pouch D.wat is the lymph drainage A.bone marrow E.2nd pharyngeal cleft C.3rd brachial cleft 20. B.Erythropoises occurs in middle trimester mainly from A.kidney C.1st pharyngeal pouch D. supplies both ventricles D.A pt presented with an opening in the ant wall of sternocledomastoid muscle with pussy discharge coming out of it.internal iliac B.ectoderm of 2nd brachial pouch B.3rd pharyngeal cleft 17.2nd brachial arch C.liver B.this problem occured due to abnormality in one of the following A.arises from anterior aortic sinus (correct).1st pharyngeal cleft B.endoderm of 2nd brachial pouch C.Superior parathyroid gland develops from A.RT coronary artery A.Most imp hormone involve in gluconeogenesis is .both liver n kidney 19. supplies left atrium 25.superficial inguinal E.enters the rt heart border after passing through sulcus on posterir surface C.spleen(correct) D.deep inguinal (correct) 21.external iliac C. E.1st brachial arch B.endoderm of 3rd brachial pouch (correct) E. he is diagnosed to have acute appendicitis on flexing his rt thigh inward n medially he cries with pain.retrocolic C.pelvic (correct) D. Activation of P-53 D.anticentromere antibodies E.insulin C.A. Bcl-2 inhibition C.C3b n C5a are anaphylactant E 34.subcecal 1.anti SS antibodies B.paracolic B.Apoptosis is inhibited by A. B.antimitochondrial antibodies .C3a is opsonin B.anti DS antibodies (correct) C.paracecal E.C5a is potent anaphylactic C.C5a (correct) C. apoptosis related protein ?? E… 32.cortisol (correct) E.thyroxine D.rash n joint pain.arachdonic acid metabolites 33.growth hormone B.Most potent chemotactic factor is A.Pt presented with photosensitivity.dermatomiositis 35.what is the most appropiate answer A.SLE B.capsases activation (correct).A pt presented with rt iliac fossa pain.RA D.C5a is a potent chemotactic agent (correct) D.wat is the type of appendicitis he has A.IL-1 D.rheumatid arthritis D.which is the daignostic in this case A.multiple sclerosis E.ankylosing spondiolitis C. Glycogen 26.C3a B.Low serum complement seen in which condition A.viruses E. 55.ESR is decreased with A..ammonia 54.largest of all cells E.karyotyping (correct) D.macula densa of kidney D.uric acid(correct). C.Digoxin (correct) B.Ct scan E. B.ultrasound B..urea and water B. C.have nucleus D.albumin is decreased (correct) C.47.nonfragile C.RBC's are A.End product of purine metabolism is A.biconvex B.decreased plasma volume 56. E. E.temprature B.have glycolytic enzyme activity (correct).liver cell B. D.water and ammonia E. VLDL HDL IDL 49.barr bodies C.peritubular interstitium of kidney(correct). .Which of the following has highest cholesterol content A.Griseofulvin C.Androgen E.decreased blood volume E. 53.Which is the best option in identification of Turner's syndrome A.Which drug do not cause gynaecomasia? A.globulin is decreased D.spironolactone 50.Cimetidine D.Erythropoitin is secreted from A.. Chylomicron remnants LDL(correct). C..glucose D. mRNA has anticodon C.increase in size n no of cell C. E.. 61.cytplasm n nucleus ratio is derranged D.tRNA is largest E.are the same as parental cell C.infections D.increase in size of cell B.tRNA involve in protein synthesis 65.subacute degenaration of spinal cord E.toxin C.chronic irritation (correct).Wat is the differentiating point of hyperplasia fron hypertrophy A.mRNA has a codon B.muscle wasting (correct) C.deficiency of THAIMINE(B1)causes all except A. 64.What is the most true among following A.radiations E.cns abnormalities in alcoholics ..Dysplasia is seen mainly seen in A. C.increase in no of cell (correct).is premalignant ??????? E...cartilage C.Commonest Cause of PDA A. B.defects in alcohlics D.congenital abnormality E.drugs B.involve viscera only 62.bone marrow (correct) B.deep tissue 63..epithelia D.59.Wat is the differentiating feature of hyperplasia from benign tumor A.rRNA is most abundant n involve as ribosomes on endoplasmic reticulum(correct) D.fibrous tissue E.is reversible D.infection 60.ultrasonography B.cardiac anomalies B..Most common Cause of metaplasia is A.permaturity (correct) D. thiamine D.channels 71.CO2 is tranported in lungs through A.12 B.niacin 68.fasilitated diffusion C.transferrin(correct).STD E.folic acid (correct).CA vulva C.pyridoxine E.HSV is associated with A.iron B.cardiac anomalies? D.66. D.ascorbic acid(correct).congenital deafness? C. B.VITAMIN involve in collagen synthesis is A.AIDS is associated with all of the following except A.simple diffusion (correct).primary tumor of brain (correct).leukemia 67.52 75.lymphoid tumors C.Iron is stored in the form of A.32 D. B.biotin C. B.CA cervix B.vulval papules? 74.ferritin B.vaginal wart D. What is the most appropriate answer A. C 69.apoferritin 70.Optic groove appears on left side of forebrain on day A.congenital cataract (correct).42 E.limb deformaties E .active transport D.22 (correct) C.kaposi sarcoma B.Germen measles causes which abnormality.Neural tube defects occur due to deficiency of A. C.carriar proteins E. .sulphonyl urea B.Psudomembranous colitis is caused by A.E coli? B.is caused by fungal B.Cl.insulin (correct)..intestinal malformation 77.eosinophils C.omphalocele (correct) B.neutophils (correct).partial parathyroidectomy 78. 86.Which of the following is opportunistic organism A..difficile (correct) B. D.wat type of cell will present at this time of disease A.now he developed same pain again.Treatment of diabetes in pregnant lady is A. 84... 83.A newborn with anterior abd wall swelling and umbilical cord attached to it.CL perfiranges D.biguanides D.klebsella E.76.hypophysectomy D. D.wat is the condition called A.side effect of streptomycin A.Cl.direct suppresion of T4 (correct) C.lymphocytes . C.administration of testosterone E.impairment of hearing B.chlostridium (correct) 85.umblical hernia D. C.usually seen in preexisting lung diseases? C.shigella D.Cl.gastroschiaosis C.About aspargillosis which is not correct A. E.drugs blocking the action of T4? B.botulinum E. E 97.salmonella C. B.PT has exophthalmos n his T3 n T4 are increased.omentocele E.is caused by long term antibiotic use.wat is the option for treatment of exophthalmos A.tetani C.suphonylurea n biguanides E.Pt has a history of infection 4 weeks back. 45XO C.group O with B 104.actinomycosis B.Ca-Calmodulin complex is present in smooth muscles 109.basophils E.mefanamic acid 102.aspirin B.D.Most common organism involved in infection caused by IUCD A.testosterone D.estrogen C. high conc.mast cells 98.heparin (correct) C.Ig M (correct).glucose B.47XX .ECF differ from ICF in A.warfarin D.group A with O B.diffrence of smooth n skeletal muscle A.bacteroide D.trichomonas 107.Type 2 hypersensitivity reaction includes A.amino acid C.antiandrogens (correct) B.low Na n Ca high K D. Of K & Ca with low Na E. of Na n Ca (correct) B.Which drug is used in the treatment of hirsutism A.group A with AB (correct) C.E coli C.CO2 105.group AB with A D.Transfusion reaction will not occur in a pt if we transfuse the A.karyotype is A.erythroblastis fetalis all other options were wrong 103.gonorrhea E.indomethacin E.46XX B. E.Volume … 108.In partial mole.What is the drug of choice for DVT in ist trimester of pregnancy A.Ig G D.Which of the following does not cross placenta easily A.cryptoterone acetate E.diacrone 106.group B with O E.high conc.high K low Na C. 1 Paracetamol a) increases PT b) its overdose causes Nephrotoxicity c) is a poor anti-inflamatory (correct) d) is more stronger than codeine e) causes met-Hb.If carbohydrate is separated from proteins wat will be the fate A.D.eliminate fron kidney? C.Na D.emia more frequently than Phanacetin Q.. hydrolases (correct) E..alkaline phosphate 122.urea but normal s.Lysozome contains A.3 Congenital cataract is associated with a) chiken pox b) chromosomal abnormality c) small pox d) Rubella (correct) Q.creatinnine d) Q.. urine in initial stages (correct) c) increase in s.47XY E. a) Hyperkalemia b) conc.aggregation n precipitation (correct) B.69XXY (correct) 110..5 All of the following have ability of phagocytosis EXCEPT a) Microgila b) Lymphocytes c) Kuffer cells (correct) d) Neutrophils . Following is Autosomal Dominant Disorder a) Phenylketonuria b) Hereditary Spherocytosis (correct) c) Hemophilia A d) Cystic Fibrosa Q. a) Lewy bodies b) Patchy demylination & white fiber gliosis c) axonal neuropathy d) (correct) Q.increase uptake int muscle 111.2 ATN is associated with.glucose B.4 MS is characterized by.Ca C. a) Hemolysis b) Vit B 9 def. Hb= 7.7 in an Asthamtic female. on investigation he is having Un-conjugated Bilirubinemia. the long needle goes far into greater palatine canal.causing anaesthesia of an autonomic ganglion.11 Metabolic Alkalosis is caued by a) Chronic Hypoventilation b) Pancreatic Fistula (correct) c) hyperglycemia d) ATN Q.6 Pulmonary Arterial pressure increases with. the anemia is due to . ..12 Glucronide conjugation of drugs a) causes inactivation of drug b) is Type 1 reaction c) makes drug insoluble in water (correct) d) increases its effects Q.immeditae treatment of HPTN during intra-operative HPTN..4 md/dl MCV. a) Essential HPTN b) Sympthatic stimulation c) Nitroglycerine d) Excercise e) Hypoxia (correct) Q. c) Vit B 12 def.13 A patient having generalized body weakness & numbness.venous nitroglycerine (correct) c) propranalol d) Q.9 Tissue which has in-ability to replace dead cells a) Heart (correct) b) Kidney c) Liver d) Bone e) Bowl Mucosa Q..10 Non-essential amino acid includes a) Leucine b) Iso-leucine c) Methionine d) Tyrosine (correct) Q..8 During giving local anaestheia.Q.. is a) sublingual glycerine trinitate b) intra.as a result along with dryness of nasal mucosa also occurs a) decreased secretions of Parotid b) decreased secretion of Submandibular & sublingual glands (correct) c) Q.> 120 f l . the confirmatory test of the diagnosis will be a) Tensilon Test b) Ach Receptor Antibody Test c) blood culture d) CT scan Brain Q.S4 b) Pelvic splanchnic Nerves (correct) c) vagus nerve d) Q.15 Cause of joint pain in Deep sea divers a) arterial embolism b) venous embolism c) DIC d) thromboembolism Q.22 A young man had sinjury to the neck with sum sharp object.20 which does not take part in inflamatory reactions a) Adrenaline b) Histamine c) 5-hydroxy tryptamine d) Dopamine (correct) Q.S3.14 Tissue with Dual autonomic supply but not reciprocal suuply is a) Skin b) Blood vessels (correct) c) Pupil d) Salivary glands e) Ciliary muscle Q.in a fasting man the cause of concenterated urine is a) Skin (correct) b) Blood vessels c) Pupil d) Salivary glands e) Ciliary muscle Q.16 In summer..d) Hypersplenisim (correct) Q. .. Now he notices a bulging swelling above the clavicle on sneezing & coughing. 21 Parietal & Visceral Pericardium is supplied by a) Phrenic Nerve (correct) b) Sympthatic Nerves c) Vagus neve d) Cardiac Plexus Q.18 if a High Dextrose water is infusion given to a person.3 yrs ago.17 A 35 yr old man is having bilateral ptosis of eye during evening since last few weeks.it inhibits a) Cotisol b) ADH (correct) c) Aldosterone d) Insulin Q. this swelling most probably due to.19 Parasympthatic supply of Sigmoid Colon is a) Sacral segments S2. 27 A Transplant from identical twin is a) Allograft b) Heterograft c) Homograft d) Isograft (correct) e) Xenograft Q.29 Pharmakokinetic interaction among drugs is examplified by : a) Shortening of action of Procaine by Adrenaline b) increase peripheral toxicity of Levodopa with Carbidopa c) increase toxicity of Methotrexate by Aspirin d) prevention of Nitroglycerine induced tachycardia by Propranolol e) blokade of acetylcholine receptors by atropine Q.0.the Haemorrhage will be a) Epidural (correct) b) Subdural c) Intracranial d) Q.30 A 30 years old 75 kg Diabetic man with recent diagnosis of Hypertension.25 Muscles of Hand supplied by a) ventral rami of C-8 (correct) b) Ventral branches of T-1 c) Median Nerve d) Ulnar Nerve e) Radial Nerve Q.5 mg/dl s.Urea -.140 mmol .26 A person has sharp knife injury to upper medial aspect of arm.Uric Acid-. On ECG : Left Ventricular Hypertrophy s.7 mg/dl s.20 mg/dl s.Creatinine -. after which He is unable to Flex his elbow & having loss of cutaneous sensations on lateral aspect of forearm.7.24 Dorsal Rami of the cervical nerves innervates a) Abductor Pollices b) Extensors of Limbs (correct) c) Extensors of Trunk d) Stabilizers of Shoulder Q. the nerve injured is a) Median Nerve b) Axillary nerve c) Radial Nerve d) Musculocutaneous Nerve (correct) Q.28 Therapeutic Index of a drug indicates its a) Effectivity b) Efficacy (correct) c) Potency d) Toxicity Q.23 Middle meningeal artery if ruptured while meinges are intact.a) Injury to deep fascia of neck b) demage to supra-pleural membrane c) Un-united first Rib Fracture (correct) Q.Na -. 35 What is INAPPROPRIATE about Vagus Nerve. a) Lewy Bodies in Neucleus b) Ptachy demylination with white fiber gliosis c) Axonal degeneration d) (correct) Q.80 mg/dl The cause is a) Essential HPTN (correct) b) Diabetic Nephropathy c) Pheochromocytoma d) Addison Disease e) Primary HyperAldosteronism Q.. a) its longest Cranial Nerve b) passes in between IJV & ICA c) passes posterior to Carotid Sheath (correct) d) Gives Recurrent Laryngeal Nerve e) gives Superior Laryngeal Nerve Q.38 Kleinifilter syndrome a) contains single X chromosome b) less chances of Mental retardation (correct) (correct) .5 mmol s.23 mg/dl 24 Hr Urinary Proteins-..Cl --.32 GFR is measured by clearance of a) Inulin (correct) b) Creatinine c) Urea d) Glucose Q.36 Thymus is a) Rich in Lymphocytes (correct) b) Present at birth but shorly regree after birth c) derived from 4th Pahryngeal Pouch d) is a part of Thyroid e) Lies posterior to Trachea Q.Urea is increased & s. K-..31 Acute Tubular Necrosis is characterized by a) Hyper-kalemia b) Initially concenterated urine (correct) c) Metabolic Alkalosis d) s.HCO3 -.103 mmol s.s.33 Sarcoidosis is diagnosed microscopically by a) Granuloma with Asteroid b) Non-caseating Granuloma c) Caseating Granuloma d) Macrophages & Giant cells (correct) Q.30 & Fasting BSL--.37 True statement about Down Syndrom is a) Trisomy 23 b) contains single X chromosome c) more chances with increasing maternal age Q.2.creatinine is normal Q.34 Multipule sclerosis is characterized by. 46 All are the branches of Vagus nerve EXCEPT a) Auricular b) Lacrimal (correct) c)Pharyngeal d) Recurrent Laryngeal .c) phenotypically female Q.40 At 35 weeks of Gestation.the site woud be a) Stomach b) Sigmoid Colon c) Transverse Colon d) Ascending Colon (correct) e) Small intestine Q.43 @ Leprotomy Retroperitoneal Abscess was found.its most probable cause is...in sample taken from amniocentesis. a) recieves Lymph from both right & left sides b) lies in posterior mediastinum (correct) c) passes just right to the thoracic aorta d) extends from upper abdoment to neck e) has a dilatation called Cisterna Q..44 Source of Estrogen & Progesterone in last trimester is a) Chorionic Villus b) Placenta (correct) c) Overy d) Adenohypophysis e) Corpus Leuteum Q. a) Down syndrome b) Turner syndrome c) Spina Cystica (correct) d) Hydrocephalous e) Q.41 Thoracic Duct.45 A patient having Facial deviation along with loss of sensations on anterior 2/3 rd of tongue..the leision lies in a) Facial Canal (correct) b) Internal auditary meatus c) Facial Nerve Nucleus d) Stylomastoid Foramen Q. there is increased Alpha-Fetoprotein.42 Rate of passage of food in esophagus depends upon a) Gravitational Force b) Neuromuscular excitation different @ upper & lower oesophagus c) progressive peristalsis in esophagus (correct) d) Acidity Q.39 Following is Autosomal Dominant Disorder a) Phenylketonuria b) Hereditary Spherocytosis (correct) c) Hemophilia A d) Cystic Fibrosa Q. 53 When a branch of Pulmonary Artery is blocked by embolus.51 Von-Willebrand Disease is a) Autosomal Dominant (correct) b) Autosomal Recessive c) in some family members asoociated with Factor IX deficiency d) in some family members associated with normal Factor IX Q.the Bitemporal Hemianopia occurs due to leison of a) Central part of Optic chiasma (correct) b) Optic Nerve c) Optic Tract d) peripheral Part of Optic chiasma e) Lateral Geniculate body Q.54 Difference between Adenoma & Hyperplasia is a) increase in size of cell .DD) Blood group.51 A male wd A+ (Ao.52 Vital Capacity a) measured directly by Spirometry b) is decreased in some Lung diseases c) is the sum of Tidal Volume+ERV+IRV (correct) Q.Q.Dd ) blood group.48 In a Pituitary Adenoma.49 Muscle of Forced Expiration is a) Internal Intercostal b) External Intercostal c) Sternoceidomastoid (none of them) d) Scalenus Anterior e) Scalenus Medius Q.following Lung function increases a) Alveolar Co2 b) Alveolar O2 c) Pulmonary artery O2 d) Pulmonry artery Co2 (correct) e) Ventilation Perfusion Q. married to a female having B+ (Bo.50 Na Retension is associated with a) Dehydration b) increase ADH (correct) c) Decreased Aldosterone d) Heart Failure e) Hyperthyroidism Q.47 Following nerve lies in the Cavernous Sinus a) Opthalmic b) Occulomotor c) Abducent (correct) d) Trigeminal Q.children can have all blood gp EXCEPT a) AB +ve b) AB –ve (correct) c) A + ve d) B +ve e) O +ve Q. following is correct a) K is 98% in the cells b) normally completely reabsorbed in Distal tubule c) increase water affects K-balance (correct) d) decrease in D.61 Stomach gets its arterial supply from all EXCEPT a) Ceiliac artery b) Splenic Artery c) Superior Mesenteric Artery (correct) d) Hepatic Artery Q.60 An unconsious man with pin-point pupils not responding to Naloxone.62 The Hormone which stimulates release of HCO3 from Pancreas without stimulating pancreatic enzyme secretions. a person has inability to stand from sitting position.b) increase in No.the probable cause is a) Heroin (correct) b) Hasheesh c) Morphine d) Phenobarbitone Q.63 Supra-Renal Gland arterial supply a) recieves all braches directly from Aorta (correct) . the nerve demaged is a) Infragluteal (correct) b) Supragluteal c) Schiatic d) Femoral e) Obturator Q.is a) CCK b) Secretin (correct) c) Gastrin d) Pepsin e) Mucin Q.58 About K .57 After a trauma. of cells c) Capsule (correct) d) Q. d) PPI e)whole blood (correct) Q.55 In a CLD patient presenting with Haematemesis.the preferred immediate treatment is a) Octeriotide b) Vit K c) formed Platelet Conc.K Q. the immediate treatment is with a) Carbamazepine (correct) b) Valporate c) Lorazepan d) Q.56 Among most important initial steps in apoptosis a) Activation of Bcl-2 Receptors b) Activation of Caspases (correct) c) Water influx into the cell d) Q.Regulation .59 A man with persistant fits without regaining consciousness for 30 minutes. .70 Basal Cell Carcinoma involves a) Buccal Mucosa b) Hard Palate c) Soft Palate d) Lower Lip e) Oral Cavity (correct) Q.65 Which infection is not caused by DNA virus a) Chicken Pox b) Herpes Simplex c) Herpes Zoster d) Mumps (correct) e) Small Pox Q..given Parenteral Vit B-12.68 On standing from a sitting position.64 Stenson's duct a) arises from posterior surface of Parotid b) pierces the Buccinator muscle c) opens in the oral cavity against ..it is a) Squamous Cell CA b) Adenocarcinoma c) Rhabdomyosarcoma d) wart (correct) Q. the inability to absorb B-12 is due to loss of. a) Parietal Cells (correct) b) Gastric Cells c) Cardiac Ceell question no 46:mothers milk differs from cows milk in that it contains: .71 A female underwent Gastrectomy for Zollinger-Ellison Syndrome.69 Metastatic Calcification occurs mostly in a) Kidney (correct) b) Bone c) Lung d) Intestine e) Pancreas Q.@ apex of lung there is increase in a) Blood Flow b) Ventilation c) pO2 d) pCO2 Q.b) recieves blood from 3 sources c) recieves branches from both renal arteries d) recieves blood from inferior phrenic artery Q...a plaque like leision seen on Postero-superior aspect of Vagina. Q.66 On examination of a female.67 About CSF most appropriate statement is a) it is ultrafilterate of Plasma b) has more glucose than blood c) has cushioing effect (correct) d) absorbed in Choroid Plexus Q. underlying cause: a)congenital adrenal hyperplasia (correct) b)chromosomal defect c)failure f development of genital tubercle . lipids n proteins in cells is also known as hormone of abundance: a)growth hormone (correct) b)cortisol c)thyroid hormone d)estrogen e)progesterone question no 50:hyperglycemia is caused by: a)insulin b)glucagon (correct) c)estrogen d)progesterone e)growth hormone question no 51:ketoacidosis is caused by: a)increased production of glucose b)decreased production of glucose (correct) c)decreased utilisation of carbohydrate in body d)insulin excess question no 52:insulin secretion is stopped by :a)glucagon b)decreased k levels c)somatostatin (correct) question no 53:hormone that stores carbohydrates.most definitive diagnostic test: a)presence of barr body b)chromosomal analysis (correct) question no 62:14 yrs girl has primary amenorrhea.he has excess of: a)ACTH b)aldosterone (correct) c)cortisol d)estrogen e)progesterone question no 49:hormone that stores carbohydrates.usg revealed absent uterus.a)calcium b)lactose (correct) c)proteins d)vitamin D question no 48:patient has endocrine disorder has metabolic alkalosis and hypokalemia.short blind ending vagina.secondary sexual characteristics normal.normal breast development.slight pubic hair.lipids n proteins in body is: a)growth hormone (correct) b)thyroid hormone c)cortisol d)insulin e)PTH question no 61:turners syndrome. Regarding palm: Superficial palmer arch is below palmer apeneurosis. A 10 year old boy with shoulder injury lost abduction of his shoulder up to 30 degree which muscle is lost: Supraspinatus Deltoid (correct) 02. Which of the following muscle have dual nerve supply from median and ulner nerve: Flexor digitorum profundus. Which of the following muscles is flexor at hip and extensor at knee: . 6. abduction of thumb with intact sensation due to: Injury to Radial nerve at elbow 0 3 .d)defect of paramesonephric duct question no 65:repeated blood transfusions (AB n Rh compatible) in an individual will lead to: a)haemochromatosis of liver b)hypocalcemia (correct) c)haemoglobinuria d)transfusion reaction question no 66: Rh incompatibility occurs in a mother if she has: a)Rh negative fetus (correct) b)Rh positive fetus question no 67:the best way to prevent Rh isoimmunisation in a woman who has given birth to Rh positive fetus: a)blood transfusion b)platelet transfusion c)serum d)Rh immunoglobulins (correct) e) question no 68:a person brought in er had burns.he was admitted later on he developed fever and bleeding diathesis. Infection of the first pulp space lymph nodes will first drained. 5. 0 7 .he is suffering from: a)DIC (correct) b)hepatorenal failure c)hepatic failure of prothrombin production d)renal failure question no 69:following is not a cause of DIC: a)pre eclamsia b)amniotic fluid embolism c)cervical carcinoma d)abruptio placentae (correct) e)PPH 01. Which of the following muscle is not supplied by median nerve: Adductor polices. 04. Epitrochlear lymph nodes. A boy is brought to a hospital with injury at elbow joint lost extension of medial 4 fingers at MTP joint. 16. A. Which of the following structure arches over root of left lung: A. 1 7 . 9. Triangle: Base is form by middle 3rd of clavicle. 12. In an injury to knee joint a man is unable to extend his knee the root value affected: L3-L4. 21. 13. Lies in epidural space. Parasympathetic nerves supplying the urinary bladder are: A. Border of thyroid cartilage. 8. Subclavian artery grooves over: A. A pathologist wants to pass dye to the liver he will reach thru: A. Middle rectal artery. In the injury to neck of fibula the artery damaged: Ant. 18. T3-4 . Transversalis fascia. Ligamentum Teres. It enters the thoracic cavity thru esophageal opening. A 40 years old man feels pain in his gastrocnemeus muscle after he walks 100 meter which relieves when he stops walking is likely due to obstruction of: A. Regarding post. Post. Pelvic splanchnic nerves. Angle of the rib. 19. 14.Sartorius. Vertebral venous plexus: Has no communication with cerebral sinuses. The branch of internal carotid artery which remains in true pelvis throughout its course: A. What is inappropriate about azygus vein. Weakest point of rib is: A. Tibial artery. Border of sternocladomastoid at the sup. 20. Is formed by two vertebral veins. (correct) 10. During a per rectal examination the examiner will not reveal: A. Tibial artery. 22. Nerve supply to the lower airways: A. Arch of aorta. Ureter 23. 1st rib. 15. Internal spermatic fascia is a continuation of: A. 1 1 . Bifurcation of common carotid artery: Beneath the ant. Great saphenous vein has how many valves: 20. 24. Peroneal artery. . 1 2 . Derivative of Ectoderm include: A. Lateral umbilical ligament. Sphincter pupili muscle. 2. What will happen immediately after birth: Anatomical closure of Ductus arteriosus. 9. Embryonic period is: A. Secretary Epithelium of parotid gland. Slow growing tumor of head of pancreas will compress: A. 610 weeks. Anatomical closure of foramen ovale. Common Bile Duct. 1. left umbilical vein will regress to: A. Superficial perineal pouch. (correct) Illiohypogastric nerve. Tumor of the head of pancreas will compress: A. Anatomical closure of Ductus venosus. Regarding Mesothelium: A. Physiological hernia occurs in between: A. (correct) Obliteration of Left umbilical vein. During surgery of right colon for carcinoma which structure would not be injured: A. Regarding Allantois: …… 7. Lines the body cavities. 11. Aorta. During an injury to the bulbar part of urethra the urine will extravasate to: A. Regarding Decidua: A. EMBYROLOGY. 6. Which of the following bone is derived from 2 n d Pharyngeal a r c h : A. 4. 27. 38 weeks. Which of the following is a derivative of neuro ectoderm: A. Nerve supply to the distal scrotum is thru: Illioinguinal nerve. 29. Obliteration of Right umbilical vein. Styloid process. 26. Decidua basilis forms the maternal part of placenta. Ligamentum Teres. 10. Common Bile Duct. 28. Genitofemoral nerve.25. 8. 3. Which of the following structure is not an Embryological remnant: A. What is the suitable I/V fluid for the patient of acidurea: Normal Saline. Primordial Germ cells are derived from: A. 7. 12. 3. 2. Albumin. 10. There is rapid influx of Na+. B. Indirectly measures left atrial pressure. Will be inherited by Autosomal recessive pattern 9. ADH will act on: A. Will not found in any other tissue than blood. Sympathetic stimulation (correct) 5. Muscles innervated by hypoglossal nerves are derived from: A.13. Yolk sac endoderm. Cortisol. O+ve. Cushing syndrome. ????? 15. Pulmonary Wedge Pressure: A. . Ventricular Depolarization on ECG strip is represented by: A. Blood flow to the left ventricles is increase by: Acetylcholine infusion. Ringer lactate. QRS Complex. 4. 12. 1 4 . Simple Diffusion depends on all of the following factors except: A. Blood group antigen: A. 14. Inhibitory factor released by hypothalamus against which of the following hormone: Prolactin Growth hormone. ???? Pyloric stenosis. IPSP is generated by opening of: A. Water. 5% dextrose 10% dextrose. 1 1 . 16. Which of the following hormone is called STRESS HORMONE: A. Bain Bridge reflux. 13. Which of the following causes Hyperkelemia: Zollinger Ellison syndrome. Ventricular preload is measured by: A. Conn`s syndrome. Proximal Tubules. 17. Renal absorption of glucose thru secondary transport with sodium occurs at: A. 8. LVEDV. ESR will increase with the decrease in: A. Distal Tubules. Occipital Somites. …………… 6. If father’s blood group is B+ve & mother’s blood group is AB+ve there child can not have which of the following blood group: A. Bile of the Liver differs from the GB bile because GB bile contains decrease amount of: A. During Depolarization: A. Magnetic field. Clchannels. (correct) Dorrow`s solution. PHYSIOLOGY 1. 4. The most probable underlying pathological process is: A. Sustained rhythmic reflex tremors induce by sudden movements is: A. Gangrenous Necrosis. Increase vasomotor tone. Fever in inflammation is caused by: A. Clonus. Cortisol decreases which of the following cell: A. Irreversible cell injury starts with: A. Type II pneumocytes. 27. Cardiac muscles are prevented by tetanization due to its: Rythmicity Automaticity Conductivity Long refractory period. 28.18. 33. Most active form of thyroid hormone present in circulation is: A. A 36 years old women presented with acute abdomen. Regarding Bradykinin: It formation is activated by killkeran. 31. Bile Salts. Leukocyte Adhesion. GENERAL PATHOLOGY 1. Which of the following is not a mediator of acute inflammation: A. Superimposed infection on necrosis is called: A. Heat loss mainly depends on: Temperature of the surrounding. Regarding Interferon: ………. Dopamin. Transfer of Cl. . Increase peripheral resistance is due to: A. IL1 & TNF α. 20. (correct) Evaporation. Rupture of the Lysosomal membrane. Sharp odors have the quality of water & lipid solubility. Hepatocytes. 26. 30. Resection of distal ileum will impair the absorption of: A. 25. 3. T3. Skeletal muscle. Which of the following cell can never reproduce: Erythrocyte. Regarding conduction of visual pathways: ………. 24. At laparoscopy most of the bowl loops were dark purple black. Lymphocyte. 6. Unilateral Anosmia is due to: A. Her mesenteric veins were patent. (correct) 29. Smooth muscle. Wet Gangrene. Affection of Nasal mucosa.. ICAM & VCAM mediates: A. 2. Regarding olfaction: A. Which of the following is the NOT non dividing cell: A. Chloride shift means: A. 22.in erythrocytes in exchange of HCO3. 32. (correct) 19. Surfactant is released by: A. Neuron. 23. 5. 21. Cystic hyperplastic endometrium 20. the diagnosis of Tuberculosis in this patient is confirmed by: A. 14. blind ended vagina with no uterus & ovary. A patient presented with enlarge lymph nodes. A 16 years old boy deeply jaundiced presented with gum bleeding due to: A. 1 Year. 2 1 .. Dystrophic Calcification is seen in all of the following except: Malarial parasite. B. 8. Which of the following is a premalignant condition: Condyloma Metaplasia Endocervix. Hydatid Cyst. 16. Lungs. After 2 days of accident she suddenly collapsed & died. 23. 13. 1 9 . Dead fetus. Edema due to increase hydrostatic pressure is seen in: A. Warthin`s tumor. RBC. 10. 6 days. Fat Embolism.7. 46 XY. . Leukoplakia. 11. Congestive cardiac failure. 3 Months. Which of the following is a benign tumor: A. 15. Regarding Fat Embolism: It is Fetal in >80% of cases. The primary source of Creatinin is: Liver. What is inappropriate about fat necrosis: A. 9. The most probable pathological process involve in her death: A. 1 7 . Her probable Karyotype is: A. 12. Erythroplakia. Fat necrosis occurs in: A. Over an unpredictable period of time. Metaplasia does not occur in: Brain. A 25 year old lady with 14 weeks pregnancy had a road traffic accident brought to the emergency department with a large open wound on thigh and femur fracture. 22. Fluid accumulates in acute inflammation contains: A. Which of the following is a premalignant condition: A. Proteins >3gm/dl. An 18 year old girl came to family doctor complaining of primary amenorrhea. Acute pancreatitis. 1 8 . IT manifest within 12 hours. on examination she found to have well formed breasts. Skeletal muscles. Vitamin K deficiency. Vitamin A deficiency. Cardiac muscles. Presence of caseous necrosis in lymph nodes. Maximal tensile Strength of a wound is attained in. Only caused by trauma to the fat tissue. A 30 year old women brought to emergency department with bleeding…. 10 days. Which of the following is peculiar for crohn`s disease is: A. respiratory failure is due to: A. Adenocarcinoma. 5. A 6 year old boy is complaining of proximal muscular weakness was found to be Ca++ channel antibodies positive.24. 9. Increase plasma volume. 10. Toxic shock 3 0 . Vinyl Chloride. Ventilation / perfusion mismatch. Effects of hyper parathyroidism in Ca. Left ventricular failure will lead to: A. SPECIAL PATHOLOGY 1. The mechanism of injury of ionization radiation is: A. 14. Resection of Anterior lobe of pituitary will lead to: A. diagnosed as case of gas gangrene dies due to: A. 7. 6. 12. A. 26. Immediate effect after injury to a vessel is: Vasoconstriction 16. Infective Endocarditis. Which of the disease is common in I/V drug abusers: A. CEA. heat intolerance on examination her pulse is 112b/min. Regarding autosomal recessive disorders: ……………… 25. She is most probably a case of: A. 11. 2. Anemia of pregnancy is due to: A. Hepatic hemengioma is associated with: A. R/R 20 br/min. BP 120/80. Usually asymptomatic. 8. Which of the following is the confirmatory test of A I D S : Western blot. Free radical formation. Biochemical changes 29. Mysthenia Gravis. Hyperthyroidism. 4. 3. There is increase PO4 renal absorption. In a patient there is atrophy of submandibular gland due to its duct obstruction. 27. ELISA. Hepatocytes. Apoptosis. Major complication of severe burn: Formation of granulation tissue. The most probable diagnosis is: Lambert Eaten Syndrome. Embryological marker that reappears in circulation in Ca Colon is: A. Typhoid carriers are: A. Atrophy of the gland is because of: A. Increase pulmonary arteriolar Pressure. A patient admitted to the hosp. 13. lung is due to: Parathyroid like protein Parathyroid hormone . What is Inappropriate about hyper parathyroidism: A. LASER act by: Cutting. Perianal Lesions. Which of the following are not non-dividing cells. Decrease glucocorticoids. Carcinoma formation. Intestinal metaplasia due to reflux esophagitis leads to: A. 28. In pulmonary embolism. 15. A 25 year old lady complaining of palpitations. Pethidine. 8. By aspiration of trophozoits of Entameoba histolitica. diagnosed as Klienfelters syndrome. 19. During surgery antibiotics should be given at: A. After a traffic accident a lady is brought to a hosp the IMMEDIATE step you do: A. Gout. 18. 20. The most likely diagnosis is: A. In Hypothyroidism there is increase in: A. H2 Receptor. 4. I/V nitroglycerin. Which of the following is suitable antihypertensive for asthma & IHD patient during surgery: I/V sodium nitroprusside. Basal cell carcinoma. Septic arthritis. 5. (correct) PHARMACOLOGY 1. Ameobic infection reaches lungs via: Direct extension from liver. Thyroglobin. The karyotype involved is A} 45 XO B} 46 XX . B. 6. It relieves pain as well.17. In thyroid disease the antibodies are directed against: A. C. Via Portal vein. Malignant Tumor that will not metastasize: A. Clot propagation. Clear airway. Spleenectomy will help in treating which type of anemia: A. gram stain = negative. A 50 years old man is awakened in the middle of the night with acute pain & tenderness of the right knee. Labetalol. Metaclopromide. Hereditary Spherocytosis. 22. tender & swollen. Paracetamol 9] A male with Gynaecomastia. 7. B. small testes. 9. Cholesterol 23. Ketamin is used as anesthetic in repeated dressings of burn patient because: A. Analysis of fluid from right knee shows: leukocytes 70. His knee is hot.000/mm3 with 75% neutrophils. crystal analysis = negative birefringent. Acid suppression is done by blocking: A. Absence of Myenteric plexus. By aspiration of ova of Entameoba histolitica. 3. 2. At the time of induction of anesthesia. Drug of choice for acute pancreatitis. Clot organization. 24. Which of the following is drug of choice for patient of status asthematicus: I/V aminophylin Oral steroid I/V salbutamol. Achlasia is due to: A. Morphine. Heparin will inhibit: A. He has a low grade fever. 21. Which of the following blocks α and β receptors: A. Pseudo gout. Drug that decreases the tone of lower esophageal sphincter & increases gastric emptying: A. Which one of the following is most likely? A} Horse shoe kidney B} Constricted ectopic kidney C} Pelvis kidney D} Non rotating kidney E} Unilateral renal agenesis 12] Which one of the following is not formed from urogenital sinus? A} Bartholin’s gland B} Urethral glands C} Para urethral glands D} Seminal vesicle E} 13] Virus damage the cell by A} making the pores in cell membrane B} Forming free radicals C} Altering the formation of proteins D} Nuclear damage E} using energy for their own metabolism c 14] The tissue damage by Ionization Radiation is due to A) Damage to Golgi bodies B} Formation of free radicals C} Hydropic degeneration D} Metaphase of cell E} Swelling of cells 15] All are features of IRREVERSIBLE cell injury EXCEPT A} Karyolysis B} Karyorhexis C} Autolysis D} Shrinkage of mitochondria E} Appearance of myelin figure 16] Which one of the following is least likely involved in increasing Osteoporosis A} Increased wt: on bone B} Renal cell carcinoma C} Carcinoma prostate D} . Instead there is a small shadow just above the bladder on the left side.C} 47 XXX D} 47 XXy (correct) E} 47 XX 10] Somites are derived from A} Intermediate mesoderm B} Paraxial mesoderm C} Lateral plate mesoderm (correct) D} Ectoderm E} Endoderm 11] An I/V Urograph of 8 year old boy shows Excretion of dye normal I kidney but absence of shadow on left side. C5 complex E} 23] Which one of the following is potent antioxidant? A} Vitamin A B} Vitamin C C} Vitamin E D} Transferrin (correct) . Granuloma but no necrosis. What is the diagnosis? A} Sarcoidosis B} Silicosis C} Tuberculosis D} E} (correct) 18] In women the most common form of necrosis after trauma is due to A} Trauma to fatty tissue B} C} D} E} 19] Dystrophic calcification is present in which of the following A} Acute pancreatitis B} Malaria C} Tuberculosis (correct) D} E} 20] Which one of the following is most likely feature of reversible injury? A} Cell swelling B} Karyorhexis C} Decreased glycogen D} Myelin figures E} (correct) 21] A person fell on road and got an abrasion on elbow the first event likely occur would be A} Vasoconstriction (correct) B} Platelet adhesion C} Platelet aggregation D} E} 22] Which one of the following facilitates phagocytosis in neutrophils? A} C5a B} C3a C} C3b (correct) D} C3 .E} 17] Patient X-Ray shows hilar lymph nodes. Which one of the following is correct? A} Autosomal Dominant disorder B} Autosomal recessive disorder C} X.E} 24] Which one of the following among Arachidonic acid metabolite is most likely function mediated in phagocytosis? A} Chemo taxis B} Diapedisis C} Transmigration D} E} (correct) 25] Which one of the following is potent COX-II INHIBITOR? A} Aspirin B} Celocoxib (correct) C} Indomethacin D} Meloxicam E} Piroxicam 26] In case of pulmonary embolism extensive infarction causes sudden death due to A} Air embolism B} Fat embolism C} Thromboembolism D} E} (correct) 27] A patient was operated for abdominal surgery.linked disorder D} E} (correct) 30] In a renal transplantation. Which one of the following should be given immediately? Fresh frozen plasma B} 20 days old blood C} 3 days old blood D} E} (correct) 29] A boy was born with the condition of polycystic kidney disease. which one of the following is single most significant test to be done? A} ABO blood grouping of donor and recipient . Which one of the following is most likely? A} endothelial injury B} hypercoagubility C} stasis and endothelial injury D} E} (correct) 28] A soldier has profuse bleeding. He was admitted in the hospital. A few days after he developed pain and swelling in his right leg. B} HLA typing and matching (correct) C} Donor and recipient should be close relatives D} E} 31] A organ was transplanted to the patient. Which one of the following is likely etiology? A} Antibody mediated B} T-Cell mediated C} Preformed antibodies D} E} (correct) 32] AIDS occur in the presence of A} Kaposi’s sarcoma B} C} D} E} 33] A 2 year old boy is suspected as HIV positive. which one of the following is seen? A} Anti thyroglobulin B} C} D} E} . which one of the following type of anemia is developed postoperatively A} Megaloblastic anemia (correct) B} Iron deficiency anemia C} Normocytic norm chromic anemia D} E} 35] Autoimmunity is not involved in which one of the following A} Pernicious anemia B} Grave’s disease C} Diabetes mellitus D} Hashimoto’s thyroiditis E} Thymoma (correct) 36] In case of hashimoto’s thyroiditis. but within 10 mins the organ failed. which one of the following is the confirmatory test in the boy A} ELISA test B} Western Blot test (correct) C} PCR D} Increased levels of HIV in T lymphocytes E} 34] A 10 year old boy was operated for gastrectomy. 37] To differentiate Chron’s disease from Ulcerative colitis. following is finding A} Mallory bodies B} C} D} E} 39] A 20 year old girl has developed Galactorrhea due to prolactinoma.R most likely A} Synthesizes protein B} Synthesizes lipids C} (correct) . Which of the following drug has been taken by this patient? A} Morphine B} Heroine (correct) C} Pethidine D} Phenobarbital E} 41] Strongest layer of small intestine is A} Mucosa B} Sub mucosa C} Circular layer D} Serosa E} Longitudinal layer 42] Regarding adrenal medulla which one of the following is true? A} Secretes epinephrine and Norepinepinephrine in 80/20 ratio B} C} D} E} 43] R.E. Bromocriptine is given to treat it the drug causes A} Inhibition of prolaction from pituitary gland (correct) B} Decreased prolaction from hypothalamus C} Inhibition of prolaction from breast D} Stimulation of prolaction of anterior pituitary E} 40] An unconscious patient presented in ED does not respond to Naloxone. which one of the following is likely significant regarding Chron’s disease A} has increased frequency to cancer B} Involves both small and large intestine C} D} E} (correct) 38] Regarding alcoholic hepatitis. D} E} 44] Half life of insulin is A} 5 mins B} 90 mins C} 120 mins D} E} 45] Insulin A} Converts glycogen to glucose B} Stimulates protein synthesis C} Stimulates lipolysis D} E} (correct) 46] Which one of the following most likely contains abundant cholesterol? A} VLDL B} LDL (correct) C} Chylmicrons D} HDL E} 47] Which one of the following most likely causes Increase in GFR? A} Constriction of afferent arteriole B} Constriction Of efferent arteriole C} D} E} (correct) 48] Gastric emptying is delayed by A} Gastrin B} Secretin C} CCK (correct) D} E} 49] Which one of the following pancreatic enzyme is most likely involved in fat absorption A} Amylase B} Lipase C} D} E} 50] Smooth muscle A} Contain sarcomere B} Ttubules are present . Mediators of inflammation a. Inc ICP. hypoTN. 90 yrs old lady with purplish large patch on Rt hand and arm. Free T3 c. Mast cell stabilizer. veins e 162. venules a b. Bradycardia b. dec ACh release in bronchi a b. pH Inc. artery b. Reason? a. dec ICP. IL1 d. Bile from liver is different from that from GB in: 160. Blood flow regulation is mediated by: a.a. tachycardia d. HTN. Inferior wall MI d.most effect. arterioles c. Warfarin. precepillary sphincter c. PCT a b. Sec released from thyroid gland entering circulation is: a. B6 d. Capillary abn b. venules e. hypoTN. HCO3. Prothrombin c. vasodil c. tachycardia 1 2 2 . ARF. No comorbids. Largest total cross-sectional and surface area is of: a. Thiamine a b. Lateral wall MI e. Inferior wall MI 150. DCT c. Thyroxine b. CD 4 1 . HTN. Vit K dependent clotting factors 4 7 . factor III c. capillaries d. Ibpratopium bromide given by nebulization: a. renal failure 4 0 . C3a b. Ca++ dec. Pt complains of chest pain for more than 30 m i n . PTH 43. TSH e. Uncontrolled D M 152. True b. ARF b. tachycardia e. Hb 8. Inc ICP. arterioles capillaries d. false 1 1 5 1 . C5a b c. Anion gap increases i n : a . TNF alpha 161. bradycardia c. Folic acid 44. LOH d. H T N . Prothrombin c 48. folic acid deficiency d. Na chromoglycate 52.V 4 . effects o n : a.0. TRH 53.C} Gap junctions are present D} Forms motor end plate E} (correct) 51] Steady pressure on skin is perceived by A} Pacinian’s corpuscle (correct) B} Ruffini’s corpuscle C} Meissener’s Corpuscle D} Golgi tendon organs E} 39. It denotes: a. Cushing's triad: a. Vit D def c.inc (35). ECG shows changes in V 1 . anterolateral MI c. High output cardiac failure a. Metabolic alkalosis a. . Otherwise Normal. inc secretion 49. B2 c. anterior wall MI a b. free T4 d. No itching or pain. inc ICP. inc ICP. a. Cause a. Factor VIII b. Medullary respiratory group of neurons t of ascending reticular system a. C is stimulated by: A} Factor X B} Factor XII C} Thromboplastin D} Factor IX E} Fibrinogen d 13] In enchymal cells Iron is deposited in the form of A} Tranferrin B} Ferritin bC} Heamatin D} Heamoglobin E} Lipifuscin 19] Regarding Von Willbrand disease A} Autosomal Dominant B} Autosomal Recessive C} Some family members have low level of Factor IX D} Some family members have abnormal level of Factor IX E} has positive family history 20] Which of the following condition is related to lymphatics? A} Angiosarcoma a B} Rhabdomyosarcoma C} Cystic hygroma 21] The condition in which pulmonary trunk fails to divide leading A} PDA arteriosus C} TOF aB} Truncus 22] Basal Cell Carcinoma is located at A} Lower lip B} Upper lip C} Tongue D} Hard palate E} Soft palate 25] Which of the following is not Carcinogen? A} Alcohol B} Benzathracine C} Cyclophosphamide D} Benzidine E} Dimethylsulfate 26] The Initial step in APOPTOSIS is A} Inhibition of P53 genes B} Activation of BCL-2 C} Activation of Caspases c D} pores formed by lymphocytes 28] Which t of kidney produces ERYTROPOITEN A} Macula Densa B} Peritubular Capillary b C} PCT D} DCT E} LOH 29] A patient with signs and symptoms of megaloblasstic anemia. likely abnormality is in A} Glucose}-6-Phosphate deficiencyB} Immune hemolysis C} oxysmal Nocturnal Haemoglobinuria D} Malaria d E} Sickle cell disease 5] Thiamine deficiency causes A} Peripheral Neuropathy a B.4] A 16 year old boy was given CO-TRIMAXAZOLE for fever.I. after few hours he developed red color urine. PellagraC} Chelosis D} Dermatitis 6] Milk is notoriously deficient in A} Vitamin C B} Iron C Riboflavin D} Pantothenic acid E} Vitamin A 7] Dry heat kills the bacteria by A} Coagulation of proteins B} Cell lysis C} Free radical formation D} Direct killing d E} Oxidation 8] Virus damage the cell by A} Making the pores in cell membrane B} Forming free radicals C} Altering the formation of proteins c D} Nuclear damage E} using energy for their own metabolism 9] Which one of the following lipoprotein has highest protein content? A} HDL} C} VL} D} IDL E} CHYLMICRONS a} B} LDL 10] All are related to PLATELET FUNCTION EXCEPT A} Capillary resilience test a B} BT C} Clot retraction 11] ALLELLE is: A} Fragmented genes B} Non-identical genes C} Structural genes D} Regulatory gene E} Autosomal Dominant gene 12] D. The likely cause is . Stool DR: Ova of Intestinal asite. The most likely causative asite is A} Ankylostomata Duodenale B} Ascris Lumbricoidis C} Diphylobothrium Latum c D} Strangyloids Stercoralis E} Taenia Saginata 34] A 40 year old male after road traffic accident had received massive blood transfusion.5%. His LFT shows Direct Billirubin 11 mg/100 m l . Platelet count: 110 x 10 9/L.5 gm/dl. One large cell with multiple nuclei arranged close to membrane is also seen. The most likely change in central area would b e: A} Caseous necrosis B} Coagulation necrosis C} Fatty changesD} Hydropic changes E} Liquefective necrosis 33] A 28 year fisherman presented with slowly developing lethargy. Healing of this burn will take place from which of the following layers of the skin A} Basal layer of epidermis} Keratinized layer C} Layer of upper dermis c D} Lower dermis E} Subcutaneous layer 38] A 40 year old male presented with jaundice and generalized itching. He is expected to have all EXCEPT A} Hypocalcaemia B} Hypokalemia b C} Hypothermia D} Left shift of Oxyheamoglobin dissociation curve E} Metabolic acidosis 35] The endothelial molecules ICAM-1 & VCAM-1 are responsible for A} Direct endothelial injury B} Endothelial cell contraction C} Junctional re Traction D} Leukocyte adhesion d E} Leukocyte margination 36] Eating raw or under cooked meat can cause infestation by A} Dracunculous Medinensis B} Echinecoccus Granulosis C Liver Fluke D Schistosomiasis E} Tape Worm e 37] A pt: has sustained burn that is very painful with blisters. easy fatigue and palpitations. C} Failure to thrive. Alka Phosphate 2300 IU/L 500IU/L. Physical examination: Pallor. SGPT 75U/L. D} Skin pigmentation E} Thrombocytopenia 32] Liver biopsy from a pati ent shows a lesion which comprises of central collection of structure less material surrounded by epitheliod cells and lymphocytes. His diet comprises of fish an d rice only. Loss of touch sense in both feet and legs Hb: 7. a B} Loss of subcutaneous fat.A Decreased GIT motility B} Deficiency of intrinsic factor b C} Malabsorption 30] Stimulation to increase Serum Calcium is A} Hypo-phosphatemia B} atharmone Decreased ECF b C} 31] Malnutrition causes all of the following EXCEPT A} Pitting Edema. The most likely cause is A} Drug induced jaundice B} Extra hepatic Cholestasis b C} Autoimmune Hepatitis D} Alcoholic Hepatitis E} Intrahepatic Cholestasis 39] The tissue damage by Ionization Radiation is due to A} Damage to Golgi bodies B} Formation of free radicals b C} Hydropic degeneration D} Metaplasia of cell E} Swelling of cells 42] Increase in Light Chain Immunoglobulins Is the characteristic feature of A} Endocrine type Amyloidosis B} Hereditary Amyloidosis C} Localized type Amyloidosis D} Reactive Amyloidosis E} Systemic type Amyloidosis 43] Chemotaxis is caused by A} C5a a B} C5b C} Lymphokines D} Leucotriene B3 E} Opsonins . Reticulocyte count: 0. Which variant rises with rise in BP TPR* Pulse Venous tone Arterial tone 9.In a thyroid surgery mass was excisied which microscopically showed amyloids post surgically which marker would be taken in account PTH Calcitonin* Ca Po4 8.When r the ventricles most filled with blood ' Diastasis Rapid inflow Atrial systole Ventricular systole .44] Following are the actions of Archidonic acid metabolites in inflammation EXCEPT A} Chemotaxis B} Increased vascular permeability 1.In liver of a patient under the microscope a lesion was seen with central cheesy appearance surrounded by large cells with lymphocytes and fibroblast what type of necrosis is present Coagulative* Liquefative Fatty tuberculous 5.Which harmone of the following stimulates thirst Adh* Aldosterone Renin oxytocin 6.Pre cancerous in aids: Cmv Ebv* Hsv Hpv 4.Which has no antigens AB* O-O+ A 7.What is most lateral in cerebral hemisphere : Insula Pars triangularis * Pars interemedia Amygdala 2.Below the umblicus anterior abdomen is formed by: External intercostal and part of Internal oblique* Internal intercostals and part of transverse abdominis uscle External intercostals and transverse abdominis muscle Internal intercostals and transverse abdominis muscle 3. Which of the following tissue is most radiosensitive Skeletal muscle Cartilage* Bone 17.In dorsal column lesion which of the following will be lost Pain Temp Crude touch Itch Proprioception 18.Example of wet gangrene Thromboembolism Bacterial endocarditis Pancreatitis 11.Common feature in nitrates.10.Asthma has low Fev1* Fvc Vc Tlc 19. isoprenaline .feature of Downs Syndrome Cardiac anomolies Fallots Hypospadiasis Early death* 14Turner syndrome has Chromosomal defect Xlinked Short stature* Tall stature 15.Which of the following harmone stops ovulation during lactation Estrogen Progestrone Prolactin* Oxytocin 12Which of the follwing delays wound healing Vit c* vita Immobilization Vit E 13. Throphylline is Postural hypotension Rapid pulse Vasodlation .Pre carcinogenic is Berryliosis Asbestosis* Benzethe Pentamidine 20.X linked dominant Pku Alkaptonuria Familial polyposis* Glycogen storage 16. Water reabsorption through ADH is done in Proximal Distal Collecting Loop of henle 26.The organ placed retroperitoneally is Spleen Transverse Bladder Ascending* Liver 26.Long term granulomatous disease is Crohn Celiac Sarcoidosis Ulcerative colitis* 25.Midarm circumference is the measure of Proteins* Fats Carbohydrates Minerals 22.Venodilation 21.Chemotactic factors functions mostly in Diapedisis .In food poisoning death occur by Exotxin of samonella Vibro cholera * Shigella Endotoxin of Shigella 24.Temporal Arteritis is diagnosis by Esr* Biopsy B/c WBC count 23.Urinary bladder in male is Completely covered by peritoneum Superiorly covers From rectum separated by fascia of Denon Villiers 27.Hydrocephalus occurs because of obstruction in I/v foramen* Sylvius Magendie Lushka 28.What organ lesion causes hypothermia Preoptic Lateral hypothalamus Post hypothalamus * Pitutary 28.Hipocampal injury causes Sex rage Hunger Loss of Memory* Temperature disturbances 27. Musle traversing the shoulder joint Crochobrachilis Sspinatus Ispinatus 31.TGlycerides are absorbed in Chylomicrons Vldl Hdl Ldl 38. medially rotate 36.laterally rotate Adducts .Margination ' Migration Acute inflammation 29.Hb is enclosed in RBC at stage of Normoblast .Juvenile polyps are also called Hamartomas Rectal polyps Internal haemorrhoids 37.Example of synchondrosis is Symphisis Involves hyaline crtlg b/w 2 bones Costal joint 34. minimus Adducts.Blood spread of CA occurs 1st in Metaphysis Diaphysis Epiphyses Epiphyseal plate Cortex 32.medially rotate Abducts. Gluteus medius . laterally rotate Flexes .A boy presented in the ER with fracture with less seum calcium.He is suffering from Primary hypoparathyroidism Vit D deficiency 2ndry hypoparathyroidism 33.In gastrectomy loss of following occurs Hcl Intrinsic facor Fats Vit D 39. Paramesonephric duct remnant in male is Appendice testis Appendix Ovary 30. medially rotate Abducts.Erythropoietin is secreted from Yellow marrow Jg complex Macula densa Preitubualr capillaries Loop of henle 35. Lateral part of breast supplied by Ant group Post group Lateral group Central group 46.About 1st lumbrical Arises from 2nd meta carpal Is bipennate Supplied by median nerve 44.Posterior triangle in the neck has follwing boundries Ant ant border of SCM post omohyoid Post ant border of SCM and ant diagastric Ant post border of SCM and post ant border of trapezius 50.The muscle of passive inspiration is Diaphragm External intercostal Internal intercostal Scalenes muscles 47.Adrenal gland is separated from kidney by Peritoneal fascia Renal fascia Perinephric fat 41.Late normoblast Erythroblast 40.Lymph drainage of lateral 2/3 of tongue occurs through Sup deep cervical nodes Inferior deep cervical nodes Submandibular nodes Submental nodes 48.Lumbricals are supplied by Median neve deep br of median n Deep br of ulnar n 45.Left recurrent laryngeal nerve has Shorter route than right recurrent laryngeal nerve Branch of vagus nerve Supplies all the muscles of soft palate Supplies all the muscles of larynx 51.Boil on the tip of the tongue will drain into following lymph nodes Submental submandibular Submental nd submandibular 49.Right hepatic artery is branch of Hepatic artery gastroduodenal artery Ceoliac artery .Median nerve supplies Palmar medial 1/3 Palmar and dorsal medialm1/3 Dorsal medial 1/3 Dorsal 2/3 lateral 43.About antibodies Have varible2 heavy chains IgM is abundant 42. Inferior rectal artery is a branch of Ext illiac Int illiac Sup epigastric Int pudendal artery Femoral artery 58. he has Metabolic acidosis Respiratory alkalosis wirh ion gap Metabolic alkalosis 56.After illeal resection absorption of which of the following components does not take place Chylomicrons Bile Tglyceric acid Vit d Vit c 54.One of the following is not a feature of anaerobe Cough Tetanus Pus Granuloma formation Gangrene 61.Granuloma formation does not occur in TB Cat scratch fever Toxoplasmosis Pneumonia 60.Patientt has Babinski + lesion is in UMN LMN Dorsal column Ant column 57.Patient after radiation vomited for 4hrs Cl is raised.In C R F all of following occurs except Anemia Hyperlaemia Hypocalcemia Hypoposphatemia Hyperuricemia 55.Peristalsis in the intestine inhibited by CCK Gastrin Secretin GIP Histamine 59.Splenic artery 52.Subcapsularis sinus is present in Spleen Lymph nodes Payers patches Palatine tonsils 53.Melanocytes originate from Neural crest Mesoderm . Structures which passes through the aortic opening is vagus and azygous vn Azygous vn Esophagus Phrenic nerve 63.Liver helps in immunity by Detoxfying noxious agents Kupffer cells Hepatocytes 65.Sour and hot is perceived by tongue through Von ebner cells Folate papillae Pain fibers Nociceptors 66.Cholestatic cholangitis is caused by Colonercis selecans diphyllobothrum latum Schistosoma Entameoba 71.The mc burney point indicates the presence of tip of appendix.IJV drain in Petrosal sinus Sagittal sinus Straight sinus Sigmoid sinus 64.Following is true about appendix Appendix ha incomplete muscular coat .Ectoderm 62.A boy is malnutrioned and has edema due to protien loss his Plasma colloid pressure is decreased Increase hydrostatic pressure Blocked lymphatics 70.After appendicectomy what may be damaged Inguinal ligament Superior epigastric artery Illhypogastric nerve Spermatic cord 72.ADH is inhibited by Alcohol Angiotensin Insulin Renin 67.Constricting afferent ducts in kidney reduces Gfr Rpf Creatinine clearance Glucose claerance 69.In testis seminferous tubules show following under M/S Sertoli cells Spermatogonia Mixed cells Spermatocytes Sperms 68. Injury to the neck and head of fibula causes damage to Sural nerve Sciatic nerve Tibial nerve Common peroneal nerve 80.Medial side of hand is supplied by Median nerve Radial nerve Axillary nerve 77.Femoral artery can be palpated in Mid inguinal lateral to pubic tubercle Mid point of inguinal ligament Adductor canal 75.Vomiting center is persent in Hypothalamus Pitutary Pons Medulla Midbrain 78.Appendix not attached to mesentry Supplied by ceoliac artery 73.One of the following is not a branch of External carotid artery Lingual A Ophtalmic A Occipital A Maxillary A Suprficial temporal A 74.Patient wirh Gravida 4 Par4 Abortio 0 with anemia and occult blood in stool Fe deficiency anemia Pernicious anemia Anemia of chronic disease Sideroblastic anemia 79.Nerve to ligamentum teres capitis is Femoral n Obturator n Medial femoral n Lat circumflex femoral n 81.Artery to head of femur is Medial femoral circumflex a Lateral femoral circumflex a Obturator a Femoral a 82.Apex of the femoral triangle Lies under inguinal ligament Contains small saphenous vein Contains nodes Is pointed downwards 76.Muscranic receptors are present in Postgaglionic parasympathetic Postgaglionic sympathetic Preganglionic parasympathetic Preganglionic sympathetic . On bronchoscopy bronchoscope will 1st enter Sup bronchus Apical bronchus Inf bronchus Middle apical bronchus 84.1 gram of protein contains calories? a) 4 calories .Essential fatty acid is Linoleic Palmitic Citric Oxaloacetic 85.Following is not a feature of DIC Dec FDF Dec BT Dec PT Dec thrombin time Dec platelets 87.In tissue bleeding occurs because of Loss of coagulation factors Congenital disorders Endothelial damage dec FDF 86.Direct inguinal hernia lies Lateral to inf epigastric a Medial ti sup epigastric a medial to inf epigastric a Medial to pubic tubercle 89.Occulocardiac reflex mediated by a)CN5 2.83.Most drug metabolism occur in Kidney Liver Spleen Intestine 1.Role of middle ear bones is Transmit sound Amplify sound Does not respond to low noise Are sesamoid bones 88.Thrombosis is initiated by Leukotrienes Prostacyclin Arachidonic metabolites Thromboxane A2 90. a) loss of taste to anterior 2/3 of tongue b)Myelohoid muscle is unable to stabalizse mandible? c) Loss of sensation to lower teets? 6-Head of femer a) has anger of 125 with shaft b)posteriorly completely covered by capsule? c)epicondyls are in parallel line with shaft 7-Lymph nodules are present in a)Spleen b)Thymus c)Lymph nobes 8-In DIC .3-7th lobe of lung is called a) medial basal 4-A marathion runner wins a race.there is a)decreased palatlets 9-what is inferior in perenium? a) pereniul membrane 10)spleen is a) mesodermal b)endodermal 11)according to recent recommendations.followin if true about it onset of action a) 5 min b)5 sec .What will be with blood insulin and Glucogan levels at the time when he crosses the win line? 5.2 15)Head of Humerous is supplied by a) Anterior circumflex artery b)Posterior circumflex artery c)Suprescapular artery d)Subscapular artery 16) Gene are composed of a) RNA b)DNA c)Extrons d)Introns 17) I/V dopamine is given to a patiend. recurrent MI should be treated with? a) Anticoagulants b) Lipid lowering drugs c) Beta blockers 12)Stratified squamous epithelium is present in a )palatine tonsils b)pharynx c)Ovary 13) Low voltage QRS comples ios seen in a) MI b) Bundel branch block c)Hypertension d)IHD e)Pleuritis 14)Isthmus of thyroid is present at a)C1.Mandibular fracture occurs just brfore the mandibular foramen. c) 60 sec 18)I/V sedation is given to a patient. the contents will go to a) Right iliac fossa b)Left Iliac fossa c)Anterior subhepatic space d)posterior subhepatic space e) small bursae 25) Metabolic alkalosis results from a)Ingestion of ammonium chloride b)ingestion of carbonic anhydrase inhibitors 26)A Diabetic patient fas urine ketones and urine glucose ++ the cause is ? a)Hyperglycemia b)Insulin difficiency 27 Response to chemotherapy is because it causes a)Atrophy b)Apaptosis 28)Dorsal column damage results in Ataxia because a)loss of pathway from dorsal column to thalamus b)Loss of proprioception input to cerebellum c)Loss of proprioception to Thalamus d) Loss of proprioception to Hypothalamus . Following is true about its arm-brain circulation a)60 sec b)5min c)30min d)5 sec e)? 19) DVT is most common in a)Femoral vein b)popleteal vein c)Superficial vericosed veins of leg 20) Which of the following is not a tumor marker a)Acid phosphatase b)PAP c)Hcg d)AFP e)CEA 21) Pus contains a)Dead bacteria b)Dead neutrophils 22) The most common cause of fatty liver in our society is a) Alcohol b)Hepatitis B and C c Drugs 23) Carotid bodies respond to a) increase blood H+ b)Increase blood co2 24) In anterior Duodenal perforation . increases frequency and chills and riggors.What structure prevents the the joint to go back in cavity? a) Tubercles on articular surface of mandible b)Tempomandibular ligament c)Lateral Ptyregoid muscle d)Medial Ptyregoid muscle 36)Where the gangliom joining the superior petrosal nerve and inferior petrosal nerve is located? 37) A patient has hypercarboxyhemoglobin.29)Which of following nerve passes dorsal to brain stem? a) 3. with Hypercellular bone marrow/\ a)Due to drug he is using for anaemia b) Leukemia c) Iron deficiency anaemia d)G6PD deficiency 31) The sign of cerebellar disease is a)Adiadohakinesia b)Static tremor c)Hypertonia 31)Neurohypophysis drain secretions a) Adrenergic neurons b)cholinergic neurons c)free nerve endings 32)CSF a)has osmolarity <1040 b) pressure <10 mm of water c) protein >200 33) Stimulation of Alpha adrenergic neurons 34)End arteries are present at a)Brain b)Heart c)Bone 35) In Tempomandibular Joint Dislocation. c)5. After how much time .his blood will have 100% oxygen? a) 1 min b)2 min c)5 sec d) 5 min 38) FRC is? a) TV + ERV b)IRV + ERV 39)Aldosterone stimulate a) Melatonin? b)? 40)A patient has fever 40c with flank pain . b) 4. d) 12.\ a) Blood culture b)urine culture . He is breathing 100% oxygen.000. palatelet 450. e) 9 30-A patient has hb 6. What is it? a) Prostate 50)25 cholecalciferol is not converted to 1.peritoneum b) skin.superficial fascia.peritinium 45) Carpus callosum a) connects the two hemispheres 46)Most constricted part of male urethra is a) External meatus 47)Superior Thyroid glands a) Are located behind the Thyroid gland outside the facia b)Have blood supply from the superior thyroid arteries 48) Which of the following is true a) Piriformis attach to inferion trochanter b) Schiatic nerve passes through superior sciatic foramen 49) A structure lined by cuboidal epithelium. and its center contains two calcified bodien of pink colour.extaperitoneal fat.Lactation does not occur because a) Increased Level of progesterone and Estrogen 54) Which of the following have Pulmonary cycle? .c) Blood and urine culture 40)Following is not true? a) HCV positive carriers have not been reported 41) Pulse pressure is increased in a)Arteriols b)capalleries C)Aorta 42) A grand father was playing with his grandson with holding his wrist and rotation him in a circule. What happent ? a) Proximal Radio ulnar joint dislocation b)Distal Radio ulnar joint dislocation c)Fracture of acromion 43) Regarding typhoid fever a) Asypmtomatic b)Carriers do not excreate causative agent in fees c) Asypmtomtic carriers should be isolated and treated 44) Incision is given on Rt Lumbar area.Suddenly the child starded crying and sit with holding his forarm in porn position. The layers incised will be a) Skin-superficial facia-deep facia.external oblique-internal oblique-transverse –Transverse fascia-exta peritoneal fat.25 cholecalciferol in a)Hepatic failure b)Renal Failure 51) Which of the following is not present in posterior relations of kidney? a) colic flexure 52) Rt border of heart is formed by a)Rt atrium 53) In pregnancy . Na is absorbed totally neutrally active transport? a) PCT b)DCT c)ATT 76) Cell mediated immunity is mediated by a) T cells b)B cells 77) Virus does not cause a) Lipoma b) Kaposi sarcoma c)cervical cancer 78) Iodine is stored in thyroid follicles as a) Thyroglobulin b)DIT and MIT 79) A patient died due to colonic cancer. The cause could be a)Adenomatous polyps? . cell swelling occurs due to a) Entry of water in cell fromj surrounding b) Entry of lipids 64)What is potent Antoxident ( Anti Aging) a)VIT E 65) A boy is excreting meconium from rectum and has severe constipation. It could be a) Damage to inferior epigestric artery? 63) In Hypoxia.joint pain. The condition is called A) Intussusceptions B) Hirschsprung’s disease 66) In what portion of nephrone.On autopsy found thousands of polyps. The rectal examination showed no ganglion cells. Flank pain could have a) SLE 60) A young patient has difficulty in swallowing a) Sclerodermia 61) A patient with SLE have a) Possitive ANA b) Possitive anti SS 62) Incision on Supra pubic area caused heavy bleed. Patient's father also had colonic cancer.a) Ascariasis 55)Causative agent of meningitis in 60 years old man is a) St pneumonia 56) Cause of death in pneumonia bue to shock is a)Gram –ve bacteria b)Pseudomonas c)st pneumonia 57) Diphtheria exotoxin hane powerful effect on a) Larynx b)Heart c)Nerve endings d)Kidneys 58) Aspiration of peanut will go in a) Right lower bronchus 59) A female with butterfly ace. What could be happen? a) Increased body volume? b) decreased intracellular osmolarity c)Increased extracellular osmolrity 94) Inhibition of heart Rate will be in a) Stimulation of parasympathetic nervous system 95) Aldosterone is stimulated by a) Increase of serum K 96) Follic Acid difficiency causes a)Megaloblastic erythropoises b) Foliate deficiency 97) Difference B/w Plasma and interstitial fluid is a) Protien 98) C02 is mainly carried by a) Hco3 99) In Inguinal hernia. there is predominate a) Fibroblasts b)Macrophages c)neutrophils 89)Characterics of malignant cancer is a)Metastasis 90 )Characteristic of malignant cancer is a) pleomorphism 91) Pretraceal fascia a) completely encircle the Thyroid gland b)Completely encircles trachea 92) 6 months after the resection of terminal ilium. posterior wall is formed by a) Conjoint tendon b)Internal oblique muscles 100) What is part basal ganglia are a) Putamen ( 32 mistakes ) Paper 2 101) Blood supply of vertebral column is a)Vertibral artery 102)Suprascapular nerve arises from . It could be a) anomaly is due to premature separation of ureteric bud 81) Chylomicrons contains a) Fatty acids with vitamins 82) Transudate has a) Low Albumin 83) common mediator of Basophils and mast cells have a)Histamine 84)In Skeletal muscle . contraction occurs due to a) Sacromere b) Actin c)Myosin 88) In chronic inflammation.80) A patient has double ureters and double pelvis. the patient a) iron deficiency anemia 93) A patient has sweating 2L . and he took 2 L plan water. there might damage a) spinal part of accessory nerve b) CNS part of accessory nerve c) Vagus nerve 117)A patient in RTA came in EAC with hypovomumic shock. Lost adduction of thumb also. lined by stratified squamous epithelium. First what to treat? a) I/V Fluids 118) Clostridia cause a) Gas Gangren 119)Angiotensin 1 is converted to Angiotensin 2 in a) Lung capillary endothelium 120) First week diagnosis of Typhoid is a) Blood culture 121)A soldier posted at high altitude for many years came witg PVC > 40 a) Secondary polycythemis 122) Buerger's disease is strongly associated with a)Smoking 123)Acute appendicitis is associated with a) Neutrophelic leukocytosis 124) Medial arcuate Ligament encloses at its upper end a)Psoas muscle .? Palatine tonsils 114)Regarding the Iron requirement of a pregnant woman. a) Deep branch of ulnar nerve 112)Femoral canal is formed by what fascias? a) Fascia Lata + memberenous Layer b) c) 113) Organ .a) Thyrocerv ical trunk 103) Ist pharyngeal pouch forms a) Auditory tubes 104)Diapharm is supplied by a) c 345 105)Urachus is remnant of a) Allantois 106)A patient had surgery of breast and he is now unable to lift his arm. Gastroduodenal ligament 111) A patient can not abduct and adduct his fingers. a patient might develop a) Megaloblastic anaemia 116) A patient got injury in the posterior triangle of neck. What is lost in feeces? a) HCO3 109)phasic fast adaptive structure is a) Mickle disk b)Carotid baroreceptors c)Rufini's carpuscles 110)The lesser omentum forms which structures? a) Hepatoduodenal ligament. What is damaged? a) serratus anterior muscle 107)Thurst is stimulated by a) ADH 108) A patient has dirrhoea and developed metabolic acidosis. following test is appropriate. a) TIBC b)serum ferritin 115)After gastrectomy. Falciparum Ligament. rounded structure. RR 20. He might developed a)Left homonymus hemianopsia 128)In Burkit Lymphome. pulse 120/min. therer is a) Pulsus deficit 145)A patient lost weight .and drained by single vein 130) Repture of middle meningeal artery cause hematoma in a)Between the two layers of dura mater 131)Middle esophageal constriction is by a)Arch of Aorta b)Left bronchus 132) T. DB 120/80 a) Hyperthyroidism 146) Alpha receptors stimulation may cause a) Lipolysis b)Gluconeogenesis . hot dry skin.cells a) Matured in Thymus after birth b)Both T cells and B cells are derived by same precursors 133)Neurovascular bundle is located at ribs at a)inferior border of ribs 134)Which of the following does not predispose the cancer? a) Radiation b)Bacterial products c)virus d)Chemicals 135)Methicilline resistant staph are a) Usually sensitive to Vancomycin 136) Cardiac muscles can not be tetanized becosed a)They have long refractory period 137) Severe blood transfusion reactoion occurs if transfused all except a) A+ blood to 0+ b)A + blood to AB+ c) B+ to O+ d) B+ tp AB+ e)O +ve blood to OA+ 138)Xenograft is a) Transplant of tissues of different species 139) Lysosomes contains a) Acid Hydrolase 140) Rt Gastro epiploic artery is the branch of a) splenic artery 141) Right gastroduodenal artery is the branch of a) Hepatic artery 142 )An asthematic patient may have FEV1/FCV <75 143)Ligamentum arteriosum connect with a) Aorta with Left pulmonary artery 144)In atrial fibrillation. The causative agent is a) EBV 129)Regarding adrenal glands a) Right is Larger than left b)Supplied by numerous arteries .125) QRS comples is due to a)Ventricular systole 126) The transitional epithelium have a) Couoidial epithelium lined by more larger and rounded cells? 127) A patient have right optic tract damage. B1 . Granuloma but no necrosis.c)glycogenolysis d) Glycogenesis 147)Trypsinogen in the duodenum will be activated by a) PH < 7 148) Pulmonary embolism most commonly involves a) Left pulmonary artery 149 A patient had streptococcal infection. What is the diagnosis? A} Sarcoidosis B} Silicosis C} Tuberculosis 165) Strongest layer of small intestine is a)Circular b)longitudinal c)Mucosa d)Submucosa ok? 166). What blood test you will advise him after one week? a) Anti streptolysin O titer 150)A patient developed hemolytic anaemia after blood transfusion. Hemoglobin released will be attached with a) Heptoglobin b) Albumin 151) A patient with itching. fever with Antimitochondrial antibody +ve a) Primary billiary cirrhosis 152) PDA occurs in a) Prematurity 153) Fertilization occurs in a)Oviduct 154) Liquificative necrosis occurs in a) Brain 155) Temperature regulatory center is in a) Hypothalamus 156) Skin cancer occurs a) In Sun exposed area 157) Giant cells have a) Macrophages 158) Inflamatory mediator is a) C5a 159) Optic tract ends in a) Lateral geniculate body 160) Gastric motility is increased by a) Acetylcholine 161)Wound healing is delayed by a) infection 162)Achalasia is due to a)loss of Aurbach plexus 163)Patient has severe chest pain with ST elevation a) Disectening aorta b)Pleuritis c)Pneumothorax d)costochondral e)Pneumonias 164)Patient X-Ray shows hilar lymph nodes. High output cardiac failure a. I.b. B6 167)Valve less vessel is A) Aorta B) Pulmonary artery C) Coronary Sinus D) Pulmonary Trunk E) SVC e 168) Within 1 hr of the Acute M. B2 c. which of the following enzyme will be raised? A) CK-MB a B) LDH C) Alkaline Phosphatase D) AST E) Troponin T 169) Replacement of cells of other normal site of body is called a) Metaplasia 170)Heart sound produced by rapid ventricular filling is A) 1st heart sound B) 2nd heart sound C) 3rd heart sound c D) 4th heart sound 171)Corticosteroids decrease a) Neutrophils b)Lymphocytes b c)Eiosinophils d)Monocytes e)Basophils 172) DVT occurs in females by use of OCPs 173) Visceral Pericardium is supplied by a) Phrenic Nerve b) Sympthatic Nerves c) Vagus neve d) Cardiac Plexus 174) Basal Cell Carcinoma involves a) Buccal Mucosa b) Hard Palate c) Soft Palate d) Lower Lip e) Oral Cavity 175) Primary spermatocyte divide by a)Mitosis b)Spermiosytosis c)Primary meiotic devision d)Sec Meiotic devision 176) The volume of distribution of drug is not influenced by a)Sex b)Age . ferritin B.c)Heart failure d)Renal Failure 177)In Hypothyroidism a) Increased cholesterol level 178)craniophrangioma in the center of optic chiasmata may cause a) Bitemporal hemianopsia 179)Which one of the following most likely causes Increase in GFR? a)Constriction of afferent arteriole b)Constriction Of efferent arteriole 180) cardic output measured by thermodilution method is called a)fick law? 1 8 1 . arterioles b.tachycardia E. capillaries d. Blood flow regulation i s mediated b y : a.increased urine output D.apoferritin 68) Which drug do not cause gynaecomasia? a) Digoxin a b) Griseofulvin c) Cimetidine d) Androgen e) Estrogen . venules 182)Thiamine deficiency causes A} Peripheral Neuropathy B} Pellagra C} Chelosis D} Dermatitis 183) Which of the following is not a feature of shock A.restlessness 184) Iron is stored in the form of A.decrease cardiac output B.transferrin C.muslce weakness C. precepillary sphincter c. wht is the least likely complication in fetus? a) Bone deformity b) Mental retardation b c) Cleft palate d) Nail Hypoplasia 92) Anticoagulants are indicated in all EXCEPT: a) Thrombocytopenia a b) Cerebral infarction c) Prolong bed rest d) DVT 95) Which investigation is least likely done in a suspected case of DIC? a) Clotting time b) FDP c) D-Dimer d) Platelet count e) PT e 96) Which drug is not used in Dysmenorrhea? a) Ibuprofen b) Mefenamic acid c) Aspirin d) COX inhibitor 13)germinal epithelium of ovary contain? a)cuboidal epithelium a c)columnar etc 14)columnar epithelium line which structure? a)thymus b)choroid plexus c)uterine tubes c 15)which method is most helpful in identifying chromosomal abnormalities? a)karyotyping a b)u\s etc question no .Photosensitivity.69) Pt developed Morning stiffness.rash which test will be positive? a) RA b) ANA b) antimitochondrial 70) Wht is the end product of glucose breakdown ? a) Galactose b) Lactose c) Fructose d) Pyruvate d 71) In a pre eclamptic pt which drug is used to lower BP before surgery? a) Hydralazine a b) Amlodipine 91) A pregnant women is on Phenytoin. side effects mentioned. around 35 brought to emergency. Atropine (correct) (Organo phosphate poisoning. Monotherapy to be prescribed ACE-I a Thiazide Frusimide Ca Channel blockers 10)Pregnancy Induced Hypertension. Heart rate 45. newly diagnosed. whats the treatment. whats the treatment? Alpha blockers a Beta blockers Both Alpha+ Beta 85) Effects of Insulin on body fat deposits/metabolism 37) Which drug binds to COX-2 recepters and not to COX-1 Aspirin Indomethacin Piroxicam Meloxicam Ibuprofen . which drug is responsible.9) Mild CCF symptoms. salivation. Rx? Methyldopa 11) Side Effects of ACE inhibitors Hyperkalemia a HypoKalemia Hypercalemia Hypocalcemia 24) Which drug has a n ATROPINE LIKE ACTION Physostigmine Scopolamine 25)Old obese Diabetic. BP 60/40. Clorpromazine SSRI Etc 105) A man.) Lignocaine Digoxin 58) H-K-ATPase pump is blocked by Omeprazole a Cimetidine Bismuth 82) Pheocromocytoma. choice of Rx? Biguanides a Biguanides plus sulphonylurea Insulin 104) A patient on a psychiatric drug. sweating. adrenals 6. Cimetidine 5. Inc K* b. blood drug conc c. Digoxin toxicity a. How will u assess the drug? a. dec venous return d. Enzyme induction occurs in: a. t1/2 is 3-5 min 3. urine drug conc 2. Bromocriptine is given to treat it the drug causes A} Inhibition of prolaction from pituitary gland B} Decreased prolaction from hypothalamus C} Inhibition of prolaction from breast D} Stimulation of prolaction of anterior pituitary E} 40] An unconscious patient presented in ED does not respond to Naloxone. venodilator c. extensive 1st pass effect b. dec Ca** a . dec HR e. liver a b. Inc Mg* c.39] A 20 year old girl has developed Galactorrhea due to prolactinoma. Warfarin. kidney c. all are true except a. heart d. Regarding Nitroglycerine. Which of the following drug has been taken by this patient? A} Morphine B} Heroine C} Pethidine D} Phenobarbital d E} 45] Insulin A} Converts glycogen to glucose B} Stimulates protein synthesis C} Stimulates lipolysis If a drug is taken orally. plasma drug conc b. effect inc by a. all post-gang parasymp are cholinergic 22. distal convoluted tubule e. K* d. thiazide b. plasma colloid osmotic pressure b c. intersitial hydrostatic press d. Which is true. ACEi c d. Na* b c. ascending loop of Henle c. corticosteroids c. amiodarone 24. collecting duct 23. furosemide c. Cause: a. Treatment started. a. Loop diuretics + thiazides enhance each others effect ( Dec NaCl abs) at: a. aspirin a b. hypertension b. 1 week later pt complains of severe pain at Rt big toe. all sympathetic are adrenergic c. all parasymp are cholinergic b. Drug to be given: a. proximal convoluted tubule d. alkalosis 7. streptomycin b. bleomycin . Secondary active transport occurs thru a. corticosteroids c d. glucose b. altered permeability 10. PG in inflammation dec by: a. Urea 9.d. Pt with mild CCF. dec Na* e. Metastatic pleural effusion. clindamycin c. phosphodiesterase inhibitors 11. descending loop of Henle b. Capillary permeability is inc by a. MOA? a. SA node b. digoxin b. Monotherapy for mild CCF: a. quinidine c. pH e. purkinje e. Terminals shd be in: a. Beta blocker c. Angiotensin b. Class IA antiarrythmic drugs a. Infant with renal abnormality (renal artery stenosis). inc PR interval c. Lidocaine b. inc QRS f. Rt ventricle (correct) e 27. AV node c.Drug that inc extracellular K* (moves K* out of the cell): a. dec AV conduction e. Drug responsible: a. amiodarone 32. captopril b c. dec abnormal tissue conduction and no effect on normal tissue b. amiodarone 34. Lidocaine as antiarrrythmic: a. H2CO3 c. Carbonic anhydrase d. Ca** channel blocker c.Verapamil as antiarrythmic ? 36. Rt atrium d. procainide d. blocks Na* channels b.25. Lidocaine. K* channel blocker 35. dec atrial contraction d.Pacemaker for complete heart block. ACEi b. inc QT 33. nifedipine d. propranolol 26. exercise . Reason a. clindamycin b. post spinal lig c. Sec K* in renal cells a. Cell wall synthesis inhib a. Regarding chemotherapy all are true. Ethambutol d. 1 week after starting of ATT. ant spinal lig b. pt c/o pain in big toe. carbonic anhydrase 51. cephalosporins d. cerebrum > cerebellum > sp. Dark urine after starting ATT. PZA 62. erythromycin 64. req daily dosing of multiple drugs for long time 72. Pseudomembrane colitis linked to: a. Needle inserted at paramedian vertebral canal pierces: a. tetrahydrofolate in bacteria 60. interspinal log e. cord > cerebellum > cerebrum . DOC a. supraspinal lig d. Tamoxifen 58. Sulphonamides inhibit a. Rifampin c. a. penicillin c. Blood flow regulation a. often multi therapy is not curative b. except: a. sp. cord b. Rifampin 61. metabolic alkalosis b. Post menopausal women with brease ca.37. ligamentum flavum 63. INH b. Penicillin 59. most imp. 83.c. has likely taken: A} Phenobarbital a B} Phenothiazine C} Morphine D} Diazepam E} Alcohol 1. cord > cerebellum 77. PG released in inflammation are inhibited by: a. He has history of taking some unknown drug. stress hormone ACTH* Epinephrine Norepinephrine Growth hormone 2. Which drug the pt. tributery of external carotid vein Anterior jugular vein* Facial vein . corticosteroids c. Bupivacaine . aspirin a b. wat is not in relation with pitutary gland Facial nerve* Abducent nerve Sphenoid sinus Trigeminal 4. wat is not in relation with right kidney decending colon* ascending colon duodenum spleen 3. serotonin 54] A semiconscious patient is brought to the emergency room. NaHCO3 reverses the action of the drug. cerebrum > sp. Myocardial toxicity to a L/A? a. Spinal* Internal carotid Vertebral artery and ant cerebellar artery . relation with left kidney Deudenum* Ascending colon Descending colon 9. Nor-epinephrine & serotonin degration by MAO* COMT 7. PICA gives post. side effect of thophyllin nausea and vomiting* hypertension bradycardia seizures 6. chemotaxic factor leukoterine B4* C3b C5 complex histamine 8. wat is not in post. wat cannot pass placenta Ig G Ig A Ig M* Ig D Ig E 12. ant and post spinal arteries arise from vertebral artery gives anterior spinal. what is secreted in breast milk Ig G* Ig A Ig M Ig D Ig E 11. wat is in between celeiac trunk and superior messentry artery Pancreas* Pancreas + jujenum Transverse colon + ilium 10.5. A man with noticed weakness of left side of the body. where is the lesion Cerebral hemisphere Left cerebellum * superior c puduncle Red neucleus 18.. loss of memory but resolve ….. if there is damage to sciatic nerve. supply to the dorsum of foot is by sephenous nerve* Sural nerve Common peroneal Deep peroneal . a new born presented with obstructive jaundice. adrenalcortical insuficancy causes hypokelimia hyponatremis hypoglycemia* alkalosis 19.13. In resting stage actin is attached to tropomyocin troponin titin myosin 20.. gall bladder looks ok in U/s. When r the ventricles filled w i t h blood Diastasis* Atrial systole Rapid in flow 15. cause Atresia of CBD* Atresia of entire extra hepatic apparatus Atresia of hepatic duct Choledochal cyst 17.he is agressive in his mood. hypotonia of the left limb. from which foramen facial nerve enters temporal lobe internal accustic meatus* stylomastoid foramen foramen ovale foramen spinosum 14.. A boy had behavior problem. where is damage Mammillory bodies frontal lobe* temporal lobe parital lobe occipital lobe 16. he swangs to left side.. utrine cancer spread to labia majore via Round ligament* utrosacral ligament 27. true Scaphoid. Basalic vein .. drain in axillary ven* runs on radial side starts from palmar surface 28. median nerve supply lateral 2 lumbricals* causes wrist drop 29. injury of neck of the humerus.2 1 .. wat vein run with Post. Gluteus Maximum* Gluteus medius Tensor fascia lata Quadricep femoris 2 2 .. stucture compressing Esophagus Left bronchus (correct) Left Venticle Right Atrium Right ventricle Thymus 24. interventricle artery great cardiac vein middle cardiac * small cardiac 26. Esophagus constriction at Aortic arch* Left ventricle Rt atrium 25. wat is not in relation with left recurrent nerve SVC* Arch of aorta Lig arteriousum Trachea Esophagus 23. wat is in proximal layer of carpal bone. patient with difficulty in standing from sitting postion. nerve damage Axillary nerve* Musculocutaneous Radial Ulnar 3 0 . lunate triqutral & pisiform* . fuctional n unit of the lung* contains lobar bronchus pulmonary veins run in intersegmental tissue 32. diagnostic test for strptococuss is ASO titer Blood culture* Urine Dr 37. wat is true supplied by anterior division of internal iliac* separated from uterus by pouch of Douglas in males has seminal vesical above and vas deferens below 33.5cm inferior n lateral to pubic tubercle* has only 1 tributary connected to short sphenous by perforators 34. 6 5 % of Na and water reabsorbed from PCT* DCT Loop of Henle Collecting duct 38. urinary bladder. sephenous nerve wat is true drain into femoral 3. broncopulmonary segment wat is true anatomical.Which of the following tissue is most radiosensitive Skeletal muscle Cartilage* Bone 36.3 1 . down syndrome trisomy 2 1 * 18 15 13 35. injury to hippocampus Memory loss* 40.Erythropoietin is secreted from Yellow marrow Macula densa Preitubualr capillaries* . . .. diapharam contraction cause increase in thoracic pressure increase in thoracic diameter 43. medial part of breast supply which node internal thoracic* lateral grp ant grp medial grp 42. sypathetic supply by alpha receptors dilator pupilary musscle of eye* brochoconstriction cardioacceleration 45. wat is not supplied by internal thoracic artery Inferior portion of rectus muscle* Ant abd wall upto umbilicus Pericardium Thymus 49. thoracic duct passes with aorta * 44. parasympathetic supply inc. gastric emptying delayed by CCK* Secretin Gastrin Vagus 47. wat is the cause of edema Plasma colloid pressure is decreased* Increase hydrostatic pressure Increased capillary permeability 48..PCT 4 1 . A boy has edema due to protien loss of 3. Patient Gravida 4 with anemia and occult blood in stool Iron deficiency anemia * Sideroblastic anemia Thalassemia Megaloblastic anemia . secretioin of slivary gland* 46.5g in urine. Of left coronary artery Posterior interventricular artery Main Left coronary artery Main right coronary artery 55. Cerebral Anterior and pst. wat is true abt CSF secreted at rate of 500 ml/day* choroid villi are seen by naked eye th aqueduct transmits CSF from 4 ventricle to spine 54. Bile salt absorb from Ileum* Proximal jejunum Distal jejunum doudenum 52.wat is true C6* C4 C5 Cricoid cartilage 53. Circumflex artery a br. metastatic carcinoma least common site brain lung liver spleen(correct) 56. iron absorb in ferruos form* oxalate 5 1 . wat the reflex gagtrogastric reflex gastro coic reflex (correct) . arterial supply to primary visual area PCA* ACA MCA PICA 58. Trachea start from …. premotor area . blood supply to Bundle of His. A boy defecate just after he takes meal. Cerebral 57. which artery supplies Anterior and middle cerebral (correct) Anterior Middle Middle and post.50. 59.MHC complex is related to HLA typing* blood grp antigens same in identical twins 68. A surgeon did a study on patients with heamorroids he included 100 pt in grpA and treated them surgicall whil kept the other 100pt on high fiber diet he interviewd them after 12mons and found that the grp with surgical treatment had a better response…… wat type of study is this? Case control Cohort Prospective Randomized control trial 63. nerve supply to latismus dorsi thoracodorsal* subscapular long thoracic 6 1 . Median of 20.20.30.5* 30 60.35. first part of axillary artery ends at border of pectoralis major border of pec minor * .25. Right bronchus a s compare t o l e f t is longer ant to pul artery right bronchus is more vertical than left* 62.40 25 20 27.30.sup parathyroid gland is related to sup thyroid artey* lies outside the pretracheal fasciae lies anterolatral to thyroid 67. Doctor patient relations and how good a doctor is known by his degrees his records his pleasant behavior and bed side manners* 64.Benedicts test is done for blood sugar urinary glucose urinary reducing sugars* 66. if a patient develops a fatal disease it is best to hide it from him tell it to him and his family as soon as possible* 65.25. Kleinfelters syndrome chromosomal deficiency autosomal dominant autosomal recessive extra X chromosome* extra Y chromosome . A patient w i t h multiple fractures and BP 9 0 / 6 0 pulse 6 0 / m i n w a t is the most appropriate initial treatment? IV fluids* Reduction os fractures 73. Factor 9 def wat is the appropriate treatment? Cryoppt* FFP Whole blood WBC 74. Myleination of the brain wat is most inappropriate? Starts at birth Occurs hapharzadly Sensory neuros are myelinated first Occurs by oligodendrogliocytes 7 2 . DIC most inappropriate low FDP* consumptive coagulopathy treated with heparin low platelets 75.Esophagus is related to which structure whose dilatation can cause dysphagia l atrium* left vent SVC Rt atrium 70. If RCA is occluded distal to the origin of right marginal artery which part of the heart will be affected most? Rt atrium SAnode AVnode* Apex of the heart 7 1 .border of teres major 69. which organ has least chance of metastasis lung bone liver kidney spleen* 80.influenzae 81. OCP increase the risk of breast cancer endometrial cancer thromboembolism* osteoporosis 83. thromboembolism is most common in? leg* liver brain kidney 82. Fat embolism most unlikely urinary test is diagnostic* occurs due to trauma to fatty tissues caused bt acute pancreatitis 77. which organism causes meningitis secondary to pyogenic lung abscess? Strep Staph Pneuococcus h. Highest mortality rate Hepatitis A Hepatitis B Hepatitis C Hepatitis D Hepatitis E 78. Diphtheria is transmitted by sexual contact respiratory * feco oral 79. Osteoporosis is less in premenopausal women due to effect of estrogen* testosterone progesterone androgens th 84. A trauma patient in shock with stab wound along left 10 rib which organ is damaged? Liver Spleen* .76. Bladder S. About large gut descending colon has no haustra appendix has no tenia* all large gut is supplied by inf mesenteric artery ascending colon has a mesentry 93. most imp for wound contraction myoblasts fibroblasts myofibroblasts* epithelium collagen 89. hyperparathyroidism causes least likely hypertension dystrophic calcification* inc PTH 9 1 .Colon Duodenum 85. in autosomal recessive disease all siblings are affected both parents are affected only mother is affected all siblings have a chance of one into four of contracting the disease* 86. . Ductus deferens ends into Prostate u. most probable abt gall bladder. Most likely finding in CSF inc glucose in fungal meningitis dec glucose in viral meningitis lymphocytosis in aseptic meningitis dec glucose in pyogenic meningitis 88.which of these is not caused by bacteria histoplasmosis* 87. smooth muscles are striated voluntary mostly arranged in circular and longitudinal layers * 90.vesicles Prostatic urethra Ejaculatory duct* 92. what Is true abt Hb consisits of alpha and gamma chains in adults imp buffer of H+ * not assoc with CO2 transport 100.After gastrectomy patient will develop iron def anemia megaloblastic anemia due to folic acid def pernicious anemia* sideroblastic anemia 98. Which HBpathy presents with crises sickle cell trait sickle cell disease* thalassemia Hb C 96. unconjugated bile is carried by which protein? Albumin * Pre albumin A globulin B globulin Fibrinogen 97.Has thick submucousa Mucousa has extensive folds* 94.PDGF is secreted from which part of platelets alpha granules dense bodies cytoplasm lysosomes . a patient with known HBV infectio comes with HBSAg + HBe ab+ HBV DNA + HBC IGM ab+ Wat is the diagnosis? Acute dis * Chronic carrier Chronic active 95. 60 yr male with lymphandenopathy and low hb low platelets and increased wbc microscopy show large mature cells wats the diagnosis? CLL* Burkitts Follicular NHL 99. which cells produce antibodies plasma cells* T cells Bcells Platelets 102.* Has parthyroids ant to it 106. Heparin is released from which cells. Mast cells* Eosinophils IgE 103. wat is true abt pituitary ant pitutiary derived from neurons no portal vessels btw hypothalamus and post pituitary* 107. Epitheloid cells in granulomas are formed by which cells macrophages lymphocytes langerhans cells* 104.1 0 1 . at the end of pregnancy uterus is most sensitive to which hormone? Estrogen Progesterone Prolactin Oxytocin* 110. D & C at 18 days after LMP endometrium will show which stage of uterine cycle? Follicular Ovulatory Secretory* Proliferative 109. Most abundant phagocytic cells in circulation basophils monocytes . Satiety center is located in which part of hypothalamus anterolateral VPL VL Medial* 108. thyroid is invested in which fascia? Pretracheal* Deep cervical Prevertebral 105. wat is true abt thyroid consists of collecting ducts and acini epi changes shape according to state of activity. macrophages neutrophils* 111. which structure is most ant in popliteal fossa popliteal vein politeal artey * tibail nerve . which of these muscles is a part of the pelvic diaphragm deep transverse perinei puborectalis* bulbospongiosus ischeocavernosus 118. Pallegra is due to def of riboflavin thiamine niacin * vit C 114. ICF & ECF differ in inc K+ in ICF 112. most potent stimulus for release of aldosterone is inc K+ * inc Na+ inc glucose 113. in females which structure lies btw ureter and peritoneum uterine artery* inf vesicle artery 117. wat is not true abt adrenal gland fetal gland is 1 / 3 size of the adult gland it is invested by renal fascia adrenal cortex is derived from mesoderm 119. Trauma to the middle meningeal artey causes subdural hemorrhage epidural hemorrhage* SAH 115. patient with loss of fine touch proprioception and vibration injury to which part of spinal cord? Fasiculus gracilis Fasiculus cuneatus Post white column* Lateral white column 116. a bullet pierces the intercostals space which layer will it come after the intercostals muscles parietal pleura visceral pleura endothoracic fascia* pleural cavity 126. major part of energy utilized during breathing is to overcome elastic recoil of lungs* resistance of chest wall large airway resistance small airway resistance 125. which of these are the major resistance vessels arteries veins capillaries arterioles* venules 122. a person in a room with optimum enviormental conditions how will the heat loss occur convections sweating breathing radiation and conduction* voiding 124. which vessel will be damaged if the phrenic nerve is cut musculophrenic int thoracic pericadiophrenic* 127. patient losses 1 litre of body fliud in 1 hour wat will happen first inc PR inc BP inc veno spasm* 123. which of these is not a branch of internal iliac .tendon of which of these muscles is intracapsular bicep femoris plantaris popliteus* semitendinosus 121.bicep femoris gastrocnemeus 120. metastatic tumors are identified by invasion of other structures* pleomorphism inc NC ratio inc mitosis 133. wat is seen in dysplasia pleomorphism inc NC ratio* inc mitosis 132. most unlikely abt glomerulus is . pseudo hypoparathyroidism. sarcomas have rapid growth inc vascularity* capsule benign 134. Due to inc ca Dec vit D PTHrP 131. lymphatic of uterus do not drain into int iliac nodes ext iliac nodes sup ing nodes inf mesenteric nodes* 129.sup rectal* middle rectal inf vesicle sup vesicle 128. epi of a smoker will show inc ciliated epi mixture of st sq and pseudostratified cells* dec goblet cells 135. a patient with hoarseness and lyrangeal nodule which is most unlikely atrophy * nodule hypertrophy hyperplasia 130. most unappropraite abt liver has dual blood supply hepatic artey gives only 3 5 % of O2 portal vein has 7 0 % O2 * 138. not part of the portal tract portal vein hepativ artery bile duct hepatic vein* 139. in males uretric stones mostly lodge at which point just below kidney* at crossing of ext iilac vessels at pelvic brim at ischeal spine 142.has urinary space btw 2 layers is cup shaped is blind ended part of tubule has stratified e p i * has podocytes 136. Clavipectoral fascia completely covers pectoralis minor* forms suspensory lig of breast forms axillary tail 141. if there is fracture of the acetabulum post and superiorly which bones r inv ileum and ischium * ileum and pubis isschium and pubis 143. not a part of axillary nodes ant grp medial grp apical grp central grp deep cervical* 140. which vessel is involved and dilated in portal HTN portal vein* hepatic artey hepativ vien SMV 137. most constricted part of the male urethra? Ext meatus* Infandibukum Navicular fossa . middle menigeal artery enters thru which foramen? Foramen spinosum* Foramen rotundum Foramen ovale 150. Great cerebral vein does not drain deep cerebral vein sup cerebral vein basal vein thalamic vein 152. pain of the ovary is transmitted to the medial side of thigh thru which nerve femoral obturator* ilioinguinal 146. epidural space contains venous plexus* contiues into skull at foramen magnum attached to dorsal coccyx upto S2 148. Otic ganglion lies under foramen ovale* foramen spinosum maxillart nerve stylomastoid foramen 149.144. left ventricle does not contain supraventricular crest chordae tendinae aortic vestibule papillary muscles 147. Superior petrosal sinus lies in falx cereberi fakx cerebelli tentorium cerebellli* diaphragma sella 151.which of these is not a basic tissue of the body epi nerve . at the level of second part of duodenum wat is unlikely aorta at the right of SVC* left renal vein 145. which vessel lies ant to IVC left renal vein left renal artery right renal vein right renal artery right testicular vein* 158. Osmotic pressure depends on electrical equivalence conc gradient yenp no of particles mol size 154. if inguinal canal of a female is opened wat does it contain round ligament and ilioinguinal nerve 159. lymphatic drainage of testis is into para aortic nodes sup ing nodes int iliac nodes ext iliac nodes . chemoreceptors respond to inc PCO2* dec PO2 inc Ph inc temp 155. Processus vaginalis covers testis only* covers ductus deferens forms ext spermatic fascia arises from parietal peritoneum arises from visceral peritoneum 156. Dorsal rami of nerves supply ext of trunk ext of limbs 157.muscle blood CT 153. . Starts at the lower border of cricoid cartilage 6. Cimetidine c.012 24-Mar-12 4:32:00 PM Fcps part 1 paper of medicine n allied including both paper 1 n paper 2 held on 13 march 2011. Regarding trachea a. Omeprazol b. Recurrent laryngeal nerve pases anteriorly d. Isthmus of thyroid lying in front of 4rt or 5th tracheal ring e.due to a. which drug can b given to decrease gastropresis a. Migration of tumor cells b. Musculocutaneous nerve d. Metclopramide e. Defective embryogenesis d. Long thoracic nerve c. Damaging extracellular matrix e. Antacids d. 2. Antibodies against thyroid hormone crossing placenta e. On motor bike accident fracture of neck of humerus n can. Maternal iodide deficiency b. What is most common cause of congenital hypothyroidism: a. Tumor spread by: a. sepeation of tumor cells from one another d. Breakdown of e-cadherin c. 5. Diabetic pat taking oral hypoglycemic drugs complain of abdominal fullness. 3. Inborn error of metabolism c. 1. 15 cm in length c.t abduct his arm. Gap in the C shape cartilages are lying anteriorly b. Release of acid from stomach is mediated through . 4. e. Axillary nerve b. food in stomach d. Increase globulin and albumin ratio e. HEV e. e. 9. Iincrease action of procain by epinephrine b.5 d. Increase plasma globulin b. HAV b. C3. Increase fibrinogen c. H2 receptor c. Best example of pharmaco kinetics drug drug interaction: a.she has returned from remote village. Temperature 1 1 . C4. Azygus vein b. A female at 36weeks gestation cmwith bili.5. C3 b.a. HBV c. HCV d. Increase plasma Albumin d.20mg/dl raised SGPT & SGOT. What is the structure arches around the root of left lung a.4. What can b the cause of infection? a. C4. 7. Left recurrent laryngeal nerve 10. Increase action of by Calcium c. Left phrenic nerve d. Left vagus nerve e. H1 receptor b. Toxicity of lithium with thiazide diuretics .6 e. Diaphragm is supplied by a. Arch of aorta c. 8. Regarding ESR decreased by increase in which of the following a.5 c. e. e. e. Mesoderm c. 1 7 . Middle cardiac vein b. Notochord arises from a. Mesoderm c. Endoderm d. 14. Alpha 1 antitrypsin b. Thyroid is derived from a. z lines c. 13. Ectoderm b.d. Ethyl alcohol d. Which one of the following is a chemical carcinogen a. which vein continues as coronary sinus a. Reverse the action of aspirin by NaHCO3 e. Benzidine c. Contractile unit in skeletal muscle is between a. Following veins drain into coronay sinus. h discs b. t tubules d. Reverse the action of Morphine by Naloxone 12. Great cardiac vein c. Endoderm d. Anterior cardiac vein d. A / P radiograph of chest which structure forms right border of heart a. Oblique vein e. 16. 15. Propyl alcohol e. SVC . Ectoderm b. corticosteroid e. 22. IVC c. Thirst stimulated by a. e. Salt depletion d. Uv light d. Regeneration least effected by a. 20. Diabetes b. e. 60 percent of work of breathing a. Arch of aorta e. 40%motor 60%sensory c. Stroke due to essential hypertension c. 19. Infection c. ECF volume depletion b.b. 2 1 . 23. Resistance by large bronchi e. Primary cartilaginous joint a. Regarding nerve supply to skeletal muscle contains a. Costochondral . Alzhiemers disease b. Resistance offered by small size bronchi d. Lower motor neuron lesion e. 60%motor 40%sensory fibers b. ICF volume depletion c. Right ventricle d. 60%motor 40%sensory and few postganglionic sympathetic fibers d. is to overcome elastic recoil of lungs b. Upper motor neuron lesion d. In downs syndrome at 40 years of age a. Counteract chest wall compliance c. Left Atrium 18. Endothelial damage. . Transcobalamin d. 26. 2 yr old child presented with anemia and has hyper segmented neutrophil. reuptake and enzyme catalysis c. MCV raised. how will you confirm your diagnosis a. fibrin c. high blood flow.b. AST e. After 6 hours of myocardial infarction which enzyme level most raised a. Inferior tibiofibular joint d. altered blood constituents b. ALKALINE PHOSPHATASE 28. Enzyme catalysis 25. Diffusion and reuptake b. CPK b. Plasma high blood flow e. Epinephrine reuptake by a. e. slow blood flow. ALT d. Endo damage eddy blood flow d. Estrogen b. LDH c. Intrinsic factor antibodies b. 27. Estrogen and progesterone c. Thrombus formation triad a. Progesterone d. Symphysis Pubis e. In adult female breast atrophy is commonly due to decrease a. Teeth and jaw c. Reuptake e. Diffusion. Prolactin e. Endothelial damage. Diffusion d. 24. Vit b12 level c. Coffee b. 2x2 table in biostats? a. A 30 year man participated in marathon race the most common hormone changes you will find a.48. Spleen and splenic vessels e. Splenic flexure of transverse colon 3 1 . Thymus d. Chi Square Test c. A person has pH 7. Low Insulin. Low Glucagon c. String beans 30. Left kidney is not related anteriorly to which of the following structures? a. Diaphragm b. High Glucagon b. T-Test b. High Insulin. CO2 66 and HCO3 27 . taking which of the following could cause hypertensive crisis? a. What is the . 33. High Insulin. 32. Stomach c. Low Insulin. High Glucagon d. Payer's patches e. d. Pancrease d. Pine apple e. Tonsils 34. Lymph node c. Patient taking MAO inhibitors. Low Glucagon e.29. Which of the following lymphoid structure does NOT contain lymph nodules: a. e. Spleen b. Cola d. Chocolate c. A patient suffers from appendicitis. Complex endometrial c. IL 6 e. Peronei cause eversion d. d. Aschli tendon plays imp role when we r standing on toes e. Eosinophilia e. Serotonin d. In myocardial infarction.diagnosis a. Partially compensated metabolic alkalosis 35. Partially compensated metabolic acidosis e. Tibialis anterior n tibialis posterior cause inversion c. enkephalin 36.. which earliest change occurs in formation of aschoff bodies a. Interstitium . Acute respiratory acidosis b. Regarding movements of foot: a. Bradykinin and prostaglandin E c. IL 1 and TNF alpha b. 45y lady comes with vaginal bleeding doc advise her for hysterectomy due to premalignant condition which is it? a.. Partially compensated respiratory acidosis d. Inflamaton c. Fibrinoid necrosis b. Pain will be mediated by a. Inversion n eversion occur at subtalar joint b. Peripheral blood shows leukocytosis. 39. 38. Child with generalized edema and proteinurea 6g per day which part of the kidney involved a. Squamous metaplasia e. 37. Acute metabolic acidosis c. Simple endometrial. Atypical endometrial hyperplasia b. Giant cell d. 44. Initial response to tissue injury is: a.b. HDL c. Distal tubules 40. VLDL d. Platelet adherence d. Pulmonary vasculature b. Vasoconstriction b. Loop of henle c. Hypoxia causes vasoconstriction in a. 42. Chylomicrons b. LDL e. 4 1 . Brain d. Dital convuluted tubules e. After 6 hours of myocardia infarction which enzyme level most . Clot formation c. More phagocytic in bloodstream c. Collecting tubules d. Heart c. 43. Decrease in infective conditions d. Proximal convulated tubules e. High levels of protein associated with a. Young female developed acute renal failure after post partum hemorrhage. Collecting tubules d. Basement membrane c. Proximal convuluted tubules b. Kidney e. Decrease when corticosteroid given e. Regarding neutrophil correct a. e. which part of the kidney is most likely damaged a. Migrate into and out of blood stream b. 45. Hepatitis B c. Basillar artery d. Hepatitis A e. ALKALINE PHOSPHATASE 46. Blood supply of motor cerebral cortex a. Duration c. what is most inappropriate a. CPK b. present in midbrain 50. a. 49. Hepatitis C d. Diff b/w first n second heart sounds. is present in frontal lobe b. 47. With carotid pulse d. 3rd ventricle. LDH c. choroid plexus is supplied by internal carotid and basillary artery c. ALT d. e. Hepatitis D 48. Anterior and posterior cerebral artery c. is connected to 4th ventricle via cerebral acqueduct e. Frequency b.raised a. . e. AST e. Doesn't receive any sensory input c. choroid plexus is present in its floor b. Alcoholism b. What is most characteristic of cerebral motor cortex a.. is connected to lateral ventricles via interventricular formina d. is essential to carry out voluntary movements d. essential for stretch reflex e.. Anterior and middle cerebral artery b. Most common cause of liver cirrhosis in an adult male? a. Right bronchus a. Secondary diabetes mellitus e. Ionized state b. Internal carotid artery . 54.5 1 . Dry gangrene b. Internal jugular vein b. Is shorter b. Ischemia in Diabetes mellitus commonly results from a. Fasting blood sugar of patient is 6. e. Electric equivalence d. 55. Which structure doesn’t lie deep to the parotid gland? a. Lipid solubility c. Diabetes mellitus b. long lag phase d. Endarteritis obliterans d. GABA b.7 mol(194 mg/dl) what is the diagnosis.8 mol(122mg/dl) after 1 hour of gtt its 10. Glycine c. a. Is vertical c. Gas gangrene c. is infront of pulm artey d. e. Wet gangrene e. Impaired gluc tolerance c.which neurotransmitter? a. 56.. 52. 53. Glutamate d.. Rods and cones wid excitatory potentials to ganglion and bipolar cells. is infront of pulm vein e.9 mol and later after 2 hours post prandial it was 10. Distribution of drug enhanced by? a.. CIN I I c. Bad news. ovary c. xi nerves in the carotid sheath 57. first part of the duodenum e. HPV 6 1 . Their friends b. round ligament of uterus 62. Which of the following tumors doesn’t cause distant metastases? a. Other pts d. fallopian tubes b. All the staff involved in patient's care should be present c. At bedside b. Regarding counseling. when delivered to patient. is good for a. Ca in situ e. 60. CIN I I I d. CIN 1 b. in environment of exclusivity e. in a formal session. dysgerminoma c. Themselves e. Family members c. In doctor's office. verucous carcinoma d. when done to pts. 59. Squamous cell carcinoma e. Pharynx d. In front of all family members d.c. Parotid duct e. 58. basal cell Ca b. should ideally be a. gonads d. Which one of the following organ is present inside the peritoneal cavity? a. What is the most common cause of increased vascular . x. cx shows mild dysplasia with squamous vacuoles: a. 66. 1 / 3 of tbw c. 5 min b. Inflammation b. e.permeability? a. Trauma c. Regarding icf a. Regarding Vitamin K a. Immunological d. 63. prevents coagulation e. d. a. 2hr c. glans and prepuce of penis b. Which of the following non hairy regions does not have sebacous glands? a. is a water soluble vitamin b. When given in neonatal hemorrhage. 67. 2 / 3 of tbw b. produced by intestinal bacteria c. Psoas muscle a. 65. present b/w lumber plexus c. originate from all lumbar vertebra b. passes in front of inguinal ligament d. Allergy e. e. palms and soles of feet . e. present in large amount in human and cow's milk d. Half life of carboxyhaemaglobin. labia minora c. margins of lips d. 64. 6 hr d. D d. skin lesions d. fibrinogen e. Patient presented with painless heamaturia along with fever. diagnosis is a. Renal Cell Carcinoma . gatrointestinal hemorrhage c. cholesterol in blood b. latearl lemniscus c. C 7 1 .. A c. e. a. decrease in height of action potential d. inc. In 90 percent of anthrax infections a. temperature 69.e. gray commisure e. ultrasound abdomen plain is normal. Hyperexcitability c. Hemorrhagic pneumonia b. An alcoholic patient comes with wernikes korasakoff syndrome. It could be due to def of which of the following vitamins a. nipple 68. globulin c. 73.. Hypokalemia causes which of the following. night sweats for 2 months. Not related to spinal cord a. B1 b. Hyperpolarization b. plasma albumin d. 70. bacteremia e. 72. K e. ant gray horn b. posterolateal sulcus d. ESR decreases with increase in: a. CXR shows Bilateral Hilar Lymphadenopathy. heart . 76. diagnosis is a. IV Corticosteroides c. Starvation e. Which of the following suppress GH release? a. Acute Pyelonephritis c.which of the following is indicated a. 1st degree heart block b. d. Somatomedin d. MI d. 75. Stress 78. 74. Azathioprine b. In which of the following conditions. Sleep c. Which organ has a highest arteriovenous shunts a. 77. e. A patient with cervical lymphadenopathy. liver b. Biopsy shows caseating granulomaous pattern. e. Puberty b.b. Acute tubular Necrosis e. lungs c. Sulfasalazine d.atrial repolarization is evident on ECG a. A 21 year old having severe Ulcerative Colitis. Tuberculosis c. Deep inspiraion e. Renal tuberculosis d. Sarcoidosis b. 3rd degree heart block c. There was some question from biostats that which is most inappropriate? a. 79. iv heparin c..2 d. 80.01 d. 82. kidneys e. s1 c. dec end-systolic volume b. oral anticoagulant e. exploratory p<0. A boy engulfs peanut and it got stuck in the air way where would it occlude? . d. L1. through diaphragm e. L3 L4 b. e. A pregnant lady has mitral stenosis. Lymphatics c.d. Root value of knee jerk a. to prevent atrial fibrillaton treatment is a. analysis of variance that it can be done by one thing e. dec end-diastolic volume c. If ejection fraction is increased there will be a. im heparin b. experimental p<0. Haematogenous d.05 c. 84. Direct b. 8 1 . 83. e. Amoebic liver abscess spread to lungs by a. heparin and oral anticoagulant d. something was abt variance b. stimulating production of NK cells b. hydrolysis e. How many ATPs are produced after complete metabolism of one molecule of Glucose? a.a. Female child born then sent home. karyolysis b. 4 0 e. TRANSPOSITION OF GREAT VESSELS with no communication like VSD/ASD . left lower bronchus e. Cyclosporin acts by a. outline irregular a. right main bronchus b. 15 hours later she presented with cyanosis + weak pulse & diagnosis? a. 3 2 c. 85. bcl2 c. 8 7 . structure less material in center. right lower bronchus d. 8 8 . Acidophillic cytoplasm with basophilic granules. VSD b. e. hydrops degeneration c. e. 4 4 8 9 . apoptosis d. inhibiting NK cells d. ABL d. modifying maturation of T cells c. 3 0 b. 3 8 d. 8 6 . Which of the following is a tumour supressor gene? a. Ras b. right middle broncus c. they are seen in the a. ACTH e. amygdale . Lymph flow from the foot is a. Oxytocin 94. Collagens b. increased by massaging the foot c. Starc e. Somatostatin b. hypoplastic Left ventricle d. Dopamine d. most powerful stimulus for rennin a. cerebellar cortex c. increased when capillary permeability is decreased d. mediobasal portion of the hypothalamus b. midbrain reticular formation d. Cortisol c. Peptidoglycans c. dec NA Cocent to tbules b. e. PDA 90. Pectin d. decreased when the valves of the leg veins are incompetent e.c. increased when an individual rises from the supine to the standing position b. decreased by exercise 93. Dietary fibers a. In addition. Which of the following is not synthesized in both endocrine glands and the brain? a. Regular rhythmic fluctuations in electrical activity are observed in the cerebral cortex and thalamus. TOF e. 9 1 . hypotention c. 92. sympathetic stimulation d. b. curve the most likely value of T50 would b? a. c. e. the dominant EEG rhythm observed with electrodes over the occipital lobes is a.homonymous hemianopia. b. 20 b.homonymous hemianopia 98. alpha (8-13 Hz) d. theta (4-7 Hz) c. b. The part of basal nuclei is ? a. irregular low-voltage activity 96. bitemporal hemianopia. lt. In right shift of oxygen dissc. Absent cones.e. What will be the symptoms if there is leison in left optic radiation? a. delta (0. A person cannot differentiate between red n green colour while making presentation on computer. what is the most likely defect in this patient ? a. d. Absent rods. pons 95. 18 c. 99. e. putamen. alert adult sitting with the eyes closed. macula lutea d.5-4 Hz) b. beta (18-30 Hz) e. rt. In a healthy. Hypothalamus . Subthalamus d. 26 d. c. fast. 97. 35 e. amygdolid. Blind left eye. binasal hemianopia. c. sup mesenteric artery d. 1 0 1 . QRS b. QT SEG c. e. A male 60 yr Generalized lymphadenopathy. Damage to median nerve a. ALL d. on peri. e. CLL b. loss of adduction of thumb c. loss off sensattion on lateral side of hand d. mesocolon attachment b. 104. aut. 2nd part of duodenum is crossed anteriorly by a. smear lymphocytes looks mature cell a. 1 0 2 . 100.ressecive c. portal vein e. ST seg d. left renal vein c. CML c. x linked d. autisomoal dominant b. e. loss of abduction of thumb b. 103. 105. e.e. 20_40mg/protein . CSF composition approprate is a. Isoelectric segment of ECG during which the complete ventricular depolirization occurs a. adult polycystic kidney disease a. sensory fibers pass thru ant limb c. and post. during sleep hormone level is more than awake time a. Deciduas e. necrosis e. Amnion c. male 40 yr. ADH c. e. e. e. b. 107. 1 1 1 .. 109. 106. food poisoning d. anarobes infectionall all true execpt a. gangrene c. Primordial germ cells deriveted from. a. Sodium 130 meq/ml c. granuloma formation b. biguanides b. e. Internal capsule inapprorate is a. 108. epinephrine d. cortisol b. corona radiate transmit impulses to cortex d.b. acarbose d. glybenclamide c. limb contains cortiospinal n corticocerbellar fiber b. Glucose 120mg/dl d.15kg overwt with 20mmol blood sugar i treatment in addition to diet and excirse is a. Trophoblast d. lock jaw 110. wall of yolk sac. ant. Myasthenia garvis diz is . filtration.. penicillus arteries b. e..homonymeous hemianopia d. a. 113. type 1 c. a. 117. type 2 H. type 3 d. type 4 e. lt. facilitated transport e. 112.S reaction b. bitemporal hemianopia b. rt homonymeous hemianopia c..S reaction b.fetus will never have blood group ……….. b. 116.. type 2 H. Water flow from plasma to interstial fluid a. type 3 d.. a. e. lymph filtration in spleen occurs in a. valve in veins may be c.a. active process d. trabicular c. muscle contration of leg b. . type 1 c. 114. 115. Venous blood from the leg is increased during exercise. with sodium c.. father A . peripheral e.. poststreptococcal G. Optic chiasma lesion causes. intra thoracic pressure d.mother AB .N is. type 4 e. centre d. lysosome c. O+ve b. Female patient having ch.. e. facial nerve b. a. B-ve d. renal failure . b. a. acid . a. 120. sheehans synd.. twin b. O b. baby has blood group…. 122.. Mother –ve . B c. step brother e. Pituitary gland not related with…. cavvernous sinuus d. fibroid d. anemia c.. 118. father +ve 1st child with hepatospleenomegaly now 2nd child (erythroblastosis fetalesis).best donor for transplatation . optic chiasma e. endometrial hyperplasia e. step sister d. 1 2 1 .. O-ve c. lady with PPH can not lactate her baby inspite of willing for lactation due to a.. 119.a. husband c. AB d. In saliva bactriocidal agent is ……. a. lysozymes b. sphenoid sinus c. A e. a. 127. dec salt retention d. Aldosterone . Rationale treatment of essenial HTN a. a. Antibiotic given with serum conc. alpha blkers b. a. 1 branch from renal art c. aldosterone c. amylin e. a. e. drug to be given to antagonize symptoms …. e. Nephrotic synd. Atropine c. omeprazole d. Alcohol b.d. e. Epinephrine d. salivation. 124. by …… a. e. Adrenal glands blood supply most inappropriately. 128. dec hydrostatic pressure c. ADH dec. b. Angiotensin receptor blocker d. Pralidoxime b.. all branches of arteries from aorta. e. colloidal osmotic pressure b. vancomycin b. with Generalized . cyclosporine c.. 126. mointering . dec. Gardeners spraying inscticieds become sunconseous. IgA 123. 125. 1 br from phrenic d.edema due to…. 134. sarcoidosis e. inc hcl d. inc salt d.s drug is … a. decrease body water e. Palantopharngeus muscles moves laterally b. Palate move downwards d. 132. Ulcerative colitis c. PPI. H2 blockers … a. Basophilia of cell due to… a. e. dec HCL secretion b. 133. corticosteroid c. omeprazole. b. anticholerngic agent b.c. e. Larynx moves down c. e. Metoclopramide d. delay gastric emptying c. starvation 129. Erythroplakia d. Osteomlytis b. 130. 131. Which is not malignant lesion a. Swallowing stage… a. e. Which is not immunosuprresnt…? a. Cimetidine c. RER . methotrexate d. all intercostals veins drain into azygous vein c. D n C d.. e.. e. 138. middlle zone d. . 135. apex of lung b. ultrasound c. e. a. a. Ventilation is very high at …. post triangle c. e. base of lung c. posteriorly external ilaic vessels 137. e. 139. 136. CIN of cervix most easy cheaper and valuable tst . enter rt angle in blabber b. Thoracic duct turn to left side d. golgi bodies c. lies above utrine artery c. 140. Pacemaker for cerebral cortex which show changes on EEG …. Correct about ureter……… a. is not close to cervix d. pap smear b. a. regarding posteror mediastinum. axilla d.. aorta gives 9 post intercostal br b.b.. nnucleus d. 3rd part of subclavian artery anyersum swelling present in… a. present in broad ligament e. ant triangle b. e.joint pain . GH c. 143. 142. drug of choice is… a. 144. HB type present will be a. dopamine c. resp distress. HBsc c. b.. i/v frisemide b. HB A2 d. HB ss b. isthamus at c123 d. Insulin e. Thyroid gland . e. anemia. e. streptokinase d. Ac ventricular failure. start at uppper border of thyroid cartilage c. glucogan. oxygen e. . a. Thalamus b.chest infection. Seotonin d. reticular formation c. 145. 1 4 1 . Patient with fever . Hormone causing Gluconeogenesis … a. dopamine b. Nigrostratail fibers secrete neurotransmitter … a.. Histological active follicles contain more colloid b. ADH d. Ach c. hypothalamus d.a. e. bladder sphincter e. dec gasticc emptying c. pain n filling sensation lost b. naproxen c. 149. hemophilia d. Tumor with least likely metastasis a. e.normal CT . all these findings are due to . dec tone of ext. Sympathetic block to urinary baldder. 1 5 1 . Vagal stimulation result in.. Alpha adrenergic .normal Aptt with clott retriction problem a. a.146. which will result?? a. 148. patient wth BT prolong .bladder sphincter d. block acid secretion d. inc tone of int. SCC c. e. lymphoma d. inc male fertility 147. Which of the following drug irreversibly blocks platelet action… a. FEMALE hyperthoriodism w i t h palpitaion and tachycardia. 150. asprin b. afibrinogenmia b... thrompocytopenia c. a. BCC b. inc gasticc emptying b. e. e. pulse rate 110. unable to empty bladder c. diclofenac d. beta adrgic response b. 98 % potassium is present intra cellularly 157. 153..all are correct except a. 152. GFR is measured by a.3 c. formation d. joins hypoglossal d. e. e.csf production is 500ml/day b. in under water e. Clinically. supply thyrohyoid muscle e. About ansa cervicalis…….O2 toxicity result from a. Heparin prevent thrombus a. hyperkalemia occurs in sever diabetes mellitus c.platelates degranulation play major role b. mast basophiles . Anti thrombin 3 is activated by . a. vagus stimulation d.Sensitivity. antibody antigen complex c. 155. Inappropriate for type 2 Hyper.c. Which one of the following is correct a. prolong o2 therapy b. 154. c1. organization 158.2. at high altitude d. inc PCO2 c.. PAH d. Pul. embolization c. supply to carotid body b. circulating complex 156. propagation b. inulin c. creatine b. This is most likely due to a.HSV c. Which of the following is inappropriate a. increase PR interval b. At alveolar level. In case of hyperthyroidism. rubella 160. decrease PR interval c.VWF 159. hypercalcemia e. hypertophy is inc in size c. which is inappropriate a. warferine c. hyper phosphatemia d. Regarding parasympathetic stimulation a. increase bone demineralization b. venous thrombosis . IgA c.a.. heparin b. mucus 164.hyperphosphaturia 163. decrease QRS interval 162. Which of the following organism produce hemorrhagic fever when person is infected for the 2nd time a. after some days patient developed skin ulcer at site of radiation. hyperplasia n hypertrophy can not occur in the same tissue 1 6 1 . microbicidal function is done by a. alveolar macrophages b. hyperplasia is inc in number of cell b.PAF d. A patient has bone sarcoma.CMV b. malignancy d. endartritus obliterans b. which is not healing. dengue d. radiations were given. infection c. calciuria c. core body temperature 99 F. branches in neck c. heat exhaustion b. in addition to blindness from . In which of the following vein venous PaO2 n PaCO2 is appropriate with arterial blood a. femoral vein b.. so increase dose should be given 168. pH d.PCO2 b. mitochondria c. veins on dorsum of a warm hand 169. can be given in increased intra cranial pressure c. subclavian vein e. peroxisome b. can be safely given with mono amine oxidase inhibitor b. was found to be unconscious.165. nucleus 166. more pulsatile b. Regarding Opiods a. External carotid artey is differentiated from internal carotid artery a. lysosome d.RER e. H ion concentration 167. n has hypotension. Respiratory center in brain is sensitive to changes in a. on examination he has cold clamy extremities. older persons are less sensitive to it. heat stroke c. anticubetal vein c. A person after some kind of thermal stress. intenal juglar vein d. which of the following phenonmenon occurred a. O2 n H2O2 is produced in neutrophil from a.PO2 c. hypoyhermia 1 7 1 . A patient has pituitary tumor. deep 170. hyperphagia b. Which of the following will result from hypothalamus lesion a. hyperthermia c. TOF d. it has also eroded lower part of hypo thalamus. Central artey of retina is related to a. RA c. VSD c.PDA 173. About chalamydia a.compression of optic chiasma. hypothermia 172. kaposi sarcoma c. dec urea secreted in urine 177. cystic fibrosis 174. optic canal d. cause trachoma c. burkitt’s lymphoma 175. facial vein . dec osmolality of tubular fluid c. nasopharyngeal carcinoma b. ASD b. is club shaped 176. superior orbital fissure b. dec interstitium osmolality.so dilute uurine is produced b. which one of the following is correct a. EBV causes which malignancy a. SLE b. Most common congenital heart disease a. dec calcium in urine d. cavernous sinus e. When loop diuretic given. inferior orbital fissure c. Which of the following autoimmune disease involves only single organ a. is aerobic organism d. cause choriorretinitis b. pituitary gland . right gastroepiploic artey is branch of gastrodudenal artery c. About broncho pulmonary segment a. Blood supply of stomach a. somatostatin release d. right gastric artery supply lesser curvature b. loss of hypoglossal nerve function on same side b. the presence of a halo is an ominous sign 183.178. supplied by tertiary bronchiole b. Medullary branch of vertebral artery is damaged on right side. short gastric artery is branch of hepatic artery 182. Insulin secretion is decreased by a. which of the following will result a. number of solute particles b. velocity c. vein is posterior to bronchus 180. What is true regarding fibroadenoma of the breast ? a. food intake c. is independent of each other c. loss of voluntary movement on opposite side 184. Osmlarity increase with increase in a. macro calcifications are usually seen d. size of solute particles c. Turbulence in vessel is increased by increase in a. glucagon secretion b. diameter b. artey is anterior to bronchus d. radius d. posterior acoustic enhancement in less than 5 0 % b. viscosity 179. loss of all sensation on opposite side d. molecular weight of solute particles 1 8 1 . regresses after menopause e. is likely to be tender to palpation c. loss of hypoglossal function on opposite side c. meso +endo 186. ecto + meso e. Question about Passive congestion of liver kindly complete the missing options n questions PAPER 1 and 2. Question about autoimmune hemolytic anemia 187.1 8 5 . mesoderm d. Connective tissue in all body except head n neck.physiology. ectoderm b. Question about MHC 1 8 8 .org/cgi/content/full/292/3/R1320) Mitral stenosis Aortic regurgitation Ans: A: 2 About NEURAL TUBE a) b) c) d) e) Forms primitive streak Forms notochord Cns develops from it Forms three germ layers … . is derived from a. endoderm c. paper 1 2009 DO CONFIRM THE ANSWERS 1 ECG changes can be seen in the following conditions EXCEPT a) Change in body position b) c) d) e) MI Sleep (http://ajpregu. org/wiki/Neural_tube ) 3 SACRALIZATION a) Union of 1st sacral vertebra with 5th lumbar b) Union of 5th lumbar with 1st sacral c) Fusion of all sacral vertebra to form sacrum d) Flexion at sacrum e) … Ans: B http://wiki.wikipedia. in haemorrhage b) Dec in gram negative septicemia c) Dec in heart failure d) … e) … ans.com/Q/What_is_sacralization_of_the_l-5 4 Stroke volume a) Cardiac output depends on it b) Heart rate determines stroke volume c) Increases in haemorrhage d) Independent of venous return e) … Ans. C (http://en.Ans.answers. DNT KNW EXACTLY 5 Central venous pressure a) Inc. b 6 What change occurs from lying to standing position? a) Venous pressure inc b) Arterial pressure inc c) Sweating d) Cutaneous vasoconstriction . bleeding from venae sites.htm 7 A simple senario of hurshspring disease 8 Which hepatitis dangerous in pregnancy a) A b) B c) C d) D e) E Ans. A http://www. who’s been deliverd in a remote village. presents with shock.whats diagnosis DIC 10. e 9 A lady.com/Cardiac%20Function/CF017. septicemia. B 11.cvphysiology. Regarding vit. Most common cause of pulmonary embolism? a) b) c) d) Heart failure Dvt Cancer lung Pneumonia e) … ans. Ans.e) …. D synthesis a) b) c) d) e) 1 hydroxylation ocurs in kidney 25 ocurs in kidney 1 ocurrs in liver 25 in lung Both in skin . Ans. A http://en.org/wiki/Vitamin_D 12.wikipedia. c 15. WHICH IS NOT SUPLIED BY OCULOMOTOR N. Auricular v. 13. Treatment of erythroblastosis fetalis? a) Exchange transfusion with b +ve b) With b _ve c) Ab +ve d) Anti D e) … ans. b) Content of carotid triangle c) Pierces deep fascia behind sternocleidomastoid d) … e) … ans. a) Medial rectus b) Lateral rectus c) Inf oblique d) Sup rectus e) Inf rectus 14. Cephalic vein . About external juglar vein a) Formed by retromandibular and ant. a) Present in deltopectoral groove 16. perferringens ws not givn) a) C. e weakness and paresthesia along ulnar nerv course 17. A qs about CSF findings. C. Viridans c) Strep pneumonae d) Staph epidermiditis e) … ans. which 1 true etc 18 Diagnosis of typhoid in 1st week Ans… blood culture 19. Cause of gas gangrene (C. Most common cause of SUBACUTE BACTERIAL ENDOCARDITIS a) Staph aureus b) Sterp.wikipedia. ljungdahlii Tetanus toxin C. septicum … ans. Type of omental necrosis? a) Caseous .botulinum b) c) d) e) C. d http://en.org/wiki/Gas_gangrene 21. A lil senario of cervical rib i. b 20. com. C http://books. tube b) Distal c. Stimulus for aldosterone release a) Hypernatremia b) Hypokalemia c) Hyperkalemia d) Alkalosis e) … ans. tube c) Loop of henle d) Collecting duct e) …. There were 4_5 qs about aldosterone 24. Ans.google. Most of bicarbonate absorption ocurs in a) Proxial c.pk/books?id=c41BJkHrniEC&pg=PA25&dq=omental +fat+necrosis&hl=en&ei=8iopTKykIaWlsQaB9JzEBA&sa=X&oi=book_result& ct=result&resnum=4&ved=0CDYQ6AEwAzgK#v=onepage&q=omental%20fa t%20necrosis&f=false 22.b) Gangrene c) Fat d) Coagulative e) Fibrinoid Ans.com. c .pk/books?id=pekX0WUKzMoC&pg=PA414&dq=bica rbonate+reabsorption&hl=en&ei=oCspTPu0CIO0lQftobDBw&sa=X&oi=book_result&ct=result&resnum=1&ved=0CCYQ6AEwAA#v=on epage&q=bicarbonate%20reabsorption&f=false 23.google. A http://books. Aldosterone causes a) Hypercalcemia b) Hyperkalemia b) Acidosis c) Hypernatremia d) … ans.org/wiki/Circadian_rhythm . For circardian rhythm optic nerve sends fibers to a) b) c) d) Optic chiasma Medial geniculate body Lateral geniculate body Suprachiasmatic n. c 26. D http://en. Dec aldosterone will lead to a) Hyponatremia 27. tube c) Collecting duct d) Bowman capsule Ans c 28. Ans.wikipedia. e) Upraoptic n. ADH acts on a) Loop of henle b) Distal c.25. D 32. D 31 Natural anticoagulant a) Heparin b) Warfarin c) Aspirin d) Plasminogen e) Fibrin Ans. C 30. A person suffering from dehydration. which ion replacement is most important? a) Ca b) Mg c) Na d) K e) Cl Ans. What to give to Inc vitamen in diet a) Nuts b) Yougurt c) Egg d) Green vegetables e) Margarine (sumthin like it) Ans.29. Endogenous pigment in chronic hemolysis a) Hemosiderin b) Lipofuscin c) Anthracotic pigment . htm 36.d) … e) … Ans A 33. b http://www. all the children there have some permanent disabilitythe parents of such children are suffering from a) Denial b) Anger c) Bargaining d) Depression e) Acceptance 34.berkeley. In a ward. . Square root of variance a) Mean b) Variation c) Standard deviation d) Median e) Acuracy Ans C 35. Mean Inc with a) Inc in sample size b) Dec in sample size c) Inc in standard deviation d) … e) … ans.stat.edu/~stark/Java/Html/SampleDist. at a children hospital. Which will not b present in a lung hamartoma (it ws senario of pulmonary hamartoma and qs ws asked at the end of senario) a) Coin lesion on xray chest b) Mostly asymptomatic c) Mostly Discovered incidently d) Benign e) Cartilage in lesion area Ans E 40. Drug for hyperthyroidism in pregnancy a) Methimazolepropylthyrouracil b) Ppu c) Radioactive iodine d) Total thyrodectomy e) Ligol solution Ans B 39. Difference BW hyperplasia and benign tumor .What is therapeutic index of a drug? 37. Drug contraindicated in renal failure a) Amikacin b) Chloramphenicol c) Vancomycin d) Pencillin e) … ans. a 38. e seizures and convulsions then asked that what other symptom can be found in that child and ans ws GLUCOMA .a) b) c) d) e) E Hyperplasia invades surrounding tissues Benign tumor metastasize to tissues Benign tumor shows aggressive growth Hyperplasia shows pleomorphisim Benign tumor is surrounded by capsule Ans 41. A senario was about sturge webber syndrome in which he gave neurological symptoms I. Atrophy means a) Dec in cell size 42. Submucosal glands are present in a) Stomach b) c) d) e) C Gall bladder Duodenum Small intestine Pancreas Ans . Point to note that they did not talk about PORT WINE STAIN (birth mark) on face or forehead A 44. Premalignant condition of oral cavity a) Erythroplakia b) Leukoplakia c) SCC d) Lichen planus e) … ans b 43. Bile salts are absorbed where? a) Terminal ileum 50.45. Which hormone Inc gastric contraction? . Absorption of long chain fatty acids? a) Ileum 49. Bile acids are conjugated with which amino acid to form bile salts a) Taurine 51. After gastrectomy B12 is not absorbed due to deficiency of a) Parietal cells b) Chief cells Ans A 47. After gastrectomy what can happen a) Malabsoprtion b) Steatorhea c) Anemia d) … e) … ans c 46. Gastroparesis treatment (last part1 me b tha) a) Metoclopramide 48. Stretch reflex maintains a) Muscle tone b) Balance c) Length d) … . choices were not straight forward e.g askin sum tract lesion etc dts y not remeberd 56.A) GASTRIN B) CCK C) Secretin c) Gip Ans A 52.T8 level and dy askd about wat will happen . There was a qs about injury at T4. Rupture of post wall of duodenal ulcer will damage a) Gastroduodenal artery 53 Left renal vein relation to aorta a) Ant b) Post Ans A 54. Final common motor pathway a) Corticospinal tract b) Alpha motor neurons c) Upper motor neurons d) Cerebral cortex e) … ans b 55. During systole a) b) c) d) Both ventricles contract simultaneously 5 0 % filling of ventricles occurs Blod flow inc in coronary arteries . e) . ans a 60. Regarding pulmonary wedge pressure a) Called so cz measured by a wedge shaped catheter b) c) d) e) Usually bw 10-15 mmhg Measure of rit atrial pressure … … ans b 59. There was qs in which options were (forgot qs) a) Pancinian corpucles b) c) d) e) Messiners corposccles Ruffinis end organs Golgi tendons …... 58.e) … ans c 57. Fastest conducting fibers in heart a) Purkinje . Most imp point of exudative inflamation a) Sp gravity < 1.020 b) Neutrophils c) Protein > 2 g | dl d) . e) … ans .61. Isoniazid side effects can be prevented by a) Pyridoxine 65. AFB +ve a) Leprosy 67. Not a cause of cardiogenic shock a) Cardiac temponade b) Mi c) Arrythmias d) Hemorrhage e) Aortic dissection Ans d 66. Treatment of acute asthma a) Terbutaline 64. A pt with deformed nose. Glomerular pressure inc due to a) Afferent constriction b) Efferent constriction Ans b 62. A senario was about dijoxin toxicity 63. Which tumor does not hav bone metastasis? a) b) c) d) Thyroid Liver Lung Breast e) Kidney Ans b 72.c 68. Monocytes a) Most abundant cells in circulation b) Do not go out of vessels c) Not phagocytic d) … e) … 69. Bladder ca caused by a) Schistosoma haematobiam 71. Tuberculous meningitis a) Inc lymphocytes in csf 70. ESTROGEN OCPs can cause a) Breast ca b) Ovary ca c) Endometrial ca Ans a . posteriorly by post. max expired volume is vc 75. Vital capacity a) After a max inspiration. Which is not covered by deep cervical fascia a) b) c) d) Thyroid Parotid Sternocleidomastoid Submandibular gland e) Sublingual Ans e 79. Belly of digastric. Belly 78. Regarding sup parathyroid gland a) Located in close proximity to INF thyroid artery and recurrent laryngeal nerve at the level of cricothyroid junction.its imp) 77. Digastric triangle a) Bounded anteriorly ant. (Remembr that sup glands hv more constant position while INF r variable in position. Circumduction a) Combination of flexion. a) Amount of air left in the lung after a max exhalation 74.that’s y dy askd about sup glands location. extension.73. aduction and abduction . Residual vol. Cardiac output a) Measured by fick principle 76. Micturation reflex centre is located in a) Brainstem (pontine micturation centre). then chest pain that relieved by sitting up a) Pericarditis 85. Approach to intercostal space for pleural effusion drainage a) Lower part of space (neurovascular bundle in upper part os SPACE but lower part of RIB) 84. In nephron fluid which substance conc is higher than plasma a) Na b) Bicarbonate c) Glucose d) Urea e) Albumin Ans . There s also a sacral micturation centre but it ws not in the choices 82. Pt having uppr respiratory tract INF. sory B) C) D) 81. why a) Dnt remember choices. Diagnosis of pneumothorax? a) Xray chest 83. For surgery tibia should b aproached from medial side.80. Renal plasma flow is determined by a) PAH 87. Tissue for HLA typing (there were two same qs in each paper) a) Buccal mucosa b) Leukocytes c) Skin d) Bone marrow Ans . type of hypersensitivity reaction? a) 2 90. blood suply. lymph. WHAT IS TRUE ABOUT UPPER HALF OF ANAL CANAL A) Options wr about epithelium.d 86. nerve sup. Dnt remember exactly 88. hepatitis. diarrhea 10days after bone marrow transplant a) Graft vs host disease 89 Erythroblastosis fetalis. WHICH VACCINE IS NOT LIVE A) B) C) D) Measles Mumps Rubella Yellow fevr E) Tetanus Ans e 91. Jaundice. thanks ALLAH Dy did NT gv both choices simultaneously : .b 92. Cauda equina a) Collection of nerve roots and rootlets 94 Pilocarpine is nicotinic a) Agoist b) Antagonist 95 Diagnosis of klinefeltrs syndrome a) Barr body 96 Which drug interers with warfarrin a) Cemetidine 97 Adverse effect of chlorpromazine a) Extrapyrimidal 98 Mechanism of action of captopril a) Ace inhibitor 99 Feature of ca . LP done at a) L4-L5 (l3-l4 also correct but it ws not in choices.) 93. a) Plomorphisim b) Nuclear ratio c) Hyperplasia d) Invasion Ans d 100 Most cancers hav which fillament in them a) Kerratin b) Desmin c) Vimentin d) Neurofibrilary e) … ans a 101 Paraneoplastic syndrome is associated with a) Ca breast b) Adeno ca lung c) Small cell ca lung d) Ca prostate Ans C 102 Hepatitis b INF is monitored by a) b) c) d) Surface antigen Srface antibody Core antibody E antibody 103 . Hepatitis A inf. which test to perform a) Hep A nd B VIRUS b) Bilirubin c) Sgpt d) Albumin Ans c 104 Which hormone inhibits insulin secretion? a) Secretin b) Cck c) Somatostatin d) Glucagan e) Vip Ans c (As a general rule all git hormone stimulate insulin release and knwn as INCRETIN) 105 MOST IMP STRESS HORMONE (it was also in last part1) a) Epinephrine b) c) d) e) Insulin Growth Cortisol Acth Ans d 106 In Cushing syndrome a) Neutrophils are Inc. lymphocytes and eiosiniphils are Dec 107 A simple senario of graves disease 108 . M. Mononeucleosis Walders (sumthin like that) granulomatosis … e) … ans a (nt sure) 111 Pt with arithritis. photosensitivity (these symptoms wr in the form of a simple senario). butterfly rash. M 15gm. G in different ratios and %ages e. g IgA 5gm. WATS diagnosis a) SLE 112 Platelets contraindicated in a) Splenomegaly b) Acute immune thrombocytopenia c) Coagulopathy d) Dic Ans b 113 Water moves through .Whats pattern of antibodies in multiple myeloma (options were vry weird. G 20gm b) IgM: igG S 1:20 109 HLA type in RA a) HLA DR4 110 ABSOLOUTE LYMPHOCYTOSIS NOT PRESENT IN a) b) c) d) Sle Inf. nt exactly remembrd) a) They hav igA. pdf+active+liste ning+for+a+doctor&hl=en&gl=pk&pid=bl&srcid=ADGEEShL3aencNeNL6PcD L75akSth1myGNOsIBbWqXMO-BxP5X7QKVmiu16qKkAOUhwMcNerrFuT2x-kVNZZz84Xr5Gy1K7AJJfrUhJoOWFb7PnEI52oZ3VXT9v3ZAKmA31w_EhEa0&sig=AHIEtbTDPrtyezt44aYcvJMnrIDzau8oZw 116 Which is more aggressive (also in last part1) a) Basal cell ca b) Squmaous cell ca c) Melanoma Ans b .racgp.google.au/afp/200512/200512robinson.a) Pores b) Membrane matrix c) Protein channels d) Fascilitated difusion Ans A (water channels) 114 Fascilitated diffusion a) Passive transport trough protein channels or carier protiens 115 Which factor strenthen pt doc relationship a) High professional skills b) High social skills c) Highly qualified d) Logical answers to qs e) Active listening Ans E http://docs.org.com/viewer?a=v&q=cache:B90RsYl_QIJ:www. which N. is damaged a) Fascial b) c) d) e) Hypoglossal Submandibular Edenger westphal … ans b 120 A pt with diastolic murmur in aortic area and colapsing pulse. diagnosis .117 Nissel bodies are a) b) c) d) a RER SER Mitichondria Golgi bodies Ans 118 Greatest area of cerebral cortex is covered by a) b) c) d) Elbow Knee Thumb Shoulder e) Ankle Ans C 119 Deviation of tongue on protrusion. a) Aortic stenosis b) Mitral stenosis c) Aortic R. which will b infected a) Spleen . d) Mitral R e) … ans c 121 SUB-DURAL HEMATOMA CAUSE A) B) C) D) SUP CEREBRAL VEIN INF CEREBRAL VEIN ANT DIVISION OF MA Post. Division of MA Ans A 122 Csf absorbed by a) Arachnoid villi b) Choroid plexuses Ans A 123 Which is pierced during LP A) Dura matter 124 Inf spreads retroperitonealy. b) Jejunum c) Transverse colon d) Descending colon e) … ans d 125 Ant pituitary loss will result in Dec in size of a) Zona glomerulosa b) Zona fasiculata c) Parafolicular cells of thyroid d) Adrenal medulla e) … ans b 126 Factor Dec wound healing a) Vit c deficiency 127 How thrombocytes play role in thrombosis a) Sorry forgotn the choices 128 Accomoation reflex intact. Damage to? a) Edenger westphal n. b) Optic nerve c) Optic tract d) Optic chiasma e) Pretectal area Ans E . light reflex absent. 129 Part of portal system draining esophagus a) b) c) d) D Azygous v.com.google. Hemiazygous Rit gastric Lft gastric Ans http://books.pk/books?id=21e4wArL7hQC&pg=PA18&dq=portal +system+draining+esophagus+is&hl=en&ei=N7w0TNLEDKjonQefitGDBA&sa =X&oi=book_result&ct=result&resnum=1&ved=0CCkQ6AEwAA#v=onepage &q&f=false 130 Bronchopulmonary segment a) Anatomical and functional unit iof lung 131 Temp set point is in a) Ant hypothalamus b) Post hypothalamus 132 Antibodies are produced by a) Plasma cells 133 Virulence of bacteria is associated with a) b) c) d) Dose Duration of exposure Toxin production Body resistence . e) … ans c 134 For a blood donor what is not required (choices not well remembrd.actualy dy askd that which factor should not b there in a GENERAL HEALTHY DONOR) a) Age b) Sex c) Systemic disease 135 Which one is not a epithelium tumor a) Adenocarcinoma b) Sq ca c) Liposarcoma Ans C 136 Which receptor r involvd in acid production a) H1 b) H2 c) Acetylcholine d) … e) … ans b 137 Which is present in slow wave sleep a) Dopa . b) Acetylcholine c) Serotonin d) Norepinephrine e) … ans c and d 138 Hemibalismus due to damage to a) Subthalamic N. It’s due to . now his urine is concentrated. 139 A child having dyspnea when lyng down a) Retrosternal goiter 140 Best buffer of body a) Protein b) Hb c) Hco3 d) Phosphate Ans C 141 Protein utilization is chekd by a) Urinary NO2 b) Blood NO2 Ans A 142 A pt has fluid loss. 4 po2 65 pco2 33 hco3 19 a) Metabolic acidosis b) Metabolic alkalosis c) Compensated resp alkalosis d) Resp alkalosis 144 What happens on mountain aclimitization? a) Ventilation inc 145 Syphil dignosis material taken from a) Blood b) Urine c) Semen d) Genital sores (lesion) e) Saliva Ans D 146 Best indicator of serum iron stores a) Serum iron b) Serum ferritin c) Hemosiderin .a) Aldosterone b) Adh c) Acth d) Cortisol Ans B 143 Ph 7. To biceps . 3.s1 149 Hypospedias is due to defect in a) Urogenital tubercle b) Urogenital fold c) Urachus Ans B 150 Cephalic vein a) Begins in anatomical snuff box b) Lies lat. 3 c) L2. c) Lat. 4.d) TIBC Ans B 147 Why more oxygen goes to alveoli at apex than at base? a) b) c) d) Inc compliance Inc blood flow I n v\q Inc pulmonary pressure Ans A 148 Urinary incontinence with overflow and excessive voiding. To radial a. damage to (qs nt remembrd well) a) S2.4 b) S2. 200 Ans D 153 One qs was about CVP. 120 d) 120…. 60 b) 60….d) Ends in axilla Ans C 151 Upper part of anal canal is a) b) c) d) Lined by stratified sq epi Drained by sup inguinal nodes Drained by inf rectal vein Sensitive to touch e) Sensitive to pain 152 Normal fetal heart rate a) 40…. 160 e) 160…. 100 c) 100….about its inc or dec in specific condition 154 Inc in systemic filling preesure causes a) Inc venous return 155 Beta-blockers do not cause a) Inc renin . no complication. cause a) Cerebellar lesion b) Upper motor neuron .its 1st visit to a doc. obese. What shold be the treatment? a) b) c) d) Insulin Insulin + sulphonylurea Sulphonylurea Biguanides e) Biguanides +sulphonylurea Ans D 158 Pendulous knee jerk.b) Vasoconstriction c) Bronchiodilation d) … e) … ans a 156 Lidocaine a) Inc PR interval b) Dec AP c) Dec automaticity Ans A 157 A diabetic pt with BS 20mmole\l . c) Lower motor neuron d) S2.influnzae meningitis a) Csf culture b) Blood culture c) Inc lymphocyte in csf d) … e) … an a 161 Antibody to TB bacteria is (dnt remember exact choices but they did ask regarding antibody against TB) a) b) c) d) e) Cell bound Membrane bound in plasma In saliva … 162 . 3.4 Ans A 159 In which parasite the respiratory symptoms predominate? a) Ascaris b) Cystecercosis c) … d) … e) … 160 Diagnosis of H. In which phase os cell dividion the chromosomes are arranged in chromatids a) Prophase b) Metaphase c) Anaphase d) Telophase 163 Atrial contraction coresponds to a) C wave b) P wave c) Qrs 164 Dorsal roots supply which musscles? a) Internal intercostal b) Externat intercostal c) Innermost intercostal d) Some back musscle 165 Internal spermatic fascia is derived from a) b) c) d) External oblique Internal oblique Transvesalis fascia Transversus abdominus Ans C 166 Inputs to cerebellum a) From vestibular system 167 . Resolving power of lens a) That enables to see closely related subjetcs seperatly 168 Parasympathetic stimulation causes a) Ciliary muscle contraction 169 RTA. skin is calm and cold due to a) Vasoconstriction b) Vasodilation 170 Most common manifestation of septic pt a) Tachycardia b) Hypotension c) Inc GFR Ans B 171 Premalignant condition a) Basal cell nevus b) Dysplastic nevus syndrome Ans B 172 Tumor involving lymph vessels a) Angiosarcoma b) Cystic hygroma .5L blood loss. 1. Ans D 174 What u find in primary hyperaldosteronism a) Inc k b) Dec k c) Hypotension d) Inc renin e) Inc angiotensin 2 Ans B 175 Which take part in synthesis of aldosterone a) b) c) d) e) Angiotensin 1 Angiotensin 2 Cortisol … … .Ans B 173 A female having very high levels of prolactin. suffering from homonymous hemianopia. It’s due to a) b) c) d) Compression of optic nerve Damage to optic tract Damage to optic radiation Compression of upper part of optic chiasma e) Compression of oculomotor n. What type of hypersensitivity reaction occurred? a) 1 b) 2 c) 3 d) 4 e) 5 179 Blood supply of eye except cones and rods a) Central artery of retina 180 They gv senario of klinefeltr and askd what u will find most common a) Gynecomastia 181 Aldosterone causes a) Hyperkalemia .176 Inc k causes a) Inc aldosterone b) Inc ADH c) Dec aldosterone d) Dec ADH 177 HYDROCHLORTHIAZIDE CAUSES A) Inc k B) Inc ca C) Inc mg D) Inc na 178 Hydrops fetalis occurs to RH –ve mother and RH +ve father. but forgot to tell about the subject in which he took the exam. ?subject Contributed by Dr Farhan Karim. 1.b) Hyponatremia c) Hypercalcemia d) Hypokalemia e) Acidosis OCTOBER 08 Questions.Pre cancerous in aids: Cmv Ebv* Hsv Hpv 4.Which harmone of the following stimulates thirst .Below the umblicus anterior abdomen is formed by: External intercostal and part of Internal oblique* Internal intercostals and part of transverse abdominis uscle External intercostals and transverse abdominis muscle Internal intercostals and transverse abdominis muscle 3.What is most lateral in cerebral hemisphere : Insula Pars triangularis * Pars interemedia Amygdala 2.In liver of a patient under the microscope a lesion was seen with central cheesy appearance surrounded by large cells with lymphocytes and fibroblast what type of necrosis is present Coagulative* Liquefative Fatty tuberculous 5. When r the ventricles most filled with blood ' Diastasis Rapid inflow Atrial systole Ventricular systole 10.Which of the following harmone stops ovulation during lactation .Example of wet gangrene Thromboembolism Bacterial endocarditis Pancreatitis 11.In a thyroid surgery mass was excisied which microscopically showed amyloids post surgically which marker would be taken in account PTH Calcitonin* Ca Po4 8.Adh* Aldosterone Renin oxytocin 6.Which variant rises with rise in BP TPR* Pulse Venous tone Arterial tone 9.Which has no antigens AB* OO+ A 7. Which of the following tissue is most radiosensitive Skeletal muscle Cartilage* Bone 17.In dorsal column lesion which of the following will be lost Pain .Estrogen Progestrone Prolactin* Oxytocin 12Which of the follwing delays wound healing Vit c* vita Immobilization Vit E 13.X linked dominant Pku Alkaptonuria Familial polyposis* Glycogen storage 16.feature of Downs Syndrome Cardiac anomolies Fallots Hypospadiasis Early death* 14Turner syndrome has Chromosomal defect Xlinked Short stature* Tall stature 15. Temp Crude touch Itch Proprioception 18.Common feature in nitrates.Midarm circumference is the measure of Proteins* Fats Carbohydrates Minerals 22. Throphylline is Postural hypotension Rapid pulse Vasodlation Venodilation 21.Pre carcinogenic is Berryliosis Asbestosis* Benzethe Pentamidine 20.Temporal Arteritis is diagnosis by Esr* Biopsy B/c WBC count 23. isoprenaline .In food poisoning death occur by .Asthma has low Fev1* Fvc Vc Tlc 19. The organ placed retroperitoneally is Spleen Transverse Bladder Ascending* Liver 26.Long term granulomatous disease is Crohn Celiac Sarcoidosis Ulcerative colitis* 25.Water reabsorption through ADH is done in Proximal Distal Collecting Loop of henle 26.Urinary bladder in male is Completely covered by peritoneum Superiorly covers From rectum separated by fascia of Denon Villiers 27.Exotxin of samonella Vibro cholera * Shigella Endotoxin of Shigella 24.Hydrocephalus occurs because of obstruction in I/v foramen* Sylvius Magendie Lushka . A boy presented in the ER with fracture with less seum calcium.Musle traversing the shoulder joint Crochobrachilis Sspinatus Ispinatus 31.Chemotactic factors functions mostly in Diapedisis Margination ' Migration Acute inflammation 29.28. Paramesonephric duct remnant in male is Appendice testis Appendix Ovary 30.Hipocampal injury causes Sex rage Hunger Loss of Memory* Temperature disturbances 27.What organ lesion causes hypothermia Preoptic Lateral hypothalamus Post hypothalamus * Pitutary 28.Blood spread of CA occurs 1st in Metaphysis Diaphysis Epiphyses Epiphyseal plate Cortex 32.He is . Example of synchondrosis is Symphisis Involves hyaline crtlg b/w 2 bones Costal joint 34.medially rotate Abducts.TGlycerides are absorbed in Chylomicrons Vldl Hdl Ldl .Juvenile polyps are also called Hamartomas Rectal polyps Internal haemorrhoids 37. medially rotate Abducts.suffering from Primary hypoparathyroidism Vit D deficiency 2ndry hypoparathyroidism 33. Gluteus medius .Erythropoietin is secreted from Yellow marrow Jg complex Macula densa Preitubualr capillaries Loop of henle 35. laterally rotate Flexes . minimus Adducts. medially rotate 36.laterally rotate Adducts . 38.About 1st lumbrical Arises from 2nd meta carpal Is bipennate Supplied by median nerve 44.Lumbricals are supplied by Median neve deep br of median n Deep br of ulnar n .In gastrectomy loss of following occurs Hcl Intrinsic facor Fats Vit D 39.Median nerve supplies Palmar medial 1/3 Palmar and dorsal medialm1/3 Dorsal medial 1/3 Dorsal 2/3 lateral 43.Adrenal gland is separated from kidney by Peritoneal fascia Renal fascia Perinephric fat 41.About antibodies Have varible2 heavy chains IgM is abundant 42.Hb is enclosed in RBC at stage of Normoblast Late normoblast Erythroblast 40. Lymph drainage of lateral 2/3 of tongue occurs through Sup deep cervical nodes Inferior deep cervical nodes Submandibular nodes Submental nodes 48.Lateral part of breast supplied by Ant group Post group Lateral group Central group 46.Right hepatic artery is branch of Hepatic artery .Left recurrent laryngeal nerve has Shorter route than right recurrent laryngeal nerve Branch of vagus nerve Supplies all the muscles of soft palate Supplies all the muscles of larynx 51.45.Posterior triangle in the neck has follwing boundries Ant ant border of SCM post omohyoid Post ant border of SCM and ant diagastric Ant post border of SCM and post ant border of trapezius 50.The muscle of passive inspiration is Diaphragm External intercostal Internal intercostal Scalenes muscles 47.Boil on the tip of the tongue will drain into following lymph nodes Submental submandibular Submental nd submandibular 49. he has Metabolic acidosis Respiratory alkalosis wirh ion gap Metabolic alkalosis 56.Patientt has Babinski + lesion is in UMN LMN Dorsal column Ant column .In CRF all of following occurs except Anemia Hyperkalemia Hypocalcemia Hypoposphatemia Hyperuricemia 55.Subcapsularis sinus is present in Spleen Lymph nodes Payers patches Palatine tonsils 53.gastroduodenal artery Ceoliac artery Splenic artery 52.Patient after radiation vomited for 4hrs Cl is raised.After illeal resection absorption of which of the following components does not take place Chylomicrons Bile Tglyceric acid Vit d Vit c 54. 57.Melanocytes originate from Neural crest Mesoderm Ectoderm 62.Structures which passes through the aortic opening is .Peristalsis in the intestine inhibited by CCK Gastrin Secretin GIP Histamine 59.One of the following is not a feature of anaerobe Cough Tetanus Pus Granuloma formation Gangrene 61.Inferior rectal artery is a branch of Ext illiac I n t illiac Sup epigastric Int pudendal artery Femoral artery 58.Granuloma formation does not occur in TB Cat scratch fever Toxoplasmosis Pneumonia 60. Liver helps in immunity by Detoxfying noxious agents Kupffer cells Hepatocytes 65.Constricting afferent ducts in kidney reduces .IJV drain in Petrosal sinus Sagittal sinus Straight sinus Sigmoid sinus 64.Sour and hot is perceived by tongue through Von ebner cells Folate papillae Pain fibers Nociceptors 66.In testis seminferous tubules show following under M/S Sertoli cells Spermatogonia Mixed cells Spermatocytes Sperms 68.ADH is inhibited by Alcohol Angiotensin Insulin Renin 67.vagus and azygous vn Azygous vn Esophagus Phrenic nerve 63. The mc burney point indicates the presence of tip of appendix.Cholestatic cholangitis is caused by Colonercis selecans diphyllobothrum latum Schistosoma Entameoba 71.Following is true about appendix Appendix ha incomplete muscular coat Appendix not attached to mesentry Supplied by ceoliac artery 73.One of the following is not a branch of External carotid artery Lingual A Ophtalmic A Occipital A Maxillary A Suprficial temporal A 74.Gfr Rpf Creatinine clearance Glucose claerance 69.Femoral artery can be palpated in .After appendicectomy what may be damaged Inguinal ligament Superior epigastric artery Illhypogastric nerve Spermatic cord 72.A boy is malnutrioned and has edema due to protien loss his Plasma colloid pressure is decreased Increase hydrostatic pressure Blocked lymphatics 70. Mid inguinal lateral to pubic tubercle Mid point of inguinal ligament Adductor canal 75.Patient wirh Gravida 4 Par4 Abortio 0 with anemia and occult blood in stool Fe deficiency anemia Pernicious anemia Anemia of chronic disease Sideroblastic anemia 79.Medial side of hand is supplied by Median nerve Ulnrr nerver Radial nerve Axillary nerve 77.Apex of the femoral triangle Lies under inguinal ligament Contains small saphenous vein Contains nodes Is pointed downwards 76.Vomiting center is persent in Hypothalamus Pitutary Pons Medulla Midbrain 78.Injury to the neck and head of fibula causes damage to Sural nerve Sciatic nerve Tibial nerve Common peroneal nerve . Essential fatty acid is Linoleic Palmitic Citric Oxaloacetic 85.On bronchoscopy bronchoscope will 1st enter Sup bronchus Apical bronchus Inf bronchus Middle apical bronchus 84.80.Artery to head of femur is Medial femoral circumflex a Lateral femoral circumflex a Obturator a Femoral a 82.In tissue bleeding occurs because of Loss of coagulation factors Congenital disorders Endothelial damage dec FDF .Nerve to ligamentum teres capitis is Femoral n Obturator n Medial femoral n Lat circumflex femoral n 81.Muscranic receptors are present in Postgaglionic parasympathetic Postgaglionic sympathetic Preganglionic parasympathetic Preganglionic sympathetic 83. Following is not a feature of DIC Dec FDF Dec BT Dec PT Dec thrombin time Dec platelets 87.86.Thrombosis is initiated by Leukotrienes Prostacyclin Arachidonic metabolites Thromboxane A2 90. End artery Spleen Bone Brain .Most drug metabolism occur in Kidney Liver Spleen Intestine 9 1 .Direct inguinal hernia lies Lateral to inf epigastric a Medial ti sup epigastric a medial to inf epigastric a Medial to pubic tubercle 89.Role of middle ear bones is Transmit sound Amplify sound Does not respond to low noise Are sesamoid bones 88. left umbilical vein will regress t o : A. Sphincter pupili muscle. Which of the following structure is not an Embryological remnant: A. 4. 6-10 weeks. 2. Derivative of Ectoderm include: A. 5. Ligamentum Teres. 1. Which of the following is a derivative of neuro ectoderm: A.EMBYROLOGY. 9. Anatomical closure of Ductus venosus. Lateral umbilical ligament. Embryonic period is: A. 3-8 weeks. 8. 6. Secretary Epithelium of parotid gland. 10. Regarding Allantois: …… 7. Regarding Mesothelium: A. Decidua basilis forms the maternal part of placenta. Regarding Decidua: A. Lines the body cavities. 1 1 . 3. What will happen immediately after birth: Anatomical closure of Ductus arteriosus. Physiological hernia occurs in between: A. . Secretary Epithelium of parotid gland. Derivative of Ectoderm include: A. What is inappropriate regarding Glossopharyngeal nerve: A. It is related to the auditory pathway.Anatomical closure of foramen ovale. Styloid process. Which of the statement regarding Lateral Horn of spinal cord is inappropriate: A. L1-2. Obliteration of Left umbilical vein. It is entirely sensory. Injury to L1 vertebra will directly damage which of the following spinal structure: A. 2. 6. Primordial Germ cells are derived from: A. Anterior Hypothalamic Nucleus. It is present at the cervical portion of spinal cord. Yolk sac endoderm. Occipital Somites. In adults spinal cord ends at the intervertebral disc between: A. 13. In spinal tap the needle which will go across: A. 8. NEUROANATOMY 1. Obliteration of Right umbilical vein. Corticospinal Tract. Which of the following bone is derived from 2nd Pharyngeal arch: A. Regarding Trapezoid Body: A. 7. 4. 14. Loss of temperature regulation is due to damage at: A. Hypothalamus. Epidural Space è Dura Matter è Sudural Space è Arachnoid Matter è Subarachanoid space. Temperature regulation centre is located i n : A. 3. 5. 12. Conus Medullaris. Muscles innervated by hypoglossal nerves are derived from: A. 9. Loss of Fine and skilled movements of the hand is due to damage of: A. . Maintains the integrity of the cell. …………… 6. Compact bone contains: A. Sympathetic stimulation 5. 2. Straitifed Squamous Keratinizing Epithelium . PHYSIOLOGY 1. Ventricular Depolarization on ECG strip is represented b y : A. Large n o : of Elastin fibers. Bain Bridge reflux. Newly formed Elastic cartilage looks yellow & dense because of: A. Simple columner epithelium is present: A. Regarding spinal cord what is inappropriate: A. 2. IPSP is generated by opening of: . Multiple nuclei located at their periphery. Mast cell. Blood flow to the left ventricles is increase b y : Acetylcholine infusion. 8. Indirectly measures left atrial pressure. 6. Ventricular preload is measured by: A. LVEDV. Choroidal plexus of ventricles. 3. Skeletal muscles have: A. 4. Its dura matter has two layers HISTOLOGY 1. Cytoskeleton: A. 4.10. Osteoblast cells in the lacuner spaces. Nasal cavity. 3. Which part of the respiratory tract contains mucous glands: A. ………. Pulmonary Wedge Pressure: A. 7. Surfaces which are prone to great amount of friction have: A. QRS Complex. 5. Cl. 12. Renal absorption of glucose thru secondary transport with sodium occurs at: A. 16. ESR will increase with the decrease i n : A.A. 15. 14. Magnetic field. Proximal Tubules. 10. Inhibitory factor released by hypothalamus against which of the following hormone: Prolactin Growth hormone. 1 1 . Will not found in any other tissue than blood. During Depolarization: A. 12. Simple Diffusion depends on all of the following factors except: A. Bile of the Liver differs from the GB bile because GB bile contains . What is the suitable I/V fluid for the patient of acidurea: Normal Saline. B. If father’s blood group is B+ve & mother’s blood group is AB+ve there child can not have which of the following blood group: A. 7. 13. O+ve. Which of the following hormone is called STRESS HORMONE: A. Ringer lactate. 17. Blood group antigen: A. Which of the following causes Hyperkelemia: Zollinger Ellison syndrome. Albumin. 8. Will be inherited by Autosomal recessive pattern 9. Pyloric stenosis. There is rapid influx of Na+. Conn`s syndrome. Dorrow`s solution. Distal Tubules. Cortisol. Cushing syndrome. ADH will act o n : A.channels. 5% dextrose 1 0 % dextrose. Cortisol decreases which of the following cell: A. Which of the following is the NOT non dividing cell: A. Cardiac muscles are prevented by tetanization due to its: Rythmicity Automaticity Conductivity Long refractory period. Regarding Bradykinin: It formation is activated by killkeran. 18. Resection of distal ileum will impair the absorption of: A. 27. Water. 30. Sustained rhythmic reflex tremors induce by sudden movements is: A. Skeletal muscle. Lymphocyte. T3. 3 1 . 23. 22. Increase peripheral resistance is due t o : A. Smooth muscle. Regarding Interferon: ………. Clonus. 19. 20. Type II pneumocytes. 25. Surfactant is released b y : A. 28. Heat loss mainly depends o n : Temperature of the surrounding. 26. 24.decrease amount of: A. Regarding conduction of visual pathways: . Most active form of thyroid hormone present in circulation is: A. Evaporation. 29. Transfer of Cl. Hepatocytes. Neuron. 2 1 . Bile Salts. Chloride shift means: A. Increase vasomotor tone.in erythrocytes in exchange of HCO3. Which of the following cell can never reproduce: Erythrocyte. 33. Sharp odors have the quality of water & lipid solubility. Affection of Nasal mucosa. Fluid accumulates in acute inflammation contains: A. 4. Fat Embolism. Rupture of the Lysosomal membrane. IT manifest within 12 hours. Fat necrosis occurs in: A. 9. After 2 days of accident she suddenly collapsed & died.………. Superimposed infection on necrosis is called: A. Irreversible cell injury starts with: A. Fever in inflammation is caused by: A. Dopamin. 2. 3. 10. A 25 year old lady with 14 weeks pregnancy had a road traffic accident brought to the emergency department with a large open wound on thigh and femur fracture. Only caused by trauma to the fat tissue. Proteins >3gm/dl. The most probable pathological process involve in her death: A. Leukocyte Adhesion. What is inappropriate about fat necrosis: A. . 5. 32. 6. Her mesenteric veins were patent. Regarding Fat Embolism: It is Fetal in > 8 0 % of cases. At laparoscopy most of the bowl loops were dark purple black.. IL1 & TNF α. 11. Which of the following is not a mediator of acute inflammation: A. GENERAL PATHOLOGY 1. ICAM & VCAM mediates: A. Regarding olfaction: A. The most probable underlying pathological process is: A. Gangrenous Necrosis. 7. Wet Gangrene. Acute pancreatitis. Unilateral Anosmia is due to: A. A 36 years old women presented with acute abdomen. 8. 16. Cystic hyperplastic endometrium 20. Congestive cardiac failure. . Erythroplakia. blind ended vagina with no uterus & ovary.12. Skeletal muscles. An 18 year old girl came to family doctor complaining of primary amenorrhea. RBC. the diagnosis of Tuberculosis in this patient is confirmed b y : A. Lungs. 17. A 16 years old boy deeply jaundiced presented with gum bleeding due to: A. A 30 year old women brought to emergency department with bleeding…. on examination she found to have well formed breasts. Which of the following is a benign tumor: A. B. 15. 13. Which of the following is a premalignant condition: A. A patient presented with enlarge lymph nodes. Vitamin K deficiency. 46 XY. Her probable Karyotype is: A.. 22. 2 1 . Dystrophic Calcification is seen in all of the following except: Malarial parasite. Leukoplakia. Vitamin A deficiency. Cardiac muscles. Warthin`s tumor. Which of the following is a premalignant condition: Condyloma Metaplasia Endocervix. 19. Presence of caseous necrosis in lymph nodes. Edema due to increase hydrostatic pressure is seen i n : A. 18. Metaplasia does not occur i n : Brain. 14. The primary source of Creatinin is: Liver. Biochemical changes 29. Infective Endocarditis. Intestinal metaplasia due to reflux esophagitis leads t o : A. ELISA. 28. Hepatic hemengioma is associated with: A. Typhoid carriers are: A. Hydatid Cyst. diagnosed as case of gas gangrene dies due t o : A. SPECIAL PATHOLOGY 1. Hepatocytes. 2. Adenocarcinoma. 24. 1 Year. 10 days. 4. Toxic shock 30. Over an unpredictable period of time. 27. 3 Months. Regarding autosomal recessive disorders: ……………… 25. In a patient there is atrophy of submandibular gland due to its duct obstruction. Vinyl Chloride. LASER act by: Cutting. The mechanism of injury of ionization radiation i s : A. Free radical formation. A patient admitted to the hosp. 3. Which of the following is the confirmatory test of AIDS: Western blot. 6 days. Which of the disease is common in I/V drug abusers: A. 23. Maximal tensile Strength of a wound is attained in. A. Atrophy of the gland is because of: A. . 26.Dead fetus. Usually asymptomatic. Which of the following are not non-dividing cells. Apoptosis. 1 1 . Carcinoma formation. Major complication of severe burn: Formation of granulation tissue. There is increase PO4 renal absorption. 13. The most probable diagnosis is: Lambert Eaten Syndrome. 8.5. Which of the following is peculiar for crohn`s disease is: A. Basal cell carcinoma. CEA. 15. In pulmonary embolism. Spleenectomy will help in treating which type of anemia: A. Hyperthyroidism. 12. 9. Achlasia is due t o : . Perianal Lesions. BP 120/80. lung is due t o : Parathyroid like protein Parathyroid hormone 17. 14. Immediate effect after injury to a vessel i s : Vasoconstriction 16. Resection of Anterior lobe of pituitary will lead t o : A. 6. Increase plasma volume. Left ventricular failure will lead t o : A. Hereditary Spherocytosis. A 25 year old lady complaining of palpitations. 7. 19. heat intolerance on examination her pulse is 112b/min. She is most probably a case of: A. respiratory failure is due t o : A. Mysthenia Gravis. Effects of hyper parathyroidism in Ca. 18. What is Inappropriate about hyper parathyroidism: A. Malignant Tumor that will not metastasize: A. Anemia of pregnancy is due t o : A. Increase pulmonary arteriolar Pressure. R/R 20 br/min. Embryological marker that reappears in circulation in Ca Colon is: A. 10. Ventilation / perfusion mismatch. A 6 year old boy is complaining of proximal muscular weakness was found to be Ca++ channel antibodies positive. Decrease glucocorticoids. In Hypothyroidism there is increase in: A. I/V nitroglycerin. Gout. After a traffic accident a lady is brought to a hosp the IMMEDIATE step you do: A. 4. C. tender & swollen. Metaclopromide. 2. gram stain = negative. In thyroid disease the antibodies are directed against: A. 3.A. By aspiration of ova of Entameoba histolitica. Which of the following is drug of choice for patient of status asthematicus: . 24. Ameobic infection reaches lungs via: Direct extension from liver. Absence of Myenteric plexus. The most likely diagnosis is: A. B. He has a low grade fever. Which of the following is suitable antihypertensive for asthma & IHD patient during surgery: I/V sodium nitroprusside. Septic arthritis. It relieves pain as well. Pseudo gout. Ketamin is used as anesthetic in repeated dressings of burn patient because: A.000/mm3 with 75% neutrophils. Via Portal vein. Clear airway. 21. Labetalol. By aspiration of trophozoits of Entameoba histolitica. A 50 years old man is awakened in the middle of the night with acute pain & tenderness of the right knee. 22. 5. Drug that decreases the tone of lower esophageal sphincter & increases gastric emptying: A. crystal analysis = negative birefringent. PHARMACOLOGY 1. Cholesterol 23. 20. His knee is hot. Which of the following blocks α and β receptors: A. Analysis of fluid from right knee shows: leukocytes 70. Thyroglobin. Heparin will inhibit: A. Clot propagation. At the time of induction of anesthesia. 7. Acid suppression is done by blocking: A. 9. Pethidine. Clot organization. During surgery antibiotics should be given a t : A. 6. Morphine. 8. Drug of choice for acute pancreatitis. Paracetamol . H2 Receptor. B.I/V aminophylin Oral steroid I/V salbutamol. Ulnar e. Musculocutaneous Q 3. Bicep femoris b. Radial c. Ventral primary rami . Upper border of Illiac crest crosses a. paper A is still not received PAPER B Q 1. Sartoriusl d.it will help us in our board exam. A person is unable to flex his thigh and knee a. Visceral peritoneum! b. Semitendinious c. Fracture of surgical neck of humerus injured which nerve a. Median 1 b. Body of L4l Q 5. Abdominal organs are lined by a. Ulnar Q 2. Muscle of the back are supplied by a.medicine 24-Mar-12 4:32:00 PM FCPS PART 1 JUNE 2010 MEDICINE PAPER B • by DR RAKHSHANI » Wed Jul 07. Ant sup iliac spine b. likely lesion is a. Parietal peritoneum c. Median d. A person undergoing an 8 hour long explorative laprotomy now can not oppose thumb with index finger. Rectus femoris Hidden content: UNLOCKED Q 4. 2010 7:39 am HI Friends i m posting the mcqs. Auxillary! b. Omentum Q 6. In Asthma there is a. Which of the following is not true regarding Hemi section of the spinal cord? a. S1 Q 8. Loss of joint position sense on same side Q 9. FEV1 = FVC b. What initial investigation would u like to do a. Lateral 1 c. Most common feature of atypical pneumonia is a. A plumber is having chronic cough. Consolidation Q 13. ECG c.b. Knee joint is supplied by a. L5. Loss of pain and temp sense on same side! b. Posterior Q 10. A known smoker is having sudden onset chest pain and dyspnea . L2. Spinothalamic tract is a. L4. Dry cough] c. ABG’s Q 12.3. A young boy was climbing stairs experiences chest pain and dyspnea. L4l c. FEV1 < 7 5 % OF FVCl Q 14. Ipsilateral Babinski sign c. Dorsal primary rami of spinal nerve] Q 7. V / Q mismatch] Q 11. What is the mechanism of dyspnea a. HCO3 18 meq/L a. Echo d. Low pO2 b. L5 d. L3 b. CXRl b. o/e breath sounds are absent and percussion is hyperresonant. Compensated Respiratory Alkalosis . his PH is 7. PCO2 30mmHg. L3. Fever b. Anterion b. Combine d. A patient GFR drops from 100 to 20. 2 c.5 b. Venous volume c. PO2 c. Compensated Metabolic Acidosis c. 20 Q 19. Migraine c. . Deviation of uvela to left lesion will be at a. Cerebellar Hemingioblastoma] b. Etc 9/27/08Ashfaq Q 18. In polycythemia which parameter is increased a. The best index of Afterload is a. his s. Right Vagus] b. Mean Arterial Pressure] b. Cardiac output Q20. Cardiac output Q 2 1 .b. Which of the following tumor is associated with amplification . Thyrotoxicosis Q 23. Total Peripheral Resistance is determined by a. 5] d. Caliber of Arteriole] b. WBC count Q 22. 10 e. Blood pressure c. Polycythemia is associated with a. Which one is wrong about ECG a. MI d. Uncompensated respiratory Alkalosis] Q 15. P wave precede pacemaker potential] b. Q 16. Question about respiratory failure types associated with chronic bronchitis. creatinine will rise from 1 to a. Right hypoglossal Q 17. Blood Pressure] b. Burkitt’s lymphoma c. A ML I b.of oncogenes a. Schwanomma Q 24. Endometrial carcinoma b. Infectious mononucleosis! b. Ovarian carcinoma Q 30. her FBC shows 8 5 % lymphocytosis. Which one is not a feature of cirrhosis? a. The causative agent of lymphoma in AIDS is a. investigation shows hypocalcemia and low vit D. A 25 year old female with chronic diarrhea and weakness. the diagnosis is a. Cervical carcinoma Q 29. Typhoid Fever Q 27. Ascities . Giardiasis! b. Kaposi’s sarcoma d. Celiac Disease! Q 26. HSV-2 is a risk factor for a. CMV Q 28. Portal hypertension d. Acute piecemeal necrosis! b. Q 26. Cervical carcinoma! c. EBV! c. Esophageal motility is affected in which disease? a. HSV b. Of the following which one is not cause by a virus? a. Pertusis c. Regenerative nodules c. Neuroblastoma! c. the diagnosis is a. Retinoblastoma d. Scleroderma! Q 25. Question abt Hashimoto’s thyroditis ie autoimmune. SLE b. A young child with sore throat and fever. Nephroblastoma b. Q 3 1 . Lymphocyte d. IL-2 b. Macrophage! c. Monocyte Q35. Basophill e. b. Ig A deficiency C1 esterase inhibitor deficiency complement deficiency! CD4 helper deficiency Q34. IL-4 c. SVC and right atrium! . On X-Ray right heart border is made by a. Neutrophill b. IL-12! Q36. Most common cell of chronic inflammation? a. A 30 year old lady who is 34 weeks pregnant is having fits and raised BP with proteinuria. IV Diazepam] b. aPTT b. Bordetela pertussis! c. IL-10 e. Pseudomonas! Q37. Clotting time Q32. Bleeding time d. cutaneas vasculitis and arthritis will show? a. Vibrio cholera b. MgSO4 9/27/08Ashfaq Q33. c. PTl c. Which gram -ve bacteria do not produce endotoxin? a. A 23 yr old male with malar rash. Which chemical mediator is involved in cell mediated immune response? a. d. A patient on oral anticoagulant which parameter should be monitored? a. IL-6 d. What is the most appropriate initial management? a. SVC. Neonatal Rubella infection is screened by measuring which antibody? a. Lyshnyan syndrome Q39. Q47. LHRH b. Somatostain d. Pancreatodudenal Artery. IVC and right atrium Q38. Insisura angularis] Q42. Gastrodudenal Artery] b.DQ. Meningococal vaccine! c. Serotonin . Which of the following vaccine has got the protein component to enhance immune response? a. SCIDI b. In peptic ulcer disease bleeding commonly occur from which artery? a. Which of the following is not true regarding Immunoglobulin? a. Hematobium? Q41. IgMl b. A question about HLA-II . teania solium] b. Polio b. right atrium and IVC c.DR. Light and Heavy chain have the same domain Q46.b. Adenosine Deaminase deficiency is seen in which patients? a. Q43. IgG c. teania echinococccus Q40. igM and IgG Q45. Vasopressin e. Which neurotransmitter is not involved in memory? a.cause can b a. A question regarding S. TRH c. DP. A notch between lesser curvature of stomach and pylorus is called? a. Rabies Q44. A Patient presented wid cysts in liver. Q48. A question about alzhiemer disease whether it affect short or long term memory. Q49. Genes are made of a. b. c. d. Intron Exon Ds DNA] RNA e. Ribosome 9/27/08Ashfaq Q50. Thiazide diuretic causes a. Hypocalcemia 1 Q51. Thiazide diuretic causes a. Hypokalemia! Q52. Antidote of morphine poisoning is a. Nalaxonel Q53. Pt. on MAO inhibitor is given 1st dose of barbiturate will develop a. comal b. Insomnia Q54. In acute heart failure the drug of initial choice is a. I / V frucemidel Q55. A patient with carpopedal spasm following throidectomy, the drug of initial choice is a. I / V Ca gluconate.! Q56. A patient on MAO inhibitor is given 1st dose of barbiturate, he will develop a. Coma ! b. Insomnia Q57. A 20 yr old girl with type 1 diabetes is brought to emergency in semicomatose condition. An injection of insulin given to her will raise her a. b. c. d. Her blood PH I Blood sugar K+ level Urinary excretion of ketones Q58. Fine tremors are associated with which drug? a. Alcohol b. Terbutalinel c. Theophyline Q59. Which drug is teratogenic? a. Alcohol! b. c. d. e. Coffee Heroin Phenothiazine Tobacco Q60. Most common Souce of pulmonary emboli a. femoral vein] b. popliteal vein] c. periprostatic veins d. periovarian veins e. superficial saphenous vein Q61. The normal esophagus is a. 10 inches in length] b. lies entirely in the thoracic cavity c. consists entirely of skeletal muscle d. lined entirely by stratified squamous epithelium.] Q62. A child sufferd from repeated episodes of mild jaundice. Diagnosis after his labs showed unconjugated hyperbilirubinemia 1.Dubin johnson syndrome 2. Bile duct obstruction 3. Gilbert syndrome] 4. Rotor syndrome Q63. A pt z having anosmia and hypogonadism, CT will show lesion of which area? a. Amygdala b. Arcuate] c. preoptic d. paraventricular e. supraoptic. Q64. Menopause is assosciated with 1. Low estrogen and high fsh and high lh| 2. Low estrogen and high fsh and low lh 3. High estrogen and low fsh and lh 4. Low estrogen and normal fsh and lh 9/27/08Ashfaq Q65. A patient is having oligospermia, which of the following hormone is likely to be low? a. FSH] b. LH c. Estrogen Q66. Which one of the following is not a vasodilator? a. PGI2 b. Endothelinl c. NO Q67. A question abt Androgen. Q68. Which drug is given to close ductus arteriosus if it does not closes spontaneously? a. Indomethacinel b. Paracetamol Q69. A normal umbilical cord contain a. Two artries and left umbilical vein] b. Two artries and right umbilical vein Q70. A pregnant lady is likely to have risen following value a. TIBCl b. Serum iron c. Serum ferritin Q71. A patient on broad spectrum antibiotic starts to get bleeding, likely cause is a. Vitamin k deficiency Q72. The posterior relation of left kidney include following except a. Quadrates lamborum b. Psoas c. Diaphragm d. Splenic flexure! Q73. A patient with swallowing difficulty and bird beak appearance on Ba meal is suggestive of a. Achlasial b. Esophageal carcinoma Q74. A question about anatomy of spinal arteries. Q75. A question about allergic urticaria. Q76. Which anti tubercular drug is associated with Hyperuricemia. a. Isoniazid b. Rifampicin c. Pyrazinamidel Q77. A patient on corticosteroid will show a decline in which cell line a. Neutrophils b. Monocyte c. Lymphocyte! Q78. A question about long acting thyroid stimulating antibody (LATS) Q79. Which property of motion is affected in cerebellar lesion? a. Range of moton Q80. Which lobe of cerebellum is concerned with Balance and eye movement? a. Vermis b. Flocunodular lobe] c. Lateral hemisphere Q81. Pubic symphasis is an example of which type of joint? a. Fibrous b. Fibrocartillagenousl c. Synovial Q82. Tissue bleed is a feature of a. Collagern defect! b. Clotting problem Q83. Which feature differentiates malignant from benign? a. Metastasis! b. Recurrence c. High vascularity 9/27/08Ashfaq Q84. A feature of malignancy include a. Invasiveness! b. Hardness c. Vascularity Q85. Blast injury is associated with deafness of which type a. Sensory b. Neural! c. Combine d. Toxic Q86. In neural pathway unilateral lesion above which nucleus cause bilateral deafness? a. Lateral leminisci b. Inferior colliculi c. Cochlear nucleus! d. Medial geniculate body I am not giving this answer let see can you do it. Q87. Which enzyme is measured to differentiate raised alkaline phosphate whether of hepatic or bone in origin? a. ALT b. AST c. Gama GT.l Q88. A hypertensive patient is having raised renin activity in left renal vein and in plasma, and a decreased renin activity in right renal vein, he is having a. Right renal artery stenosis b. Left renal artery stenosis] c. Bilateral stenosis Q89. A child aspirated a peanut. In which lobar bronchus it must have logged 1. Left upper 2. Right inferior] 3. Right middle 4. Right superior 5. Left inferior Q90. Drug of choice for sub-arachnoid Hemorrhage a. Nifedipine b. Nimodipinel c. Verpamil d. Amlodipine e. Diltiazem Q91. Recketsia infect which cells a. Epithelial b. Endothelial] c. Fat cell d. Schwann cell e. Macrophage f. Muscle cell Q92. Which one is anaphylatoxin a. C3b b. C5b c. C3al Q93. Normal quiet expiration is brought about by contraction/recoil of 1. Diaphragm] 2. Elastic tissue in thoracic and lung wall 3. Abdominal muscles 4. Sternocleidomastoid Q94. A child died of severe pneumonia. At autopsy, the lung shoed Warthin- fnkendly cells with many nuclei. Causative organism: 1. Ebola virus 2. CMV 3. Measles virus] 4. Rubella virus 5 pneumocystis carinii Q95. The following disease most severely affects the esophageal phase of swallowing 1. Myasthenia gravis 2. Scleroderma! 3. Poliomyelitis 4. Stroke 9/27/08Ashfaq Q96. Pt presented in emergency in unconscious state report indicates hematoma n rite parietal region, cause z 1. Internal carotid artery 2. Basilar artery 3. Middle cerebral artery 4. Middle meningeal artery! Q97. A young patient with hypertension and hematuria, investigation shows anti basement membrane antibodies, the likely diagnosis is a. Good pastuer syndrome] b. SLE c. Membranous Gn 9/27/08Ashfaq Answer the ans with blocks mean correct option. MEDICINE JUNE 2009 1. H+ ion secretion in the kidney causes bicarbonate reabsobtion bicarbonate excretion 2. which is pivot joint temporomandicular atlanto occpital Atlanto axial 3. Drug used for mountain sickness acetazolamide scopolamine 4.Corneal opacities r caused by ethambutol phenothiazides 5. Captopril causes hypokalemia hyper kalemia 6. Ph =7.5 HCO3= 48 PCO2= 45 diagnosis is metabolic alkalosis respiratory alkalosis partially compensated metabolic alkalosis partially compensated respiratory alkalosis 7. Which is P450 inducer isoniazid phenobarbitone cometidine ketoconazole 8. Which causes decreased gastric motility Magnesium Sulphate Aluminium Hydroxide 9. Serum gastrin levels are incresed by prolonged use of antacids PPI H2 blockers anticholinergics beta blockers 10.Esophago gastric junction competence is maintained by lying supine incresed intra abdominal pressure diaphragm paralysis use of metoclopramadie use of morphine 1 1 . Patient has acute blood loss the mechanism triggered imediately is baro receptor mechanism renin angiotensin system 12. Carotid body receptors respond to arterial PO2 arterail Pco2 13. propylthyouracil acts by decreasing iodine uptake decreasing TSH levels decreasing thyro globulin levels 14. which is increased in first responce Ig G Ig M Ig A 15. which crosses placenta Ig G Ig M Ig A Ig E 16. which is secreted in breast milk Ig G Ig A Ig M 17. which has memory function Ig G Ig M Ig A Ig E 18. which drug causes abnormal differentiation between red and green colours ethambutol amiodarone pyrazinamide 19. for best verbal communication skill one should hav good vacabulary competence in presentation fluent speech 20. which is not caused by parasympathelic stimulation increased GIT motility micturation miosis sweating 2 1 . ejaculation is caused by sympathetic parasypathetic both 22. Alpha receptors stimulation causes pupilaary dilation increased heart rate increased contractility 23. fracture of surgical neck of humerus, patinet cant raise arm and sensory loss on lateral surface of arm. damage to axillary nerve musculo cutaneous nerve 24. action of gluteus medius n minimus is abduction n medial rotation abduction n lateral rotation adduction n medial rotation adduction n lateral rotation 25. thymus is formed by 3rd pharangeal pouch 4th pharangeal pouch 26. bromocriptine causes decresed prolaction levels by acting on dopamine receptors cholinergic receptors 27. patients ecg shows increasing PR intervals and then a missed beat first degree block sinus arrythmia Mobitz type 1 block Mobitz type 2 block 3rd degree block 28. Digoxin is drug of choice in Atrial flutter Atrial Fibrillation Ventricular Tachycardia Bradycardia 29. patient after ischemic attack has ventricular tachycardia. drug that must b used is lidocaine amiodarone verapamil 30. apex beat is 4th left intercostal space mid clavicular line 8cm to the left of midline in 5th intercostal space 3 1 . young patient has bruises on the body and no other significant history n examination unremarkable. he has idiopathic thrombocytopenic purpura aplastic anemia Hemophilia 32. Hemophilia is X linked recessive Autosomal dominant Autosomal recessive 33. which investigation to be done in patient with hemophilia PT APTT bleeding time complete blood count 34. patient has hemoptysis and glomerulonephritis. diagnosis is good pasture syndrome wegeners granulomatosis 35. P falciparum causes black water fever 36. C perferingens acts by producine lecithinase depleting ATP causing hypoxia 37. Gonococcus in easily identified in exudate specimen by gram staining Z N staining culture 38. which is not oncogenic Hep B virus Hep D virus Herpes Simplex virus EBV 39. Middle meingeal artery passes through foraman spinosum foraman lacerum foraman rotundum foraman ovale 40. damage to middle meningeal artery causes hematoma formation between duramater n calvaria duramater n arachnoid mater arachnoid mater n piamater 50. which is present in cavernus sinus abducent nerve 5 1 . which cranial nerves r parasympathetic I I I , VII, IX, X 52. patients right eye has moved upward n has his neck tilted to left to avoid diplopia. damage to superior oblique inferiour oblique 53. patient has homonymous hemianopia. lesion is at optic nerve optic tract retina optic chiasma 54. germ cells r derived from ectoderm endoderm mesoderm 55. foreign body in trachea goes in which bronchus right upper right middle right inferior left inferior 56. which lobe has only 2 brocho pulmonary segments right middle right upper left upper left middle 57. during iso volumetric contraction atrial pressure decreases semilunar valves r open coronary blood flow decreses first heart sound is produced second heart sound is produced 58. which is thick filament actin myosin 59. a sacromere is present between 2 Z lines H band A band 60. in muscle cells calcium is released from endoplasmic reticulum sarco plasmic reticulum 6 1 . in kidney select the best order of arteries renal > arcuate>interlobar>interlobular>efferent arteriole renal>interlobular>interlobar>arcuate>efferent arteriole renal>interlobar>interlobular>arcuate>afferent arteriole renal>arcuate>interlobular>interlobar>afferent arteriole 62.after normal saline infusion blood volume increases Na excretion decreases osmolality increases renin secretion increases 63. after renal transplant which malignancy is common ??? 64 babinski sign is positive in pyramidal lesion cerebellar lesion 65. ventral spino thalamic tract ends at lumbar region mid thoracic end thoracic 66. commonest site of lumbar puncture is L4-L5 L2-L3 S1S2 67. which is not punctured during lumbar puncture duramater ligamentum flavum longitudinal spinal ligament 68. after Rh incompatabilty what is given to the mother to prevent it next time anti D antibodies 69. patient has blood group A+ which cannot b given to him O+ O-A-AB-70. hamartoma is benign tumor malignat tumor 7 1 . medullary rays r present is cortex medulla 72. in Addisons disease what does not occur hyper kalemia hypokalemia 73. which is essential amino acid phenyal alanine 74. which is teratogenic alcohol 75. which is associated with cancer methyl alcohol propyl alcohol 76. HLA- DR4 is associated with rheumatoid arthritis SLE scleroderma 77. which is present in SLE anti DNA antibodies anti jones antibodies anti smith antibodies 78. which causes dysphagia ankylosing spondylitis scleroderma dermatomyositis 79. iron deficincy anemia is hypochromic microcytic 80. patients had gastrectomy , after 3 months she presents with iron deficiciency anemia pernicious anemia hemolytic anemia 8 1 . pregnant lady has MCV of 70 f l . and Hb 8.1 g/dl, she is suffering from iron deficiency anemia megaloblastic anemia 82. which is tributary of portal vein superior rectal inferior rectal 83. what arches in front of the root of left lung Arch of aorta Azygus vein 84. selective beta blockers do not cause broncho spasm 85. how will the effect of warfarin immediatelty reversed vitamin K FFP protamine sulphate 86. RCA supplies right atrium only right atrium n right ventricle 87. tunica vaginalis is derived from transversalis muscle transversalis fascia peritoneum inter oblique muscle 88. turner syndrome has karyotype 45 XO 45 XX 45 XXy 45 XXX 89. which muscles divied submandicular gland in to super ficial n deep part omohyoid mylohyoid anterior belly of digastric 90. patient has difficulty closin right eye and distorted facial appearance. he has damage to right fascial nerve left fascial nerve right trigeminal nerve left trigeminal nerve 9 1 . which does not cause thenar muscle wasting carpel tunnel syndrome C8 neuritis cervical rib scalene muscle spasm cervical spondylosis 92. a pregnat lady from hilly area presents to. fetal scan at term shows no anomaly. which of the following can still b present in the newborn ASD VSD PDA pulmonary stenosis 93. what is end product of glucose metabolism in the presence of oxygen lactic acid 1 molecule of pyruvate 2 molecule of pyruvate 94. beetle chewing causes submucuous fibrosis leukoplakia keratosis 95. which is last mediator in septic shock IL-1 IL- 6 96. antigen antibody reactions r most reduced in Liver failure Low neutrophils 97. Clavulanic acid destroys B lactamases causes decreased penicillin excretion 98. Dose of Gentamicin is reduced in elderly due to reduced renal function 99. GFR can b best estimated by creatinine clearance serum urea levels serum creatinine levels 100. best test for diabetic nephropathy is serum urea levels serum creatinine levels urinary albumin 1 0 1 . which protein maintains red cell shape integrin secretin spectrin 102. Cyanosis is due to increased deoxy hemoglobin in blood decresed PO2 in blood. 103 which of the following causes natriuresis stretch receptors in atria carotid bodies 104. patient has gall stones , which enzyme could b most likely raised Alkaline Phosphatase LDH ALT 105. diffusion through alveoli is directly proportion to thickness of membrane total cross sectional area 106. transpot of materials by carrier proteins is example of simple diffusion facilitated diffusion active transport 107. which does not has lymphoid follicles speen thymus lymphnodes payer patches 108. in congestive cardiac failure there is incresed pressure in venous system 109. ventricular contraction causes which wave in JVP a cv 110. site for venesection great sephanous short sephanous 1 1 1 . which is not part of limbic system vermis of cerebellum amygdala anterior thalamus hippo campus 112. right gastric artery is branch of gastro duadenal right gastro epiploic 113. thiazide diuretics donot require potassium supplemets 114. behaviour sciences is branch of science which deals with behaviour of persons in different social , psychological conditions 115. pateint has loss of proprioception on right n loss of pain n temperature on left, lesion is right hemisection of spinal cord left hemisection of spinal cord 116. cerebral cortex is concerned with voluntary movements of body 117. DVT is causes by stasis of blood 118. lacrimal duct opens in inferior meatus middle meatus 119. in asthma FEV1/FEV less than 65% FVC is reduced 120 6 years old girl has meningitis. most common organis is streptocossus pneomonie Neiserria meningitidis Hemophilus Influenzae E Coli 1 2 1 . exudate has protein more than 3g/dl specific gravity is less than 1.010 122. a young boy has distorted nose and cervical lymphadenopathy. lymph node biopsy shows non caseating granuloma and sputum AFB is positive, diagnosis is TB leprosy sarcoidosis syphilus 123. Lambda chain is present in amyloidosis 124. which does not follow oro fecal route tenia solium trichuris trichuria entrobius vermicularis Ankylostroma duodenale 125. cholangio carcinoma is caused by C Sinensis paragonimus watermani taenia solium 126. young boy has bilaterla parotid enlargement. diagnosis bacterial parotitis mumps infectious mononuclosus 127.about Mycobacterium TB , which is false causes disease only in humans favors aerobic conditions 128. which is false about spores produced under nutrient rich conditions B Anthracis producs spores C tetani produes spores they r killed under 121 degree for 15 min in autoclave 129. fetal brain development is caused by Growth hormone thyroid hormone 130. gastric Acid increases most after taking carbohydrate protein fats 1 3 1 . which hormone effects carbohydrate, protein and fat metabolism insulin Growth Hormone 132. which is not an epithelial tumor sq cell CA adeno carcinoma sarcoma 133. Bence jones protein are produced in multipla myeloma 134. which hormone effects both nor epinephrine n serotonin metabolism Mono amine oxidase 135. patient presents with epistaxis and bruising on body. best invertigation is complete blood count PT Aptt 136. ADH acts on Proximal tubule Distal Tubule Collecting Tubule 137. which is most slow growing malignancy of thyroid Follicular CA Medullary Papillary Anaplastic 138. Cerebellar lesion causes Ataxia 139. patient with cerebellar lesion cannot touch objects with his finger tip 140. PTH remains normal in renal insufficiency osteoporosis 1 4 1 . Neonatal Rubella infection is screened by measuring which antibody? IgM IgG IgM and IgG also 6 -7 questions about aldosterone and blood pressure control MEDICINE JANUARY 2009 By 'Crack Smart'. Medicine paper (Jan 2009) Q#1.a 45 yr old male comes with the complaint of acute abdominal pain with migratory thrombophilibitis. what is the diagnosis? a. Acute pancreatitis b. Chronic relapsing pancreatitis c. C.a pancrease (ans) d. D.M Q#2.Iron is stored in parenchyma as: a. Ferritin b. Hemosiderrin c. Transferrin Q # 3 . Least site of metastasis i s : a. Spleen b. Liver c. Lung d. Kidney e. Lymph nodes Q#4.Major stress hormone is: a. ACTH b. Epinephrine c. Norepinephrine Q#5. Benedict test is used for: a. renal glycosuria b. urinary ketone. c. urine glucose test Q#6.A lady with hepatitis B; Serology shows: HbsAg +, HbeAg +ve,Anti HBe -ve, Hbv Dna +ve. Anti HBc IgM +ve...what does it show? a. Acute Hepatitis B (ans) b. Chronic hepatitis B c. Viral load is decreasing d. Immunized Q#7.A healthy baby comes apparently with the complaint that he does not take breast milk instead he prefers the bottle feed. What is the diagnosis? a. Cleft hard and soft palate (ans; most probably) b. Cleft soft palate c. Hormonal deficiency d. Nasolacrimal duct block e. Laryngeal weakness Q#8.Penetrating injury in left 4th ics will damage which of the following structure? a. Intercostal membrane b. Intercostal muscle c. left lung d. left ventricle e. Left atrium Q#9.In Hyperglycemia, which 1 will increase: a. Epinephrine b. Cortisol c. Glucagon It is autosomal recessive c. Insulin (ans) Q#10.Non-myelinated nociceptor fibre directly synapses with: a. Only females suffer e. Hemolytic Anemia e. Genetic mutation d. Macrocytic Anemia Q#12. Only male suffer Q#11. It is autosomal dominant b. Microcytic hyperchromic anemia c. Dorsal spinothalamic tract sorry cant remember other options . Pt will develop: a.d.Regarding polycystic kidney: a.After 3 months to gastrectomy. Microcytic hypochromic anemia b. Megaloblastic anemia (ans) d. Failure of closure of foramen ovale c. Mesoderm Q#14.Q#13.Somites are derived from a. PDA (ans) Q#16. Helicobacter pylori Q#15. Mast cell . Coarctation of aorta b.Which is the major circulating phagocyte? a.A 2 yr old child (premature). Neutrophil b.Pseudomembranous colitis is caused by a.perferinges c. Clostridium difficile (ans) b. C. which cardiovascular anomaly will most likely develop: a. ileum . Has increased K+ (ans) c. Glycine b. Macrophage (ans) Q#17.Bile salts are normally absorbed from a. Ach d. Dopamine c. GABA Q#19. Duodenum b.Neurotransmitter of corticostriatal fibres is a. Jejunum c. Has increased Ca+ Q#18.ICF differs from ECF i n : a. Monocyte d. Has increased Na+ b.c. Stomach Q20. S1Q3T3: cant remember options for this Qz Q#21. Mitochondria Q#23. Nalaxone (ans) Q#22. R. Is type 1 reaction c.Antidote for Morphine: a.E. Microfilaments e. Cause inactivation of drug b.E. S.Glucronide conjugation of drugs a.R (ans) b. Makes drug insoluble .R c.d. Microtubules d.which of the following is absent in Axon hillock a. MACula densa b. Extensors of limbs c. Abductor pollices b.C6 Q#25.Q#23.Which part of kidney produces Erythropoiten: a. Pct d. One large cell with multiple nuclei arranged close to membrane .Liver biopsy from a pt shows a lesion which is comprised of central collection of structureless material surrounded by epitheloid cells and lymphocytes.Loop of henle Q#26. Extensors of trunk d. Stabilizers the shoulder Q#24. pertubular capillary (ans) c.Upper end of esophagus is at the level of a. DCT e.Dorsal rami of cervical nerves innervate a. Seizure 29.Posterior belly of diagastric is supplied b y : .Side effects of thiopental a. Fatty change d.Tissue damage by Ionization Radiation is due t o : a. swelling of cell Q#28. metaplasia of cell e. Hydropic change e. Formation of free Radicles c. Damage to Golgi bodies b. Liquefective necrosis Q#27. Nausea b.The most likely change is: a. Coagulation Necrosis c. Caseous necrosis b. Hydropic change d.is also seen. IgM c. IgM b.a. IgG .Which immunoglobulin has pentameric structure: a. Glossopharngeal nerve b. IgE e. IgG Q#31. IgD e. IgE d.Which is highest in human milk? a. IgD d. Facial nerve c. IgA b. IgA c. Trigeminal Q#30. Hepatic c.Q#32. Posterior cerebral artery Q33. Splenic d. Celiac b. Thoracic duct Q34. Ant:Cerebral Artery b. Superior mesentric . Middle cerebral Artery c.Gastroduodenal artey arises from a. Esophagus b. Left main bronchus c.Blood supply of Visual area: a.Aneurysm of arch of aorta will most like compress a. Phrenic nerve d. Ulnar .Q35. HIV c. Anaplastic b. Follicular d. Radial d. which nerve is damaged? a. Melanosis Q37. HBV d. Iodine deficiency b. papillary Q36. Slow growing tumor of thyroid is: a. Median c. Axillary b.A 34yr old man can’t abduct his arm due to fracture of humerus. Medullary c.Which does not lead to Pre-malignancy a. Estrogen (ans) B.Which is not “ Major Basic tissue” . Increased Acidophilic cells (most probably diz was the ans) Q40. Progestron 41. Increased ACTH Q39.A 35 yr old female is protected against osteoporosis due to A. Histology of pitutary will show. Musculocutaneous Q38. Increased serum K+ b. a.A 14 yr Boy comes with the complaint of his abnormal Height and secretion from mammary glands. Increased serum Na+ c.e.Aldosterone will increase w i t h : a. Increased Chromophobes b. decreased plasma volium d. portal vein C} Hepatic vein. Bone d. portal vein B} Bile duct. Epithelium b.a. Blood e. cystic artery. Nerve (ans) c. Muscle Medicine March 2008 PREPARED BY: DR:MUHAMMAD SHEERAZ AKHTAR SOOMRO 1] Which one of the following is COX-II INHIBITOR? A} Aspirin B} Ibuprofen C} Indomethacin D} Meloxicam E} Piroxicam 2] Which one of the following vitamin deficiency causes HOMOCYSTENURIA? A} Vitamin B1 B} Vitamin B2 C} Vitamin B6 D} Vitamin B12 E} Biotin 3] Which one of the following structures is present at free edge of LESSER OMENTUM? A} Bile duct. portal vein . hepatic artery. hepatic artery. likely abnormality is in A} Glucose-6-Phosphate deficiency B} Immune hemolysis C} Paroxysmal Nocturnal Haemoglobinuria D} Malaria E} Sickle cell disease 5] Thiamine deficiency causes A} Peripheral Neuropathy B} Pellagra C} Chelosis D} Dermatitis 6] Milk is notoriously deficient in A} Vitamin C B} Iron C} Riboflavin D} Pantothenic acid E} Vitamin A 7] Dry heat kills the bacteria by A} Coagulation of proteins B} Cell lysis C} Free radical formation D} Direct killing E} Oxidation 8] Virus damage the cell by A} Making the pores in cell membrane B} Forming free radicals C} Altering the formation of proteins D} Nuclear damage E} using energy for their own metabolism . after few hours he developed red color urine.4] A 16 year old boy was given CO-TRIMAXAZOLE for fever. 9] Which one of the following lipoprotein has highest protein content? A} HDL B} LDL C} VLDL D } IDL E} CHYLMICRONS 10] All are related to PLATELET FUNCTION EXCEPT A} Capillary resilience test B} BT C} Clot retraction 11] ALLELLE is: A} Fragmented genes B} Non-identical genes C} Structural genes D} Regulatory gene E} Autosomal Dominant gene 12] D.C is stimulated b y : A} Factor X B} Factor XII C} Thromboplastin D} Factor IX E} Fibrinogen 13] In parenchymal cells Iron is deposited in the form of A} Tranferrin B} Ferritin C} Heamatin D} Heamoglobin E} Lipifuscin 14] a mother is having blood group A+(Ao. Which of the following blood group cannot be inherited by their children? . Dd). father is having blood group B+(Bo.I. DD). the IPSILATERAL sign will be A} Loss of pain B} Loss of temperature C} Loss of sensation of touch and vibration D} Lower motor neuron paralysis E} Upper motor neuron paralysis 16] Valve less vessel is A} Aorta B} Pulmonary artery C} Coronary Sinus D} Pulmonary Trunk E} SVC 17] A lady has undergone colostomy is now on the TPN. She is prone to develop: A} Hyperglycemia B} Hypocalcaemia C} Hypophosphatemia D} Hyperviscosity E} Hypoglycemia 18] Turner syndrome A} XXY B} 45XO C} 46XY D} XXX E} 69XX .A} A + B} B + C} AB + D} AB-E} O+ 15] The person has suffered the injury to spinal cord leading to HEMISECTION of the cord. 19] Regarding Von Willbrand disease A} Autosomal Dominant B} Autosomal Recessive C} Some family members have low level of Factor IX D} Some family members have abnormal level of Factor IX E} has positive family history 20] Which of the following condition is related to lymphatics? A} Angiosarcoma B} Rhabdomyosarcoma C} Cystic hygroma 21] The condition in which pulmonary trunk fails to divide leading to A} PDA B} Truncus arteriosus C} TOF 22] Basal Cell Carcinoma is located at A} Lower lip B} Upper lip C} Tongue D} Hard palate E} Soft palate 23] All are branches of Vagus Nerve except A} Auricular Nerve B} Lacrimal Nerve C} Recurrent Larengeal Nerve D} Internal Larengeal Nerve 24] Permanent Atrophy is caused by A} Loss of Innervation B} Hormonal Deficiency C} Immobilization D} Nutritional deficiency . E} blockage of arterial supply 25] Which of the following is not Carcinogen? A} Alcohol B} Benzathracine C} Cyclophosphamide D} Benzidine E} Dimethylsulfate 26] The Initial step in APOPTOSIS is A} Inhibition of P53 genes B} Activation of BCL-2 C} Activation of Caspases D} pores formed by lymphocytes 27] Which of the following is content of Cavernous Sinus? A} Oculomotor B} Trochelear C} Abducent D} Trigeminal (V3) 28] Which part of kidney produces ERYTROPOITEN A} Macula Densa B} Peritubular Capillary C} PCT D} DCT E} LOH 29] A patient with signs and symptoms of megaloblasstic anemia. The likely cause is A Decreased GIT motility B} Deficiency of intrinsic factor C} Malabsorption 30] Stimulation to increase Serum Calcium is A} Hypophosphatemia . B} Loss of subcutaneous fat.B} Paratharmone C} Decreased ECF 31] Malnutrition causes all of the following EXCEPT A} Pitting Edema.5 gm/dl. Physical examination: Pallor. Reticulocyte count: 0. Loss of touch sense in both feet and legs Hb: 7. C} Failure to thrive. The most likely causative parasite is A} Ankylostomata Duodenale B} Ascris Lumbricoidis C} Diphylobothrium Latum D} Strangyloids Stercoralis E} Taenia Saginata 34] A 40 year old male after road traffic accident had received massive blood transfusion. One large cell with multiple nuclei arranged close to membrane is also seen. His diet comprises of fish and rice only. D} Skin pigmentation E} Thrombocytopenia 32] Liver biopsy from a patient shows a lesion which comprises of central collection of structure less material surrounded by epitheliod cells and lymphocytes. He is expected to have all EXCEPT A} Hypocalcaemia B} Hypokalemia C} Hypothermia . The most likely change in central area would be: A} Caseous necrosis B} Coagulation necrosis C} Fatty changes D} Hydropic changes E} Liquefective necrosis 33] A 28 year fisherman presented with slowly developing lethargy. Stool DR: Ova of Intestinal parasite. Platelet count: 110 x 10 9/L. easy fatigue and palpitations.5%. His LFT shows Direct Billirubin 11 mg/100 m l . Alkaline Phosphate 2300 IU/L 500IU/L. SGPT 75U/L.D} Left shift of Oxyheamoglobin dissociation curve E} Metabolic acidosis 35] The endothelial molecules ICAM-1 & VCAM-1 are responsible for A} Direct endothelial injury B} Endothelial cell contraction C} Junctional retraction D} Leukocyte adhesion E} Leukocyte margination 36] Eating raw or under cooked meat can cause infestation by A} Dracunculous Medinensis B} Echinecoccus Granulosis C} Liver Fluke D} Schistosomiasis E} Tape Worm 37] A p t : has sustained burn that is very painful with blisters. Healing of this burn will take place from which of the following layers of the skin A} Basal layer of epidermis B} Keratinized layer C} Layer of upper dermis D} Lower dermis E} Subcutaneous layer 38] A 40 year old male presented with jaundice and generalized itching. The most likely cause is A} Drug induced jaundice B} Extra hepatic Cholestasis C} Autoimmune Hepatitis D} Alcoholic Hepatitis E} Intrahepatic Cholestasis 39] The tissue damage by Ionization Radiation is due to A} Damage to Golgi bodies . B} Formation of free radicals C} Hydropic degeneration D} Metaplasia of cell E} Swelling of cells 40] Formation of free radicals cause by A} Ionization Radiation 41] Edema of renal origin is most likely due to Albuminuria & sodium retention B} Decreased water excretion C} Hypertension D} Renal stone E} Sodium retention by the kidney 42] Increase in Light Chain Immunoglobulins Is the characteristic feature of A} Endocrine type Amyloidosis B} Hereditary Amyloidosis C} Localized type Amyloidosis D} Reactive Amyloidosis E} Systemic type Amyloidosis 43] Chemotaxis is caused by A} C5a B} C5b C} Lymphokines D} Leucotriene B3 E} Opsonins 44] Following are the actions of Archidonic acid metabolites in inflammation EXCEPT A} Chemotaxis B} Increased vascular permeability C} Thrombosis D} Vasoconstriction E} Vasodilatation . C3s E } C 1 . C2. The blood condition is most likely due to A} Iron deficiency B} Decreased bone marrow activity C} Folic acid deficiency D} Increased placenta size E} Plasma Volume Expansion 47] The Sensations of Anterior Triangle of the neck is mediated through A } C 1 . C2. C4 C} C2. C3 B } C 1 .0 g/dl with normocytic normochromic picture. He is most likely to have A} Anterior infarction B} Anterolateral infarction C} Anteroseptal infarction D} Inferior Infarction E} True Posterior infarction 46] A female at 30 wks of gestation has Hb: 9. C4 D } C2. C2 48] All are features of IRREVERSIBLE cell injury EXCEPT A} Karyolysis B} Karyorhexis C} Autolysis D} Shrinkage of mitochondria E} Appearance of myelin figure 49] A young athlete has difficulty in flexion of knee and extension of hip. And has ST segment elevation in LEADS V1 to V4. Which of the following muscle is likely paralyzed A} Rectus femoris B} Sartorius C} Semitendinosus . C3.45] A 65 year old male pt: presented with chest pain for last 30 mins. C3. He has history of taking some unknown drug. NaHCO3 reverses the action of the drug.D} Vastas lateralis E} Popliteus 50] In the Aortic opening I the Diaphragm is constricted. urine will leak into A} Deep Perineal Pouch B} Superficial Perineal Pouch C} Fascia lata D} Scrotum E} Ischiorectal fossa 52] The most common site of fertilization in humans is A} Ovary B} Uterus C} Fallopian tube D} Cervix E} Peritoneal cavity 53] The initial event in the transfer of RBC to site of inflammation A} Margination B} Emigration C} Pavementation D} Diapedesis E} Phagocytosis 54] A semiconscious patient is brought to the emergency room. has likely taken: A} Phenobarbital . which of the following structure will be compressed along with Aorta A} Azygous vein and Vagus nerve B} Thoracic duct and Vagus nerve C} Thoracic duct and Azygos vein D} Azygos vein and both phrenic nerves E} Inferior vena cava 51] When the bulbur urethra wil injure. Which drug the pt. I. which of the following enzyme will be raised? A} CK-MB B} LDH C} Alkaline Phosphatase D} AST E} Troponin T 56] Which organelle is not present at axon hillock A} Microtubules B} microfilaments C} Mitochondria D} RER E} SER 57] Which is not related to embolus? A} Liquid B} Gas C} Thrombus D} Amniotic fluid E} Fat 58] Which of the following muscle compensate for supinator? A} Biceps Brachi B} Brachialis C} Brachioradialis 59] A patient presented with anterior wall M.I Artery involved is A} LAD .B} Phenothiazine C} Morphine D} Diazepam E} Alcohol 55] Within 1 hr of the Acute M. of cell D} Decrease in size of cell 63] Diabetic patient using Roziglitazone and metformin. Which investigation for follow up A} Lipid profile + Renal profile B} Renal profile + Hepatic profile C} Renal profile + Hepatic profile + Cardiac 64] If the whole movement of Abduction of arm is lost the likely damage is in the Brachial Plexus A} Upper trunk + Posterior cord B} Upper trunk + Medial cord C} Lower trunk + Posterior cord D} Lower trunk + Medial cord 65] Cords of Brachial Plexus are named according to Relation with A} Axillary artery B} Subclavian artery 66] Myelination in CNS is done by A} Schwann cells B} Oligodendrocytes C} Astrocytes . of cell C} Decrease in no.60] Which of the following molecule easily crosses Blood Brain Barrier? A} CO2 B} H2O 61] Hypervitaminosis A cause A} Ataxia B} Anemia C} Scaly Dermatitis 62] Hypertrophy indicates A} Increase in size of cell B} Increase in no. D} Microglia cells 67] Captopril A} can be given in large doses in hypertensive crisis 68] Which of the nucleus is NOT related to Facial Nerve? A} Main Dorsal Nucleus B} Nucleus of Tractus Solitarius C} Nucleus Ambiguas D} Sensory nucleus 69] Platelets A} Are formed from precursor cells in the bone marrow B} Has no nucleus 70] Primary auditory cortex is present in A} Occipital lobe B} Superior Temporal Gyrus C} Frontal lobe D} Prefrontal lobe 71] Patient is suffering from factor VIII deficiency A} Genetically Engineered Factor VIII is available 72] Most Common cause of Thyrotoxicosis A} Graves disease B} Solitary nodule C} Toxic nodular goiter D} Diffuse goiter 73] Which of the following is involved in maintaining RESTING MEMBRANE POTENTIAL? A} Outward Potassium current B} Outward Sodium current . and also difficulty coming downstairs the likely nerve damaged is A} Oculomoter nerve B} Trochlear nerve C} Trigeminal nerve D} Optic nerve E} Abducent nerve 76] Substance has to cross semi permeable membrane in simple diffusion A} Is not saturable B} Require carriers C} Require energy 77] Carcinoma prostate drain into A} Para-aortic nodes B} External iliac nodes C} Internal iliac nodes D} External and internal iliac nodes 78] Pulmonary capillary wedge pressure is so A} Catheter used is wedge shaped B} Is direct measurement of left atrial pressure C} Is direct measurement of Right atrial pressure .C} Inward Sodium current D} Inward Chlorine current E} Sodium/potassium ATPase Pump 74] Heart sound produced by rapid ventricular filling is A} 1st heart sound B} 2nd heart sound C} 3rd heart sound D} 4th heart sound E} 5th heart sound 75] A patient presented with vertical diplopic. head tilting. Microcytic hyperchromic anemia c. It is autosomal recessive c. Only male suffer Q#11.Regarding polycystic kidney: a. Only females suffer e. Glucagon d. Hemolytic Anemia e. Genetic mutation d. Insulin (ans) Q#10. It is autosomal dominant b.After 3 months to gastrectomy. Microcytic hypochromic anemia b.b. Pt will develop: a. Macrocytic Anemia Q#12. Megaloblastic anemia (ans) d. Cortisol c.Non-myelinated nociceptor fibre directly synapses with: . Clostridium difficile (ans) b. Coarctation of aorta b.a.Pseudomembranous colitis is caused by a.Somites are derived from a.perferinges c. Mesoderm Q#14. Dorsal spinothalamic tract sorry cant remember other options Q#13. PDA (ans) . Failure of closure of foramen ovale c. C. which cardiovascular anomaly will most likely develop: a. Helicobacter pylori Q#15.A 2 yr old child (premature). Which is the major circulating phagocyte? a. GABA Q#19.Q#16. Glycine b. Dopamine c. Macrophage (ans) Q#17.Bile salts are normally absorbed from .ICF differs from ECF i n : a. Neutrophil b. Ach d.Neurotransmitter of corticostriatal fibres is a. Has increased Ca+ Q#18. Has increased Na+ b. Mast cell c. Has increased K+ (ans) c. Monocyte d. R. Microfilaments e.Glucronide conjugation of drugs .R c. Duodenum b.a. S. Mitochondria Q#23. Microtubules d. Stomach Q20. Jejunum c.which of the following is absent in Axon hillock a. Nalaxone (ans) Q#22.Antidote for Morphine: a.E. S1Q3T3: cant remember options for this Qz Q#21.E. ileum d.R (ans) b. Is type 1 reaction c. Cause inactivation of drug b.Dorsal rami of cervical nerves innervate a. MACula densa b. Pct d. DCT .C6 Q#25. Extensors of trunk d.a. Stabilizers the shoulder Q#24. Abductor pollices b. Extensors of limbs c. pertubular capillary (ans) c.Upper end of esophagus is at the level of a. Makes drug insoluble Q#23.Which part of kidney produces Erythropoiten: a. One large cell with multiple nuclei arranged close to membrane is also seen. Damage to Golgi bodies b. swelling of cell Q#28.The most likely change is: a. Caseous necrosis b. Fatty change d. Coagulation Necrosis c. Hydropic change e. metaplasia of cell e.Side effects of thiopental .Tissue damage by Ionization Radiation is due t o : a.Loop of henle Q#26. Hydropic change d. Formation of free Radicles c.e.Liver biopsy from a pt shows a lesion which is comprised of central collection of structureless material surrounded by epitheloid cells and lymphocytes. Liquefective necrosis Q#27. Seizure 29. IgA . IgA b.Which is highest in human milk? a. IgM b.Posterior belly of diagastric is supplied by: a. IgE e.Which immunoglobulin has pentameric structure: a. IgM c.a. Nausea b. Glossopharngeal nerve b. IgG Q#31. Trigeminal Q#30. Facial nerve c. IgD d. Gastroduodenal artey arises from a.Blood supply of Visual area: a. Esophagus b.c. Thoracic duct Q34. Posterior cerebral artery Q33. Middle cerebral Artery c.Aneurysm of arch of aorta will most like compress a. Left main bronchus c. Celiac . IgG Q#32. IgE d. Ant:Cerebral Artery b. Phrenic nerve d. IgD e. Splenic d. Medullary c. Melanosis Q37.Which does not lead to Pre-malignancy a. Follicular d. HBV d. which nerve is damaged? a. papillary Q36. Iodine deficiency b. HIV c. Anaplastic b. Superior mesentric Q35. Slow growing tumor of thyroid is: a.A 34yr old man can’t abduct his arm due to fracture of humerus.b. Hepatic c. Axillary . Musculocutaneous Q38. Increased ACTH Q39. decreased plasma volium d. Histology of pitutary will show.A 35 yr old female is protected against osteoporosis due to . a. Increased Acidophilic cells (most probably diz was the ans) Q40. Median c. Ulnar e.Aldosterone will increase with: a. Radial d. Increased serum K+ b. Increased serum Na+ c.A 14 yr Boy comes with the complaint of his abnormal Height and secretion from mammary glands. Increased Chromophobes b.b. Blood e. Nerve (ans) c.A. Muscle FCPS . Bone d. Progestron 41. Estrogen (ans) B. Epithelium b.Which is not “ Major Basic tissue” a. 2010 4:44 pm 1) If a person came to u with infertility and azospermia what would be the best test a) FSH +LH b) prolactin c) buccal smear d) LH e) t e s t o s t e r o n e 2) a) b) c) 3) a) b) c) whats Hb H disease?? omission of 3 alpha chains of Hb presence of barts hemoglobin requires frequent blood transfusion which one is not the part of brainstem? pons cerebral peduncles cerebellum .Medicine June 2008 Paper • by drwatson » Thu Jul 22. d) superior colliculus e) mesencephalon 4) a) b) c) d) Radiation affects/causes injury to cell membrane nucleus cytoplasm mitochondria 5) a) b) c) cerebellar lesion causes adiadokinesia sensory loss motor weakness d) 6) a) b) hypertonia ovarian ca metastasizes early to lung peritoneum c) stomach d) opposite ovary 7) Isthmus of thyroid gland lies against the tracheal rings: a) 1st and 2nd b) c) d) e) 2nd and 3rd 3rd and 4th 4th and 5th 5th and 6th 8) Lymphoid nodules are not present in a) spleen b) thymus c) tonsil d) peyers patches 9) Most common Souce of pulmonary emboli a) femoral vein b) popliteal vein c) periprostatic veins d) periovarian veins . In which lobar bronchus it must have lodged a) Left upper b) c) d) e) Right inferior rt middle rt superior left inferior 13) Left 7th bronchopulmonary segment is called a) lateral basal b) anteromedial basal c) lingual lobe d) apicoposterior e) posterior basal . Diagnosis after his labs showed unconjugated hyperbilirubinemia a) Dubin johnson syndrome b) Bile duct obstruction c) Gilbert syndrome d) Rotor syndrome 12) A child aspirated a peanut. The muscle most likely affected a) Biceps femoris b) Rectus femoris c) Semimebranosus d) sartorius e) semitendinosus Hidden content: UNLOCKED 11) A child sufferd from repeated episodes of mild jaundice.e) superficial saphenous vein 10) A lady suffered an injury in a RTA and became unable of flex both her leg and thigh. 14) The following does not have an associated valve a) superior vena cava b) c) d) e) inferior vena cava coronary sinus aorta pulmonary trunk 15) The following is not present in relation to Posterior relations of left kidney a) quadratus lumborum b) psaos c) colonic flexure d) diaphragm 16) Normal quiet expiration is brought about by contraction/recoil of a) b) c) d) diaphragm elastic tissue in thoracic and lung wall abdominal muscles sternocleidomastoid 17) A child died of severe pneumonia. the lung shoed Warthinfikelendenberg cells with many nuclei. At autopsy. causative organism: a) Ebola virus b) CMV c) measles virus d) rubella virus e) pneumocystis carinii 18) The following disease most severely affects the esophageal phase of swallowing a) myasthenia gravis b) scleroderma c) poliomyelitis d) stroke 19) The normal esophagus is . a) 10 inches in length b) lies entirely in the thoracic cavity c) consists entirely of skeletal muscle 20) A 26 yr old girl 28 week of gestation developed vomiting.bilirubin 20mg/dl..2 weeks back she went to meet her parents in a remote village.whats her diagnosis a) hepatitis A b) hep B c) hep c d) hep d e) hep e 21) Genes are a) b) c) d) e) intron exon DNA RNA ribosome 22) Lymphatic drainage of medial quadrant of breast a) axillary b) internal memory lymph node 23) Renal excretion of a drug a) depndnt on GFR b) 24) Menopause is associated with a) low estrogen and high FSH and high LH b) low estrogen and high FSH and low LH c) high estrogen and low FSH and LH d) low estrogen and normal FSH and LH 25) A 20 yr old girl with type 1 diabetes is brought to emergency in semi ..SGOT 272 and SGPT 220 . she has no history of any blood transfusion or past surgery. a person has a) high insulin and low glucagon b) high glucagon and high insulin c) high glucagon and low insulin d) high cortisol . An injection of insulin given to her will raise her a) blood sugar level b) pH of blood c) urinary excretion of ketones 26) Which one of the Following is terratogenic a) alcohol b) c) d) e) coffee heroin Phenothiazine tobacco 27) Lamia propria of vagina is made up of a) collgen fibres b) reticular c) elastic 28) Which of the following is not forming double peritoneum layer a) broad ligament b) suspensory lig c) mesovarium d) mesosalpinx e 29) Richest in triglycerides a) b) c) d) chylomicrons LDL VLDL HDL 30) At the end of a marathon race.comatosed condition. the new serum creatinine would be a) b) c) d) e) 0.31) A pts GFR dropped from 100 to 20.5 2.0 5. What will u do? a) volume replacement b) oral airway c) tracheostomy d) splinting of fractures e) .0 30 20 32) Target tissue of Rickettsia?? a) endothelial cells b) nervous tissue c) macrophages d) muscle cells e) blood cells 33) Most effected organ by dental radiologya) brain b) thyroid gland c) lungs d) heart 34) Which nerve is most likely to damage during 3rd molar surgical removal? a) mandibular nerve b) inferior alveolar nerve c) lingual nerve d) baccal nerve 35) A patient presented to emergency in a semicomatosed condition after an RTA with multiple long bone fractures with BP 80/50 mmHg. If the serum creatinine was 1 with GFR 100ml/min. diagnosis is a) aids b) infectious mononucleosis 40) Changes in V1 & avf indicates a) anterior wall MI b) inferoir wall MI 41) A Patient presented with cysts in liver.cause can be a)teania solium b)teania echinococccus 42) 3rd heart sound is due to a) rapid ventricular filling b) AV valves closure . blood picture shows atypical lymphocytes.36) Which of the following drugs is proved to be beneficial in the treatment of Subdural haematoma? a) Atenolol b) nifedipine. c) losartan d) nimodipine 37) Which of the following tumors in children is due to gene amplification? a) Neuroblastoma b) retinoblastoma c) Wilms tumor 38) Chronic diarrhea can be due to a) giardiasis b) crohn’s disease 39) A patient presented with sore throat & fever. investgation to evaluate cause is a) chest x-ray b) ABG'S 47) In adults...resport indiactes hematoma in right parietal region.43) Following is responsible for CNS myelination a) shwan cell b) oligodendrocyte 44) Pt presented in emergency in unconcious sate.5gm protein/dl a) CSF b) Lymph c) plasma . spinal cord ends at level of a) L1 b) L3 c) L2 Perhaps it was middle meningeal artery in the above question 48) Upper end of esophagus is at the level of a) C6 b) C7 c) T1 49) Fluid having 0. examination shows decreased breath sounds & hyper -resonance. cause is a) internal carotid artery b) basilar artery c) middle cerebral artery 45) Functional residual capacity a)can b measured by spirometry b) RV+ERV 46) A Diagnosed case of emphysema presents in emergency with dyspnea. . cardiac enzymes are normal. lesion is in a) cerebellum b) basal ganglia 56) There were 2 or 3 questions regarding acid-base balance but dont rememer the values. .What is the diagnosis? a) Acute MI b) Ruptured Aortic Aneurysm 55) A patient presents with scanning speech & intention tremors.ECG shows ST segment elevation. there is a) Decreased resting tone of LES b) Increased no of ganglia in myenteric plexus c) Decreased no of ganglia in myenteric plexus 52) Renal clearnace a) depends on GFR b) decrease in infants 53) Cerebellum lesion can cause a) b) c) d) adiadokinesia static tremors hypertonia sensory loss in body 54) A patient presented in the ER with an acute onset of severe chest pain radiating towards the left arm and jaw with a BP of 130/80 mm of Hg.d) serum 50) Severe blood transfusion reaction will occur a) A+ given to A-b) AB+ given to O+ 51) In achlasia cardia. the treatment of choice will be a) MGSO4 b) methyldopa c) clonidine 61) Which of the following develop from endoderm ? a) b) gonads c) d) palatine tonsil 62) Cannon wave is associated with a) tricuspid regurgitation b) complete heart block 63) Which of the following property prevent heart muscles from .57) Increase level of erythropoetin is seen in a) lung carcinoma b) renal CA c) cerebellar tumor 58) A young 20yr old guy while climbing stairs had sudden sharp Rt sided chest pain. on MAO inhibitor is given 1st dose of barbiturate will develop a) coma b) insomia c d 60) A lady in 3rd trimester comes with hypertension and edema .. she has history of seizures . 59)A pt. Cause a) b) c) d) ventilation-perfusion mismatch blockage of pulm capillary Fall in PO2 Fall in atm pressure e) ……. how long it'll take to reach steady state concentration? a) 4 mins b) 9 mins c) 16 mins 68) The pt. serum cardiac enzymes are not raised . most likely to have (exact scenario not memorized) a) b) 67) Half life of a drug is 2 mins .3 . pt.hco3 18 . comes to emergency with severe SOB for 4 hours.ecg shows ST elevation . the most likely diagnosis is a) MI b) pulmonary emboli c) retrosternal goiter 65) Jaw angle is innervated by a) b) c) 66) pH 7.tetanization? a) rythymicity b) prolong refractory period 64) A pt. pCO2 30 . the diagnosis is confirmed by a) urine culture b) blood culture c) urine + blood culture 69) The most potent stimulant for erythropoietin? a) b) hypoxia 70) The true statement about growth hormone is . is suffering from UTI . IGE+T cells d. Gastro duodenal artery Lt. C. T cells+macrophages b.malariae P. B. P. In type 4 Hypersensitivity mainly cells are a. B+T cells e.falciparum b.vivax 3: In duodenal perforation the artery involves.Gastric Artery Splenic Artery Rt. B cells+macrophages c. P. Gastric Artery 4: Parasite transmission e no feco-oral route .RIZWAN RIAZ 1. None.a) b) c) d ) its a polypeptide MEDICINE Oct 2008 15th OCT 2008(medicine & Allied) BY DR. A.ovale P. c. e. D. d. 2: Malignant tertian malaria caused by a.knowlasea P. Has 3 arterial supply C. Heart B. Liver C. E. D. Has same size in adults & infants B.Vernicularis C. Choroquine D. Pyramidal in shape D. Lateral plate 9: Most of drugs are metabolize in A. Arthemether B. Roth spots on 7th day C. Organism is B-hemolytic.A. Quinine 6: About RT adrenal gland choose correct A. Basoquine C. Ent histolytica B. C. Mefloquine E. B. non_lactose 8: Melanocytes derived from A. Kidney D. Separate form kidney by perinephric fat 7: Typhoid fever A. All pts becomes carrier B. Lungs . Tenia solium 5: Antimalarial drug e can be used other than malaria A. Endoderm Ectoderm Neural crest cells Pharyngeal arch E. Myeloprolefrative 1 2 : In Cushing decrease amount of A. 1 0 : ½ life of a drug Options not remember 1 1 : Premalignant condition 1. Midbrain ? 1 5 : Aspiration of gastric contents go in the A. Lymphocytes C. Pons B. Basophiles D. D. Rt lower lobe of lungs 1 6 : Comatose patient are more prone to regurgitation A. Cervical erosion 2. Monocytes 1 3 : Babinski sign -ve in A.. Stomach full B. B. Lt lateral position 1 7 : Vein that doesn’t directly drain in the inferior vena cava . Rt lateral position C. Neutrophils B. C. Medulla C. UMNL Sleep New born LMNL 1 4 : Loss of accommodation reflex due to occulumotor nerve site of lesion A. Ribs D. Aspirin 2 2 : Rx of salicylate poisoning which can increase the excretion of salicylate A.A. headache Mydriasis associated with no option of meningitis Options not remember 2 0 : An I/v urograph of old boy shows Excretion of dye normal at Rt side but absence of shadow of Lt kidney instead there is a small shadow above the bladder on Lt side Most likely i. I/v NAHCO3 B. Horse shoe kidney 2 1 : Drug with inhibit COX-1 as well as COX-2 A. Lumber vein 1 8 : Rt kidney doesn’t relate to A. Unilateral agenesis iii. Diclofenic B. Ascending colon C. Crossed ectopic kidney i i . Pelvis kidney v. Descending colon 1 9 : Fever. Liver B. Thymus . Diuretics D. 1 0 % D/W 2 3 : organ having subcapsularis & receives afferents A. Palatine tonsils B. Dehydration state C. Non rotating kidney iv. Naproxen C. Hepatic vein B. Lingual tonsils C. D.D. ADH B. Receptors destruction C. Glucagons lipogenic Insulin lipolysis Body fat <5 g/dl Epinephrine has lipolytic effect 2 6 : IDDM main disturbance by causing A. Aldosterone C. B. 1st heart sound B. C. None of the above . Spleen 2 4 : End arteries are in A. 3rd heart sound D. Resistance to insulin B.C space in midclavicular line 2 9 : AV valve closure is responsible to A. Lymph node E. Heart 2 5 : Fat stores in body A. Kidney D. Angiotensin 2 2 8 : Sound of mitral valve is best heart at Left 5th I. Lungs E. Spleen B. Eryropoietin D. 4th heart sound E. Increase ketogenesis by liver 2 7 : Thirst increase due to A. 2nd heart sound C. Brain C. H. D. Atrial systole B. Rapid ejection C.O C. Len oleic C. Isovolumetric relaxation D.A RT auricle Sulcus terminalis Upper part of Crista Terminals of R. Venous Compliance B. B. Due to decrease interstitial fluid osmotic pressure C. Decrease C. Increase C.O 3 2 : In Hemorrhage after compensation with one of the following decrease A. Rapid inflow 3 1 : In neurogenic stock A. Lower part of Crista Terminals of R. Decrease plasma colloid osmotic pressure B. C. Cardiac output 3 3 : CCF pt developed tachycardia best management of RX OPTIONS NOT REMEMBER 3 4 : Site of SA node A. Loss of vasomotor tone B. Decrease capillary pressure . Saturated F.A 3 6 : Boy having edema due to renal A.R C. Palmitic acid B.A 3 5 : Which one is essential Fatty Acid A.3 0 : During cardiac cycle ventricles are filled in A. inflammtion 4 1 : X-linked disorder A. APTT C. OXYTOCIN 3 9 : Which 1 of the following is aggressive in nature? A. Duchene muscular dystrophy D. Small cell carcinoma B.3 7 : Histamine theophyline & Aminophylline have same action A. Nevis cell 40: CHEMOTAXIS is a feature of A. CH.inflammation B. To increase the gastric acid secretion 3 8 : I/v large amount of DEXTROX will inhibit A. Plat. AC.count D. THYROID D. Thrombin time E. Cystic fibrosis 4 2 : In factor 8 deficiency specific test is A. CORTISOL B. Fibrin products 4 3 : Vomiting centre located in . PT B. Glycogen storage disease C. Basal cell carcinoma D. Squamous cell carcinoma C. Marfan syndrome B. ADH INSULIN C. A. Vertebral artery B. Choose correct 1. Covered by pretracheal fascia C. Basophiles C. Medulla C. Lymph node C. OPTIONS NOT REMEMBER 4 8 : Regarding parathyroid gland. Located anterolateral 2 thyroid gland B. Reticular Formation 4 5 : Arterial supply of spinal cord A. A. Derived 4om 1st pharyngeal arch 4 9 : Radiosensitive tumor A. Lymphocytes 4 7 : Regarding Glomerulotubular balance. Muscle D. Choroidal artery C. Choroidal artery 4 6 : Mast cells r released 4om A. Thyroid artery D. Midbrain D. Esinophils B. Ant. Cartilage . Hypothalamus B. Pituat. Cerebellum 4 4 : Arousal is controlled by A.gland C. Neutrophils D. Supplied by sup. Bone B. Pons B. Post. Minimus C. C. C.R Inc. Urine O. H. PT APTT PT & aPTT Thrombin E. G.O Inc. E.Maximus B. Fibrin products 5 6 : Pt. Sartorius . Inc. B. H. D.R 5 4 : Regarding Microscopic feature of CASEOUS Necrosis OPTINS NOT REMEMBER 5 5 : Female pt.5 0 : Rh incompatibility related 2 Type 1 hypersensitivity reaction Type 2 hypersensitivity reaction Type 3 hypersensitivity reaction Type 4hypersensitivity reaction None of the above 5 1 : Regarding IGG2 cells OPTIONS NOT REMEMBER 5 2 : Regarding Th-2 cells OPTIONS NOT REMEMBER 5 3 : Hypovolumic shock A.P Dec. came after 1 day of RTA with c/o unable of standup after seated position. on oral contraceptive & u want to give her oral anticoagulant u must chk A. G. Hot peripheries B. C. A. Probably muscle injury. D. C. L3 B. Q. Injury to A. C5 C.P. Median nerve Radial nerve Ulnar nerve Long thoracic nerve E. Horner syndrome 6 1 : Pt.P C. I. its due to. L4 C. Foot. expire due to sudden inc.COZ OF DEATH A. C8 58:Diagnosed pt. Gracilis E.C. B. C7 E. Berry aneurysm B. feel tingling sensation on rt. Mys. Little toe with difficulty in eversion of Rt. C6 D. Marfan aneurysm . S2 5 9 : Below the medial epicondyle of humerus passes A. of sciatica cam in o.D. Femorus 57:Pt. S1 E.p. D. L5 D. C4 B. Dissecting aneurysm C. Gravis B.Autopsy shows thinning of tunica media. Musculocutaneous nerve 6 0 : Intermittent Diplopia is a classical sign of A. B. Thumb during c-setion after giving epidural anesthesia. A. Inc.d with c/o loss of sensation of RT. Adrenaline B. Dopamine & Dobutamine C.Coli . External Carotid C. Triazolam D. Temporal 6 3 : 1 of the following is not epileptogenic A. Maxillary Artery E. D. I/V NaHCO3 D. Internal Carotid D. Common Carotid B. flouxitin B. Amitriptyline C. depression 6 5 : 1 of the following is not classified as non barbiturate hypnotic A. Temporal arteritis 6 2 : Artery enters in cranium & becomes the principle artery of brain A. Nausea & vomiting Psychosis Pin point pupil Resp.Cholrea B. V.Botilinium C. B. 1 0 % D/W 67: Coz of FATAL DIARRHEA A. C. E.D. Choral hydrate C. Imipramine 6 4 : Opoid Toxicity causes A. Chlormethimazole 66: In CARDIOGENIC SHOCK A. C. Bromide B. B6 D. 3 B. 5 D. C. E. B. D. 9 7 2 : Tactile two point discrimination by A. C C. C. Blood transfusion Folic Acid Iron E-poi tin 6 9 : Pellagra is due to def. D. 7 E. hypokinesia A. Dialysis B. Pacinian Ruffini Free nerve endings Merkel disc 7 3 : Patient with resting tremors. Cerebellum . D.6 8 : Anemia of CRF is best treated with A. 4 C. Vita. Vita. Folic acid 7 0 : Site of constriction of esophagus. Cricopharangeus Left Bronchus crosses it When enters in diaphragm At cardiac end 7 1 : Single cranial nerve which doesn’t contain parasympathetic fibers A. Chose correct one A. rigidity. B. NIACIN BIOTIN B. C. of A. Temporal gyrus D. UMNL D. Fine touch C. Parkinson disease C. D. B. Post. Lat. Hyper sexuality . Brocas area 7 5 : Regarding lumber puncture which one is correct A. Preoptic hypothalamus (NO OPTION OF ANT. Temperature D. Vibration 7 8 : The first step occurs when light falls on retina A.B. Pain B. Between L4 & L5 CSF pressure in mm of Hg Spine fully extended Angle of needle must be 45 degree E. Hypothalamus D. Prefrontal gyrus C. C.P 76:Heat loose of body is controlled by A. Hippocampus 7 7 : Damage to dorsal column tract causes ipsilateral loose of A. Wernicke’s area B. HYPOTHALAMUS) C. Sup. Hypothalamus B. LMNL 7 4 : Motor aphasia due to lesion of A. Fear & rage B. 11-Cisretinal to all Trans retinal 7 9 : Lesion of mamillary body causes A. Advice to patient stands up immediately after L. Thalamus E. Supplies to middle ear D.L.C.R.V C. Physiological shunt . Tidal volume 84: Diagnosis of Emphysema A. E.C D.54. T. glucose 40 & mainly lymphocytes. Supplies to stapedius muscle 8 1 : CSF having proteins 400 mg/dl. Insula B. Encephalitis B. Hysterical hypoventilation C.V B. I. Purely sensory B. Carbon mono oxide poisoning B. Occipital lob 83: PH-7. R. Diagnosis A. FEV E. C. Loose of Recent memory D. Give chorda tympani to posterior 1/3 of tongue C. Hyperphagia 8 0 : Regarding facial nerve which one is correct? A. O2-55 & CO2-25 cause of deranged ABG’S A. Salicylate poisoning 83: Asthma diagnosed by A. Cryptococcus virus TBM Bacterial meningitis Viral meningitis 8 2 : Lateral sulcus of cerebellum A. Physiological dead space B. D. Alpha-2 Beta-1 Beta-2 Nicotinic 8 9 : Stress hormone of body called A. Diaphragm B. ACTH B. E. Sternocladomastoid D. Gap junction D.C. V/Q. Intercalated disk . Contains iron in ferric form C.affinity D. Tight junction E. Scalenius posterior 8 6 : Muscle of forceful inspiration A. V/Q-1 8 5 : Muscle of quite inspiration A. D. C. Causes acidosis when add with H+ 8 8 : Receptors mediates increase heat rate A. Sarcomere B. SER C. Intercostals muscle C. Alpha-1 B. External intercostals muscle 8 7 : Regarding hemoglobin A. Adult contains alpha-2 & gama-2 B. V/Q-0 E. Epinephrine NO OPTIONS OF CORTISOL 9 0 : Heart muscle works as syncytium because of A. 24 hours urinary out put B. High altitude 9 7 : 1/3 of total body water contains A. S. E.C.A pressure 9 2 : Diagnosed case of thyrotoxicosis came to you with heart rate of 190b/mn.9 1 : Preload depends on A. When deoxihaemoglobin is more then 5g/dl 9 6 : Pulmonary vasculature resistance increase in A.U. Atrial flutter B. EDV of L. Digoxin B. Plasma D.V pressure B.creatinine C.C.F B. B. Anemia hypoxia B.N .F C. Lidocain C.A) Blocks ACE enzyme 9 5 : Cyanosis due to A. Propanalol 9 3 : Saw tooth P waves in ECG associated with A.V block 9 4 : Captoprill (M.O. I. Verapamil D. EDV of R. Best treatment is A. A. Interstitial 9 8 : GFR can be calculated by A. C. Alcohol B.O. B. Pain D.T muscles A. Aldosterone causes commonly A. Secretin D. Juxtaglomerular cells C.I. D.D. V. Urea 9 9 : S/Chloride maintained by kidney through A. Acidosis DEC.H 100: INC.O. CCK E. Gastrin C.O. Na+ -K+ . Rennin Sodium depletion K+ retention 1 0 1 : M.I. Peritubular cells B.P .I.H is inhibited by A.H B.P B.D.A spironolactone OPTIONS NOT REMEMBER 102: A. G. Hemorrhagic Shock C. Exercise 103: Erythropoietin secreted from A. Macula densa 104: Hormone which causes relaxation of all G.2Cl co transportation in thin ascending L. Na+-K+-2Cl co transportation in thick ascending L. C cells 109: Amniocentesis shows 47XXY A. Klinefelter syndrome 110: In pregnancy cause of Hyper Parathyriodism A. Puituicytes 108: In autoimmune diseases of thyroid antibodies against A. Adrenergic C.coz of disturbance is A. Fructose Lactose Sucrose Maltose 106: Pt. Glucose B. TSH & when TRH given TSH slightly rises. TSH B. Turner syndrome B. Cholinergic B. Primary Hypothyroidism due to Hypothalamus 107: Neurphyposis contains A. C. Secretory cells D.105: Which causes diarrhea when it reaches to large colon? A. Adrenogenital syndrome C. Thyroglobulin C. D. E. Follicular cells D. Hypoparathyroidism . with puffy eyes & edema with dec. Prim. L. Scortal ligament C. Reiters Syndrome C. S. Nor adrenaline C. Gubernaculums Testis B.C. Epidydimis .E. D.B. Thyroid gland 115: Hydrocele is the aculummation of fluid A. serum Cholesterol D. Inc.A Calcitonin Alpha fetoprotein Beta H. Cortisol D. Ankylosing spondilytis B. B.A D.G 114: Which hormone maintains body temperature by oxidative phosphorylation A. C. R. Becoz estrogen causes inhibition of bone resorption 1 1 1 : Estrogen doesn’t have action A. C. Vas deferens E. Delayed bone loss after menopause B. Tunica vaginalis D. 112: HLA DR 27 associated with A. Adrenal B. Follicular growth C.E 113: Tumor marker of Thyroid carcinoma is A. 1 2 1 : C.8mg/dl. Breast B. D. came with jaundice Hb-10g/dl. Due to bile stones B.E.whats Ur diagnosis. Acute Hepatitis Rotor syndrome Dubin Johnson Syndrome Gilbert Syndrome E.S.U/S abdomen normal. Globulin Synthesis 119: Cholangio-Carcinoma caused by A. anemia B.A is a tumor marker of A. B. Colorectal Carcinoma C. A. absorption from gut C. A. Hepatitis B 118: IN Tumor immunity liver has a function of A.Retic 10%. E. To detoxify the toxic substances B. Echunococcus Granuloses 120: Old age pt. Defi. Protein synthesis C. Of VIT. Diagnosed as a case of OBSTRUCTIVE JAUNDICE & now cam With c/o bleeding tendency. S. Which 1 of the following cause. Megaloblastic anemia due to folate C. S. Clonorchis –Sinesis B. Testis . came with c/o A. Pernicious anemia. 117: Old age Pt.116: After gastrectomy pt. D.Hematobonium C.Japanicum D. C. Iron def.K due to DEC.Bili-2. Desmin 126: Which 1 of the following is the diff.B Sarcoidosis Leprosy None of the above 125: Which 1 of is not a Tumor marker A.V pts. T.whats Ur diagnosis A. Tobacco B. T. Colonic adenocarcinoma 123: Granuloma formation is not associated with A. Coal C. came with distortion of nose.D. Pneumococcal Pneumonia 124: Pt. AFB +ve. Have tendency to get infection of . Thyroid 122: Juvenile Polyps are A. Alpha 1 antitrypsin B. C. Asbestos worker 1 2 8 : H.I. b/w malignant & benign tumor A. Hereditary non polpyposis D.B C. Metastasis 127: Workers Having Increase chances 2 develop Malignancy A. B. Hyperplasic polyps C. Sarcoidosis B. lymph nodes palpable. Vesmin C. Meckels diverticulum’s B. D. C Deficiency Vit.V B.V C. Throboembolism C. conjunctivitis sicca. Sjogran Syndrome B.T. Less CD4 cells count B.M B. E deficiency 134: Diagnosed case of tuberculosis PPD or Tuberculin test found to be negative Because of one of the following factor A. Cox 1 Cox 2 133: Which 1 of the following factor causes Delayed wound healing Inc.B. Bacterial Endocarditis 132: Platelets aggregation is due 2 A.A. Thromboxane A2 B. Scleroderma 1 3 1 : Which 1 of the following is not associated with Dry gangrene? A. A. Xerostomia. Chicken Pox 129: Diagnosis of H. H. Prostaglandins C. Steroid intake .P D. D.T B. associated with A.V when A. Less CD8 cells count 1 3 0 : Joint pain.S. Immunosuppressive C. Pneumocystitis carnii E. E. H.I. Blood flow Vit.P. what mechanism is . Lactic Acid E. Urea C. shock like condition of a child. Uric Acid D.135: Blood group having no antigen to A & B A+ B+ AB+ OO+ 136: About leukocyte adhesion factor responsible Integrin Selectin 138: In humans the End product of PURINE metabolism is A. Xanthin B. flushed. Pyruvate FCPS MEDICINE Q's July 07 Part-1 • by drwatson » Thu Jul 22. 2010 5:02 pm 1) What structure is located superio-posterior to pituatory fossa Mastoid air cells Ethmoid sinus Sphenoid sinus 2) Which Lung function Test is diagnostic of Asthma? FEV1 FVC FRC RLC 3) Bee sting. Monotherapy to be prescribed ACE-I Thiazide .involved Type 1 reaction Type 2 Etc 4) What sensory receptors sense VIBRATION? Pacinion corpuscles Ruffini Etc 5) Cause of anemia in Pregnancy Fall in Hb Decreased heme Volume Expansion. a nerve is cut. 6) Cut to the postero lateral area of FOREARM near the head of Radius. injury to Radial Median Ulner 7) Structure not passing through FLEXOR RETINACULUM of forearm Ulnar nerve Median Nerve Flexor policis longus Etc 8)Cause of SQUAMOUS METAPLASIA in a 38yrs old lady Multiparity HSV infection IUCD 9) Mild CCF symptoms. no sensory loss. cant appose thumb. difficulty (I guess extending) thumb. 20)Body concerves body water by . (Circle of Willis was a hot thing this time. Like. Rx? Methyldopa 11) Side Effects of ACE inhibitors Hyperkalemia HypoKalemia Hypercalemia Hypocalcemia 12)BARRETT’S Esophagus. 16) Branches of BASILAR Artery 17) Vertebral Artery Branches. the basic concept. which is a branch of it. whats the cause? Loss of Myenteric nerve plexus Failure of relaxation of LES 13) Which is not a branch of VAGUS Lacrimal Nerve 14) Volume of Distribution.Frusimide Ca Channel blockers 10)Pregnancy Induced Hypertension. Defination? 15) First pass Hepatic Metabolism. Basic concept. more than 3 Q) 18)Which of these form Circle of Willis Post communicating artery Vertebral A Basilar A 19) RENIN ANGIOTENSIN System Mechanism of Aldosterone release. Location? 30) Histological section of a lymphoid tissue. choice of Rx? Biguanides Biguanides plus sulphonylurea Insulin 26) BARRETT ESOPHAGUS can lead to: Adenocarcinoma esophagus Adenocarcinoma Stomach Sq carcinoma Esophagus Stricture Hidden content: UNLOCKED 27) Location of PSEUDO STRATIFIED SQUAMOUS Epithelium 28) STRATIFIED SQUAMOUS Epithelium. Defination. newly diagnosed. Which mechanism is activated Renin angio aldo system ADH Etc 24) Which drug has a n ATROPINE LIKE ACTION Physostigmine Scopolamine 25)Old obese Diabetic. . 22) HYPERPLASIA Definition. a cortex and a medulla seen.ADH release Cortisol release Aldosterone 21) METAPLASIA. 23) VOLUME DEPLETION. characters 29) Pseudo Stratified COLUMNER CILIATED Epithelium. and some ?ducts visible. 31) Which cells cannot PHAGOCYTOSE Kuffners cells Mast cells Monocytes Polymorphs 32) SENSORY RECEPTERS in Joints 33) Complement system is activated by the interaction of ?some factor with A (cant remember the options) B C 33) Most Important use of ECF Excretion of CO2 from body Electrolyte exchange Cell nutrition 34) Reasons for susceptibility to infections in AIDS Decrease in CD4 count CD8 35) HAIRY LEUKOPLAKIA is seen in Fungal infection HIV Etc 36) Following PARTIAL HEPATECTOMY. remaining part can regenerate in 10 days 15-25 days 5 weeks 7 weeks 37) Which drug binds to COX-2 recepters and not to COX-1 . Identify. 39) What is the result of a lesion at the OPTIC CHIASMA 40) What structure is Not related to OPTIC TRACT? Lateral Geniculate body Medial ~ 41) Area of brain affected in BITEMPORAL HEMIANOPIA 42) CSF pressure is increased when what structure is blocked : Internal Jugular vein Common carotid 43)Functions of CEREBELLUM 44) Venous drainage of the HEART Anterior cardiac sinus Inferior vena cava Cardiac vein 45) RIGHT HEART BORDER is formed by RA RA+SVC RA+RV 46) CIRCUMFLEX ARTERY supplies LA LV Apex LA .Aspirin Indomethacin Piroxicam Meloxicam Ibuprofen 38) What is transmitted through corpus callosum. whats the cause/where is the lesion.L auricle 47) Whats correct regarding HEART? Atria and Ventricles contract simultaneously. 53) In Cervical part of spinal cord. occlusion of: Superior Mesenteric A Inferior Mesenteric A 49) Lymph Nodes involved in Ca CERVIX are? 50) Ca Cervix is caused by: IUCD HSV HIV 51) Sympathetic stimulationof Beta recepters causes Bronchial dilatation 52) Mix lesion of loss of pain and Temperature of one side and Proprioception of the other side. the following happens: Dreams . 54) In REM sleep. 48) What structures are present in the free border of the LESSER SAC Cause of abdominal angina. Left atria is 1st to contract Excitation passes from Atria to Ventricles directly. whats deficient? Grey matter White matter Anterior Horn Lateral horm Posterior horn. Contraction starts from Left apex Left Atria is 3 times larger than Right atria. whats the differenciating lesion . 55) Lesion in the Broca’s area: Can’t write Can’t speak except in few words 56) Surfactant: Decreases compliance Decreases when alveolar size decreased Has a low turnover rate 57) An athlete at rest: Increased Stroke rate Increased cardiac output Increased Heart rate 58) H-K-ATPase pump is blocked by Omeprazole Cimetidine Bismuth 59) Murmur of MS best heard at Apex Tricuspid area Etc 60) In Hypertension. Renin release is mediated by Renal artery dilation Na in distal limb loop of Henle 61) Vasodilation is mediated by Histamine 62) Tidal Volume is a part of which capacity 63) Pneumoconiosis.Sleep Walking. how will body respond Decrease in insulin release Increase in Glucagon Cortisol release Somatostatin release 70) Bone lesion in the elderly. carbon dioxide 44mmhg. Whats the metabolic disorder. Hco3 raised. 68) Histology of nephron.Fibrosis Pleural plaques 64) Mesothelioma: Pleural Plaques 65)Asthma. which part is in the medulla Loop of Henle Collecting duct 69) Infusion of I/V dextrose. Where is the defect – in Heme or Globin 67) CCF. . Hypertension. raised Ph. bone matrix is being eroded. which investigation is diagnostic: (BRS-Physio Page 136 to confirm) Decreased Fev1 Decreased FRC 66) Thellasemia. which of these changes CANNOT happen: Raised Potassium Decreased Calcium Decreased Hb Decreased Po4 72) Release of ADH happens under what circumstances. which hormone is responsible 71) CRF. on lab test major finding : Raised TLC Water levels in R Iliac fossa 76) Polypeptide hormone: Growth Hormone And 4 steriod hormones mentioned 77) Protein synthesis in secretory cells: RER SER Golgi A 78) Peroxisomes release: Hydrolysing enzymes 79) Hormone/s responsible for BREAST ATROPHY E P E+P 80) Dumb Question>> Which Hormone is responsible for respiration during the leutinisation phase FSH LH . differenciating lesion from UC: Perianal disease Etc 75) Appendicitis.(Basic concept) 73) Acid is released in stomach when Proteins are ingested Fat Carbohydrate 74) Crohn’s disease. whats the treatment? Alpha blockers Beta blockers Both Alpha+ Beta. 83) Hypertension. whats the cause? 82) Pheocromocytoma. Increased Na. whats the reason Ans: Hyperaldosteronism 84) Non Anionic Gap Metabolic Acidosis is seen in 85) Effects of Insulin on body fat deposits/metabolism 86) Gametogenesis 87) Modes of inheritance. Whats the diagnosis Shigella Whipples Celiac disease 91) Young girl with emlarded clitoris Androgenital syndrome Down .E (Do leave a comment if you think this Q makes any sense) P LH 81) Hashimoto’s Thyroiditis (Scenario given). ( 2+ questions were there) Like which is what. Decreased K. has been on glutin free diet for 8-10 days. Autosomal Dominant was asked 88) Autosomal Recessive disorders are: 89) Wheat contains which vitamin/mineral 90) Child having diarrhea. Left pelvis sinks down. 94) Right foot in the air. Is located in lacrimal fossa on the lateral side Both glands drained by single duct. low fibre (correct) 96) Man. which nerve is affected? L Superior Gluteal Nerve (correct) R Superior Glut L Inferior Glut R Inferior Glut 95) About Ca Colon. in a locked-in position.Etc 92) An old man hit by car bumper> #fibula. which nerve is affected. . Right sided present early Cause is increased fat intake. whats the cause: Nitrosamine Schistosoma 97) Lateral quadrant of breast drains into which group of Lymph node. course. Option: joins lingual nerve when exits skull. previously had a disorder in U bladder. which group of lymph nodes are involved: L anterior superior submandibular R L Post Sup submandibular R post sup submandibular. now came for follow up and is found to have Ca. 100) About Lacrimal gland. Ans: common paroneal 93) Knee fully extended. 98) CA anterior 2/3 tongue. 99) Chorda Tympani. which nerve is affected. salivation. Globus pallidus. around 35 brought to emergency. cause? Sub mucous fibrosis Misplaced Injection Ankylosis of TM joint The Q's below are courtesy of 'Dr Daffodil'.) Lignocaine Digoxin 106) This area is normally resonant to percussion 2-4 Intercostal space on Right 2-4 Intercostal space on Left 5-6 I/c space on Right 5-6 I/c space on 107) Boy. whats the treatment. Heart rate 45. which area is involved 103) Functions of Corpus callosum. Atropine (correct) (Organo phosphate poisoning. history of trauma to chin 2 years ago presents with inability to open mouth fully.101) Lacrimal duct opens at: Superior Inf Middle Spheno ethmoidal recess 102) In pure motor stroke. . age 10. which drug is responsible. side effects mentioned. Clorpromazine SSRI Etc 105) A man. 104) A patient on a psychiatric drug. sweating. Putamen. BP 60/40. uterusd. n ext.illiacb.Deep Inguinal 124) Radiotherapy effect the most a.Int.proteins .B6c.Dopamine 118) Proencephalin is abnduntly found ina.Atonic bladderb.neuropathic 116) Dopamine regulates a.uric acidb.800-1200127) Pri. min pressure difference b/w Lt.Norepinephrinc.500-600c.C3b Medicine October 2007 . supply mucus membrane of int.ovaryb.108) Vitamin raised in disturbed homocystine metabolisma.Basal ?Gangliab.b.depp inguinald.glutamted. Ut.gives secretomotor fibers to Parotid gland 122)Disease of cervix inolves lymh nodesa. contains more Triglyseridesb.apoptosis 113) Corpus callosum containsa.degenerationb.automaticc.C5ab.Carboxylation 110) End product of Purinea.Para aorticc.C3ac. earb.rifampicin 126) Insensible lossoesa.IgGd.paralysis of muscles of palate/toungueb.Somatostatin 117) Neurotransmitter related to emotionsa.low glucose .200-400b. resistance is (dont remember % or ratio) less than systemic circulation 120) During Ventricular ejectiona.due to damage of frontal cortex in dominant hemisphere 115) Tabes dorsalis causes bladder incontinence due to a. opsonin in classic pathwaya.Int.B12 109)Vit B 6 involve ina. Vent.Para aortic 123) prostate CA which lymph nodes be involveda. and Aorta 121)Facial nerve/chorda tympani?( dont exactly remember)a. illiacc.reductionc.cervix 125)CSF showing raised neutro.which drug to givea.Prolactineb.oxidationb. illiacb.ACHb. circulationa.Thalmus 119) Regarding pul.absorb fats in duodenum 112) Chemotherapy causesa. tubesc.ceftriaxoneb.B1.Cerebellumc.commisural fibersassociation fibers 114) Aphasia a.Int.urea 111) Miscellesa. a) Hyperkalemia b) conc. a) Lewy bodies b) Patchy demylination & white fiber gliosis c) axonal neuropathy d) Q.Q.creatinnine d) Q.emia more frequently than Phanacetin Q.3 Congenital cataract is associated with a) chiken pox b) chromosomal abnormality c) small pox d) Rubella Q. urine in initial stages c) increase in s.urea but normal s....4 MS is characterized by.2 ATN is associated with.5 All of the following have ability of phagocytosis EXCEPT a) Microgila b) Lymphocytes c) Kuffer cells d) Neutrophils ..1 Paracetamol a) b) c) d) e) increases PT its overdose causes Nephrotoxicity is a poor anti-inflamatory is more stronger than codeine causes met-Hb. 7 in an Asthamtic female.6 Pulmonary Arterial pressure increases with. the long needle goes far into greater palatine canal.9 Tissue which has in-ability to replace dead cells a) Heart b) Kidney c) Liver d) Bone e) Bowl Mucosa Q..Q.10 Non-essential amino acid includes a) Leucine b) Iso-leucine c) Methionine .causing anaesthesia of an autonomic ganglion.8 During giving local anaestheia. is a) sublingual glycerine trinitate b) intra.immeditae treatment of HPTN during intra-operative HPTN.venous nitroglycerine c) propranalol d) Q.as a result along with dryness of nasal mucosa also occurs a) decreased secretions of Parotid b) decreased secretion of Submandibular & sublingual glands c) Q. a) b) c) d) e) Essential HPTN Sympthatic stimulation Nitroglycerine Excercise Hypoxia Q. > 120 fl. Hb= 7.4 md/dl MCV. on investigation he is having Un-conjugated Bilirubinemia. d) Hypersplenisim Q. a) Hemolysis b) Vit B 9 def...13 A patient having generalized body weakness & numbness.14 Tissue with Dual autonomic supply but not reciprocal suuply is a) Skin b) c) d) e) Blood vessels Pupil Salivary glands Ciliary muscle Q.d) Tyrosine Q. the anemia is due to . c) Vit B 12 def.12 Glucronide conjugation of drugs a) causes inactivation of drug b) is Type 1 reaction c) makes drug insoluble in water d) increases its effects Q..11 Metabolic Alkalosis is caued by a) Chronic Hypoventilation b) Pancreatic Fistula c) hyperglycemia d) ATN Q.15 Cause of joint pain in Deep sea divers a) arterial embolism b) venous embolism c) DIC d) thromboembolism .. S4 b) Pelvic splanchnic Nerves c) vagus nerve .Q.17 A 35 yr old man is having bilateral ptosis of eye during evening since last few weeks.the confirmatory test of the diagnosis will be a) Tensilon Test b) Ach Receptor Antibody Test c) blood culture d) CT scan Brain Q.16 In summer.S3.18 if a High Dextrose water is infusion given to a person.19 Parasympthatic supply of Sigmoid Colon is a) Sacral segments S2.15 Cause of joint pain in Deep sea divers a) arterial embolism b) venous embolism c) DIC d) thromboembolism Q.in a fasting man the cause of concenterated urine is a) ADH b) No Oral intake c) Aldosterone d) increased sweeting Q.16 In summer.in a fasting man the cause of concenterated urine is a) Skin b) c) d) e) Blood vessels Pupil Salivary glands Ciliary muscle Q.it inhibits a) Cotisol b) ADH c) Aldosterone d) Insulin Q. 24 Dorsal Rami of the cervical nerves innervates a) Abductor Pollices b) Extensors of Limbs c) Extensors of Trunk d) Stabilizers of Shoulder Q. 21 Parietal & Visceral Pericardium is supplied by a) Phrenic Nerve b) Sympthatic Nerves c) Vagus neve d) Cardiac Plexus Q. Now he notices a bulging swelling above the clavicle on sneezing & coughing.3 yrs ago.23 Middle meningeal artery if ruptured while meinges are intact.22 A young man had sinjury to the neck with sum sharp object. this swelling most probably due to. a) Injury to deep fascia of neck b) demage to supra-pleural membrane c) Un-united first Rib Fracture Q..25 Muscles of Hand supplied by .d) Q.20 which does not take part in inflamatory reactions a) Adrenaline b) Histamine c) 5-hydroxy tryptamine d) Dopamine Q..the Haemorrhage will be a) Epidural b) Subdural c) Intracranial d) Q. a) ventral rami of C-8 b) Ventral branches of T-1 c) Median Nerve d) Ulnar Nerve e) Radial Nerve Q.26 A person has sharp knife injury to upper medial aspect of arm. the nerve injured is a) Median Nerve b) Axillary nerve c) Radial Nerve d) Musculocutaneous Nerve Q. 27 A Transplant from identical twin is a) b) c) d) Allograft Heterograft Homograft Isograft e) Xenograft Q. after which He is unable to Flex his elbow & having loss of cutaneous sensations on lateral aspect of forearm.28 Therapeutic Index of a drug indicates its a) Effectivity b) Efficacy c) Potency d) Toxicity Q.29 Pharmakokinetic interaction among drugs is examplified by : a) b) c) d) e) Shortening of action of Procaine by Adrenaline increase peripheral toxicity of Levodopa with Carbidopa increase toxicity of Methotrexate by Aspirin prevention of Nitroglycerine induced tachycardia by Propranolol blokade of acetylcholine receptors by atropine Q.30 A 30 years old 75 kg Diabetic man with recent diagnosis of . 103 mmol s.20 mg/dl s.0.2.5 mg/dl s. On ECG : Left Ventricular Hypertrophy s.Urea is increased & s. a) Lewy Bodies in Neucleus b) Ptachy demylination with white fiber gliosis c) Axonal degeneration .creatinine is normal Q.30 & Fasting BSL--..33 Sarcoidosis is diagnosed microscopically by a) Granuloma with Asteroid b) Non-caseating Granuloma c) Caseating Granuloma d) Macrophages & Giant cells Q.140 mmol s.Urea -.Uric Acid-.HCO3 -.31 Acute Tubular Necrosis is characterized by a) b) c) d) Hyper-kalemia Initially concenterated urine Metabolic Alkalosis s.Na -.7 mg/dl s.5 mmol s.Creatinine -.34 Multipule sclerosis is characterized by.Cl --.32 GFR is measured by clearance of a) Inulin b) Creatinine c) Urea d) Glucose Q.Hypertension.80 mg/dl The cause is a) Essential HPTN b) Diabetic Nephropathy c) Pheochromocytoma d) Addison Disease e) Primary HyperAldosteronism Q..7. K-.23 mg/dl 24 Hr Urinary Proteins-. there is increased Alpha-Fetoprotein.in sample taken from amniocentesis.its most probable cause is.38 Kleinifilter syndrome a) contains single X chromosome b) less chances of Mental retardation c) phenotypically female Q.40 At 35 weeks of Gestation. a) Down syndrome b) Turner syndrome .36 Thymus is a) Rich in Lymphocytes b) c) d) e) Present at birth but shorly regree after birth derived from 4th Pahryngeal Pouch is a part of Thyroid Lies posterior to Trachea Q.39 Following is Autosomal Dominant Disorder a) Phenylketonuria b) Hereditary Spherocytosis c) Hemophilia A d) Cystic Fibrosa Q.d) Q..35 What is INAPPROPRIATE about Vagus Nerve.37 True statement about Down Syndrom is a) Trisomy 23 b) contains single X chromosome c) more chances with increasing maternal age Q.. a) its longest Cranial Nerve b) passes in between IJV & ICA c) passes posterior to Carotid Sheath d) Gives Recurrent Laryngeal Nerve e) gives Superior Laryngeal Nerve Q. .the leision lies in a) Facial Canal b) Internal auditary meatus c) Facial Nerve Nucleus .41 Thoracic Duct.45 A patient having Facial deviation along with loss of sensations on anterior 2/3 rd of tongue.44 Source of Estrogen & Progesterone in last trimester is a) b) c) d) Chorionic Villus Placenta Overy Adenohypophysis e) Corpus Leuteum Q.. a) recieves Lymph from both right & left sides b) lies in posterior mediastinum c) passes just right to the thoracic aorta d) extends from upper abdoment to neck e) has a dilatation called Cisterna Q.42 Rate of passage of food in esophagus depends upon a) b) c) d) Gravitational Force Neuromuscular excitation different @ upper & lower oesophagus progressive peristalsis in esophagus Acidity Q.c) Spina Cystica d) Hydrocephalous e) Q.43 @ Leprotomy Retroperitoneal Abscess was found..the site woud be a) Stomach b) Sigmoid Colon c) Transverse Colon d) Ascending Colon e) Small intestine Q. the Bitemporal Hemianopia occurs due to leison of a) b) c) d) Central part of Optic chiasma Optic Nerve Optic Tract peripheral Part of Optic chiasma e) Lateral Geniculate body Q.47 Following nerve lies in the Cavernous Sinus a) Opthalmic b) Occulomotor c) Abducent d) Trigeminal Q.48 In a Pituitary Adenoma.46 All are the branches of Vagus nerve EXCEPT a) Auricular b) Lacrimal c)Pharyngeal d) Recurrent Laryngeal Q.50 Na Retension is associated with a) Dehydration b) increase ADH c) Decreased Aldosterone d) Heart Failure e) Hyperthyroidism .49 Muscle of Forced Expiration is a) Internal Intercostal b) External Intercostal c) Sternoceidomastoid d) Scalenus Anterior e) Scalenus Medius Q.d) Stylomastoid Foramen Q. of cells c) Capsule d) Q.following Lung function increases a) Alveolar Co2 b) Alveolar O2 c) Pulmonary artery O2 d) Pulmonry artery Co2 e) Ventilation Perfusion Q.53 When a branch of Pulmonary Artery is blocked by embolus.Dd ) blood group.54 Difference between Adenoma & Hyperplasia is a) increase in size of cell b) increase in No.DD) Blood group.52 Vital Capacity a) measured directly by Spirometry b) is decreased in some Lung diseases c) is the sum of Tidal Volume+ERV+IRV Q.the preferred immediate treatment is a) Octeriotide b) Vit K .51 Von-Willebrand Disease is a) Autosomal Dominant b) Autosomal Recessive c) in some family members asoociated with Factor IX deficiency d) in some family members associated with normal Factor IX Q.children can have all blood gp EXCEPT a) AB +ve b) AB -ve c) A + ve d) B +ve e) O -ve Q.Q.55 In a CLD patient presenting with Haematemesis.51 A male wd A+ (Ao. married to a female having B+ (Bo. K Q. the nerve demaged is a) Infragluteal b) Supragluteal c) Schiatic d) Femoral e) Obturator Q. the immediate treatment is with a) Carbamazepine b) Valporate c) Lorazepan d) Q.c) formed Platelet Conc.57 After a trauma.60 An unconsious man with pin-point pupils not responding to Naloxone.59 A man with persistant fits without regaining consciousness for 30 minutes.56 Among most important initial steps in apoptosis a) Activation of Bcl-2 Receptors b) Activation of Caspases c) Water influx into the cell d) Q. d) PPI e)whole blood Q.Regulation .the probable cause is a) Heroin b) Hasheesh c) Morphine d) Phenobarbitone .58 About K . a person has inability to stand from sitting position. following is correct a) K is 9 8 % in the cells b) normally completely reabsorbed in Distal tubule c) increase water affects K-balance d) decrease in D. is a) CCK b) Secretin c) Gastrin d) Pepsin e) Mucin Q.63 Supra-Renal Gland arterial supply a) b) c) d) recieves all braches directly from Aorta recieves blood from 3 sources recieves branches from both renal arteries recieves blood from inferior phrenic artery Q.66 On examination of a female..65 Which infection is not caused by DNA virus a) Chicken Pox b) Herpes Simplex c) Herpes Zoster d) Mumps e) Small Pox Q.61 Stomach gets its arterial supply from all EXCEPT a) Ceiliac artery b) Splenic Artery c) Superior Mesenteric Artery d) Hepatic Artery Q.Q.it is a) Squamous Cell CA b) Adenocarcinoma . Q.a plaque like leision seen on Postero-superior aspect of Vagina...64 Stenson's duct a) arises from posterior surface of Parotid b) pierces the Buccinator muscle c) opens in the oral cavity against .62 The Hormone which stimulates release of HCO3 from Pancreas without stimulating pancreatic enzyme secretions. 70 Basal Cell Carcinoma involves a) Buccal Mucosa b) c) d) e) Hard Palate Soft Palate Lower Lip Oral Cavity Q.68 On standing from a sitting position.given Parenteral Vit B-12.c) Rhabdomyosarcoma d) wart Q.69 Metastatic Calcification occurs mostly in a) Kidney b) Bone c) Lung d) Intestine e) Pancreas Q. the inability to absorb B-12 is due to loss of.. a) Parietal Cells b) Gastric Cells c) Cardiac Ceells .71 A female underwent Gastrectomy for Zollinger-Ellison Syndrome..67 About CSF most appropriate statement is a) it is ultrafilterate of Plasma b) has more glucose than blood c) has cushioing effect d) absorbed in Choroid Plexus Q.@ apex of lung there is increase in a) Blood Flow b) Ventilation c) pO2 d) pCO2 Q. no sensory loss. 6) Cut to the postero lateral area of FOREARM near the head of Radius. injury to Radial Median Ulner 7) Structure not passing through FLEXOR RETINACULUM of forearm . flushed. difficulty (I guess extending) thumb. cant appose thumb.MEDICINE Q's July 07 Part-1 1) What structure is located superio-posterior to pituatory fossa Mastoid air cells Ethmoid sinus Sphenoid sinus 2) Which Lung function Test is diagnostic of Asthma? FEV1 FVC FRC RLC 3) Bee sting. shock like condition of a child. a nerve is cut. what mechanism is involved Type 1 reaction Type 2 Etc 4) What sensory receptors sense VIBRATION? Pacinion corpuscles Ruffini Etc 5) Cause of anemia in Pregnancy Fall in Hb Decreased heme Volume Expansion. Monotherapy to be prescribed ACE-I Thiazide Frusimide Ca Channel blockers 10)Pregnancy Induced Hypertension.Ulnar nerve Median Nerve Flexor policis longus Etc 8)Cause of SQUAMOUS METAPLASIA in a 38yrs old lady Multiparity HSV infection IUCD 9) Mild CCF symptoms. Basic concept. 16) Branches of BASILAR Artery . whats the cause? Loss of Myenteric nerve plexus Failure of relaxation of LES 13) Which is not a branch of VAGUS Lacrimal Nerve 14) Volume of Distribution. Rx? Methyldopa 11) Side Effects of ACE inhibitors Hyperkalemia HypoKalemia Hypercalemia Hypocalcemia 12)BARRETT’S Esophagus. Defination? 15) First pass Hepatic Metabolism. 17) Vertebral Artery Branches. more than 3 Q) 18)Which of these form Circle of Willis Post communicating artery Vertebral A Basilar A 19) RENIN ANGIOTENSIN System Mechanism of Aldosterone release. the basic concept. Which mechanism is activated Renin angio aldo system ADH Etc 24) Which drug has a n ATROPINE LIKE ACTION Physostigmine Scopolamine 25)Old obese Diabetic. choice of Rx? Biguanides Biguanides plus sulphonylurea Insulin 26) BARRETT ESOPHAGUS can lead to: . (Circle of Willis was a hot thing this time. Defination. newly diagnosed. 22) HYPERPLASIA Definition. which is a branch of it. 20)Body concerves body water by ADH release Cortisol release Aldosterone 21) METAPLASIA. 23) VOLUME DEPLETION. Like. and some ?ducts visible. characters 29) Pseudo Stratified COLUMNER CILIATED Epithelium. 31) Which cells cannot PHAGOCYTOSE Kuffners cells Mast cells Monocytes Polymorphs 32) SENSORY RECEPTERS in Joints 33) Complement system is activated by the interaction of ?some factor with A (cant remember the options) B C 33) Most Important use of ECF Excretion of CO2 from body Electrolyte exchange Cell nutrition 34) Reasons for susceptibility to infections in AIDS Decrease in CD4 count CD8 . a cortex and a medulla seen. Location? 30) Histological section of a lymphoid tissue. Identify.Adenocarcinoma esophagus Adenocarcinoma Stomach Sq carcinoma Esophagus Stricture 27) Location of PSEUDO STRATIFIED SQUAMOUS Epithelium 28) STRATIFIED SQUAMOUS Epithelium. 35) HAIRY LEUKOPLAKIA is seen in Fungal infection HIV Etc 36) Following PARTIAL HEPATECTOMY. 39) What is the result of a lesion at the OPTIC CHIASMA 40) What structure is Not related to OPTIC TRACT? Lateral Geniculate body Medial ~ 41) Area of brain affected in BITEMPORAL HEMIANOPIA 42) CSF pressure is increased when what structure is blocked : Internal Jugular vein Common carotid 43)Functions of CEREBELLUM 44) Venous drainage of the HEART Anterior cardiac sinus . remaining part can regenerate in 10 days 15-25 days 5 weeks 7 weeks 37) Which drug binds to COX-2 recepters and not to COX-1 Aspirin Indomethacin Piroxicam Meloxicam Ibuprofen 38) What is transmitted through corpus callosum. Inferior vena cava Cardiac vein 45) RIGHT HEART BORDER is formed by RA RA+SVC RA+RV 46) CIRCUMFLEX ARTERY supplies LA LV Apex LA L auricle 47) Whats correct regarding HEART? Atria and Ventricles contract simultaneously. occlusion of: Superior Mesenteric A Inferior Mesenteric A 49) Lymph Nodes involved in Ca CERVIX are? 50) Ca Cervix is caused by: IUCD HSV HIV 51) Sympathetic stimulationof Beta recepters causes Bronchial dilatation . Contraction starts from Left apex Left Atria is 3 times larger than Right atria. Left atria is 1st to contract Excitation passes from Atria to Ventricles directly. 48) What structures are present in the free border of the LESSER SAC Cause of abdominal angina. whats deficient? Grey matter White matter Anterior Horn Lateral horm Posterior horn. 53) In Cervical part of spinal cord.52) Mix lesion of loss of pain and Temperature of one side and Proprioception of the other side. whats the cause/where is the lesion. 54) In REM sleep. the following happens: Dreams Sleep Walking. 55) Lesion in the Broca’s area: Can’t write Can’t speak except in few words 56) Surfactant: Decreases compliance Decreases when alveolar size decreased Has a low turnover rate 57) An athlete at rest: Increased Stroke rate Increased cardiac output Increased Heart rate 58) H-K-ATPase pump is blocked by Omeprazole Cimetidine Bismuth 59) Murmur of MS best heard at Apex Tricuspid area . Hco3 raised. how will body respond Decrease in insulin release Increase in Glucagon Cortisol release . Whats the metabolic disorder. 68) Histology of nephron. carbon dioxide 44mmhg. Renin release is mediated by Renal artery dilation Na in distal limb loop of Henle 61) Vasodilation is mediated by Histamine 62) Tidal Volume is a part of which capacity 63) Pneumoconiosis.Etc 60) In Hypertension. raised Ph. whats the differenciating lesion Fibrosis Pleural plaques 64) Mesothelioma: Pleural Plaques 65)Asthma. which part is in the medulla Loop of Henle Collecting duct 69) Infusion of I/V dextrose. Where is the defect – in Heme or Globin 67) CCF. which investigation is diagnostic: (BRS-Physio Page 136 to confirm) Decreased Fev1 Decreased FRC 66) Thellasemia. Somatostatin release 70) Bone lesion in the elderly. differenciating lesion from UC: Perianal disease Etc 75) Appendicitis. (Basic concept) 73) Acid is released in stomach when Proteins are ingested Fat Carbohydrate 74) Crohn’s disease. on lab test major finding : Raised TLC Water levels in R Iliac fossa 76) Polypeptide hormone: Growth Hormone And 4 steriod hormones mentioned 77) Protein synthesis in secretory cells: RER SER Golgi A . which hormone is responsible 71) CRF. bone matrix is being eroded. Hypertension. which of these changes CANNOT happen: Raised Potassium Decreased Calcium Decreased Hb Decreased Po4 72) Release of ADH happens under what circumstances. whats the treatment? Alpha blockers Beta blockers Both Alpha+ Beta. 83) Hypertension. Autosomal Dominant was asked 88) Autosomal Recessive disorders are: .78) Peroxisomes release: Hydrolysing enzymes 79) Hormone/s responsible for BREAST ATROPHY E P E+P 80) Dumb Question>> Which Hormone is responsible for respiration during the leutinisation phase FSH LH E (Do leave a comment if you think this Q makes any sense) P LH 81) Hashimoto’s Thyroiditis (Scenario given). whats the cause? 82) Pheocromocytoma. Increased Na. whats the reason Ans: Hyperaldosteronism 84) Non Anionic Gap Metabolic Acidosis is seen in 85) Effects of Insulin on body fat deposits/metabolism 86) Gametogenesis 87) Modes of inheritance. Decreased K. ( 2+ questions were there) Like which is what. 94) Right foot in the air. . in a locked-in position. low fibre (correct) 96) Man. Whats the diagnosis Shigella Whipples Celiac disease 91) Young girl with emlarded clitoris Androgenital syndrome Down Etc 92) An old man hit by car bumper> #fibula. which nerve is affected? L Superior Gluteal Nerve (correct) R Superior Glut L Inferior Glut R Inferior Glut 95) About Ca Colon. previously had a disorder in U bladder. which nerve is affected. has been on glutin free diet for 8-10 days. now came for follow up and is found to have Ca. Left pelvis sinks down. Right sided present early Cause is increased fat intake. Ans: common paroneal 93) Knee fully extended. whats the cause: Nitrosamine Schistosoma 97) Lateral quadrant of breast drains into which group of Lymph node. which nerve is affected.89) Wheat contains which vitamin/mineral 90) Child having diarrhea. Clorpromazine SSRI Etc 105) A man. which drug is responsible. 104) A patient on a psychiatric drug. sweating. which area is involved 103) Functions of Corpus callosum. which group of lymph nodes are involved: L anterior superior submandibular R L Post Sup submandibular R post sup submandibular. 101) Lacrimal duct opens at: Superior Inf Middle Spheno ethmoidal recess 102) In pure motor stroke. Is located in lacrimal fossa on the lateral side Both glands drained by single duct. BP 60/40. Atropine (correct) (Organo phosphate poisoning. 100) About Lacrimal gland. whats the treatment. Option: joins lingual nerve when exits skull. side effects mentioned.98) CA anterior 2/3 tongue. course. 99) Chorda Tympani. around 35 brought to emergency. salivation. Heart rate 45.) Lignocaine Digoxin 106) This area is normally resonant to percussion . Putamen. Globus pallidus. history of trauma to chin 2 years ago presents with inability to open mouth fully. Vent. contains more Triglyseridesb.Cerebellumc.degenerationb.B12 109)Vit B 6 involve ina.uric acidb. min pressure difference b/w Lt.Int.Basal ?Gangliab.glutamted.absorb fats in duodenum 112) Chemotherapy causesa. supply mucus membrane of int.Atonic bladderb.reductionc.Int.B1. earb. illiacc.ACHb.2-4 Intercostal space on Right 2-4 Intercostal space on Left 5-6 I/c space on Right 5-6 I/c space on 107) Boy. illiacb. 108) Vitamin raised in disturbed homocystine metabolisma.Carboxylation 110) End product of Purinea.due to damage of frontal cortex in dominant hemisphere 115) Tabes dorsalis causes bladder incontinence due to a.Norepinephrinc.Somatostatin 117) Neurotransmitter related to emotionsa. circulationa.Thalmus 119) Regarding pul. n ext. resistance is (dont remember % or ratio) less than systemic circulation 120) During Ventricular ejectiona.b.urea 111) Miscellesa.automaticc.Prolactineb. age 10.apoptosis 113) Corpus callosum containsa.gives secretomotor fibers to Parotid gland 122)Disease of cervix inolves lymh nodesa. and Aorta 121)Facial nerve/chorda tympani?( dont exactly remember)a. cause? Sub mucous fibrosis Misplaced Injection Ankylosis of TM joint The Q's below are courtesy of 'Dr Daffodil'.Dopamine 118) Proencephalin is abnduntly found ina.oxidationb.neuropathic 116) Dopamine regulates a.commisural fibersassociation fibers 114) Aphasia a.paralysis of muscles of palate/toungueb.B6c.depp . IgGd.Para aortic 123) prostate CA which lymph nodes be involveda. opsonin in classic pathwaya.500-600c.uterusd.illiacb.C3b .ceftriaxoneb. Ut.inguinald.800-1200127) Pri.cervix 125)CSF showing raised neutro.ovaryb.rifampicin 126) Insensible lossoesa.Int.C5ab.proteins .Deep Inguinal 124) Radiotherapy effect the most a.C3ac.200-400b.which drug to givea.low glucose . tubesc.Para aorticc. T.ARRTHYMIA B. 2010 4:33 pm SURGERY JUNE 2009 SURGERY PAPER 24 JUNE 2009-07-11 By Dr Sana Memon.B ULCER MARGIN? A.EVERETED B.TINNITUS B. Q1.DOES NOT HAVE CARRIER B.UPER LOBE B.EARLIEST SIGN OF ASPIRIN OVER DOSE? A.RT LOWER LOBE C.WHICH LOBE OF LUNG HAS 2 SEGMENT A.RT.I.LEFT UPPER LOBE Q2.LIMITED TO G.MAIN SIDE EFFECT OF LIDOCAINE A.LEFT LOWER LOBE E.INDIFINE 24-Mar-12 4:32:00 PM .PROTZOA Q5.ENTAMOBEA HISTOLYTICA? A.RT MIDDLE LOBE D.GASTRIC UPSET C.T C.NAUSEA VOMITING Q4.INVERTED C.surgery FCPS ..TONIC CLONIC SEZIRES C.METABOLIC ALKLOSIS Q3.SURGERY June 2009 Paper2 Dby drwatson » Thu Jul 22. D.INDURATED Q6.SECTRION OF HARMONE INCREASE AS DARKNESS INCREASING? A.CORTISOL B.GROWTH HARMONE C.MELATONINE D.INSULIN Q7.INDICATION OF SPLEENECTOMY? A.AUTOIMMUNE HEMOLYTIC ANEMIA B.SICKEL CELL ANEMIA C.G SIX PHOSPATES DIF D.THROMBOCYTOPENIA Hidden content: UNLOCKED Q8.RBC CELL MEMBRANE? A.ACTIN B.MYOCIN C.SPECTRIN D.HEMOGLOBIN Q9.CLOSTRIDIUM DIFFIC? A.CAUSES LOCAL HYPOXIA B.AEROBES C.ALPHA TOXIN D.CONVERSVATIVLY TREAT. Q10.INGUINAL HERNIA? A.INDIRECT PASSES ONLY SUPER;RING B.DIRECT PASSES THROUGH HESSAL BECH TRIANGLE C.DIRECT LYING ON MEDIAL SIDE OF INF:EPIGASTIC ARTERY Q11.CONTENT OF INGUINAL HERNIA? A.INF:EPIGASTRIC ARTERY B.LYMPH OF FUNDUS OF UTERUS. C.OVARIAN ARTERY. D.FEMORAL ARETY Q12.DIALYSING FLUID COMPOSTION SAME AS PLASMA AXCEPT WHICH ONE HIGH? A.GLUCOSE B.UREA C.K* D.CA Q13.COMMON ORGAINSM INFECTION IN LIVER TRANSPLANT PT: A.HEP.B B.HEB C C.CYTOMEGALO VIRUS D.INFLUENZA Q14.RT:CORNOARY ARETY? A.RISES FROM RT:POST AROTIC SINCUS B.SUPPLY BOTH ARTIUM C.RUN IN POST INTERVENTRICULAR GROOVE D.BR:CIRCUMFELX Q15.BRUN PT; HYPOTENSION,TACYCARDIA? A.D.I.C B.INFECTION Q16.BOY WITH INCREASE BLEEDING TIME AFTER CIRCUMCISION,HIS COUSIN ALSO HAVE SAME PROBLEM? A.BT B.CLOTTING TIME C.APTT D.PT Q17.SECOND HEART SONUD? A.CLOUSER OF TRICUSPID VALVES B.FILLING OF VENTRICLE C.CLOSURE OF SEMILUNAER VALVES Q18.BLADER CA? A.TRANSITIONAL CELL CA. B.Sq: CELL CA C.EPITHOID CA Q19.CALCITONIN? A.INCREASE BLOOD CA LEVEL B.INC;ABSORTION OF CA INTESTINE C.INC:ABSORTION FROM RENAL TUBULES D.INC:BONE REABSORBATION Q20.AFTER MULTIPULE BLOOD TRANSFUSION? A.HYPOKALEMIA B.HYPERKALEMIA C.HYPOCALCEMIA D.RAISED BUN Q21.COMPENSATION AFTER MILD BLOOD LOSS? A.TACYCARDIA B.DEC: COMP OF VEIN C.NORMAL B.P Q22.IN ALL TYPE OF SHOKE? A.HYPOVOLEMIA B.TACYCARDIA C.UNCONSIOUS Q23.FAT EMBOLSIM? A.SYPMTOMS AFTER 12 HRS B.80% FATAL C.COMON CAUSE THROMBPHILIBITIES D.BREAST TRUMA Q24.CHILD THIN,DEPIGMENTED,APATHY A.MARASMUS B.KASHIKOR C.DIARREHA Q25.PT:WITH GRANULOMATOUS LESION IN BLADDER DEVELOPED CA.CAUSE? A.SHISTSOMIA MANSONI B.CIGGRATE SMOKING C.SHIS;HEMATOBOIUM D.DIYES Q26.RETROGRADE AMNEISA? A.FRONTAL LOBECTOMY B.AGGREVATE BY TEMPORAL LOBE TRUMA Q27.METAPLASIA,INCORRECT? A.IRREVERSIBLE B.BRONCHIAL EPI; C.GASTRO ESOPHAGEAL JUNCTION D.CERVIAL JUNCTION Q28.APPENDIC EPIPLOIC PRESENT IN? A.DEUDENUM B.RECTUM C.JEUJENIUM D.ILIUM Q29.COMMON COMPLICATION OF BETAL NUTS? A.SUBMUCOSAL FIBROSIS B.EPITHEIAL POLYP C.CA; EPITHILIUM Q30.FEMORAL SHEATH? A.FASICA ILISCA B.FASICA TRAVERSALIS C.FASCIA TRANSVERSALIS AND ILISCAP Q31.LOCALLY MALIGNANT TUMOR? A.CHOLESTATOMA B.NEUROBLASTOMA. Q32.COMMON SIGN OF OF AGRUNLOSYTOSIS CAUSES BY ANTIEPILEPTICS? A.WT:LOSS B.SORE THOART C.BLEEDING D.GIT UPSET Q33.DURING OPERATION OF POST;CERVICAL LYMH NODE BIOPSY? A.CRANIAL PORTION OF ACCERY NERVE. B.SPINAL PORTION OF ACCESSRY NERVE. C.AXILLARY NERVE D.LOWER CORD OF BRACHIAL PLEXUSES E.PHERINIC NERVE. Q34.PT: WITH ROAD TRAFFIC ACCEDIENT,UNABLE DORSIFLEX AND EVERT FOOT? A.SUPER; PERONEAL NERVE B.DEEP PERONEAL NERVE C.COMON PERNEAL NERVE D.TIBEAL NERVE Q35.ESTABLISHED ANTIOXIDANT? A.VIT A B.VIT D C.VIT E D.VIT C E.VIT B Q36.URORECTAL SEPTUM? A.SEPRATE RECTUM AND URINARY BLDER B.SEPRATE RECTUM AND UROGENITAL SEPTUM C.SEPRATE RECTUM AND SIGMOID COLON D.SEPRATE RECTUM AND URETHRA Q37.70% OXYGEN EXTRACT IN RESTING CONDITION IN WHICH TISSUE? A.HEART B.BRAIN C.KIDNEY D.LUNG E.SK;MUSLES Q38.PT: W I T H THE S / S OF PANCREATITIS BUT NORMAL AMYLASE,WHAT NEXT DIAGNOSTIC TEST? A.LFT B.CHLESTROL C.GLUCOSE D.AST/ALT Q 4 9 3 0 YRS OLD PERSON AHS STAB WOUND ON RT;SIDE OF CHEST? A.IPSILATERAL LUNG COLLPASE AND IPSILATERAL CHEST WALL SPRING OUT B.IPSILATERAL LUNG COLLAPSE AND CONTRALATERAL CHEST WALL COLLAPSE. C.NO CHANGE IN CHEST WALL D.CONTRALATERAL LUNG COLLAPSE AND IPSELATERAL CHEST WALL COLLAPSE Q40.DURING OPERATION ILIUM REMOVE? A.DEC;ABSORTION OF AMINO ACID B.DEC ABS:OF WATER C,INC ABSORBATION OF FAT D.DEC; IRON ABSORBATION Q41.COMA(DEMAGE OF PART OF BRAIN) A.NUCLEUS CERULOSIS B.RETICULAR FORMATION AREA C.AREA POSTREMA Q42.TRUE HERMOPHADISM? A.XX/XY B.XXY C.XXX D.XYY Q43.AFTER APENDICTOMY,PT;DEVELOPED CHRONIC WOUND ABBCESS AND DRAING SINUS,CONTAING YELLOW GRANULES? A.NOCARDIA B.ACTINOMYCOSIS C.H.PYLORI D.E.COLI E.ENT.HISTOLYTICA Q44.RT:TESTIS LUMPH NODE DRAINGE? A.DEEP INGUINAL NODE B.SUP:MEDIAL GROUP INGUINAL NODE C,PARAORTIC LYMPH NODE Q45.FAT,CHO AND LIPID DEPOSTION BY WHICH HARMONE? A.INSULIN B.GLUCAGON C.CORTISOL. D.GROWTH HARMONE Q46.MICROCYTIC,HYPOCROMIC ANEMIA? A.IRON DEF;ANEMIA B.CHORNIC BLOOD LOSS C.SICKEL CELL ANEMIA D.HEMOLYTIC ANEMIA Q47.CYNOSIS? A.DEC CONTENT O F H B % B.INC:CONCENTRATION OF DEOXY;HEMOGLOBIN C.ANEMIA Q48.UNLOCKING MUSCLES? A.POPLITIUS B.SEMITANDANNIOUS C.SEMIMEMBRANOUS D.SARTORIUS Q49.URETER CONSTRICTION?INCORRECT A.PUJ JUNCTION B.PELVIC BRIUM C.CROSS THE COMON ILLIC ARTERY D.RUN ON PSOS MUSCLE Q50.COMON SITE OF RIB FRACTURE? A.HEAD OF RIB B.ANGLE OF RIB C.BODY OF RIB Q51.INCREASE BLEEDING TIME CAUSE BY?ACCEPT A.HENOCH,SHIL:PUPURA B.CIROHSIS C.DEC PLATELETS D.THROMBOCYTOSIS. Q52.SLOWEST GROWING TUMOR? A.PAPILLARY CA B.MEDULLARY CA C.FOLLICULAR CA Q53.TURNER SYNDROME? A.LONG HIGHT B.MENTAL RETARDATION C.CARRING ANGLE REDUCED D.HIGHT 4.5CM Q54PELVIC DIAPHAGRM FORMED BY MUSCLES EXCEPT? A.COCCYGUS B.PYriformus MUSCLES C.ILOCOCCGUS Q55.STD CAUSES ULCER ACCPET? A.SYPHILIS B.HERPES C.GONOCOCCUS Q56.PT WITH HYPOVOLEMIC SHOKE,TREATED.WHAT WILL BE DECREASE? A.HEART RATE B.urine OUTPUT C.B.P Q57.after 48 HRS OF STRAVATION,WHT WILL BE BREAKDOWN TO PROVIDE ENEGRY A.CHO B.FAT C.PROTIEN Q58.CHRONIC HEMOLYSIS AND INTRACELLULAR PIGMENT ACCUMULATION? A.BILIRUBIN B.HEMOSIDRIN C.BIVIRDIN Q59.BASCILIC VEIN? A.RUN OF RADIAL SIDE OF FORARM B.FORM ON THE PALMER SURFACE OF HAND C60.CONTINUE AS AXILLARY VEIN Q.DRUG LEAST NEPHROTOXIC A.GENAMYCIN B.NEOMYCIN C.CLINDAMYCIN Q61.ANT:DISLOCATION OF SHOULDER JOINT WHICH NERVE DEMAGE? A.AXILLARY B.RADIAL C.MUSCULOCUTANEOS D.MEDIAN E.LOWER BRACHIAL PLUXES Q62.TESTOSTERONE PRODUCED BY ? A.LYDING CELL B.EPIDYDEMUS C.SRTOLI CELL Q63.APEX OF HEART? A.3RTH INTERCOSTAL SPACE. B.TOWARDS RIGHT C.8CM FROM MID LINE Q64.AFLATOXIN B1 CAUSES CA? A.HEPATOCELLUAR CA B.BLADDER CA C.LEUKEMIA D.SKIN CA Q65.BLOOD GROUP CAUSES REACTION? A.B+ B.A+ C.ABD.O+ Q66.MUSCLES DIVIDE SUBMEDIBULAR GLAND? A.STYLOPHARAGEUS MUSCLES B.MYLOID C.STYLOGLOSSIUS Q67.PT:CANNT ABDUCT THE ARM UPTO 30 DEGREE? A.INFRASPINATUS B.SUPRA SPINATUS C.DELTOID Q68.MAJOR INSPIRATORY MUSCLES? A.INTERNAL INTERCOSTAL MUSCLES B.EXT:INTERCOSTAL MUSCLES C.DIAPHGRAM Q69.LEFT SUP:RENAL VEIN DRAIN IN WHICH VEIN? A.RENAL VIEN B.AZYGOS VIEN C.HEMIAZYGOS VIEN D.ACCESSARY AZYGOS VIEN Q70.PARASYMPATHATIC SYS CAUSES ALL OF FOLLWING AXCEPT? A.SPINCTER CONSTRICTION B.DILATE THE PUPIL C.EMPTY BLADDER Q71.PT:LOSS OF ABDUCTION, ADDUCTION OF FINGERBUT NO LOSS OF SENSATION? A.MEDIAN NERVE B.ULNER NERVE. C.RAIDAL NERVE D.DEEP BR;OF ULNER NERVE Q72.MALE W I T H T4N1M1 SERVIVAL RATE LESS 5 0 % , S / S ? A.CACHXIA B.ANEMIA C.ARRTHEYMIA Q77.FUNCTION OF LIMIC SYS AXCEPT? A.MEMORY B.AGGRESTION C.SPINAL REFLAX Q78.STRONGEST LIGAMENT? A.ILIOFEMORAL B.ISCHEO FEMORAL C.DELTA LIGAMENT/MEDIAL LIGAMENT Q79.SHORT GASTRIC ARTERY BR:OF A.SUP:EPIGASRIC ARTERY B.HEPATIC ARTERY C.SPLENIC ARTERY Q80.TYMPANIC MEMBRANE? A.FORMED LATERAL WALL OF TYMPANIC CAVITY B.SUPPLY BY VAGUS AND TRIGEMINAL NERVE C.OPENING OF AUDITARY TUBE IN POST WALL. Q81.HEPARIN A.EMBOLISM BSECRTED BY .BASOPHIL C.DISLODGMENT D.DISOLVMENT OF CLOT Q82.VERBAL COMUNICATION? A.BY PRESENTATION B.INVOLVE IN RESEARCH WORK Q83.37%POST MENUPOSAL WOMEN TAKING HRT,63 WOMEN TAKING NO MEDICATION,HOW YOU SHOW IN CHART A.BAR CHART B.PIE CHART C.PICTORIAL CHART Q84.THYROXINE CAN BE USE OTHER THEN THYROID TREATMENT? A.WEIGHT LOSS B.INCREASE APPEPITTE. C.BMR Q85.THYMUS? A.REGRESS AFTER BIRTH B.FORMED BY 4TH PHARENGEAL ARCHES C.PRODUCED T CELL Q86.GROOVE OF SUBCLAVIN ARTERY? A.1ST RIB B.2ND RIB C.3RD RIB D.CLAVICLE Q87.PIVOT JOINT? A.ATLANTOAXIAL JOINT B.MANDIBULAR JOINT C.ELBOW JOINT Q88.CA REALSE FROM SKETAL MUSCLES FROM? A.SARCOPLASMIC RETICULIUM B.MITOCHONDRIA C.LYSOSOME D.T-TUBULES Q89.PROJECTION FIBERS PRESENT IN? A.CORPUS CALOSUM B.INTERNAL CAPSULES C.RETICULAR FORMATION Q90.PERIPHERAL AND CENTRAL CHEMO RECPTORS BOTH RESPONES TO? A.PCO2 B.PO C.OXYGEN CONCENTRATION D.H+ Q91.ESSENTIONAL AMINO ACID A.TYROSINE B.ALANINE C.ARGININE D.PHENYALANINE Q92.LUMBER PUNCTURE? A.L2 TO L3 B.L4 TO L5 C.POST;SUP;ILIC SPINE Q93.CEREBRAL CORTEX A.RECEIVE NO SENSORY INFORMATION B.PRESENT ONLY IN FRONTAL LOBE C.ESSENTIONAL FOR INITATION OF VOLUENTRY MOVEMENT Q94.WATER ABSORBTION IN PROXIMAL CONVULATED TUBULES? A.55% ABSORED B.ASECNDING LIMB IS PERMIALE FOR WATER C.WATER ABSORED THORUGH FASCILITED DIFFUSION Q95.SUBARCHANID LYER END AT? A.L2 B.S2 C.S5 Q96.DORSAL COLUMN MEDIAL LAMINUSCUS COTAIN? A.FINE TOUCH B.CROUDE TOUCH C.PAIN D.TEMP Q97.GLYCOLYSIS? A.GIVE 1 MOLECULES OF PYRUVATE B.2 MOLECULE OF GLUCOSE Q98.THICK FILAMENT? A.MYOSIN B.ACTIN C.ACTOMYOSIN Q99.ORGAN TRANSPLANT REJECTED IN 10 MIN A.CELL MEDIATED IMUNITY B.ANTIBODY C.PLASMA CELL D.NATURAL KILLER CELL Q100.WHICH FOLLOWING CHEMICAL EXPOSURE CAUSES CA? A.METHYL ALCOHAL B.BENZENE C.CARBON TETRACHLORIDE Q101.CRANIAL NERVES CARRINYING PARASYMPATHATIC FIBERS? A.3,7,8,10 B.3.7.9.10 C.4,7,9,10 Q102.DURING PREGNANCY HICH DRUG SAFEST IN HYPOTHYRIOD PT: A.BETA BLOCKERS B.DIAZEPAM C.PROPYTHIORACIL D.CARBAMAZIPINE Q103.ARCHES OF LEFT LUNG? A.AZYGOS VEIN B.THORACIC DUCT C.ARCH OF AORTA Q104.DIAPHGRAM SUPPLY BY? A.C,3,4,5 B.INTERCOSTAL NERVE C.C 2,3.4 Q105.WHICH DRUG CAUSES CORNEAL OPACITY A.CHLOROQUINE B.ERTHROMYCIN Q106.CAVERNOUS SINUS COTAIN? A.TROCHLER NERVE B.INTERNAL JUGULAR VEIN C.ABDUCENT NERVE Q107.WHICH DRUG INCREASE CYTO P450 SYS A.BARBITURATES B.SULFA DRUGS C.ISONIAZID D.KETOKANAZOLE Q108.EASIEST METHOD TO DIAGNOSED MENINGOCOCCUS? A.CULTURE B.STAINING C.SERUM ANTIGEN Q109.BALCK WATER FEVER CAUSES BY? A.PLAS;MALARIA B.PLAS:FALCIPARUM C.QUATRUN MALARIA Q110.OXYTOCIN AND ADH ORIGENATED BY? A.POST PITUTARY B.HYPOTHALAMUS C.ANT PITUTARY Q111.CEREBLLAR DISORDER? A.UNABLE TO PERFORM VOL;MOVEMENT B.STATIC TREMORS C.INTENSTION TREMORS SURGERY June 2009 SURGERY JUNE 2009. Recalled By Safwan. Thankyou. (C) Means correct. 1 . Gland o f cloquet i s located i n : a. inguinal canal 2. Below is a carcinogen: a. methyl alcohol b. propyl alcohol c. benzidine (C) 3. Incidence is: a. number of new cases in a population (C) 4. Following is correct about femur: a. the head is in line of medial condyle b. the angle between the neck and shaft is average 125 degrees. (C) 5. Subclavian artery passes: a. above the 1st rib (C) b. below the 1st rib 6. Abdominal angina is caused due t o : a. Superior mesenteric artery (C) b. inferior mesenteric artery c. celiac trunk d. abdominal aorta 7. Left Suprarenal vein drains into: a. inferior vena cava b. left renal vein (C) c. phrenic vein d. portal vein 8. Squamous cell carcinoma of urinary bladder is caused due to: a. shistosoma (C) 9. The species which causes carcinoma in urinary bladder is: a. shistosoma mansoni b. schitosoma hematobium (C) 10. Oxidative breakdown of glucose during glycolysis generates: a. 1 pyruvate b. 2 pyruvate (C) c. lactic acid 1 1 . Referred pain to shoulder is via: a. C 1 , 2 b. C2, 3 c. C3, 4 , 5 (C) d. C5, 6 12. Following is not palpable on per rectal examination: a. b. c. d. e. Ureter Prostate Seminal vesicle ?? ?? 13. Spinal cord ends at: a. S2 b. L1-2 (C) c. L3 d. L5 14. Femoral sheath is composed of: a. transversus abdominis fascia b. internal oblique fascia c. iliacus fascia (C) 15. Fracture at neck of fibula: a. injury to common peroneal nerve (C) b. superficial pernoeal nerve c. deep perneal nerve d. tibial nerve 16. Posterior displacement of knee is prevented by: a. Anterior cruciate ligament b. Posterior cruciate ligament (C) c. Tibial collateral ligament d. fibular collateral ligament 17. A patient had cut in the center of wrist during a suicide attempt, he has no abduction or adduction of fingers plus no abduction of the thumb, the injury is t o : a. ulnar nerve b. median nerve c. radial nerve 18. Lymphatic supply of mammary glands is: a. parasternal lymph nodes b. abdominal lymph nodes c. axillary lymph nodes (C) 19. Defecation is because of: a. mass reflex b. myenteric reflex c. S2, 3, 4 (C) d. Parasympathetic reflex 20. Prolonged coma is because of lesion of: a. both frontal lobes lesions (C) b. one frontal lobes lesion c. periaquaductal area lesion d. midbrain lesion 2 1 . Retrograde amnesia: a. can be abolished via prefrontal lobotomy b. appears in amygdala lesions and propranolol therapy 22. Projection fibers of cerebral cortex travel through: a. internal capsule (C) 23. Regarding neural tube which one is correct: a. Involved in CNS development (C) b. Involved in meninges development 24. Primordial germ cells are of: a. ectodermal origin b. mesodermal (C) c. endodermal 25. Urorectal septum divides the cloaca into: a. anus and urinary bladder b. anorectal canal and urogenital sinus (C) 26. ADH and Oxytocin are released from: a. suprachaismatic b. preoptic c. adenohypophysis d. neurohypophysis (C) 27. Termination of subarachnoid space is at: a. L2 b. S2 (C) c. L5 d. S4 28. Following is true about middle meningeal artery: a. Passes through foramen spinosum (C) 29. Mass reflex is elicited b y : a. decortication b. complete section of spinal cord c. lesions of cervical or thoracic spinal cord (C) 30. Medial lemniscus system mediates: a. Pain b. Temperature c. Tickling d. Fine touch (C) 3 1 . What prevents muscles from over-stretching: a. Golgi spindles b. muscle spindles (C) 32. Hamartoma is a: a. neoplastic condition with no malignant potential b. non-neoplastic condition (C) c. completely benign d. malignant 33. In chronic hemolysis, following will deposit in tissues: a. bilirubin (C) b. c. d. e. biliverdin conjugated bilirubin lipofuscin iron 34. Premalignant lesion of mouth is: a. ?? b. ?? 35. Complication of betel nuts chewing is: a. haemorrhage ?? b. ulceration ?? ?? ?? ?? 36. Cleft palate is caused due t o : a. Viral b. carbamazepine c. multifactorial (C) d. antiepileptics 37. A House surgeon is operating on posterior triangle, which structure is at risk: a. spinal portion of accessory nerve (C) b. cranial portion of accessory nerve 38. During surgery on submandibular glands, how will you protect labial branch of facial nerve: a. b. c. d. lift the deep fascia of hyoid bone nerve stimulation studies protecting it as it enters surgical field lifting the blood vessels 39. Enzyme for breakdown of serotonin and Norepinephrine is a. Catechol - o - methyltransferase b. Monoamine oxidase (C) 40. Inducer of drug metabolism is: a. ketoconazole b. phenobarbitone (C) 4 1 . Lesions of cerebellum causes: a. b. c. d. failure of initiation resting tremor rigidity ataxia (C) 42. Essential amino acid is: a. Glycine b. glutamate c. tyrosine d. phenylalanine (C) 43. Slow growing tumour of thyroid is: a. papillary b. follicular c. adenoma 44. Following are features of thyroid tumour: a. ?? 45. Premalignant condition of colon carcinoma: a. b. c. d. inflammatory polyp hyperplastic polyp hamartomatous adenomatous (C) 46. Among the following statements about parotid gland, which is false: a. It has two lobes b. parotid duct arises from deep lobe c. retromandibular vein is related to its inferior pole d. ?? e. ?? 47. Which nerve lies in the cavernous sinus cavity: a. b. c. d. III,V IV, VI VI (C) IX 48. Muscles of facial expression are all of the following except: a. b. c. d. e. orbicularis oris orbicularis oculi buccinator platysma ?? (C) 49. Taste is sensed by all of the following except: a. fungiform b. filliform (C) c. circumvellate 50. Tympanic membrane forms: a. lateral wall of tympanic cavity 5 1 . Thyroid gland is surrounded by which fascia: a. investing b. pretracheal (C) c. prevertebral d. deep 52. Azygous vein: ?? 53. Facilitated diffusion i s : a. energy dependent b. carrier mediated (C) 54. The most significant prepotential is that of: a. atria b. ventricles c. SA node (C) 55. Best mechanism for temperature loss: a. Vasonconstriction b. thirst ?? ?? 56. Acute inflammation: a. transudate b. exudate (C) 57. alpha-adrenergic discharge leads to: a. cardioacceleration b. baroreceptor reflex 58. Which organ gets less than required oxygenation during exercise: a. Skeletal muscle (C) b. c. d. e. kidney skin brain heart 59. H+ ion secretion is i n : a. hyperkalemia b. acidosis (C) c. alkalosis 60. H+ secretion is associated with: a. excretion of bicarbonate b. reabsortion of bicarbonate (C) 6 1 . Central and peripheral regulation of respiration is via: a. b. c. d. PCO2 PO2 pH CO 62. A fisherman came to emergency department with pallor. The most probable cause for this is: a. Diphylobothrium (C) 63. Ileal resection leads t o : a. Increased iron absorption b. Decreased amino acid absorption. 64. Aldosterone production is stimulated via: a. Heart failure b. Decreased perfusion of kidney c. Renin (C) d. Increased Na 65. Another use of thyroid hormone is: a. Weight loss b. Appetite c. Hair loss d. ?? e. ?? 66. Anabolic hormone is: a. Insulin (C) b. Glucagon 67. Adult Polycystic Disease i s : a. Autosomal recessive b. X- linked recessive c. Autosomal dominant (C) d. X linked dominant e. Mitochondial 68. Lymphatic supply of ovary is: a. Internal iliac lymph nodes b. external iliac lymph nodes c. deep inguinal d. lumbar/paraaortic (C) e. sacral 69. Remnant of mullerian factor is: a. Duct of gartner b. Round ligament c. Appendix of testis (C) 70. Tuberculous ulcer has which type of edges: a. Everted Punched out c. Undermined (C) e. Definition of Behaviour sciences i s : a. Within 10 hours acute transplant rejection is due t o : a. Type I b. Influenza virus b. d. Staphylococcus d. Scientific study involving epidemiology. Streptococcus c. c. Elevated 7 1 . e. Following is true about cryptorchidism: a. Scientific study of human behaviour b. Failure of descent of testis from abdomen into scrotal sac. 75. (C) .b. Very common 74. psychology. biostatistics. Widespread resistant to TB drugs b. Antibody-mediated (C) Type I I I Cytotoxic T cells Type V 73. Only causes lung disease c. Sliding d. Severe infection in a post-transplant patient is due t o : a. ethics. Following is true about MAI: a. sociology. Cytomegalovirus (C) 72. Is formed between two Z-discs (C) 79. Tonsils 8 1 . Calcium is released into muscles via: a. Protein synthesis (C) b. Spleen c. RER function is: a. Thymus (C) b. Sarcoplasmic reticulum (C) 80. Lung collapses and chest indraws (C) Lung expands and chest expands lung expands and chest indraws lung expands and chest remains same . Following is true about sarcomere: a. e. Peyers patches d. d. In Pnemothorax following happens: a. Good communication skills are in a person who: a. Post-translational modification 78. RER c. Lung collapses and chest expands b. Mitochondrion b. Is formed between two intercalated discs b. Lymphatic nodules are not present i n : a. Transcription c.76. c. Who does presentations (C) 77. Clostridium difficile 87. Vaccine of Bacillus-Calmette-Guerin is which type of vaccine: a. c. Which one is true about corynebacterium diphtheria: a. Clostridium tetani b. b. Secondary immunity c. Neisseria Streptococcs pneumonia E. Active (C) b. Produces powerful endotoxin 85. Pseudo membranous colitis is caused by: a. Produces powerful exotoxin (C) b. Formaline killed Toxoid vaccine b. Streptococcu pyogenes 86. Clostridium perfringens causes injury via following mechanism: a. Clostridium botulinum c. d.82. Passive 84. Clostridium perfringens d. Live attenuated (C) 83. Vaccine of Bacillus-Calmette-Guerin induces which type of immunity: a. The most common cause of pneumonia in 6 months to 2 years age group i s : a. Heat killed organism c. free radicals . Coli Haemophilus influenza e. Trauma c. Neisseria best diagnosed most easily via: a. Invasion 88. Culture 89. metabolic alkalosis c. HCO3=35. Cause is: a. PCO2=45 (35=45). Gram stain (C) b. hypochromic normochromic 92. Normochromic normocytic anemia b. metabolic acidosis b. MCH=28. Eggs (C) 9 1 . Immobility (C) . partially compensated metabolic alkalosis 93. Ethambutol 90. hypoxia (C) c. it is: a. Normchromic microcytic c. Ocular opacity can be caused by: a. Contraceptive pills b.b. Surgery d. Cyst b.25. Patient is having MCV=78. hypochromic microcytic d. Most common cause of Deep Venous Thrombosis is: a. Patient is vomiting: ph = 7. Hydatid disease caused due to ingestion of: a. Calcitonin works by: a. Osteopenia d. Increasing bone resorption. Pnemococcus c. Ca++ is raised. Iluem d. Pseudomonas (C) 95. Large intenstine 97. PO4 is decreased. Submucous glands are present i n : a. Following is an important component of Counter current exchange mechanism: a. Streptococcus b. Vasa recta (C) b. Proximal tubule c. Most common cause of hospital acquired pnuemonia is: a. Decreasing bone resorption (C) 99. Hyperparathroidism (C) 98. Dudenum (C) c. Pagets disease b. Osteoporosis c. most probable cause is: a. Stomach b. Distal tubule 96.94. b. Patient is having bone pains. True hermaphrodite genotype is: . Q1. SURGERY JUNE 2009 SURGERY PAPER 24 JUNE 2009-07-11 By Dr Sana Memon.UPER LOBE B. Testosterone is produced by: a.TONIC CLONIC SEZIRES .METABOLIC ALKLOSIS Q3.LEFT LOWER LOBE E.EARLIEST SIGN OF ASPIRIN OVER DOSE? A.RT.MAIN SIDE EFFECT OF LIDOCAINE A. The vasculature of kidney is: a. Afferent -> glomerulus -> efferent -> vasa recta Alongwith these there were few questions about premalignant conditions of mouth and a question about which is correct about cholesteatoma. XY b.TINNITUS B..GASTRIC UPSET C. X0 c.WHICH LOBE OF LUNG HAS 2 SEGMENT A.RT MIDDLE LOBE D. XXY (C) 100.a.RT LOWER LOBE C. Leydig cells 1 0 1 .LEFT UPPER LOBE Q2.ARRTHYMIA B. There was a Murad MCQ about pie chart. Sertoli cells (C) b. HEMOGLOBIN Q9.RBC CELL MEMBRANE? A.CAUSES LOCAL HYPOXIA B.B ULCER MARGIN? A. .PROTZOA Q5.INDICATION OF SPLEENECTOMY? A.ACTIN B.EVERETED B.T.I.C.THROMBOCYTOPENIA Q8.AEROBES C.CLOSTRIDIUM DIFFIC? A.G SIX PHOSPATES DIF D.ENTAMOBEA HISTOLYTICA? A.LIMITED TO G.ALPHA TOXIN D.CORTISOL B.SECTRION OF HARMONE INCREASE AS DARKNESS INCREASING? A.AUTOIMMUNE HEMOLYTIC ANEMIA B.NAUSEA VOMITING Q4.INDIFINE D.INDURATED Q6.CONVERSVATIVLY TREAT.MELATONINE D.DOES NOT HAVE CARRIER B.INVERTED C.MYOCIN C.GROWTH HARMONE C.INSULIN Q7.SICKEL CELL ANEMIA C.T C.SPECTRIN D. BRUN PT.OVARIAN ARTERY.RT:CORNOARY ARETY? A. C.B B.DIRECT PASSES THROUGH HESSAL BECH TRIANGLE C.I.CYTOMEGALO VIRUS D.BR:CIRCUMFELX Q15.CA Q13.RING B. D.DIALYSING FLUID COMPOSTION SAME AS PLASMA AXCEPT WHICH ONE HIGH? A.INGUINAL HERNIA? A.FEMORAL ARETY Q12. HYPOTENSION.INFECTION .INDIRECT PASSES ONLY SUPER.HEP.HEB C C.DIRECT LYING ON MEDIAL SIDE OF INF:EPIGASTIC ARTERY Q11.RUN IN POST INTERVENTRICULAR GROOVE D.GLUCOSE B.Q10.SUPPLY BOTH ARTIUM C.UREA C.RISES FROM RT:POST AROTIC SINCUS B.INF:EPIGASTRIC ARTERY B.C B.CONTENT OF INGUINAL HERNIA? A.TACYCARDIA? A.LYMPH OF FUNDUS OF UTERUS.K* D.D.COMMON ORGAINSM INFECTION IN LIVER TRANSPLANT PT: A.INFLUENZA Q14. SECOND HEART SONUD? A.NORMAL B.HIS COUSIN ALSO HAVE SAME PROBLEM? A.Sq: CELL CA C.INC:BONE REABSORBATION Q20.FILLING OF VENTRICLE C.Q16.HYPOCALCEMIA D.HYPERKALEMIA C.TRANSITIONAL CELL CA.HYPOKALEMIA B.CLOSURE OF SEMILUNAER VALVES Q18.IN ALL TYPE OF SHOKE? A.BOY WITH INCREASE BLEEDING TIME AFTER CIRCUMCISION.CALCITONIN? A.COMPENSATION AFTER MILD BLOOD LOSS? A.INCREASE BLOOD CA LEVEL B.CLOUSER OF TRICUSPID VALVES B.P Q22.APTT D.ABSORTION OF CA INTESTINE C.INC.EPITHOID CA Q19.BLADER CA? A.DEC: COMP OF VEIN C.PT Q17.HYPOVOLEMIA .AFTER MULTIPULE BLOOD TRANSFUSION? A. B.INC:ABSORTION FROM RENAL TUBULES D.CLOTTING TIME C.RAISED BUN Q21.BT B.TACYCARDIA B. HEMATOBOIUM D.CIGGRATE SMOKING C.DIYES Q26.INCORRECT? A.B.DEPIGMENTED.BRONCHIAL EPI.CHILD THIN.COMON CAUSE THROMBPHILIBITIES D.APPENDIC EPIPLOIC PRESENT IN? A.BREAST TRUMA Q24.SYPMTOMS AFTER 12 HRS B.GASTRO ESOPHAGEAL JUNCTION D.METAPLASIA.APATHY A.AGGREVATE BY TEMPORAL LOBE TRUMA Q27.TACYCARDIA C.UNCONSIOUS Q23.KASHIKOR C.RETROGRADE AMNEISA? A.DIARREHA Q25.RECTUM C.FRONTAL LOBECTOMY B.CERVIAL JUNCTION Q28.JEUJENIUM .IRREVERSIBLE B.MARASMUS B.FAT EMBOLSIM? A.SHIS.PT:WITH GRANULOMATOUS LESION IN BLADDER DEVELOPED CA.CAUSE? A.DEUDENUM B.SHISTSOMIA MANSONI B. C.80% FATAL C. B.FASICA TRAVERSALIS C.COMMON COMPLICATION OF BETAL NUTS? A.CERVICAL LYMH NODE BIOPSY? A.PHERINIC NERVE.GIT UPSET Q33.CHOLESTATOMA B.D.FASCIA TRANSVERSALIS AND ILISCAP Q31.EPITHEIAL POLYP C.SUBMUCOSAL FIBROSIS B.LOCALLY MALIGNANT TUMOR? A.AXILLARY NERVE D.CRANIAL PORTION OF ACCERY NERVE. Q34.FASICA ILISCA B.SORE THOART C.TIBEAL NERVE .LOWER CORD OF BRACHIAL PLEXUSES E.UNABLE DORSIFLEX AND EVERT FOOT? A.CA.SPINAL PORTION OF ACCESSRY NERVE.COMMON SIGN OF OF AGRUNLOSYTOSIS CAUSES BY ANTIEPILEPTICS? A.COMON PERNEAL NERVE D.WT:LOSS B.NEUROBLASTOMA.SUPER. EPITHILIUM Q30.ILIUM Q29. PERONEAL NERVE B.DURING OPERATION OF POST.DEEP PERONEAL NERVE C. Q32.FEMORAL SHEATH? A. C.BLEEDING D.PT: WITH ROAD TRAFFIC ACCEDIENT. CONTRALATERAL LUNG COLLAPSE AND IPSELATERAL CHEST WALL COLLAPSE .VIT C E.URORECTAL SEPTUM? A.VIT D C.LFT B.IPSILATERAL LUNG COLLPASE AND IPSILATERAL CHEST WALL SPRING OUT B.ESTABLISHED ANTIOXIDANT? A.IPSILATERAL LUNG COLLAPSE AND CONTRALATERAL CHEST WALL COLLAPSE.HEART B.VIT E D.AST/ALT Q4930 YRS OLD PERSON AHS STAB WOUND ON RT.SK.70% OXYGEN EXTRACT IN RESTING CONDITION IN WHICH TISSUE? A.GLUCOSE D.SEPRATE RECTUM AND SIGMOID COLON D.MUSLES Q38.SEPRATE RECTUM AND URINARY BLDER B.CHLESTROL C.SIDE OF CHEST? A.SEPRATE RECTUM AND URETHRA Q37.LUNG E.NO CHANGE IN CHEST WALL D.WHAT NEXT DIAGNOSTIC TEST? A.SEPRATE RECTUM AND UROGENITAL SEPTUM C.VIT B Q36.BRAIN C. C.VIT A B.PT: WITH THE S/S OF PANCREATITIS BUT NORMAL AMYLASE.Q35.KIDNEY D. ENT.DEC.INSULIN B.NUCLEUS CERULOSIS B.XXX D.CHO AND LIPID DEPOSTION BY WHICH HARMONE? A.CONTAING YELLOW GRANULES? A.PYLORI D.SUP:MEDIAL GROUP INGUINAL NODE C. IRON ABSORBATION Q41.CORTISOL.AREA POSTREMA Q42.INC ABSORBATION OF FAT D.DEC.DEC ABS:OF WATER C.XXY C.FAT.COMA(DEMAGE OF PART OF BRAIN) A.GROWTH HARMONE .RETICULAR FORMATION AREA C.NOCARDIA B.HISTOLYTICA Q44.DEVELOPED CHRONIC WOUND ABBCESS AND DRAING SINUS.Q40.PARAORTIC LYMPH NODE Q45.GLUCAGON C.DURING OPERATION ILIUM REMOVE? A.TRUE HERMOPHADISM? A.RT:TESTIS LUMPH NODE DRAINGE? A. D.XX/XY B.PT.H.COLI E.DEEP INGUINAL NODE B.AFTER APENDICTOMY.ABSORTION OF AMINO ACID B.E.ACTINOMYCOSIS C.XYY Q43. ANEMIA B.SLOWEST GROWING TUMOR? A.INC:CONCENTRATION OF DEOXY.PELVIC BRIUM C.PUJ JUNCTION B.SEMITANDANNIOUS C.CYNOSIS? A.MICROCYTIC.Q46.SHIL:PUPURA B.HEMOLYTIC ANEMIA Q47.POPLITIUS B.THROMBOCYTOSIS. Q52.BODY OF RIB Q51.DEC CONTENT OF HB% B.DEC PLATELETS D.HEMOGLOBIN C.SEMIMEMBRANOUS D.HEAD OF RIB B.CHORNIC BLOOD LOSS C.CROSS THE COMON ILLIC ARTERY D.HENOCH.UNLOCKING MUSCLES? A.RUN ON PSOS MUSCLE Q50.ANGLE OF RIB C.COMON SITE OF RIB FRACTURE? A.SARTORIUS Q49.CIROHSIS C.HYPOCROMIC ANEMIA? A.SICKEL CELL ANEMIA D.INCREASE BLEEDING TIME CAUSE BY?ACCEPT A.IRON DEF.URETER CONSTRICTION?INCORRECT A.ANEMIA Q48.PAPILLARY CA . HEART RATE B.PROTIEN Q58.PYriformus MUSCLES C.STD CAUSES ULCER ACCPET? A.after 48 HRS OF STRAVATION.HIGHT 4.TREATED.B.FAT C.LONG HIGHT B.BIVIRDIN .PT WITH HYPOVOLEMIC SHOKE.BILIRUBIN B.FOLLICULAR CA Q53.HEMOSIDRIN C.CARRING ANGLE REDUCED D.HERPES C.SYPHILIS B.P Q57.ILOCOCCGUS Q55.5CM Q54PELVIC DIAPHAGRM FORMED BY MUSCLES EXCEPT? A.WHT WILL BE BREAKDOWN TO PROVIDE ENEGRY A.urine OUTPUT C.WHAT WILL BE DECREASE? A.CHRONIC HEMOLYSIS AND INTRACELLULAR PIGMENT ACCUMULATION? A.COCCYGUS B.MEDULLARY CA C.GONOCOCCUS Q56.TURNER SYNDROME? A.B.MENTAL RETARDATION C.CHO B. APEX OF HEART? A.Q59. B.BASCILIC VEIN? A.RUN OF RADIAL SIDE OF FORARM B.NEOMYCIN C.LOWER BRACHIAL PLUXES Q62.B+ B.CONTINUE AS AXILLARY VEIN Q.A+ .3RTH INTERCOSTAL SPACE.LEUKEMIA D.BLADDER CA C.SRTOLI CELL Q63.HEPATOCELLUAR CA B.MEDIAN E.8CM FROM MID LINE Q64.LYDING CELL B.MUSCULOCUTANEOS D.BLOOD GROUP CAUSES REACTION? A.AFLATOXIN B1 CAUSES CA? A.DRUG LEAST NEPHROTOXIC A.TESTOSTERONE PRODUCED BY ? A.RADIAL C.AXILLARY B.GENAMYCIN B.CLINDAMYCIN Q61.SKIN CA Q65.TOWARDS RIGHT C.FORM ON THE PALMER SURFACE OF HAND C60.EPIDYDEMUS C.ANT:DISLOCATION OF SHOULDER JOINT WHICH NERVE DEMAGE? A. PT:CANNT ABDUCT THE ARM UPTO 30 DEGREE? A.C.DILATE THE PUPIL C.RENAL VIEN B.ABD.STYLOGLOSSIUS Q67.MUSCLES DIVIDE SUBMEDIBULAR GLAND? A.RAIDAL NERVE D. ADDUCTION OF FINGERBUT NO LOSS OF SENSATION? A.PT:LOSS OF ABDUCTION.SUPRA SPINATUS C.DEEP BR.O+ Q66.SPINCTER CONSTRICTION B.OF ULNER NERVE .HEMIAZYGOS VIEN D.DIAPHGRAM Q69.EXT:INTERCOSTAL MUSCLES C.INTERNAL INTERCOSTAL MUSCLES B.MAJOR INSPIRATORY MUSCLES? A.DELTOID Q68.ACCESSARY AZYGOS VIEN Q70.AZYGOS VIEN C.ULNER NERVE.EMPTY BLADDER Q71.LEFT SUP:RENAL VEIN DRAIN IN WHICH VEIN? A.STYLOPHARAGEUS MUSCLES B.INFRASPINATUS B. C.MYLOID C.MEDIAN NERVE B.PARASYMPATHATIC SYS CAUSES ALL OF FOLLWING AXCEPT? A. 37%POST MENUPOSAL WOMEN TAKING HRT.CACHXIA B.OPENING OF AUDITARY TUBE IN POST WALL.HEPATIC ARTERY C.63 WOMEN TAKING NO .SUPPLY BY VAGUS AND TRIGEMINAL NERVE C.ISCHEO FEMORAL C.HEPARIN A.ANEMIA C.MEMORY B.BY PRESENTATION B.STRONGEST LIGAMENT? A. Q81.SPLENIC ARTERY Q80.S/S? A.MALE WITH T4N1M1 SERVIVAL RATE LESS 50%.DISLODGMENT D.BASOPHIL C.VERBAL COMUNICATION? A.INVOLVE IN RESEARCH WORK Q83.TYMPANIC MEMBRANE? A.ILIOFEMORAL B.SPINAL REFLAX Q78.AGGRESTION C.EMBOLISM BSECRTED BY .FORMED LATERAL WALL OF TYMPANIC CAVITY B.SHORT GASTRIC ARTERY BR:OF A.DISOLVMENT OF CLOT Q82.Q72.DELTA LIGAMENT/MEDIAL LIGAMENT Q79.SUP:EPIGASRIC ARTERY B.ARRTHEYMIA Q77.FUNCTION OF LIMIC SYS AXCEPT? A. LYSOSOME D.PRODUCED T CELL Q86.1ST RIB B.FORMED BY 4TH PHARENGEAL ARCHES C.PIVOT JOINT? A.ATLANTOAXIAL JOINT B.T-TUBULES Q89.PROJECTION FIBERS PRESENT IN? A.INCREASE APPEPITTE.PICTORIAL CHART Q84.MANDIBULAR JOINT C.MEDICATION.CA REALSE FROM SKETAL MUSCLES FROM? A.3RD RIB D.WEIGHT LOSS B.CORPUS CALOSUM B.CLAVICLE Q87.BAR CHART B.HOW YOU SHOW IN CHART A.ELBOW JOINT Q88.THYMUS? A.2ND RIB C.THYROXINE CAN BE USE OTHER THEN THYROID TREATMENT? A.PIE CHART C.REGRESS AFTER BIRTH B.RETICULAR FORMATION .BMR Q85. C.SARCOPLASMIC RETICULIUM B.GROOVE OF SUBCLAVIN ARTERY? A.INTERNAL CAPSULES C.MITOCHONDRIA C. TYROSINE B.ESSENTIONAL FOR INITATION OF VOLUENTRY MOVEMENT Q94.RECEIVE NO SENSORY INFORMATION B.55% ABSORED B.PERIPHERAL AND CENTRAL CHEMO RECPTORS BOTH RESPONES TO? A.PRESENT ONLY IN FRONTAL LOBE C.ESSENTIONAL AMINO ACID A.L4 TO L5 C.POST.Q90.H+ Q91.ALANINE C.LUMBER PUNCTURE? A.PHENYALANINE Q92.ILIC SPINE Q93.PO C.FINE TOUCH B.SUBARCHANID LYER END AT? A.S5 Q96.OXYGEN CONCENTRATION D.S2 C.PCO2 B.WATER ABSORED THORUGH FASCILITED DIFFUSION Q95.WATER ABSORBTION IN PROXIMAL CONVULATED TUBULES? A.CROUDE TOUCH .SUP.ASECNDING LIMB IS PERMIALE FOR WATER C.L2 B.DORSAL COLUMN MEDIAL LAMINUSCUS COTAIN? A.L2 TO L3 B.CEREBRAL CORTEX A.ARGININE D. GLYCOLYSIS? A.METHYL ALCOHAL B.GIVE 1 MOLECULES OF PYRUVATE B.NATURAL KILLER CELL Q100.ANTIBODY C.CARBAMAZIPINE Q103.2 MOLECULE OF GLUCOSE Q98.THORACIC DUCT .4.3.DURING PREGNANCY HICH DRUG SAFEST IN HYPOTHYRIOD PT: A.9.CELL MEDIATED IMUNITY B.CARBON TETRACHLORIDE Q101.7.WHICH FOLLOWING CHEMICAL EXPOSURE CAUSES CA? A.9.ACTIN C.C.MYOSIN B.7.BETA BLOCKERS B.BENZENE C.PROPYTHIORACIL D.7.ORGAN TRANSPLANT REJECTED IN 10 MIN A.CRANIAL NERVES CARRINYING PARASYMPATHATIC FIBERS? A.ARCHES OF LEFT LUNG? A.TEMP Q97.THICK FILAMENT? A.3.10 C.PLASMA CELL D.ACTOMYOSIN Q99.10 Q102.AZYGOS VEIN B.10 B.8.PAIN D.DIAZEPAM C. 4.STAINING C.CAVERNOUS SINUS COTAIN? A.SERUM ANTIGEN Q109.POST PITUTARY B.ERTHROMYCIN Q106.CHLOROQUINE B.MALARIA B.BALCK WATER FEVER CAUSES BY? A.TROCHLER NERVE B.KETOKANAZOLE Q108.QUATRUN MALARIA Q110.C 2.OXYTOCIN AND ADH ORIGENATED BY? A.3.4 Q105.ISONIAZID D.CULTURE B.3.BARBITURATES B.5 B.HYPOTHALAMUS .INTERNAL JUGULAR VEIN C.PLAS.PLAS:FALCIPARUM C.WHICH DRUG INCREASE CYTO P450 SYS A.ARCH OF AORTA Q104.WHICH DRUG CAUSES CORNEAL OPACITY A.C.INTERCOSTAL NERVE C.ABDUCENT NERVE Q107.EASIEST METHOD TO DIAGNOSED MENINGOCOCCUS? A.DIAPHGRAM SUPPLY BY? A.C.SULFA DRUGS C. Following is correct about femur: a.INTENSTION TREMORS Qby drwatson » Thu Jul 22.UNABLE TO PERFORM VOL. the angle between the neck and shaft is average 125 degrees. inguinal canal 2. 1. number of new cases in a population (C) 4. (C) 5. propyl alcohol c. benzidine (C) 3. Below is a carcinogen: a. the head is in line of medial condyle b.CEREBLLAR DISORDER? A.C. methyl alcohol b.ANT PITUTARY Q111. Incidence is: a.STATIC TREMORS C.MOVEMENT B. Gland of cloquet is located in: a. 2010 4:26 pm • FCPS SURGERY June 2009 Paper (C) Means correct. Subclavian artery passes: . shistosoma (C) 9. inferior mesenteric artery c. inferior vena cava b. 2 . below the 1st rib 6. The species which causes carcinoma in urinary bladder is: a. Superior mesenteric artery (C) b. shistosoma mansoni b. 2 pyruvate (C) c. schitosoma hematobium (C) 10. Oxidative breakdown of glucose during glycolysis generates: a. left renal vein (C) c. Squamous cell carcinoma of urinary bladder is caused due to: a. 1 pyruvate b. phrenic vein d.a. portal vein 8. above the 1st rib (C) b. Left Suprarenal vein drains into: a. lactic acid 1 1 . Abdominal angina is caused due t o : a. C 1 . Referred pain to shoulder is via: a. abdominal aorta 7. celiac trunk d. L3 d. Fracture at neck of fibula: a. C2. 6 12. C5. 4 . L1-2 (C) c. transversus abdominis fascia b. Femoral sheath is composed of: a. Prostate Seminal vesicle ?? ?? Hidden content: UNLOCKED 13. e. iliacus fascia (C) 15. d. injury to common peroneal nerve (C) b. superficial pernoeal nerve c. Following is not palpable on per rectal examination: a. L5 14. deep perneal nerve d. C3. 5 (C) d. Posterior displacement of knee is prevented by: a. 3 c.b. Ureter b. Anterior cruciate ligament . S2 b. c. internal oblique fascia c. tibial nerve 16. Spinal cord ends at: a. one frontal lobes lesion c. radial nerve 18. Prolonged coma is because of lesion of: a. median nerve c. fibular collateral ligament 17. 4 (C) d. Posterior cruciate ligament (C) c. Parasympathetic reflex 20. axillary lymph nodes (C) 19. can be abolished via prefrontal lobotomy b. appears in amygdala lesions and propranolol therapy . periaquaductal area lesion d. Defecation is because of: a. A patient had cut in the center of wrist during a suicide attempt. S2. he has no abduction or adduction of fingers plus no abduction of the thumb. Lymphatic supply of mammary glands is: a. ulnar nerve b. myenteric reflex c. Retrograde amnesia: a. midbrain lesion 2 1 . both frontal lobes lesions (C) b. Tibial collateral ligament d. the injury is t o : a. parasternal lymph nodes b. abdominal lymph nodes c. 3.b. mass reflex b. Involved in meninges development 24. Urorectal septum divides the cloaca into: a. Termination of subarachnoid space is at: a. S2 (C) c. neurohypophysis (C) 27. L2 b. anorectal canal and urogenital sinus (C) 26. anus and urinary bladder b. L5 d. S4 28. preoptic c. Projection fibers of cerebral cortex travel through: a. internal capsule (C) 23. Primordial germ cells are of: a. endodermal 25.22. mesodermal (C) c. ectodermal origin b. adenohypophysis d. Regarding neural tube which one is correct: a. ADH and Oxytocin are released from: a. Involved in CNS development (C) b. Following is true about middle meningeal artery: . suprachaismatic b. Passes through foramen spinosum (C) 29. neoplastic condition with no malignant potential b. iron 34. In chronic hemolysis.a. non-neoplastic condition (C) c. biliverdin c. Pain b. malignant 33. conjugated bilirubin d. Hamartoma is a: a. Fine touch (C) 3 1 . decortication b. muscle spindles (C) 32. Tickling d. Premalignant lesion of mouth is: . bilirubin (C) b. What prevents muscles from over-stretching: a. completely benign d. Golgi spindles b. following will deposit in tissues: a. complete section of spinal cord c. Mass reflex is elicited b y : a. lipofuscin e. Medial lemniscus system mediates: a. lesions of cervical or thoracic spinal cord (C) 30. Temperature c. which structure is at risk: a. cranial portion of accessory nerve 38. A House surgeon is operating on posterior triangle. ?? 35. Catechol .methyltransferase b. antiepileptics 37. Complication of betel nuts chewing is: a. ulceration ?? ?? ?? ?? 36.o . ?? b. haemorrhage ?? b. nerve stimulation studies c. carbamazepine c.a. lifting the blood vessels 39. Monoamine oxidase (C) . Cleft palate is caused due t o : a. Viral b. spinal portion of accessory nerve (C) b. During surgery on submandibular glands. Enzyme for breakdown of serotonin and Norepinephrine is a. protecting it as it enters surgical field d. multifactorial (C) d. lift the deep fascia of hyoid bone b. how will you protect labial branch of facial nerve: a. Among the following statements about parotid gland. adenoma 44. phenobarbitone (C) 4 1 . follicular c. Following are features of thyroid tumour: a. Premalignant condition of colon carcinoma: a. Lesions of cerebellum causes: a. inflammatory polyp hyperplastic polyp hamartomatous adenomatous (C) 46. Essential amino acid is: a. ketoconazole b.40. papillary b. failure of initiation b. Inducer of drug metabolism is: a. ataxia (C) 42. c. phenylalanine (C) 43. d. resting tremor c. Glycine b. ?? 45. Slow growing tumour of thyroid is: a. rigidity d. tyrosine d. which is false: . b. glutamate c. Tympanic membrane forms: a. III. pretracheal (C) c. b. lateral wall of tympanic cavity 5 1 . orbicularis oris orbicularis oculi buccinator platysma ?? (C) 49. It has two lobes parotid duct arises from deep lobe retromandibular vein is related to its inferior pole ?? ?? 47. filliform (C) c. VI VI (C) IX 48. Thyroid gland is surrounded by which fascia: a. d. e. fungiform b. c. Which nerve lies in the cavernous sinus cavity: a. circumvellate 50. Taste is sensed by all of the following except: a.V IV.a. d. d. c. b. b. investing b. Muscles of facial expression are all of the following except: a. prevertebral . c. e. energy dependent b. carrier mediated (C) 54. baroreceptor reflex 58. Acute inflammation: a.d. atria b. Best mechanism for temperature loss: a. deep 52. Which organ gets less than required oxygenation during exercise: . The most significant prepotential is that of: a. ventricles c. cardioacceleration b. transudate b. SA node (C) 55. Azygous vein: ?? 53. alpha-adrenergic discharge leads to: a. exudate (C) 57. thirst ?? ?? 56. Vasonconstriction b. Facilitated diffusion i s : a. Central and peripheral regulation of respiration is via: a. Increased iron absorption b. skin d. PO2 c. alkalosis 60. brain e. acidosis (C) c. Skeletal muscle (C) b. kidney c. Ileal resection leads t o : a.a. PCO2 b. pH d. reabsortion of bicarbonate (C) 6 1 . H+ ion secretion is i n : a. CO 62. H+ secretion is associated with: a. Aldosterone production is stimulated via: a. Heart failure b. 64. excretion of bicarbonate b. Diphylobothrium (C) 63. The most probable cause for this is: a. Decreased perfusion of kidney . hyperkalemia b. heart 59. Decreased amino acid absorption. A fisherman came to emergency department with pallor. X. Glucagon 67. Autosomal recessive b. external iliac lymph nodes c. Insulin (C) b. Remnant of mullerian factor is: a. Appetite Hair loss ?? ?? 66.linked recessive Autosomal dominant (C) X linked dominant Mitochondial 68. d. Round ligament .c. Adult Polycystic Disease i s : a. c. Weight loss b. e. lumbar/paraaortic (C) e. e. Increased Na 65. Anabolic hormone is: a. deep inguinal d. Duct of gartner b. sacral 69. Internal iliac lymph nodes b. c. Renin (C) d. Another use of thyroid hormone is: a. d. Lymphatic supply of ovary is: a. Within 10 hours acute transplant rejection is due t o : a. d. Punched out c. Widespread resistant to TB drugs b. Elevated 7 1 . Cytomegalovirus (C) 72. Staphylococcus d. Only causes lung disease c. Very common 74. Sliding d. c. Undermined (C) e. Antibody-mediated (C) Type I I I Cytotoxic T cells Type V 73. Severe infection in a post-transplant patient is due t o : a. Everted b. Following is true about cryptorchidism: a.c. Streptococcus c. Type I b. Influenza virus b. Appendix of testis (C) 70. Tuberculous ulcer has which type of edges: a. Failure of descent of testis from abdomen into scrotal sac. 75. Following is true about MAI: a. Definition of Behaviour sciences i s : . e. psychology. Scientific study of human behaviour b. (C) 76. Protein synthesis (C) b. Lung collapses and chest expands . biostatistics. sociology. Transcription c. Post-translational modification 78. Scientific study involving epidemiology. Who does presentations (C) 77. Is formed between two intercalated discs b. Thymus (C) b. Is formed between two Z-discs (C) 79. Peyers patches d. Mitochondrion b.a. RER function is: a. Sarcoplasmic reticulum (C) 80. Spleen c. Calcium is released into muscles via: a. Following is true about sarcomere: a. Tonsils 8 1 . RER c. In Pnemothorax following happens: a. Good communication skills are in a person who: a. Lymphatic nodules are not present i n : a. ethics. E. Produces powerful endotoxin 85. Vaccine of Bacillus-Calmette-Guerin is which type of vaccine: a. Live attenuated (C) 83. Active (C) b. Clostridium tetani b.b. The most common cause of pneumonia in 6 months to 2 years age group i s : a. Neisseria b. Streptococcs pneumonia c. Passive 84. Heat killed organism c. Secondary immunity c. Clostridium perfringens . Streptococcu pyogenes 86. Lung expands and chest expands d. Vaccine of Bacillus-Calmette-Guerin induces which type of immunity: a. Which one is true about corynebacterium diphtheria: a. Formaline killed Toxoid vaccine b. Produces powerful exotoxin (C) b. Lung collapses and chest indraws (C) c. Clostridium botulinum c. lung expands and chest indraws e. Pseudo membranous colitis is caused by: a. Coli d. Haemophilus influenza e. lung expands and chest remains same 82. Culture 89. Hydatid disease caused due to ingestion of: a. Neisseria best diagnosed most easily via: a. Clostridium difficile 87. Cyst b. Cause is: a. free radicals b. PCO2=45 (35=45). hypoxia (C) c. partially compensated metabolic alkalosis 93. metabolic acidosis b. it is: a. MCH=28. Patient is vomiting: ph = 7.25. Clostridium perfringens causes injury via following mechanism: a. HCO3=35. hypochromic microcytic d. hypochromic normochromic 92. Gram stain (C) b. Eggs (C) 9 1 . Ocular opacity can be caused by: a. Most common cause of Deep Venous Thrombosis is: . Normchromic microcytic c. Invasion 88.d. metabolic alkalosis c. Patient is having MCV=78. Normochromic normocytic anemia b. Ethambutol 90. Calcitonin works by: . Pagets disease Osteoporosis Osteopenia Hyperparathroidism (C) 98. Following is an important component of Counter current exchange mechanism: a. Vasa recta (C) b. PO4 is decreased. Iluem d. Dudenum (C) c. c. Distal tubule 96. Proximal tubule c. Trauma c. Immobility (C) 94. Surgery d. Large intenstine 97. Most common cause of hospital acquired pnuemonia is: a. Pnemococcus c. Pseudomonas (C) 95. b. Contraceptive pills b. Ca++ is raised. most probable cause is: a. Stomach b.a. d. Submucous glands are present i n : a. Patient is having bone pains. Streptococcus b. a) Vit B 12 4.D (Semipalatinsk. M. taste sent co taste centre by a) Chemoreceptor . b. Regulation of PO2 and breathing rate is controlled by a) Carotid bodies 3. XY b. Hot souses on tongue. Afferent -> glomerulus -> efferent -> vasa recta SURGERY October 2008 Dr Qasim Hussain. Decreasing bone resorption (C) 99. M. The vasculature of kidney is: a.Muhammad Irfan. Lithargic patient with difficulty of walking have Hb 5. OCT 2008 1.a. Kazakhstan) FCPS -1 SURGERY 15.D (Semipalatinsk. Testosterone is produced by: a. Sertoli cells (C) b. Kazakhstan) Dr. Leydig cells 1 0 1 . XXY (C) 100. Pallor. Increasing bone resorption. TLC 3500cm. True hermaphrodite genotype is: a. A boy with lean body mass with fracture of arm osteopenia a) VIT D deficiency 2. X0 c. Interstitial fluid is a) 1/3 of TBW 13) Not includes in enzymes of necrosis a) Protease b)Catalase c) Hyaluronic acid 14. seen a) Caseous necrosis 15. Which blood group have no agglutinin? a) AB+ 10. In lymph nodes. Right Gastric artery is a branch of a) Hepatic artery 7) GFR is increased by increase in resistance a) Efferent arterioles 8. Iron deficiency anemia best diagnosed? a)Increase TIBC b) Serum ferritin . Which is not a cause of anaerobic? a) Granulation formation 6. Atypical Tuberculosis a) Does not responds to common Anti TB drugs 9.5. Essential fatty acid is a) Leniolenic acid 1 1 . If large amount of Dextrose water is given to a patient a) Decrease ADH 1 2 . Bones of Ear a) Amplify sound waves 17) Immune mechanism of Liver lies in a) Kuffer cells 18. SA node is located on a) Above the sulcus terminals 26. Heat Loss Mechanism lies in a) Posterior hypothalamus 2 1 . Insulin deficiency a) Ketogenesis in liver 23. Death from food poisoning occur with a) C. Smell fibers from nose a) Does not reach Thalamus 20. SA node is supplied by a) RCA 25. Lymph nodes of lower lip drain into a) Submental and submandibular lymph nodes 22. in Liver Cirrhosis a) Increase PT 19. Botulism .16. Subcapsular afferent lymph is in a) Lymph nodes 24. 10 30. Helps in diagnosis of colonic carcinoma a) CEA 33. 7. Ansa cervicalis is .000 Glucose 3. What causes depolarization? a) Na+ influx 32. LP shows in CSF neutrophils 40.27.9. Which is a characteristic of Cancer? a) Pleomorphism---ANSWER b) Increase mitotic figure 35. Increase of Platelets aggregation is b y : a)Thromboxane A2 36. Contusion to Lateral border of Fibula result in a) Common peroneal nerve damage 3 1 .5mg a) Acute Bacterial meningitis 37. A 20 year old boy with Headache and photobphobia With high grade fever. Narrowest part of male urethra is a) External meatus 29. Pretracheal fascia completely covers a) Thyroid 28. Which cranial nerves are parasympathetic? ANS: CN 3. What is called juvenile polyp? a) Hamartomatous 34. Patient with some pituitary tumor has infertility. Cause of lymphoid tumor in HIV patient is a) EBV 47. Biopsy will show a) Normal hepatic architecture 48. 2. a) Decreased bile salts reabsorption 40. a patient developed HAV completely recovered.a) Hpoglossal nerve and C 1 . Wound heals poorly if absent a) VIT C in diet 4 1 . 3 38) Mitral valve lies between a) Left atrium and left ventricle 39. If resected 10inch ilium. possibly due to a) Increase level of prolactin in serum 46. High level of creatinine is seen in a) ESRD 42. Neurogenic shock is a) Vasomotor injury 43. In Down syndrome seen a) Short stature 44. Melanocytes are derived from . XXY is a) Klinefilter syndrome 45. Neurohypophysis contains a) Pititicytes 49. A patient has some gastric problem a) Iron Deficiency anemia 63. What is buried in lateral sulcus? a) Insula 57. Complements are decreased in a) SLE 60. Anterior Cardiac veins into . A patient can utter few words. If infection of abdomen spread to retroperitonium. IgE is present on a) Basophils 62. can reach to colon? a) Descending colon 6 1 .a) Neural crest cells 50. In adults. ADH works on a) Collecting ducts 52. the spinal cord ends in a) Below L1 54. Erythropoietin is produced by a) peri tubular capillaries 5 1 . Malignant malaria is caused by a) Falciparam malaria 55. Where is lesion? a) Broca's area 58. Dorsal column damage leads to loss of a) Proprioception 56. Spinal cord is supplied by a) Vertebral artery 59. Posterior to Urinary bladder is a) Fascia Donovillaris 53. If Gastric mucosal permeability is increased a) H+ comes out in lumen of stomach and damage mucosal barrier 64. S3 is produced by a) Rapid filling of left ventricle 85. the cause of hepatitis is a) CMV 8 1 . Medially rotate and abduct the thigh is a) Gluteus Medius and and Gluteus Minimus 78. END diastolic volume i s a) 120ml 82. Structure not related to Right kidney is a) Descending colon 89) Temporal arteritis best diagnosed by a) Temporal artery biopsy 90. Cause of edema is a) Decreased colloidal osmotic pressure 87. Liver transplant. 1st heart sound is best listened on a) On Mid clavicular line. Most common cause of malignancy is a) Asbestosis 66. Source of folic Acid is a) Vegetables 79. most probably due to a) CA breast 83. in bronchial Asthma. Palpable anterior Auxiliary lymph nodes.a) Right Atrium 65. Patient with bronchial asthma has a) Decreased FEV1 88. END arteries are seen in a) Spleen 77. Neurovascular bundle is in a) inner and innermost muscles 80. Aneurysm of abdominal aorta compresses a) Thoracic duct . 5th ICS 84. seen a) Eosinophilia 67. Berry aneurysms are mostly found in a) cerebrum 86. Remittent of paramesonephric duct in male is a) Appendix testis 93. Which structure passes through shoulder joint cavity? a)tendon of long head of biceps 100. Most important muscle of inspiration is a) Diaphragm 102. During fasting. young patient came with fracture of arm. Myasthenia gravis improves on taking a) Anti cholinesterase drugs 98. Myelination of peripheral nerves are by a) Schwann cells 97.9 1 . Most sensitive to radiotherapy a) Lymphocytes 103 What is branch of ICA? ANS: Ophthalmic artery 104. Hydrocephalas occurs blockage of a) Aqueduct salvias 99. Achalasia is due to a) Absent ganglions in LES 95. Impaired metabolism of purine cause in urine a) Increased uric acid 1 0 1 . Direct inguinal hernia in a) Medial to inferior epigestric artery\ 94. in EAC.P. Squamous cell is cervix is an example of ANS: Metaplasia 1 0 5 : Thoracic duct drain all except ANS: cerebrum 106: Increase blood in heart chamber is ANS: Isovolumatric relaxation 1 0 7 : If Clavicle is fractured. circumference of mid arm is decreased after one month due to a) Decreased protein of body 96. mostly due to a) Vit. On attempt to measure the B. its lateral part is pulled down b y : . tetany is seen. D deficiency 92. damaged ANS: Lavator Ani muscles 115: Broadmann's area 312 is also called ANS: somatosensory area 116: Surfactant is produced by? ANS: Pneumocyte II 117: Sigmoid sinus is continuous with ANS: Internal carotid artery 118: In shock. which hormone? ANS: Thyroid hormone 109: Diarrhea improves on fasting is ANS: Osmotic 1 1 0 : Source of energy to brain during fasting: ANS: Amino Acid 1 1 1 : Which of the following is not antiseptic? ANS: Acetyl Salicylic Acid 112: Aging determined by bones? ANS: Angle of mandible is less obtuse 113: Lacrimal gland is supplied by? ANS: Ptyrigopalatine ganglion 114: During Episiotomy. what will decrease? ANS: Venous compliance 119: Is a sign upper motor neuron lesion? ANS: Babinsky sign 120: If numerous epitheliod cells are surrounded by lymphocytes. called ANS: Caseous necrosis (Granuloma) 1 2 1 : Fracture of medial epicondyl of humerous. For uncoupled oxidative phosphorylation. on hand lost sensations? ANS: medial 1/3 of hand palm.ANS: Subclavius muscle 108. and dorsal 1/3 of hand 122: Winging of scapula is due to damage of? ANS: Serratus anterior muscle 123: Right adrenal gland is ANS: Pyramidal shape 124: Submucosal gland is in ? ANS: Duodenum 125: Which is true? . ANS: Right brochus is longer 126: In bronchial asthma? ANS: increased breathing by stimulating beta –adrenergic 1 2 7 : inferior rectal artery is a branch of ANS: internal pudendal artery 128: Is not a carcinogen? ANS: Cyclophosphamide 129: Microsomal metabolism is ANS: Liver 1 3 0 : Half life is ANS: time to metabolism of half of drug 1 3 1 : Most of the drug is metabolize in ANS: Liver 132 Diverticulosis occurs: ANS: sigmoid colon 133: Stimulation of RBC from bone marrow by? ANS: Erythropoietin 1 3 4 : Occult blood in stool is in ANS: Iron deficiency anemia 135: Partial gastrectomy ANS: Pernicious anemia 136: Fat store is increased by ANS: insulin 137: In stress which hormone is released? ANS: Cortisol 138: Temperature regulator centre is in? ANS: Hypothalamus 1 3 9 : In heart ANS: Right atrium lies anterior to left atrium 140: In hypovolumic shock ANS: Rapid intravenous fluids 1 4 1 : Not seen in DIC ANS: Thrombocytosis 142: Best method of diagnosing ectopic pregnancy ANS: Laparoscopy 1 4 3 : superior parathyroid is . Widal test is positive? Don’t Know A) 1: 80 for O antigen B) 1:180 for O antigen C ) 1: 100 for O antigen D ) 1:180 for O and H antigen E) 1:80 for H antigen 155. anterior fibers of internal oblique. but if supported up to 30 degree. Actin physically binds with A) Troponin . Rectus muscle Posterior fibers of internal oblique.ANS: supplied by superior thyroid artery 144: The supra renal gland ANS: The supra renal gland is supplied artery which is direct branch of aorta 145: Gas gangrene is not caused by ANS: Bacterial endocarditis 146: permeability of a substance depends on ANS: Total crosses sectional area 147: A patient can't abduct arm. can do. A patient developed type I anaphylactic reaction. What is? ANS: Supraspinatus muscle 148: incision between umbilicus and ribs passes ANS: external oblique. Treatment of choice? ANS: Adrenaline 1 5 3 : Amputated leg is the result of ANS: Amniotic bands 154. transverse muscle 149: Lymph from posterior 1/3 drain into ANS: deep superior cervical lymph nodes 150) Negative Montoux test is seen in a) Steroid therapy b) Immunosuppressive Therapy 151) Permeability of a substance depends on ANSWER a) Total cross sectional area-b) Carrier c) Receptors 152. Insertion of 1st lumbrical 160. tip of conical ceacum has appendix attached—ok? 165. Appendicectomy can damage to Inferior epigestric artery . At Mac burny point located Tip of appendix Base of appendix In infants. Contraindication for platelet a. mansonei b) S. Colonarisa ----ok 1 6 1 . InG is in Largent quantity 164. DIC c. Cholangiocarcinoma caused by a) S. Japonicum c) S. Splenomegaly 163.B) Tropomyosin ANS C) Myosin 156. Ptyrigopalatine canal is supplied by a) Deep branch of petrosal nerve.Polysithemia rubra vera associated with a) bone tumors----ANS b) lung diseased c)Living on Himalayan mountain 158. What passes through ligament teres of head of femur a) obturator nerve branch---ok b)branch of femoral artery 162. Auto antibodies to platelets ---ok b. congenital omphaloseal? A) Not associated with any congenital Heart diseases B) Associated with VSD? c) Associated with ASD? wrong D) B and C? 157.ANS OK b) superficial branch of petrosal nerve c) maxillary nervr branch 159. Don’t know a) Hypertension b) increased bronchial constriction c)Lacrimation d) ? 170.iliohypogastric nerve ilioinguinal nerve\ I wrote This answer .I wrote This answer . Common side effect of Isoprenlene. portal entry of infection by hematogenous route a) Through epiphysis-ok b) Metaphysis c)Diaphysis 167. is. In Osteomylitis of long bones. Histamine. The premalignant condition is a) Psoriasis b) Cervical erosion c) myelodisplastic syndrome ok d) leukoplakia 169. Nitroglycerine. Primary cartilageneous joint a) Connect the to bones together. Don’t know exact 166. Stratified cuboidial epithelium is present in a) Ducts of Salivary glands ok . Muscrinic receptors respond to a) preganglionic parasympathetic b) Postganglionic parasympathetic Sok 1 7 1 . Don’t know exact b) Present in pinna of ears c) is fibrocartilage 168. What begins with Premalignant conditions? a)Adenocarcinoma stomach b) c) 172. what is its renal clearance? HINT: use formula UV/ p ANS: 20 174. Autosomal Dominant is a) 176. ADH will decrease in a) Nausea b) Pain c) Anxiety d) Decrease serum osmolrity 180. The cause? ANS: Insulin a) insulin b) Estrogen 175. Does not cause Gangrene a) Bacterial endocarditis b)???? ANS 179. Varicose veins . The flow rate of urine is if 2 ml / min . Amyloidosis result in a) Adenocarcinoma stomach 177. The concentration of a substance in the blood is 10mg and in in urine is 100mg.b) Distal convoluted tubules of nephron c) Lacrimal gland ducts 173. Regarding RBC a) buffer as accepter of oxygen b)Only mature forms contain Hemoglobin c) Adult hemoglobin is alpha and Gamma d)Not transport oxygen 178. 35 year old female Type I diabetic got fracture due to osteoporosis. Primary brain vesicle is a) Mesencepholan b) Telencepholan c) Diencephalan 186.a) Somewhat associated with smoking b) Only in lower limbs c due to defect in adventitia 1 8 1 . intestinal motility is decreased by a) C C K b) Gastrin c) Trypsin d)lipase e) Insulin 184. Gross increase level of HCO3 a) Persistent vomiting b) CRH c) Lung fibrosis 185. Primary ossification centre is a) Epiphysis b) Metaphysis c) Diaphysis 182. Microtubules are part of a) Centrioles ok . A patient with sepsis is best diagnosed with a) Pulse > 100 b) Pulse > 120 c) Positive bacterial culture 183. Femoral artery is best palpated on? ANS: mid inguinal ligament 190. Cardiac Muscles are 189.D Semipalatinsk FCPS -1 SURGERY 25.1 gram of protein contains calories? a) 4 calories 3-7th lobe of lung is called a) medial basal .Occulocardiac reflex mediated by a)CN5 2. M. In brochhocscopy.b) Cell membrane 187. Carotid cartilage is ANS: on base of thyroid 1 9 1 . june 2008 1. Fatty Acids are transferred from adipose to liver in the form of? a) Chylomicrons b) HDL c) LDL d) VLDL SURGERY. Fist structure to be visualized is: ANS: Right lower bronchus 193. June 2008 From Dr Qasim Hussain. BP is increased when there is a) Increased sympathetic output b)Increased total peripheral resistance 188. a) loss of taste to anterior 2/3 of tongue b)Myelohoid muscle is unable to stabalizse mandible? c) Loss of sensation to lower teets? 6-Head of femer a) has anger of 125 with shaft b)posteriorly completely covered by capsule? c)epicondyls are in parallel line with shaft 7-Lymph nodules are present in a)Spleen b)Thymus c)Lymph nobes 8-In DIC .Mandibular fracture occurs just brfore the mandibular foramen.What will be with blood insulin and Glucogan levels at the time when he crosses the win line? 5. recurrent MI should be treated with? a) Anticoagulants b) Lipid lowering drugs c) Beta blockers 12)Stratified squamous epithelium is present in a )palatine tonsils b)pharynx c)Ovary 13) Low voltage QRS comples ios seen in a) MI b) Bundel branch block c)Hypertension .there is a)decreased palatlets 9-what is inferior in perenium? a) pereniul membrane 10)spleen is a) mesodermal b)endodermal 11)according to recent recommendations.4-A marathion runner wins a race. followin if true about it onset of action a) 5 min b)5 sec c) 60 sec 18)I/V sedation is given to a patient. Following is true about its arm-brain circulation a)60 sec b)5min c)30min d)5 sec e)? 19) DVT is most common in a)Femoral vein b)popleteal vein c)Superficial vericosed veins of leg 20) Which of the following is not a tumor marker a)Acid phosphatase b)PAP .2 15)Head of Humerous is supplied by a) Anterior circumflex artery b)Posterior circumflex artery c)Suprescapular artery d)Subscapular artery 16) Gene are composed of a) RNA b)DNA c)Extrons d)Introns 17) I/V dopamine is given to a patiend.d)IHD e)Pleuritis 14)Isthmus of thyroid is present at a)C1. c)Hcg d)AFP e)CEA 21) Pus contains a)Dead bacteria b)Dead neutrophils 22) The most common cause of fatty liver in our society is a) Alcohol b)Hepatitis B and C c Drugs 23) Carotid bodies respond to a) increase blood H+ b)Increase blood co2 24) In anterior Duodenal perforation . the contents will go to a) Right iliac fossa b)Left Iliac fossa c)Anterior subhepatic space d)posterior subhepatic space e) small bursae 25) Metabolic alkalosis results from a)Ingestion of ammonium chloride b)ingestion of carbonic anhydrase inhibitors 26)A Diabetic patient fas urine ketones and urine glucose ++ the cause is ? a)Hyperglycemia b)Insulin difficiency 27 Response to chemotherapy is because it causes a)Atrophy b)Apaptosis 28)Dorsal column damage results in Ataxia because . b) 4 .What structure prevents the the joint to go back in cavity? . c)5. with Hypercellular bone marrow/\ a)Due to drug he is using for anaemia b) Leukemia c) Iron deficiency anaemia d)G6PD deficiency 31) The sign of cerebellar disease is a)Adiadohakinesia b)Static tremor c)Hypertonia 31)Neurohypophysis drain secretions a) Adrenergic neurons b)cholinergic neurons c)free nerve endings 32)CSF a)has osmolarity <1040 b) pressure <10 mm of water c) protein >200 33) Stimulation of Alpha adrenergic neurons 34)End arteries are present at a)Brain b)Heart c)Bone 35) In Tempomandibular Joint Dislocation. e) 9 30-A patient has hb 6. d) 12. palatelet 450.000.a)loss of pathway from dorsal column to thalamus b)Loss of proprioception input to cerebellum c)Loss of proprioception to Thalamus d) Loss of proprioception to Hypothalamus 29)Which of following nerve passes dorsal to brain stem? a) 3. increases frequency and chills and riggors.his blood will have 100% oxygen? a) 1 min b)2 min c)5 sec d) 5 min 38) FRC is? a) TV + ERV b)IRV + ERV 39)Aldosterone stimulate a) Melatonin? b)? 40)A patient has fever 40c with flank pain .a) Tubercles on articular surface of mandible b)Tempomandibular ligament c)Lateral Ptyregoid muscle d)Medial Ptyregoid muscle 36)Where the gangliom joining the superior petrosal nerve and inferior petrosal nerve is located? 37) A patient has hypercarboxyhemoglobin.\ a) Blood culture b)urine culture c) Blood and urine culture 40)Following is not true? a) HCV positive carriers have not been reported 41) Pulse pressure is increased in a)Arteriols b)capalleries C)Aorta 42) A grand father was playing with his grandson with holding his wrist and rotation him in a circule.Suddenly the child starded crying and sit with . After how much time . He is breathing 100% oxygen. peritoneum b) skin.25 cholecalciferol in a)Hepatic failure .extaperitoneal fat. The layers incised will be a) Skin-superficial facia-deep facia. What is it? a) Prostate 50)25 cholecalciferol is not converted to 1. and its center contains two calcified bodien of pink colour. What happent ? a) Proximal Radio ulnar joint dislocation b)Distal Radio ulnar joint dislocation c)Fracture of acromion 43) Regarding typhoid fever a) Asypmtomatic b)Carriers do not excreate causative agent in fees c) Asypmtomtic carriers should be isolated and treated 44) Incision is given on Rt Lumbar area.peritinium 45) Carpus callosum a) connects the two hemispheres 46)Most constricted part of male urethra is a) External meatus 47)Superior Thyroid glands a) Are located behind the Thyroid gland outside the facia b)Have blood supply from the superior thyroid arteries 48) Which of the following is true a) Piriformis attach to inferion trochanter b) Schiatic nerve passes through superior sciatic foramen 49) A structure lined by cuboidal epithelium.external oblique-internal oblique-transverse – Transverse fascia-exta peritoneal fat.holding his forarm in porn position.superficial fascia. It could be a) Damage to inferior epigestric artery? 63) In Hypoxia. cell swelling occurs due to .b)Renal Failure 51) Which of the following is not present in posterior relations of kidney? a) colic flexure 52) Rt border of heart is formed by a)Rt atrium 53) In pregnancy . Flank pain could have a) SLE 60) A young patient has difficulty in swallowing a) Sclerodermia 61) A patient with SLE have a) Possitive ANA b) Possitive anti SS 62) Incision on Supra pubic area caused heavy bleed.joint pain.Lactation does not occur because a) Increased Level of progesterone and Estrogen 54) Which of the following have Pulmonary cycle? a) Ascariasis 55)Causative agent of meningitis in 60 years old man is a) St pneumonia 56) Cause of death in pneumonia bue to shock is a)Gram –ve bacteria b)Pseudomonas c)st pneumonia 57) Diphtheria exotoxin hane powerful effect on a) Larynx b)Heart c)Nerve endings d)Kidneys 58) Aspiration of peanut will go in a) Right lower bronchus 59) A female with butterfly ace. The rectal examination showed no ganglion cells. Na is absorbed totally neutrally active transport? a) PCT b)DCT c)ATT 76) Cell mediated immunity is mediated by a) T cells b)B cells 77) Virus does not cause a) Lipoma b) Kaposi sarcoma c)cervical cancer 78) Iodine is stored in thyroid follicles as a) Thyroglobulin b)DIT and MIT 79) A patient died due to colonic cancer.On autopsy found thousands of polyps. The condition is called A) Intussusceptions B) Hirschsprung’s disease 66) In what portion of nephrone. The cause could be a)Adenomatous polyps? 80) A patient has double ureters and double pelvis.a) Entry of water in cell fromj surrounding b) Entry of lipids 64)What is potent Antoxident ( Anti Aging) a)VIT E 65) A boy is excreting meconium from rectum and has severe constipation. Patient's father also had colonic cancer. It could be a) anomaly is due to premature separation of ureteric bud 81) Chylomicrons contains a) Fatty acids with vitamins 82) Transudate has . contraction occurs due to a) Sacromere b) Actin c)Myosin 88) In chronic inflammation. there is predominate a) Fibroblasts b)Macrophages c)neutrophils 89)Characterics of malignant cancer is a)Metastasis 90 )Characteristic of malignant cancer is a) pleomorphism 91) Pretraceal fascia a) completely encircle the Thyroid gland b)Completely encircles trachea 92) 6 months after the resection of terminal ilium.a) Low Albumin 83) common mediator of Basophils and mast cells have a)Histamine 84)In Skeletal muscle . the patient a) iron deficiency anemia 93) A patient has sweating 2L . What could be happen? a) Increased body volume? b) decreased intracellular osmolarity c)Increased extracellular osmolrity 94) Inhibition of heart Rate will be in a) Stimulation of parasympathetic nervous system 95) Aldosterone is stimulated by a) Increase of serum K 96) Follic Acid difficiency causes a)Megaloblastic erythropoises b) Foliate deficiency 97) Difference B/w Plasma and interstitial fluid is a) Protien . and he took 2 L plan water. posterior wall is formed by a) Conjoint tendon b)Internal oblique muscles 100) What is part basal ganglia are a) Putamen ( 32 mistakes ) Paper 2 101) Blood supply of vertebral column is a)Vertibral artery 102)Suprascapular nerve arises from a) Thyrocerv ical trunk 103) Ist pharyngeal pouch forms a) Auditory tubes 104)Diapharm is supplied by a) c 345 105)Urachus is remnant of a) Allantois 106)A patient had surgery of breast and he is now unable to lift his arm. What is lost in feeces? a) HCO3 109)phasic fast adaptive structure is a) Mickle disk b)Carotid baroreceptors . What is damaged? a) serratus anterior muscle 107)Thurst is stimulated by a) ADH 108) A patient has dirrhoea and developed metabolic acidosis.98) C02 is mainly carried by a) Hco3 99) In Inguinal hernia. rounded structure. Gastroduodenal ligament 111) A patient can not abduct and adduct his fingers. a) TIBC b)serum ferritin 115)After gastrectomy. Falciparum Ligament. following test is appropriate. lined by stratified squamous epithelium. a patient might develop a) Megaloblastic anaemia 116) A patient got injury in the posterior triangle of neck. Lost adduction of thumb also. First what to treat? a) I/V Fluids 118) Clostridia cause a) Gas Gangren 119)Angiotensin 1 is converted to Angiotensin 2 in a) Lung capillary endothelium 120) First week diagnosis of Typhoid is a) Blood culture 121)A soldier posted at high altitude for many years came witg PVC > 40 a) Secondary polycythemis 122) Buerger's disease is strongly associated with .? Palatine tonsils 114)Regarding the Iron requirement of a pregnant woman.c)Rufini's carpuscles 110)The lesser omentum forms which structures? a) Hepatoduodenal ligament.there might damage a) spinal part of accessory nerve b) CNS part of accessory nerve c) Vagus nerve 117)A patient in RTA came in EAC with hypovomumic shock. a) Deep branch of ulnar nerve 112)Femoral canal is formed by what fascias? a) Fascia Lata + memberenous Layer b) c) 113) Organ . and drained by single vein 130) Repture of middle meningeal artery cause hematoma in a)Between the two layers of dura mater 131)Middle esophageal constriction is by a)Arch of Aorta b)Left bronchus 132) T.cells a) Matured in Thymus after birth b)Both T cells and B cells are derived by same precursors 133)Neurovascular bundle is located at ribs at a)inferior border of ribs 134)Which of the following does not predispose the cancer? a) Radiation b)Bacterial products c)virus d)Chemicals 135)Methicilline resistant staph are a) Usually sensitive to Vancomycin 136) Cardiac muscles can not be tetanized becosed a)They have long refractory period 137) Severe blood transfusion reactoion occurs if transfused all except . He might developed a)Left homonymus hemianopsia 128)In Burkit Lymphome.a)Smoking 123)Acute appendicitis is associated with a) Neutrophelic leukocytosis 124) Medial arcuate Ligament encloses at its upper end a)Psoas muscle 125) QRS comples is due to a)Ventricular systole 126) The transitional epithelium have a) Couoidial epithelium lined by more larger and rounded cells? 127) A patient have right optic tract damage. The causative agent is a) EBV 129)Regarding adrenal glands a) Right is Larger than left b)Supplied by numerous arteries . therer is a) Pulsus deficit 145)A patient lost weight .RR 20. pulse 120/min. Hemoglobin released will be attached with .a) A+ blood to 0+ b)A + blood to AB+ c) B+ to O+ d) B+ tp AB+ e)O +ve blood to OA+ 138)Xenograft is a) Transplant of tissues of different species 139) Lysosomes contains a) Acid Hydrolase 140) Rt Gastro epiploic artery is the branch of a) splenic artery 141) Right gastroduodenal artery is the branch of a) Hepatic artery 142 )An asthematic patient may have FEV1/FCV <75 143)Ligamentum arteriosum connect with a) Aorta with Left pulmonary artery 144)In atrial fibrillation. DB 120/80 a) Hyperthyroidism 146) Alpha receptors stimulation may cause a) Lipolysis b)Gluconeogenesis c)glycogenolysis d) Glycogenesis 147)Trypsinogen in the duodenum will be activated by a) PH < 7 148) Pulmonary embolism most commonly involves a) Left pulmonary artery 149 A patient had streptococcal infection. hot dry skin. What blood test you will advise him after one week? a) Anti streptolysin O titer 150)A patient developed hemolytic anaemia after blood transfusion. fever with Antimitochondrial antibody +ve a) Primary billiary cirrhosis 152) PDA occurs in a) Prematurity 153) Fertilization occurs in a)Oviduct 154) Liquificative necrosis occurs in a) Brain 155) Temperature regulatory center is in a) Hypothalamus 156) Skin cancer occurs a) In Sun exposed area 157) Giant cells have a) Macrophages 158) Inflamatory mediator is a) C5a 159) Optic tract ends in a) Lateral geniculate body 160) Gastric motility is increased by a) Acetylcholine 161)Wound healing is delayed by a) infection 162)Achalasia is due to a)loss of Aurbach plexus 163)Patient has severe chest pain with ST elevation a) Disectening aorta b)Pleuritis c)Pneumothorax d)costochondral e)Pneumonias 164)Patient X-Ray shows hilar lymph nodes. Granuloma but no necrosis. What is the diagnosis? .a) Heptoglobin b) Albumin 151) A patient with itching. B1 b. which of the following enzyme will be raised? A) CK-MB B) C) D) E) LDH Alkaline Phosphatase AST Troponin T 169) Replacement of cells of other normal site of body is called a) Metaplasia 170)Heart sound produced by rapid ventricular filling is A) 1st heart sound B) 2nd heart sound C) 3rd heart sound . High output cardiac failure a.I.A} Sarcoidosis B} Silicosis C} Tuberculosis 165) Strongest layer of small intestine is a)Circular b)longitudinal c)Mucosa d)Submucosa ok? 166). B2 c. B6 167)Valve less vessel is A) B) C) D) E) Aorta Pulmonary artery Coronary Sinus Pulmonary Trunk SVC 168) Within 1 hr of the Acute M. D) 4th heart sound 171)Corticosteroids decrease a) Neutrophils b)Lymphocytes c)Eiosinophils d)Monocytes e)Basophils 172) DVT occurs in females by use of OCPs 173) Visceral Pericardium is supplied by a) Phrenic Nerve b) Sympthatic Nerves c) Vagus neve d) Cardiac Plexus 174) Basal Cell Carcinoma involves a) Buccal Mucosa b) Hard Palate c) Soft Palate d) Lower Lip e) Oral Cavity 175) Primary spermatocyte divide by a)Mitosis b)Spermiosytosis c)Primary meiotic devision d)Sec Meiotic devision 176) The volume of distribution of drug is not influenced by a)Sex b)Age c)Heart failure d)Renal Failure 177)In Hypothyroidism a) Increased cholesterol level . muslce weakness C.tachycardia E. venules 182)Thiamine deficiency causes A} Peripheral Neuropathy B} Pellagra C} Chelosis D} Dermatitis 183) Which of the following is not a feature of shock A.restlessness . precepillary sphincter c.178)craniophrangioma in the center of optic chiasmata may cause a) Bitemporal hemianopsia 179)Which one of the following most likely causes Increase in GFR? a)Constriction of afferent arteriole b)Constriction Of efferent arteriole 180) cardic output measured by thermodilution method is called a)fick law? 1 8 1 .increased urine output D.decrease cardiac output B. arterioles b. Blood flow regulation is mediated by: a. capillaries d. ferritin B.transferrin C. It is not covered with amnion. The rectal examination showed no ganglion cells. The condition is A} Gastroschisis B} omphalocele 2] Which of the following is not related to Meckel’s Diverticulum? A} is remnant of urachas B} lies 60 cm proximal to illeocecal Valve 3] A boy is excreting meconium from rectum and has severe constipation.apoferritin SURGERY March 2008 COURTESY: DR: ALTAF GHUMRO & PREPARED BY DR: MUHAMMAD SHEERAZ AKHTAR SOOMRO 1] A newborn child has abdominal swelling on the right side of the umbilical cord. B} they develop from lateral plate mesoderm 5] Regarding diaphragmatic hernia the most likely is .184) Iron is stored in the form of A. The condition is called A} Intussusceptions B} Hirschsprung’s disease 4] Regarding somites A} They develop on both sides of notochord. small testes. Which one of the following is not the developmental remant A} lateral umbilical ligament B} Medial umbilical ligament C} Median umbilical ligament D} ligamentum teres E} ligamentum venosum 8] The part of mouth derived from ectoderm is A} Epithelium of parotid salivary gland B} Epithelium of Tongue C} Floor f mouth D} E} 9] A male with Gynaecomastia. diagnosed as Klienfelters syndrome.A} Absence of septum transversum B} Failure of pluiroperitoneal membrane 6] Which one of the following is not the derivative of 1st (mandible) pharyngeal arch? A} Incus B} Malleus C} Stapes D} E} 7] Embrologically. The karyotype involved is A} 45 XO B} 46 XX C} 47 XXX D } 47 XXy E} 47 XX . 10] Somites are derived from A} Intermediate mesoderm B} Paraxial mesoderm C} Lateral plate mesoderm D} Ectoderm E} Endoderm 11] An I/V Urograph of 8 year old boy shows Excretion of dye normal I kidney but absence of shadow on left side. Which one of the following is most likely? A} Horse shoe kidney B} Constricted ectopic kidney C} Pelvis kidney D} Non rotating kidney E} Unilateral renal agenesis 12] Which one of the following is not formed from urogenital sinus? A} Bartholin’s gland B} Urethral glands C} Para urethral glands D} Seminal vesicle E} 13] Virus damage the cell by A} making the pores in cell membrane B} Forming free radicals C} Altering the formation of proteins D} Nuclear damage E} using energy for their own metabolism . Instead there is a small shadow just above the bladder on the left side. 14] The tissue damage by Ionization Radiation is due to A) Damage to Golgi bodies B} Formation of free radicals C} Hydropic degeneration D} Metaphase of cell E} Swelling of cells 15] All are features of IRREVERSIBLE cell injury EXCEPT A} Karyolysis B} Karyorhexis C} Autolysis D} Shrinkage of mitochondria E} Appearance of myelin figure 16] Which one of the following is least likely involved in increasing Osteoporosis A} Increased w t : on bone B} Renal cell carcinoma C} Carcinoma prostate D} E} 17] Patient X-Ray shows hilar lymph nodes. Granuloma but no necrosis. What is the diagnosis? A} Sarcoidosis B} Silicosis C} Tuberculosis D} E} 18] In women the most common form of necrosis after trauma is due to A} Trauma to fatty tissue B} C} D} . C5 complex E} 23] Which one of the following is potent antioxidant? .E} 19] Dystrophic calcification is present in which of the following A} Acute pancreatitis B} Malaria C} Tuberculosis D} E} 20] Which one of the following is most likely feature of reversible injury? A} Cell swelling B} Karyorhexis C} Decreased glycogen D} Myelin figures E} 21] A person fell on road and got an abrasion on elbow the first event likely occur would be A} Vasoconstriction B} Platelet adhesion C} Platelet aggregation D} E} 22] Which one of the following facilitates phagocytosis in neutrophils? A} C5a B} C3a C} C3b D} C3 . A} Vitamin A B} Vitamin C C} Vitamin E D} Transferrin E} 24] Which one of the following among Arachidonic acid metabolite is most likely function mediated in phagocytosis? A} Chemo taxis B} Diapedisis C} Transmigration D} E} 25] Which one of the following is potent COX-II INHIBITOR? A} Aspirin B} Celocoxib C} Indomethacin D} Meloxicam E} Piroxicam 26] In case of pulmonary embolism extensive infarction causes sudden death due to A} Air embolism B} Fat embolism C} Thromboembolism D} E} 27] A patient was operated for abdominal surgery. Which one of the following is most likely? . A few days after he developed pain and swelling in his right leg. Which one of the following is correct? A} Autosomal Dominant disorder B} Autosomal recessive disorder C} X. Which one of the following should be given immediately? Fresh frozen plasma B} 20 days old blood C} 3 days old blood D} E} 29] A boy was born with the condition of polycystic kidney disease. but within 10 mins the organ failed. He was admitted in the hospital.A} endothelial injury B} hypercoagubility C} stasis and endothelial injury D} E} 28] A soldier has profuse bleeding. Which one of the following is likely etiology? . which one of the following is single most significant test to be done? A} ABO blood grouping of donor and recipient B} HLA typing and matching C} Donor and recipient should be close relatives D} E} 31] A organ was transplanted to the patient.linked disorder D} E} 30] In a renal transplantation. which one of the following is the confirmatory test in the boy A} ELISA test B} Western Blot test C} PCR D} Increased levels of HIV in T lymphocytes E} 34] A 10 year old boy was operated for gastrectomy. which one of the following type of anemia is developed postoperatively A} Megaloblastic anemia B} Iron deficiency anemia C} Normocytic norm chromic anemia D} E} 35] Autoimmunity is not involved in which one of the following A} Pernicious anemia B} Grave’s disease .A} Antibody mediated B} T-Cell mediated C} Preformed antibodies D} E} 32] AIDS occur in the presence of A} Kaposi’s sarcoma B} C} D} E} 33] A 2 year old boy is suspected as HIV positive. C} Diabetes mellitus D} Hashimoto’s thyroiditis E} Thymoma 36] In case of hashimoto’s thyroiditis. which one of the following is seen? A} Anti thyroglobulin B} C} D} E} 37] To differentiate Chron’s disease from Ulcerative colitis. Bromocriptine is given to treat it the drug causes A} Inhibition of prolaction from pituitary gland B} Decreased prolaction from hypothalamus C} Inhibition of prolaction from breast D} Stimulation of prolaction of anterior pituitary . following is finding A} Mallory bodies B} C} D} E} 39] A 20 year old girl has developed Galactorrhea due to prolactinoma. which one of the following is likely significant regarding Chron’s disease A} has increased frequency to cancer B} Involves both small and large intestine C} D} E} 38] Regarding alcoholic hepatitis. R most likely A} Synthesizes protein B} Synthesizes lipids C} D} E} 4 4 ] Half life of insulin is .E. Which of the following drug has been taken by this patient? A} Morphine B} Heroine C} Pethidine D} Phenobarbital E} 41] Strongest layer of small intestine is A} Mucosa B} Sub mucosa C} Circular layer D} Serosa E} Longitudinal layer 42] Regarding adrenal medulla which one of the following is true? A} Secretes epinephrine and Norepinepinephrine in 80/20 ratio B} C} D} E} 43] R.E} 40] An unconscious patient presented in ED does not respond to Naloxone. A} 5 mins B} 90 mins C} 120 mins D} E} 45] Insulin A} Converts glycogen to glucose B} Stimulates protein synthesis C} Stimulates lipolysis D} E} 46] Which one of the following most likely contains abundant cholesterol? A} VLDL B} LDL C} Chylmicrons D} HDL E} 47] Which one of the following most likely causes Increase in GFR? A} Constriction of afferent arteriole B} Constriction Of efferent arteriole C} D} E} 48] Gastric emptying is delayed by A} Gastrin B} Secretin . C} CCK D} E} 49] Which one of the following pancreatic enzyme is most likely involved in fat absorption A} Amylase B} Lipase C} D} E} 50] Smooth muscle A} Contain sarcomere B} T-tubules are present C} Gap junctions are present D} Forms motor end plate E} 51] Steady pressure on skin is perceived by A} Pacinian’s corpuscle B} Ruffini’s corpuscle C} Meissener’s Corpuscle D} Golgi tendon organs E} 52] Which one of the following is related to pain? A} Serotonin B} Substance P C} D} E} . 53] Free nerve endings have receptor for A} Light touch B} Pressure C} Pain D} Vibration E} 54] Which one of the following has highest pressure? A} Pulmonary artery B} Left atrium C} Right atrium D} SVC E} 55] Flow of current through junctions in the muscle A} Smooth muscle B} Skeletal muscle C} Cardiac muscle D} E} 56] A profusely bleeding patient develops hypotension.P A} Baroreceptor reflex B} Chemoreceptor reflex C} Secretion of ADH D} Aldosterone E} Renin angiotensin mechanism 57] Water enters into the cell by . which one of the following would be immediate mechanism to maintain B. A} Pinocytosis B} through pores C} Simple diffusion D} Facilitated diffusion E} Active transport 58] Insulin secretion is controlled by A} Glucagon B} increased glucose C} Decreased glucose D} E} 59] Systolic B.P is highest in A} Pulmonary arteries B} Cerebral arteries C} Renal arteries D} Brachial arteries E} 60] End diastolic volume depends on A} Atrial contraction B} Distensibility of ventricles C} Duration of diastole D} Venous return E} 61] Mean electrical axis of heart is deviated to left if A} Heart is angulated on right side B} Pulmonary stenosis C} Right bundle branch block D} COPD . is dark skin B} Increased conc: of Hb A C} Increased Conc: of Hb F D} Aortic stenosis E} 63] Secretion of potassium by distal collecting tubule decrease by A} Increase in potassium in diet B} Hyperaldosteronism C} Metabolic alkalosis D} Spironalactone intake E} Thiazide diuretic 64] Absorption of iron is markedly affected by A} Ingestion with meal B} Partial gastrectomy C} Trivalent form D} E} 65] Patient with pancreatitis has developed steatorrhea due to deficiency of A} Lipase B} Pepsin C} Amylase D} Renin E} .E} 62] Pulse Oxymeter measurement gives error in the reading if A} Pt. ANATOMY 0 1 . Some questions do contain other choices but you consider “A” the right choice. FCPS Part 1 SURGERY AND ALLIED 5TH JULY 2007 Note: Only correct answers are given as choice “A”. Man is A} 12 gm B} 120gm C} 200 gm D} 150 gm E} 68] Source for metabolic fuel for liver in post absorptive state is A} Glucose B} Ketones C} Glycerol D} Fatty acids E} SURGERY July 07 Contributed By Dr Ahsan Siraj.66] Absorptive surface of small intestine is A} 50 B} 100 C} 200 D } 350 E} 500 67] Daily dietary protein for 70 k g . A 10 year old boy with shoulder injury lost abduction of his shoulder up . abduction of thumb with intact sensation due to: Injury to Radial nerve at elbow 3. Regarding post. In an injury to knee joint a man is unable to extend his knee the root value affected: L3-L4. Regarding palm: Superficial palmer arch is below palmer apeneurosis. Which of the following muscle have dual nerve supply from median and ulner nerve: Flexor digitorum profundus. 09. Tibial artery. Epitrochlear lymph nodes. 7. A boy is brought to a hospital with injury at elbow joint lost extension of medial 4 fingers at MTP joint. Which of the following muscle is not supplied by median nerve: Adductor polices. Infection of the first pulp space lymph nodes will first drained. 4. Peroneal artery. 8. In the injury to neck of fibula the artery damaged: Ant. . 10.to 30 degree which muscle is lost: Supraspinatus Deltoid 02. 6. 05. Triangle: Base is form by middle 3rd of clavicle. Which of the following muscles is flexor at hip and extensor at knee: Sartorius. It enters the thoracic cavity thru esophageal opening. Border of thyroid cartilage. Internal spermatic fascia is a continuation of: A. 16. Angle of the rib. 18. A. 19. 15. 20. A 40 years old man feels pain in his gastrocnemeus muscle after he walks 100 meter which relieves when he stops walking is likely due to obstruction of: A. Weakest point of rib is: A. Ligamentum Teres. Which of the following structure arches over root of left lung: A. 17. 13. Border of sternocladomastoid at the sup. 1st rib. Tibial artery. The branch of internal carotid artery which remains in true pelvis . What is inappropriate about azygus vein.1 1 . Is formed by two vertebral veins. Post. Great saphenous vein has how many valves: 20. 2 1 . A pathologist wants to pass dye to the liver he will reach t h r u : A. Arch of aorta. 14. Transversalis fascia. Vertebral venous plexus: Has no communication with cerebral sinuses. Bifurcation of common carotid artery: Beneath the any. 12. Lies in epidural space. Subclavian artery grooves over: A. throughout its course: A. Nerve supply to the lower airways: A. 29. Ureter 23. Surgery Q's July 2007 FCPS-1 . Aorta. Middle rectal artery. 22. During an injury to the bulbar part of urethra the urine will extravasate to: A. 28. T3-4 25. Common Bile Duct. Common Bile Duct. Slow growing tumor of head of pancreas will compress: A. 26. Nerve supply to the distal scrotum is t h r u : Illioinguinal nerve. Superficial perineal pouch. Parasympathetic nerves supplying the urinary bladder are: A. Tumor of the head of pancreas will compress: A. Genitofemoral nerve. During a per rectal examination the examiner will not reveal: A. Illiohypogastric nerve. During surgery of right colon for carcinoma which structure would not be injured: A. Pelvic splanchnic nerves. 27. 24. remains of vitelline duct(sumthin like dat) 4.10 .25 7.present in all da body cavities .the ca head of pancreas may compress .mesothelioma is ..cholecalciferol .15 .first formed blood vessels(sumthin like dat) . .surfactant is produces by .common bileduct .how many valves r present in great saphenous vein? .duodenal artery .but ven supported upto 30 drgrees abduction is normal.lined by simple cuboidal epi 2.spenic artery .kidney .teres major 6...liver.renin 5.pneumocytes2 3.1.a pt cums 2 u wid loss of abduction of right arm upto 30degrees.20 .supraspinatuis .pneumocytes1 .deltoid .5 .activation of one of followin occurs in kidney .infraspinatus .which muscle is involved? .allantois is . wot structure arches over left bronchus .azygous vein 9.a pt is unable 2 flex metacarpels.the central regulation of temperature occurs in .ulnar nerve below da elbow joint . and unable 2 flex wrist joint(i think).8..but one of da option was FAT EMBOLISM 13.cortico spinal tract 12..cerebellum 10.pons ..hypothalamus ..s4-s5 14.there was another stem regarding da temperature regulation but it was asked peripheral temp regulation..which sensory part is involved in da flexion of knee joint? (sumthin like dat) .pons .i dun xactly remember da clinical correlated stem.s3-s4 .fine voluntary skilled movements of upper xtremities r conducted by .s1-s2 .aortic arch .upper motor neuron ...which nerve is damaged? .medulla .ulnar nerve above da elbow joint .cerebellum .n one of da option was skin in dat plus anterior hypothamus.inability 2 abduct fingers..i dun remember da xact stem! 11..there was another stem regardin fracture of bone n its complication..medulla . A 10 year old boy with shoulder injury lost abduction of his shoulder up to 30 degree which muscle is lost: .stratified squamous epi .an unaffected stage in amoeba . ducts of salivary glands 19.ligamentum teres .median nerve 15. Some questions do contain other choices but you consider “A” the right choice.which structure has no embryonic bacground? .adult amoeba (sumthin lyk dat) 16.. ANATOMY 01.medial longitudnal lig 17.neutro penia and hepatosleenomagaly is present in .simple cuboidal epi is present in .all those structures in contact wid air hav da following epi . 2010 5:00 pm Note: Only correct answers are given as choice “A”.non keratinzed sq epi .radial nerve .amoebic cyst .pseudo startified sq epi 18.keratinized sqamous epi .amoebic larva .median longitudnal lig .chagas disease FCPS Part 1 SURGERY AND ALLIED 5TH JULY 2007 Dby drwatson » Thu Jul 22.falciparum malaria .anemia.ligamentum venosus . 10. 8. Which of the following muscles is flexor at hip and extensor at knee: Sartorius. Infection of the first pulp space lymph nodes will first drained.Supraspinatus Deltoid 02. Triangle: Base is form by middle 3rd of clavicle. 1 1 . Regarding post. A boy is brought to a hospital with injury at elbow joint lost extension of medial 4 fingers at MTP joint. Regarding palm: Superficial palmer arch is below palmer apeneurosis. Which of the following muscle is not supplied by median nerve: Adductor polices. 7. Peroneal artery. Bifurcation of common carotid artery: . 4. 09. In an injury to knee joint a man is unable to extend his knee the root value affected: L3-L4. 05. abduction of thumb with intact sensation due to: Injury to Radial nerve at elbow 3. Epitrochlear lymph nodes. 6. In the injury to neck of fibula the artery damaged: Ant. Which of the following muscle have dual nerve supply from median and ulner nerve: Flexor digitorum profundus. Tibial artery. Beneath the any. Is formed by two vertebral veins. 15. A 40 years old man feels pain in his gastrocnemeus muscle after he walks 100 meter which relieves when he stops walking is likely due to obstruction of: A. A pathologist wants to pass dye to the liver he will reach t h r u : A. Lies in epidural space. Internal spermatic fascia is a continuation of: A. Tibial artery. 17. 13. 18. A. 20. Border of sternocladomastoid at the sup. Transversalis fascia. The branch of internal carotid artery which remains in true pelvis throughout its course: . It enters the thoracic cavity thru esophageal opening. 19. 12. Post. Arch of aorta. Angle of the rib. 14. 2 1 . Border of thyroid cartilage. Which of the following structure arches over root of left lung: A. Ligamentum Teres. What is inappropriate about azygus vein. Great saphenous vein has how many valves: 20. 16. Weakest point of rib is: A. Subclavian artery grooves over: A. 1st rib. Vertebral venous plexus: Has no communication with cerebral sinuses. Genitofemoral nerve. 29. Regarding Decidua: A. Nerve supply to the distal scrotum is t h r u : Illioinguinal nerve. Decidua basilis forms the maternal part of placenta. Pelvic splanchnic nerves. Common Bile Duct. Superficial perineal pouch. 24. Tumor of the head of pancreas will compress: A. 2. Nerve supply to the lower airways: A. Parasympathetic nerves supplying the urinary bladder are: A. During a per rectal examination the examiner will not reveal: A. During an injury to the bulbar part of urethra the urine will extravasate to: A. 1. Common Bile Duct. Slow growing tumor of head of pancreas will compress: A. T3-4 25. 27.A. Embryonic period is: . 28. 22. Aorta. 26. Illiohypogastric nerve. Ureter 23. Middle rectal artery. EMBYROLOGY. During surgery of right colon for carcinoma which structure would not be injured: A. Which of the following bone is derived from 2nd Pharyngeal arch: A. left umbilical vein will regress t o : A. Lateral umbilical ligament. Styloid process. 4. Sphincter pupili muscle. . Derivative of Ectoderm include: A. Which of the following is a derivative of neuro ectoderm: A. 10. Regarding Allantois: …… 7. 6-10 weeks. 6. Secretary Epithelium of parotid gland. 3-8 weeks. Regarding Mesothelium: A. What will happen immediately after birth: Anatomical closure of Ductus arteriosus. Secretary Epithelium of parotid gland. 1 1 . 12. Which of the following structure is not an Embryological remnant: A. 9.A. 8. Obliteration of Right umbilical vein. 5. Physiological hernia occurs in between: A. Lines the body cavities. 3. Anatomical closure of Ductus venosus. Ligamentum Teres. Anatomical closure of foramen ovale. Obliteration of Left umbilical vein. Derivative of Ectoderm include: A. Temperature regulation centre is located i n : A. Regarding Trapezoid Body: A. 8. Injury to L1 vertebra will directly damage which of the following spinal structure: A. In adults spinal cord ends at the intervertebral disc between: A. Regarding spinal cord what is inappropriate: A. What is inappropriate regarding Glossopharyngeal nerve: A. 10. Primordial Germ cells are derived from: A. L1-2. 6. It is present at the cervical portion of spinal cord. Hypothalamus. Its dura matter has two layers HISTOLOGY 1. Epidural Space è Dura Matter è Sudural Space è Arachnoid Matter è Subarachanoid space. 4. Loss of temperature regulation is due to damage at: A.13. Anterior Hypothalamic Nucleus. In spinal tap the needle which will go across: A. It is related to the auditory pathway. 14. Simple columner epithelium is present: . NEUROANATOMY Hidden content: UNLOCKED 1. 5. Conus Medullaris. Which of the statement regarding Lateral Horn of spinal cord is inappropriate: A. It is entirely sensory. Muscles innervated by hypoglossal nerves are derived from: A. Corticospinal Tract. Occipital Somites. Yolk sac endoderm. 7. 3. 2. Loss of Fine and skilled movements of the hand is due to damage of: A. 9. …………… 6. 2. 2. Cytoskeleton: A. Will not found in any other tissue than blood. Sympathetic stimulation 5. Will be inherited by Autosomal recessive pattern . 4. 3. Pulmonary Wedge Pressure: A. Straitifed Squamous Keratinizing Epithelium . Indirectly measures left atrial pressure. Compact bone contains: A. 7. Bain Bridge reflux. 6. Osteoblast cells in the lacuner spaces. Choroidal plexus of ventricles. QRS Complex. 4. Blood flow to the left ventricles is increase b y : Acetylcholine infusion. During Depolarization: A. LVEDV. Surfaces which are prone to great amount of friction have: A. Ventricular Depolarization on ECG strip is represented b y : A. Newly formed Elastic cartilage looks yellow & dense because of: A. 8. There is rapid influx of Na+.A. 7. 5. Multiple nuclei located at their periphery. Skeletal muscles have: A. Nasal cavity. Maintains the integrity of the cell. 8. Mast cell. Clchannels. IPSP is generated by opening of: A. Which part of the respiratory tract contains mucous glands: A. Blood group antigen: A. PHYSIOLOGY 1. Ventricular preload is measured by: A. 3. ………. Large n o : of Elastin fibers. B. Dorrow`s solution. 16. Inhibitory factor released by hypothalamus against which of the following hormone: Prolactin Growth hormone. ADH will act o n : A. Cushing syndrome. 12. What is the suitable I/V fluid for the patient of acidurea: Normal Saline.9. Simple Diffusion depends on all of the following factors except: A. ESR will increase with the decrease i n : A. . 18. O+ve. Proximal Tubules. If father’s blood group is B+ve & mother’s blood group is AB+ve there child can not have which of the following blood group: A. 10. Ringer lactate. 15. 1 1 . Distal Tubules. Neuron. 12. Water. Bile of the Liver differs from the GB bile because GB bile contains decrease amount of: A. Magnetic field. Skeletal muscle. 17. 13. 14. Renal absorption of glucose thru secondary transport with sodium occurs at: A. Pyloric stenosis. 5% dextrose 1 0 % dextrose. Albumin. Cortisol. Which of the following cell can never reproduce: Erythrocyte. Conn`s syndrome. Which of the following causes Hyperkelemia: Zollinger Ellison syndrome. Which of the following hormone is called STRESS HORMONE: A. Regarding Interferon: ………. Regarding conduction of visual pathways: ………. Lymphocyte. . 27. Resection of distal ileum will impair the absorption of: A. Sustained rhythmic reflex tremors induce by sudden movements is: A. Chloride shift means: A. 23. Regarding olfaction: A. Transfer of Cl.in erythrocytes in exchange of HCO3. T3. Bile Salts. Heat loss mainly depends o n : Temperature of the surrounding. Regarding Bradykinin: It formation is activated by killkeran. 26. Evaporation. 33.Smooth muscle. Which of the following is the NOT non dividing cell: A. Most active form of thyroid hormone present in circulation is: A. Cardiac muscles are prevented by tetanization due to its: Rythmicity Automaticity Conductivity Long refractory period. Increase peripheral resistance is due t o : A. 19.. 32. Hepatocytes. Increase vasomotor tone. 24. Sharp odors have the quality of water & lipid solubility. 22. Affection of Nasal mucosa. 28. Unilateral Anosmia is due t o : A. Cortisol decreases which of the following cell: A. 3 1 . 30. Clonus. Type II pneumocytes. 20. Surfactant is released b y : A. 29. 25. 2 1 . The most probable underlying pathological process is: A. 9. 7. Which of the following is not a mediator of acute inflammation: A. Leukocyte Adhesion. Her mesenteric veins were patent. ICAM & VCAM mediates: A. 8. A 25 year old lady with 14 weeks pregnancy had a road traffic accident brought to the emergency department with a large open wound on thigh and femur fracture. 3. Fat Embolism. Regarding Fat Embolism: It is Fetal in > 8 0 % of cases. Dopamin. 2. What is inappropriate about fat necrosis: A. A 16 years old boy deeply jaundiced presented with gum bleeding due to: . 11. Proteins >3gm/dl. At laparoscopy most of the bowl loops were dark purple black. A 30 year old women brought to emergency department with bleeding….. Only caused by trauma to the fat tissue.GENERAL PATHOLOGY 1. A 36 years old women presented with acute abdomen. 4. Wet Gangrene. 13. 12. IL1 & TNF α. Superimposed infection on necrosis is called: A. Fat necrosis occurs in: A. After 2 days of accident she suddenly collapsed & died. 6. Gangrenous Necrosis. Fluid accumulates in acute inflammation contains: A. 5. Irreversible cell injury starts with: A. The most probable pathological process involve in her death: A. Rupture of the Lysosomal membrane. IT manifest within 12 hours. 10. Fever in inflammation is caused by: A. Acute pancreatitis. 3 Months. 22. 18. Hydatid Cyst. B. . blind ended vagina with no uterus & ovary. Which of the following is a benign tumor: A. RBC. Congestive cardiac failure. An 18 year old girl came to family doctor complaining of primary amenorrhea. Her probable Karyotype is: A. Leukoplakia. Warthin`s tumor. Erythroplakia. Edema due to increase hydrostatic pressure is seen i n : A. Vitamin A deficiency. Which of the following is a premalignant condition: A. Cystic hyperplastic endometrium 20. The primary source of Creatinin is: Liver. 1 Year. Metaplasia does not occur i n : Brain. Cardiac muscles. 2 1 . 46 XY. 17. the diagnosis of Tuberculosis in this patient is confirmed b y : A. Over an unpredictable period of time. Maximal tensile Strength of a wound is attained in. Vitamin K deficiency. 16. Which of the following is a premalignant condition: Condyloma Metaplasia Endocervix. Presence of caseous necrosis in lymph nodes. 15. 14. A patient presented with enlarge lymph nodes.A. Dead fetus. Dystrophic Calcification is seen in all of the following except: Malarial parasite. 19. Skeletal muscles. 23. Lungs. on examination she found to have well formed breasts. Usually asymptomatic. 26. The mechanism of injury of ionization radiation is: A. 4. 3. SPECIAL PATHOLOGY 1. . Typhoid carriers are: A. Increase plasma volume. LASER act by: Cutting. Atrophy of the gland is because of: A. Regarding autosomal recessive disorders: ……………… 25. In a patient there is atrophy of submandibular gland due to its duct obstruction. Anemia of pregnancy is due t o : A. Perianal Lesions. 28. Hepatocytes. Which of the following are not non-dividing cells. A patient admitted to the hosp. Apoptosis. Adenocarcinoma. A. Hepatic hemengioma is associated with: A. Which of the following is the confirmatory test of AIDS: Western blot. Biochemical changes 29.6 days. Infective Endocarditis. diagnosed as case of gas gangrene dies due t o : A. 7. Which of the disease is common in I/V drug abusers: A. Embryological marker that reappears in circulation in Ca Colon is: A. 10 days. 2. 5. Free radical formation. 6. 27. 24. ELISA. Which of the following is peculiar for crohn`s disease is: A. Vinyl Chloride. Toxic shock 30. CEA. Intestinal metaplasia due to reflux esophagitis leads t o : A. Absence of Myenteric plexus. Major complication of severe burn: Formation of granulation tissue. heat intolerance on examination her pulse is 112b/min. Decrease glucocorticoids. Malignant Tumor that will not metastasize: A. 10. 20. In pulmonary embolism. 18. Carcinoma formation. BP 120/80.8. Clear airway. There is increase PO4 renal absorption. Achlasia is due t o : A. respiratory failure is due t o : A. 1 1 . 14. 19. Hyperthyroidism. The most probable diagnosis is: Lambert Eaten Syndrome. Hereditary Spherocytosis. 2 1 . After a traffic accident a lady is brought to a hosp the IMMEDIATE step you do: A. Ventilation / perfusion mismatch. Basal cell carcinoma. . What is Inappropriate about hyper parathyroidism: A. Effects of hyper parathyroidism in Ca. Resection of Anterior lobe of pituitary will lead t o : A. R/R 20 br/min. Mysthenia Gravis. 9. A 25 year old lady complaining of palpitations. Increase pulmonary arteriolar Pressure. Ameobic infection reaches lungs via: Direct extension from liver. 13. Left ventricular failure will lead t o : A. She is most probably a case of: A. 15. A 6 year old boy is complaining of proximal muscular weakness was found to be Ca++ channel antibodies positive. lung is due t o : Parathyroid like protein Parathyroid hormone 17. Immediate effect after injury to a vessel i s : Vasoconstriction 16. Spleenectomy will help in treating which type of anemia: A. 12. By aspiration of trophozoits of Entameoba histolitica. 24. PHARMACOLOGY 1. Heparin will inhibit: . I/V nitroglycerin. Gout. Septic arthritis. His knee is hot. Labetalol. Ketamin is used as anesthetic in repeated dressings of burn patient because: A. Which of the following is drug of choice for patient of status asthematicus: I/V aminophylin Oral steroid I/V salbutamol. Pseudo gout. Metaclopromide. At the time of induction of anesthesia. Drug that decreases the tone of lower esophageal sphincter & increases gastric emptying: A. Which of the following blocks α and β receptors: A. In thyroid disease the antibodies are directed against: A. It relieves pain as well. tender & swollen. Thyroglobin.000/mm3 with 75% neutrophils. He has a low grade fever. 22. The most likely diagnosis is: A. Cholesterol 23. 3. In Hypothyroidism there is increase in: A.By aspiration of ova of Entameoba histolitica. crystal analysis = negative birefringent. Via Portal vein. 6. 4. A 50 years old man is awakened in the middle of the night with acute pain & tenderness of the right knee. gram stain = negative. Analysis of fluid from right knee shows: leukocytes 70. 7. C. During surgery antibiotics should be given at: A. 5. B. 2. Which of the following is suitable antihypertensive for asthma & IHD patient during surgery: I/V sodium nitroprusside. 9. Paracetamol FCPS Surgery July 2007 paper • by drwatson » Thu Jul 22. Morphine. B. Acid suppression is done by blocking: A. Drug of choice for acute pancreatitis.A. Clot propagation. H2 Receptor. Pethidine. 8. 2010 5:00 pm 1 trapezious body relating to auditory pathway MLF dorsal column-medial leminiscus laterl column 2 which one of these cells is not a non dividing cell neurons nerve cells myocardiocytes hepatocytes 3 papillary muscle derived from ectoderm endoderm mesoderm neuroectoderm 4 LASER works by cutting thermal . Clot organization. Occulocardiac reflex mediated by .crystalization 5 which of the followings is the benign tumor wilms tumor mesothlioma hamartoma 6 internal cremastaric fascia dervied from fascia tranversalis fascia internal oblique fascia tranversus abdominis 7 Regarding bone regular lamellae irreagular lamellae horizontal canals oblique canals 8 ADH works at distal convulated tubule collecting tubule distal convulated n collecting ttubule proximal convulated tubule 9 commonly rib fractures at angle neck shaft body 10 right umblical artery becomes left medial umblical ligament right median umblical ligament left medial umblical ligament median umblical ligament FCPS past papers surgery june 2005 Dby drwatson » Fri Aug 13. 2010 11:53 am 1. What will be with blood insulin and Glucogan levels at the time when he crosses the win line? 5.1 gram of protein contains calories? a) 4 calories 3-7th lobe of lung is called a) medial basal 4-A marathion runner wins a race.Mandibular fracture occurs just brfore the mandibular foramen.a)CN5 2. a) loss of taste to anterior 2/3 of tongue b)Myelohoid muscle is unable to stabalizse mandible? c) Loss of sensation to lower teets? 6-Head of femer a) has anger of 125 with shaft b)posteriorly completely covered by capsule? c)epicondyls are in parallel line with shaft 7-Lymph nodules are present in a)Spleen b)Thymus . recurrent MI should be treated with? a) Anticoagulants b) Lipid lowering drugs c) Beta blockers 12)Stratified squamous epithelium is present in a )palatine tonsils b)pharynx c)Ovary 13) Low voltage QRS comples ios seen in .c)Lymph nobes 8-In DIC .there is a)decreased palatlets 9-what is inferior in perenium? a) pereniul membrane 10)spleen is a) mesodermal b)endodermal 11)according to recent recommendations. followin if true about it onset of action .2 15)Head of Humerous is supplied by a) Anterior circumflex artery b)Posterior circumflex artery c)Suprescapular artery d)Subscapular artery 16) Gene are composed of a) RNA b)DNA c)Extrons d)Introns 17) I/V dopamine is given to a patiend.a) MI b) Bundel branch block c)Hypertension d)IHD e)Pleuritis 14)Isthmus of thyroid is present at a)C1. a) 5 min b)5 sec c) 60 sec 18)I/V sedation is given to a patient. Following is true about its arm-brain circulation a)60 sec b)5min c)30min d)5 sec e)? 19) DVT is most common in a)Femoral vein b)popleteal vein c)Superficial vericosed veins of leg 20) Which of the following is not a tumor marker a)Acid phosphatase b)PAP c)Hcg . the contents will go to a) Right iliac fossa b)Left Iliac fossa c)Anterior subhepatic space d)posterior subhepatic space e) small bursae .d)AFP e)CEA 21) Pus contains a)Dead bacteria b)Dead neutrophils 22) The most common cause of fatty liver in our society is a) Alcohol b)Hepatitis B and C c Drugs 23) Carotid bodies respond to a) increase blood H+ b)Increase blood co2 24) In anterior Duodenal perforation . with Hypercellular bone marrow/\ . palatelet 450. c)5. e) 9 30-A patient has hb 6.25) Metabolic alkalosis results from a)Ingestion of ammonium chloride b)ingestion of carbonic anhydrase inhibitors 26)A Diabetic patient fas urine ketones and urine glucose ++ the cause is ? a)Hyperglycemia b)Insulin difficiency 27 Response to chemotherapy is because it causes a)Atrophy b)Apaptosis 28)Dorsal column damage results in Ataxia because a)loss of pathway from dorsal column to thalamus b)Loss of proprioception input to cerebellum c)Loss of proprioception to Thalamus d) Loss of proprioception to Hypothalamus 29)Which of following nerve passes dorsal to brain stem? a) 3. d) 12.000. b) 4 . a)Due to drug he is using for anaemia b) Leukemia c) Iron deficiency anaemia d)G6PD deficiency 31) The sign of cerebellar disease is a)Adiadohakinesia b)Static tremor c)Hypertonia 31)Neurohypophysis drain secretions a) Adrenergic neurons b)cholinergic neurons c)free nerve endings 32)CSF a)has osmolarity <1040 b) pressure <10 mm of water c) protein >200 33) Stimulation of Alpha adrenergic neurons . After how much time .34)End arteries are present at a)Brain b)Heart c)Bone 35) In Tempomandibular Joint Dislocation. He is breathing 100% oxygen.What structure prevents the the joint to go back in cavity? a) Tubercles on articular surface of mandible b)Tempomandibular ligament c)Lateral Ptyregoid muscle d)Medial Ptyregoid muscle 36)Where the gangliom joining the superior petrosal nerve and inferior petrosal nerve is located? 37) A patient has hypercarboxyhemoglobin.his blood will have 100% oxygen? a) 1 min b)2 min c)5 sec d) 5 min 38) FRC is? a) TV + ERV . b)IRV + ERV 39)Aldosterone stimulate a) Melatonin? b)? 40)A patient has fever 40c with flank pain .\ a) Blood culture b)urine culture c) Blood and urine culture 40)Following is not true? a) HCV positive carriers have not been reported 41) Pulse pressure is increased in a)Arteriols b)capalleries C)Aorta 42) A grand father was playing with his grandson with holding his wrist and rotation him in a circule. What happent ? .Suddenly the child starded crying and sit with holding his forarm in porn position.increases frequency and chills and riggors. a) Proximal Radio ulnar joint dislocation b)Distal Radio ulnar joint dislocation c)Fracture of acromion 43) Regarding typhoid fever a) Asypmtomatic b)Carriers do not excreate causative agent in fees c) Asypmtomtic carriers should be isolated and treated 44) Incision is given on Rt Lumbar area.external oblique-internal oblique-transverse Transverse fascia-exta peritoneal fat.superficial fascia. The layers incised will be a) Skin-superficial facia-deep facia.peritinium 45) Carpus callosum a) connects the two hemispheres 46)Most constricted part of male urethra is a) External meatus 47)Superior Thyroid glands a) Are located behind the Thyroid gland outside the facia b)Have blood supply from the superior thyroid arteries 48) Which of the following is true .peritoneum b) skin.extaperitoneal fat. a) Piriformis attach to inferion trochanter b) Schiatic nerve passes through superior sciatic foramen 49) A structure lined by cuboidal epithelium. and its center contains two calcified bodien of pink colour. What is it? a) Prostate 50)25 cholecalciferol is not converted to 1.25 cholecalciferol in a)Hepatic failure b)Renal Failure 51) Which of the following is not present in posterior relations of kidney? a) colic flexure 52) Rt border of heart is formed by a)Rt atrium 53) In pregnancy .Lactation does not occur because a) Increased Level of progesterone and Estrogen 54) Which of the following have Pulmonary cycle? a) Ascariasis 55)Causative agent of meningitis in 60 years old man is a) St pneumonia . 56) Cause of death in pneumonia bue to shock is a)Gram -ve bacteria b)Pseudomonas c)st pneumonia 57) Diphtheria exotoxin hane powerful effect on a) Larynx b)Heart c)Nerve endings d)Kidneys 58) Aspiration of peanut will go in a) Right lower bronchus 59) A female with butterfly ace. Flank pain could have a) SLE 60) A young patient has difficulty in swallowing a) Sclerodermia 61) A patient with SLE have a) Possitive ANA b) Possitive anti SS .joint pain. Na is absorbed totally neutrally active transport? a) PCT b)DCT c)ATT 76) Cell mediated immunity is mediated by a) T cells b)B cells . cell swelling occurs due to a) Entry of water in cell fromj surrounding b) Entry of lipids 64)What is potent Antoxident ( Anti Aging) a)VIT E 65) A boy is excreting meconium from rectum and has severe constipation.62) Incision on Supra pubic area caused heavy bleed. The condition is called A) Intussusceptions B) Hirschsprung's disease 66) In what portion of nephrone. It could be a) Damage to inferior epigestric artery? 63) In Hypoxia. The rectal examination showed no ganglion cells. 77) Virus does not cause a) Lipoma b) Kaposi sarcoma c)cervical cancer 78) Iodine is stored in thyroid follicles as a) Thyroglobulin b)DIT and MIT 79) A patient died due to colonic cancer. It could be a) anomaly is due to premature separation of ureteric bud 81) Chylomicrons contains a) Fatty acids with vitamins 82) Transudate has a) Low Albumin 83) common mediator of Basophils and mast cells have a)Histamine 84)In Skeletal muscle .On autopsy found thousands of polyps. contraction occurs due to . Patient's father also had colonic cancer. The cause could be a)Adenomatous polyps? 80) A patient has double ureters and double pelvis. there is predominate a) Fibroblasts b)Macrophages c)neutrophils 89)Characterics of malignant cancer is a)Metastasis 90 )Characteristic of malignant cancer is a) pleomorphism 91) Pretraceal fascia a) completely encircle the Thyroid gland b)Completely encircles trachea 92) 6 months after the resection of terminal ilium. What could be happen? . the patient a) iron deficiency anemia 93) A patient has sweating 2L . and he took 2 L plan water.a) Sacromere b) Actin c)Myosin 88) In chronic inflammation. a) Increased body volume? b) decreased intracellular osmolarity c)Increased extracellular osmolrity 94) Inhibition of heart Rate will be in a) Stimulation of parasympathetic nervous system 95) Aldosterone is stimulated by a) Increase of serum K 96) Follic Acid difficiency causes a)Megaloblastic erythropoises b) Foliate deficiency 97) Difference B/w Plasma and interstitial fluid is a) Protien 98) C02 is mainly carried by a) Hco3 99) In Inguinal hernia. posterior wall is formed by a) Conjoint tendon b)Internal oblique muscles 100) What is part basal ganglia are a) Putamen . What is damaged? a) serratus anterior muscle 107)Thurst is stimulated by a) ADH .( 32 mistakes ) Paper 2 101) Blood supply of vertebral column is a)Vertibral artery 102)Suprascapular nerve arises from a) Thyrocerv ical trunk 103) Ist pharyngeal pouch forms a) Auditory tubes 104)Diapharm is supplied by a) c 345 105)Urachus is remnant of a) Allantois 106)A patient had surgery of breast and he is now unable to lift his arm. Lost adduction of thumb also. Falciparum Ligament. following test is appropriate. Gastroduodenal ligament 111) A patient can not abduct and adduct his fingers.108) A patient has dirrhoea and developed metabolic acidosis. a) Deep branch of ulnar nerve 112)Femoral canal is formed by what fascias? a) Fascia Lata + memberenous Layer b) c) 113) Organ . lined by stratified squamous epithelium. . What is lost in feeces? a) HCO3 109)phasic fast adaptive structure is a) Mickle disk b)Carotid baroreceptors c)Rufini's carpuscles 110)The lesser omentum forms which structures? a) Hepatoduodenal ligament.? Palatine tonsils 114)Regarding the Iron requirement of a pregnant woman. rounded structure. a patient might develop a) Megaloblastic anaemia 116) A patient got injury in the posterior triangle of neck.a) TIBC b)serum ferritin 115)After gastrectomy.there might damage a) spinal part of accessory nerve b) CNS part of accessory nerve c) Vagus nerve 117)A patient in RTA came in EAC with hypovomumic shock. First what to treat? a) I/V Fluids 118) Clostridia cause a) Gas Gangren 119)Angiotensin 1 is converted to Angiotensin 2 in a) Lung capillary endothelium 120) First week diagnosis of Typhoid is a) Blood culture 121)A soldier posted at high altitude for many years came witg PVC > 40 . and drained by single vein .a) Secondary polycythemis 122) Buerger's disease is strongly associated with a)Smoking 123)Acute appendicitis is associated with a) Neutrophelic leukocytosis 124) Medial arcuate Ligament encloses at its upper end a)Psoas muscle 125) QRS comples is due to a)Ventricular systole 126) The transitional epithelium have a) Couoidial epithelium lined by more larger and rounded cells? 127) A patient have right optic tract damage. The causative agent is a) EBV 129)Regarding adrenal glands a) Right is Larger than left b)Supplied by numerous arteries . He might developed a)Left homonymus hemianopsia 128)In Burkit Lymphome. cells a) Matured in Thymus after birth b)Both T cells and B cells are derived by same precursors 133)Neurovascular bundle is located at ribs at a)inferior border of ribs 134)Which of the following does not predispose the cancer? a) Radiation b)Bacterial products c)virus d)Chemicals 135)Methicilline resistant staph are a) Usually sensitive to Vancomycin 136) Cardiac muscles can not be tetanized becosed .130) Repture of middle meningeal artery cause hematoma in a)Between the two layers of dura mater 131)Middle esophageal constriction is by a)Arch of Aorta b)Left bronchus 132) T. a)They have long refractory period 137) Severe blood transfusion reactoion occurs if transfused all except a) A+ blood to 0+ b)A + blood to AB+ c) B+ to O+ d) B+ tp AB+ e)O +ve blood to OA+ 138)Xenograft is a) Transplant of tissues of different species 139) Lysosomes contains a) Acid Hydrolase 140) Rt Gastro epiploic artery is the branch of a) splenic artery 141) Right gastroduodenal artery is the branch of a) Hepatic artery 142 )An asthematic patient may have FEV1/FCV <75 143)Ligamentum arteriosum connect with . What blood test you will advise him after one week? a) Anti streptolysin O titer 150)A patient developed hemolytic anaemia after blood transfusion.RR 20. therer is a) Pulsus deficit 145)A patient lost weight .a) Aorta with Left pulmonary artery 144)In atrial fibrillation. pulse 120/min. DB 120/80 a) Hyperthyroidism 146) Alpha receptors stimulation may cause a) Lipolysis b)Gluconeogenesis c)glycogenolysis d) Glycogenesis 147)Trypsinogen in the duodenum will be activated by a) PH < 7 148) Pulmonary embolism most commonly involves a) Left pulmonary artery 149 A patient had streptococcal infection. hot dry skin. Hemoglobin released will be attached with . fever with Antimitochondrial antibody +ve a) Primary billiary cirrhosis 152) PDA occurs in a) Prematurity 153) Fertilization occurs in a)Oviduct 154) Liquificative necrosis occurs in a) Brain 155) Temperature regulatory center is in a) Hypothalamus 156) Skin cancer occurs a) In Sun exposed area 157) Giant cells have a) Macrophages 158) Inflamatory mediator is a) C5a .a) Heptoglobin b) Albumin 151) A patient with itching. What is the diagnosis? A} Sarcoidosis B} Silicosis .159) Optic tract ends in a) Lateral geniculate body 160) Gastric motility is increased by a) Acetylcholine 161)Wound healing is delayed by a) infection 162)Achalasia is due to a)loss of Aurbach plexus 163)Patient has severe chest pain with ST elevation a) Disectening aorta b)Pleuritis c)Pneumothorax d)costochondral e)Pneumonias 164)Patient X-Ray shows hilar lymph nodes. Granuloma but no necrosis. which of the following enzyme will be raised? A) CK-MB a .I. High output cardiac failure a. B6 167)Valve less vessel is A) Aorta B) Pulmonary artery C) Coronary Sinus D) Pulmonary Trunk E) SVC e 168) Within 1 hr of the Acute M. B1 b. B2 c.C} Tuberculosis 165) Strongest layer of small intestine is a)Circular b)longitudinal c)Mucosa d)Submucosa ok? 166). B) LDH C) Alkaline Phosphatase D) AST E) Troponin T 169) Replacement of cells of other normal site of body is called a) Metaplasia 170)Heart sound produced by rapid ventricular filling is A) 1st heart sound B) 2nd heart sound C) 3rd heart sound c D) 4th heart sound 171)Corticosteroids decrease a) Neutrophils b)Lymphocytes b c)Eiosinophils d)Monocytes e)Basophils 172) DVT occurs in females . by use of OCPs 173) Visceral Pericardium is supplied by a) Phrenic Nerve b) Sympthatic Nerves c) Vagus neve d) Cardiac Plexus 174) Basal Cell Carcinoma involves a) Buccal Mucosa b) Hard Palate c) Soft Palate d) Lower Lip e) Oral Cavity 175) Primary spermatocyte divide by a)Mitosis b)Spermiosytosis c)Primary meiotic devision d)Sec Meiotic devision 176) The volume of distribution of drug is not influenced by . arterioles b. precepillary sphincter .a)*** b)Age c)Heart failure d)Renal Failure 177)In Hypothyroidism a) Increased cholesterol level 178)craniophrangioma in the center of optic chiasmata may cause a) Bitemporal hemianopsia 179)Which one of the following most likely causes Increase in GFR? a)Constriction of afferent arteriole b)Constriction Of efferent arteriole 180) cardic output measured by thermodilution method is called a)fick law? 1 8 1 . Blood flow regulation is mediated by: a. venules 182)Thiamine deficiency causes A} Peripheral Neuropathy B} Pellagra C} Chelosis D} Dermatitis 183) Which of the following is not a feature of shock A.c.decrease cardiac output B. capillaries d.transferrin .muslce weakness C.tachycardia E.ferritin B.increased urine output D.restlessness 184) Iron is stored in the form of A. apoferritin .C. .....(smethng like dat exactly nt 5-which 1 contain granulosa cell... Recalled By Syeda.diagnosis of failure of ovulation by ?? . a-palatine shelves b-maxillary prominence c-med nasal prominence d-lat nasal prominence e-incisive foramen 3-germ cells derived from. 6-in primary infertility.. a-anticonvulsant b-multiple preg c-multifactorial d-inc maternal age 2-left cleft palat dt malformation of. 1-Etiology of cleft palate.gynae n obs 24-Mar-12 4:32:00 PM GYNAE OBS JUNE 2009 Gyn Obs JUNE 2009.... remembr) a-45XO .. a-graffian follicle b..... a-ectoderm bendoderm cmesoderm d-neural crest cell 4-karyotype of turner syndrom.. ........a-wid lower abdominal pain......... endotoxemia . d-thick mucosa sec frm cx wch nt show fern pattern. 11-shigella. 9-cornybacterium diphtheria......??? ....... 7-pentamer immunoglobulin is??? a-IgA B-IgG c-IgM* d-IgD e-IgE 8-Regarding entameba histplytica. a-it is a nematode b-it causes liver disease c-occurs in both trophozoite nd cyst stage d-it exist normally in intestine e-......... b..... c- 10-mode of cell injury by clostridium a-ATP depletion....... b-rise in basal body temp c-middle of menstual cycle. a-gm -ve bacilli bclub shaped* . endemic sporadiac pandemic epidemic ..... . ......... IL1 IL6 TNF ALPHA LIPOPOLYSACHARRIDE 15-yellow fever inf occurs .............. ethambutol quinine phenothiazine chloroquine . 13-drug inducer by p450 mechanism........................ 14-last mediator of endotoxic shock....... ....neurotoxin verotoxin enterotoxin exotoxin 12-corneal opacity caused by... . cimitidine ketokonazole phenobarbitone ........... ...... ..16-wall of inguinal canal Not formed by??????? ext oblique apponeurosis lacunar lig internal oblique conjoint tendon transversalis fascia 17-endocervical polyp in pregnancy..........wat type of sampling it is Random sampling (other options nt remember. ...... (these sort of alien words were written) 22-bone scan ...nt sure) 20-ultrasound........… 18-vaccination of mycobacterium T.sampling is taken a/c to age …....nd methaemoglobin. (name of sme disesases were written) ... 4-8 weeks transabdominal u/s.......stratified etc ......B based on Type I hypersensitivity Type II Type I I I TYPE IV Cell mediated 19-after subdivided no of peoples into gps . ............. Physiological . fetal heart sounds can detected Cresenteric line seen w/h cnfirm pregnancy....(smething like dat ) 21-MRI Frequencies... fats ..23-incidence.. No.and protein storage Insulin 28-fetal brain development Growth hormone Thyroid hormone 29-w/h of the following amino acid should be taken in diet Alanine Tyrosine Glycine’glutamine 30-folllowing phagocytic cell present in circulation? Monocyte . of new cases in a population in a time given 24-pie chart 25-wch 1 is absent in muscles Glucose 6 po4ase Hexokinase 26-end product of glucose metabolism Lactic acid 1molecule of pyruvic acid 2 molecule of pyruvic acid 27-hormone ass with glucose ... Rectovag fistula Rectouterine fistula Imperforate anus Rectoperineal fistula 34-structure lying b/w labium majus nd minus Urethral gland Bulbourethral gland Minor urethral gland 35-w/c 1 of following muscle not attached to tendon of perineal body Ischiocavernosus Ext urethral sphincter Bulbospongiosus Transverse perineal muscle 36-lymphoide nodule nt present in ..Macrophage Lymphocyte Neutrophils 31-morphine can be given in Biliary cirrhosis Terminal ill patient of cancer Acute pancreatitis 32-functional layer of endometrium consists of Stratum basal Stratum compactum nd stratum spongiosum Stratum basal nd compactum 33-maldevelopment of uro-rectal septum . Synovial... nd elasticity..... 41-hamartoma is.......Thymus Spleen Lymphnode 37-Notochord forms Neural tube Cns 38-epithelium of ovary Simple cuboidal 39-pivot jt include Atlanto occipital jt Atlanto axial jt 40-sacroiliac jt Fibrous .....tubular glands in lamina propria Vagina Esophagus Oropharynx Duodenum 43-post relation of left ureter .. vascularity... 42-an organ lined by stratified sq epi with inc cellularity....... ........ . Puckering of skin dt subcut ductal infiltration....delivery nd postpartum 51-pacinians corpuscles Tactile touch ..........lie in deep relations Can mov freely over retromammary space Peau’d orange caused by lymphatic obstruction.............. 50-lady died dt amniotic fluid embolism During delivery During c/s nd post partum After post partum During labour.... seratus ant..Root of mesentry Iliac vessel 44-outer lat quadrant of mamry gland drain into Ant gp axillary lymph node 45-xerophthalmia dt Vit A def 46-anti-oxidant Vit E 47-w/h st having 3 layers of muscles Fundus of stomach Esophagus 48-fat embolism 49-regarding mammary tissues w/h is Not appropriate Pect major... .... 57-in left shift O2 hbdissociation curve w / h 1 is increase Fetal hb .PH=7..HCO3=INC-............Fine touch Temp Pain Vibration(I think it was nt in option nt sure) 52-H+ exchange in proximal tubules with HCO3 reabsorption K secretion 53-pat wid severly vomiting came wid breathlessness..PCO2=55 Metabolic alkalosis Metabolic acidosis Compensatory metabolic alkalosis 54-uterovag prolapse w / h lig is injured Broad lig Round lig Transverse cx lig 55-alkaline po4ase rises in Prostatic ca BPH Pancreatic ca 56-in ca mammary gland w/h drug is given in post menopausal lady Tamoxifen Clomiphen .....5. 60-lady wid reddish cheeks..... .................. 59-intravascular haemolysis diagnosed by Reticulocytosis Inc in hepatoglobulin binding Inc TIBC . 62-acute inflammation Transudate Exudate Inc protein content 63-w/h drug causing minimal GI side effects Paracetamol Aspirin Ibuprofen Ketorolac Diclofenac ...................58-in thalassemia there is Poor hemoglobinization Dyserythropoiesis ....……......... SLE 61-in metaplasia w/h is inappropriate Irreversible ....arthritis... .........…....R >100 HR>.64-in 6 yrs old child meningitis occurs dt w/h organism H-Infuenza Ecoli Niesseria 65-sepsis is diagnosed most appropriately by Blood culture Granulocyte <2000--->15000 H.......... 67-virus not associated with ca in human HSV HBV HDV HPV Papova virus(papilloma vitus +polyoma virus) 68-w/h is more in human milk as compare to cow’s Lactulose 69-gonococcus is best easily diagnosed by Gram staining Culture ZN stain 70-w/h is completely covered by pretracheal fascia . 66-w/h is transmitted by mosquito Yellow fever .. .Thyroid gland 71-is not an embryonic remanant Lat umbilical ligament 72-oxytocin nd vasopressin originate frm Hypothalamus Neyrohypophysin Autonomic ganglion 73-sucrose Is a diasaccride Metabolized into 2 glucose molecules Reducing or non reducing (nt remember) 74-w/h of the following dis not ass with carcinoma Lysosomal storage dis ..definition 78-verbal communication skill Speak fluently Education nd research training Presentation ........................ 75-rough endoplasmic reticulum Protein synthesis 76-fragility of rbcs membrane Spectrin 77-behavioural sciences .......... ......... ................ Ataxia Rigidity Initiate muscle movement 84-pre-motor cortex lie in Frontal lobe 85-coccygeal part of spinal cord lie at the level of w/h vertebra C2 L2 .... 82-w/h 1 decrease the muscle length Golgi tendon organ Muscle spindle ...Good vocabulary 79-Not a function of beta blockers -ve ionotropic -ve chronotropic ....................alpha blockers 81-centrally acting HTNsive Clonidine Methyldopa ......... 80...... 83-cerebellar lesion............ S1 S2 86-lie in true pelvis nd crosses frm med to lat side of pelvis Sup rectal art Middle rectal art Inf rect art Internal iliac 87-inf fascia of urogenital diaghphram also called as Perineal membrane 88-female pelvis also called as Android Anthropoid Platypelloid Mesatipellic 89-at crowning w/h muscle is cut during episotomy Bulbospongiosis 90-lumbar puncture L2 L3 L3 L4 L4 L5 S1 S2 91-Lactation...wh stimulate ... Hormones.... .. 97-teratogenic Alcohol Caffeiene 98-carcinogenic Methyl alcohol Benzidine 99-pat with blood gp A should nt given A+ A_ O AB .. 93-pregnancy is diagnosed by Gonadotrophin hormone 94-urogenital diagphram inserted on Ishchial tuberosity Ischial spine Sub pubic arch Ischial ramus 95-w/h muscle div submandibular gland into superficial nd deep part Mylohyoid Ant belly of diagastric 96-root of left lung Azygous vein arched at root of left lung..92-prolactin hormone Stimulated by........ . Biparital diameter... 9.... Regarding kidney . wks ...... Abd circumference...w/h sequence is correct renal art > arcuate>interlobar>interlobular>glomerular>efferent arteriole renal>interlobular>interlobar>arcuate>efferent arteriole>glomerular renal>glomerular>interlobar>interlobular>arcuate>afferent arteriole renal>arcuate>interlobular>glomerular>interlobar>afferent arteriole 101-in muscle ca+2 release frm sarcoplasmic reticulum T-tubule Sarcomere 102-sarcomere b/w 2 z line 103-parasympathetic nerves r 3.100-. 10 104-w/c is present in de cavity of cavernous sinus Abducent nerv Trochlear Occulomotor 105-in haemophilia PT INC APTT INC Inc bleeding time 106-gestational age estimated frm. 7..... 16 sec b….half life of dopamine is 5 min .2008 Gynaecology n obstetrics By SHEHER BANO.wt is the most potent stimuli for aldosterone release.size at birth is equal to adult size ..10 sec 2.Crown rump length .which statement regarding adrenals is wrong.15 sec b….. a…..8 sec c….ACTH b….. a….2008 June 25..wt is the circulation time b/w brachial n cerebral circulation.30 sec c…. how much time will it take to reach the steady state concentration.. 107-norepi + serotonin degraded by MAO 108-at last trimester w/c uterus is sensitive to w/c hormone Oxytocin 109-to prevent heat loss Shivering Vasoconstriction 110-inc K Aldosterone sec Gyn Obs June 25. a…. 1. 8 weeks. a….60 sec 3.Renin 4. thyroid isthmus is at level of a….1 kcal 11.4.B cells 8.5th 6th tracheal ring 12.1 k cal 10.actions of glucagons are all except 6.3 k cal c…..internal juglar arteries 13 end arteries are present in a…..spleen b….1 k cal b…. Min 9.heating at 151 degree C for ….IgG b…...5.spinal arteries c….T cells d….3rd 4th tracheal ring c….Ig M c….blood supply of spinal cord is a…...brain .vertebral arteries b….9.which of the following are features of type 2 hypersensitivity reaction a…..5.2nd 3rd tracheal ring b….actions of growth hormone r all except 7.9.energy liberated on fat metabolism is a…..energy liberated on complete metabolism of protein is a….wt is the requirement for complete sterilization a…. .derivative of endoderm is a…....4th week b…5th week c….pudendal nerve b….epthelial lining of ureter is 21.patient feels pain on defecation nerve involved is a….almost 10 to 15 questions on NSAIDs and prostaglandins 19.superficial transverse perineal + bulbospongiosus 17.wt makes right border of heart a….derivatives of pharyngeal arches (2 to 3 questions) 24.muscular layer of ureter in lower third is 23....one of the following regarding levator ani is true 20..muscle cut on episiotomy is a….rt ventricle c….factors affected by vitamin K deficiency .laryngeotracheal bud appears during a…..chemoatractant for neutrophil is. 26.lymph node 14.c…..7th week 16.rt atrium + rt ventricle 25.mesentry 15.inferior rectal nerve 18.rt atrium b…... cortisol causes decrease in a….RBCs c….int n ext iliac nodes b…......ext iliac nodes 28.27.. 35...point of meeting of intrinsic n extrinsic pathway 29.oxyhemoglobin dissociation curve (2 questions) 30.CSF 33.platelets b….which has highest protein content a….lymph c….neutrophils 38 difference b/w benign n malignant tumors is( 2 almost similar questions) .structure palpable through lateral vaginal wall is 37.int iliac nodes c….autosomal dominant n recessive disorders (2 questions like name was give n was asked wt is wt) 34. the site for direct inguinal hernia.protein content of CSF is 32.plasma b….. are… 36.simple sqamous epithelium is present in all except 31..boundaries of inguinal triangle.lymphatic drainage of cervix is a….lymphocytes d….origin of ext oblique muscle is from….. coccyx B.Lymph drainage of perianal skin is from one of the following A.forms falciform ligament C.capsule OBST&GYNEA FCPS-1(OCT 23 0 7 ) Q & A’s.contributed by Dr.A pt presented with pain on defecation after severe constipation.related to left adrenal gland .vertical group of nodes 4.sacrospinous ligament 2.Lesser omentum A.Huma 1.autonomic nerves B.medial group of horizontal superficail nodes E.pudendal nerve D.sacrotuberous ligament E.Ashh & Dr.Internal pudendal artey C.metastasis c….pain in constipation occurs due to ? A.perineal nerve E.lateral group of horizontal superficail nodes D.external iliac nodes C.internal iliac nodes B.a….invasion b….ishcial tuberosity D.pubic symphsis C.Which of the following does not form the wall of perineum A.connects with deudenum B.inguinal nerve 3. vesicouterine pouch 7.rectum C.retrouterine pouch E.A young girl of 25 yrs went with her husband on honey moon after one month of marriage.perineal membrane D fascia E fascia 8.she suddenly experianced severe spasmotic pain in her rt iliac fossa.which of the following structure is not palpable through vagina A.diuretic use D.the inferior fascial layer also called as A.ureteric stone D.ectopic Pregnancy E..colle's fascia B.acute appendicitis B.The peritoneum covers the anterior abdominal wall which is continous with the perineum.If a known hypertensive pt took some diuretic n died in 6 hours.n presented in emergency in state of shock.intestinal obstruction D.renal colic C.most probable diagnosis is A.D.it has fascia which has superior n inferior layers.acute cholecystitis 6.related to left kidney E.hypokalemia ? .cervix B.chronic renal failure C.hypophosphatemia E.related to rt kidney 5.scarpe fascia C.acute renal failure B.wat is the cause A. superior mesenteric C.9.increased urine output D.left renal vein 12.has an abd ostiium which is 0.moves laterally to open medialy in the ovaries B.supply of sphincter urethra comes from A.labia majora B.is 6cm long in adults D.ilioinguinal nerver B.sacral splanchnic nerves D.muslce weakness C.pelvis splanchnic nerves C.Which of the following is not a feature of shock A.restlessness 10.decrease cardiac output B.labia minora C.present at the base of broad ligament C.inferior vena cava B.clitoris .inferior mesenteric D.5mm when dilated E.Genital tubercle in females give rise to A.autonomic nerves E.rt renal vein E.What is most appropiate about uterine tube A.extend above the fundus of uterus 13.Rt ovarian vein drains the following A.tachycardia E.Pudendal nerves 11. ectoderm of 3rd brachial pouch D.3rd pharyngeal cleft 17. absence of pleuro-pericardial membrane B.1st pharyngeal cleft B. absence of septum transversum C.A pt presented with an opening in the ant wall of sternocledomastoid muscle with pussy discharge coming out of it.endoderm of 2nd brachial pouch C.D.endoderm of 3rd brachial pouch E.2nd pharyngeal pouch E.mesoderm of 3rd brachial pouch 16.Thymus develops from A.this problem occured due to abnormality in one of the following A.mekel's diverticulum C D E 15.allantois B.ectoderm of 2nd brachial pouch B.1st pharyngeal pouch D. absence of arch? .vestibular glands E. absence of pleuro-peritoneal membrane D.2nd pharyngeal cleft C.Diaphragmatic hernia occurs due to A.bulbourethral glands 14.urachal cyst is the remnant of A. absence of central tendon E. arises from anterior aortic sinus B.spleen D. supplies left atrium 22.deep inguinal 21.liver B.3rd brachial pouch D. supplies both ventricles D.A node E.Superior parathyroid gland develops from A.3rd brachial cleft 20.both liver n kidney 19.2nd brachial arch C.superficial inguinal E.1st brachial arch B.In cervical carcinoma.external iliac C. supplies S.bone marrow E.18.estrogen .kidney C.wat is the lymph drainage A.Erythropoises occurs in middle trimester mainly from A.enters the rt heart border after passing through sulcus on posterir surface C.internal iliac B.RT coronary artery A.both internal n external iliac D.4th brachial pouch E.Fat deposition n ductal development of breast is the function of which hormone A. dopamine D.oxytocin E.prolactin D.Most imp hormone involve in gluconeogenesis is A.GnRH 24.Inhibition of ovulation is the function of which hormone A.decrease blood volume B.B.prolactin C.estrogen E.progesterone B .decrease plasma volume E. he is diagnosed to have acute appendicitis on flexing his rt thigh inward n medially he cries with pain.A pt presented with rt iliac fossa pain.paracolic B.blood pressure 25. Glycogen 26.cortisol E.progesterone C.wat is the type of appendicitis he has A.growth hormone B.angiotension 2 C.insulin C.Thirst is least stimulated by A.pelvic .salt depletion D.hcG 23.thyroxine D.retrocolic C. short spanchnic nerves E.ovarian ligament C.vagina D.inguinal nerves 29.round ligament of ovary E.hypogastric plexus B.which of the following structure has both intra n extra pelvic extension A.anus 31.One slide of a specimen shows stratified squamous epithelium with vascular smooth muscle and elastic fibres most probably the specimen is A.subcecal 27.Nerve supply of rectum A.transverse cervical ligament D.Stab wound above the piriformis muscle will damage which structure A.paracecal E.uterine tube C.D.uterus E.Apoptosis is inhibited by .pelvis splanchnic nerves D.inferior gluteal nerves C. inferior mesenteric plexus 30.sciatic nerve D.superior gluteal nerves? B.greater splanchnic nerves C.round ligament of uterus 28.broad ligament B.cervix B.tendon of obturator internus E. SLE B.C3a B.C5a is potent anaphylactic C.dermatomiositis 35.ankylosing spondiolitis C.viruses E.C3b n C5a are anaphylactant E 34.what is the most appropiate answer A. Activation of P-53 D.anti SS antibodies .C5a ? C.C5a is a potent chemotactic agent D. Bcl-2 inhibition C.IL-1 D.which is the daignostic in this case A.rheumatid arthritis D.rash n joint pain.arachdonic acid metabolites 33.C3a is opsonin B.Low serum complement seen in which condition A.A.capsases activation B.Most potent chemotactic factor is A.Pt presented with photosensitivity.multiple sclerosis E. apoptosis related protein ?? E… 32. to optimise its function? B.Platelets r stored at room temprature because A.she wil most probably hav increased risk of A.thrombophlebitis D..to maintain its number C.anticentromere antibodies E.osteoperosis n fractures D E.Most dangerous Side effect of estrogen therapy is A. 38. to increase number E 39.ammonia C.antimitochondrial antibodies 36..osteoporosis E 37..to avoid allergic reaction D.none of above D.If a pt is on long term corticosteroid therapy.urea B.uric acid .B.decrease wound healing C. hirsutism C.RA D. suppression of lactation B.anti DS antibodies C.repeated infections B.Which is most imp indicator of muscle protein loss A. 47XXY D.wat is true about DNA A. ? 40. E.proline B.47XY C....normal breasts.Can exist as helix in single stranded form as well as in Double stranded form D.absent uterus.Double stranded in which two strands are antiparallel to each other B.A normal looking Girl came to you with primary amenorrhea.47XXY D.45XO 45. E. .E.46XY B.short stature diagnosed as Turner syndrome..webbed neck.phosphoric acid? D.Hydrolysis of DNA gives A.A pt came to you with primary amenorrhoea. scanty pubic n axillary hairs. wht is her karyotype? A.46XX E 46. 41.on examination she was having short blind vagina wid normal vulva.her karyotype is? A.45XO B.ribose C.46XY C. a single turn of DNA is 2nm C. Which of the following has highest cholesterol content A. 51.Digoxin B. B.spironolactone 50. C..47.Pyruvic acid is used as intermedite in which step of cell metabolism A.karyotyping D.Which hormone convert to its active form at target tissue with the help of 5 a reductase? A.Estrogen C.. D.Griseofulvin C.Cimetidine D.dihydoepiandosterone D.Which is the best option in identification of Turner's syndrome A. Chylomicron remnants LDL VLDL HDL IDL 48. 49.Progesterone E.Ct scan E..carbohydrate to acetyl co A . E.barr bodies C.Androgen E.. Testosterone B.Which drug do not cause gynaecomasia? A.ultrasound B. proline D.Epinephrine. E.tyrosine E.alanine B.phenylalanine 53.liver cell B.peritubular interstitium of kidney C...End product of purine metabolism is A.Erythropoitin is secreted from A.norepinephrine n dopamine are derivatives of which aminoacid A.acetic acid to acetyl co A D E.uric acid C.macula densa of kidney D..temprature B.globulin is decreased D. 55.urea and water B.albumin is decreased C.glucose to acetyl co A C.water and ammonia E.glucose D.ESR is decreased with A.arginine C.ammonia 54.. 52.decreased blood volume ..B. CVS B.cisplatin B.biconvex B.liver E.infection 60.drugs B.vincristine D.congenital abnormality E.cyclophosphamide C.permaturity D.have nucleus D.Commonest Cause of PDA A.breast D.nonfragile C.have glycolytic enzyme activity 57.toxin C..Tamoxifen has effect on which system A.blephamide E 59.CNS C.kidney 58.largest of all cells E.RBC's are A.E.Which of the following drug is most notorious for causing renal toxicity A.ultrasonography .decreased plasma volume 56.Most common Cause of metaplasia is A. tRNA involve in protein synthesis .is reversible D..involve viscera only 62..are the same as parental cell C.mRNA has anticodon C.What is the most true among following A.rRNA is most abundant n involve as ribosomes on endoplasmic reticulum D...increase in size n no of cell C..fibrous tissue E.infections D.increase in no of cell B.radiations E.epithelia D.. 64.deep tissue 63.chronic irritation C.mRNA has a codon B. E. 61.is premalignant ? E.Dysplasia is seen mainly seen in A.Wat is the differentiating point of hyperplasia fron hypertrophy A.B.tRNA is largest E.bone marrow B..cytplasm n nucleus ratio is derranged D.increase in size of cell B.Wat is the differentiating feature of hyperplasia from benign tumor A.cartilage C. cns abnormalities in alcoholics 66.STD E.kaposi sarcoma B.transferrin C.apoferritin .ferritin B.65.lymphoid tumors C.Neural tube defects occur due to deficiency of A.defects in alcohlics D.iron B.niacin 68.leukemia 67.muscle wasting C.biotin C.AIDS is associated with all of the following except A.thiamine D.VITAMIN involve in collagen synthesis is A.ascorbic acid B.pyridoxine E.folic acid C 69.deficiency of THAIMINE(B1)causes all except A.subacute degenaration of spinal cord E.cardiac anomalies B.primary tumor of brain D.Iron is stored in the form of A. secretory? 74.ovulatory C.CA vulva C.Optic groove appears on left side of forebrain on day A.vulval papules? 72.fibroepithelial lesion B.simple diffusion B.CO2 is tranported in lungs through A.HPV 73.If a pt has presented with the wart on lateral wall of introitus.squamous cell CA of skin C.active transport D.12 B.polyp D.CA cervix B.32 D.carriar proteins E.channels 71. Which phase will she be in? A.Pt presented on 18th day of menstrual cycle her normal cycle is such dat she ovulates on day 14.vaginal wart D.52 .pre mestural D.fasilitated diffusion C.squamous CA of vagina E.42 E.the most probable daignosis will be A.follicular B.proliferative E.22 C.HSV is associated with A.70. wat is the option for treatment of exophthalmos A. 79.wat is the condition called A.biguanides D.A newborn with anterior abd wall swelling and umbilical cord attached to it.partial parathyroidectomy 78.hypophysectomy D.PT has exophthalmos n his T3 n T4 are increased.congenital deafness? C.administration of testosterone E.gastroschiaosis C.Treatment of diabetes in pregnant lady is A.omentocele E. What is the most appropriate answer A.Statistical average applicable to the measurement of a population is A.75..insulin C.umblical hernia D.drugs blocking the action of T4? B.direct suppresion of T4 C.sulphonyl urea B.intestinal malformation 77.cardiac anomalies? D.congenital cataract B.median .limb deformaties E 76.suphonylurea n biguanides E.mean? B.Germen measles causes which abnormality..mode C.omphalocele B. case control B. oxytocin receptors on cervix B. prospective D.dec prostaglandin recp on myometriun 82. C. 84.standard deviation e.DNA D.E coli? B. random 81.cell membrane B.dec prostaglandin recp on cervix D.central 80.oxytocin receptor on myometrium and endometrium C.50S 83.Beta lactum acts through A.cell wall C.d. E.impairment of hearing B.30S E.salmonella .it has effect o n : A. D. cohort C. retrospective E.side effect of streptomycin A.Which of the following is opportunistic organism A.Two groups one with CHD and associated risk factors is compared to another group with CHD but no associated risk factors. What type of study is it? A.Oxytocin is given to a pt in labour. E 87.is caused by fungal B.chalymadia E.mumps C.shigella D.About aspargillosis which is not correct A. 86.CL perfiranges D.Another question associted with aspargilosis 89.wat is the diagnostic finding in this case for T.usually seen in preexisting lung diseases? C.bacteriodes B.klebsella E.botulinum E.rhinovirus 90.Psudomembranous colitis is caused by A.syphlis B.B .gonorrhea 88..tetani C.Most common cause of puerperal sepsis is A.HIV E.cytomegalovirus D.difficile B.A pt with cervical lymphadenopathy.chlostridium 85.Cl.Cl.trichomonas D.Cl..C.E coli C.is caused by long term antibiotic use. he has granuloma on histopathology..Inutero infection is not caused by A. D. presense of delta antigen shows( HDV) A.paramesonephric duct C..AFB B.endometrial cyst C.present breast but absent uterus.A 25 yrs old multiparaous presented with lower abd pain.Chronicity D.gonorhea D.wat is the cause A.well developed secondary sexual characterstics.teratoma 92..recovery C.genital tubercle D 93.chlamydia C..A pt presented with primary amennorhea.ovarian CA D.on histology it showed solid n cystic areas intermixed but on laproscopy it was semi solid with brown centre. 94. Acute phase .A woman presented with vulvar itching n pinkish purulent discharge.Infectivity B.mesonephric duct B.caseous necrosis C.the organ not found is due to abnormal development of A.montoux test D.dermoid cyst B.trichomanas vaginilis B.enodmetrial CA E.A.lactobacillis E.x-ray E 91.wat is the most probable daignosis A. factor 5 .factor 7 B.trichomonas E.E coli B.om laproscopy fallopian tubes were sticked together n ruptured during operation.Deficiency of Vit K will effect all of the following except A.mast cells 98.chlamydia D.acute salpingitis B.aspirin B.lymphocytes D.wat type of cell will present at this time of disease A.indomethacin E.pyosalpinx D.Pt has a history of infection 4 weeks back.What is the drug of choice for DVT in ist trimester of pregnancy A.gomorrhea 96.neutophils B.Most common organism involve in post operative abdominal & gynaecological infection is A.eosinophils C.basophils E.warfarin D.Measure of severity/fatality of the disease 95.now he developed same pain again.chronic salpingitis C.heparin C.E.mefanamic acid 100.wat is the condition involved A.bacteroides C.A pt presented with lower abd pain n purulent white dischrge.hydrosalpinx 97. erythroblastis fetalis all other options were wrong 103.estrogen C.CO2 105.amino acid C.cryptoterone acetate E.activated 12 D.activated 11 C.Type 2 hypersensitivity reaction includes A.prothrombin 101.diacrone .prothrombin E.group B with O E.group A with AB C.Ig M E.Ig G D.C.factor 9 D.activater 10 B.activeted 9 102.testosterone D.group AB with A D.group O with B 104.fibrinogen E.At which step intrinsic n extrinsic pathways converge A.glucose B.group A with O B.Which drug is used in the treatment of hirsutism A.Transfusion reaction will not occur in a pt if we transfuse the A.Which of the following does not cross placenta easily A.antiandrogens B. actinomycosis B. of Na n Ca B.47XY E.If carbohydrate is separated from proteins wat will be the fate A.low Na n Ca high K D.high K low Na C.E coli C.aggregation n precipitation B.bacteroide D.In partial mole.Na D.69XXY 110.high conc.46XX B.Ca-Calmodulin complex is present in smooth muscles 109. hydrolases E.Volume … 108.karyotype is A.glucose B.ECF differ from ICF in A.45XO C.trichomonas 107.gonorrhea E.alkaline phosphate .106.Lysozome contains A.Most common organism involved in infection caused by IUCD A.Ca C. high conc.eliminate fron kidney? C.increase uptake int muscle 111. Of K & Ca with low Na E.diffrence of smooth n skeletal muscle A.47XX D. cysteine B.What is not true about levator anni muscle A.can be palpated at pelvic brim E.Accurate C.cystedine C.supplied by L1 n L2 B. MCQ on Significance A. From a population two groups are selected on age basis and then subgroups are assigned randomly for the study.Which of the following does not have sulpha group A.112.Multistage sampling B.What is not true about ureter A.enter the pelvis infront of common iliac artery B. 116.Sensitive D… 117.simple random sampling 118.Specific B.wht type of sampling is it? A.threonine 113.methionine E. Metabolic Alkalosis is caused by a) Chronic Hypoventilation .homocystine D.not hav sphincteric valve at opening into bladder D.?? 115.Stratified sampling C.puborectalis forms part of it 114.enter the bladder on superio lateral surface of trigone C. If a test is able to both diagnose and eliminate diseased and non-diseased wht is it called? A.Random stratified sampling D. Crossmatch blood to mothers plama C. Following is Autosomal Dominant Disorder a) Phenylketonuria b) Hereditary Spherocytosis c) Hemophilia A d) Cystic Fibrosa 123. For Rh blood transfusion to the anemic baby A.Crossmath blood to mothers blood D.On examination of a female. following is correct a) K is 9 8 % in the cells b) normally completely reabsorbed in Distal tubule c) increase water affects K-balance? d) decrease in D.K.Regulation .A 125.a plaque like lesion seen on Postero-superior aspect of Vagina. About K .it is .Q on Metabolic acidosis 120.give wbc depleted transfusion E.Q on compensated respiratory alkalosis 1 2 1 .b) Pancreatic Fistula c) hyperglycemia d) ATN 119.crossmatch blood to babys plasma B. Source of Estrogen & Progesterone in last trimester is a) Chorionic Villus b) Placenta c) Overy d) Adenohypophysis e) Corpus Leuteum 124.Give red cell concentrates crossmatched wid fetal blood 122. a) Squamous Cell CA b) Adenocarcinoma c) Rhabdomyosarcoma d) wart? GYN OBS Questions JULY 2007 Hi just got to know of dis site through cpsp community otherwise would have mailed u questions way back anyways great effore am mailing fcps part 1 obs gynae mar n jul 2007 question no 1:eversion of endocervical epithelium in pregnancy is due to: a)physiological change b)chronic cervicitis c)carcinoma in situ d)estrogen stimulation e)epidermadization of epithelium 2:primary oocyte surrounded by a non cellular layer: a)theca interna b)theca externa c)corona radiata d)cumulus oophurus e)zona pellucida question no 3: simple columnar epithelium is the lining of: a)epithelium lining the ducts b)cornea c)testis d)uterine tubese) question no 4:48 years lady has carcinoma cervix that has involved the cervix and has involved the vagina but not its lower third and has not . she has now watery vaginal discharge.causative organism: a)bacteroides b)chlamydia c)herpes simplex d)gonococcus e)trichomonas vaginalis 7: a married woman has right sided tubulo ovarian abscess which was removed in a surgery had an iucd placed before.extended onto pelvic walls stage of ca: a)stage 1ab)stage 1bc)stage 11d)stage 111e)stage 4 question no 5:38 years lady has endometrial ca that has involved upto half the tickness of myometrium and ovaries are normal stage: a)stage 1a b)stage 1b c)stage 1c d)stage 11 e)stage 111 question no 6:20 yrs old girl has been raped.causative organism: a)chlamydia b)gardenella c)tuberculous d)bacteroides e)gonococcus 8.has painful red papules n ulcers on vagina and vulva has fever and tender inguinal lymph nodes.the late secretory phase of menstrual cycle is not characterised by: a)the spongy layer becomes thick b)corpus luteum secretes estrogen n progesterone c)the endometrial thickness is doubled d)endometrial cells have subnuclear vacuolations e)endometrial glands become tortuous . LH is increased: a)2 fold b)2 to 4 fold c)2 to 6 fold d)6 to 8 fold e)8 to 10 fold 10.72 hours 12.paracetamol: a)is more potent analgesic than codeine b)is a weak anti inflammatory agent c)overdosage can cause nephrotoxicity question no 15:anticoagulants are contraindicated i n : .the volume of distribution of a drug is not influenced b y : a)age b)sex c)cardiac failure d)renal failure 13.sperms stay active in female genital tract: a)12 hours b)12 .menopause is characterised by: a)atrophy of uterus b)decrease in ovarian size c)derease estrogen increased FSH AND LH d)decreased estrogen increased FSH decreased LH 11.24 hours c)12 -36 hours d)24 .the bioavailability of a drug determines : a)efficacy b)efficency 14.48 hours e)24 .9.just 2 days before ovulation. a)cerebral infarction b)thrombocytopenia 16.examination revealed: a)multinodular goitre b)retrosternal goitre c)colloid goitre d)cold nodule 18:cortisol: a)increases amino acids n glucose in blood b)is a glucocorticoid c)is found in circulation bound to proteins d)metabolised in liver e)formed from cholesterol 19.baby cephalic.which of the following investigation is least likely to benefit: a)clotting time b)FDPs c)prothrombin time d)platelet counte)serial fibrinogen levels 17.and head at zero station.a multigravida admitted with labour pains examination revealed os 6 cm dilated.20 yrs lady has endotoxic shock after septic abortion.30 yrs male has dyspnea on lying down.JVP is differentiated from carotid pulse: a)does not change with abdominal pressure b)does not change with posture c)does not change with respiration d)pulsation varies with respiratione)prominent outward movement 20.the point of reference is: a)ischial tuberosity b)pubic arch c)pubic symphysis d)ischial spines e)sacral promontory . question o 23:beta hCG:a)it is a polypeptideb)it has action similar to FSHc)it prevents involution of corpus luteumd)e) 24:adrenal medulla will selectively secrete norepinephrine in the condition:a)exerciseb)sleepc)cold exposured)hemorrhagee) 25:cardiac output is decreased in:a)pregnancyb)standing or sitting from lying positionc)exercised)e) 26:cardiac output is decreased in all except:a)sleepb)starvationc)sittingd)e) 27:epinephrine will selectively onstrict muscles in:a)arteriolesb)bronchiolesc)ciliary muscled)pupilse)intestinal walls 28:spores should be disinfected by:a)dry heat at 100cb)dry heat at 60cc)dry heat at 160cd)cidex solutione)moist heat 29:following is not a disinfectant: a)derivatives of salicylic acid b)alcohol c)soap d) .at LSCS the artery damaged in superficial fat in suprapubic incision: a)inferior epigaSTRIC artery b)superior epigastric artery c)internal iliac artery 22.following is not a tumor marker: a)acid phosphatase b)beta hCG c)alpha fetoproteins d)CEA e)placental alkaline phosphatase 23.21. e) question no 30: that she had urinary incontinence.muscle damaged is: a)bulbospongiosis n superficial perineal muscles b)superficial n deep perineal muscles c)external anal sphincter n deep perineal muscles d)ischiocavernosus n sphincter urethrae e)sphincter urethrae n deep perineal muscles 32:pudendal nerve: a)formed from ant primary rami of second third n fourth sacral spinal nerves b)crosses the ischial spine medial to internal pudendal artery c)enters the pudendal canal on the lateral side of ischiorectal fossa d)leaves the pelvis by passing through the upper part of greater sciatic foramen e)reenters the pelvis through lesser sciatic foramen 33:following does not form wall of inguinal canal: a)aponeurosis of external oblique b)internal obliquec)conjoint tendon d)lacunar ligament e)fascia transversalis 34:following does not pass through lesser sciatic foramen: a)inferior gluteal vessels b)tendon of obturator internus c)nerve to obturator internus d)pudendal nervee)internal pudendal artery .due to injury and communication i n : a)ureter n bladder b)ureter n vagina c)bladder n cervix d)bladder n vagina e)bladder n uterus question no 31:perineal body is injured. 35:after LSCS patient had peritoneal hemorrhage she was reopened bleeding most likely from: a)branch of ovarian artery b)branch of uterine artery c)internal iliac artery d)obturator artery e)external iliac artery no 36:left ovarian artery is a branch of: a)abdominal aorta b)left renal artery c)uterine artery d)left inferior mesenteric arterye)obturator artery question no 37:patient had laparotomy for ovarian mass mass was burried in ovarian fossa artery likely to be injured is:a)common iliac vesselsb)internal iliac vesselc)external iliac vesseld)ovarian arterye)obturator artery questoion no 38:insulin stimulates transport of glucose into: a)renal tubular cells b)neurons of cerebral cortex c)skeletal muscled)intestinal cells e)liver question no 39:the secretion of following is not increased in pregnancy: a)cortisol b)growth hormone c)thyroid hormone d)prolactine)PTH question no 40:a patient had hypophsectomy for pituitary tumor she has amenorrhea for 8 months ovulation induction can be done in her by: a)clomiphene citrate b)pulsatile hCG c)hMG . d)hMG followed by pulsatile hCG question no 41:following does not occur in pregnancy: a)increase in fasting blood sugar b)increase in uric acid level c)increase in serum urea level d)decrease in serum albumine)decrease in serum proteinse)decrease in nitrogen levels question no 42:respiratory acidosis occurs i n : a)emphysema b)hyperventilation c)ingestion of excess amount of sodium bicarbonate d)starvatione) vomiting question no 43:CO2 is transported from the alveoli of lungs into blood in lungs via: a)diffusion b)active transport c)facilitated diffusion d)secondary active transporte)osmosis question no 44:O2-Hb dissociation curve delivering O2 in normal limits will have: a)PaO2 40mmHg SO2 60% b)PaO2 68mmHg SO2 78% c)PaO2 96mmHg SO2 90% d)PaO2 123mmHg SO2 98% e)PaO2 256mmHg SO2 99% question no 45:b-hCG has alpha n beta units.each unit has aminoacids:a)102 alpha 210 betab)100alpha n 160 betac)92 alpha n 156 betad)e) qyuestion no 46:mothers milk differs from cows milk in that it contains:a)calciumb)lactosec)proteinsd)vitamin D . question no 47:pelvic outlet: a)can b increased in supine position b)begins at the level of ischial spines c)dimensions change by movements of coocyx d)contracture is more common than mid cavity probleme) question no 48:patient has endocrine disorder has metabolic alkalosis and hypokalemia.lipids n proteins in body is:a)growth hormoneb)thyroid hormonec)cortisold)insuline)PTH question no 54:8 yrold girl has vaginal bleeding she is taller than her age and examination revealed an abdominal mas arising from pelvis she is suffering from: a)constitutional. lipids n proteins in cells is also known as hormone of abundance:a)growth hormoneb)cortisolc)thyroid hormoned)estrogene)progesterone question no 50:hyperglycemia is caused by:a)insulinb)glucagonc)estrogend)progesteronee)growth hormone question no 51:ketoacidosis is caused by: a)increased production of glucose b)decreased production of glucpoose c)decreased utilisation of carbohydrate in body d)insulin excess question no 52:insulin secretion is stopped by:a)glucagonb)decreased k levelsc)somatostatind)e) question no 53:hormone that stores carbohydrates. symptoms b)hypothalamic tumors c)hypothalamic mass .he has excess of:a)ACTHb)aldosteronec)cortisold)estrogene)progesterone question no 49:hormone that stores carbohydrates. slight pubic hair.he is suffering from:a)metastatic carcinomab)hogkins lymphomac)non hogkins lymphomad)CMLe) question no 59:patien is taking glucocorticoids for 10 days.most definitive diagnostic test:a)presence of barr bodyb)chromosomal analysis question no 62:14 yrs girl has primary amenorrhea.she wont suffer from:a)moon faceb)hypotensionc)thin skind)e) question no 60:gestational age can best be estimated from:a)abdominal circumference at 36 wksb)biparietal dia at 28 wksc)crown rump length at 8 wksd)femur length at 32 wkse) question no 61:turners syndrome.weight loss positive.usg revealed absent uterus.secondary sexual characteristics normal.d)granulosa cell tumor e)adrenal gland tumor question no 55:least positive titre for widal test isa)1:80b)1:160c)1:260d)1:320e)1:156 question no 56:all of the following can b stored at 4c except:a)shigellab)vibrio cholerac)d)e) question no 57:vaccine is available for all except:a)EBVb)typhusc)pneumococcusd)e) question no 58:patient has presented with fever n generalised lymphadenopathy.short blind ending vagina.normal breast development.underlying cause: a)congenital adrenal hyperplasia b)chromosomal defect c)failure f development of genital tubercle d)defect of paramesonephric duct . bone marrow exam normal. organ most likely to b affected:a)fallopian tubesb)cervixc)ovaryd)endometriume)vagina question no 65:repeated blood transfusions (AB n Rh compatible) in an individual will lead to:a)haemochromatosis of liverb)hypocalcemiac)haemoglobinuriad)transfusion reactione) question no 66: Rh incompatibility occurs in a mother if she has:a)Rh negative fetusb)Rh positive fetusc)d)e) question no 67:the best way to prevent Rh isoimmunisation in a woman who has given birth to Rh positive fetus:a)blood transfusionb)platelet transfusionc)serumd)Rh immunoglobulinse) question no 68:a person brought in er had burns.he is suffering from:a)DICb)hepatorenal failurec)hepatic failure of prothrombin productiond)renal failuree) question no 69:following is not a cause of DIC:a)pre eclamsiab)amniotic fluid embolismc)cervical carcinomad)abruptio placentaee)PPH question no 70:following is not a cause of DIC:a)abruptio placentaeb)burnsc)malignant tumorsd)e) question no 71:multigravida P9+0 delivered a boy in a village 2 hours later she started bleeding profusely had oozing from gums she was brought to hospital her labs showed hb 6 TLC 15000/ul platelets 30000/ul she is suffering from:a)DICb)PPHc)clotting disorderd)bleeding diathesise)ITP .now she has visited a gynaecologist after some years.he was admitted later on he developed fever and bleeding diathesis.question no 63:the lumen of uterine cavity is obliterated by fusion of: a)decidua basalis n spongy layer b)decidua basalis n decidua capsularis c)decidua capsularis n decidua parietalis d)decidua parietalis n decidua basalise) question no 64:a woman has been exposed to radiation in young life. she is suffering from: a)DIC b)PPH c)ITP d)von willebrand disease question no 73:vit k directly influences d reaction:a)conversion of fibrinogen to fibrinb)prothrombin to thrombinc)activation of factor 7d)formation of fibrinogen question no 74:intrinsic n extrinsic pathways converge upon:a)activation of factor 7ab)activation of factor 5ac)factor 12ad)prothrombine)tissue factor question no 75:bacteria that grow at 42c:a)vibrio cholerab)gonococcusc)shigellad)Ecolie)salmonella question no 76:following is not true about endocarditis:a)blood cultures for causative organisms r usually negativeb)is caused by streptococcus group of bacteriac)is more common in elderly groupd)valvular involvemente) question no 77:cause f subacute bacterial endocarditis:a)H.question no 72:27 year old female had profuse vaginal bleeding 2 hrs after delivery her labs showed hb 6 TLC 15000/ul plt 30000/ul peripheral blood film showed burr bodies.influenzaeb)Ecolic)streptococcus viridansd)e) question no 78:interferon:a)binds to DNAbbinds to RNAc)is a protein derivatived)inhibits protein synthesis n their phosphorylation question no 79:highest energy compound:a)ATPb)glucosec)hexose phosphated)starche)glycogen question no 80:the end product of carbohydrate metabolism:a)fructoseb)glucosec)galactosed)starche)pyruvate question no 81:L-dopa taken by a patient will decrease . :a)estrogenb)progesteronec)prolactind)cortisole) question no 82:isoniazid:a)cant b administered with rifampicinb)is contraindicated in nephropathyc)pyridoxine is used prophylactically to prevent its neurotoxic effectsd)e) question no 83:hepatitis b following is not true:a)has an incubation period less than that of hep ab)caused by sexual contactc)caused by DNA virusd)e) question no 85:actinomyces does not cause disease in:a)boneb)lungc)brain question no 86:disseminated coccidiodomyces following is not true:a)infection will not provide life time immunityb)c)d)e) question no 87:middle aged man crossing the road stumbled n hit by a car he fell down had injury to neck of fibula of his left leg was taken to hosp examination revealed that he is unable to dorsiflex and evert his left foot the injury is t o : a)common peronel nerve b)tibialis post c)superficial peroneal nerve d)deep peroneal nervee) question no 88:ECF:a)provides nutrients to cellsb)provides glucose to cellsc)bathes the cellsd)takes away electrolytese)takes away wastes question no 89:spermatogenesis is stimulated by:a)FSHb)testosteronec)LHd)FSH n testosteronee) question no 90:patient brought in shock in er successful fluid replacement will b indicated by:a)decrease in tachcardiab)increase in bpc)increase in urine outputd)e) question no 91:cardoiac cycle:a)60% of ventricular filling occurs in diastoleb)end diastolic left ventricular vol is 100 mlc)ejection fraction is 75%d)the notch of d pulse is due to atrial systolee) . question no 92:a man has a pulse rate of 72/min impulse originates in:a)AV nodeb)SA nodec)left bundle branchd)purkinje fibrese) question no 93:muscles r derived from:a)mesodermb)ectodermc)endoderm question no 94:in our country d cause of cell inujury is:a)alcoholb)hypoxiac)ischemiad)ionising radiatione)malnutrition question no 95:in our country cause of fatty liver is:a)alcoholismb)amoebiasisc)due to hep b n cd)increase intake of fatsdecreased protein utilisation question no 97:in d female d neck of urinary bladder is located :a)in front of rectumb)below d urogenital diaphramc)in d urogenital diaphramd)above d urogenital diaphrame)behind symphsis pubis question no 98:the highest content of triglycerides is in:a)chylomicron remnantsb)HDLc)LDLd) IDLe)VLDL question no 99:following is not a phospholipid:a)cephalinb)lysolecithinc)lecithind)plasminogene)sphingolipid question no 100:a human being can survive without:a)calciumb)proteinsc)carbohydratesd)lipidse) question no 101:patient has pyuria but has negative urine culture for organisms this happens in all conditions except: a)acute post streptococcal glomerulonephritis b)polycystic kidneys c)urine stored at room temp for long d)alkaline urinee)vesical calculus question no 102:broad ligament a double layered fold of peritoneum .ovary is attached to it by: . short stature diagnosed as Turner syndrome. If Carcinoma involves nipple of breast which lymph nodes r likely to involve first? a) Pectoral b) Apical c) Central d) inernal mammary 3. A pt came to you with primary amenorrhoea. came in OPD With enlarged Ant. wht is her karyotype? a) 45XO b) 46XY c) 47XXY . 1.a)mesoovarium b)suspensory ligament of ovary c)round ligament of ovary d)cardinal ligament Gynae Obs Paper July 2007 Part-1 Questions sent by Dr Samra. Eversion of cervix during pregnancy occur in response t o : a) Estrogen b) Physiological change c) Ca Cervix d) Gonadotropins e) Cervicitis 2. reapeated again 5. which region is most likely involved? a) Upper Outer Quadrant b) Lower Inner Quadrant 4.Axillary lymph nodes and diagonosed a case of Ca Breast .webbed neck. A Pt. More or less same question as # 3. Sperms life span in female genital tract is ? a) 24 to 48 hrs b) 24 to 72 hrs 9. c) Biparietal diameter at 18th wk.Tubocurarine d) Bronchodilation --. b) Femur length at 28th wk.Propranolol b) Miosis --.her karyotype is? a) 46XY b) 47XY c) 47XXY 7. d) Abdominal circumference at 8. b)Pudendal Nerve c)Internal Pudendal Vessels d)sup. Almost same as above 1 1 . In drug receptor relations which one is incorrect? a) Bradycardia --.Salbutamol 12. scanty pubic n axillary hairs. A normal looking Girl came to you with primary amenorrhea. gluteal vessels .6. Spermatogenesis is controlled by? a) Testosterone b) FSH+ Testosterone c) LH d) Estrogen 10. & inf.Pilocarpine c) Skeletal Muscle Paralysis --.absent uterus.on examination she was having short blind vagina wid normal vulva.normal breasts. Wht is the best way to detect exact gestational age of fetus? a) CRL at 8 wk. Which one of them do not pass through Lesser Sciatic Foramen? a)Tendon of obturator internus. Ureter is found at : a) the bifurcation of common iliac artery b) pelvic brim 17.Cervix metastasize to Labia majora which one of these is most likely responsible for spread? a) Round lig. If Ca. To reach the uterus. If ovaries are removed frm ovarian fossa.which nerve is likely to b damaged? a)Common Peroneal b)Deep peroneal c)Superficial Peroneal 14. wht is most likely to b damaged? a) Internal iliac b) uterine artery c)Ovarian artery d) External iliac artery 18. In a motorbike accident neck of fibula fractured and lead to a loss of dorsiflexion n eversion of foot.13. uterine artery crosses the: a)Ureter b)Fallopian Tube c)Ligament of ovary 19. Of uterus . If perineal body is cut which muscle likely to b damaged? a) Bulbospongiosus + superficial Transverse Perieal b) Deep + Superficial Transverse Perineal 16. Wht is most likely to b true about pudendal nerve? a) Enter the ischiorectal fossa thru its lateral wall b) Leave the pelvis thru upper part of greater schiatic notch c) Re enter the pelvis thru lesser schiatic notch 15. Hct 20 with same peripheral film wht is the cause? a) b) c) d) Physiological change Iron deficiency Folate Deficiency Vit B12 deficiency 24) Ovaries are attached to broad ligament by? a) Mesoovarium b) Mesosalpinx c) suspensory ligament 25) In females Neck of bladder ends : . c) Ovarian artery d) uterine vein 22) If on lying down pt became dyspnoec wht is the cause? a) Retrosternal goiter b) Multinodular goiter c) 23) If a women at 10th week pregnancy hav Hb 12.b) pubocervical ligament c) pectineal ligament 20)which structure do not participate in forming the walls of inguinal canal? a) Transversalis fascia b) Internal Oblique c) External Oblique d) Conjoint tendon e) Lacunar ligament 21) If after 1 hr of ceaserian section pt start b bleeding internally which one is most likely damaged? a) Uterine artery b) Internal iliac a.9 .Hct 25 Normocytic Normochromic film but at 32 wk pregnancy her Hb is 10. a) above the urogenital diaphragm b) below the urogenital diaphragm c) 26) Most common cause of hosp. acquired infection is? a) Indwelling catheters b) I/V lines c) 27) Which method is used 2 kill spores of surgical instruments? a)dry heat at 160c for 1 hr b)dry heat at 160 c for 4 hr c)moist heat at 160c for1hr d)Gension violet for 4 hrs 28) Which one is not used as insecticidal? a) Derivatives of salicylic acid b) Phenol c) Sodium hypochlorite d) Soap 29) Most common cause of subacute bacterial endocarditis is: a) sterpt viridans b) sterpt epidermidis c) staph aureus 30) Food poisoning cause by staph. Is mainly due t o : a) exotoxin b) endotoxin c) enterotoxin 31) Now a days the best way to diagnose viral infection is: a) ELISA b) by detecting virus from secretion or wound c) immunofloresence . 32) If pt hav persistent pyuria all can be the cause except: a) polycystic kidney disease b) long standing alkaline urine at room temperature c) Post Streptoccocus glomerulonephritis 33) wht is the least positive value for widal to diagnose typhoid? a) 1:80 b) 1:120 c)1:360 34) Two days before ovulation LH surge rises to a) 1 to 5 fold b) 4 to 6 fold c) 6 to 10 fold 35) villous surrounded by cyto n syncitiotrophoblast i s : a) Primary villous b) secondary villous c) tertiary villous 36)Following is not a proven cause of hirsuitism? a) Androgen b) PCOs c) steven leventhal sundrome d) 37) CO2 is transported frm blood to alveoli by : a) Facilitated diffusion b) Active transport c) Diffusion 38) Active transport differs frm Passive due t o : a) depend on osmosis b) require carriers but no energy c) requires energy but no carriers d) require energy by phosphate anhydride bond . 39) Mean Arterial pressure is a) difference b/w systolic and diastolic pressure b) Diastolic + 1/3 pulse pressure c) 40) Columnar epithelium is found i n : a) uterine tubes b) Thymus c) ovaries 41) Obese are at risk of all except: a) Hyepertension b) Diabetes c) Thyrotoxicosis d) IHD 42) Following will lead to Respiratory Acidosis? a) emphysema? b) heavy intake of NaHCO3 c) Vomiting 43) Hormone which causes an increase carbohydrate protein and lipid stores is? a) GH b) insulin c) cortisol d) Aldosterone e) thyroxine 44) same question 45) insulin secretion is inhibited b y : a) glucagon b) secretin c) . 46) Acellular layer surrounding an ovum is : a) Zona pellucida b)Granulosa layer c) Theca interna d) Corona radiate 47) Primordial germ cells arises f r m : a) Yolk sac 48) Highest energy containing compound is: a) Starch b) Glucose c)Hexose phosphate d) ATP 49) Which contain high triglyceride? a) Chylomicron remnants b) LDL c) VLDL d) HDL e) IDL 50) Which is not a phospholipid? a) Lecithin b) ISolecithin c) Cephalin d) Plasmalogen 51)Low PO2 Stimulate respiratory centre thru: a) Directly acting on medulla b) stimulating peripheral chemoreceptor c) acting on central chemoreceptor 52) If Rh -ve mother deliver an O +ve fetus wht is the best way to protect subsequent pregnancies? . wht is most likely? a) Cushing disease b) Cushing Syndrome c) Addisons disease 58)Wht change will not seen in a women having inc.polycythemia.lymphocytopenia and raised ACTH level.a) Immunoglobulin 53) wht is the landmark for biscop scoring: a) Ishial Spine b) Ishial Tuberosity 54) In a women of craniopharyngioma wht is the best opetion to induce ovulation? a) Clomiphen citrate b) Menopausal Gonadotropins 55) If you give L Dopa to the pt of Parkinsonism which hormone will decrease? a) Prolactin b) FSH. cortsiol level? a) Hypotension b) c) d) e) Moon face Buffalo Torso Thin skin Hirsuitism 59) Main support of uterus is: .LH c) Testosterone 56) Which hormone convert to its active form at target tissue with the help of 5 α reductase? a) Testosterone b) Estrogen 57) Pt came to you with eosinopenia. wht is the most likely cause? a) Vesicovaginal Fistula b) Vesicouterine Fistula 64) Which one is the supporting ligament of uterus tht is palpable on Perrectal examination? a) Uterosacral ligament 65) If 35 yr old women developed Generalized Lymhadenopathy wht is the most likely cause: a) Hodgkin Lymphoma b) Nonhodgkin lymphoma c) Malignant disease d) CML e) ALL 66) If women accidently exposed to radiation 3 to 4 yrs back which organ is most likely effected? a) Cervix b) Fallopian tube . is : a) Optimize the platelet function b) platelet number do not dec.a) Cardinal ligament 60) Rational for storage of platelet at room temp. 61) Women brought to you in ER in shock who had a 10 pint transfusion 10 yrs back due to PPH is most likely having: a) Sheehan syndrome 62) Therapeutic index indicate: a) Bioavailability b) Efficacy c) 63) If after 15 days of c-section due to Obstructed Labour pt. developed Urinary incontinence. rash which test will be positive? a) RA b) ANA b) antimitochondrial 70) Wht is the end product of glucose breakdown ? a) Galactose b) Lactose c) Fructose d) Pyruvate 71) In a pre eclamptic pt which drug is used to lower BP before surgery? a) Hydralazine b) Amlodipine 72) Amniotic fluid embolism is related t o : a) DIC 73) Which hormone is necessary for brain development of fetus? .Photosensitivity.c) Ovaries d) Vagina 67) Wht is the best method to check bone density? a) CT scan spine b) MRI spine c) Calcium Level d) D scan of spine 68) Which drug do not cause gynaecomasia? a) Digoxin b) c) d) e) Griseofulvin Cimetidine Androgen Estrogen 69) Pt developed Morning stiffness. T4 raised in pregnancy due to: a) Inc TBG b) Inc TSH c) Inc TRH 76) Which one is not a tumor marker? a) b) c) d) e) HCG CEA α fetoprotein Placental Acid phosphatase(PLAP) Alkaline Phosphatase 77) Hysterosalpingography is more accurate than MRI in: a) Adenomyosis b) blocked tubes 78) TORCH can lead to all EXCEPT: a) Pnuemonia b) Cataract c) Deafness d) Cardiac anomalies e) Blindness 79) DNA is: a) Double stranded in which two strands are antiparallel to each other b) all other wrong .a) GH b) Thyroid c) Cortisol 74) If α feto protein is high wht is most likely? a) Anencephaly b) Spina bifida Occulta c) Down syndrome 75) T3 . 80) Which antithyroid drug is given during Pregnancy? a) Propylthiouracil 81) Tx of raised sugar in pregnancy is : a) Insulin b) Sulfonylurea 82) Menopause is diagnosed by: a)Raised FSH.Raised LH.dec Estrogen b)dec FSH n LH 83) Tuboovarian abcess caused by IUCDs…wht is the most likely causative agent? a) Actinomycosis b) Streptococcus 84) Which component is low in Cow`s milk as compared to mother`s milk? a) Lactose b) Fat c) Calcium d) Protein 85) Excess of Aldosterone will cause: a) Hypokalemia b) Hyperkalemia 86) Which method is most useful in identifying chromosomal abnormalities? a) Karyotyping b) U/S c) Bar bodies 87) DIC is related to all EXCEPT: a) Cancer b) Amniotic fluid embolism c) Sepsis d) RPOCs . she was taller acc.e) answer 88) In C-section insicion is given in subcutaneous fat suprapubically will likely to damage? a) Superior mesenteric artery b) Deep Circumflex iliac artery c) Superior epigastric artery d) inferior epigastric artery e) Perforating arteries 89) Insulin increase absorbtion of glucose in? a) Hepatocytes b) Mucosa of small intestine c) Renal tubular cells 90) Glucose is absorbed in renal tubules through: a) Secondary Active transport b) Facilitated diffusion c) Diffusion 91) A pregnant women is on Phenytoin.wht is the least likely complication in fetus? a) Bone deformity b) Mental retardation c) Cleft palate 92) Anticoagulants are indicated in all EXCEPT: a) b) c) d) Thrombocytopenia Cerebral infarction Prolong bed rest DVT 93) A mother brought her 8 yr old daughter in OPD with complain of vaginal bleeding. To her age with well developed breasts.she was otherwise normal.wht is the cause: . protein in body? a)cortisol b)growth hor c)thyroxine 2)which hormone is responsible 4 storage of fat CHO n protein in body n called hormone of abundance? a)costisol .fats.Rabeeka Khan (Expect repetition of questions) 1)which hormone is resposible 4 storage of carbohydrate.a) Ovarian tumor b) Hyperpituitrism c) Hypothalamic disorder 94) Main function of Extracellular fluid is: a) Transport electrolyte b) Remove waste products c) Provide nutrition 95) Which investigation is least likely done in a suspected case of DIC? a) Clotting time b) c) d) e) FDP D-Dimer Platelet count PT 96) Which drug is not used in Dysmenorrhea? a) Ibuprofen b) Mefenamic acid c) Aspirin d) COX inhibitor Questions sent By Dr. b)throxine c)grwoth hormone 3)which hormone is needed 4 brain development of fetous during intranatal life? a)growth horm b)thyroid etc 4)tubo ovarian abcess in a woman using iucd is caued by? a)actinomycosis b)streptococi etc 5)which component is low in cow milk as compare 2 mothr milk? a)lactose b)protien c)calcium d)fat 6)among following which is not antiseptic? a)soap b)salicylic acid c)sodium hypochlorite d)ethanol 7)4 tx of eclampsia which drug is used? a)hydralazine b)minoxidil etc 8)which method is used 2 kill spore? a)dry heat at 160c for 1 hr b)dry heat at 160 c for 4 hr c)moist heat at 160c for1hr . 9)excess of aldosteron will cause? a)hypokalemia b)hyperkalemia etc 10)structure of 2ndry villus unclude? a)mesoderm cytotrophoblast syncytium b)capillaries cytotrophoblast syncytium etc 11)a 45 yrs old lady is post menopausal from last 9 months.which is most likely 2 show onset of menopause? a)anovulatory cycle b)atrophic uterus etc 12)spermatogenesis is supported by? a)FSH n testosteron b)gonadotrophin etc 13)germinal epithelium of ovary contain? a)cuboidal epithelium c)columnar etc 14)columnar epithelium line which structure? a)thymus b)choroid plexus c)uterine tubes 15)which method is most helpful in identifying chromosomal abnormalities? a)karyotyping b)u\s etc . her wt n ht for age were normal bt absence of uterus.she was othrwise a normal lookinggirl karyotype was xy a)46xy b)46xx etc 17)DIC is related 2 all of the following except? a)amniotic fluid embolism b)cancer c)gram-ve sepsis d)retained dead fetus e)etc(answer) 18)a women brought 2 hospital after having difficult delivery at home wid bleeding diathesis wat is the cause a)dic etc etc 19)which of the following is leat likely 2 b effected by actinomycosis a)bone b)brain c)oral cavity 19)which is not the manifestation of child born wid toxoplasma infection? a)pneumonia b)cataract c)mental retardation etc 20)a woman having morning stiffness of hand n pericardial effusion which lab finding woud help in dx? a)anti nuclear DNA b)strptolysin .16)a normallooking female cam 4 primary amenorrhea on examination she was having short blind vagina wid normal vulva n scanty hair. c)rheumatic factor etc 21)which structure pass thro inguinal canal from uterus 2 labium majus in female? a)round ligament b)broad ligament etc 22)which ligament can b palpated on per rectal examination? a)uterosacral b)broad ligament c)round ligament etc 23)main support of uterus? a)broad ligament b)transverse cervical ligament etc 24)an incision given supra pubically in a female for LSCS would damage which artery? a)superior epigastric b)inferior epigastric etc 25)ovary is attached 2 back of braod ligament by? a)mesovarium etc etc 26)which sturcture does not form walls of inguinal canal? a)lacunar b)external oblique apenerosis c)conjoint etc . which is the least likely complication? a)bone deformity b)mental retardation c)cleft palate 32)anticoagulant are contraindicated in pt wid? a)thrombocytopenia etc etc 33)a pt showing eosiniphilia.27)insulin help in absorption of glucose through? a)musoca of small intestine b)renal tubules c)hepatocytes etc 28)glucose is absorbed in renal tubules thro? a)2ndry active transport b)facilitated diffusion etc 29)after lscs(c\section) a woman is bleeding intaperitonealy which artery can b damaged? a)uterine b)ovarian c)internal iliac etc 30)which organism require selective media 4 its diagnosis? a)vibrio cholera b)e coli c)campylo bacter etc 31)a pregnant woman on phenytoin.lymphoctytopenia polycythemia his blood levels show increase ACTH level which of following is the cause? a)cushing syndrome . b)sudhing disease c)addisons disease etc 34)ca breast will effect which of the breast quadrant mostly? a)upper n outer b)medial c)inner n medial etc 35)a women having ca breast which group pf lypmh nodes is effected? a)anterior axillay etc 36)a mother brought her daughter wid vaginal bleeding she is 8 yrs old taller 4 her age wid well develop breast n odr wise normal which coud b the cause? a)constitutional b)hypothalamic etc 37)a man wid full burns in icu develop bleeding diathesis which coud b the cause? a)dic b)infection etc 38)a women brought 2 ER unconcious wid history of postpartum hemorrhage 10 yrs back which of the following is dx? a)sheehan syndrome b)low bp etc 39) Main functions of extracellular fluid is? a)transport electrolytes b)removal of waste material . etc 40)therapeutic index show a)drug efficacy b)safety . tributery of external carotid vein Anterior jugular vein* Facial vein 5. side effect of thophyllin nausea and vomiting* hypertension bradycardia seizures 6. Correct answers are marked >> * << 1. stress hormone ACTH* Epinephrine Norepinephrine Growth hormone 2. See the june papers below the January papers. wat is not in relation with pitutary gland Facial nerve* Abducent nerve Sphenoid sinus Trigeminal 4. Nor-epinephrine & serotonin degration by 24-Mar-12 4:32:00 PM . wat is not in relation with right kidney decending colon* ascending colon duodenum spleen 3.adiology RADIOLOGY JANUARY 2009. Most imp. By 'Bolanian'. wat cannot pass placenta . wat is not in post.MAO* COMT 7. what is secreted in breast milk Ig G* Ig A Ig M Ig D Ig E 1 1 . chemotaxic factor leukoterine B4* C3b C5 complex histamine 8. wat is in between celeiac trunk and superior messentry artery Pancreas* Pancreas + jujenum Transverse colon + ilium 10. relation with left kidney Deudenum* Ascending colon Descending colon 9. . PICA gives post. loss of memory but resolve …. from which foramen facial nerve enters temporal lobe internal accustic meatus* stylomastoid foramen foramen ovale foramen spinosum 14.... When r the ventricles filled with blood Diastasis* Atrial systole Rapid in flow 15.he is agressive in his mood. A boy had behavior problem. Spinal* Internal carotid Vertebral artery and ant cerebellar artery 13. a new born presented with obstructive jaundice. ant and post spinal arteries arise from vertebral artery gives anterior spinal.. gall bladder looks ok . where is damage Mammillory bodies frontal lobe* temporal lobe parital lobe occipital lobe 16.Ig G Ig A Ig M* Ig D Ig E 12. patient with difficulty in standing from sitting postion. hypotonia of the left limb. cause Atresia of CBD* Atresia of entire extra hepatic apparatus Atresia of hepatic duct Choledochal cyst 17. In resting stage actin is attached to tropomyocin troponin titin myosin 20. adrenalcortical insuficancy causes hypokelimia hyponatremis hypoglycemia* alkalosis 19. where is the lesion Cerebral hemisphere Left cerebellum * superior c puduncle Red neucleus 18. A man with noticed weakness of left side of the body.. if there is damage to sciatic nerve. he swangs to left side. Gluteus Maximum* Gluteus medius Tensor fascia lata .. supply to the dorsum of foot is by sephenous nerve* Sural nerve Common peroneal Deep peroneal 2 1 .in U/s.. Esophagus constriction at Aortic arch* Left ventricle Rt atrium 25. interventricle artery great cardiac vein middle cardiac * small cardiac 26. Basalic vein .Quadricep femoris 22. wat is not in relation with left recurrent nerve SVC* Arch of aorta Lig arteriousum Trachea Esophagus 23. wat vein run with Post. stucture compressing Esophagus Left bronchus (correct) Left Venticle Right Atrium Right ventricle Thymus 24. drain in axillary v e n * runs on radial side starts from palmar surface . utrine cancer spread to labia majore via Round ligament* utrosacral ligament 27. lunate triqutral & pisiform* 3 1 . urinary bladder. nerve damage Axillary nerve* Musculocutaneous Radial Ulnar 30. wat is true supplied by anterior division of internal iliac* separated from uterus by pouch of Douglas in males has seminal vesical above and vas deferens below 33.28. broncopulmonary segment wat is true anatomical. sephenous nerve wat is true drain into femoral 3. true Scaphoid. fuctional n unit of the lung* contains lobar bronchus pulmonary veins run in intersegmental tissue 32. median nerve supply lateral 2 lumbricals* causes wrist drop 29. wat is in proximal layer of carpal bone.5cm inferior n lateral to pubic tubercle* has only 1 tributary connected to short sphenous by perforators 34. down syndrome trisomy 2 1 * 18 15 . injury of neck of the humerus. diagnostic test for strptococuss is ASO titer Blood culture* Urine Dr 3 7 . injury to hippocampus Memory loss* 40. 6 5 % of Na and water reabsorbed f r o m PCT* DCT Loop of Henle Collecting duct 38. .13 35.Erythropoietin is secreted from Yellow marrow Macula densa Preitubualr capillaries* PCT 4 1 .Which of the following tissue is most radiosensitive Skeletal muscle Cartilage* Bone 36. medial part of breast supply which node internal thoracic* lateral grp ant grp medial grp . . wat is not supplied by internal thoracic artery Inferior portion of rectus muscle* .. thoracic duct passes with aorta * 44. wat is the cause of edema Plasma colloid pressure is decreased* Increase hydrostatic pressure Increased capillary permeability 48. parasympathetic supply inc. diapharam contraction cause increase in thoracic pressure increase in thoracic diameter 43. gastric emptying delayed by CCK* Secretin Gastrin Vagus 47.42.5g in urine. sypathetic supply by alpha receptors dilator pupilary musscle of eye* brochoconstriction cardioacceleration 45.. A boy has edema due to protien loss of 3. secretioin of slivary gland* 46. blood supply to Bundle of His. Patient Gravida 4 with anemia and occult blood in stool Iron deficiency anemia * Sideroblastic anemia Thalassemia Megaloblastic anemia 50. wat is true abt CSF secreted at rate of 500 ml/day* choroid villi are seen by naked eye aqueduct transmits CSF from 4th ventricle to spine 54. Bile salt absorb from Ileum* Proximal jejunum Distal jejunum doudenum 52.wat is true C6* C4 C5 cartilage Cricoid 53. iron absorb in ferruos f o r m * oxalate 5 1 .Ant abd wall upto umbilicus Pericardium Thymus 49. Trachea start from …. Of left coronary artery . Circumflex artery a br. nerve supply to latismus dorsi thoracodorsal* .35.Posterior interventricular artery Main Left coronary artery Main right coronary artery 55. Cerebral Anterior and pst.30. wat the reflex gagtrogastric reflex gastro coic reflex (correct) 59. Cerebral 57.20. Median of 20.5* 30 60. premotor area .25.40 25 20 27. A boy defecate just after he takes meal.25.30. which artery supplies Anterior and middle cerebral (correct) Anterior Middle Middle and post. arterial supply to primary visual area PCA* ACA MCA PICA 58. metastatic carcinoma least common site brain lung liver spleen(correct) 56. Benedicts test is done for blood sugar urinary glucose urinary reducing sugars* 66. Doctor patient relations and how good a doctor is known by his degrees his records his pleasant behavior and bed side manners* 64.subscapular long thoracic 6 1 .sup parathyroid gland is related to sup thyroid artey* lies outside the pretracheal fasciae lies anterolatral to thyroid . Right bronchus as compare to left is longer ant to pul artery right bronchus is more vertical than left* 62. if a patient develops a fatal disease it is best to hide it from him tell it to him and his family as soon as possible* 65. A surgeon did a study on patients with heamorroids he included 100 pt in grpA and treated them surgicall whil kept the other 100pt on high fiber diet he interviewd them after 12mons and found that the grp with surgical treatment had a better response…… wat type of study is this? Case control Cohort Prospective Randomized control trial 63. first part of axillary artery ends at border of pectoralis major border of pec minor * border of teres major 69. If RCA is occluded distal to the origin of right marginal artery which part of the heart will be affected most? Rt atrium SAnode AVnode* Apex of the heart 7 1 .67.Esophagus is related to which structure whose dilatation can cause dysphagia l atrium* left vent SVC Rt atrium 70.MHC complex is related to HLA typing* blood grp antigens same in identical twins 68. Myleination of the brain wat is most inappropriate? Starts at birth Occurs hapharzadly Sensory neuros are myelinated first Occurs by oligodendrogliocytes 72. A patient with multiple fractures and BP 90/60 pulse 60/min wat is the most appropriate initial treatment? IV fluids* Reduction os fractures . 73. DIC most inappropriate low FDP* consumptive coagulopathy treated with heparin low platelets 75. Fat embolism most unlikely urinary test is diagnostic* occurs due to trauma to fatty tissues caused bt acute pancreatitis 77. Kleinfelters syndrome chromosomal deficiency autosomal dominant autosomal recessive extra X chromosome* extra Y chromosome 76. Factor 9 def wat is the appropriate treatment? Cryoppt* FFP Whole blood WBC 74. Highest mortality rate Hepatitis A Hepatitis B Hepatitis C . OCP increase the risk of breast cancer endometrial cancer thromboembolism* osteoporosis 83.which organ has least chance of metastasis lung bone liver kidney spleen* 80.influenzae 8 1 . thromboembolism is most common in? leg* liver brain kidney 82. Osteoporosis is less in premenopausal women due to effect of estrogen* . Diphtheria is transmitted by sexual contact respiratory * feco oral 79.Hepatitis D Hepatitis E 78. which organism causes meningitis secondary to pyogenic lung abscess? Strep Staph Pneuococcus h. testosterone progesterone androgens 84. in autosomal recessive disease all siblings are affected both parents are affected only mother is affected all siblings have a chance of one into four of contracting the disease* 86. A trauma patient in shock with stab wound along left 10th rib which organ is damaged? Liver Spleen* Colon Duodenum 85.which of these is not caused by bacteria histoplasmosis* 87. Most likely finding in CSF inc glucose in fungal meningitis dec glucose in viral meningitis lymphocytosis in aseptic meningitis dec glucose in pyogenic meningitis 88. most imp for wound contraction myoblasts fibroblasts myofibroblasts* epithelium collagen 89. smooth muscles are striated . voluntary mostly arranged in circular and longitudinal layers * 90.Bladder S. most probable abt gall bladder. a patient with known HBV infectio comes with HBSAg + HBe ab+ HBV DNA + HBC IGM ab+ Wat is the diagnosis? Acute dis * Chronic carrier Chronic active 95. Which HBpathy presents with crises . About large gut descending colon has no haustra appendix has no tenia* all large gut is supplied by inf mesenteric artery ascending colon has a mesentry 93.vesicles Prostatic urethra Ejaculatory duct* 92. Ductus deferens ends into Prostate u. hyperparathyroidism causes least likely hypertension dystrophic calcification* inc PTH 9 1 . Has thick submucousa Mucousa has extensive folds* 94. sickle cell trait sickle cell disease* thalassemia Hb C 96. 60 yr male with lymphandenopathy and low hb low platelets and increased wbc microscopy show large mature cells wats the diagnosis? CLL* Burkitts Follicular NHL 99. unconjugated bile is carried by which protein? Albumin * Pre albumin A globulin B globulin Fibrinogen 97.After gastrectomy patient will develop iron def anemia megaloblastic anemia due to folic acid def pernicious anemia* sideroblastic anemia 98. what Is true abt Hb consisits of alpha and gamma chains in adults imp buffer of H+ * not assoc with CO2 transport . thyroid is invested in which fascia? Pretracheal* Deep cervical Prevertebral 105. wat is true abt pituitary ant pitutiary derived from neurons no portal vessels btw hypothalamus and post pituitary* .100.PDGF is secreted from which part of platelets alpha granules dense bodies cytoplasm lysosomes 1 0 1 . Epitheloid cells in granulomas are formed by which cells macrophages lymphocytes langerhans cells* 104. Mast cells* Eosinophils IgE 103. Heparin is released from which cells.* Has parthyroids ant to it 106. which cells produce antibodies plasma cells* T cells Bcells Platelets 102. wat is true abt thyroid consists of collecting ducts and acini epi changes shape according to state of activity. ICF & ECF differ in inc K + in ICF 112. Satiety center is located in which part of hypothalamus anterolateral VPL VL Medial* 108. at the end of pregnancy uterus is most sensitive to which hormone? Estrogen Progesterone Prolactin Oxytocin* 110. D & C at 18 days after LMP endometrium will show which stage of uterine cycle? Follicular Ovulatory Secretory* Proliferative 109. Most abundant phagocytic cells in circulation basophils monocytes macrophages neutrophils* 1 1 1 .107. most potent stimulus for release of aldosterone is inc K + * . which of these muscles is a part of the pelvic diaphragm deep transverse perinei puborectalis* bulbospongiosus ischeocavernosus 118. Trauma to the middle meningeal artey causes subdural hemorrhage epidural hemorrhage* SAH 115.inc Na+ inc glucose 113. wat is not true abt adrenal gland fetal gland is 1/3 size of the adult gland it is invested by renal fascia . in females which structure lies btw ureter and peritoneum uterine artery* inf vesicle artery 117. Pallegra is due to def of riboflavin thiamine niacin * vit C 114. patient with loss of fine touch proprioception and vibration injury to which part of spinal cord? Fasiculus gracilis Fasiculus cuneatus Post white column* Lateral white column 116. adrenal cortex is derived from mesoderm 119. which structure is most ant in popliteal fossa popliteal vein politeal artey * tibail nerve bicep femoris gastrocnemeus 120. which of these are the major resistance vessels arteries veins capillaries arterioles* venules 122. a person in a room with optimum enviormental conditions how will the heat loss occur convections sweating breathing radiation and conduction* voiding .tendon of which of these muscles is intracapsular bicep femoris plantaris popliteus* semitendinosus 1 2 1 . patient losses 1 litre of body fliud in 1 hour wat will happen first inc PR inc BP inc veno spasm* 123. a patient with hoarseness and lyrangeal nodule which is most unlikely atrophy * . which of these is not a branch of internal iliac sup rectal* middle rectal inf vesicle sup vesicle 128.124. major part of energy utilized during breathing is to overcome elastic recoil of lungs* resistance of chest wall large airway resistance small airway resistance 125. which vessel will be damaged if the phrenic nerve is cut musculophrenic int thoracic pericadiophrenic* 127. lymphatic of uterus do not drain into int iliac nodes ext iliac nodes sup ing nodes nodes* inf mesenteric 129. a bullet pierces the intercostals space which layer will it come after the intercostals muscles parietal pleura visceral pleura endothoracic fascia* pleural cavity 126. Due to inc ca Dec vit D PTHrP 1 3 1 . metastatic tumors are identified by invasion of other structures* pleomorphism inc NC ratio inc mitosis 133.nodule hypertrophy hyperplasia 130. wat is seen in dysplasia pleomorphism inc NC ratio* inc mitosis 132. pseudo hypoparathyroidism. sarcomas have rapid growth inc vascularity* capsule benign 134. epi of a smoker will show inc ciliated epi mixture of st sq and pseudostratified cells* dec goblet cells . not part of the portal tract portal vein hepativ artery bile duct hepatic vein* 139. which vessel is involved and dilated in portal HTN portal vein* hepatic artey hepativ vien SMV 137.135. not a part of axillary nodes ant grp medial grp apical grp central grp deep cervical* . most unappropraite abt liver has dual blood supply hepatic artey gives only 3 5 % of O2 portal vein has 7 0 % O2 * 138. most unlikely abt glomerulus is has urinary space btw 2 layers is cup shaped is blind ended part of tubule has stratified e p i * has podocytes 136. pain of the ovary is transmitted to the medial side of thigh thru which nerve femoral obturator* ilioinguinal 146. in males uretric stones mostly lodge at which point just below kidney* at crossing of ext iilac vessels at pelvic brim at ischeal spine 142. if there is fracture of the acetabulum post and superiorly which bones r inv ileum and ischium * ileum and pubis isschium and pubis 143. most constricted part of the male urethra? Ext meatus* Infandibukum Navicular fossa 144. at the level of second part of duodenum wat is unlikely aorta at the right of SVC* left renal vein 145.140. Clavipectoral fascia completely covers pectoralis minor* forms suspensory lig of breast forms axillary tail 1 4 1 . left ventricle does not contain supraventricular crest . middle menigeal artery enters thru which foramen? Foramen spinosum* Foramen rotundum Foramen ovale 150.chordae tendinae aortic vestibule papillary muscles 147. Great cerebral vein does not drain deep cerebral vein sup cerebral vein basal vein thalamic vein . Superior petrosal sinus lies in falx cereberi fakx cerebelli tentorium cerebellli* diaphragma sella 1 5 1 . epidural space contains venous plexus* contiues into skull at foramen magnum attached to dorsal coccyx upto S2 148. Otic ganglion lies under foramen ovale* foramen spinosum maxillart nerve stylomastoid foramen 149. 152. chemoreceptors respond to inc PCO2* dec PO2 inc Ph inc temp 155.which of these is not a basic tissue of the body epi nerve muscle blood CT 153. Dorsal rami of nerves supply ext of trunk ext of limbs 157. which vessel lies ant to IVC . Osmotic pressure depends on electrical equivalence conc gradient yenp no of particles mol size 154. Processus vaginalis covers testis only* covers ductus deferens forms ext spermatic fascia arises from parietal peritoneum arises from visceral peritoneum 156. hep B . although I cannot commit to my memory all the mcqs but an effort is made to recall as many possible as it can be. Which hepatitis has the most mortality a.to benefit my colleagues in future. lymphatic drainage of testis is into para aortic nodes sup ing nodes int iliac nodes ext iliac nodes RADIOLOGY JAN 2009 FCPS part-1 january 2009 ( RADIOLOGY) Contributed by Dr. if inguinal canal of a female is opened wat does it contain round ligament and ilioinguinal nerve 159. Gastric emptying inhibited by Ans. hepA b. Looking forward for your prayers.com) Well the paper consisted of paper 1 and paper 2 .i have tried to give the correct answers which I have later confirmed from authenticated sources but the margin of error is still there . 1.left renal vein left renal artery right renal vein right renal artery right testicular vein* 158. I hope it will be benefical. anyone requiring any advice regarding fcps 1 feel free to contact on my e mail. Usman ( doc82me@yahoo. CCK 2. DCT 5. thichness of the membrane (ans ) 8. increase no of particles (ans) c. regarding heamoglobin a doesnot carry CO2 b. hyponatremia b. gastroileal reflex b. contains 2 alpha and 2 gamma chains 7. diffusion is inversely proportional to a. appendix a. Aldosterone secretion increases in response to a. hyperkalemia (ans) 4. hep D ( ans) e. cause of achlasia is a. gastrocolic reflex (ans) 9. hep C d. does not have tenia coli (ans ) . increase molecular size b. loss of myenteric plexus (ans ) 10. PCT (ans ) b.c. infants tend to defecate immediately after they eat because of a. increase ACTH c. Hep E 3. osmotic pressure depends on a. acts as a buffer (answer ) c. 6 5 % Na reabsorption is in a. electric charge 6. hepatic artery b. Benedict test is used for a. connective tissue portal vein hepatic vein (ans) bileduct 13. microcytic anemia c. muscle proteins (answer) 12. increase portal diameter (ans ) 14. urine glucose b. iron def anemia b. growth hormone b. pernicious anemia (ans) 17. e. which is not the part of portal tract a. ferrous form (ans) 16. serum glucose 15.1 1 . urine ketones (ans) c. muscle glycogen b. prolactin c. what results after portal hypertension a. c.which hormone is responsible for fetal brain development a. what is the energy source after 48 hrs of starvation a. d. thyroid hormone (ans) . iron absorption increases in a. liver glycogen c.women with gravida 4 hb 8 occult blood in stool cause a. iron def anemia (ans) 18. total gasrectomy results in a. diagnostic criteria for pre malignant condition a. pleomorphism b. local invasion 24.pseudohyper PTH is seen in a. Lymphocytes normal appearance cause a. hyperglycemia b. vit D def b. tuberculous lymphadenitis b. diagnostic criteria for soft tissue tumours a. dystrophic calcification (ans) 21. CLL 26. patient with decrease hb.19. increase vascularity 25. brown atrophy b. decrease platelets but increase TLC with generalized lymphadenopathy.adrenal cortical def results in a. pleomorphism b. hyponatremia (ans) 20. uric acid (ans) . non Hodgkin lymphoma c. increase nuclear cytoplasmic ratio b. which investigation a. mitotic figure c. hyperPTH will be in all except a. mitotic figures 23. pleomorphism c. hypokalemia c. mitotic figures c.patient with CML develop joint pain. hairy cell d. diagnostic criteria for malignant tumours a. paraneoplastic PTH release from tumours ( I think it is the answer ) 22. leukotriene B4 b. c. histamine 28. neutrophils 29. thromboembolism ( ans) 33. most common side effect of estrogen contraceptive a. RA factor 27.least site of metastasis a. hypertension b.sideeffect of theophylline a. spleen ( ans ) 31. mast cells b. b. most radiosensitive tumour a. C5 complex (ans) c. lymphnode d. brain lung liver lymphnode e.b. apnea 32. lung c. most important chemotactic agent a. platelets d. basophils ( I think so it is the ans ) c. heparin is released by which cells a. nausea vomiting d. brain 30. bone b. which hormone decrease cholesterol . seizures (ans) c. d. severe dehydration will result in a. which is a stress hormone a. most imp ICF electrolyte in excess a. HCO3 39. secretory c.women with lower abdominal pain on 18 day of LMP presented with secondary infertility . Cld. K+(ans) b. jejunum 40. ileum (ans) b. progesterone b. ovulatory b. premenstrual 35. estrogen ( ans) 34. NE 37. Bilesalts absorb from a.factor 1X def what should be given a. insulin def will result in a. loss of ICF only b. cryoprecipitate . glycogen syntheseis c. proliferative d. protein synthesis b. ketogeneis in liver (ans) 36. loss of ECF only c.a. cortisol (ans) b. diagnostic D&C will show endometrium in which sage a. Na c. loss of total body water (ans) 38. mitochondia b. stratified epithelium (ans) 43. PDGF released from which organelles of platelets a. atrophy 42.b. cardiac muscle is a specialized smooth muscle (ans) 48. podocytes b. cell membrane 47.heavy smoker with hoarseness vocal cord will show a. whole blood 41. cytoplasm d. smooth muscle is a. alpha granules (ans) b.what is not present in bowman capsule a. pseudo stratified epi with patches of stratified sq epi 44. dense granules c. nissl substance (ans) 46. produced by chroidplexus which is cauliflower in shape coverd by smooth muscle b. hyperplasia (ans) b. continuous stratified sq epi b. regarding CSF a. daily production is 500 ml per day(ans) 45. tumour c.somites are formed from . striated b. habitual smoker the epithelium will be a. axon doesnot contain a. FFP c. which musle is involved a.gluteusmaximus (ans) c.pt has difficulty rising from sitting position but flexion of leg is normal.which enzyme common to metabolism of both NE and serotonin a. T10(ans) b.narrowest part of urethra a. thin filament is covered by a. flexors of neck c.dorsal rami contribute to a. mesenchyma 49. MAO (ans ) 52. sartorius 50. tropomyosin ( ans) c. navicular fossa 54. 7th costal cartilage c. troponin t 53. external meatus (ans) b. ectoderm b. extensors of trunk(ans) b.pellagra is due to a. extensors of leg 55.ileopsoas b. niacin def (ans) 51. mesoderm (ans) c. membranous urethra c.9th rib . titin b.a.diapgragm doesnot arise from a. COMT b. lattismus dorsi nerve supply a. increase thoracic volume (ans) .artery which is adjacent to the phrenic nerve a. lunate.enlargement of which part of the heart causes esophageal obs a.decrease abdominal pr c. which vein of heart is located in posterior interventricular sulcus a middle cardiac vein(ans) b.triquetral. rt atrium c. pericardiophrenic (ans) 59.bones of carpel a. L1 56.foreign body enters into rt lung because a. proximal row contains scaphoid. left atrium (ans) b. musculophrenic c. great cardiac vein 63.increase thoracic pr b.contraction of diaphragm causes a. pisiform(ans0 57. rt ventricle 62. arch of aorta b. internal thoracic b. anterior cardiac vein c.d. thoracodorsal n (ans) 58. trachea and esophagus d. sup vena cava (ans) 60.ligamentum arterisus c.lt recurrent laryngeal n not associated with a. it is more vertical and wider than left lung(ans) 61. tumour of cervix will spread to labius majora via a. popliteal vein b. tibial n . most anterior structure in popliteal fossa is a. spleen c. blockage of rt marginal artery will result in interruption of blood supply to a.structure precent b/w celiac trunk and sup mesenteric artery a. SA node b. axillary nodes 65. common peroneal n c. tail of pancrease 7 1 .thoracic duct a.64. internal thoracic nodes (ans) b. pancrease (ans) 70. passes thru aortic opening (ans) 69. rt atrium (ans) 68. pancrease and jejunum b. posterior nodes c. interatrium wall c. structure not lying posterior to left kidney is a.medial quadrant of breast lymph drainage a. roung ligament of uterus (ans ) 66clavipectoral fascia a. is pierced by axillary artery c. duodenum (ans) b.is continuous with prevertebral fasia 67. invests pectoral minor muscle (ans) b. gastrohepatic+hepatoesophageal+hepatoduodenal+gastrodenal (ans) 78. diaphragmatic sella b. falx cebri 74.d. rt hepatic artery c. retroduodenal+infraduodenal+supraduodenal+intraduodenal(ans) 77. it is a remanant of embryonic yolk sac (ans) b. regarding mekels diverticulum a. chances of siblings to be affected is one in four 73. straight sinus 75. sella turtica d. pelvic brim (ans) c. it is 60cm distal to ileocecal valve 79. middle cerebral vein drains into a. left hepatic artery 76. cystic artey (ans) b. sup petrosal sinus is related to a. lateral wall of pelvis b. cavernous sinus b. upper part of ureter . what is the commenest site of ureter stone obstruction in a standing male patient a. popliteal artery (ans) 72. lesseromentum is divided inti following portions a. regarding autosomal recessive trait a. tentorium cerebelli c. it is precent in 1 0 % of population c.cbd is divided in to following portions a. proximal portion of CBD is supplied by a. 30 b.80. 28. knowledge 82.25. take place with damage to fat tissue only (ans) b. chronic recurrent pancreatitis . trauma to the breast 86. b. how is the professionalism of a doctor judged a.antibodies are produced by which cells a. acute pancreatitis b. in order to disclose news regarding fatal disease a. neutrophils plasmacells(ans) macrophages lymphocytes 85fat embolism all are true except a.in atherosclerosis following are seen a. reputation c. calculate the median in following data 20.24.35 a. c. regarding behavioural sciences .30. d. 35 8 1 . migratory thrombophlebitis is seen in a. foam cells (ans) 87. patient should know fist b. information should be hidden 83. prospective cohort study(ans) 84. punctuality b. relatives should be told first c.30. two gps of patients given haemmorhoidal treatment and their effect seen with time a.5(ans) c. Infections.Endoderm 4-Memory centre is located i n : Insula.Brachial A.HyperPTH.Axillary.cord.Median.Adrenal cortex 7-Least chances of renal stones is associated with: Hyperlipidemia.Vertebral canal 3-Vertebrae is derived f r o m : Myotome.Medulla.Temporal lobe.Parietal lobe.tubercle 12-Following is not a tumor marker: .Soleus 2-Adult derivative of notochord is: Nucleus pulposis.Ectoderm.granulosa.Submandibular. pancreatic tumour (ans) 88.pituatary gland causes dec.Annulus fibrosis.which structure crosses the aorta a.Dermatome.Frontal lobe 5-The most commom type of salivary gland tumor is: Salivary glands. left ovarian vein RADIOLOGY OCT 2007 BY KASHIF NADEEM NISHTAR HOSPITAL.Sclerotome.c.Lateral and medial cord 9-Nerve injured in Ant.disloacation of Shoulder joint i s : Musculocutaneous. 11-Common site of fracture at rib is: Angle.Ulnar A.Suprascapular 10-Damage to scaphoid bone causes injury t o : Radial A.Plantaris.Costochondral joint.Parotids.Radial.Medial cord.interosseous A.functioning of Glom.Hyper vit.Post cord.D.Rectus femoris.Glom.Ant.left renal vein (ans) b rt renal vein c.Vertebre.Shaft.MULTAN 1-The muscle involved in unlocking of knee joint is: Poplitus.fasiculata.Sublingual 6-Distruction of Ant.Hyperurecimia 8-Nerve supply to extensors of arm is supplied by: Lat. Dec power of muscles below the leson on same side.Ant.Hypertrophied pituatry.Inferior meatus.TLC:3.Sup.most likely have: Dec.Middle meatus.Cerbral A.Hunger pangs 20-Foetal period starts after which week: 11th.21st 21-Patient having dec levels of factorII.meatus.AFP. 15-Soldier comes with heavy bleeding.12th.PLAP.Acid Phospatase 13-Most common histology found in lung tumors is: Squamous.0.having Hb:6. .VII.Colloids.Lyrnx 24-Olfactory area is present i n : Ant.Stress.Aplastic anaemia 23-Nucleus ambigus supplies to: Teeths.Cerbral.Fungal inf..Occipital lobe.Folate def.temporal gyrus.most prabable finding would be: Hypertrophied PTH gland.IX.Extensor plantar on left side..Parietal lobe 25-Patient with injury to left 8th cervical segment of spinal cord will not show following sign: Dec sense of position.deviation of tongue.Parasitic inf.Dec.perforating material.Hypertrophied adrenals.Mixed.Whole blood for 18 days 16-Maxillary sinus opens into: Sup.Tongue.Spinal.Dec Vit.bHCG.. 18-Autospy done on the patient having CRF.CEA.the artery commonly involved in brain is: PICA.Crystallines.Vit B12 def.Post.Viral inf.5/ul.pituatary would cause: Sweeting.Hypertrophied thyroids 19-Excitation of post.Small CA 14-Patients comes with Rt.K 22-Young pt.sense of touch and vibartions.Jaw.AdenoCA.X.AICA.Inf.the ideal fluid replacement would be: Packed RBCs.antithrombin III.8th.Shivering.16th.Nasal septum 17-Patient having increased levels of IgE.Dec sense of pain and temprature below the lesion on same side.Whole blood for 3 days.Plts:1lac is suffering from: Iron def.most probably suffering from: Bacterial inf.vibration below lesion on same side. best marker to monitor his thyroid status is: T3.Dec.Damaging nuclear membrane.at PUJ. 38-Hypothyroid patient in on thyroxine.T4 and TSH levels 39-Patient having hyperplastic bone marrow.Fallopian tubes 35-Least common site for ectopic pregnancy would be at: Ovaries.Mitochondrial shrinkage 32-Ureter isnot constricted a t : Pelvic brim.Vit D.Nephrons.Invloving protien synthesis 29-Most sensitive cells to hypoxia are: RBCs..Only Cystic A.Platelets 30-Myelin in CNS is formed by: Microglia.Schawan cells.Ca.Inc PTH.Neurons.Greater omentum.TSH.T3.anaemia.External Iliac V.Post.Pouch of douglus.Bones 28-Virus acts on cells by: Damaging cell membrane.Karyolysis.bone density 27-Most common site of malignancy in pts suffering from nuclear outbreak Haematopoitic.T3 and T4.26-Feature of Rickets and Osteomalacia is: Inc.Cervix 36-Gall bladder is supplied b y : Cystic A and Left gastrcA.Megaloblastic.Inc Platelets is probably suffering from: Aplastic anaemia.Inc.Cystic A and Left Hepatic.Ovaries.Commom Iliac V.T4.Astrocytes 31-Feature not associated with irreversible cell injury i s : Mylein figures.Lung...Thyroid...Psoas muscle.Openning at bladder 33-Common relation of ureter is: Ant.to gonadal vessels and ant. 37-Venous drainage of urinary bladder is into: Internal Iliac V.to iliac A.Oligodendrocytes.to gonadal vessels and post.to iliac A.structure injured would be: Ureters.Internal and external V.Myelofibrosis.Breast.Leaukemia 40-Stimulus for Eryrthropoiten secreation is: Hypoxia 41-Erytropoiten is always raised i n : .Fallopian tubes. 34-Patient feels pain after she underwent surgery for fibroids.Karyorrehxis.WBCs. Sickle cell anaemia 47-Type of defect in Heredetary spherocytosis is: Enzymatic defect.Gonadal.Structural defect 48-Patient with jaundice..it would be: FAP.Thalasemia.Ileum.Hereditary spherocytosis.Nor adrnaline.Hausstral folds.Tubulovillous.Jejunum.Adrenaline.Cholesterol .Goblet cells.Tubular adenoma.Primary polycythemia.Fatty thrombus.Embolus.Villous adenoma.Polycythemia vera.epigastric.the cells lost in the gastrectomy are: Mucous.Bladder.anaemia. 44-Fat tags attached to the walls of large intestine are known as: Taenia coli.Dopamine 55-Precursor of steriod hormone is: Progesterone.lie posteriorly.parietal.Metaplastic polyp 54-Tyrosine derivateve doesnt include: TSH.Hemolytic anaemia 49-Ejaculatory duct opens into: Ureter.cheif.Pregnenolone.on histologic evaluation pathologist labelled it as benign growth with no chances into malignant transformation.Air embolism.Prostatic urethera 50-Terminal part of CBD in relation to pancrease is: Embeded into pancrease.lie anteriorly 51-Following would ause massive infarction and destruction leading to patient death: Thrombus. 43-Patient after gastrectomy is on VitB12.Appendeces epiplocea 45-Bile salts are reabsorbed from the: Duedenum.polycythemia 42-Following vein would be dilated in portal HTN: Inf.most likely cause of death i s : Amniotic fluid embolism..Ureteric crest.Rectum 46-Following is an autosomal disease is: Cystic fibrosis.Fat embolism 53-Patient with old history of adenoCA of colonis operated for polypectomy.Sec.with 13th week of pregnancy after having crush fractures in RSA dies after 3 days.Iron def.Colon.Thromboembolism 52-22yrs female pt.Left colic V.high reticulocyte count is having: Folate def.Renal vein.Prolactin..VitB12 def. Internaland Innermost layers.XY.transversus.Kidneys.most likely suffering from: Pagets.internal oblique.Internal oblique 63-Patient with bone pains having normal Ca.Superantigen infection 61-Following doesnt form the layer of inguinal canal: External oblique.Ulnar 59-The base of urinary bladder is made by: Post surface.Investing fasica.Transversus muslce 65-In Turner syndrome.Osteomalacia 64-External spermatic fasica is formed by: External oblique.loss of sensation over thumb and index finger.rectus abdominus.nerve involved is: Median.lymphocyctic infiltartion 69-Thyroid gland moves with swallowing because it is enclosed i n : Pretracheal fasica.XO.Hyoid 58-Wasting of thenar eminence.Gram positive inf.Axillary.+ve AFB sputum.inc Alk.HyperPTH.Vertebral fascia. 62-Femoral sheath is formed by: Fascia transversalis..Bone mets.Innermost and enothoracic fasica.Cremastric muscle.Heart.with cervical lymphadenopaty.and Fasica iliaca.Liquefactive.Glissons 57-Most common site of thyroglossal cyst is: Suprahyoid.Liver.phosp.Hyper VitD.Inferolateral.Fasica trans.Radial.Ant surface.56-Capsule of liver in known as: Disse.XYY 66-Organ having least chances of infarction: Lungs.Carotid fascia 70-Neurovascular bundle in chest wall lies between: External and Internal intercostal mucles.Infrahyoid.Gram negative inf.Internal oblique.sign to look for TB on microscopy is: Chronic inflammation.Fat.Traumatic 68-Pt.Beneath Endothoracic fascia .the genotype would be: XX.XXY.caseous necrosis.Spleen 67-Type of necrosis in brain infarction: Couglative.Superior surface 60-Shock without having vasodilation is likely associated with: Burns.External oblique. Dysplasia 81-Main difference between primary and secondary intention wound healing is: Granulation tissue.Iron def.Amenorrhea is due to: Inhibition of LH and FSH thru Prolactin.the lympahtic drainage of testicle is into: Para aortic L.Carbon 78-Child having meckels diverticulum is having bleeding per rectum.Metaplasaia.Vit C.the venous blood returns to the heart by: Muscle pump in calves 74-Circulation in heart is maintained by: Symphthatic sys.ANF 85-Tumor compressing optic chiasma would cause: ..Ectopic gastric tissue 79-During lactation.it is due to: Volvulus.Pre aortic.Dec.Neoplasia. 72-Man is suffering from testicular ca.Foriegn body 77-Mesothelioma is associated with: Vinyl chloride.haemturia.Dec oestrogen 80-Presence of pancratic tissue in gastric mucosa is termed as: Hamartoma.Asbestos.Parasymphthetic sys.cause would be: Minimal change disease.he is suffering from: Folate def.Inflammatory cells 82-Child with sore throat after 3 weeks developed oedema.Vit C def.71-Fibrous pericardium and medicatinal pleura is supplied by: Vagus N.Focal segmental GS. 76-Facor affecting collagen synthesis during healing is: Infections.Local mechanism 75-Fisherman is presented with gingival bleeding and ecchymosis..post streptococcal GN.Angiotension I.Phrenic N..Superficial inguinal.Internal iliac.Autonomic N.Wound contraction.Silica.nodes 73-In exercise.Aldosterone.nodes..Fibrinoid necrosis 84-Angiotension II exerts its effects by activating: Renin.. 83-Microscopic feature of polyarterites nodusa i s : Granuloma.Intercostal N.Choristoma.Copper dust.Interssusption.External iliac L..IgA nephropathy.prolactin.Vit B12 def. Bilateral haemianopia.Anxiety .Excercise.Bitemporal haemianopia.Complete blindness 86-Growth hormone is maximally rasies i n : Sleeping.Binasal haemianopia.Hunger. Regarding Nitroglycerine. Warfarin. blood drug conc c. adrenals 6. Cimetidine 4. a. dec plasma protein binding 5. c. Digoxin toxicity a.26th MARCH' 08 ANESTHESIA 1. effect inc by a. d. e.ANESTHESIA ANESTHESIA MARCH' 08 FCPS I . c. urine drug conc 2. dec metabolism b. If a drug is taken orally. Enzyme induction occurs i n : liver kidney heart d. Inc K* b. How will u assess the drug? a. Why the dose of theophylline in infant is smaller than in adults a. b. t1/2 is 3-5 min 3. plasma drug conc b. d. a. Inc Mg* dec C a * * dec Na* alkalosis 24-Mar-12 4:32:00 PM . c. b. all are true except extensive 1st pass effect venodilator dec venous return dec HR e. phosphodiesterase inhibitors 1 1 . ACh c. all sympathetic are adrenergic c. d. equal no. b. of atoms 9. Secterin . ACh c. 2 liters of 2 different gases of diff mol wt contain a. K* d. Somatostatin 13. all parasymp are cholinergic b. c. Na* c. Capillary permeability is inc by a. corticosteroids c. glucose b. of molecules b. Gastric acid is inc by a. inc gastrin b. Urea 8. equal no. all post-gang parasymp are cholinergic 12. aspirin b. Gastric motility is inc b y : a. PG in inflammation dec by: a. a. Secondary active transport occurs thru a. Which is true. Secterin d. hypertension plasma colloid osmotic pressure intersitial hydrostatic press altered permeability 10. Gastrin b.7. constipation 18. inc sweating c. b. Basal level of bile 15. ACh effect? a. ADH aldosterone Ag I I baroreceptor efferent e. c. dec contractility .d. Thirst is dec by: a. dec HR c. Somatostatin 14. Inc HR and force of contraction 17. pt is hypotonic. e. Inc salivation b. After induction of anesthesia by 10 mg/kg thiopentone. amytriptyline and reserpine c. d. hypersensitivity b. atropine and reserpine b. lipogenesis midriasis piloerector contraction bronchodilator 16. alpha recptors effects ? a. inc Hematocrit 19. atropine and amitryptyline 20. inc HR b. Beta effect ? a. promotes each others actions a. Cause ? a. c. d. In hypertonic obs cardiomyopathy. distal convoluted tubule e. Loop diuretics + thiazides enhance each others effect ( Dec NaCl abs) at: a. amiodarone 24. furosemide c. descending loop of Henle b. which Ca** channel blocker is used: a. AV node c. corticosteroids d. Terminals shd be i n : a. purkinje .21. Treatment started. Monotherapy for mild CCF: a. Drug to be given: a. clindamycin c. Rt atrium d. captopril c. nifedipine d. ACEi d. Nicardipine 22. collecting duct 23. ascending loop of Henle c. propranolol 26. thiazide b. Nifedipine b. streptomycin b. proximal convoluted tubule d. Pt with mild CCF. digoxin b. SA node b. bleomycin 25. 1 week later pt complains of severe pain at Rt big toe. Metastatic pleural effusion. Cause: a.Pacemaker for complete heart block. d.Lungs inactivate Bradykinin b y : a. c. inc QT 33. dec abnormal tissue conduction and no effect on normal tissue b. enz involved in converting kallikrein to plasma protein 29. inc contractility b. Class IA antiarrythmic drugs a. inc QRS f. dec atrial contraction d. Lidocaine b. Drug responsible: a.After loss of 1 litre of blood in 5 m i n : a. dec AV conduction e. b. ACEi b. Large hump on left of cardiac pacing mentions: a. quinidine c. Lidocaine. MOA? a. CO = VR. amiodarone 28. procainide d. Amiodarone > MOA 32. Infant with renal abnormality (renal artery stenosis). Rt ventricle (correct) 27.e. amiodarone . C a * * channel blocker c. diastolic press dec systolic press dec aldosterone inc no change 30. inc end-diastolic vol 3 1 . inc HR c. inc PR interval c. renal failure 40. ARF b. CD 4 1 . a.inc (35). Cause a.Drug that inc extracellular K* (moves K* out of the cell): a.0. diarrhea c. LOH d. blocks Na* channels b. prolong vomiting b. Vit D def . Hb 8. Metabolic alkalosis a. carbonic anhydrase 38. exercise 37. Carbonic anhydrase d. ureters attached to colon 39. pH e. PCT b.34. Consistent finding in hypoKalemia a. Angiotensin b.Verapamil as antiarrythmic ? 36. Sec K* in renal cells a. ARF. DCT c. Beta blocker c. Lidocaine as antiarrrythmic: a. K* channel blocker 35. HCO3. pH Inc. Ca++ dec. H2CO3 c.most effect. metabolic alkalosis b. Anesthetics effect at cerebral level and analgesics at spinal cord level c. mild jaundice. Death d/t embolism after accident a. deferroxamine b. B2 c. B6 d. Ferritin 1000. PTH 42. Reason? a. 10 yrs old girl from Baluchistan c/o fatiguw. c. d. b. blood transfusion c. iron transfusion 46. Capillary abn b. 90 yrs old lady with purplish large patch on Rt hand and arm. Hb 8. enlarged spleen. S. Factor VIII b. Otherwise Normal. Folic acid 44. lethargy. Vit K dependent clotting factors 45. MCV 58. Appropriate treatment? a. No itching or pain. factor I I I .c. effects o n : a. High output cardiac failure a. Fat tumor cells air thromboembolism 47. only alter A delta and C fibers b. No comorbids. discoloration of skin. Prothrombin c. Anesthetics differ from anelgesics i n : a. affects only pain and temp and no other sensory modalities 43. Warfarin. folic acid deficiency d. Thiamine b.0. c. Sec released from thyroid gland entering circulation is: a. Bradycardia b. Inc ICP. b. Na chromoglycate 50. vasodil c. b. HTN. tachycardia 54. inc secretion 49. a. HTN. Cushing's triad: a. d. Post menopausal w o m e n w i t h brease c a . bradycardia inc ICP. Thyroxine Free T3 free T4 TSH e. Ibpratopium bromide given by nebulization: a. d. tachycardia inc ICP. production rate 20 ml/hr .c. dec ACh release in bronchi b. tachycardia dec ICP. Inc ICP. d. Tamoxifen 52. hypoTN. HTN. TRH 53. 20-50 hrs 50-100 hrs 100-200 hrs 36-200 hrs (correct) 5 1 . DOC a. Mast cell stabilizer. CSF a. Prothrombin 48. T1/2 of desmethyl diazepam. total 150 ml b. hypoTN. e. an inactive metabolite of diazepam is: a. c. c. produced by ependymal cells of choroid plexus d. inc Ca resorption from bones c. b. Rifampin 6 1 . Cell wall synthesis inhib a. Penicillin 59. Inc K* than plasma b. inc PO4 abs from renal tubules b. c. CRF d. dec specific gravity 56. 1 week after starting of ATT. PTH a. a. Sulphonamides inhibit a. drained by lymphatics 55. d. H2CO3 Hb plasma pr. tetrahydrofolate in bacteria 60. dec glucose c. Needle inserted at paramedian vertebral canal pierces: . Dark urine after starting ATT. Ethambutol d.c. Anemia 57. PZA 62. Reason a. INH b. CSF: a. PO4 58. Imp buffer in interstitial fluid a. pt c/o pain in big t o e . Rifampin c. req use of normal vaporizer 6 7 . often multi therapy is not curative b. MAC of Sevoflurane with 6 0 % NO a.660.68 67.9 b. autonomic stimulation . MAC 1. ant spinal lig b. dec HR b. dec HR 66. all unconscious pts must be screened by this drug c. ligamentum flavum 63. req daily dosing of multiple drugs for long time 65. req special vaporizer c. 0. Regarding chemotherapy all are true. clindamycin penicillin cephalosporins erythromycin 64. c. supraspinal lig d. onset of action 30-60 min b. Flumazenil a. Ether causes a. interspinal log e. max safe dose 1 mg/day 66. except: a.a. Pseudomembrane colitis linked t o : a. d. paradoxical tachycardia d. Sevoflurane a. b. post spinal lig c. Isoflurane a. N2O shd not be used in elderly with intestinal obs d/t: a. cord > cerebellum > cerebrum c. inc ICP d. Cylinder half filled with liquid N2O. Halothane 7 1 . More pungent. risk of inc distension 74. EEG typical of seizures b.5 3 . Pressure in the cylinder? 75. sp. desfurane < isoflurane < enflurane < sevoflurane < halothane < methoxyflurane 0. Enflurane a. enflurane c. does not undergo metab and is exc unchanged 77. cerebrum > cerebellum > sp. Pressure in the cylinder till the last drop evaporates? 76. nonevident facial spasm 69. Myocardial toxicity to a L/A? . T1/2 N2O ? a. cord > cerebellum 73. cerebrum > sp. Isoflurane b. Blood flow regulation a.68. not concerned to dose / conc adm c. a.02 0. cord b.2 2. Metabolism of halides to release fluroide levels in descending order: a. desflurane at MAC <1 70.5 20 72. Pressre of N2O cylinder is 750 psi. Muscle relaxant that can be given to an asthmatic pt a. Bupivacaine 8 1 . b. Hyperoxia. Muscle relaxant. Prilocaine 80. dec surfactant prod d. corticosteroids c. d. anemia 83. Fracture 5. long duration of action a. All true except 1. tubocurare . retrolental fibroplasia b. oil / gas solubility c. alveolar air conc. cardiovascular collapse a. atracurium tubocurare cisatracurium suxa 85. longest acting L/A to be given a. PG released in inflammation are inhibited by: a. c. CV depression e. bupivacaine 79.a. renally exc. 6th rib. 82. aspirin b. Bupivacaine 78. MAC related t o : a. MetHbemia a. atelectasis c. Bl / gas solubility b. serotonin 84. a. flow of O2 > N2O c. CaO formation c. Laminar flow (MURAD QS) a. 1 0 % gluteraldehyde b . Reason for heat generation when CO2 is passed thru soda lime a. resistance independent . The operating room shd b cleaned with the following drug. 2% gluteraldehyde d. friction b/w soda and CO2 90. pancuroniun d. formation of CaCO3 b. radiation (UV light) d. H2O2 88. flow directly prop pressure b. alcohol with chlorhexidine c. dry heat e. breakage / disruption of control valve. For disinfection hands shd be washed with a. 9 1 . formaldehyde 87. formaldehyd 89. autoclaving is heating objects at 121 degC at 15 psi for 3 min c. repeated heating denatures polyvenyl tubings b. Following is true regarding sterilization: a. 1 % phosphoenol c. phenol b. vancuronium c. following surgery of HIV p t : a. gellamine 86. Flowmeter stops working when a.b. flow of N2O > O2 b. density b. 9/5 * F . Regarding transducer all are true except a. cannot be attached to the ventilator 94. changes signal from 1 form of energy to another b. temp at which liquid converts to vapour / gas without the utilization of external heat b. not affected by humidity d. capacitanc transducer can be used on body 95. density dec c. used towards expiration b. the temp at which gas must be cooled to become liquid 97.32 . kept vertical to keep the dial needle at zero c. diffusion b. doppler effect 96. Relation of vol of a subs to its weight i s : a. conc 99. velocity dec 93. viscosity dec b. To convert Celcius scale to Farenheit scale: a.92. Movement of molecules from high conc they layer/ surface: a. osmosis c. Critical temp is: a. viscosity c. Flow meter a. When Rynolds no. Latent heat of vaporization: a. exceeds 3000 a. becomes zero with gain of energy 98. 5/9 *F x 32 c. Conduction b. gas is heated b. Apart frm measuring end tidal CO2 conc. Pt with ETT tube. Regarding Capnograph a. As the gas flows thru the vaporizer a. Max heat is lost by: a. 5/9 *F + 32 d. convection c. d. hygrometer b. evaporation 102. surface vapour with highest energy is evaporated 103. Pt lying uncovered in operating room. rise of height indicate valve failure .. c. evaporation 1 0 1 . rise of baseline indicate failure of valve b. add 32 and multiply by 9/5 100. convection c. b.b. CO2 saturation PO2 reduced Hb oxyHb 105. Conduction b. Humidity is measured by: a. Heat is lost by: a. Subract 32 and multiply by 5/9 e. radiation d. capnograph can also measure a. outside of the container is cooled c. thermometer 104. radiation d. light emitted by filtered light source 113. no current flows when bridge is balanced 112. pt may get a burn even at ECG electrodes ( Burn can occur at all frequencies ) b. mass spectrometry (MURAD QS) 107. alpha waves with eyes closed a. measures changes of resistsnce b. but independent of freq ) a. inc as the wire is stretched b. Angioplasty 110. known as strain gauge used as pressure transducer c. convert to beta when eyes open b.106. convert to delta in REM c. followed by slow frequency. Wheatstone bridge a. proportional to current flow 1 1 1 . Strong magnetic instruments may not be used in a remote village d/t side effects. MRI c. high altitude wave 108. Instrument that can be used i s : a. depends it is operating at cutting or coagulation mode c. During diathermy when neutral plate is disturbed: ( Burn inc as current density inc. signals occur at 0. Pulse oxymetery a. depends on current frequency 109. O2 is measured by all techniques except a. Resistance in series: a. Variance is related t o : .5 sec b. CT scan b. d. O2 carried most in fetal circ. cohort study c. randomized control trials 117. Treatment of strep pneumonia by sulphonamide and penicilline by random selection is called: a. square root of std deviation c.a. After 1 yr he reasses the pts and finds that surgically treated pts have better outcome. Aorta . range 114. Superior vena cava d. shows relation of 2 paired means 116. Which measure of central tendency is used most frequently in clinical practise a. t-test a. variability b. This is called: a. Surgeon treats 200 pts of heamorrhoids. mode median 50th percentile Wilcoxon rank sum test 115. Umblical vein c. a. case . c. mean b. He randomly divides the pts into surgically treated and pharmacologically treated groups. e. Umblical artery b. tripple blind 118. double blind c. prospective cohort d. single blind b.control study b. in lower 1/3 has 3 layers of muscles esophageal opening acting as anatomical sphincter 120. sq. d. d. c. acts as anatomic sphincter to the stomach 122. has str. Length of larynx. Esophagus histology a. cricoid to T2 cartilage 1 2 1 . e. sq in upper 1/3 has str. accessory nerve b. anterolateral MI Inferior wall MI Lateral wall MI Inferior wall MI 123. c.V4. aorta c. 6th rib 124. e. at level of T12 vert d. 15 cm average b.119. 2nd rib d . clavicle b. anterior wall MI b. Left phrenic nerve also passes thru this opening c. Pt complains of chest pain for more than 30 min . superior vena cava d. Structures entering thorasic inlet a. Groove for subclavian artery at a. I t denotes: a. at left crus of diaphragm b. covered by squamous ep b. ECG shows changes i n V 1 . a. Esophageal opening a. 1st rib c. azygous vein . Left recurrent laryngeal nerve 125. internal intercostal muscle b. delayed closure of pulmonic valve 1 3 1 . esophageal plexus d. 1st branch of ascending aorta is: a. T10 vertebral body b. Diaphragm is directly not attached t o : a.e. S2 splitting occurs d/t a. Needle inserted at 5th intercostal space on the left of the sternum pierces: a. 10th rib 126. LCA from arch of aorta c. xiphisternum d . Spinal cord ends at the level of lower border of: a. right ventricle 128. left ventricle e. closure of aortic and pulmonary valves 130. left subclavian 129. Which is untrue. RCA from anterior aortic sinus b. L2 vertebra . L1 vertebral body c. intercostal nerves 127. left atrium d. Fibrous pericardium and parietal layer of serous pericardium is supplied by ä. internal membrane c. cardiac plexus c. phrenic nerve b. 2nd heart sound is produced d/t : a. L3 vertebra c. isovolumetric contraction 134. Aortic pressure tracing is highest at which phase of cardiac cycle: a. pudendal nerve b. deep inspiration c. S1 vertebra 132. myocin c.b. actin and myocin e. rapid ejection phase c. L5 vertebra e. posterior cutaneous nerve of thigh common peroneal nerve nerve to quadratus femoris perforating cutaneous nerve 133. troponin 137. Example of carrier mediated counter transport: . L4 vertebra d. Which is a branch of posterior division of sacral plexus a. Thich filaments: a. irregular breathing e. actin b. diaphragm 135. slow ejection phase d. c. d. tropomyocin d. apnea 136. Muscle of quiet inspiration a. Damage to pneumotaxic centre leads t o : a. isovolumetric relaxation b. short expiratory phase d. apneusis b. e. Knee joint is supplied by: a. 0. 30 ms c.glucose transport 138. 3 ms d. Chloride shift 146. Vagus forms the efferent pathway of: a. Inhaled foreign body lodges in which part of lung a. Bohr effect b.03 ms 142. Haldane effect c. Blow to the lateral surface of knee joint damages: a. Na influx 139. 143. medial collateral lig and anterior cruciate lig.a. 300 ms b.Breur reflex b. Hering. Rt apical bronchus 140. Leftwatd shift of oxy-Hb curve is called: a.receptors of pulmonary capillaries c. medial meniscus.3 ms e. 0. Ventilation perfusion mismatch in which Ventilation > Perfusion is . Action potential thru large nerve fiber: a. L3-L4 144. Depolarization of cell is maintained b y : a. J. Bain Bridge reflex 145. At the end of expiration: a. intra-alveolar pressure is subatmospheric b. Na. intra-pleural pressure is subatmospheric 1 4 1 . medulla 149. has more HCO3b. Most important buffer is interstitial fluid a. Pacinian corpuscles related to a. pressure d. Proteins 148. rapidly adapting pain e. Uncontrolled DM 152. slowly adapting pain 150. True b. touch b. vibration c. false 1 5 1 . Medullary respiratory group of neurons part of ascending reticular system a.called as: a. Hb b. Dry mouth increases all except a. Bile from liver is different from that from GB i n : a. shunt b. has more mucous and pigments c. Damage to what part of brain will result in central cease of respiration a. H2CO3 c. Anion gap increases i n : a. H2PO4 d. thrist b. dead space 147. ADH . has more anions 153. Mediators of inflammation a. IL1 d. inc hydrostatic pressure of cap c. Middle cerebral artery supplies a. H* sec from nephron is inc by a. artery . leg foot area of motor and sensory cortex 159. sensory neurons blocked 1st d. Increased filtration thru capillary is fascilitated b y : a. C5a c. autonomic neurons blocked 1st 158. acidosis b. General anesthesia influences a. pain neurons blocked 1st c. alkalosis 155. Plsma osmolarity e. motor is affected 1st b. TNF alpha 1 6 1 . Coronary blood flow is maintained by: a. Largest total cross-sectional and surface area is of: a. Stretch reflex afferent 1a fibers true / false true/false 160.c. autoregulation 156. inc oncotic pressure of ISF 157. Plasma vol 154. C3a b. Ag I I d. inc hydrostatic pressure of ISF d. Inc plasma colloid osmotic pressure b. Blood flow regulation is mediated by: a. arterioles b. SSK b. d. veins 162. Lung function that cannot be measured by spirometry: a. Somatostatin dec sec of which hormone a. capillaries d. FRC = ? a. venules e.16 sec 164. VIP e. e. Residual vol 165. Somatostatin c. arterioles c.0. Secretin 167. Hypoglycemia increases a.12 . c. capillaries d. Gastric acid is stimulated b y : a. precepillary sphincter c. b. If heart rate is 80 beats/min a. venules 163.b. ERV + RV 166. duration of PR interval wld be b/w 0. Gastrin CCK Secterin GIP VIP 168. Gastrin d. ADH . c. A postmenopausal women suffered fracture d/t osteoporosis.. Thyroid hormone 169. Oxytocin d. b. PO4 is dec. Calcitonin 170. Reason? a.b. Prolactin e. d. Her Ca** is Inc. Capacitance .. Estrogen Cortison PTH CRF e. Insulin c. Physics. CD count >300 Ans A Vitamin D causes: A.extras from other blogs 24-Mar-12 4:32:00 PM Re: 2011 FCPS part 1 past papers Dby Grecian » Sat May 28. Best method to detect malignancy A.Immunochemical Ans B HIV diagnosis is through A.Clara cells B.western blot B.Elisa C.increased renal Ca re-absorption B.Histopathology C.Cytology B.Macrophages Ans C surfactant contained within the lamellar which is released via exocytosis.Type 2 alveolar cells D.increased serum phosphate levels Ans A Antithrombin .Type 1 alveolar cells C. 2011 7:28 pm Lamellar bodies are found in: A. Cerebral cortex B.A.Has a structure similar to thrombin B.Is transported down attached to neurohypophysin Ans C HIV infection in female will cause A.Spinal cord at T10 level C.leukemia Ans C Following is not a sign of irreversible cell injury: A.Has double size compared to oxytocin B.Pons D.cervical CA D.Karyolysis C.is a valine protease inhibitor C.Mitochondrial shrinkage .endometrial cancer C.When binds to protease action is increased by heparin Ans C it is a serine protease inhibitor.Medulla Ans A Tricky! I guess what they are referring to is the involvement of motor cortex!! ADH A. Head injury causes paraplegia with no sensory loss which are is involved? A. In antithrombin I I I deficiency heparin's anticoagulant effect is decreased.Karryorhexis B.Released on decreased osmolarity C.vulvar cancer B. Which of the following causes viral mediated host DNA transfer: A.type 4 sensitivity B. mediated C.Transduction B.Conjugation Ans A Which of the following most probably has an animal reservoir? A.coagulative C.Cytotoxic rejection D. but if the conditions don't improve irreversible injury will develop Type of necrosis in omentum: Aliquefactive B.Ab.smallpox .GVHD Ans B a question of hyper-acute rejection.D.myelin figures Ans D still reversible at this stage.Transformation C.fat necrosis Ans D A kidney transplant immediately turns blue and is rejected within minutes: A.gangernous D.influenza Bmeasles C. Ans A Vitamin K deficiency following is unusual: A.palatal palsy D.cirrhosis C.coumarin therapy B.G-6-P-D deficiency C.neuromuscular incoordiantion Ans D Heinz bodies are found in A. Necrosis in omentum is A.Bile duct obstruction C.plummer-vinson syndrome C.mycoplasma D.in newborn due to lack of transport through placenta Ans C Liquid dysphagia is due to: A.fat necrosis .hypoglossal nerve palsy B.SLE Ans A All the others.hereditary spherocytosis D.red meat ingestion D.acute myeloid anemia B.post splenectomy Ans B Hemolytic anemia is a feature of A.autoimmune hemolytic anemia B. GFR B.active congestion B.renal creatinine clearance Ans C Earliest side-effects of aspirin over-dose A.it is 17-ketosteroid E.increases hair growth on scalp B. This is due to A.renal plasma flow D.active hyperemia C. vertigo .increase protein synthesis in bones and muscles C. he becomes red faced.caseous necrosis Cgangrenous necrosis Dfibrinoid necrosis Ans A When a question is asked from a student during lecture.passive hyperemia D.renal blood flow C.parasympathetic stimulation Ans B Testosterone is secreted by interstitial cells of leydig it A.tinnitus.B.inhibits secretions of sebaceous glands D.is converted into androsterone in target cells Ans B Para amino hippuric acid is used to determine A. enzyme assay E.histopathological examination C. his elder brother also has a history of repetitive blood transfusions.cytological examination B.tumor marker Ans B .basal chylomicron transport proteins E.apoE B.Sickling test E.immunochemical assay D.Fragile test Ans C Diagnosis of a tumor is made by A.HDL receptors C.Electrophoresis D.B. mother and elder brother are all having this high level of cholesterol in their blood.VLDL receptor Ans C A scenario of child having Hb 6gm/dl.vomiting Cseizures Ans B A scenario of child having serum cholesterol of 600mg/dl.LDL receptors D. his father. ow you come to a diagnosis for this child? A.Immunocytochemistry C. WBC is 5200 and platelets 150000/mm3.What gene mutation is responsible for this condition? A.CBC B. He develops abnormal ECG changes not consistent with ischemia. What is the cause of the ECG changes? A.glucagon has different guanyl cyclase than ACTH C.hypercalcemia C.brain Ans A A scenario of patient having Factor VIII deficiency.metabolic alkalosis B.14 C. The genes responsible for this disease are at chromosome? A.Y Ans D A surgical patient is transfused 3-5 units of blood before and after surgery.11 D.hyperkalemia Ans C Glucagon increases glycogenolysis in liver while ACTH does not due to? A.lungs D.liver has different receptors for ACTH D.heart B. .ACTH is readily degraded by the liver enzymes Ans A ACTH via cortisol causes gluconeogenesis.X E.8 0 % of oxygen is extracted by which organ? A.21 B.skeletal muscle C.ACTH increase plasma glucose B. celiac trunk C.bone D.lung B.inferior vena cava B.pancreas C.spleen D.kidney E.breast .brain D.liver B.aorta D. Most commonly this tumor has metastasized from A.kidney B.bone C.adrenal E.superior mesenteric artery Ans A Carcinoma breast metastasizes to A.liver Ans B Lung contains metastatic tumor.renal arteries E.If celiac trunk is blocked which of the following will not suffer from ischemic injury? A.gall bladder Ans B Head of pancreas is in front of of aorta A. histopathology reveals clear cells.liver C. psuedomonas .staphylococci C.streptococci B.wuchereria bancrofti C.increases during sleep C.caused by salivatory nucleus of medulla/pons B.paragonimus westermani B.schistosoma hematobium Ans C Which pus forming organism in lung also causes meningitis? A.increased blood flow to skeletal muscles C.pneumococci E.increases in response to Beta-adrenergic stimulation Ans D Beta adrenergic stimulation causes A.increase peristalsis Ans B Which organism is found in urine sample A.mydriasis B.bacteriodes D.Ans A Regarding salivation A.inhibited by pilocarpine D. 3 Blood brain barrier absent in _ A. B.Area postema.Anterior pituitary.lichen planus B. C. B.Wall of optic reces.hyperkeratosis Ans A Most lethal premalignant condition is A. ANS: A True B True C False D True E True Q. Constriction of the pupil.4 Parasympathetic stimulation causes_ A. D. C. D.erythroplakia D.solar keratosis Ans C Q.lichen planus B.Poserior pituitary. .Pineal gland. Increased blood flow to the skin and visceras. E. Increase peristalsis of GI tract. Lowing of blood pressure.Ans D Common premalignant condition of mouth in adults A. E. Slowing down of heartbeat.leukoplakia C. ANS_ A False B False C True D True E False Q. B. Hemibalismus D. Decrease muscle tone. E. D. Bradykinesia ANS: A True B True C True D True E True Q.6. Babiniski sign present. Acidosis B. Hypocalcaemia D. Pronounced muscle wastig.8 Factors stimulating synaptic transmition_ A. Flacid paralysis. B.7 Excitatory neurotrasmitters_ A. Tetanus toxin . C. Akinesia E.ANS: A True B True C True D True E True Q. Presance of clonus. Acetylchline D. Chorea. Athetosis C. Seretonin ANS: A False B False C true D True E False Q. Lesions of basal ganglia produces_ A. Hypoxia E. Glutamate E.5 Upper motor neuron lesion produces_ A. Alkalosis C. Gaba B. Glycine C. ANS: A False B True C True D False E False Q. Pendular knee jerk D. Dysarthria ANS: A True B False C True D True E True Q 1. Ataxia E. Akinesia C.A:True B:True C: True D: True E: True Fcps pharmacology past paper ? 2008 Dby drwatson » Fri Aug 13. Dendrites (A) carries sensory information's (B) lack myelin sheath and neurolema (C) cytoplasm doesn't contain nucleus & mitochondria (D) may transmit impulses to adjacent one (E) there is nissl granules in the cytoplasm Answer. 2010 11:59 am . B. Nystagmus.9 Lesions of cerebellum causes_ A. Medulla contains cranial nerve nuclei of (A) Glossopharyngeal (B) Vagus (C) Cranial part of Accessory (D) Hypoglossal Nerve (E) there is nissl granules in the cytoplasm Answer.A:True B:True C: True D: True E: True Q 2. Photosensitivity.rash which test will be positive? a) RA b) ANA b) antimitochondrial 70) Wht is the end product of glucose breakdown ? a) Galactose b) Lactose c) Fructose d) Pyruvate d 71) In a pre eclamptic pt which drug is used to lower BP before surgery? a) Hydralazine a b) Amlodipine .68) Which drug do not cause gynaecomasia? a) Digoxin a b) Griseofulvin c) Cimetidine d) Androgen e) Estrogen 69) Pt developed Morning stiffness. 91) A pregnant women is on Phenytoin.wht is the least likely complication in fetus? a) Bone deformity b) Mental retardation b c) Cleft palate d) Nail Hypoplasia 92) Anticoagulants are indicated in all EXCEPT: a) Thrombocytopenia a b) Cerebral infarction c) Prolong bed rest d) DVT 95) Which investigation is least likely done in a suspected case of DIC? a) Clotting time b) FDP c) D-Dimer d) Platelet count e) PT e . 96) Which drug is not used in Dysmenorrhea? a) Ibuprofen b) Mefenamic acid c) Aspirin d) COX inhibitor 13)germinal epithelium of ovary contain? a)cuboidal epithelium a c)columnar etc 14)columnar epithelium line which structure? a)thymus b)choroid plexus c)uterine tubes c 15)which method is most helpful in identifying chromosomal abnormalities? a)karyotyping a b)u\s etc question no . choice of Rx? . Rx? Methyldopa 11) Side Effects of ACE inhibitors Hyperkalemia a HypoKalemia Hypercalemia Hypocalcemia 24) Which drug has a n ATROPINE LIKE ACTION Physostigmine Scopolamine 25)Old obese Diabetic. newly diagnosed.9) Mild CCF symptoms. Monotherapy to be prescribed ACE-I a Thiazide Frusimide Ca Channel blockers 10)Pregnancy Induced Hypertension. whats the treatment. whats the treatment? Alpha blockers a . BP 60/40. side effects mentioned. around 35 brought to emergency. Clorpromazine SSRI Etc 105) A man. which drug is responsible. salivation. Atropine (correct) (Organo phosphate poisoning.) Lignocaine Digoxin 58) H-K-ATPase pump is blocked by Omeprazole a Cimetidine Bismuth 82) Pheocromocytoma. Heart rate 45. sweating.Biguanides a Biguanides plus sulphonylurea Insulin 104) A patient on a psychiatric drug. Bromocriptine is given to treat it the drug causes A} Inhibition of prolaction from pituitary gland B} Decreased prolaction from hypothalamus C} Inhibition of prolaction from breast D} Stimulation of prolaction of anterior pituitary E} 40] An unconscious patient presented in ED does not respond to Naloxone.Beta blockers Both Alpha+ Beta 85) Effects of Insulin on body fat deposits/metabolism 37) Which drug binds to COX-2 recepters and not to COX-1 Aspirin Indomethacin Piroxicam Meloxicam Ibuprofen 39] A 20 year old girl has developed Galactorrhea due to prolactinoma. Which of the following drug has been taken by this patient? A} Morphine . Enzyme induction occurs i n : a. blood drug conc c. Regarding Nitroglycerine. dec venous return d. kidney . liver a b.B} Heroine C} Pethidine D} Phenobarbital d E} 45] Insulin A} Converts glycogen to glucose B} Stimulates protein synthesis C} Stimulates lipolysis If a drug is taken orally. Cimetidine 5. Warfarin. extensive 1st pass effect b. t1/2 is 3-5 min 3. How will u assess the drug? a. urine drug conc 2. venodilator c. effect inc by a. dec HR e. plasma drug conc b. all are true except a. dec C a * * d. Inc K* a b. dec Na* e. descending loop of Henle b. phosphodiesterase inhibitors 1 1 .c. glucose Na* b K* Urea 9. corticosteroids c. altered permeability 10. all post-gang parasymp are cholinergic 22. Inc Mg* c. PG in inflammation dec by: a. Digoxin toxicity a. plasma colloid osmotic pressure b c. aspirin a b. c. b. alkalosis 7. intersitial hydrostatic press d. Secondary active transport occurs thru a. hypertension b. Loop diuretics + thiazides enhance each others effect ( Dec NaCl abs) at: a. all parasymp are cholinergic b. Capillary permeability is inc by a. adrenals 6. a. heart d. ascending loop of Henle . all sympathetic are adrenergic c. d. Which is true. c. proximal convoluted tubule d. Terminals shd be i n : a. Metastatic pleural effusion. nifedipine d. 1 week later pt complains of severe pain at Rt big toe. bleomycin 25. streptomycin b. SA node AV node Rt atrium purkinje e. ACEi c d. Rt ventricle (correct) e 27. captopril b c. Pt with mild CCF. ACEi b. d. b. Infant with renal abnormality (renal artery stenosis).Pacemaker for complete heart block. Treatment started. furosemide c. C a * * channel blocker c. clindamycin c.c. propranolol 26. Drug responsible: a. Monotherapy for mild CCF: a. Cause: a. corticosteroids c d. digoxin b. amiodarone . collecting duct 23. distal convoluted tubule e. Drug to be given: a. thiazide b. amiodarone 24. quinidine c. inc QRS f.32. dec abnormal tissue conduction and no effect on normal tissue b. pH e.Drug that inc extracellular K* (moves K* out of the cell): a. Lidocaine as antiarrrythmic: a. Sec K* in renal cells a. Lidocaine. amiodarone 34. Lidocaine b. Beta blocker c. H2CO3 c. blocks Na* channels b. Class IA antiarrythmic drugs a. inc QT 33. Post menopausal women with brease ca. metabolic alkalosis b. Carbonic anhydrase d. Tamoxifen . carbonic anhydrase 5 1 . dec AV conduction e. Angiotensin b. K* channel blocker 35. dec atrial contraction d. exercise 37.Verapamil as antiarrythmic ? 36. procainide d. inc PR interval c. MOA? a. DOC a. ant spinal lig b.58. clindamycin b. cord . Blood flow regulation a. supraspinal lig d. Reason a. Dark urine after starting ATT. Rifampin 6 1 . Penicillin 59. cerebrum > cerebellum > sp. pt c/o pain in big t o e . Needle inserted at paramedian vertebral canal pierces: a. except: a. post spinal lig c. often multi therapy is not curative b. erythromycin 64. tetrahydrofolate in bacteria 60. Ethambutol d. INH b. cephalosporins d. Sulphonamides inhibit a. interspinal log e. Cell wall synthesis inhib a. Rifampin c. PZA 62. req daily dosing of multiple drugs for long time 72. a. Regarding chemotherapy all are true. Pseudomembrane colitis linked t o : a. ligamentum flavum 63. 1 week after starting of ATT. penicillin c. Which drug the pt. aspirin a b. cord > cerebellum 77. NaHCO3 reverses the action of the drug.Plantaris.Soleus . He has history of taking some unknown drug. cerebrum > sp. serotonin 54] A semiconscious patient is brought to the emergency room. cord > cerebellum > cerebrum c. PG released in inflammation are inhibited by: a.Rectus femoris. Myocardial toxicity to a L/A? a. sp.b. has likely taken: A} Phenobarbital a B} Phenothiazine C} Morphine D} Diazepam E} Alcohol FCPS past paper ? July 2008 Dby drwatson » Fri Aug 13. 83. corticosteroids c.. Bupivacaine . 2010 11:58 am 1-The muscle involved in unlocking of knee joint is: Poplitus(correct). pituatary gland causes dec.Sublingual 6-Distruction of Ant.Glom.Ulnar A.granulosa.Infections.Lateral and medial cord 9-Nerve injured in Ant.Endoderm 4-Memory centre is located i n : Insula.Hyper vit.Dermatome.Parotids(correct)..Annulus fibrosis..Frontal lobe 5-The most commom type of salivary gland tumor is: Salivary glands.Hyperurecimia 8-Nerve supply to extensors of arm is supplied by: Lat.Brachial A. .Medial cord(correct).Axillary(correct).Sclerotome.Medulla.Vertebre.cord.disloacation of Shoulder joint i s : Musculocutaneous.Ectoderm..Radial.interosseous A.2-Adult derivative of notochord is: Nucleus pulposis(correct).Post cord.Vertebral canal 3-Vertebrae is derived from: Myotome.D.Median.functioning of Glom.fasiculata.Submandibular. (correct).HyperPTH.Temporal lobe.Adrenal cortex(correct) 7-Least chances of renal stones is associated with: Hyperlipidemia(correct).Parietal lobe.Ant.Suprascapular 10-Damage to scaphoid bone causes injury t o : Radial A. Crystallines.Whole blood for 18 days 16-Maxillary sinus opens into: Sup.AICA.Hypertrophied thyroids .Mixed.AFP..Colloids.Hypertrophied pituatry.Viral inf.Hypertrophied adrenals.Post.Inferior meatus (correct).Cerbral A..Ant.Dec.CEA.the artery commonly involved in brain is: PICA.Whole blood for 3 days(correct).bHCG.the ideal fluid replacement would be: Packed RBCs..meatus.most probably suffering from: Bacterial inf. 13-Most common histology found in lung tumors is: Squamous(correct).Costochondral joint.Sup.tubercle 12-Following is not a tumor marker: PLAP.Acid Phospatase(correct).11-Common site of fracture at rib is: Angle(correct)...AdenoCA.Shaft.Parasitic inf(correct).Middle meatus. 15-Soldier comes with heavy bleeding.most prabable finding would be: Hypertrophied PTH gland(correct).sense of touch and vibartions.Nasal septum 17-Patient having increased levels of IgE.deviation of tongue..Small CA 14-Patients comes with Rt.Spinal(correct). 18-Autospy done on the patient having CRF.Cerbral.Fungal inf. Jaw.Occipital lobe.8th(correct).IX.Inf.TLC:3.vibration below lesion on same side.Stress.temporal gyrus.Dec Vit.Tongue.Aplastic anaemia(correct) 23-Nucleus ambigus supplies to: Teeths.0..having Hb:6.Vit B12 def.antithrombin III.VII.Hunger pangs 20-Foetal period starts after which week: 11th..Extensor plantar on left side.12th.pituatary would cause: Sweeting.perforating material.Dec power of muscles below the leson on same side.Dec sense of pain and temprature below the lesion on same side.Shivering.Plts:1lac is suffering from: Iron def.most likely have: Dec.. (correct) .Parietal lobe 25-Patient with injury to left 8th cervical segment of spinal cord will not show following sign: Dec sense of position.X.16th.K(correct) 22-Young pt.Lyrnx(correct) 24-Olfactory area is present i n : Ant.Folate def.19-Excitation of post.5/ul.21st 21-Patient having dec levels of factorII. Oligodendrocytes(correct).structure injured would be: .Thyroid.at PUJ.Astrocytes 31-Feature not associated with irreversible cell injury is: Mylein figures(correct).Invloving protien synthesis(correct).Neurons(correct).Karyorrehxis.Dec.Ca. 34-Patient feels pain after she underwent surgery for fibroids.Psoas muscle(correct).to iliac A(correct).Bones 28-Virus acts on cells by: Damaging cell membrane..Karyolysis..to gonadal vessels and ant.Schawan cells.Openning at bladder 33-Common relation of ureter is: Ant.Nephrons.Damaging nuclear membrane.Platelets 30-Myelin in CNS is formed by: Microglia.WBCs.Breast..Inc.to gonadal vessels and post.bone density(correct) 27-Most common site of malignancy in pts suffering from nuclear outbreak Haematopoitic.Post. 29-Most sensitive cells to hypoxia are: RBCs.to iliac A.Vit D..Inc PTH.26-Feature of Rickets and Osteomalacia is: Inc.Lung(correct).Mitochondrial shrinkage 32-Ureter isnot constricted a t : Pelvic brim. TSH.T4.Fallopian tubes(correct) 35-Least common site for ectopic pregnancy would be at: Ovaries(correct).Megaloblastic.Only Cystic A. (correct).Myelofibrosis.Commom Iliac V.Fallopian tubes.Primary polycythemia(correct).Cervix 36-Gall bladder is supplied b y : Cystic A and Left gastrcA.External Iliac V.anaemia...Cystic A and Left Hepatic.. EPO.polycythemia (Polycythemia vera is only type of polycythemia in which there is Dec.Leaukemia(correct) 40-Stimulus for Eryrthropoiten secreation i s : Hypoxia 41-Erytropoiten is always raised i n : Polycythemia vera...T3.Sec.Internal and external V.Ovaries.T3 and T4.Pouch of douglus.T4 and TSH levels(correct) 39-Patient having hyperplastic bone marrow.Greater omentum.Inc Platelets is probably suffering from: Aplastic anaemia. . 38-Hypothyroid patient in on thyroxine.Ureters. 37-Venous drainage of urinary bladder is into: Internal Iliac V(correct).best marker to monitor his thyroid status is: T3. 43-Patient after gastrectomy is on VitB12.Ileum(correct).Rectum 46-Following is an autosomal dominant disease is: Cystic fibrosis.Renal vein.parietal(correct)..Prostatic urethera(correct) 50-Terminal part of CBD in relation to pancrease is: .anaemia.Gonadal. 44-Fat tags attached to the walls of large intestine are known as: Taenia coli(correct).Ureteric crest.Hausstral folds.high reticulocyte count is having: Folate def.Sickle cell anaemia 47-Type of defect in Heredetary spherocytosis is: Enzymatic defect.Appendeces epiplocea 45-Bile salts are reabsorbed from the: Duedenum.Iron def.Left colic V(correct).Structural defect(correct).Hereditary spherocytosis(correct).Colon.Goblet cells.cheif.epigastric.Hemolytic anaemia(correct) 49-Ejaculatory duct opens into: Ureter.. 48-Patient with jaundice.VitB12 def..Thalasemia.Bladder.42-Following vein would be dilated in portal HTN: Inf.Jejunum.the cells lost in the gastrectomy are: Mucous... .it would be: FAP.Glissons(correct).Dopamine 55-Precursor of steriod hormone is: Progesteron.22yrs female pt.Fatty thrombus.Prolactin.lie posteriorly.on histologic evaluation pathologist labelled it as benign growth with no chances into malignant transformation.Embolus.most likely cause of death i s : Amniotic fluid embolism.with 13th week of pregnancy after having crush fractures in RSA dies after 3 days.Air embolism.Cholesterol(correct) 56-Capsule of liver in known as: Disse.Tubular adenoma.Tubulovillous(correct).lie anteriorly 51-Following would cause massive infarction and destruction leading to patient death: Thrombus.Metaplastic polyp 54-Tyrosine derivateve doesnt include: TSH.Thromboembolism(correct) 52.Nor adrnaline.Villous adenoma.Adrenaline.Pregnenolone.Embeded into pancrease.Fat embolism(correct) 53-Patient with old history of adenoCA of colon is operated for polypectomy. Superior surface 60-Shock without having vasodilation is likely associated with: Burns..External oblique.Transversus muslce .rectus abdominus..Gram negative inf.Bone mets(correct).internal oblique.Ulnar 59-The base of urinary bladder is made by: Post surface(correct).and Fasica iliaca(correct).57-Most common site of thyroglossal cyst is: Suprahyoid.Cremastric muscle.phosp.Internal oblique.Superantigen infection 61-Following doesnt form the layer of inguinal canal: External oblique.Radial.Hyoid(correct) 58-Wasting of thenar eminence.Ant surface.inc Alk.Inferolateral. (correct) 62-Femoral sheath is formed by: Fascia transversalis.Axillary.Fasica trans.loss of sensation over thumb and index finger.Osteomalacia 64-External spermatic fasica is formed by: External oblique(correct).HyperPTH.nerve involved is: Median(correct).Hyper VitD.Internal oblique 63-Patient with bone pains having normal Ca.Infrahyoid.Gram positive inf.most likely suffering from: Pagets.transversus.. Intercostal N.lymphocyctic infiltartion 69-Thyroid gland moves with swallowing because it is enclosed i n : Pretracheal fasica(correct).XXY.Fat.XY.Spleen 67-Type of necrosis in brain infarction: Couglative.Traumatic 68-Pt.Internal .nodes(correct)...65-In Turner syndrome.Superficial inguinal.the genotype would be: XX.Pre aortic..Beneath Endothoracic fascia 71-Fibrous pericardium and medicatinal pleura is supplied by: Vagus N.Phrenic N(correct).+ve AFB sputum. Innermost and enothoracic fasica.with cervical lymphadenopaty. 72-Man is suffering from testicular ca.Liver(correct).XYY 66-Organ having least chances of infarction: Lungs...Kidneys.sign to look for TB on microscopy is: Chronic inflammation..Heart.Liquefactive(correct).Carotid fascia 70-Neurovascular bundle in chest wall lies between: External and Internal intercostal mucles.the lympahtic drainage of testicle is into: Para aortic L.caseous necrosis(correct).Autonomic N.Internal and Innermost layers(correct).Investing fasica.Vertebral fascia.XO(correct). ..Choristoma.it is due to: Volvulus.he is suffering from: Folate def.Interssusption.nodes 73-In exercise.Vit C.Vit B12 def.the venous blood returns to the heart by: Muscle pump in calves 74-Circulation in heart is maintained b y : Symphthatic sys.Dec oestrogen 80-Presence of pancratic tissue in gastric mucosa is termed as: Hamartoma.Dysplasia .Parasymphthetic sys.Vit C def(correct) 76-Facor affecting collagen synthesis during healing is: Infections(correct).Metaplasaia(correct).External iliac L.Foriegn body 77-Mesothelioma is associated with: Vinyl chloride.Ectopic gastric tissue(correct) 79-During lactation..Silica.Asbestos(correct).Copper dust.Amenorrhea is due to: Inhibition of LH and FSH thru Prolactin(correct).Carbon 78-Child having meckels diverticulum is having bleeding per rectum...Neoplasia.Dec.prolactin.iliac.Iron def.Local mechanism(correct) 75-Fisherman is presented with gingival bleeding and ecchymosis. Hunger.post streptococcal GN(correct).81-Main difference between primary and secondary intention wound healing is: Granulation tissue.Fibrinoid necrosis(correct) 84-Angiotension II exerts its effects by activating: Renin...cause would be: Minimal change disease.Complete blindness 86-Growth hormone is maximally rasies i n : Sleeping(correct).Wound contraction(correct).Angiotension I.external iliac nodes .Bilateral haemianopia.Excercise.haemturia.internal iliac nodes B.ANF 85-Tumor compressing optic chiasma would cause: Binasal haemianopia.IgA nephropathy.Aldosterone(correct).Inflammatory cells 82-Child with sore throat after 3 weeks developed oedema. 83-Microscopic feature of polyarterites nodusa i s : Granuloma.Focal segmental GS.Bitemporal haemianopia(correct).Anxiety 3.Lymph drainage of perianal skin is from one of the following A. connects with deudenum B.ammonia .related to left adrenal gland D.restlessness 39.increased urine output(correct) D.C.medial group of horizontal superficail nodes(correct) E.related to left kidney E.related to rt kidney 9.urea a B.vertical group of nodes 4.Which of the following is not a feature of shock A.lateral group of horizontal superficail nodes D.forms falciform ligament b C.decrease cardiac output B.Lesser omentum A.tachycardia E.Which is most imp indicator of muscle protein loss A.muslce weakness C. 46XY (correct) B.45XO (correct) .her karyotype is? A.A normal looking Girl came to you with primary amenorrhea. 41. webbed neck. scanty pubic n axillary hairs. E.Can exist as helix in single stranded form as well as in Double stranded form D.45XO 45. short stature diagnosed as Turner syndrome.on examination she was having short blind vagina wid normal vulva. a single turn of DNA is 2nm C.wat is true about DNA A.47XY C.uric acid E.47XXY D. wht is her karyotype? A.C.none of above D.A pt came to you with primary amenorrhoea.normal breasts. ? 40.Double stranded in which two strands are antiparallel to each other B.absent uterus. 46XX E 10.endoderm of 3rd brachial pouch (correct) E.this problem occured due to abnormality in one of the following .Pudendal nerves (correct) 15.autonomic nerves E.47XXY D.pelvis splanchnic nerves C.endoderm of 2nd brachial pouch C.46XY C.B.sacral splanchnic nerves D.Thymus develops from A.supply of sphincter urethra comes from A.ilioinguinal nerver B.ectoderm of 3rd brachial pouch D.A pt presented with an opening in the ant wall of sternocledomastoid muscle with pussy discharge coming out of it.ectoderm of 2nd brachial pouch B.mesoderm of 3rd brachial pouch 16. absence of pleuro-pericardial membrane B. absence of septum transversum C.liver B.2nd pharyngeal pouch (correct) E.1st pharyngeal cleft B.3rd pharyngeal cleft 17.spleen(correct) D.Erythropoises occurs in middle trimester mainly from A.bone marrow E. absence of pleuro-peritoneal membrane(correct) D.Superior parathyroid gland develops from .1st pharyngeal pouch D.Diaphragmatic hernia occurs due to A.both liver n kidney 19. absence of central tendon E.2nd pharyngeal cleft C.A. absence of arch? 18.kidney C. 2nd brachial arch C.enters the rt heart border after passing through sulcus on posterir surface C. supplies left atrium 25.4th brachial pouch (correct). B.3rd brachial cleft 20.deep inguinal 21.arises from anterior aortic sinus (correct). supplies both ventricles D.wat is the lymph drainage A. E.external iliac C.superficial inguinal E.RT coronary artery A.both internal n external iliac (correct) D.3rd brachial pouch D.A node E. supplies S.Most imp hormone involve in gluconeogenesis is .A.internal iliac B.1st brachial arch B.In cervical carcinoma. subcecal 1.growth hormone B. B.A pt presented with rt iliac fossa pain.retrocolic C.cortisol (correct) E.capsases activation (correct). apoptosis related protein ?? E… . he is diagnosed to have acute appendicitis on flexing his rt thigh inward n medially he cries with pain.pelvic (correct) D.insulin C.Apoptosis is inhibited by A. Glycogen 26.A. Bcl-2 inhibition C.paracolic B.thyroxine D.paracecal E. Activation of P-53 D.wat is the type of appendicitis he has A. SLE B.arachdonic acid metabolites 33.viruses E.C3b n C5a are anaphylactant E 34.multiple sclerosis .32.C5a is a potent chemotactic agent (correct) D.what is the most appropiate answer A.C3a B.C5a is potent anaphylactic C.Low serum complement seen in which condition A.C3a is opsonin B.IL-1 D.Most potent chemotactic factor is A.C5a (correct) C.ankylosing spondiolitis C.rheumatid arthritis D. Androgen .RA D.Cimetidine D.anti DS antibodies (correct) C. VLDL D. Chylomicron remnants B. HDL E.anticentromere antibodies E.E.Which of the following has highest cholesterol content A. LDL(correct).Pt presented with photosensitivity.antimitochondrial antibodies 47.anti SS antibodies B. C.Griseofulvin C.Which drug do not cause gynaecomasia? A.which is the daignostic in this case A.Digoxin (correct) B.dermatomiositis 35.rash n joint pain. IDL 49. peritubular interstitium of kidney(correct).liver cell B.ultrasound B.Which is the best option in identification of Turner's syndrome A.glucose D. C.water and ammonia E.Ct scan E. C. 53..spironolactone 50..macula densa of kidney D. .Erythropoitin is secreted from A.urea and water B.karyotyping (correct) D.uric acid(correct).ammonia 54.E.End product of purine metabolism is A.barr bodies C. E.decreased plasma volume 56.Commonest Cause of PDA A.globulin is decreased D.have nucleus D.ESR is decreased with A.decreased blood volume E.drugs B.toxin C..temprature B.largest of all cells E.have glycolytic enzyme activity (correct).nonfragile C.. 55.RBC's are A.congenital abnormality .albumin is decreased (correct) C.biconvex B.permaturity (correct) D. 59. epithelia D.Wat is the differentiating point of hyperplasia fron hypertrophy A.ultrasonography B.E.increase in size n no of cell C.fibrous tissue .is premalignant ??????? E.Dysplasia is seen mainly seen in A.chronic irritation (correct)..radiations E.infection 60.increase in size of cell B.bone marrow (correct) B.infections D.is reversible D.cartilage C. 61. C.Most common Cause of metaplasia is A.involve viscera only 62. deep tissue 63.mRNA has anticodon C. 64.are the same as parental cell C.tRNA is largest E.cytplasm n nucleus ratio is derranged D.deficiency of THAIMINE(B1)causes all except A..tRNA involve in protein synthesis 65... B.defects in alcohlics .What is the most true among following A. E.mRNA has a codon B..Wat is the differentiating feature of hyperplasia from benign tumor A.cardiac anomalies B..increase in no of cell (correct).E.muscle wasting (correct) C.rRNA is most abundant n involve as ribosomes on endoplasmic reticulum(correct) D. pyridoxine E.leukemia 67.D.VITAMIN involve in collagen synthesis is A.ascorbic acid(correct).niacin 68.lymphoid tumors C. C .subacute degenaration of spinal cord E. D.kaposi sarcoma B.primary tumor of brain (correct).iron B.STD E. B.folic acid (correct).AIDS is associated with all of the following except A.biotin C.Neural tube defects occur due to deficiency of A.cns abnormalities in alcoholics 66.thiamine D. ferritin B.active transport D.apoferritin 70. B.vulval papules? 74.22 (correct) .CA vulva C.simple diffusion (correct).channels 71.transferrin(correct).fasilitated diffusion C. C.CO2 is tranported in lungs through A.Iron is stored in the form of A.CA cervix B.Optic groove appears on left side of forebrain on day A.69.12 B.carriar proteins E.vaginal wart D.HSV is associated with A. 42 E.limb deformaties E 76.52 75.gastroschiaosis C.omentocele E.congenital cataract (correct).Germen measles causes which abnormality.umblical hernia D.wat is the condition called A.PT has exophthalmos n his T3 n T4 are increased.32 D.congenital deafness? C. B.wat is the option for treatment of exophthalmos A.C.intestinal malformation 77. What is the most appropriate answer A.cardiac anomalies? D.omphalocele (correct) B.drugs blocking the action of T4? .A newborn with anterior abd wall swelling and umbilical cord attached to it. insulin (correct). C.administration of testosterone E.. 83.side effect of streptomycin A..Which of the following is opportunistic organism A.hypophysectomy D.E coli? . 84.suphonylurea n biguanides E.Treatment of diabetes in pregnant lady is A.sulphonyl urea B. C.B.partial parathyroidectomy 78.biguanides D.direct suppresion of T4 (correct) C.impairment of hearing B. E. D. Cl.botulinum E. D.shigella D.klebsella E.now he developed same pain again..Pt has a history of infection 4 weeks back. E 97.About aspargillosis which is not correct A..B. 86.is caused by long term antibiotic use.CL perfiranges D.wat type of cell will present at this time of disease .Psudomembranous colitis is caused by A.Cl.chlostridium (correct) 85..difficile (correct) B.usually seen in preexisting lung diseases? C.Cl.salmonella C.is caused by fungal B.tetani C. group A with AB (correct) C.warfarin D.group AB with A .mast cells 98.aspirin B.mefanamic acid 102.lymphocytes D.eosinophils C.Type 2 hypersensitivity reaction includes A.neutophils (correct).group A with O B. B.erythroblastis fetalis all other options were wrong 103.What is the drug of choice for DVT in ist trimester of pregnancy A.heparin (correct) C.indomethacin E.A.Transfusion reaction will not occur in a pt if we transfuse the A.basophils E. testosterone D.group O with B 104.bacteroide .amino acid C.glucose B.cryptoterone acetate E.Which drug is used in the treatment of hirsutism A.Which of the following does not cross placenta easily A.Ig M (correct).group B with O E.CO2 105.Most common organism involved in infection caused by IUCD A.D.E coli C.actinomycosis B.diacrone 106.estrogen C. E.Ig G D.antiandrogens (correct) B. Volume … 108.In partial mole.D.high conc. Of K & Ca with low Na E.gonorrhea E.45XO C.aggregation n precipitation (correct) .If carbohydrate is separated from proteins wat will be the fate A.trichomonas 107.low Na n Ca high K D.karyotype is A.high K low Na C. high conc.ECF differ from ICF in A.69XXY (correct) 110.Ca-Calmodulin complex is present in smooth muscles 109.diffrence of smooth n skeletal muscle A.47XX D.46XX B.47XY E. of Na n Ca (correct) B. Na D.B.increase uptake int muscle 111.Lysozome contains A.1 Paracetamol a) increases PT b) its overdose causes Nephrotoxicity c) is a poor anti-inflamatory (correct) d) is more stronger than codeine . hydrolases (correct) E. Following is Autosomal Dominant Disorder a) Phenylketonuria b) Hereditary Spherocytosis (correct) c) Hemophilia A d) Cystic Fibrosa Q.Ca C.alkaline phosphate 122.glucose B.eliminate fron kidney? C. 4 MS is characterized by.3 Congenital cataract is associated with a) chiken pox b) chromosomal abnormality c) small pox d) Rubella (correct) Q.creatinnine d) Q.. urine in initial stages (correct) c) increase in s...urea but normal s.e) causes met-Hb.emia more frequently than Phanacetin Q.. a) Lewy bodies b) Patchy demylination & white fiber gliosis (correct) c) axonal neuropathy d) Q. a) Hyperkalemia b) conc.2 ATN is associated with.5 All of the following have ability of phagocytosis EXCEPT a) Microgila . a) Essential HPTN b) Sympthatic stimulation c) Nitroglycerine d) Excercise e) Hypoxia (correct) Q.6 Pulmonary Arterial pressure increases with.as a result along with dryness of nasal mucosa also occurs a) decreased secretions of Parotid b) decreased secretion of Submandibular & sublingual glands (correct) ..8 During giving local anaestheia.7 in an Asthamtic female.immeditae treatment of HPTN during intra-operative HPTN.causing anaesthesia of an autonomic ganglion. the long needle goes far into greater palatine canal.venous nitroglycerine (correct) c) propranalol d) Q.b) Lymphocytes c) Kuffer cells (correct) d) Neutrophils Q. is a) sublingual glycerine trinitate b) intra. 12 Glucronide conjugation of drugs .10 Non-essential amino acid includes a) Leucine b) Iso-leucine c) Methionine d) Tyrosine (correct) Q.c) Q.9 Tissue which has in-ability to replace dead cells a) Heart (correct) b) Kidney c) Liver d) Bone e) Bowl Mucosa Q.11 Metabolic Alkalosis is caued by a) Chronic Hypoventilation b) Pancreatic Fistula (correct) c) hyperglycemia d) ATN Q. .14 Tissue with Dual autonomic supply but not reciprocal suuply is a) Skin b) Blood vessels (correct) c) Pupil d) Salivary glands e) Ciliary muscle . c) Vit B 12 def.> 120 fl..a) causes inactivation of drug b) is Type 1 reaction c) makes drug insoluble in water (correct) d) increases its effects Q..4 md/dl MCV. a) Hemolysis b) Vit B 9 def. d) Hypersplenisim (correct) Q. Hb= 7.13 A patient having generalized body weakness & numbness. the anemia is due to . on investigation he is having Un-conjugated Bilirubinemia.. in a fasting man the cause of concenterated urine is a) Skin (correct) b) Blood vessels c) Pupil d) Salivary glands e) Ciliary muscle Q.it inhibits .Q.15 Cause of joint pain in Deep sea divers a) arterial embolism b) venous embolism c) DIC d) thromboembolism Q.17 A 35 yr old man is having bilateral ptosis of eye during evening since last few weeks.18 if a High Dextrose water is infusion given to a person.16 In summer.the confirmatory test of the diagnosis will be a) Tensilon Test b) Ach Receptor Antibody Test c) blood culture d) CT scan Brain Q. S4 b) Pelvic splanchnic Nerves (correct) c) vagus nerve d) Q.20 which does not take part in inflamatory reactions a) Adrenaline b) Histamine c) 5-hydroxy tryptamine d) Dopamine (correct) Q.S3.a) Cotisol b) ADH (correct) c) Aldosterone d) Insulin Q. 21 Parietal & Visceral Pericardium is supplied by a) Phrenic Nerve (correct) b) Sympthatic Nerves c) Vagus neve .19 Parasympthatic supply of Sigmoid Colon is a) Sacral segments S2. the Haemorrhage will be a) Epidural (correct) b) Subdural c) Intracranial d) Q.d) Cardiac Plexus Q. this swelling most probably due to. a) Injury to deep fascia of neck (correct) b) demage to supra-pleural membrane c) Un-united first Rib Fracture Q.22 A young man had sinjury to the neck with sum sharp object.24 Dorsal Rami of the cervical nerves innervates a) Abductor Pollices b) Extensors of Limbs (correct) c) Extensors of Trunk d) Stabilizers of Shoulder .23 Middle meningeal artery if ruptured while meinges are intact. Now he notices a bulging swelling above the clavicle on sneezing & coughing.3 yrs ago... 26 A person has sharp knife injury to upper medial aspect of arm. the nerve injured is a) Median Nerve b) Axillary nerve c) Radial Nerve d) Musculocutaneous Nerve (correct) Q.25 Muscles of Hand supplied by a) ventral rami of C-8 (correct) b) Ventral branches of T-1 c) Median Nerve d) Ulnar Nerve e) Radial Nerve Q. 27 A Transplant from identical twin is a) Allograft b) Heterograft c) Homograft d) Isograft (correct) .Q. after which He is unable to Flex his elbow & having loss of cutaneous sensations on lateral aspect of forearm. 30 A 30 years old 75 kg Diabetic man with recent diagnosis of Hypertension.7.140 mmol s.Na -.Uric Acid-.Creatinine -.29 Pharmakokinetic interaction among drugs is examplified by : a) Shortening of action of Procaine by Adrenaline b) increase peripheral toxicity of Levodopa with Carbidopa c) increase toxicity of Methotrexate by Aspirin d) prevention of Nitroglycerine induced tachycardia by Propranolol e) blokade of acetylcholine receptors by atropine Q.7 mg/dl s. On ECG : Left Ventricular Hypertrophy s.20 mg/dl s.5 mmol s. K-.Cl --.2.e) Xenograft Q.5 mg/dl s.28 Therapeutic Index of a drug indicates its a) Effectivity b) Efficacy (correct) c) Potency d) Toxicity Q.Urea -.103 mmol .0. 23 mg/dl 24 Hr Urinary Proteins-.32 GFR is measured by clearance of a) Inulin (correct) b) Creatinine c) Urea d) Glucose .creatinine is normal Q.30 & Fasting BSL--.31 Acute Tubular Necrosis is characterized by a) Hyper-kalemia b) Initially concenterated urine (correct) c) Metabolic Alkalosis d) s.HCO3 -.s.Urea is increased & s.80 mg/dl The cause is a) Essential HPTN (correct) b) Diabetic Nephropathy c) Pheochromocytoma d) Addison Disease e) Primary HyperAldosteronism Q. .36 Thymus is a) Rich in Lymphocytes (correct) . a) Lewy Bodies in Neucleus b) Ptachy demylination with white fiber gliosis (correct) c) Axonal degeneration d) Q.35 What is INAPPROPRIATE about Vagus Nerve.33 Sarcoidosis is diagnosed microscopically by a) Granuloma with Asteroid b) Non-caseating Granuloma c) Caseating Granuloma d) Macrophages & Giant cells (correct) Q.Q.34 Multipule sclerosis is characterized by. a) its longest Cranial Nerve b) passes in between IJV & ICA c) passes posterior to Carotid Sheath (correct) d) Gives Recurrent Laryngeal Nerve e) gives Superior Laryngeal Nerve Q... b) Present at birth but shorly regree after birth c) derived from 4th Pahryngeal Pouch d) is a part of Thyroid e) Lies posterior to Trachea Q.in sample taken from amniocentesis.40 At 35 weeks of Gestation. .37 True statement about Down Syndrom is a) Trisomy 23 b) contains single X chromosome c) more chances with increasing maternal age (correct) Q.39 Following is Autosomal Dominant Disorder a) Phenylketonuria b) Hereditary Spherocytosis (correct) c) Hemophilia A d) Cystic Fibrosa Q.38 Kleinifilter syndrome a) contains single X chromosome b) less chances of Mental retardation (correct) c) phenotypically female Q. its most probable cause is.43 @ Leprotomy Retroperitoneal Abscess was found.there is increased Alpha-Fetoprotein. a) recieves Lymph from both right & left sides b) lies in posterior mediastinum (correct) c) passes just right to the thoracic aorta d) extends from upper abdoment to neck e) has a dilatation called Cisterna Q.the site woud be ...42 Rate of passage of food in esophagus depends upon a) Gravitational Force b) Neuromuscular excitation different @ upper & lower oesophagus c) progressive peristalsis in esophagus (correct) d) Acidity Q. a) Down syndrome b) Turner syndrome c) Spina Cystica (correct) d) Hydrocephalous e) Q..41 Thoracic Duct.. a) Stomach b) Sigmoid Colon c) Transverse Colon d) Ascending Colon (correct) e) Small intestine Q.the leision lies in a) Facial Canal (correct) b) Internal auditary meatus c) Facial Nerve Nucleus d) Stylomastoid Foramen Q.44 Source of Estrogen & Progesterone in last trimester is a) Chorionic Villus b) Placenta (correct) c) Overy d) Adenohypophysis e) Corpus Leuteum Q.46 All are the branches of Vagus nerve EXCEPT a) Auricular .45 A patient having Facial deviation along with loss of sensations on anterior 2/3 rd of tongue. 47 Following nerve lies in the Cavernous Sinus a) Opthalmic b) Occulomotor c) Abducent (correct) d) Trigeminal Q.48 In a Pituitary Adenoma.the Bitemporal Hemianopia occurs due to leison of a) Central part of Optic chiasma (correct) b) Optic Nerve c) Optic Tract d) peripheral Part of Optic chiasma e) Lateral Geniculate body Q.49 Muscle of Forced Expiration is a) Internal Intercostal b) External Intercostal .b) Lacrimal (correct) c)Pharyngeal d) Recurrent Laryngeal Q. 51 A male wd A+ (Ao.Dd ) blood group.DD) Blood group.children can have all blood gp EXCEPT a) AB +ve b) AB -ve (correct) .50 Na Retension is associated with a) Dehydration b) increase ADH (correct) c) Decreased Aldosterone d) Heart Failure e) Hyperthyroidism Q.c) Sternoceidomastoid (none of them) d) Scalenus Anterior e) Scalenus Medius Q.51 Von-Willebrand Disease is a) Autosomal Dominant (correct) b) Autosomal Recessive c) in some family members asoociated with Factor IX deficiency d) in some family members associated with normal Factor IX Q. married to a female having B+ (Bo. c) A + ve d) B +ve e) O +ve Q.54 Difference between Adenoma & Hyperplasia is a) increase in size of cell b) increase in No.53 When a branch of Pulmonary Artery is blocked by embolus. of cells c) Capsule (correct) d) .52 Vital Capacity a) measured directly by Spirometry b) is decreased in some Lung diseases c) is the sum of Tidal Volume+ERV+IRV (correct) Q.following Lung function increases a) Alveolar Co2 b) Alveolar O2 c) Pulmonary artery O2 d) Pulmonry artery Co2 (correct) e) Ventilation Perfusion Q. Q.the preferred immediate treatment is a) Octeriotide b) Vit K c) formed Platelet Conc. d) PPI e)whole blood (correct) Q. a person has inability to stand from sitting position. the nerve demaged is a) Infragluteal (correct) b) Supragluteal c) Schiatic d) Femoral .55 In a CLD patient presenting with Haematemesis.56 Among most important initial steps in apoptosis a) Activation of Bcl-2 Receptors b) Activation of Caspases (correct) c) Water influx into the cell d) Q.57 After a trauma. 60 An unconsious man with pin-point pupils not responding to Naloxone.Regulation . following is correct a) K is 9 8 % in the cells b) normally completely reabsorbed in Distal tubule c) increase water affects K-balance (correct) d) decrease in D.58 About K .e) Obturator Q. the immediate treatment is with a) Carbamazepine (correct) b) Valporate c) Lorazepan d) Q.K Q.59 A man with persistant fits without regaining consciousness for 30 minutes.the probable cause is a) Heroin (correct) b) Hasheesh c) Morphine d) Phenobarbitone . 62 The Hormone which stimulates release of HCO3 from Pancreas without stimulating pancreatic enzyme secretions.Q.61 Stomach gets its arterial supply from all EXCEPT a) Ceiliac artery b) Splenic Artery c) Superior Mesenteric Artery (correct) d) Hepatic Artery Q.64 Stenson's duct a) arises from posterior surface of Parotid .63 Supra-Renal Gland arterial supply a) recieves all braches directly from Aorta (correct) b) recieves blood from 3 sources c) recieves branches from both renal arteries d) recieves blood from inferior phrenic artery Q.is a) CCK b) Secretin (correct) c) Gastrin d) Pepsin e) Mucin Q. b) pierces the Buccinator muscle c) opens in the oral cavity against ....66 On examination of a female.67 About CSF most appropriate statement is a) it is ultrafilterate of Plasma b) has more glucose than blood c) has cushioing effect (correct) .a plaque like leision seen on Postero-superior aspect of Vagina. Q.it is a) Squamous Cell CA b) Adenocarcinoma c) Rhabdomyosarcoma d) wart (correct) Q.65 Which infection is not caused by DNA virus a) Chicken Pox b) Herpes Simplex c) Herpes Zoster d) Mumps (correct) e) Small Pox Q. 69 Metastatic Calcification occurs mostly in a) Kidney (correct) b) Bone c) Lung d) Intestine e) Pancreas Q.68 On standing from a sitting position.70 Basal Cell Carcinoma involves a) Buccal Mucosa b) Hard Palate c) Soft Palate d) Lower Lip e) Oral Cavity (correct) .@ apex of lung there is increase in a) Blood Flow b) Ventilation c) pO2 d) pCO2 Q.d) absorbed in Choroid Plexus Q. lipids n proteins in cells is also known as hormone of abundance: .71 A female underwent Gastrectomy for Zollinger-Ellison Syndrome.. the inability to absorb B-12 is due to loss of. a) Parietal Cells (correct) b) Gastric Cells c) Cardiac Ceell question no 46:mothers milk differs from cows milk in that it contains: a)calcium b)lactose (correct) c)proteins d)vitamin D question no 48:patient has endocrine disorder has metabolic alkalosis and hypokalemia.Q.given Parenteral Vit B12.he has excess of: a)ACTH b)aldosterone (correct) c)cortisol d)estrogen e)progesterone question no 49:hormone that stores carbohydrates.. a)growth hormone (correct) b)cortisol c)thyroid hormone d)estrogen e)progesterone question no 50:hyperglycemia is caused by: a)insulin b)glucagon (correct) c)estrogen d)progesterone e)growth hormone question no 51:ketoacidosis is caused by: a)increased production of glucose b)decreased production of glucose (correct) c)decreased utilisation of carbohydrate in body d)insulin excess question no 52:insulin secretion is stopped by :a)glucagon . underlying cause: a)congenital adrenal hyperplasia (correct) b)chromosomal defect c)failure f development of genital tubercle d)defect of paramesonephric duct .b)decreased k levels c)somatostatin (correct) question no 53:hormone that stores carbohydrates.usg revealed absent uterus.short blind ending vagina.most definitive diagnostic test: a)presence of barr body b)chromosomal analysis (correct) question no 62:14 yrs girl has primary amenorrhea.slight pubic hair.normal breast development.secondary sexual characteristics normal.lipids n proteins in body is: a)growth hormone (correct) b)thyroid hormone c)cortisol d)insulin e)PTH question no 61:turners syndrome. he is suffering from: a)DIC (correct) b)hepatorenal failure .he was admitted later on he developed fever and bleeding diathesis.question no 65:repeated blood transfusions (AB n Rh compatible) in an individual will lead to: a)haemochromatosis of liver b)hypocalcemia (correct) c)haemoglobinuria d)transfusion reaction question no 66: Rh incompatibility occurs in a mother if she has: a)Rh negative fetus (correct) b)Rh positive fetus question no 67:the best way to prevent Rh isoimmunisation in a woman who has given birth to Rh positive fetus: a)blood transfusion b)platelet transfusion c)serum d)Rh immunoglobulins (correct) e) question no 68:a person brought in er had burns. Which of the following muscle is not supplied by median nerve: Adductor polices. A boy is brought to a hospital with injury at elbow joint lost extension of medial 4 fingers at MTP joint. abduction of thumb with intact sensation due to: Injury to Radial nerve at elbow 03. A 10 year old boy w i t h shoulder i n j u r y lost abduction of his shoulder up to 30 degree which muscle is lost: Supraspinatus Deltoid (correct) 02. 04. Regarding palm: .c)hepatic failure of prothrombin production d)renal failure question no 69:following is not a cause of DIC: a)pre eclamsia b)amniotic fluid embolism c)cervical carcinoma d)abruptio placentae (correct) e)PPH 0 1 . Peroneal artery. 09. Which of the following muscles is flexor at hip and extensor at knee: Sartorius. Epitrochlear lymph nodes. Regarding post. 05. Border of thyroid cartilage. Border of sternocladomastoid at the sup. 6. 8. Infection of the first pulp space lymph nodes will first drained. In the injury to neck of fibula the artery damaged: Ant.Superficial palmer arch is below palmer apeneurosis. Bifurcation of common carotid artery: Beneath the ant. 12. Which of the following muscle have dual nerve supply from median and ulner nerve: Flexor digitorum profundus. Great saphenous vein has how many valves: . Tibial artery. In an injury to knee joint a man is unable to extend his knee the root value affected: L3-L4. Triangle: Base is form by middle 3rd of clavicle. (correct) 10. 7. 1 1 . Weakest point of rib is: A. Subclavian artery grooves over: A. Angle of the rib. 18. A pathologist wants to pass dye to the liver he will reach t h r u : A. A. Lies in epidural space. Ligamentum Teres. 17. 19. 13. Vertebral venous plexus: Has no communication with cerebral sinuses. 1st rib. 15. A 40 years old man feels pain in his gastrocnemeus muscle after he walks 100 meter which relieves when he stops walking is likely due to obstruction of: A. Is formed by two vertebral veins. Post. It enters the thoracic cavity thru esophageal opening. 16. 14. Tibial artery. Which of the following structure arches over root of left lung: A. . Arch of aorta.20. What is inappropriate about azygus vein. During a per rectal examination the examiner will not reveal: A. Transversalis fascia. 24. 2 1 . Nerve supply to the distal scrotum is t h r u : Illioinguinal nerve. T3-4 25. Pelvic splanchnic nerves. 26. Parasympathetic nerves supplying the urinary bladder are: A. Middle rectal artery. Ureter 23. During an injury to the bulbar part of urethra the urine will extravasate to: A. (correct) Illiohypogastric nerve. The branch of internal carotid artery which remains in true pelvis throughout its course: A. 22. Genitofemoral nerve. Nerve supply to the lower airways: A.20. Superficial perineal pouch. Internal spermatic fascia is a continuation of: A. 27. During surgery of right colon for carcinoma which structure would not be . 6. Regarding Decidua: A. Derivative of Ectoderm include: A. Aorta. 1. 3-8 weeks. 4. Sphincter pupili muscle. Slow growing tumor of head of pancreas will compress: A. left umbilical vein will regress t o : . 3. Embryonic period is: A. Common Bile Duct. EMBYROLOGY. Secretary Epithelium of parotid gland.injured: A. 2. 28. Common Bile Duct. 29. Tumor of the head of pancreas will compress: A. Decidua basilis forms the maternal part of placenta. Which of the following is a derivative of neuro ectoderm: A. Regarding Allantois: …… 7. Styloid process. Occipital Somites. 1 1 . What will happen immediately after birth: Anatomical closure of Ductus arteriosus. Which of the following bone is derived from 2nd Pharyngeal arch: A. 14. Anatomical closure of Ductus venosus. Muscles innervated by hypoglossal nerves are derived from: A. 8.A. Regarding Mesothelium: A. Physiological hernia occurs in between: A. Ligamentum Teres. Which of the following structure is not an Embryological remnant: A. Obliteration of Right umbilical vein. 12. Lateral umbilical ligament. 13. Lines the body cavities. Anatomical closure of foramen ovale. Primordial Germ cells are derived from: . (correct) Obliteration of Left umbilical vein. 6-10 weeks. 9. 10. Ventricular Depolarization on ECG strip is represented b y : A. Blood flow to the left ventricles is increase b y : Acetylcholine infusion. Blood group antigen: . 7. Sympathetic stimulation (correct) 5. During Depolarization: A. QRS Complex. Ventricular preload is measured by: A.A. Clchannels. Yolk sac endoderm. Bain Bridge reflux. 8. IPSP is generated by opening of: A. Indirectly measures left atrial pressure. Pulmonary Wedge Pressure: A. PHYSIOLOGY 1. 2. There is rapid influx of Na+. 3. LVEDV. …………… 6. 4. 12. (correct) Dorrow`s solution. 13. Ringer lactate. Will be inherited by Autosomal recessive pattern 9. What is the suitable I/V fluid for the patient of acidurea: Normal Saline. Distal Tubules. 1 1 .A. O+ve. Proximal Tubules. Simple Diffusion depends on all of the following factors except: A. B. Will not found in any other tissue than blood. 5% dextrose . Albumin. ESR will increase with the decrease i n : A. Renal absorption of glucose thru secondary transport with sodium occurs at: A. ADH will act o n : A. Magnetic field. 10. If father's blood group is B+ve & mother's blood group is AB+ve there child can not have which of the following blood group: A. 12. Skeletal muscle. ???? Pyloric stenosis. Cushing syndrome. Bile of the Liver differs from the GB bile because GB bile contains decrease amount of: A. 16. Neuron. ????? 15. 17. Water. Which of the following hormone is called STRESS HORMONE: A.1 0 % dextrose. Inhibitory factor released by hypothalamus against which of the following hormone: Prolactin Growth hormone. Cortisol. Which of the following causes Hyperkelemia: Zollinger Ellison syndrome. Conn`s syndrome. 18. . Which of the following cell can never reproduce: Erythrocyte. 14. 26. 25. 24. . Surfactant is released b y : A. 23. 2 1 . Regarding Interferon: ………. Type II pneumocytes. 20. 22. Heat loss mainly depends o n : Temperature of the surrounding. Cortisol decreases which of the following cell: A.Smooth muscle. (correct) 19. (correct) Evaporation. Resection of distal ileum will impair the absorption of: A. Hepatocytes. Bile Salts. Increase vasomotor tone. Increase peripheral resistance is due t o : A. Regarding Bradykinin: It formation is activated by killkeran. Which of the following is the NOT non dividing cell: A. Lymphocyte. GENERAL PATHOLOGY . 28. Regarding olfaction: A. Affection of Nasal mucosa.in erythrocytes in exchange of HCO3. Transfer of Cl. Cardiac muscles are prevented by tetanization due to its: Rythmicity Automaticity Conductivity Long refractory period.. Unilateral Anosmia is due t o : A. 32. Sharp odors have the quality of water & lipid solubility. 3 1 . T3. Clonus. (correct) 29. Sustained rhythmic reflex tremors induce by sudden movements is: A. Most active form of thyroid hormone present in circulation is: A. Chloride shift means: A. 33.27. Regarding conduction of visual pathways: ………. 30. Dopamin. 9. ICAM & VCAM mediates: A. The most probable underlying pathological process is: A. 4. At laparoscopy most of the bowl loops were dark purple black. Fluid accumulates in acute inflammation contains: A. Leukocyte Adhesion. A 36 years old women presented with acute abdomen. IL1 & TNF α. 7. Fever in inflammation is caused by: A. Wet Gangrene. Acute pancreatitis. Fat necrosis occurs in: A. 3.1. 5. Irreversible cell injury starts with: A. Superimposed infection on necrosis is called: A. Which of the following is not a mediator of acute inflammation: A. 8. Rupture of the Lysosomal membrane. Proteins >3gm/dl. What is inappropriate about fat necrosis: . 2. Gangrenous Necrosis. 6. Her mesenteric veins were patent. A 16 years old boy deeply jaundiced presented with gum bleeding due to: A. 14. Only caused by trauma to the fat tissue. Fat Embolism. A 25 year old lady with 14 weeks pregnancy had a road traffic accident brought to the emergency department with a large open wound on thigh and femur fracture. Regarding Fat Embolism: It is Fetal in > 8 0 % of cases. the diagnosis of Tuberculosis in this patient is confirmed b y : A. 10. 15. 46 XY. An 18 year old girl came to family doctor complaining of primary amenorrhea. After 2 days of accident she suddenly collapsed & died. Her probable Karyotype is: A. Presence of caseous necrosis in lymph nodes. Edema due to increase hydrostatic pressure is seen i n : . 1 1 . A patient presented with enlarge lymph nodes. 13. on examination she found to have well formed breasts.. A 30 year old women brought to emergency department with bleeding…. 16. blind ended vagina with no uterus & ovary.A. Vitamin K deficiency. 12. The most probable pathological process involve in her death: A. IT manifest within 12 hours. RBC. 2 1 . Vitamin A deficiency. Which of the following is a benign tumor: A. Cardiac muscles. 17. Congestive cardiac failure. 18. Leukoplakia. Skeletal muscles. Warthin`s tumor. . Which of the following is a premalignant condition: A. Cystic hyperplastic endometrium 20. The primary source of Creatinin is: Liver. Erythroplakia.A. Which of the following is a premalignant condition: Condyloma Metaplasia Endocervix. B. Metaplasia does not occur i n : Brain. 19. Maximal tensile Strength of a wound is attained in. Hepatocytes. Apoptosis. Hydatid Cyst. Dystrophic Calcification is seen in all of the following except: Malarial parasite. 24. 10 days. Atrophy of the gland is because of: A. Over an unpredictable period of time. A. 22. 23. 3 Months. 26. Regarding autosomal recessive disorders: ……………… 25. In a patient there is atrophy of submandibular gland due to its duct obstruction. 1 Year. Which of the following are not non-dividing cells. . Dead fetus. 6 days.Lungs. Toxic shock 30. 2. Hepatic hemengioma is associated with: A. A patient admitted to the hosp. The mechanism of injury of ionization radiation i s : A. Usually asymptomatic. Which of the following is the confirmatory test of AIDS: Western blot. Typhoid carriers are: A. 4. Intestinal metaplasia due to reflux esophagitis leads t o : . Which of the disease is common in I/V drug abusers: A. ELISA. SPECIAL PATHOLOGY 1.27. Infective Endocarditis. Free radical formation. 3. Biochemical changes 29. Vinyl Chloride. 28. LASER act by: Cutting. diagnosed as case of gas gangrene dies due t o : A. 9. R/R 20 br/min. She is most probably a case of: A. 8. 12. Hyperthyroidism. Resection of Anterior lobe of pituitary will lead t o : A. Anemia of pregnancy is due t o : A. CEA. Adenocarcinoma. Embryological marker that reappears in circulation in Ca Colon is: A. A 25 year old lady complaining of palpitations. A 6 year old boy is complaining of proximal muscular weakness was . Carcinoma formation.A. Major complication of severe burn: Formation of granulation tissue. heat intolerance on examination her pulse is 112b/min. Which of the following is peculiar for crohn`s disease is: A. Increase plasma volume. Perianal Lesions. 6. What is Inappropriate about hyper parathyroidism: A. 7. 5. BP 120/80. 1 1 . There is increase PO4 renal absorption. 10. Decrease glucocorticoids. 15. Absence of Myenteric plexus. Spleenectomy will help in treating which type of anemia: A. 14. Left ventricular failure will lead t o : A. Effects of hyper parathyroidism in Ca. 18.found to be Ca++ channel antibodies positive. Mysthenia Gravis. The most probable diagnosis is: Lambert Eaten Syndrome. Hereditary Spherocytosis. lung is due t o : Parathyroid like protein Parathyroid hormone 17. 13. Ventilation / perfusion mismatch. . Achlasia is due t o : A. respiratory failure is due t o : A. In pulmonary embolism. Malignant Tumor that will not metastasize: A. Immediate effect after injury to a vessel i s : Vasoconstriction 16. Increase pulmonary arteriolar Pressure. 19. Basal cell carcinoma. In thyroid disease the antibodies are directed against: A.000/mm3 with 75% neutrophils. B. Via Portal vein. After a traffic accident a lady is brought to a hosp the IMMEDIATE step you do: A. Ameobic infection reaches lungs via: Direct extension from liver. 24. By aspiration of trophozoits of Entameoba histolitica. Analysis of fluid from right knee shows: leukocytes 70.20. Clear airway. 2 1 . He has a low grade fever. His knee is hot. Septic arthritis. By aspiration of ova of Entameoba histolitica. tender & swollen. A 50 years old man is awakened in the middle of the night with acute pain & tenderness of the right knee. gram stain = negative. Pseudo gout. Cholesterol 23. C. In Hypothyroidism there is increase i n : A. crystal analysis = negative birefringent. The most likely diagnosis i s : A. Gout. Thyroglobin. (correct) . 22. Drug that decreases the tone of lower esophageal sphincter & increases gastric emptying: A. Heparin will inhibit: . Which of the following blocks α and β receptors: A. 4. 7. Which of the following is suitable antihypertensive for asthma & IHD patient during surgery: I/V sodium nitroprusside. I/V nitroglycerin. 5. Labetalol.PHARMACOLOGY 1. Metaclopromide. It relieves pain as well. Ketamin is used as anesthetic in repeated dressings of burn patient because: A. Which of the following is drug of choice for patient of status asthematicus: I/V aminophylin Oral steroid I/V salbutamol. 3. 2. During surgery antibiotics should be given at: A. At the time of induction of anesthesia. 6. Morphine. Pethidine. 9. 8. Clot propagation. Paracetamol 9] A male with Gynaecomastia. Clot organization.A. small testes. The karyotype involved is A} 45 XO B} 46 XX C} 47 XXX D} 47 XXy (correct) E} 47 XX 10] Somites are derived from A} Intermediate mesoderm B} Paraxial mesoderm . H2 Receptor. Drug of choice for acute pancreatitis. B. Acid suppression is done by blocking: A. diagnosed as Klienfelters syndrome. Which one of the following is most likely? A} Horse shoe kidney B} Constricted ectopic kidney C} Pelvis kidney D} Non rotating kidney E} Unilateral renal agenesis 12] Which one of the following is not formed from urogenital sinus? A} Bartholin's gland B} Urethral glands C} Para urethral glands D} Seminal vesicle E} 13] Virus damage the cell by A} making the pores in cell membrane .C} Lateral plate mesoderm (correct) D} Ectoderm E} Endoderm 11] An I/V Urograph of 8 year old boy shows Excretion of dye normal I kidney but absence of shadow on left side. Instead there is a small shadow just above the bladder on the left side. B} Forming free radicals C} Altering the formation of proteins c D} Nuclear damage E} using energy for their own metabolism 14] The tissue damage by Ionization Radiation is due to A) Damage to Golgi bodies B} Formation of free radicals C} Hydropic degeneration D} Metaphase of cell E} Swelling of cells 15] All are features of IRREVERSIBLE cell injury EXCEPT A} Karyolysis B} Karyorhexis C} Autolysis D} Shrinkage of mitochondria E} Appearance of myelin figure 16] Which one of the following is least likely involved in increasing Osteoporosis . A} Increased w t : on bone B} Renal cell carcinoma C} Carcinoma prostate D} E} 17] Patient X-Ray shows hilar lymph nodes. Granuloma but no necrosis. What is the diagnosis? A} Sarcoidosis (correct) B} Silicosis C} Tuberculosis D} E} 18] In women the most common form of necrosis after trauma is due to A} Trauma to fatty tissue B} C} D} E} 19] Dystrophic calcification is present in which of the following . A} Acute pancreatitis B} Malaria C} Tuberculosis (correct) D} E} 20] Which one of the following is most likely feature of reversible injury? A} Cell swelling B} Karyorhexis C} Decreased glycogen D} Myelin figures (correct) E} 21] A person fell on road and got an abrasion on elbow the first event likely occur would be A} Vasoconstriction (correct) B} Platelet adhesion C} Platelet aggregation D} E} . C5 complex E} 23] Which one of the following is potent antioxidant? A} Vitamin A B} Vitamin C C} Vitamin E (correct) D} Transferrin E} 24] Which one of the following among Arachidonic acid metabolite is most likely function mediated in phagocytosis? A} Chemo taxis (correct) B} Diapedisis C} Transmigration D} E} .22] Which one of the following facilitates phagocytosis in neutrophils? A} C5a B} C3a C} C3b (correct) D} C3 . Which one of the following is most likely? A} endothelial injury B} hypercoagubility C} stasis and endothelial injury (correct) . A few days after he developed pain and swelling in his right leg.25] Which one of the following is potent COX-II INHIBITOR? A} Aspirin B} Celocoxib (correct) C} Indomethacin D} Meloxicam E} Piroxicam 26] In case of pulmonary embolism extensive infarction causes sudden death due to A} Air embolism B} Fat embolism C} Thromboembolism (correct) D} E} 27] A patient was operated for abdominal surgery. He was admitted in the hospital.linked disorder D} E} 30] In a renal transplantation.D} E} 28] A soldier has profuse bleeding. which one of the following is single most significant test to be done? A} ABO blood grouping of donor and recipient B} HLA typing and matching (correct) . Which one of the following is correct? A} Autosomal Dominant disorder B} Autosomal recessive disorder (correct) C} X. Which one of the following should be given immediately? Fresh frozen plasma B} 20 days old blood C} 3 days old blood (correct) D} E} 29] A boy was born with the condition of polycystic kidney disease. Which one of the following is likely etiology? A} Antibody mediated B} T-Cell mediated C} Preformed antibodies (correct) D} E} 32] AIDS occur in the presence of A} Kaposi's sarcoma B} C} D} E} 33] A 2 year old boy is suspected as HIV positive. but within 10 mins the organ failed.C} Donor and recipient should be close relatives D} E} 31] A organ was transplanted to the patient. which one of the following is the confirmatory test in the boy A} ELISA test . which one of the following is seen? . which one of the following type of anemia is developed postoperatively A} Megaloblastic anemia (correct) B} Iron deficiency anemia C} Normocytic norm chromic anemia D} E} 35] Autoimmunity is not involved in which one of the following A} Pernicious anemia B} Grave's disease C} Diabetes mellitus D} Hashimoto's thyroiditis E} Thymoma (correct) 36] In case of hashimoto's thyroiditis.B} Western Blot test (correct) C} PCR D} Increased levels of HIV in T lymphocytes E} 34] A 10 year old boy was operated for gastrectomy. A} Anti thyroglobulin B} C} D} E} 37] To differentiate Chron's disease from Ulcerative colitis. which one of the following is likely significant regarding Chron's disease A} has increased frequency to cancer B} Involves both small and large intestine (correct) C} D} E} 38] Regarding alcoholic hepatitis. . following is finding A} Mallory bodies B} C} D} E} 39] A 20 year old girl has developed Galactorrhea due to prolactinoma. Bromocriptine is given to treat it the drug causes A} Inhibition of prolaction from pituitary gland (correct) B} Decreased prolaction from hypothalamus C} Inhibition of prolaction from breast D} Stimulation of prolaction of anterior pituitary E} 40] An unconscious patient presented in ED does not respond to Naloxone. Which of the following drug has been taken by this patient? A} Morphine B} Heroine (correct) C} Pethidine D} Phenobarbital E} 41] Strongest layer of small intestine is A} Mucosa B} Sub mucosa C} Circular layer D} Serosa E} Longitudinal layer . E.42] Regarding adrenal medulla which one of the following is true? A} Secretes epinephrine and Norepinepinephrine in 80/20 ratio B} C} D} E} 43] R.R most likely A} Synthesizes protein (correct) B} Synthesizes lipids C} D} E} 44] Half life of insulin is A} 5 mins B} 90 mins C} 120 mins D} E} . 45] Insulin A} Converts glycogen to glucose B} Stimulates protein synthesis (correct) C} Stimulates lipolysis D} E} 46] Which one of the following most likely contains abundant cholesterol? A} VLDL B} LDL (correct) C} Chylmicrons D} HDL E} 47] Which one of the following most likely causes Increase in GFR? A} Constriction of afferent arteriole B} Constriction Of efferent arteriole (correct) C} D} . E} Re: FCPS past paper ? July 2008 Dby drwatson » Fri Aug 13. 2010 11:58 am 48] Gastric emptying is delayed by A} Gastrin B} Secretin C} CCK (correct) D} E} 49] Which one of the following pancreatic enzyme is most likely involved in fat absorption A} Amylase B} Lipase C} D} E} 50] Smooth muscle A} Contain sarcomere . Thiamine a b. B2 c.B} T-tubules are present C} Gap junctions are present (correct) D} Forms motor end plate E} 51] Steady pressure on skin is perceived by A} Pacinian's corpuscle (correct) B} Ruffini's corpuscle C} Meissener's Corpuscle D} Golgi tendon organs E} 39. LOH d. High output cardiac failure a. renal failure 40. DCT c. pH Inc. Cause a.a. Ca++ dec. Folic acid . HCO3. Vit D def c. ARF b.most effect. PCT a b. CD 4 1 . Metabolic alkalosis a. PTH 43. Hb 8.inc (35).0. folic acid deficiency d. ARF. B6 d. Pt complains of chest pain for more than 30 min . factor I I I c.V4. Uncontrolled DM . tachycardia e. inc secretion 49. effects o n : a. Ibpratopium bromide given by nebulization: a. No itching or pain. Inferior wall MI d. a. Free T3 c. Lateral wall MI e. Bradycardia b. Inc ICP. false 1 1 5 1 . 90 yrs old lady with purplish large patch on Rt hand and arm. Medullary respiratory group of neurons t of ascending reticular system a. tachycardia d. No comorbids. HTN. free T4 d. Warfarin. Inc ICP. Anion gap increases i n : a.44. anterior wall MI a b. hypoTN. Thyroxine b. Otherwise Normal. HTN. Cushing's triad: a. inc ICP. HTN. bradycardia c. Inferior wall MI 150. dec ACh release in bronchi a b. True b. tachycardia 122. TSH e. Capillary abn b. hypoTN. Prothrombin c 48. Factor V I I I b. vasodil c. Reason? a. Vit K dependent clotting factors 4 7 . Mast cell stabilizer. dec ICP. inc ICP. Na chromoglycate 52. It denotes: a. ECG shows changes in V 1 . Prothrombin c. Sec released from thyroid gland entering circulation is: a. TRH 53. anterolateral MI c. C5a b c. Blood flow regulation is mediated b y : a. Mediators of inflammation a. capillaries d. arterioles c. Bile from liver is different from that from GB i n : 160. likely abnormality is in A} Glucose}-6-Phosphate deficiencyB} Immune hemolysis C} oxysmal Nocturnal Haemoglobinuria D} Malaria d E} Sickle cell disease 5] Thiamine deficiency causes A} Peripheral Neuropathy a B. precepillary sphincter c. capillaries d. venules e. after few hours he developed red color urine. artery b. TNF alpha 1 6 1 .152. arterioles a b. PellagraC} Chelosis D} Dermatitis 6] Milk is notoriously deficient in A} Vitamin C B} Iron C Riboflavin D} Pantothenic acid E} Vitamin A 7] Dry heat kills the bacteria by A} Coagulation of proteins B} Cell lysis C} Free radical formation D} Direct killing d E} Oxidation 8] Virus damage the cell by A} Making the pores in cell membrane B} . C3a b. veins e 162. venules 4] A 16 year old boy was given CO-TRIMAXAZOLE for fever. Largest total cross-sectional and surface area is of: a. IL1 d. C is stimulated by: A} Factor X B} Factor XII C} Thromboplastin D} Factor IX d E} Fibrinogen 13] In enchymal cells Iron is deposited in the form of A} Tranferrin B} Ferritin bC} Heamatin D} Heamoglobin E} Lipifuscin 19] Regarding Von Willbrand disease A} Autosomal Dominant B} Autosomal Recessive C} Some family members have low level of Factor IX D} Some family members have abnormal level of Factor IX E} has positive family history 20] Which of the following condition is related to lymphatics? A} Angiosarcoma a B} Rhabdomyosarcoma C} Cystic hygroma .Forming free radicals C} Altering the formation of proteins c D} Nuclear damage E} using energy for their own metabolism 9] Which one of the following lipoprotein has highest protein content? A} HDL} a} B} LDL C} VL} D} IDL E} CHYLMICRONS 10] All are related to PLATELET FUNCTION EXCEPT A} Capillary resilience test a B} BT C} Clot retraction 11] ALLELLE is: A} Fragmented genes B} Non-identical genes C} Structural genes D} Regulatory gene E} Autosomal Dominant gene 12] D.I. C} Failure to thrive. D} Skin pigmentation E} Thrombocytopenia .21] The condition in which pulmonary trunk fails to divide leading A} PDA aB} Truncus arteriosus C} TOF 22] Basal Cell Carcinoma is located at A} Lower lip B} Upper lip C} Tongue D} Hard palate E} Soft palate 25] Which of the following is not Carcinogen? A} Alcohol B} Benzathracine C} Cyclophosphamide D} Benzidine E} Dimethylsulfate 26] The Initial step in APOPTOSIS is A} Inhibition of P53 genes B} Activation of BCL-2 C} Activation of Caspases c D} pores formed by lymphocytes 28] Which t of kidney produces ERYTROPOITEN A} Macula Densa B} Peritubular Capillary b C} PCT D} DCT E} LOH 29] A patient with signs and symptoms of megaloblasstic anemia. The likely cause is A Decreased GIT motility B} Deficiency of intrinsic factor b C} Malabsorption 30] Stimulation to increase Serum Calcium is A} Hypo-phosphatemia B} atharmone b C} Decreased ECF 31] Malnutrition causes all of the following EXCEPT A} Pitting Edema. a B} Loss of subcutaneous fat. 5%. The most likely causative asite is A} Ankylostomata Duodenale B} Ascris Lumbricoidis C} Diphylobothrium Latum c D} Strangyloids Stercoralis E} Taenia Saginata 34] A 40 year old male after road traffic accident had received massive blood transfusion.5 gm/dl. The most likely change in central area would b e: A} Caseous necrosis B} Coagulation necrosis C} Fatty changesD} Hydropic changes E} Liquefective necrosis 33] A 28 year fisherman presented with slowly developing lethargy.32] Liver biopsy from a pati ent shows a lesion which comprises of central collection of structure less material surrounded by epitheliod cells and lymphocytes. Stool DR: Ova of Intestinal asite. Platelet count: 110 x 10 9/L. Reticulocyte count: 0. His diet comprises of fish an d rice only. One large cell with multiple nuclei arranged close to membrane is also seen. He is expected to have all EXCEPT A} Hypocalcaemia B} Hypokalemia b C} Hypothermia D} Left shift of Oxyheamoglobin dissociation curve E} Metabolic acidosis 35] The endothelial molecules ICAM-1 & VCAM-1 are responsible for A} Direct endothelial injury B} Endothelial cell contraction C} Junctional re Traction D} Leukocyte adhesion d E} Leukocyte margination 36] Eating raw or under cooked meat can cause infestation by A} . Loss of touch sense in both feet and legs Hb: 7. Physical examination: Pallor. easy fatigue and palpitations. Alka Phosphate 2300 IU/L 500IU/L. Healing of this burn will take place from which of the following layers of the skin A} Basal layer of epidermis} Keratinized layer C} Layer of upper dermis c D} Lower dermis E} Subcutaneous layer 38] A 40 year old male presented with jaundice and generalized itching. SGPT 75U/L. The most likely cause is A} Drug induced jaundice B} Extra hepatic Cholestasis b C} Autoimmune Hepatitis D} Alcoholic Hepatitis E} Intrahepatic Cholestasis 39] The tissue damage by Ionization Radiation is due to A} Damage to Golgi bodies B} Formation of free radicals b C} Hydropic degeneration D} Metaplasia of cell E} Swelling of cells 42] Increase in Light Chain Immunoglobulins Is the characteristic feature of A} Endocrine type Amyloidosis B} Hereditary Amyloidosis C} Localized type Amyloidosis D} Reactive Amyloidosis E} Systemic type Amyloidosis .Dracunculous Medinensis B} Echinecoccus Granulosis C Liver Fluke D Schistosomiasis E} Tape Worm e 37] A p t : has sustained burn that is very painful with blisters. His LFT shows Direct Billirubin 11 mg/100 ml. 43] Chemotaxis is caused by A} C5a a B} C5b C} Lymphokines D} Leucotriene B3 E} Opsonins 44] Following are the actions of Archidonic acid metabolites in inflammation EXCEPT A} Chemotaxis B} Increased vascular permeability MEDICINE 25th MAY 2010 .physiology.org/cgi/conte . 2010 10:31 pm 25th may.FCPS I PAPER 1 & 2 Dby drwatson » Mon Nov 01.. 92/3/R1320) d) Mitral stenosis e) Aortic regurgitation Ans: A: 2 About NEURAL TUBE a) Forms primitive streak . 2010 PAPER 1 and 2. DO CONFIRM THE ANSWERS 1 ECG changes can be seen in the following conditions EXCEPT a) Change in body position b) MI c) Sleep (http://ajpregu.. answers.com. in haemorrhage b) Dec in gram negative septicemia c) Dec in heart failure d) … e) … ans.wikipedia.pk/books?id=-V3 .com/Q/What_is_sacra . DNT KNW EXACTLY 5 Central venous pressure a) Inc. b http://books.b) Forms notochord c) Cns develops from it d) Forms three germ layers e) … Ans.. C (http://en..google. of_the_l-5 4 Stroke volume a) Cardiac output depends on it b) Heart rate determines stroke volume c) Increases in haemorrhage d) Independent of venous return e) … Ans..org/wiki/Neural_tube ) 3 SACRALIZATION a) Union of 1st sacral vertebra with 5th lumbar b) Union of 5th lumbar with 1st sacral c) Fusion of all sacral vertebra to form sacrum d) Flexion at sacrum e) … Ans: B http://wiki.. is&f=false 6 What change occurs from lying to standing position? . who’s been deliverd in a remote village. bleeding from venae sites. A http://www.com/Cardiac%20F ..cvphysiology. D synthesis a) 1 hydroxylation ocurs in kidney . septicemia. Ans. B 11.whats diagnosis DIC 10.a) Venous pressure inc b) Arterial pressure inc c) Sweating d) Cutaneous vasoconstriction e) …. /CF017. e 9 A lady. Most common cause of pulmonary embolism? a) b) c) d) Heart failure Dvt Cancer lung Pneumonia e) … ans. Regarding vit. presents with shock.htm 7 A simple senario of hurshspring disease 8 Which hepatitis dangerous in pregnancy a) A b) B c) C d) D e) E Ans.. About external juglar vein a) b) c) d) e) Formed by retromandibular and ant. Treatment of erythroblastosis fetalis? a) b) c) d) Exchange transfusion with b +ve With b _ve Ab +ve Anti D e) … ans.org/wiki/Vitamin_D 12. A http://en.wikipedia. WHICH IS NOT SUPLIED BY OCULOMOTOR N. c . Content of carotid triangle Pierces deep fascia behind sternocleidomastoid … … ans. Auricular v. a) Medial rectus b) c) d) e) Lateral rectus Inf oblique Sup rectus Inf rectus 14. 13.b) 25 ocurs in kidney c) 1 ocurrs in liver d) 25 in lung e) Both in skin Ans. A qs about CSF findings. Cephalic vein a) Present in deltopectoral groove 16. C. . Viridans c) Strep pneumonae d) Staph epidermiditis e) … ans. A lil senario of cervical rib i. ljungdahlii Tetanus toxin C. e weakness and paresthesia along ulnar nerv course 17.botulinum b) c) d) e) C.15. Most common cause of SUBACUTE BACTERIAL ENDOCARDITIS a) Staph aureus b) Sterp. septicum … ans. b 20. Hidden content: UNLOCKED Cause of gas gangrene (C. perferringens ws not givn) a) C. which 1 true etc 18 Diagnosis of typhoid in 1st week Ans… blood culture 19. tube c) Loop of henle d) Collecting duct e) …. Ans. is&f=false 22.pk/books?id=c41 . on&f=false 23. Aldosterone causes ..wikipedia.com. Type of omental necrosis? a) Caseous b) Gangrene c) Fat d) Coagulative e) Fibrinoid Ans.. A http://books. c 25.google. Most of bicarbonate absorption ocurs in a) Proxial c.org/wiki/Gas_gangrene 21.google.. There were 4_5 qs about aldosterone 24.com. Stimulus for aldosterone release a) Hypernatremia b) Hypokalemia c) Hyperkalemia d) Alkalosis e) … ans.pk/books?id=pek . C http://books.. tube b) Distal c.d http://en. http://en. Dec aldosterone will lead to a) Hyponatremia 27.org/wiki/Circadian_rhythm 29. tube c) Collecting duct d) Bowman capsule Ans c 28. c 26. Upraoptic n. which ion replacement is most important? . ADH acts on a) Loop of henle b) Distal c. For circardian rhythm optic nerve sends fibers to a) Optic chiasma b) Medial geniculate body c) d) e) D Lateral geniculate body Suprachiasmatic n. Ans. A person suffering from dehydration.wikipedia.a) Hypercalcemia b) Hyperkalemia b) Acidosis c) Hypernatremia d) … ans. a) Ca b) Mg c) Na d) K e) Cl Ans. What to give to Inc vitamen in diet a) Nuts b) c) d) e) Yougurt Egg Green vegetables Margarine (sumthin like it) Ans. Endogenous pigment in chronic hemolysis a) Hemosiderin b) Lipofuscin c) Anthracotic pigment d) … e) … Ans . D 31 Natural anticoagulant a) Heparin b) c) d) e) Warfarin Aspirin Plasminogen Fibrin Ans. D 32. C 30. all the children there have some permanent disabilitythe parents of such children are suffering from a) Denial b) c) d) e) Anger Bargaining Depression Acceptance 34.edu/~stark/Jav .. In a ward.htm 36. leDist.stat. What is therapeutic index of a drug? 37.A 33. . b http://www. at a children hospital.berkeley.. Square root of variance a) Mean b) Variation c) Standard deviation d) Median e) Acuracy Ans C 35. Mean Inc with a) b) c) d) Inc in sample size Dec in sample size Inc in standard deviation … e) … ans. a 38. Drug for hyperthyroidism in pregnancy a) b) c) d) Methimazolepropylthyrouracil Ppu Radioactive iodine Total thyrodectomy e) Ligol solution Ans B 39. Difference BW hyperplasia and benign tumor a) Hyperplasia invades surrounding tissues b) Benign tumor metastasize to tissues c) Benign tumor shows aggressive growth . Which will not b present in a lung hamartoma (it ws senario of pulmonary hamartoma and qs ws asked at the end of senario) a) Coin lesion on xray chest b) Mostly asymptomatic c) Mostly Discovered incidently d) Benign e) Cartilage in lesion area Ans E 40.Drug contraindicated in renal failure a) Amikacin b) Chloramphenicol c) Vancomycin d) Pencillin e) … ans. Atrophy means a) Dec in cell size 42. . A senario was about sturge webber syndrome in which he gave neurological symptoms I. Submucosal glands are present in a) b) c) d) Stomach Gall bladder Duodenum Small intestine e) Pancreas Ans C 45. Premalignant condition of oral cavity a) Erythroplakia b) c) d) e) Leukoplakia SCC Lichen planus … ans b 43. Point to note that they did not talk about PORT WINE STAIN (birth mark) on face or forehead A 44.d) Hyperplasia shows pleomorphisim e) Benign tumor is surrounded by capsule Ans E 41.e seizures and convulsions then asked that what other symptom can be found in that child and ans ws GLUCOMA . Bile salts are absorbed where? a) Terminal ileum 50. Gastroparesis treatment (last part1 me b tha) a) Metoclopramide 48. Bile acids are conjugated with which amino acid to form bile salts a) Taurine 51. Absorption of long chain fatty acids? a) Ileum 49. Which hormone Inc gastric contraction? A) GASTRIN B) CCK C) Secretin . After gastrectomy B12 is not absorbed due to deficiency of a) Parietal cells b) Chief cells Ans A 47.After gastrectomy what can happen a) b) c) d) e) Malabsoprtion Steatorhea Anemia … … ans c 46. There was a qs about injury at T4.T8 level and dy askd about wat will happen . Rupture of post wall of duodenal ulcer will damage a) Gastroduodenal artery 53 Left renal vein relation to aorta a) Ant b) Post Ans A 54.c) Gip Ans A 52. choices were not straight forward e. Stretch reflex maintains a) Muscle tone b) Balance c) Length d) … e) … ans c .g askin sum tract lesion etc dts y not remeberd 56. Final common motor pathway a) Corticospinal tract b) c) d) e) Alpha motor neurons Upper motor neurons Cerebral cortex … ans b 55. 57. During systole a) b) c) d) Both ventricles contract simultaneously 5 0 % filling of ventricles occurs Blod flow inc in coronary arteries . Glomerular pressure inc due to a) Afferent constriction . Regarding pulmonary wedge pressure a) Called so cz measured by a wedge shaped catheter b) c) d) e) Usually bw 10-15 mmhg Measure of rit atrial pressure … … ans b 59.. Fastest conducting fibers in heart a) Purkinje 61. 58. ans a 60.. e) . There was qs in which options were (forgot qs) a) b) c) d) e) Pancinian corpucles Messiners corposccles Ruffinis end organs Golgi tendons …. Isoniazid side effects can be prevented by a) Pyridoxine 65. Treatment of acute asthma a) Terbutaline 64.b) Efferent constriction Ans b 62. e) … ans c 68. A senario was about dijoxin toxicity 63. AFB +ve a) Leprosy 67. . A pt with deformed nose.020 b) Neutrophils c) Protein > 2 g | dl d) . Most imp point of exudative inflamation a) Sp gravity < 1. Not a cause of cardiogenic shock a) b) c) d) e) Cardiac temponade Mi Arrythmias Hemorrhage Aortic dissection Ans d 66. Tuberculous meningitis a) Inc lymphocytes in csf 70. ESTROGEN OCPs can cause a) Breast ca b) Ovary ca c) Endometrial ca Ans a 73. Which tumor does not hav bone metastasis? a) Thyroid b) Liver c) Lung d) Breast e) Kidney Ans b 72. a) Amount of air left in the lung after a max exhalation . Bladder ca caused by a) Schistosoma haematobiam 71. Residual vol.Monocytes a) Most abundant cells in circulation b) c) d) e) Do not go out of vessels Not phagocytic … … 69. Regarding sup parathyroid gland a) Located in close proximity to INF thyroid artery and recurrent laryngeal nerve at the level of cricothyroid junction.that’s y dy askd about sup glands location. Cardiac output a) Measured by fick principle 76.74.its imp) 77. Vital capacity a) After a max inspiration. Digastric triangle a) Bounded anteriorly ant. extension. Belly of digastric. why a) Dnt remember choices. sory . Which is not covered by deep cervical fascia a) b) c) d) Thyroid Parotid Sternocleidomastoid Submandibular gland e) Sublingual Ans e 79. For surgery tibia should b aproached from medial side. Circumduction a) Combination of flexion. Belly 78. posteriorly by post. max expired volume is vc 75. (Remembr that sup glands hv more constant position while INF r variable in position. aduction and abduction 80. In nephron fluid which substance conc is higher than plasma a) Na b) c) d) e) d 86. Diagnosis of pneumothorax? a) Xray chest 83. then chest pain that relieved by sitting up a) Pericarditis 85. Bicarbonate Glucose Urea Albumin Ans .B) C) D) 81. Pt having uppr respiratory tract INF. Micturation reflex centre is located in a) Brainstem (pontine micturation centre). There s also a sacral micturation centre but it ws not in the choices 82. Approach to intercostal space for pleural effusion drainage a) Lower part of space (neurovascular bundle in upper part os SPACE but lower part of RIB) 84. Dnt remember exactly 88. Jaundice. nerve sup. diarrhea 10days after bone marrow transplant a) Graft vs host disease 89 Erythroblastosis fetalis. WHICH VACCINE IS NOT LIVE A) Measles B) Mumps C) Rubella D) Yellow fevr E) Tetanus Ans e 91. hepatitis. Tissue for HLA typing (there were two same qs in each paper) a) Buccal mucosa b) Leukocytes c) Skin d) Bone marrow Ans b 92. . blood suply. lymph.Renal plasma flow is determined by a) PAH 87. WHAT IS TRUE ABOUT UPPER HALF OF ANAL CANAL A) Options wr about epithelium. type of hypersensitivity reaction? a) 2 90. thanks ALLAH Dy did NT gv both choices simultaneously© 93.LP done at a) L4-L5 (l3-l4 also correct but it ws not in choices. Cauda equina a) Collection of nerve roots and rootlets 94 Pilocarpine is nicotinic a) Agoist b) Antagonist 95 Diagnosis of klinefeltrs syndrome a) Barr body 96 Which drug interers with warfarrin a) Cemetidine 97 Adverse effect of chlorpromazine a) Extrapyrimidal 98 Mechanism of action of captopril a) Ace inhibitor 99 Feature of ca a) Plomorphisim b) Nuclear ratio . c) Hyperplasia d) Invasion Ans d 100 Most cancers hav which fillament in them a) b) c) d) Kerratin Desmin Vimentin Neurofibrilary e) … ans a 101 Paraneoplastic syndrome is associated with a) Ca breast b) Adeno ca lung c) Small cell ca lung d) Ca prostate Ans C 102 Hepatitis b INF is monitored by a) Surface antigen b) Srface antibody c) Core antibody d) E antibody 103 Hepatitis A inf. which test to perform . nt exactly remembrd) . lymphocytes and eiosiniphils are Dec 107 A simple senario of graves disease 108 Whats pattern of antibodies in multiple myeloma (options were vry weird.a) Hep A nd B VIRUS b) Bilirubin c) Sgpt d) Albumin Ans c 104 Which hormone inhibits insulin secretion? a) Secretin b) Cck c) Somatostatin d) Glucagan e) Vip Ans c (As a general rule all git hormone stimulate insulin release and knwn as INCRETIN) 105 MOST IMP STRESS HORMONE (it was also in last part1) a) Epinephrine b) c) d) e) Insulin Growth Cortisol Acth Ans d 106 In Cushing syndrome a) Neutrophils are Inc. photosensitivity (these symptoms wr in the form of a simple senario). butterfly rash. G 20gm b) IgM: igG S 1:20 109 HLA type in RA a) HLA DR4 110 ABSOLOUTE LYMPHOCYTOSIS NOT PRESENT IN a) Sle b) Inf. g IgA 5gm. G in different ratios and %ages e. M 15gm.a) They hav igA. Mononeucleosis c) Walders (sumthin like that) granulomatosis d) … e) … ans a (nt sure) 111 Pt with arithritis. WATS diagnosis a) SLE 112 Platelets contraindicated in a) b) c) d) Splenomegaly Acute immune thrombocytopenia Coagulopathy Dic Ans b 113 Water moves through a) Pores b) Membrane matrix . M. google.c) Protein channels d) Fascilitated difusion Ans A (water channels) 114 Fascilitated diffusion a) Passive transport trough protein channels or carier protiens 115 Which factor strenthen pt doc relationship a) High professional skills b) c) d) e) High social skills Highly qualified Logical answers to qs Active listening Ans E http://docs. rIDzau8oZw 116 Which is more aggressive (also in last part1) a) Basal cell ca b) Squmaous cell ca c) Melanoma Ans b 117 Nissel bodies are a) RER b) SER c) Mitichondria d) Golgi bodies Ans ...com/viewer?a=v&q=cac . Mitral R … ans c . is damaged a) Fascial b) Hypoglossal c) Submandibular d) Edenger westphal e) … ans b 120 A pt with diastolic murmur in aortic area and colapsing pulse.a 118 Greatest area of cerebral cortex is covered by a) Elbow b) Knee c) Thumb d) Shoulder e) Ankle Ans C 119 Deviation of tongue on protrusion. diagnosis a) Aortic stenosis b) c) d) e) Mitral stenosis Aortic R. which N. which will b infected a) b) c) d) Spleen Jejunum Transverse colon Descending colon e) … ans d . Division of MA Ans A 122 Csf absorbed by a) Arachnoid villi b) Choroid plexuses Ans A 123 Which is pierced during LP A) Dura matter 124 Inf spreads retroperitonealy.121 SUB-DURAL HEMATOMA CAUSE A) SUP CEREBRAL VEIN B) INF CEREBRAL VEIN C) ANT DIVISION OF MA D) Post. 125 Ant pituitary loss will result in Dec in size of a) b) c) d) e) Zona glomerulosa Zona fasiculata Parafolicular cells of thyroid Adrenal medulla … ans b 126 Factor Dec wound healing a) Vit c deficiency 127 How thrombocytes play role in thrombosis a) Sorry forgotn the choices 128 Accomoation reflex intact. Damage to? a) b) c) d) Edenger westphal n. Optic nerve Optic tract Optic chiasma e) Pretectal area Ans E 129 Part of portal system draining esophagus a) Azygous v. light reflex absent. b) Hemiazygous . &q&f=false 130 Bronchopulmonary segment a) Anatomical and functional unit iof lung 131 Temp set point is in a) Ant hypothalamus b) Post hypothalamus 132 Antibodies are produced by a) Plasma cells 133 Virulence of bacteria is associated with a) Dose b) Duration of exposure c) Toxin production d) Body resistence e) … ans c 134 For a blood donor what is not required (choices not well remembrd.c) Rit gastric d) Lft gastric Ans D http://books..google.pk/books?id=21e .actualy dy askd that which factor should not b there in a GENERAL HEALTHY DONOR) a) Age b) * * * .com.. c) Systemic disease 135 Which one is not a epithelium tumor a) Adenocarcinoma b) Sq ca c) Liposarcoma Ans C 136 Which receptor r involvd in acid production a) H1 b) H2 c) Acetylcholine d) … e) … ans b 137 Which is present in slow wave sleep a) Dopa b) Acetylcholine c) Serotonin d) Norepinephrine e) … ans c and d 138 Hemibalismus due to damage to a) Subthalamic N. . It’s due to a) b) c) d) Aldosterone Adh Acth Cortisol Ans B 143 Ph 7.4 po2 65 pco2 33 hco3 19 .139 A child having dyspnea when lyng down a) Retrosternal goiter 140 Best buffer of body a) Protein b) Hb c) Hco3 d) Phosphate Ans C 141 Protein utilization is chekd by a) Urinary NO2 b) Blood NO2 Ans A 142 A pt has fluid loss. now his urine is concentrated. a) Metabolic acidosis b) Metabolic alkalosis c) Compensated resp alkalosis d) Resp alkalosis 144 What happens on mountain aclimitization? a) Ventilation inc 145 Syphil dignosis material taken from a) b) c) d) Blood Urine Semen Genital sores (lesion) e) Saliva Ans D 146 Best indicator of serum iron stores a) Serum iron b) Serum ferritin c) Hemosiderin d) TIBC Ans B 147 Why more oxygen goes to alveoli at apex than at base? a) Inc compliance b) Inc blood flow c) In v\q d) Inc pulmonary pressure . 4 b) S2.Ans A 148 Urinary incontinence with overflow and excessive voiding.s1 149 Hypospedias is due to defect in a) Urogenital tubercle b) Urogenital fold c) Urachus Ans B 150 Cephalic vein a) Begins in anatomical snuff box b) Lies lat. 4. To radial a. c) Lat. 3. damage to (qs nt remembrd well) a) S2. To biceps d) Ends in axilla Ans C 151 Upper part of anal canal is a) b) c) d) Lined by stratified sq epi Drained by sup inguinal nodes Drained by inf rectal vein Sensitive to touch . 3 c) L2. e) Sensitive to pain 152 Normal fetal heart rate a) 40…. 120 d) 120…. 60 b) 60…. 100 c) 100….about its inc or dec in specific condition 154 Inc in systemic filling preesure causes a) Inc venous return 155 Beta-blockers do not cause a) Inc renin b) Vasoconstriction c) Bronchiodilation d) … e) … ans a 156 Lidocaine a) Inc PR interval . 160 e) 160…. 200 Ans D 153 One qs was about CVP. b) Dec AP c) Dec automaticity Ans A 157 A diabetic pt with BS 20mmole\l . cause a) Cerebellar lesion b) Upper motor neuron c) Lower motor neuron d) S2.4 Ans A 159 In which parasite the respiratory symptoms predominate? a) Ascaris b) Cystecercosis .its 1st visit to a doc.no complication. obese. 3. What shold be the treatment? a) Insulin b) c) d) e) Insulin + sulphonylurea Sulphonylurea Biguanides Biguanides +sulphonylurea Ans D 158 Pendulous knee jerk. influnzae meningitis a) Csf culture b) c) d) e) Blood culture Inc lymphocyte in csf … … an a 161 Antibody to TB bacteria is (dnt remember exact choices but they did ask regarding antibody against TB) a) Cell bound b) Membrane bound c) in plasma d) In saliva e) … 162 In which phase os cell dividion the chromosomes are arranged in chromatids a) Prophase b) Metaphase c) Anaphase d) Telophase 163 Atrial contraction coresponds to a) C wave b) P wave c) Qrs .c) … d) … e) … 160 Diagnosis of H. skin is calm and cold due to a) Vasoconstriction b) Vasodilation .164 Dorsal roots supply which musscles? a) Internal intercostal b) Externat intercostal c) Innermost intercostal d) Some back musscle 165 Internal spermatic fascia is derived from a) b) c) d) External oblique Internal oblique Transvesalis fascia Transversus abdominus Ans C 166 Inputs to cerebellum a) From vestibular system 167 Resolving power of lens a) That enables to see closely related subjetcs seperatly 168 Parasympathetic stimulation causes a) Ciliary muscle contraction 169 RTA.5L blood loss. 1. suffering from homonymous hemianopia.170 Most common manifestation of septic pt a) Tachycardia b) Hypotension c) Inc GFR Ans B 171 Premalignant condition a) Basal cell nevus b) Dysplastic nevus syndrome Ans B 172 Tumor involving lymph vessels a) Angiosarcoma b) Cystic hygroma Ans B 173 A female having very high levels of prolactin. It’s due to a) b) c) d) Compression of optic nerve Damage to optic tract Damage to optic radiation Compression of upper part of optic chiasma . Ans D 174 What u find in primary hyperaldosteronism a) Inc k b) Dec k c) Hypotension d) Inc renin e) Inc angiotensin 2 Ans B 175 Which take part in synthesis of aldosterone a) Angiotensin 1 b) Angiotensin 2 c) Cortisol d) … e) … 176 Inc k causes a) Inc aldosterone b) Inc ADH c) Dec aldosterone d) Dec ADH 177 HYDROCHLORTHIAZIDE CAUSES A) Inc k .e) Compression of oculomotor n. 2010 11:55 am 1. What type of hypersensitivity reaction occurred? a) 1 b) 2 c) 3 d) 4 e) 5 179 Blood supply of eye except cones and rods a) Central artery of retina 180 They gv senario of klinefeltr and askd what u will find most common a) Gynecomastia 181 Aldosterone causes a) Hyperkalemia b) Hyponatremia c) Hypercalcemia d) Hypokalemia e) Acidosis FCPS past paper ? oct 2008 Dby drwatson » Fri Aug 13.B) Inc ca C) Inc mg D) Inc na 178 Hydrops fetalis occurs to RH –ve mother and RH +ve father.What is most lateral in cerebral hemisphere : . Insula Pars triangularis * Pars interemedia Amygdala 2.Below the umblicus anterior abdomen is formed by: External intercostal and part of Internal oblique* Internal intercostals and part of transverse abdominis uscle External intercostals and transverse abdominis muscle Internal intercostals and transverse abdominis muscle 3.Pre cancerous in aids: Cmv Ebv* Hsv Hpv 4.In liver of a patient under the microscope a lesion was seen with central cheesy appearance surrounded by large cells with lymphocytes and fibroblast what type of necrosis is present Coagulative* . Which has no antigens AB* OO+ A 7.Which harmone of the following stimulates thirst Adh* Aldosterone Renin oxytocin 6.Liquefative Fatty tuberculous 5.In a thyroid surgery mass was excisied which microscopically showed amyloids post surgically which marker would be taken in account PTH Calcitonin* Ca Po4 . Which variant rises with rise in BP TPR* Pulse Venous tone Arterial tone 9.Example of wet gangrene Thromboembolism Bacterial endocarditis Pancreatitis 11.Which of the following harmone stops ovulation during lactation Estrogen Progestrone Prolactin* .8.When r the ventricles most filled with blood ' Diastasis Rapid inflow Atrial systole Ventricular systole 10. Oxytocin 12Which of the follwing delays wound healing Vit c* vita Immobilization Vit E 13.X linked dominant Pku .feature of Downs Syndrome Cardiac anomolies Fallots Hypospadiasis Early death* 14Turner syndrome has Chromosomal defect Xlinked Short stature* Tall stature 15. Alkaptonuria Familial polyposis* Glycogen storage 16.Asthma has low Fev1* Fvc Vc Tlc .In dorsal column lesion which of the following will be lost Pain Temp Crude touch Itch Proprioception 18.Which of the following tissue is most radiosensitive Skeletal muscle Cartilage* Bone 17. Common feature in nitrates.Temporal Arteritis is diagnosis by Esr* Biopsy .19. Throphylline is Postural hypotension Rapid pulse Vasodlation Venodilation 21. isoprenaline .Midarm circumference is the measure of Proteins* Fats Carbohydrates Minerals 22.Pre carcinogenic is Berryliosis Asbestosis* Benzethe Pentamidine 20. B/c WBC count 23.Long term granulomatous disease is Crohn Celiac Sarcoidosis Ulcerative colitis* 25.Water reabsorption through ADH is done in Proximal Distal Collecting Loop of henle 26.In food poisoning death occur by Exotxin of samonella Vibro cholera * Shigella Endotoxin of Shigella 24.The organ placed retroperitoneally is . Hipocampal injury causes * * * rage Hunger Loss of Memory* .Spleen Transverse Bladder Ascending* Liver 26.Urinary bladder in male is Completely covered by peritoneum Superiorly covers From rectum separated by fascia of Denon Villiers 27.Hydrocephalus occurs because of obstruction in I/v foramen* Sylvius Magendie Lushka 28. Paramesonephric duct remnant in male is Appendice testis Appendix Ovary 30.Temperature disturbances 27.Chemotactic factors functions mostly in Diapedisis Margination ' Migration Acute inflammation 29.What organ lesion causes hypothermia Preoptic Lateral hypothalamus Post hypothalamus * Pitutary 28.Musle traversing the shoulder joint Crochobrachilis Sspinatus . Ispinatus 31.Erythropoietin is secreted from Yellow marrow .A boy presented in the ER with fracture with less seum calcium.He is suffering from Primary hypoparathyroidism Vit D deficiency 2ndry hypoparathyroidism 33.Blood spread of CA occurs 1st in Metaphysis Diaphysis Epiphyses Epiphyseal plate Cortex 32.Example of synchondrosis is Symphisis Involves hyaline crtlg b/w 2 bones Costal joint 34. Juvenile polyps are also called Hamartomas Rectal polyps Internal haemorrhoids 37.Jg complex Macula densa Preitubualr capillaries Loop of henle 35.laterally rotate Adducts . medially rotate 36.TGlycerides are absorbed in Chylomicrons Vldl Hdl .medially rotate Abducts. medially rotate Abducts. laterally rotate Flexes . minimus Adducts. Gluteus medius . In gastrectomy loss of following occurs Hcl Intrinsic facor Fats Vit D 39.About antibodies Have varible2 heavy chains IgM is abundant 42.Ldl 38.Hb is enclosed in RBC at stage of Normoblast Late normoblast Erythroblast 40.Adrenal gland is separated from kidney by Peritoneal fascia Renal fascia Perinephric fat 41.Median nerve supplies . About 1st lumbrical Arises from 2nd meta carpal Is bipennate Supplied by median nerve 44.Lumbricals are supplied by Median neve deep br of median n Deep br of ulnar n 45.Lateral part of breast supplied by Ant group Post group Lateral group Central group 46.The muscle of passive inspiration is Diaphragm .Palmar medial 1/3 Palmar and dorsal medialm1/3 Dorsal medial 1/3 Dorsal 2/3 lateral 43. External intercostal Internal intercostal Scalenes muscles 47.Lymph drainage of lateral 2/3 of tongue occurs through Sup deep cervical nodes Inferior deep cervical nodes Submandibular nodes Submental nodes 48.Boil on the tip of the tongue will drain into following lymph nodes Submental submandibular Submental nd submandibular 49.Left recurrent laryngeal nerve has Shorter route than right recurrent laryngeal nerve .Posterior triangle in the neck has follwing boundries Ant ant border of SCM post omohyoid Post ant border of SCM and ant diagastric Ant post border of SCM and post ant border of trapezius 50. After illeal resection absorption of which of the following components does not take place Chylomicrons Bile Tglyceric acid Vit d .Right hepatic artery is branch of Hepatic artery gastroduodenal artery Ceoliac artery Splenic artery 52.Branch of vagus nerve Supplies all the muscles of soft palate Supplies all the muscles of larynx 51.Subcapsularis sinus is present in Spleen Lymph nodes Payers patches Palatine tonsils 53. Patientt has Babinski + lesion is in UMN LMN Dorsal column Ant column 57.Vit c 54.Inferior rectal artery is a branch of Ext illiac .Patient after radiation vomited for 4hrs Cl is raised. he has Metabolic acidosis Respiratory alkalosis wirh ion gap Metabolic alkalosis 56.In CRF all of following occurs except Anemia Hyperlaemia Hypocalcemia Hypoposphatemia Hyperuricemia 55. Peristalsis in the intestine inhibited by CCK Gastrin Secretin GIP Histamine 59.One of the following is not a feature of anaerobe Cough Tetanus .I n t illiac Sup epigastric Int pudendal artery Femoral artery 58.Granuloma formation does not occur in TB Cat scratch fever Toxoplasmosis Pneumonia 60. Pus Granuloma formation Gangrene 61.Structures which passes through the aortic opening is vagus and azygous vn Azygous vn Esophagus Phrenic nerve 63.Melanocytes originate from Neural crest Mesoderm Ectoderm 62.Liver helps in immunity by .IJV drain in Petrosal sinus Sagittal sinus Straight sinus Sigmoid sinus 64. Detoxfying noxious agents Kupffer cells Hepatocytes 65.In testis seminferous tubules show following under M/S Sertoli cells Spermatogonia Mixed cells Spermatocytes .ADH is inhibited by Alcohol Angiotensin Insulin Renin 67.Sour and hot is perceived by tongue through Von ebner cells Folate papillae Pain fibers Nociceptors 66. A boy is malnutrioned and has edema due to protien loss his Plasma colloid pressure is decreased Increase hydrostatic pressure Blocked lymphatics 70.Constricting afferent ducts in kidney reduces Gfr Rpf Creatinine clearance Glucose claerance 69.After appendicectomy what may be damaged Inguinal ligament Superior epigastric artery .Cholestatic cholangitis is caused by Colonercis selecans diphyllobothrum latum Schistosoma Entameoba 71.Sperms 68. Illhypogastric nerve Spermatic cord 72.Following is true about appendix Appendix ha incomplete muscular coat Appendix not attached to mesentry Supplied by ceoliac artery 73.The mc burney point indicates the presence of tip of appendix.Apex of the femoral triangle .Femoral artery can be palpated in Mid inguinal lateral to pubic tubercle Mid point of inguinal ligament Adductor canal 75.One of the following is not a branch of External carotid artery Lingual A Ophtalmic A Occipital A Maxillary A Suprficial temporal A 74. Medial side of hand is supplied by Median nerve Radial nerve Axillary nerve 77.Vomiting center is persent in Hypothalamus Pitutary Pons Medulla Midbrain 78.Patient wirh Gravida 4 Par4 Abortio 0 with anemia and occult blood in stool Fe deficiency anemia Pernicious anemia .Lies under inguinal ligament Contains small saphenous vein Contains nodes Is pointed downwards 76. Artery to head of femur is Medial femoral circumflex a Lateral femoral circumflex a Obturator a Femoral a 82.Nerve to ligamentum teres capitis is Femoral n Obturator n Medial femoral n Lat circumflex femoral n 81.Injury to the neck and head of fibula causes damage to Sural nerve Sciatic nerve Tibial nerve Common peroneal nerve 80.Anemia of chronic disease Sideroblastic anemia 79.Muscranic receptors are present in . Postgaglionic parasympathetic Postgaglionic sympathetic Preganglionic parasympathetic Preganglionic sympathetic 83.Essential fatty acid is Linoleic Palmitic Citric Oxaloacetic 85.In tissue bleeding occurs because of Loss of coagulation factors Congenital disorders Endothelial damage .On bronchoscopy bronchoscope will 1st enter Sup bronchus Apical bronchus Inf bronchus Middle apical bronchus 84. Thrombosis is initiated by .Role of middle ear bones is Transmit sound Amplify sound Does not respond to low noise Are sesamoid bones 88.Following is not a feature of DIC Dec FDF Dec BT Dec PT Dec thrombin time Dec platelets 87.dec FDF 86.Direct inguinal hernia lies Lateral to inf epigastric a Medial ti sup epigastric a medial to inf epigastric a Medial to pubic tubercle 89. 2 ATN is associated with. a) Hyperkalemia b) conc...1 Paracetamol a) b) c) d) increases PT its overdose causes Nephrotoxicity is a poor anti-inflamatory is more stronger than codeine e) causes met-Hb.Medicine october 2007 paper • by drwatson » Thu Jul 22.Leukotrienes Prostacyclin Arachidonic metabolites Thromboxane A2 90.emia more frequently than Phanacetin Q.Most drug metabolism occur in Kidney Liver Spleen Intestine FCPS . 2010 4:57 pm Q. urine in initial stages . 7 in an Asthamtic female.. a) Essential HPTN b) Sympthatic stimulation c) Nitroglycerine d) Excercise e) Hypoxia Q.4 MS is characterized by..immeditae treatment of HPTN during intra-operative HPTN..c) increase in s.creatinnine d) Q. is a) sublingual glycerine trinitate b) intra.3 Congenital cataract is associated with a) chiken pox b) chromosomal abnormality c) small pox d) Rubella Q.6 Pulmonary Arterial pressure increases with.5 All of the following have ability of phagocytosis EXCEPT a) Microgila b) Lymphocytes c) Kuffer cells d) Neutrophils Q.urea but normal s.venous nitroglycerine . a) Lewy bodies b) Patchy demylination & white fiber gliosis c) axonal neuropathy d) Q. 10 Non-essential amino acid includes a) Leucine b) Iso-leucine c) Methionine d) Tyrosine Q.12 Glucronide conjugation of drugs a) causes inactivation of drug b) is Type 1 reaction c) makes drug insoluble in water .as a result along with dryness of nasal mucosa also occurs a) decreased secretions of Parotid b) decreased secretion of Submandibular & sublingual glands c) Q.11 Metabolic Alkalosis is caued by a) Chronic Hypoventilation b) Pancreatic Fistula c) hyperglycemia d) ATN Q.c) propranalol d) Q.8 During giving local anaestheia.9 Tissue which has in-ability to replace dead cells a) Heart b) Kidney c) Liver d) Bone e) Bowl Mucosa Q.causing anaesthesia of an autonomic ganglion. the long needle goes far into greater palatine canal. 4 md/dl MCV..14 Tissue with Dual autonomic supply but not reciprocal suuply is a) b) c) d) Skin Blood vessels Pupil Salivary glands e) Ciliary muscle Q. c) Vit B 12 def..16 In summer.15 Cause of joint pain in Deep sea divers a) arterial embolism b) venous embolism c) DIC d) thromboembolism .15 Cause of joint pain in Deep sea divers a) arterial embolism b) venous embolism c) DIC d) thromboembolism Q. Hb= 7. the anemia is due to .> 120 fl. on investigation he is having Un-conjugated Bilirubinemia.d) increases its effects Q..13 A patient having generalized body weakness & numbness. d) Hypersplenisim Q.in a fasting man the cause of concenterated urine is a) b) c) d) Skin Blood vessels Pupil Salivary glands e) Ciliary muscle Q.. a) Hemolysis b) Vit B 9 def. S4 b) Pelvic splanchnic Nerves c) vagus nerve d) Q.in a fasting man the cause of concenterated urine is a) b) c) d) ADH No Oral intake Aldosterone increased sweeting Q.S3. 21 Parietal & Visceral Pericardium is supplied by a) Phrenic Nerve b) Sympthatic Nerves .19 Parasympthatic supply of Sigmoid Colon is a) Sacral segments S2.18 if a High Dextrose water is infusion given to a person.the confirmatory test of the diagnosis will be a) Tensilon Test b) Ach Receptor Antibody Test c) blood culture d) CT scan Brain Q.it inhibits a) Cotisol b) ADH c) Aldosterone d) Insulin Q.16 In summer.17 A 35 yr old man is having bilateral ptosis of eye during evening since last few weeks.Hidden content: UNLOCKED Q.20 which does not take part in inflamatory reactions a) Adrenaline b) Histamine c) 5-hydroxy tryptamine d) Dopamine Q. .26 A person has sharp knife injury to upper medial aspect of arm. the nerve injured is a) Median Nerve .c) Vagus neve d) Cardiac Plexus Q. this swelling most probably due to.25 Muscles of Hand supplied by a) b) c) d) ventral rami of C-8 Ventral branches of T-1 Median Nerve Ulnar Nerve e) Radial Nerve Q.24 Dorsal Rami of the cervical nerves innervates a) Abductor Pollices b) Extensors of Limbs c) Extensors of Trunk d) Stabilizers of Shoulder Q. after which He is unable to Flex his elbow & having loss of cutaneous sensations on lateral aspect of forearm. Now he notices a bulging swelling above the clavicle on sneezing & coughing.23 Middle meningeal artery if ruptured while meinges are intact.22 A young man had sinjury to the neck with sum sharp object.3 yrs ago..the Haemorrhage will be a) Epidural b) Subdural c) Intracranial d) Q. a) Injury to deep fascia of neck b) demage to supra-pleural membrane c) Un-united first Rib Fracture Q. 30 A 30 years old 75 kg Diabetic man with recent diagnosis of Hypertension.Urea -.Uric Acid-.29 Pharmakokinetic interaction among drugs is examplified by : a) Shortening of action of Procaine by Adrenaline b) increase peripheral toxicity of Levodopa with Carbidopa c) increase toxicity of Methotrexate by Aspirin d) prevention of Nitroglycerine induced tachycardia by Propranolol e) blokade of acetylcholine receptors by atropine Q.2. K-. On ECG : Left Ventricular Hypertrophy s.23 mg/dl 24 Hr Urinary Proteins-.5 mg/dl s.Na -.103 mmol s.7 mg/dl s.5 mmol s.b) Axillary nerve c) Radial Nerve d) Musculocutaneous Nerve Q.80 mg/dl The cause is a) Essential HPTN b) Diabetic Nephropathy c) Pheochromocytoma .0.Cl --.7.20 mg/dl s.HCO3 -. 27 A Transplant from identical twin is a) Allograft b) Heterograft c) Homograft d) Isograft e) Xenograft Q.Creatinine -.30 & Fasting BSL--.140 mmol s.28 Therapeutic Index of a drug indicates its a) Effectivity b) Efficacy c) Potency d) Toxicity Q. a) Lewy Bodies in Neucleus b) Ptachy demylination with white fiber gliosis c) Axonal degeneration d) Q.34 Multipule sclerosis is characterized by.31 Acute Tubular Necrosis is characterized by a) Hyper-kalemia b) Initially concenterated urine c) Metabolic Alkalosis d) s.35 What is INAPPROPRIATE about Vagus Nerve.d) Addison Disease e) Primary HyperAldosteronism Q..Urea is increased & s...32 GFR is measured by clearance of a) Inulin b) Creatinine c) Urea d) Glucose Q.creatinine is normal Q.33 Sarcoidosis is diagnosed microscopically by a) Granuloma with Asteroid b) Non-caseating Granuloma c) Caseating Granuloma d) Macrophages & Giant cells Q. a) its longest Cranial Nerve b) c) d) e) passes in between IJV & ICA passes posterior to Carotid Sheath Gives Recurrent Laryngeal Nerve gives Superior Laryngeal Nerve Q.36 Thymus is a) Rich in Lymphocytes . a) recieves Lymph from both right & left sides b) lies in posterior mediastinum c) passes just right to the thoracic aorta d) extends from upper abdoment to neck e) has a dilatation called Cisterna ..39 Following is Autosomal Dominant Disorder a) Phenylketonuria b) Hereditary Spherocytosis c) Hemophilia A d) Cystic Fibrosa Q.in sample taken from amniocentesis...b) Present at birth but shorly regree after birth c) derived from 4th Pahryngeal Pouch d) is a part of Thyroid e) Lies posterior to Trachea Q.40 At 35 weeks of Gestation.41 Thoracic Duct. there is increased Alpha-Fetoprotein..37 True statement about Down Syndrom is a) Trisomy 23 b) contains single X chromosome c) more chances with increasing maternal age Q.38 Kleinifilter syndrome a) contains single X chromosome b) less chances of Mental retardation c) phenotypically female Q.its most probable cause is. a) Down syndrome b) Turner syndrome c) Spina Cystica d) Hydrocephalous e) Q. 46 All are the branches of Vagus nerve EXCEPT a) Auricular b) Lacrimal c)Pharyngeal d) Recurrent Laryngeal Q.42 Rate of passage of food in esophagus depends upon a) Gravitational Force b) Neuromuscular excitation different @ upper & lower oesophagus c) progressive peristalsis in esophagus d) Acidity Q.44 Source of Estrogen & Progesterone in last trimester is a) Chorionic Villus b) c) d) e) Placenta Overy Adenohypophysis Corpus Leuteum Q.43 @ Leprotomy Retroperitoneal Abscess was found.47 Following nerve lies in the Cavernous Sinus a) Opthalmic b) Occulomotor .45 A patient having Facial deviation along with loss of sensations on anterior 2/3 rd of tongue.the leision lies in a) Facial Canal b) Internal auditary meatus c) Facial Nerve Nucleus d) Stylomastoid Foramen Q.the site woud be a) b) c) d) Stomach Sigmoid Colon Transverse Colon Ascending Colon e) Small intestine Q.Q. Dd ) blood group.50 Na Retension is associated with a) Dehydration b) increase ADH c) Decreased Aldosterone d) Heart Failure e) Hyperthyroidism Q.children can have all blood gp EXCEPT a) AB +ve b) AB -ve c) A + ve . married to a female having B+ (Bo.51 Von-Willebrand Disease is a) Autosomal Dominant b) Autosomal Recessive c) in some family members asoociated with Factor IX deficiency d) in some family members associated with normal Factor IX Q.c) Abducent d) Trigeminal Q.51 A male wd A+ (Ao.48 In a Pituitary Adenoma.49 Muscle of Forced Expiration is a) b) c) d) Internal Intercostal External Intercostal Sternoceidomastoid Scalenus Anterior e) Scalenus Medius Q.DD) Blood group.the Bitemporal Hemianopia occurs due to leison of a) Central part of Optic chiasma b) Optic Nerve c) Optic Tract d) peripheral Part of Optic chiasma e) Lateral Geniculate body Q. d) B +ve e) O -ve Q.55 In a CLD patient presenting with Haematemesis. d) PPI e)whole blood Q.52 Vital Capacity a) measured directly by Spirometry b) is decreased in some Lung diseases c) is the sum of Tidal Volume+ERV+IRV Q.following Lung function increases a) Alveolar Co2 b) c) d) e) Alveolar O2 Pulmonary artery O2 Pulmonry artery Co2 Ventilation Perfusion Q. the nerve demaged is a) Infragluteal .the preferred immediate treatment is a) Octeriotide b) Vit K c) formed Platelet Conc. of cells c) Capsule d) Q.54 Difference between Adenoma & Hyperplasia is a) increase in size of cell b) increase in No.53 When a branch of Pulmonary Artery is blocked by embolus. a person has inability to stand from sitting position.56 Among most important initial steps in apoptosis a) Activation of Bcl-2 Receptors b) Activation of Caspases c) Water influx into the cell d) Q.57 After a trauma. 59 A man with persistant fits without regaining consciousness for 30 minutes.58 About K .Regulation . the immediate treatment is with a) Carbamazepine b) Valporate c) Lorazepan d) Q.is a) CCK b) Secretin c) Gastrin . following is correct a) K is 9 8 % in the cells b) normally completely reabsorbed in Distal tubule c) increase water affects K-balance d) decrease in D.60 An unconsious man with pin-point pupils not responding to Naloxone.b) Supragluteal c) Schiatic d) Femoral e) Obturator Q.62 The Hormone which stimulates release of HCO3 from Pancreas without stimulating pancreatic enzyme secretions.the probable cause is a) Heroin b) Hasheesh c) Morphine d) Phenobarbitone Q.K Q.61 Stomach gets its arterial supply from all EXCEPT a) Ceiliac artery b) Splenic Artery c) Superior Mesenteric Artery d) Hepatic Artery Q. .. Q.67 About CSF most appropriate statement is a) it is ultrafilterate of Plasma b) has more glucose than blood c) has cushioing effect d) absorbed in Choroid Plexus Q.it is a) Squamous Cell CA b) Adenocarcinoma c) Rhabdomyosarcoma d) wart Q.64 Stenson's duct a) arises from posterior surface of Parotid b) pierces the Buccinator muscle c) opens in the oral cavity against .@ apex of lung there is increase in a) Blood Flow .63 Supra-Renal Gland arterial supply a) recieves all braches directly from Aorta b) recieves blood from 3 sources c) recieves branches from both renal arteries d) recieves blood from inferior phrenic artery Q.d) Pepsin e) Mucin Q..65 Which infection is not caused by DNA virus a) b) c) d) e) Chicken Pox Herpes Simplex Herpes Zoster Mumps Small Pox Q.66 On examination of a female.a plaque like leision seen on Postero-superior aspect of Vagina.68 On standing from a sitting position. b) Ventilation c) pO2 d) pCO2 Q.69 Metastatic Calcification occurs mostly in a) Kidney b) Bone c) Lung d) Intestine e) Pancreas Q..70 Basal Cell Carcinoma involves a) Buccal Mucosa b) Hard Palate c) Soft Palate d) Lower Lip e) Oral Cavity Q. the inability to absorb B-12 is due to loss of.71 A female underwent Gastrectomy for Zollinger-Ellison Syndrome. a) Parietal Cells b) Gastric Cells c) Cardiac Ceells ..given Parenteral Vit B-12. 27 b. 3 days d. can explain ischemia of myocardium b.because a. recovery is quick d. regarding phosphodiestease inhibitor (I can just recall there was some option regarding its inactivation of cAMP. Humidity in OT should be a. volume of distribution depends on following except a..37 3. it is less toxic b. do not explains vascular flow c.regarding coanda effect what is true a. Ropivacaine is used for epidural anaesthesia instead of bupivacaine during labour. of nitrogen is 21c e.. regarding flow of gas in a tube. age b. what is true about critical temperature: a. radius . nitrogen breath test can measure it c. it is part of minute ventilation d.but don’t remember exactly now what the other options were ) 6. explains unequal distribution of air in alveoli 9. is temperature below which pressure can keep gas liquid 2. 1 week c. what is true about FRV volume. a. b. no effect on fetus c. half life of pseudocholine esterase is a.45 c.1. 2 hours b. of co2 is 31c c. gender c. 12 hours 5.directly proptional to a. of oxygen is 121c d. increased in shallow breathing b. can be measured by helium dilution metho 8. applies on mixture of gas. prolonged action 4.which I ticked. lipid solubility of a drug 7. hyperkalemia .more potent analgesic than codeine e.lisinopril 13.not an anti inflammatory d.which one of following drug worsens angina a.vasodilator 16.nitoglycerine c.verapamil b. Chances of malignant hyper thermia increases b.increases PT… b. surfactant produced b.isossorbide dinitrate c.ondansteron b.is nephrotoxic c.droperidol d. viscosity 10..analgesic..causes met-Hb. regarding paracetamol what is true… a.child with burn 6 hour back brought to ER for dressing.cyclizine c.nimodipine d.nifedipine c. pressure difference c. b.metoprolol e. a.metoclopramide e.porcholrperazine 12.diltiazem 15.which one of following used in CCF but is not a vasodilator… a.flow of inhalational gases across memebrane is directly propotional to.diltiazem 14.b.isosorbide dinitrate b.propanolol d.drug of choice in prizmental angina a. which one of following anti emetic has max bioavalability a.verapamil b.nitous oxide is a .emia more frequently than Phanacetin 17.. transalveolar pressue c. a. Family history of sibling died under GA is positive…you are cautious regarding using succinylcholine in this patient why? a. hypoxia 11. nitrous oxide 23.ketamine .2 d. pt is hypotonic. a.sensory supply of dura is by all except a. due to increased risk of prolonged muscle contractions 18.hypoglossal nerve 19.. After induction of anesthesia by 10 mg/kg thiopentone. desflurane b. sevoflurane d. hypoxia.thiopental d. dec contractility 26.trigeminal nerve b.Mac of isoflurane is a.vagus nerve d. hypersensitivity b.1. Cause ? a. inhibiting intracellular phospateases c.0. lingual vein c. duration of anaesthesia c.profol c.5 20.1. dec HR c.0. all of the following are tributaries of internal jugular vein except: a.which one is potent bronchodilator.mechanism of action ) a. which one irritates larynx.midazolam 27.c.3 b.) d.mac is reduced in… a.5 c. isoflurane c.but the main theme was how isoflurane actually suppresses cardiac activity.ketamine b. decreasing AV conduction b. sigmoid sinus b. superior thyroid d.exact term don’t remember but it was bi. oocipital vein 25.cervical nerves c. 22. bidirectional block (something like that .which one of following has steroid nucleus. a. 24.isoflurane effects cardiac activity by: (options don’t remember exactly.can be used in asthmatic patients a. tributaries similar to aorta c.if ventillation increasd.thiopental effects on cvs: a.b.heat loss is by a. diabetic patient undergoing hemicolectomy c. b. a. d.is related to bare area of liver 32. give accurate measurement when catheter in rt atrium c.. 31. inferior vena cava b. regarding left adrenal vein drains into… a. a. a.radiation n conduction 36.recommended for anaesthesia induction. 29..lies posterior to right adrenal gland d.patient in ot undressed.dead space develops 37. a.regarding isoflurane one correct statement.toxicity 34. 30.commences at L5 b. directly proportional to viscosity 38. b. left renal vein 33. correct statement regarding inferior vena cava … a. a patient of thoracotomy wid oozing wounds b. ketorolac drug of choice in… a. always low in all types of shock. adrenaline is added to local anaesthetic to restrict its… a.propofol 28.transient fall in bp b.. c. gives clue about pulmonary edema d.. reading 5cm H2o is equal to 10 mmhg 35. regarding cvp whats true….laminar flow what is true…. transducer is… . ringers lactate . b. a.just remember there was a question regarding it as well ) 42.fluid of choice is… a.ARF 40.respiratory rate d. decreases in old age 44.reaction type is type 4 41. produced by golgi apparatus b..a .in sternous exercise wats not increased….pulse pressure a.internal jugular vein b..correct statement is: a. whats true about montoux test. blood.3rd space fluid loss.dopexamine 48.Diagnosis? a.bronchial tone circadian rhythm max at …a. is differance of systolic and diastolic pressures b.adenosine mode of action… a.10%dextrose c. has hydrolases n catalases c.regarding carotid sheath. cardiac index is.. has oxidases 46.cardiac output c.convert one energy form into another 39. lysosomes a.hemolysis b.. a.( cant recall what they asked.formed by condensation of pretracheal fascia 46.CRF c. Serum Potassium is 5. 47.blood pressure 43.heart rate b.known hypertensive.prednisolone.plasma b.early morning 49.vagus nerve.patient on htnsive drug now comes in shock .contents are carotid artery. a.difference of end systolic volume and end diastolic volume 45.. cardiac output to body surface area (BSA). facial 57.. c. 56. parietal pleura 58.attached to first rib c. d. c.10-15% blood loss.patient now having bleeding from left ear n left eye pupil dilated.occulomotor b.trochlear d.road traffic accident . b.carbondioxide produced per oxygen utilization 59. lidocaine 51.supra pleural fascia is …. heart. aorta. ringers lactate 52.pulmonary ligament is a.select one false….all are mesoderm derivative except… . normal saline 50. quinidine b. supplies posterior 2/3 of tongue. lungs. a. a.leaves through styloid mastoid foramen. chorda tympani. pilocarpine c..thoracic part of sympathetic trunk supplies all except.mac of sevoflurane with 60% N2O . b .trigeminal c. ..formed by condensation of Sibson fascia 53.subclavian vessels crosses it superiorly 54. a recess in parietal pleura c. d. passes over upper part of medial side of tympanic membrane 55...moves with respiration.respiratory quotient is a.replace by .d. a. whole blood b. a.thickining of pretrachial fascia b.class three antiarhythmic name a. three times normal saline c.which cranial nerve damaged a. in which lung root sinks during respiration b. nitrogen breath test can be used for a.66-0.influenza d. 66.neisseria 64.what is true a.surgical.sternoclavicular c. incomplete cartilages in airway tract tubes c.thymus c. to pyogenic lung abcess a.anatomical .related to bare area of liver d.spleen b.functional unit of lung 63. a. right is longer than left c.lies posterior to right renal gland c. has valve b. cilliated epithelium uptil terminal bronchus 65.dorsal rami of cervical spinal nerves supply 62.lymph is drained into a.which of following organism causes menigites sec.true regarding trachea a.has a segmental bronchus b.regarding right brachiocepahilc vein.measuring dead space .staphyllococcus c. commences at level of t4 d.manubrosternal b. 68.symphsis pubis d.which of following joint is immoveable a.what is true .commences at L5 b.streptococcus b.has tributaries similar to aorta .shoulder joint 61.regarding inferior vena cava.lymph node 69. contains macrophages in alveoli b.regarding bronchopulmonary segments correct statement is a..68 60. is vertical 67.h.filling of ventricle produces which heart sound . 0.a. vagus nerve.regarding ropivacaine what is true a.1 .lidocaine d.suxamethonium causes tachycardia in a patient .because of hyperkalemia b. b.desflurane c.why ? a.basillar artey c. c.isoflurane b.a patient is inducted with an inhalational agent.you are very cautious regarding second dose.ropivacaine c.dimethocaine 77.continuation of sigmoid sinus.anterior inferior cerebellar artery is a branch of a.occulomotor nerve.regarding ropivacaine what s true: .vertebral artery d.bupivacaine b. c.pica b.70.what is true about amide local anaesthetics: a. (I think in question it was also mentioned about mac of that agent and it was < 0. b.equally effective than bupivacaine d.hypoglossal nerve.. 71.don’t remember exactly ) which agent is responsible for this: a.trochlear nerve 73.most toxic local anaesthetic is: a.regarding internal jugular vein a.it blocks muscarinic receptors at postsynaptic terminal 78..what is the mechanism ? a.you have given a dose of suxamethonium in a patient .now u have to give another dose.long duration of action c..pKa is equal to that of bupivacaine 76.which of following doesn’t have any sensory supply a.lingual nerve is its tributary.less cardiotoxic b.it blocks nicotinic receptors at autonomic ganglia c.sevoflurane 74.they are bound to albumin 75..it blocks nicotinin receptors at post synaptic junction b.because of hypocalcemia 79.he develops bronchospasm n airway irritation.hypoglossal nerve crosses it anteriorly. d.halothane d.posterior cerebral artery 72. by decreasing av conduction c.a. Prilocaine d.dosesnot release histamine 91.thyroid d.density c.lidocaine b.ace inhibitor acts in ccf by .which local anaesthetic causes methaemoglobinemia: a.progesterone 86.due to less production of metabolites b. 60% c.<50% b. 80% 90.low ph increases its shelf life b.what is true about viscosity? a.in how many percentage of population malignant hyperthermia chances exist? a.presseure difference b.pseducholine esterase is deficient 87..prolactin c.cis atrecurium is preferred over atricurium why? a.which one of following is correct : a. po2 60 spO2 90% c.radius 84.distends intestine .2 days 83.more toxic than bupivacaine c.piperacurium is preferred over pancuronium because a.bupivacaine c.cardiovascular stability is more 81. po2 50 spO2 6 0 % 89.regarding why nitrous not used in abdominal surgeries correct statement is: a.in huntingtons chorea succinyl choline is contraindicated .why? a. a.flow of gas through small orifice affected by which parameter : a.3 hours c.12-16 weeks b.it decreases on increasing temperature 85.b.by decreasing SA and AV conduction 88.in pregnancy respiratory rate increases which hormone is responsible for that : a.reducing cardiac out put .it is a recemic mixture 80. 82. po2 90 spO2 1OO% b.half life of pseudocholinesterase is: a.renal excretion is less b.estrogen b.preganglionic sympathetic supply to sublingual gland is transmitted along which cranial nerve 92.orifice of a tube is small. capillaries 100.ventricular end diastolic volume c.diffusion b. March 15.glucose tranports across membrane due to its concentration difference.93.4.pCO 2 was 60.nerve to submandibular gland c.all true except (exact options don’t remember but I guess they were like this ) a. 2012 Help me to help you guys! .organ perfusion 96.one case scenerio was given in which ph was 7.active tranport d. suxa 103..svc c. Muscle relaxant that can be given to an asthmatic pt a.bicarbonate . active tranport 101.less conc .type 2 respiratory failure and compensated respiratory acidosis d.cell is in complete depolarization phase in : a.it is called: a.give rise to greater petrosal nerve b.chloride ion .maximum map is in a.pulmonary capillary wedge pressue b.rest options don’t remember 99.About CSF most appropriate statement is a) it is ultrafilterate of Plasma b) has more glucose than blood c) has cushioing effect d) absorbed in Choroid Plexus 98. ECG of a patient shows progressively increasing PR intervals followed by dropped beat. tubocurare c.end systolic volume d.ST segment d.ivc b.respiratory acidosis b.pulmonary artery d. crosses blood brain barrier d. produced by cis than atracurium c.T wave 102.type 1 respiratory failure and compensated respiratory acidosis c. cisatracurium d.facilitated diffusion c.sec.Regarding Laudanosine.QT interval c..All are the branches of Vagus nerve EXCEPT a) Auricular b) Lacrimal c)Pharyngeal d) Recurrent Laryngeal 97.regarding intracranial part of facial nerve what is true: a.nerve to tensor tympani Posted by Dr Sultan Ahmad 0 comments Reactions: Thursday.metabolic alkalosis 95.best indicator of preload is: a.metabolite of atra and cis curium b. What is the condition? a) Third degree heart block b) Mobitz Type 1 c) Sinus arrhythmia d) Mobitz Type 2 94. atracurium b.sodium in blood is electrically balanced by .and HCO3 was 26 a.cns depression 104.qrs complex b. simple reason being that i did not receive more recalled questions from you guys! I got on with my career in UK. I tried my best while I could and still promise to update the pool when ever i get anything from you. 2010 Medicine 25th May.Dear All. Posted by Dr Sultan Ahmad 6 comments Reactions: Friday. U r doing great job. July 23. BUTTERFLY ’’ Thanks a lot. I tried my level best to recall as many qs as I can to post it at your blog. DO CONFIRM THE ANSWERS 1 ECG changes can be seen in the following conditions EXCEPT .keep doing good work 25th may. Plz do write my following name when u post ds paper to Ur blog. 2010 Hi. Your blog is very much popular among part 1 candidates. Appologies for not updating the site for sometime. BUTTERFLY from FJMC. thanks a lot. 2010 PAPER 1 and 2. Happy preparation! Dua ki derkhast ke saath. name is ‘’ DR. I am Dr. Assalam o Allaikum. Sultan. I wana see my name at the end of ds post at Ur blog. so could not actively search for latest papers and questions. LAHORE. pk/books?id=-V3k2MnDQcC&pg=PA275&dq=cvp+is+decreased+during+gram+negative+sepsis&hl=en&ei=LSE .com/Q/What_is_sacralization_of_the_l-5 4 Stroke volume a) Cardiac output depends on it b) Heart rate determines stroke volume c) Increases in haemorrhage d) Independent of venous return e) … Ans.physiology.wikipedia. in haemorrhage b) Dec in gram negative septicemia c) Dec in heart failure d) … e) … ans. b http://books. DNT KNW EXACTLY 5 Central venous pressure a) Inc. C (http://en.org/cgi/content/full/292/3/R1320) d) Mitral stenosis e) Aortic regurgitation Ans: A: 2 About NEURAL TUBE a) Forms primitive streak b) Forms notochord c) Cns develops from it d) Forms three germ layers e) … Ans.answers.google.com.a) Change in body position b) MI c) Sleep (http://ajpregu.org/wiki/Neural_tube ) 3 SACRALIZATION a) Union of 1st sacral vertebra with 5th lumbar b) Union of 5th lumbar with 1st sacral c) Fusion of all sacral vertebra to form sacrum d) Flexion at sacrum e) … Ans: B http://wiki. D synthesis a) 1 hydroxylation ocurs in kidney b) 25 ocurs in kidney c) 1 ocurrs in liver d) 25 in lung e) Both in skin . B 11. Ans.cvphysiology.htm 7 A simple senario of hurshspring disease 8 Which hepatitis dangerous in pregnancy a) A b) B c) C d) D e) E Ans. septicemia.com/Cardiac%20Function/CF017. who’s been deliverd in a remote village. Most common cause of pulmonary embolism? a) Heart failure b) Dvt c) Cancer lung d) Pneumonia e) … ans. Regarding vit. e 9 A lady. bleeding from venae sites. presents with shock. A http://www.pTPu_NMaAlAeNmY2ACA&sa=X&oi=book_result&ct=result&resnum=9&ved=0CE0Q6AEw CA#v=onepage&q=cvp%20is%20decreased%20during%20gram%20negative%20sepsis&f=fals e 6 What change occurs from lying to standing position? a) Venous pressure inc b) Arterial pressure inc c) Sweating d) Cutaneous vasoconstriction e) ….whats diagnosis DIC 10. org/wiki/Vitamin_D 12. WHICH IS NOT SUPLIED BY OCULOMOTOR N. Treatment of erythroblastosis fetalis? a) Exchange transfusion with b +ve b) With b _ve c) Ab +ve d) Anti D e) … ans. b) Content of carotid triangle c) Pierces deep fascia behind sternocleidomastoid d) … e) … ans. which 1 true etc 18 Diagnosis of typhoid in 1st week Ans… blood culture . About external juglar vein a) Formed by retromandibular and ant. 13. e weakness and paresthesia along ulnar nerv course 17. A http://en.Ans.wikipedia. c 15. A lil senario of cervical rib i. Auricular v. Cephalic vein a) Present in deltopectoral groove 16. A qs about CSF findings. a) Medial rectus b) Lateral rectus c) Inf oblique d) Sup rectus e) Inf rectus 14. septicum e) … ans. C.19.com.google.wikipedia.google. ljungdahlii c) Tetanus toxin d) C. Most of bicarbonate absorption ocurs in a) Proxial c. Type of omental necrosis? a) Caseous b) Gangrene c) Fat d) Coagulative e) Fibrinoid Ans. C http://books. Most common cause of SUBACUTE BACTERIAL ENDOCARDITIS a) Staph aureus b) Sterp. A http://books.com. Cause of gas gangrene (C. b 20.pk/books?id=c41BJkHrniEC&pg=PA25&dq=omental+fat+necrosis&hl =en&ei=8iopTKykIaWlsQaB9JzEBA&sa=X&oi=book_result&ct=result&resnum=4&ved=0CD YQ6AEwAzgK#v=onepage&q=omental%20fat%20necrosis&f=false 22.org/wiki/Gas_gangrene 21. tube b) Distal c. perferringens ws not givn) a) C.botulinum b) C. tube c) Loop of henle d) Collecting duct e) ….pk/books?id=pekX0WUKzMoC&pg=PA414&dq=bicarbonate+reabsor ption&hl=en&ei=oCspTPu0CIO0lQftobD- . Viridans c) Strep pneumonae d) Staph epidermiditis e) … ans. Ans. d http://en. Dec aldosterone will lead to a) Hyponatremia 27. c 26. . Aldosterone causes a) Hypercalcemia b) Hyperkalemia b) Acidosis c) Hypernatremia d) … ans. ADH acts on a) Loop of henle b) Distal c. c 25.Bw&sa=X&oi=book_result&ct=result&resnum=1&ved=0CCYQ6AEwAA#v=onepage&q=bicar bonate%20reabsorption&f=false 23. For circardian rhythm optic nerve sends fibers to a) Optic chiasma b) Medial geniculate body c) Lateral geniculate body d) Suprachiasmatic n. There were 4_5 qs about aldosterone 24. tube c) Collecting duct d) Bowman capsule Ans c 28. e) Upraoptic n. Stimulus for aldosterone release a) Hypernatremia b) Hypokalemia c) Hyperkalemia d) Alkalosis e) … ans. which ion replacement is most important? a) Ca b) Mg c) Na d) K e) Cl Ans.Ans. What to give to Inc vitamen in diet a) Nuts b) Yougurt c) Egg d) Green vegetables e) Margarine (sumthin like it) Ans.wikipedia. Endogenous pigment in chronic hemolysis a) Hemosiderin b) Lipofuscin c) Anthracotic pigment d) … e) … Ans A 33. . C 30.org/wiki/Circadian_rhythm 29. D 32. A person suffering from dehydration. D http://en. D 31 Natural anticoagulant a) Heparin b) Warfarin c) Aspirin d) Plasminogen e) Fibrin Ans. Square root of variance a) Mean b) Variation c) Standard deviation d) Median e) Acuracy Ans C 35.berkeley.stat. Drug contraindicated in renal failure a) Amikacin b) Chloramphenicol c) Vancomycin d) Pencillin e) … ans. b http://www. all the children there have some permanent disabilitythe parents of such children are suffering from a) Denial b) Anger c) Bargaining d) Depression e) Acceptance 34. a 38. Drug for hyperthyroidism in pregnancy a) Methimazolepropylthyrouracil b) Ppu c) Radioactive iodine .In a ward. at a children hospital. Mean Inc with a) Inc in sample size b) Dec in sample size c) Inc in standard deviation d) … e) … ans. What is therapeutic index of a drug? 37.htm 36.edu/~stark/Java/Html/SampleDist. Difference BW hyperplasia and benign tumor a) Hyperplasia invades surrounding tissues b) Benign tumor metastasize to tissues c) Benign tumor shows aggressive growth d) Hyperplasia shows pleomorphisim e) Benign tumor is surrounded by capsule Ans E 41.d) Total thyrodectomy e) Ligol solution Ans B 39.e seizures and convulsions then asked that what other symptom can be found in that child and ans ws GLUCOMA . Premalignant condition of oral cavity a) Erythroplakia b) Leukoplakia c) SCC d) Lichen planus e) … ans b 43. A senario was about sturge webber syndrome in which he gave neurological symptoms I. Atrophy means a) Dec in cell size 42. Point to note that they did not talk about PORT WINE STAIN (birth mark) on face or forehead A . Which will not b present in a lung hamartoma (it ws senario of pulmonary hamartoma and qs ws asked at the end of senario) a) Coin lesion on xray chest b) Mostly asymptomatic c) Mostly Discovered incidently d) Benign e) Cartilage in lesion area Ans E 40. Bile acids are conjugated with which amino acid to form bile salts a) Taurine 51. After gastrectomy what can happen a) Malabsoprtion b) Steatorhea c) Anemia d) … e) … ans c 46.44. Submucosal glands are present in a) Stomach b) Gall bladder c) Duodenum d) Small intestine e) Pancreas Ans C 45. Gastroparesis treatment (last part1 me b tha) a) Metoclopramide 48. After gastrectomy B12 is not absorbed due to deficiency of a) Parietal cells b) Chief cells Ans A 47. Absorption of long chain fatty acids? a) Ileum 49. Bile salts are absorbed where? a) Terminal ileum 50. Which hormone Inc gastric contraction? A) GASTRIN . Stretch reflex maintains a) Muscle tone b) Balance c) Length d) … e) … ans c 57.T8 level and dy askd about wat will happen . There was qs in which options were (forgot qs) a) Pancinian corpucles b) Messiners corposccles c) Ruffinis end organs d) Golgi tendons e) …. choices were not straight forward e.g askin sum tract lesion etc dts y not remeberd 56. Final common motor pathway a) Corticospinal tract b) Alpha motor neurons c) Upper motor neurons d) Cerebral cortex e) … ans b 55.B) CCK C) Secretin c) Gip Ans A 52. There was a qs about injury at T4. Rupture of post wall of duodenal ulcer will damage a) Gastroduodenal artery 53 Left renal vein relation to aorta a) Ant b) Post Ans A 54. . Not a cause of cardiogenic shock a) Cardiac temponade b) Mi c) Arrythmias d) Hemorrhage .. e) . Glomerular pressure inc due to a) Afferent constriction b) Efferent constriction Ans b 62. ans a 60. During systole a) Both ventricles contract simultaneously b) 50% filling of ventricles occurs c) Blod flow inc in coronary arteries d) . Treatment of acute asthma a) Terbutaline 64. A senario was about dijoxin toxicity 63.58. Isoniazid side effects can be prevented by a) Pyridoxine 65. Regarding pulmonary wedge pressure a) Called so cz measured by a wedge shaped catheter b) Usually bw 10-15 mmhg c) Measure of rit atrial pressure d) … e) … an sb 59. Fastest conducting fibers in heart a) Purkinje 61.. Which tumor does not hav bone metastasis? a) Thyroid b) Liver c) Lung d) Breast e) Kidney Ans b 72.020 b) Neutrophils c) Protein >2g| dl d) . Most imp point of exudative inflamation a) Sp gravity < 1. AFB +ve a) Leprosy 67.e) Aortic dissection Ans d 66. A pt with deformed nose. Tuberculous meningitis a) Inc lymphocytes in csf 70. ESTROGEN OCPs can cause a) Breast ca b) Ovary ca . Bladder ca caused by a) Schistosoma haematobiam 71. e) … ans c 68. Monocytes a) Most abundant cells in circulation b) Do not go out of vessels c) Not phagocytic d) … e) … 69. Vital capacity a) After a max inspiration. (Remembr that sup glands hv more constant position while INF r variable in position. why a) Dnt remember choices.its imp) 77. Which is not covered by deep cervical fascia a) Thyroid b) Parotid c) Sternocleidomastoid d) Submandibular gland e) Sublingual Ans e 79. max expired volume is vc 75. Residual vol. Circumduction a) Combination of flexion. posteriorly by post. Belly 78. Cardiac output a) Measured by fick principle 76. extension. Belly of digastric. Digastric triangle a) Bounded anteriorly ant. sory B) C) D) . For surgery tibia should b aproached from medial side.that’s y dy askd about sup glands location. Regarding sup parathyroid gland a) Located in close proximity to INF thyroid artery and recurrent laryngeal nerve at the level of cricothyroid junction. aduction and abduction 80. a) Amount of air left in the lung after a max exhalation 74.c) Endometrial ca Ans a 73. lymph. Jaundice. then chest pain that relieved by sitting up a) Pericarditis 85. Pt having uppr respiratory tract INF. blood suply.81. hepatitis. Dnt remember exactly 88. . WHAT IS TRUE ABOUT UPPER HALF OF ANAL CANAL A) Options wr about epithelium. Micturation reflex centre is located in a) Brainstem (pontine micturation centre). diarrhea 10days after bone marrow transplant a) Graft vs host disease 89 Erythroblastosis fetalis. In nephron fluid which substance conc is higher than plasma a) Na b) Bicarbonate c) Glucose d) Urea e) Albumin Ans d 86. There s also a sacral micturation centre but it ws not in the choices 82. Renal plasma flow is determined by a) PAH 87. nerve sup. Approach to intercostal space for pleural effusion drainage a) Lower part of space (neurovascular bundle in upper part os SPACE but lower part of RIB) 84. type of hypersensitivity reaction? a) 2 90. Diagnosis of pneumothorax? a) Xray chest 83. thanks ALLAH Dy did NT gv both choices simultaneously :-) 93. Cauda equina a) Collection of nerve roots and rootlets 94 Pilocarpine is nicotinic a) Agoist b) Antagonist 95 Diagnosis of klinefeltrs syndrome a) Barr body 96 Which drug interers with warfarrin a) Cemetidine 97 Adverse effect of chlorpromazine a) Extrapyrimidal . Tissue for HLA typing (there were two same qs in each paper) a) Buccal mucosa b) Leukocytes c) Skin d) Bone marrow Ans b 92. LP done at a) L4-L5 (l3-l4 also correct but it ws not in choices.WHICH VACCINE IS NOT LIVE A) Measles B) Mumps C) Rubella D) Yellow fevr E) Tetanus Ans e 91. which test to perform a) Hep A nd B VIRUS b) Bilirubin .98 Mechanism of action of captopril a) Ace inhibitor 99 Feature of ca a) Plomorphisim b) Nuclear ratio c) Hyperplasia d) Invasion Ans d 100 Most cancers hav which fillament in them a) Kerratin b) Desmin c) Vimentin d) Neurofibrilary e) … ans a 101 Paraneoplastic syndrome is associated with a) Ca breast b) Adeno ca lung c) Small cell ca lung d) Ca prostate Ans C 102 Hepatitis b INF is monitored by a) Surface antigen b) Srface antibody c) Core antibody d) E antibody 103 Hepatitis A inf. nt exactly remembrd) a) They hav igA. G 20gm b) IgM: igG S 1:20 109 HLA type in RA a) HLA DR4 110 ABSOLOUTE LYMPHOCYTOSIS NOT PRESENT IN a) Sle b) Inf. G in different ratios and %ages e. Mononeucleosis c) Walders (sumthin like that) granulomatosis d) … e) … .c) Sgpt d) Albumin Ans c 104 Which hormone inhibits insulin secretion? a) Secretin b) Cck c) Somatostatin d) Glucagan e) Vip Ans c (As a general rule all git hormone stimulate insulin release and knwn as INCRETIN) 105 MOST IMP STRESS HORMONE (it was also in last part1) a) Epinephrine b) Insulin c) Growth d) Cortisol e) Acth Ans d 106 In Cushing syndrome a) Neutrophils are Inc. M 15gm. M. g IgA 5gm. lymphocytes and eiosiniphils are Dec 107 A simple senario of graves disease 108 Whats pattern of antibodies in multiple myeloma (options were vry weird. photosensitivity (these symptoms wr in the form of a simple senario).ans a (nt sure) 111 Pt with arithritis.google. WATS diagnosis a) SLE 112 Platelets contraindicated in a) Splenomegaly b) Acute immune thrombocytopenia c) Coagulopathy d) Dic Ans b 113 Water moves through a) Pores b) Membrane matrix c) Protein channels d) Fascilitated difusion Ans A (water channels) 114 Fascilitated diffusion a) Passive transport trough protein channels or carier protiens 115 Which factor strenthen pt doc relationship a) High professional skills b) High social skills c) Highly qualified d) Logical answers to qs e) Active listening Ans E http://docs.racgp.pdf+active+listening+for+a+doctor &hl=en&gl=pk&pid=bl&srcid=ADGEEShL3aencNeNL6PcDL75akSth1myGNOsIBbWqXMOBxP5X7QKVmiu16qKkAOUhwMcNerrFuT2x-kVNZZz84Xr5Gy1K7AJJfrUhJoOWFb7PnEI52oZ3VXT9v3ZAKmA31w_EhEa0&sig=AHIEtbTDPrtyezt44aYcvJMnrIDzau8oZw .com/viewer?a=v&q=cache:B90RsYl_QIJ:www. butterfly rash.au/afp/200512/200512robinson.org. 116 Which is more aggressive (also in last part1) a) Basal cell ca b) Squmaous cell ca c) Melanoma Ans b 117 Nissel bodies are a) RER b) SER c) Mitichondria d) Golgi bodies Ans a 118 Greatest area of cerebral cortex is covered by a) Elbow b) Knee c) Thumb d) Shoulder e) Ankle Ans C 119 Deviation of tongue on protrusion. diagnosis . is damaged a) Fascial b) Hypoglossal c) Submandibular d) Edenger westphal e) … ans b 120 A pt with diastolic murmur in aortic area and colapsing pulse. which N. a) Aortic stenosis b) Mitral stenosis c) Aortic R. Division of MA Ans A 122 Csf absorbed by a) Arachnoid villi b) Choroid plexuses Ans A 123 Which is pierced during LP A) Dura matter 124 Inf spreads retroperitonealy. which will b infected a) Spleen b) Jejunum c) Transverse colon d) Descending colon e) … ans d 125 Ant pituitary loss will result in Dec in size of . d) Mitral R e) … ans c 121 SUB-DURAL HEMATOMA CAUSE A) SUP CEREBRAL VEIN B) INF CEREBRAL VEIN C) ANT DIVISION OF MA D) Post. pk/books?id=21e4wArL7hQC&pg=PA18&dq=portal+system+draining +esophagus+is&hl=en&ei=N7w0TNLEDKjonQefitGDBA&sa=X&oi=book_result&ct=result&r esnum=1&ved=0CCkQ6AEwAA#v=onepage&q&f=false 130 Bronchopulmonary segment a) Anatomical and functional unit iof lung .com. light reflex absent.google. Damage to? a) Edenger westphal n. b) Optic nerve c) Optic tract d) Optic chiasma e) Pretectal area Ans E 129 Part of portal system draining esophagus a) Azygous v.a) Zona glomerulosa b) Zona fasiculata c) Parafolicular cells of thyroid d) Adrenal medulla e) … ans b 126 Factor Dec wound healing a) Vit c deficiency 127 How thrombocytes play role in thrombosis a) Sorry forgotn the choices 128 Accomoation reflex intact. b) Hemiazygous c) Rit gastric d) Lft gastric Ans D http://books. 131 Temp set point is in a) Ant hypothalamus b) Post hypothalamus 132 Antibodies are produced by a) Plasma cells 133 Virulence of bacteria is associated with a) Dose b) Duration of exposure c) Toxin production d) Body resistence e) … ans c 134 For a blood donor what is not required (choices not well remembrd.actualy dy askd that which factor should not b there in a GENERAL HEALTHY DONOR) a) Age b) Sex c) Systemic disease 135 Which one is not a epithelium tumor a) Adenocarcinoma b) Sq ca c) Liposarcoma Ans C 136 Which receptor r involvd in acid production a) H1 b) H2 c) Acetylcholine d) … e) … ans b . now his urine is concentrated.137 Which is present in slow wave sleep a) Dopa b) Acetylcholine c) Serotonin d) Norepinephrine e) … ans c and d 138 Hemibalismus due to damage to a) Subthalamic N. It’s due to a) Aldosterone b) Adh c) Acth d) Cortisol Ans B . 139 A child having dyspnea when lyng down a) Retrosternal goiter 140 Best buffer of body a) Protein b) Hb c) Hco3 d) Phosphate Ans C 141 Protein utilization is chekd by a) Urinary NO2 b) Blood NO2 Ans A 142 A pt has fluid loss. 4 po2 65 pco2 33 hco3 19 a) Metabolic acidosis b) Metabolic alkalosis c) Compensated resp alkalosis d) Resp alkalosis 144 What happens on mountain aclimitization? a) Ventilation inc 145 Syphil dignosis material taken from a) Blood b) Urine c) Semen d) Genital sores (lesion) e) Saliva Ans D 146 Best indicator of serum iron stores a) Serum iron b) Serum ferritin c) Hemosiderin d) TIBC Ans B 147 Why more oxygen goes to alveoli at apex than at base? a) Inc compliance b) Inc blood flow c) In v\q d) Inc pulmonary pressure Ans A 148 .143 Ph 7. 3.4 b) S2.about its inc or dec in specific condition . 120 d) 120….Urinary incontinence with overflow and excessive voiding. c) Lat. To biceps d) Ends in axilla Ans C 151 Upper part of anal canal is a) Lined by stratified sq epi b) Drained by sup inguinal nodes c) Drained by inf rectal vein d) Sensitive to touch e) Sensitive to pain 152 Normal fetal heart rate a) 40…. 100 c) 100…. damage to (qs nt remembrd well) a) S2. To radial a. 4. 3 c) L2.s1 149 Hypospedias is due to defect in a) Urogenital tubercle b) Urogenital fold c) Urachus Ans B 150 Cephalic vein a) Begins in anatomical snuff box b) Lies lat.200 Ans D 153 One qs was about CVP. 60 b) 60…. 160 e) 160…. its 1st visit to a doc. cause a) Cerebellar lesion b) Upper motor neuron c) Lower motor neuron .no complication.154 Inc in systemic filling preesure causes a) Inc venous return 155 Beta-blockers do not cause a) Inc renin b) Vasoconstriction c) Bronchiodilation d) … e) … ans a 156 Lidocaine a) Inc PR interval b) Dec AP c) Dec automaticity Ans A 157 A diabetic pt with BS 20mmole\l . What shold be the treatment? a) Insulin b) Insulin + sulphonylurea c) Sulphonylurea d) Biguanides e) Biguanides +sulphonylurea Ans D 158 Pendulous knee jerk. obese. influnzae meningitis a) Csf culture b) Blood culture c) Inc lymphocyte in csf d) … e) … an a 161 Antibody to TB bacteria is (dnt remember exact choices but they did ask regarding antibody against TB) a) Cell bound b) Membrane bound c) in plasma d) In saliva e) … 162 In which phase os cell dividion the chromosomes are arranged in chromatids a) Prophase b) Metaphase c) Anaphase d) Telophase 163 Atrial contraction coresponds to a) C wave b) P wave c) Qrs . 3.4 Ans A 159 In which parasite the respiratory symptoms predominate? a) Ascaris b) Cystecercosis c) … d) … e) … 160 Diagnosis of H.d) S2. 1.5L blood loss. skin is calm and cold due to a) Vasoconstriction b) Vasodilation 170 Most common manifestation of septic pt a) Tachycardia b) Hypotension c) Inc GFR Ans B .164 Dorsal roots supply which musscles? a) Internal intercostal b) Externat intercostal c) Innermost intercostal d) Some back musscle 165 Internal spermatic fascia is derived from a) External oblique b) Internal oblique c) Transvesalis fascia d) Transversus abdominus Ans C 166 Inputs to cerebellum a) From vestibular system 167 Resolving power of lens a) That enables to see closely related subjetcs seperatly 168 Parasympathetic stimulation causes a) Ciliary muscle contraction 169 RTA. 171 Premalignant condition a) Basal cell nevus b) Dysplastic nevus syndrome Ans B 172 Tumor involving lymph vessels a) Angiosarcoma b) Cystic hygroma Ans B 173 A female having very high levels of prolactin. Ans D 174 What u find in primary hyperaldosteronism a) Inc k b) Dec k c) Hypotension d) Inc renin e) Inc angiotensin 2 Ans B 175 Which take part in synthesis of aldosterone a) Angiotensin 1 . It’s due to a) Compression of optic nerve b) Damage to optic tract c) Damage to optic radiation d) Compression of upper part of optic chiasma e) Compression of oculomotor n. suffering from homonymous hemianopia. b) Angiotensin 2 c) Cortisol d) … e) … 176 Inc k causes a) Inc aldosterone b) Inc ADH c) Dec aldosterone d) Dec ADH 177 HYDROCHLORTHIAZIDE CAUSES A) Inc k B) Inc ca C) Inc mg D) Inc na 178 Hydrops fetalis occurs to RH –ve mother and RH +ve father. What type of hypersensitivity reaction occurred? a) 1 b) 2 c) 3 d) 4 e) 5 179 Blood supply of eye except cones and rods a) Central artery of retina 180 They gv senario of klinefeltr and askd what u will find most common a) Gynecomastia 181 Aldosterone causes a) Hyperkalemia b) Hyponatremia c) Hypercalcemia d) Hypokalemia e) Acidosis . Captopril causes hypokalemia hyper kalemia 6.Wishing all the part1 candidates GUDLUCK DR.Corneal opacities r caused by ethambutol phenothiazides 5. which is pivot joint temporomandicular atlanto occpital Atlanto axial 3. September 14. 2009 MEDICINE JUNE 2009 1. BUTTERFLY FJMC. H+ ion secretion in the kidney causes bicarbonate reabsobtion bicarbonate excretion 2. Which is P450 inducer isoniazid phenobarbitone cometidine . Drug used for mountain sickness acetazolamide scopolamine 4. Posted by Dr Sultan Ahmad 21 comments Reactions: Monday.5 HCO3= 48 PCO2= 45 diagnosis is metabolic alkalosis respiratory alkalosis partially compensated metabolic alkalosis partially compensated respiratory alkalosis 7. Ph =7.LAHORE. Serum gastrin levels are incresed by prolonged use of antacids PPI H2 blockers anticholinergics beta blockers 10. Which causes decreased gastric motility Magnesium Sulphate Aluminium Hydroxide 9. Carotid body receptors respond to arterial PO2 arterail Pco2 13. propylthyouracil acts by decreasing iodine uptake decreasing TSH levels decreasing thyro globulin levels 14. which crosses placenta Ig G Ig M Ig A Ig E 16.ketoconazole 8. Patient has acute blood loss the mechanism triggered imediately is baro receptor mechanism renin angiotensin system 12.Esophago gastric junction competence is maintained by lying supine incresed intra abdominal pressure diaphragm paralysis use of metoclopramadie use of morphine 11. which is secreted in breast milk Ig G . which is increased in first responce Ig G Ig M Ig A 15. which has memory function Ig G Ig M Ig A Ig E 18. thymus is formed by . which drug causes abnormal differentiation between red and green colours ethambutol amiodarone pyrazinamide 19. fracture of surgical neck of humerus.Ig A Ig M 17. damage to axillary nerve musculo cutaneous nerve 24. for best verbal communication skill one should hav good vacabulary competence in presentation fluent speech 20. ejaculation is caused by sympathetic parasypathetic both 22. action of gluteus medius n minimus is abduction n medial rotation abduction n lateral rotation adduction n medial rotation adduction n lateral rotation 25. Alpha receptors stimulation causes pupilaary dilation increased heart rate increased contractility 23. patinet cant raise arm and sensory loss on lateral surface of arm. which is not caused by parasympathelic stimulation increased GIT motility micturation miosis sweating 21. bromocriptine causes decresed prolaction levels by acting on dopamine receptors cholinergic receptors 27. drug that must b used is lidocaine amiodarone verapamil 30. Digoxin is drug of choice in Atrial flutter Atrial Fibrillation Ventricular Tachycardia Bradycardia 29. which investigation to be done in patient with hemophilia PT APTT bleeding time complete blood count . apex beat is 4th left intercostal space mid clavicular line 8cm to the left of midline in 5th intercostal space 31.3rd pharangeal pouch 4th pharangeal pouch 26. he has idiopathic thrombocytopenic purpura aplastic anemia Hemophilia 32. patient after ischemic attack has ventricular tachycardia. Hemophilia is X linked recessive Autosomal dominant Autosomal recessive 33. young patient has bruises on the body and no other significant history n examination unremarkable. patients ecg shows increasing PR intervals and then a missed beat first degree block sinus arrythmia Mobitz type 1 block Mobitz type 2 block 3rd degree block 28. diagnosis is good pasture syndrome wegeners granulomatosis 35. VII. patient has homonymous hemianopia. damage to middle meningeal artery causes hematoma formation between duramater n calvaria duramater n arachnoid mater arachnoid mater n piamater 50. X 52. patient has hemoptysis and glomerulonephritis. C perferingens acts by producine lecithinase depleting ATP causing hypoxia 37. lesion is at optic nerve . which is not oncogenic Hep B virus Hep D virus Herpes Simplex virus EBV 39. Middle meingeal artery passes through foraman spinosum foraman lacerum foraman rotundum foraman ovale 40. P falciparum causes black water fever 36.34. which cranial nerves r parasympathetic III. patients right eye has moved upward n has his neck tilted to left to avoid diplopia. IX. damage to superior oblique inferiour oblique 53. Gonococcus in easily identified in exudate specimen by gram staining Z N staining culture 38. which is present in cavernus sinus abducent nerve 51. optic tract retina optic chiasma 54. foreign body in trachea goes in which bronchus right upper right middle right inferior left inferior 56. which lobe has only 2 brocho pulmonary segments right middle right upper left upper left middle 57. in kidney select the best order of arteries renal > arcuate>interlobar>interlobular>efferent arteriole renal>interlobular>interlobar>arcuate>efferent arteriole renal>interlobar>interlobular>arcuate>afferent arteriole . in muscle cells calcium is released from endoplasmic reticulum sarco plasmic reticulum 61. which is thick filament actin myosin 59. germ cells r derived from ectoderm endoderm mesoderm 55. a sacromere is present between 2 Z lines H band A band 60. during iso volumetric contraction atrial pressure decreases semilunar valves r open coronary blood flow decreses first heart sound is produced second heart sound is produced 58. renal>arcuate>interlobular>interlobar>afferent arteriole . after Rh incompatabilty what is given to the mother to prevent it next time anti D antibodies 69. after renal transplant which malignancy is common ??? 64 babinski sign is positive in pyramidal lesion cerebellar lesion 65.62. ventral spino thalamic tract ends at lumbar region mid thoracic end thoracic 66. patient has blood group A+ which cannot b given to him O+ O-A-AB-70. hamartoma is benign tumor malignat tumor 71.after normal saline infusion blood volume increases Na excretion decreases osmolality increases renin secretion increases 63. in Addisons disease what does not occur . which is not punctured during lumbar puncture duramater ligamentum flavum longitudinal spinal ligament 68. commonest site of lumbar puncture is L4-L5 L2-L3 S1S2 67. medullary rays r present is cortex medulla 72. HLA.DR4 is associated with rheumatoid arthritis SLE scleroderma 77. which is essential amino acid phenyal alanine 74.hyper kalemia hypokalemia 73. which is associated with cancer methyl alcohol propyl alcohol 76. which causes dysphagia ankylosing spondylitis scleroderma dermatomyositis 79. and Hb 8.1 g/dl. after 3 months she presents with iron deficiciency anemia pernicious anemia hemolytic anemia 81. which is present in SLE anti DNA antibodies anti jones antibodies anti smith antibodies 78. patients had gastrectomy . she is suffering from iron deficiency anemia megaloblastic anemia 82. which is teratogenic alcohol 75. which is tributary of portal vein superior rectal inferior rectal . iron deficincy anemia is hypochromic microcytic 80. pregnant lady has MCV of 70 fl. which does not cause thenar muscle wasting carpel tunnel syndrome C8 neuritis cervical rib scalene muscle spasm .83. how will the effect of warfarin immediatelty reversed vitamin K FFP protamine sulphate 86. he has damage to right fascial nerve left fascial nerve right trigeminal nerve left trigeminal nerve 91. selective beta blockers do not cause broncho spasm 85. tunica vaginalis is derived from transversalis muscle transversalis fascia peritoneum inter oblique muscle 88. which muscles divied submandicular gland in to super ficial n deep part omohyoid mylohyoid anterior belly of digastric 90. RCA supplies right atrium only right atrium n right ventricle 87. what arches in front of the root of left lung Arch of aorta Azygus vein 84. turner syndrome has karyotype 45 XO 45 XX 45 XXy 45 XXX 89. patient has difficulty closin right eye and distorted facial appearance. best test for diabetic nephropathy is serum urea levels serum creatinine levels urinary albumin 101. GFR can b best estimated by creatinine clearance serum urea levels serum creatinine levels 100. which protein maintains red cell shape integrin . beetle chewing causes submucuous fibrosis leukoplakia keratosis 95. fetal scan at term shows no anomaly. a pregnat lady from hilly area presents to. Dose of Gentamicin is reduced in elderly due to reduced renal function 99. which is last mediator in septic shock IL-1 IL. Clavulanic acid destroys B lactamases causes decreased penicillin excretion 98. which of the following can still b present in the newborn ASD VSD PDA pulmonary stenosis 93.6 96.cervical spondylosis 92. antigen antibody reactions r most reduced in Liver failure Low neutrophils 97. what is end product of glucose metabolism in the presence of oxygen lactic acid 1 molecule of pyruvate 2 molecule of pyruvate 94. patient has gall stones . site for venesection great sephanous short sephanous 111. which is not part of limbic system vermis of cerebellum amygdala anterior thalamus hippo campus . diffusion through alveoli is directly proportion to thickness of membrane total cross sectional area 106. which does not has lymphoid follicles speen thymus lymphnodes payer patches 108. which enzyme could b most likely raised Alkaline Phosphatase LDH ALT 105. in congestive cardiac failure there is incresed pressure in venous system 109. Cyanosis is due to increased deoxy hemoglobin in blood decresed PO2 in blood. 103 which of the following causes natriuresis stretch receptors in atria carotid bodies 104.secretin spectrin 102. ventricular contraction causes which wave in JVP a cv 110. transpot of materials by carrier proteins is example of simple diffusion facilitated diffusion active transport 107. 010 122. psychological conditions 115. exudate has protein more than 3g/dl specific gravity is less than 1. in asthma FEV1/FEV less than 65% FVC is reduced 120 6 years old girl has meningitis. lymph node biopsy shows non caseating granuloma and sputum AFB is positive. behaviour sciences is branch of science which deals with behaviour of persons in different social . cerebral cortex is concerned with voluntary movements of body 117. lacrimal duct opens in inferior meatus middle meatus 119. most common organis is streptocossus pneomonie Neiserria meningitidis Hemophilus Influenzae E Coli 121. right gastric artery is branch of gastro duadenal right gastro epiploic 113. diagnosis is TB leprosy sarcoidosis syphilus . lesion is right hemisection of spinal cord left hemisection of spinal cord 116. DVT is causes by stasis of blood 118.112. pateint has loss of proprioception on right n loss of pain n temperature on left. a young boy has distorted nose and cervical lymphadenopathy. thiazide diuretics donot require potassium supplemets 114. which hormone effects carbohydrate. which does not follow oro fecal route tenia solium trichuris trichuria entrobius vermicularis Ankylostroma duodenale 125. protein and fat metabolism insulin Growth Hormone 132. which is not an epithelial tumor sq cell CA adeno carcinoma sarcoma . gastric Acid increases most after taking carbohydrate protein fats 131. diagnosis bacterial parotitis mumps infectious mononuclosus 127. which is false about spores produced under nutrient rich conditions B Anthracis producs spores C tetani produes spores they r killed under 121 degree for 15 min in autoclave 129.123. which is false causes disease only in humans favors aerobic conditions 128.about Mycobacterium TB . young boy has bilaterla parotid enlargement. Lambda chain is present in amyloidosis 124. cholangio carcinoma is caused by C Sinensis paragonimus watermani taenia solium 126. fetal brain development is caused by Growth hormone thyroid hormone 130. which hormone effects both nor epinephrine n serotonin metabolism Mono amine oxidase 135. Neonatal Rubella infection is screened by measuring which antibody? IgM IgG IgM and IgG also 6 -7 questions about aldosterone and blood pressure control Posted by Dr Sultan Ahmad 5 comments Reactions: GYNAE OBS JUNE 2009 . Bence jones protein are produced in multipla myeloma 134.133. Cerebellar lesion causes Ataxia 139. patient with cerebellar lesion cannot touch objects with his finger tip 140. PTH remains normal in renal insufficiency osteoporosis 141. patient presents with epistaxis and bruising on body. best invertigation is complete blood count PT Aptt 136. which is most slow growing malignancy of thyroid Follicular CA Medullary Papillary Anaplastic 138. ADH acts on Proximal tubule Distal Tubule Collecting Tubule 137. remembr) a-45XO ...... a-palatine shelves bmaxillary prominence cmed nasal prominence dlat nasal prominence eincisive foramen 3-germ cells derived from. 7-pentamer immunoglobulin is??? a-IgA BIgG cIgM* dIgD . d-thick mucosa sec frm cx wch nt show fern pattern.... b-rise in basal body temp c-middle of menstual cycle....diagnosis of failure of ovulation by ?? a-wid lower abdominal pain..(smethng like dat exactly nt 5-which 1 contain granulosa cell. Recalled By Syeda. a-ectoderm bendoderm cmesoderm d-neural crest cell 4-karyotype of turner syndrom.. 1-Etiology of cleft palate... 6-in primary infertility.Gyn Obs JUNE 2009... a-graffian follicle b.. a-anticonvulsant bmultiple preg cmultifactorial d-inc maternal age 2-left cleft palat dt malformation of.......... ........ .. 10-mode of cell injury by clostridium ...... 9-cornybacterium diphtheria. a-it is a nematode b-it causes liver disease c-occurs in both trophozoite nd cyst stage d-it exist normally in intestine e-...........e-IgE 8-Regarding entameba histplytica......... . endotoxemia neurotoxin verotoxin enterotoxin exotoxin 12-corneal opacity caused by..... cimitidine ketokonazole phenobarbitone . ethambutol quinine phenothiazine chloroquine ....?? ? a-ATP depletion... 11-shigella...... 13-drug inducer by p450 mechanism......... ..... a-gm -ve bacilli b-club shaped* c-................... 14-last mediator of endotoxic shock...... b.... .......... 16-wall of inguinal canal Not formed by??????? ext oblique apponeurosis lacunar lig internal oblique conjoint tendon transversalis fascia 17-endocervical polyp in pregnancy................. ..sampling is taken a/c to age …..… 18-vaccination of mycobacterium T.. endemic sporadiac pandemic epidemic .......B based on Type I hypersensitivity Type II Type III TYPE IV Cell mediated 19-after subdivided no of peoples into gps ..... fetal heart sounds can detected Cresenteric line seen w/h cnfirm pregnancy.wat type of sampling it is Random sampling (other options nt remember..IL1 IL6 TNF ALPHA LIPOPOLYSACHARRIDE 15-yellow fever inf occurs .(smething like dat ) 21-MRI Frequencies. 4-8 weeks transabdominal u/s. .............stratified etc .......nt sure) 20-ultrasound.... Physiological ......... No....fats ....and protein storage Insulin 28-fetal brain development Growth hormone Thyroid hormone 29-w/h of the following amino acid should be taken in diet Alanine Tyrosine Glycine’glutamine 30-folllowing phagocytic cell present in circulation? Monocyte Macrophage Lymphocyte Neutrophils 31-morphine can be given in Biliary cirrhosis . (these sort of alien words were written) 22-bone scan .........nd methaemoglobin..... (name of sme disesases were written) 23-incidence... of new cases in a population in a time given 24-pie chart 25-wch 1 is absent in muscles Glucose 6 po4ase Hexokinase 26-end product of glucose metabolism Lactic acid 1molecule of pyruvic acid 2 molecule of pyruvic acid 27-hormone ass with glucose .. Terminal ill patient of cancer Acute pancreatitis 32-functional layer of endometrium consists of Stratum basal Stratum compactum nd stratum spongiosum Stratum basal nd compactum 33-maldevelopment of uro-rectal septum . Rectovag fistula Rectouterine fistula Imperforate anus Rectoperineal fistula 34-structure lying b/w labium majus nd minus Urethral gland Bulbourethral gland Minor urethral gland 35-w/c 1 of following muscle not attached to tendon of perineal body Ischiocavernosus Ext urethral sphincter Bulbospongiosus Transverse perineal muscle 36-lymphoide nodule nt present in Thymus Spleen Lymphnode 37-Notochord forms Neural tube Cns 38-epithelium of ovary Simple cuboidal 39-pivot jt include .. Synovial........tubular glands in lamina propria Vagina Esophagus Oropharynx Duodenum 43-post relation of left ureter Root of mesentry Iliac vessel 44-outer lat quadrant of mamry gland drain into Ant gp axillary lymph node 45-xerophthalmia dt Vit A def 46-anti-oxidant Vit E 47-w/h st having 3 layers of muscles Fundus of stomach Esophagus 48-fat embolism 49-regarding mammary tissues w/h is Not appropriate Pect major.. nd elasticity. 50-lady died dt amniotic fluid embolism ....... seratus ant. vascularity.................. 42-an organ lined by stratified sq epi with inc cellularity.......... Puckering of skin dt subcut ductal infiltration.......... 41-hamartoma is..Atlanto occipital jt Atlanto axial jt 40-sacroiliac jt Fibrous ..lie in deep relations Can mov freely over retromammary space Peau’d orange caused by lymphatic obstruction... ............ ...HCO3=INC Metabolic alkalosis Metabolic acidosis Compensatory metabolic alkalosis 54-uterovag prolapse w/h lig is injured Broad lig Round lig Transverse cx lig 55-alkaline po4ase rises in Prostatic ca BPH Pancreatic ca 56-in ca mammary gland w/h drug is given in post menopausal lady Tamoxifen Clomiphen . 57-in left shift O2 hbdissociation curve w/h 1 is increase Fetal hb 58-in thalassemia there is ..delivery nd postpartum 51-pacinians corpuscles Tactile touch Fine touch Temp Pain Vibration(I think it was nt in option nt sure) 52-H+ exchange in proximal tubules with HCO3 reabsorption K secretion 53-pat wid severly vomiting came wid breathlessness..5..During delivery During c/s nd post partum After post partum During labour..PCO2=55 ...........PH=7..... .... ........................ 60-lady wid reddish cheeks...........……. SLE 61-in metaplasia w/h is inappropriate Irreversible .....R >100 HR>... 59-intravascular haemolysis diagnosed by Reticulocytosis Inc in hepatoglobulin binding Inc TIBC ...Poor hemoglobinization Dyserythropoiesis ... 62-acute inflammation Transudate Exudate Inc protein content 63-w/h drug causing minimal GI side effects Paracetamol Aspirin Ibuprofen Ketorolac Diclofenac 64-in 6 yrs old child meningitis occurs dt w/h organism H-Infuenza Ecoli Niesseria 65-sepsis is diagnosed most appropriately by Blood culture Granulocyte <2000--->15000 H..arthritis....…........ .. ................................. .... 67-virus not associated with ca in human HSV HBV HDV HPV Papova virus(papilloma vitus +polyoma virus) 68-w/h is more in human milk as compare to cow’s Lactulose 69-gonococcus is best easily diagnosed by Gram staining Culture ZN stain 70-w/h is completely covered by pretracheal fascia Thyroid gland 71-is not an embryonic remanant Lat umbilical ligament 72-oxytocin nd vasopressin originate frm Hypothalamus Neyrohypophysin Autonomic ganglion 73-sucrose Is a diasaccride Metabolized into 2 glucose molecules Reducing or non reducing (nt remember) 74-w/h of the following dis not ass with carcinoma Lysosomal storage dis .............66-w/h is transmitted by mosquito Yellow fever ..... Ataxia Rigidity Initiate muscle movement 84-pre-motor cortex lie in Frontal lobe .75-rough endoplasmic reticulum Protein synthesis 76-fragility of rbcs membrane Spectrin 77-behavioural sciences ... ...alpha blockers 81-centrally acting HTNsive Clonidine Methyldopa ......................................................definition 78-verbal communication skill Speak fluently Education nd research training Presentation Good vocabulary 79-Not a function of beta blockers -ve ionotropic -ve chronotropic ....... 80...... 83-cerebellar lesion. 82-w/h 1 decrease the muscle length Golgi tendon organ Muscle spindle ..... ..... 93-pregnancy is diagnosed by Gonadotrophin hormone . Hormones.......85-coccygeal part of spinal cord lie at the level of w/h vertebra C2 L2 S1 S2 86-lie in true pelvis nd crosses frm med to lat side of pelvis Sup rectal art Middle rectal art Inf rect art Internal iliac 87-inf fascia of urogenital diaghphram also called as Perineal membrane 88-female pelvis also called as Android Anthropoid Platypelloid Mesatipellic 89-at crowning w/h muscle is cut during episotomy Bulbospongiosis 90-lumbar puncture L2 L3 L3 L4 L4 L5 S1 S2 91-Lactation...wh stimulate 92-prolactin hormone Stimulated by...... Regarding kidney . 97-teratogenic Alcohol Caffeiene 98-carcinogenic Methyl alcohol Benzidine 99-pat with blood gp A should nt given A+ A_ O AB 100-.94-urogenital diagphram inserted on Ishchial tuberosity Ischial spine Sub pubic arch Ischial ramus 95-w/h muscle div submandibular gland into superficial nd deep part Mylohyoid Ant belly of diagastric 96-root of left lung Azygous vein arched at root of left lung.w/h sequence is correct renal art > arcuate>interlobar>interlobular>glomerular>efferent arteriole renal>interlobular>interlobar>arcuate>efferent arteriole>glomerular renal>glomerular>interlobar>interlobular>arcuate>afferent arteriole renal>arcuate>interlobular>glomerular>interlobar>afferent arteriole 101-in muscle ca+2 release frm sarcoplasmic reticulum T-tubule Sarcomere 102-sarcomere b/w 2 z line . Gland of cloquet is located in: a.. 107-norepi + serotonin degraded by MAO 108-at last trimester w/c uterus is sensitive to w/c hormone Oxytocin 109-to prevent heat loss Shivering Vasoconstriction 110-inc K Aldosterone sec Posted by Dr Sultan Ahmad 4 comments Reactions: SURGERY June 2009 SURGERY JUNE 2009.103-parasympathetic nerves r 3.... 8 weeks... Biparital diameter. (C) Means correct. 1.. wks Crown rump length .... 7. Thankyou. inguinal canal .... 10 104-w/c is present in de cavity of cavernous sinus Abducent nerv Trochlear Occulomotor 105-in haemophilia PT INC APTT INC Inc bleeding time 106-gestational age estimated frm.. Recalled By Safwan.. 9... Abd circumference... Squamous cell carcinoma of urinary bladder is caused due to: a. inferior vena cava b. Superior mesenteric artery (C) b. propyl alcohol c. The species which causes carcinoma in urinary bladder is: a. Below is a carcinogen: a. the head is in line of medial condyle b. left renal vein (C) c. phrenic vein d. the angle between the neck and shaft is average 125 degrees. shistosoma mansoni b. shistosoma (C) 9. Subclavian artery passes: a. portal vein 8. schitosoma hematobium (C) 10. Abdominal angina is caused due to: a. Following is correct about femur: a. Oxidative breakdown of glucose during glycolysis generates: . Left Suprarenal vein drains into: a. below the 1st rib 6. benzidine (C) 3.2. (C) 5. above the 1st rib (C) b. number of new cases in a population (C) 4. inferior mesenteric artery c. Incidence is: a. methyl alcohol b. abdominal aorta 7. celiac trunk d. Anterior cruciate ligament b. Fracture at neck of fibula: a. L3 d. ?? e. tibial nerve 16. 1 pyruvate b. Seminal vesicle d. deep perneal nerve d. Spinal cord ends at: a. C1. iliacus fascia (C) 15. S2 b. internal oblique fascia c. lactic acid 11. Referred pain to shoulder is via: a. Prostate c. Following is not palpable on per rectal examination: a. C5. fibular collateral ligament . L5 14. ?? 13. Posterior displacement of knee is prevented by: a. injury to common peroneal nerve (C) b. Tibial collateral ligament d. Femoral sheath is composed of: a. 5 (C) d. 3 c.a. superficial pernoeal nerve c. L1-2 (C) c. 2 pyruvate (C) c. 2 b . C2. transversus abdominis fascia b. 6 12. C3. Ureter b. Posterior cruciate ligament (C) c. 4. mesodermal (C) c. internal capsule (C) 23. he has no abduction or adduction of fingers plus no abduction of the thumb. abdominal lymph nodes c.17. Parasympathetic reflex 20. A patient had cut in the center of wrist during a suicide attempt. Retrograde amnesia: a. one frontal lobes lesion c. Involved in CNS development (C) b. radial nerve 18. Regarding neural tube which one is correct: a. Defecation is because of: a. the injury is to: a. median nerve c. Involved in meninges development 24. parasternal lymph nodes b. Primordial germ cells are of: a. mass reflex b. 4 (C) d. myenteric reflex c. can be abolished via prefrontal lobotomy b. Prolonged coma is because of lesion of: a. Projection fibers of cerebral cortex travel through: a. appears in amygdala lesions and propranolol therapy 22. periaquaductal area lesion d. Lymphatic supply of mammary glands is: a. S2. midbrain lesion 21. endodermal . both frontal lobes lesions (C) b. 3 . axillary lymph nodes (C) 19. ulnar nerve b. ectodermal origin b. decortication b. Following is true about middle meningeal artery: a. anus and urinary bladder b. Hamartoma is a: a. L5 d. lesions of cervical or thoracic spinal cord (C) 30. S4 28. Pain b.25. preoptic c. Urorectal septum divides the cloaca into: a. Passes through foramen spinosum (C) 29. adenohypophysis d. Tickling d. Fine touch (C) 31. complete section of spinal cord c. suprachaismatic b. anorectal canal and urogenital sinus (C) 26. Medial lemniscus system mediates: a. Temperature c. neurohypophysis (C) 27. Termination of subarachnoid space is at: a. non-neoplastic condition (C) c. S2 (C) c. completely benign d. Mass reflex is elicited by: a. ADH and Oxytocin are released from: a. What prevents muscles from over-stretching: a. muscle spindles (C) 32. malignant . neoplastic condition with no malignant potential b. Golgi spindles b. L2 b . Monoamine oxidase (C) 40. how will you protect labial branch of facial nerve: a. Complication of betel nuts chewing is: a. Catechol .33. lift the deep fascia of hyoid bone b. In chronic hemolysis. which structure is at risk: a. lifting the blood vessels 39.o . antiepileptics 37. ulceration ?? ?? ?? ?? 36. Premalignant lesion of mouth is: a. carbamazepine c.methyltransferase b. Enzyme for breakdown of serotonin and Norepinephrine is a. spinal portion of accessory nerve (C) b. Cleft palate is caused due to: a. protecting it as it enters surgical field d. ?? b. nerve stimulation studies c. following will deposit in tissues: a. multifactorial (C) d. iron 34. bilirubin (C) b. lipofuscin e. cranial portion of accessory nerve 38. ?? 35. A House surgeon is operating on posterior triangle. conjugated bilirubin d. biliverdin c. Viral b. haemorrhage ?? b. During surgery on submandibular glands. Inducer of drug metabolism is: . retromandibular vein is related to its inferior pole d. Premalignant condition of colon carcinoma: a. III. Essential amino acid is: a. ?? 45. failure of initiation b. It has two lobes b. glutamate c. parotid duct arises from deep lobe c. adenoma 44. Slow growing tumour of thyroid is: a. ataxia (C) 42. hyperplastic polyp c. VI (C) . V b. hamartomatous d. Among the following statements about parotid gland. Lesions of cerebellum causes: a. ?? 47. Following are features of thyroid tumour: a. tyrosine d. phenobarbitone (C) 41. inflammatory polyp b. follicular c. phenylalanine (C) 43. papillary b. rigidity d.a. ?? e. which is false: a. Glycine b. VI c. resting tremor c. ketoconazole b. IV. adenomatous (C) 46. Which nerve lies in the cavernous sinus cavity: a. lateral wall of tympanic cavity 51. Facilitated diffusion is: a. Best mechanism for temperature loss: a. circumvellate 50. ventricles c. atria b. carrier mediated (C) 54. buccinator d. Taste is sensed by all of the following except: a. ?? (C) 49. Muscles of facial expression are all of the following except: a. SA node (C) 55. thirst . IX 48. platysma e. Thyroid gland is surrounded by which fascia: a. prevertebral d. Azygous vein: ?? 53. Tympanic membrane forms: a. deep 52. The most significant prepotential is that of: a. energy dependent b. fungiform b. orbicularis oris b. investing b. Vasonconstriction b. filliform (C) c.d. orbicularis oculi c. pretracheal (C) c. baroreceptor reflex 58. hyperkalemia b. H+ ion secretion is in: a. skin d. H+ secretion is associated with: a. transudate b. A fisherman came to emergency department with pallor. exudate (C) 57. PO2 c. acidosis (C) c. alkalosis 60. PCO2 b. cardioacceleration b. kidney c. . The most probable cause for this is: a. brain e. Increased iron absorption b. Acute inflammation: a. CO 62. Diphylobothrium (C) 63. Which organ gets less than required oxygenation during exercise: a. heart 59. Decreased amino acid absorption. Central and peripheral regulation of respiration is via: a. Skeletal muscle (C) b. Ileal resection leads to: a.?? ?? 56. alpha-adrenergic discharge leads to: a. pH d. reabsortion of bicarbonate (C) 61. excretion of bicarbonate b. Adult Polycystic Disease is: a. Heart failure b. Glucagon 67. Lymphatic supply of ovary is: a. ?? 66. Insulin (C) b. Everted b. Autosomal recessive b. Autosomal dominant (C) d.linked recessive c. Hair loss d. X. Decreased perfusion of kidney c. Duct of gartner b.64. Increased Na 65. Round ligament c. deep inguinal d. Punched out . Another use of thyroid hormone is: a. lumbar/paraaortic (C) e. sacral 69. Aldosterone production is stimulated via: a. Appetite c. ?? e. X linked dominant e. Mitochondial 68. Internal iliac lymph nodes b. Remnant of mullerian factor is: a. Appendix of testis (C) 70. Anabolic hormone is: a. Tuberculous ulcer has which type of edges: a. Renin (C) d. Weight loss b. external iliac lymph nodes c. Type V 73. Antibody-mediated (C) c. Influenza virus b. Type III d. Cytotoxic T cells e. Scientific study of human behaviour b. 75. Protein synthesis (C) b. Staphylococcus d.c. psychology. Following is true about MAI: a. Cytomegalovirus (C) 72. Undermined (C) e. Elevated 71. Scientific study involving epidemiology. sociology. Following is true about sarcomere: . ethics. Post-translational modification 78. Only causes lung disease c. Severe infection in a post-transplant patient is due to: a. Transcription c. Failure of descent of testis from abdomen into scrotal sac. RER function is: a. Definition of Behaviour sciences is: a. Streptococcus c. (C) 76. Good communication skills are in a person who: a. Who does presentations (C) 77. Very common 74. Sliding d. Type I b. Within 10 hours acute transplant rejection is due to: a. biostatistics. Widespread resistant to TB drugs b. Following is true about cryptorchidism: a. Vaccine of Bacillus-Calmette-Guerin induces which type of immunity: a. Produces powerful endotoxin 85. Lung collapses and chest indraws (C) c. Streptococcs pneumonia . Live attenuated (C) 83. Spleen c.a. Lymphatic nodules are not present in: a. Is formed between two Z-discs (C) 79. Lung collapses and chest expands b. In Pnemothorax following happens: a. Calcium is released into muscles via: a. Peyers patches d. RER c. lung expands and chest remains same 82. lung expands and chest indraws e. Active (C) b. Is formed between two intercalated discs b. Secondary immunity c. The most common cause of pneumonia in 6 months to 2 years age group is: a. Mitochondrion b. Tonsils 81. Sarcoplasmic reticulum (C) 80. Produces powerful exotoxin (C) b. Vaccine of Bacillus-Calmette-Guerin is which type of vaccine: a. Heat killed organism c. Which one is true about corynebacterium diphtheria: a. Neisseria b. Lung expands and chest expands d. Passive 84. Formaline killed Toxoid vaccine b. Thymus (C) b. E. Coli .c. Patient is vomiting: ph = 7. Neisseria best diagnosed most easily via: a. Eggs (C) 91. metabolic alkalosis c. hypoxia (C) c. Clostridium difficile 87. Most common cause of Deep Venous Thrombosis is: a. PCO2=45 (35=45). Normchromic microcytic c. HCO3=35. free radicals b. Normochromic normocytic anemia b. metabolic acidosis b. it is: a. Gram stain (C) b. Clostridium perfringens d. Cause is: a. partially compensated metabolic alkalosis 93.d. hypochromic normochromic 92. Clostridium perfringens causes injury via following mechanism: a.25. Cyst b. Culture 89. Streptococcu pyogenes 86. Clostridium tetani b. Clostridium botulinum c. Ethambutol 90. Pseudo membranous colitis is caused by: a. Contraceptive pills . Ocular opacity can be caused by: a. Patient is having MCV=78. hypochromic microcytic d. MCH=28. Haemophilus influenza e. Invasion 88. Hydatid disease caused due to ingestion of: a. Calcitonin works by: a. True hermaphrodite genotype is: a. X0 c. Most common cause of hospital acquired pnuemonia is: a. Surgery d. PO4 is decreased. Testosterone is produced by: a.b. Decreasing bone resorption (C) 99. Stomach b. Proximal tubule c. Immobility (C) 94. b. Vasa recta (C) b. Ca++ is raised. XXY (C) 100. Osteopenia d. Submucous glands are present in: a. Dudenum (C) c. Trauma c. Streptococcus b. Osteoporosis c. Leydig cells . Pseudomonas (C) 95. Pagets disease b. Large intenstine 97. XY b. Hyperparathroidism (C) 98. Sertoli cells (C) b. Increasing bone resorption. Patient is having bone pains. Distal tubule 96. Following is an important component of Counter current exchange mechanism: a. most probable cause is: a. Pnemococcus c. Iluem d. There was a Murad MCQ about pie chart. wat is not in relation with right kidney decending colon* ascending colon duodenum spleen 3.1 0 1 . By 'Bolanian'. Posted by Dr Sultan Ahmad 3 comments Reactions: Saturday. stress hormone ACTH* Epinephrine Norepinephrine Growth hormone 2. Correct answers are marked >> * << 1. side effect of thophyllin nausea and vomiting* hypertension bradycardia seizures . September 12. See the june papers below the January papers. Most imp. The vasculature of kidney i s : a. tributery of external carotid vein Anterior jugular vein* Facial vein 5. Afferent -> glomerulus -> efferent -> vasa recta Alongwith these there were few questions about premalignant conditions of mouth and a question about which is correct about cholesteatoma. 2009 RADIOLOGY JANUARY 2009. wat is not in relation with pitutary gland Facial nerve* Abducent nerve Sphenoid sinus Trigeminal 4. Nor-epinephrine & serotonin degration by MAO* COMT 7. wat is not in post. ant and post spinal arteries arise from . chemotaxic factor leukoterine B4* C3b C5 complex histamine 8. relation with left kidney Deudenum* Ascending colon Descending colon 9. what is secreted in breast milk Ig G* Ig A Ig M Ig D Ig E 11.6. wat cannot pass placenta Ig G Ig A Ig M* Ig D Ig E 12. wat is in between celeiac trunk and superior messentry artery Pancreas* Pancreas + jujenum Transverse colon + ilium 10. where is the lesion Cerebral hemisphere Left cerebellum * superior c puduncle Red neucleus 18. A man with noticed weakness of left side of the body. When r the ventricles filled with blood Diastasis* Atrial systole Rapid in flow 15. where is damage Mammillory bodies frontal lobe* temporal lobe parital lobe occipital lobe 16.. he swangs to left side.. gall bladder looks ok in U/s. cause Atresia of CBD* Atresia of entire extra hepatic apparatus Atresia of hepatic duct Choledochal cyst 17.. adrenalcortical insuficancy causes hypokelimia hyponatremis hypoglycemia* alkalosis .vertebral artery gives anterior spinal. from which foramen facial nerve enters temporal lobe internal accustic meatus* stylomastoid foramen foramen ovale foramen spinosum 14.. Spinal* Internal carotid Vertebral artery and ant cerebellar artery 13. hypotonia of the left limb. A boy had behavior problem..he is agressive in his mood. PICA gives post. loss of memory but resolve …. a new born presented with obstructive jaundice. supply to the dorsum of foot is by sephenous nerve* Sural nerve Common peroneal Deep peroneal 21. patient with difficulty in standing from sitting postion. Esophagus constriction at Aortic arch* Left ventricle Rt atrium 25... utrine cancer spread to labia majore via Round ligament* utrosacral ligament . if there is damage to sciatic nerve. wat is not in relation with left recurrent nerve SVC* Arch of aorta Lig arteriousum Trachea Esophagus 23. wat vein run with Post..19. stucture compressing Esophagus Left bronchus (correct) Left Venticle Right Atrium Right ventricle Thymus 24. interventricle artery great cardiac vein middle cardiac * small cardiac 26. Gluteus Maximum* Gluteus medius Tensor fascia lata Quadricep femoris 22. In resting stage actin is attached to tropomyocin troponin titin myosin 20. down syndrome trisomy 21* 18 15 13 35. drain in axillary ven* runs on radial side starts from palmar surface 28. nerve damage Axillary nerve* Musculocutaneous Radial Ulnar 30. wat is true supplied by anterior division of internal iliac* separated from uterus by pouch of Douglas in males has seminal vesical above and vas deferens below 33.27. true Scaphoid. fuctional n unit of the lung* contains lobar bronchus pulmonary veins run in intersegmental tissue 32. broncopulmonary segment wat is true anatomical. urinary bladder. Basalic vein . wat is in proximal layer of carpal bone. lunate triqutral & pisiform* 31. sephenous nerve wat is true drain into femoral 3.Which of the following tissue is most radiosensitive . median nerve supply lateral 2 lumbricals* causes wrist drop 29. injury of neck of the humerus.5cm inferior n lateral to pubic tubercle* has only 1 tributary connected to short sphenous by perforators 34. sypathetic supply by alpha receptors dilator pupilary musscle of eye* brochoconstriction cardioacceleration . diagnostic test for strptococuss is ASO titer Blood culture* Urine Dr 37. injury to hippocampus Memory loss* 40. 65% of Na and water reabsorbed from PCT* DCT Loop of Henle Collecting duct 38. thoracic duct passes with aorta * 44. medial part of breast supply which node internal thoracic* lateral grp ant grp medial grp 42. diapharam contraction cause increase in thoracic pressure increase in thoracic diameter 43. .Skeletal muscle Cartilage* Bone 36.Erythropoietin is secreted from Yellow marrow Macula densa Preitubualr capillaries* PCT 41. . iron absorb in ferruos form* oxalate 51. Patient Gravida 4 with anemia and occult blood in stool Iron deficiency anemia * Sideroblastic anemia Thalassemia Megaloblastic anemia 50. Trachea start from …. secretioin of slivary gland* 46. wat is not supplied by internal thoracic artery Inferior portion of rectus muscle* Ant abd wall upto umbilicus Pericardium Thymus 49. wat is the cause of edema Plasma colloid pressure is decreased* Increase hydrostatic pressure Increased capillary permeability 48.. parasympathetic supply inc.45. gastric emptying delayed by CCK* Secretin Gastrin Vagus 47.wat is true C6* . A boy has edema due to protien loss of 3..5g in urine. Bile salt absorb from Ileum* Proximal jejunum Distal jejunum doudenum 52. arterial supply to primary visual area PCA* ACA MCA PICA 58.30. blood supply to Bundle of His.35.25.40 25 20 27. Cerebral Anterior and pst. Cerebral 57. wat the reflex gagtrogastric reflex gastro coic reflex (correct) 59. which artery supplies Anterior and middle cerebral (correct) Anterior Middle Middle and post.25.5* 30 . wat is true abt CSF secreted at rate of 500 ml/day* choroid villi are seen by naked eye aqueduct transmits CSF from 4th ventricle to spine 54. Median of 20. A boy defecate just after he takes meal.30. Circumflex artery a br.20.C4 C5 Cricoid cartilage 53. metastatic carcinoma least common site brain lung liver spleen(correct ) 56. premotor area . Of left coronary artery Posterior interventricular artery Main Left coronary artery Main right coronary artery 55. A surgeon did a study on patients with heamorroids he included 100 pt in grpA and treated them surgicall whil kept the other 100pt on high fiber diet he interviewd them after 12mons and found that the grp with surgical treatment had a better response…… wat type of study is this? Case control Cohort Prospective Randomized control trial 63.MHC complex is related to HLA typing* blood grp antigens same in identical twins 68.60. Right bronchus as compare to left is longer ant to pul artery right bronchus is more vertical than left* 62. nerve supply to latismus dorsi thoracodorsal* subscapular long thoracic 61.sup parathyroid gland is related to sup thyroid artey* lies outside the pretracheal fasciae lies anterolatral to thyroid 67. if a patient develops a fatal disease it is best to hide it from him tell it to him and his family as soon as possible* 65. Doctor patient relations and how good a doctor is known by his degrees his records his pleasant behavior and bed side manners* 64.Benedicts test is done for blood sugar urinary glucose urinary reducing sugars* 66. first part of axillary artery ends at . border of pectoralis major . border of pec minor * border of teres major 69. A patient with multiple fractures and BP 90/60 pulse 60/min wat is the most appropriate initial treatment? IV fluids* Reduction os fractures 73. Myleination of the brain wat is most inappropriate? Starts at birth Occurs hapharzadly Sensory neuros are myelinated first Occurs by oligodendrogliocytes 72. If RCA is occluded distal to the origin of right marginal artery which part of the heart will be affected most? Rt atrium SAnode AVnode* Apex of the heart 71.Esophagus is related to which structure whose dilatation can cause dysphagia l atrium* left vent SVC Rt atrium 70. DIC most inappropriate low FDP* consumptive coagulopathy treated with heparin low platelets . Factor 9 def wat is the appropriate treatment? Cryoppt* FFP Whole blood WBC 74. thromboembolism is most common in? leg* liver brain kidney 82.influenzae 81.which organ has least chance of metastasis lung bone liver kidney spleen* 80. OCP increase the risk of .75. which organism causes meningitis secondary to pyogenic lung abscess? Strep Staph Pneuococcus h. Highest mortality rate Hepatitis A Hepatitis B Hepatitis C Hepatitis D Hepatitis E 78. Fat embolism most unlikely urinary test is diagnostic* occurs due to trauma to fatty tissues caused bt acute pancreatitis 77. Kleinfelters syndrome chromosomal deficiency autosomal dominant autosomal recessive extra X chromosome* extra Y chromosome 76. Diphtheria is transmitted by sexual contact respiratory * feco oral 79. breast cancer endometrial cancer thromboembolism* osteoporosis 83. Most likely finding in CSF inc glucose in fungal meningitis dec glucose in viral meningitis lymphocytosis in aseptic meningitis dec glucose in pyogenic meningitis 88. Osteoporosis is less in premenopausal women due to effect of estrogen* testosterone progesterone androgens 84. smooth muscles are striated voluntary mostly arranged in circular and longitudinal layers * 90. A trauma patient in shock with stab wound along left 10th rib which organ is damaged? Liver Spleen* Colon Duodenum 85.which of these is not caused by bacteria histoplasmosis* 87. hyperparathyroidism causes least likely . in autosomal recessive disease all siblings are affected both parents are affected only mother is affected all siblings have a chance of one into four of contracting the disease* 86. most imp for wound contraction myoblasts fibroblasts myofibroblasts* epithelium collagen 89. Has thick submucousa Mucousa has extensive folds* 94.Bladder S.After gastrectomy patient will develop iron def anemia . About large gut descending colon has no haustra appendix has no tenia* all large gut is supplied by inf mesenteric artery ascending colon has a mesentry 93.hypertension dystrophic calcification* inc PTH 91.vesicles Prostatic urethra Ejaculatory duct* 92. Which HBpathy presents with crises sickle cell trait sickle cell disease* thalassemia Hb C 96. unconjugated bile is carried by which protein? Albumin * Pre albumin A globulin B globulin Fibrinogen 97. Ductus deferens ends into Prostate u. most probable abt gall bladder. a patient with known HBV infectio comes with HBSAg + HBe ab+ HBV DNA + HBC IGM ab+ Wat is the diagnosis? Acute dis * Chronic carrier Chronic active 95. Mast cells* Eosinophils IgE 103. what Is true abt Hb consisits of alpha and gamma chains in adults imp buffer of H+ * not assoc with CO2 transport 100.PDGF is secreted from which part of platelets alpha granules dense bodies cytoplasm lysosomes 101. 60 yr male with lymphandenopathy and low hb low platelets and increased wbc microscopy show large mature cells wats the diagnosis? CLL* Burkitts Follicular NHL 99. Heparin is released from which cells. thyroid is invested in which fascia? Pretracheal* . which cells produce antibodies plasma cells* T cells Bcells Platelets 102.megaloblastic anemia due to folic acid def pernicious anemia* sideroblastic anemia 98. Epitheloid cells in granulomas are formed by which cells macrophages lymphocytes langerhans cells* 104. * Has parthyroids ant to it 106. Satiety center is located in which part of hypothalamus anterolateral VPL VL Medial* 108.Deep cervical Prevertebral 105. wat is true abt pituitary ant pitutiary derived from neurons no portal vessels btw hypothalamus and post pituitary* 107. wat is true abt thyroid consists of collecting ducts and acini epi changes shape according to state of activity. most potent stimulus for release of aldosterone is inc K+ * . at the end of pregnancy uterus is most sensitive to which hormone? Estrogen Progesterone Prolactin Oxytocin* 110. ICF & ECF differ in inc K+ in ICF 112. Most abundant phagocytic cells in circulation basophils monocytes macrophages neutrophils* 111. D & C at 18 days after LMP endometrium will show which stage of uterine cycle? Follicular Ovulatory Secretory* Proliferative 109. Pallegra is due to def of riboflavin thiamin e niacin * vit C 114. in females which structure lies btw ureter and peritoneum uterine artery* inf vesicle artery 117.inc Na+ inc glucose 113. wat is not true abt adrenal gland fetal gland is 1/3 size of the adult gland it is invested by renal fascia adrenal cortex is derived from mesoderm 119. which of these muscles is a part of the pelvic diaphragm deep transverse perinei puborectalis* bulbospongiosus ischeocavernosus 118. patient with loss of fine touch proprioception and vibration injury to which part of spinal cord? Fasiculus gracilis Fasiculus cuneatus Post white column* Lateral white column 116.tendon of which of these muscles is intracapsular . which structure is most ant in popliteal fossa popliteal vein politeal artey * tibail nerve bicep femoris gastrocnemeu s 120. Trauma to the middle meningeal artey causes subdural hemorrhage epidural hemorrhage* SAH 115. a person in a room with optimum enviormental conditions how will the heat loss occur convections sweating breathing radiation and conduction* voiding 124. major part of energy utilized during breathing is to overcome elastic recoil of lungs* resistance of chest wall large airway resistance small airway resistance 125. which vessel will be damaged if the phrenic nerve is cut musculophrenic int thoracic pericadiophrenic* . a bullet pierces the intercostals space which layer will it come after the intercostals muscles parietal pleura visceral pleura endothoracic fascia* pleural cavity 126.bicep femoris plantaris popliteus* semitendinosus 121. which of these are the major resistance vessels arteries veins capillaries arterioles* venules 122. patient losses 1 litre of body fliud in 1 hour wat will happen first inc PR inc BP inc veno spasm* 123. lymphatic of uterus do not drain into int iliac nodes ext iliac nodes sup ing nodes inf mesenteric nodes* 129. metastatic tumors are identified by invasion of other structures* pleomorphism inc NC ratio inc mitosis 133. epi of a smoker will show . wat is seen in dysplasia pleomorphism inc NC ratio* inc mitosis 132. which of these is not a branch of internal iliac sup rectal* middle rectal inf vesicle sup vesicle 128. a patient with hoarseness and lyrangeal nodule which is most unlikely atrophy * nodule hypertrophy hyperplasia 130. sarcomas have rapid growth inc vascularity* capsule benign 134. pseudo hypoparathyroidism. Due to inc ca Dec vit D PTHrP 131.127. not part of the portal tract portal vein hepativ artery bile duct hepatic vein* 139.inc ciliated epi mixture of st sq and pseudostratified cells* dec goblet cells 135. which vessel is involved and dilated in portal HTN portal vein* hepatic artey hepativ vien SMV 137. in males uretric stones mostly lodge at which point . most unlikely abt glomerulus is has urinary space btw 2 layers is cup shaped is blind ended part of tubule has stratified epi* has podocytes 136. Clavipectoral fascia completely covers pectoralis minor* forms suspensory lig of breast forms axillary tail 141. not a part of axillary nodes ant grp medial grp apical grp central grp deep cervical* 140. most unappropraite abt liver has dual blood supply hepatic artey gives only 35% of O2 portal vein has 70% O2 * 138. Otic ganglion lies under foramen ovale* foramen spinosum maxillart nerve stylomastoid foramen . pain of the ovary is transmitted to the medial side of thigh thru which nerve femoral obturator* ilioinguinal 146.just below kidney* at crossing of ext iilac vessels at pelvic brim at ischeal spine 142. at the level of second part of duodenum wat is unlikely aorta at the right of SVC* left renal vein 145. most constricted part of the male urethra? Ext meatus* Infandibukum Navicular fossa 144. epidural space contains venous plexus* contiues into skull at foramen magnum attached to dorsal coccyx upto S2 148. left ventricle does not contain supraventricular crest chordae tendinae aortic vestibule papillary muscles 147. if there is fracture of the acetabulum post and superiorly which bones r inv ileum and ischium * ileum and pubis isschium and pubis 143. which of these is not a basic tissue of the body epi nerve muscle blood CT 153. Processus vaginalis covers testis only* covers ductus deferens forms ext spermatic fascia arises from parietal peritoneum arises from visceral peritoneum . chemoreceptors respond to inc PCO2* dec PO2 inc Ph inc temp 155. Great cerebral vein does not drain deep cerebral vein sup cerebral vein basal vein thalamic vein 152. Osmotic pressure depends on electrical equivalence conc gradient yenp no of particles mol size 154. Superior petrosal sinus lies in falx cereberi fakx cerebelli tentorium cerebellli* diaphragma sella 151.149. middle menigeal artery enters thru which foramen? Foramen spinosum* Foramen rotundum Foramen ovale 150. what is true about critical temperature: a. half life of pseudocholine esterase is a. Dorsal rami of nerves supply ext of trunk ext of limbs 157. R o p i v a c a i n e is used for epidural a n a e s t h e s i a instead of bupivacaine during labour. b.45 c. which vessel lies ant to IVC left renal vein left renal artery right renal vein right renal artery right testicular vein* 158. of oxygen is 1 2 1 c d.156. prolonged action 4. of nitrogen is 21c e. if inguinal canal of a female is opened wat does it contain round ligament and ilioinguinal nerve 159. applies on mixture of gas. is temperature below which pressure can keep g a s liquid 2. no effect on fetus c.37 3.27 b. of co2 is 31c c. lymphatic drainage of testis is into para aortic nodes sup ing nodes int iliac nodes ext iliac nodes 1. 2 hours . Humidity in OT s h o u l d be a.because a. recovery is quick d. it is l e s s toxic b. which one of following anti emetic has max bioavalability a.which one of following drug worsens angina a. it is part of minute ventilation d. surfactant produced b. what is true about F R V volume.which I ticked. increased in shallow breathing b.isossorbidedinitrate c. nitrogen breath test c a n m e a s u r e it c.flow of inhalational g a s e s a c r o s s memebrane is directly propotional to. a. regarding p h o s p h o d i e s t e a s e inhibitor (I can just recall there w a s s o m e option regarding its inactivation of cAMP. regarding flow of gas in a tube. viscosity 10.. can explain ischemia of myocardium b.regarding coanda effect what is true a. gender c.lisinopril 13..directly proptional to a. 12 hours 5. a. do not explains vascular flow c.verapamil b. 3 days d.verapamil .porcholrperazine 12.ondansteron b.b. age b. lipid solubility of a drug 7.which one of following used in C C F but is not a vasodilator… a.cyclizine c. hypoxia 1 1 . volume of distribution d e p e n d s on following except a. explains unequal distribution of air in alveoli 9.droperidol d. pressure difference c. radius b. transalveolar p r e s s ue c. 1 week c.propanolol d. can be measured by helium dilution metho 8.metoclopramide e.but don’t remember exactly now what the other options were ) 6.. cervical nerves c .durationof anaesthesiac.emia more frequently than Phanacetin 17.5 c.hypoglossal nerve 19.is nephrotoxic c. hypoxia. Cha nces ofmalignanthyperthermiai ncre ases b. 22. a n a l g e s i c .increases P T … b. isoflurane c.isoflurane effects cardiac activity by: (options don’t remember exactly.exact term don’t remember but it w a s bi.not an anti inflammatory d.2 d..5 20. a .child with burn 6 hour b a c k brought to ER for dressing. regarding paracetamol what is true… a. due to increased risk of prolonged m u s c l e contractions 18.drug of choice in prizmental angina a.0. desflurane b.1.sensory supply of dura is by all except a. hyperkalemia c.) .diltiazem 14.more potent a n a l g e s i c than codeine e .trigeminal nerve b. inhibiting intracellular p h o s p a t e a s e s c.nitous oxide is a .but the main theme was how isoflurane actually s u p p r e s s e s cardiac activity. nitrous oxide 23.3 b.mechanism of action ) a..metoprolol e. sevoflurane d. .diltiazem 15. decreasi ngAVconduction b.b.nifedipine c. c a u s e s met-Hb. v a g u s nerve d. a.vasodilator 16.isosorbide dinitrate b.nimodipine d.mac is reduced i n … a.Mac of isoflurane is a. which one irritates larynx.0. F a m i l y history of sibling died under GA is p o s i t i v e … y o u are cautious regarding using succinylcholine in this patient why? a.nitoglycerine c.1. b. bidirectional block (something like that . 29. inferior vena c a v a b.regarding isoflurane one correct statement. 31 ..thiopental d.d.tributaries similar to aorta c.. dec H R c. After induction of a n e s t h e s i a by 10 mg/kg thiopentone. b.thiopental effects on cvs: a. dec contractility 26. left renal vein . correct statement regarding inferior vena c a v a … a. lingual vein c. w h i c h o n e is potent bronchodilator. a patient of thoracotomy wid o o z i n g w o u n d s b.can be u s e d in a s t h m a t i c patients a. pt is hypotonic.recommended for a n a e s t h e s i a induction. hypersensitivity b. oocipital vein 25. d. a.ketamine b. diabetic patient undergoing hemicolectomy c.. sigmoidsi n us b.transient fall in bp b.commences at L5 b.is related to bare area of liver 32.which one of following h a s steroid nucleus.midazolam 27 . C a u s e ? a.ketamine b. c. regarding left adrenal vein drains into… a.propofol 28. all of the following are tributaries of internal jugular vein except: a. 30. superior thyroid d. 24.lies posterior to right adrenal gland d. a.profol c. ketorolac drug of choice i n … a. a.a. decreasesinoldage 44. c a r d i a c output c.prednisolone.33. is differance of systolic and diastolic pressures b..dead space develops 37. a.if ventillation increasd..pulse pressure a. transducer is … a . a.laminar flow what is true….Ser umP otas si umis 5. Di ag nosi s? a.patient in ot undressed.difference of end systolic volume and end diastolic volume 45.. always low in all types of shock.heat loss is by a.CRF c. cardiac index is. whats true about montoux test.respiratory rate d. regarding cvp whats true…. b.heart rate b .toxicity 34.. b. lysosomes a. g i v e s clue about pulmonary edema d. a.( cant recall what they asked..known hypertensive. adrenaline is added to local anaesthetic to restrict i t s … a. produced by golgi apparatus .just remember there was a question regarding it as well ) 42.patient onhtnsive drug now comesinsho ck . a.convert one energy form into another 39.hemolysis b.radiation n conduction 36.. reading 5cm H 2 o is equal to 10 mmhg 35.insternousexercisewatsnotincreased….ARF 40.reaction type is type 4 41. give accurate measurement when catheter in rt atrium c. directly proportional to viscosity 38.blood pressure 43. c a r d i a c output to body surface area ( B S A ) . lidocaine 5 1 . .formed by condensation of S i b s o n fascia 53.adenosine mode of a c t i o n … a.plasma b. 47.formed by condensation of pretracheal fascia 46.correct statement is: a.bronchial tone circadian rhythm m a x at … a .patient now having bleeding from left ear n left eye pupil dilated.supra pleural fascia is … . a.select one false…..1 5 % blood loss.road traffic accident .3rd s p a c e fluid loss. ringers lactate d. m o v e s with respiration.all are mesoderm derivative e x c e p t … .. a. . a r e c e s s in parietal pleura c.dopexamine 48. ringers lactate 52.pulmonary ligament is a. 56. e a r l y morning 49.10%dextrose c.subclavianvess el s crossesitsuperiorly 54.replace by .contents are carotid artery.. supplies posterior 2/3 of tongue. blood. c l a s s three antiarhythmic name a. three times normal saline c. a.which cranial nerve damaged .thickining of pretrachial fascia b.vagus nerve. normal saline 5 0 . p a s s e s over upper part of medial side of tympanic membrane 55. b . chorda tympani. has oxidases 46. pilocarpine c. whole blood b.regarding carotid sheath. l e a v e s through styloid mastoid foramen. has hydrolases n catalases c. in which lung root s i n k s during respiration b.internal jugular vein b.b. quinidine b.fluid of choice i s … a. d. 1 0 .attached to first rib c . c. anatomical .which of following joint is immoveable a.occulomotor b. incomplete cartilages in airway tract tubes c.manubrosternal b.influenza d.. d. to pyogenic lung a b c e s s a.measuring dead s p a c e .surgical.respiratory quotient is a.660.which of following organism c a u s e s menigites sec.carbondioxide produced per oxygen utilization 5 9 .true regarding trachea a.h.regarding bronchopulmonary s e g m e n t s correct statement is a .trigeminal c. parietal pleura 58. 0.symphsis pubis d. lungs. right is longer than left c. aorta.functional unit of lung 63. m a cof sevofluranewith 6 0%N 2 O a.regarding right brachiocepahilc vein. c. contains ma c r op h age sin alveoli b.sternoclavicular c. 66.nitrogen breath test can be used for a.a. is vertical . commences at level of t4 d.what is true a.68 60.dorsal rami of cervical spinal nerves supply 62.streptococcus b.staphyllococcus c. cilliated epithelium uptil terminal b r o n c h u s 65.shoulder joint 61. heart. a. h a s valve b.thoracic part of sympathetic trunk supplies all except.neisseria 64. h a s a segmental bronchus b.facial 57. b.trochlear d. v a g u s nerve. b.spleen b.which of following doesn’t have a n y s e n s o r y supply a .dimethocaine .lingual nerve is its tributary. d.commences at L5 b.pica b.ropivacaine c.what is true about amide local anaesthetics: a.67.isoflurane b. c. c. l y m p h is drained into a.. b .regarding inferior vena cava.basillar artey c.regarding ropivacaine what is true a .bupivacaine b.most toxic local anaesthetic is: a.pKa is equal to that of bupivacaine 76. apatientisinductedwith aninhalational ag e n t.regarding internal jugular vein a.trochlear nerve 7 3 . 71.related to bare area of liver d. a.occulomotor nerve.they are bound to albumin 75. (I think in question it was also mentioned about mac of that agent and it was < 0.1 .lies posterior to right renal gland c.hypoglossal nerve crosses it anteriorly.equally effective than bupivacaine d..continuation of si gm oi d si n u s .lymph node 69.what is true .. 6 8 . l o n g duration of action c.don’t remember exactly ) which agent is responsible for this: a. h ed eve l o p sb ro n c h os p asm n ai r wa yirritation. h y p o g l o s s a l nerve.anterior inferior cerebellar artery is a branch of a.desflurane c.posterior cerebral artery 72.thymus c. l e s s cardiotoxic b .vertebral artery d..halothane d.sevoflurane 74.filling of ventricle produces which heart sound .has tributaries similar to aorta 70.lidocaine d. 8 0 % . b e c a u s e of hypocalcemia 79.why ? a.it is a recemic mixture 80.now u h a v e to give another dose.which one of following is correct : a. p r o g e s t e r o n e 86.pseducholine e s t e r a s e is deficient 87.bupivacaine c.because of hyperkalemia b .lidocaine b.why? a. f l o w of g a s through s m a l l orifice affected by which p a r a m e t e r : a.77.orifice of a tube is s m a l l .cardiovascular stability is more 81.it blocks nicotinin receptors at post synaptic junction b.by decreasing S A and A V conduction 88.1216 weeks b.piperacurium is preferred over pancuronium b e c a u s e a.low ph increases its shelf life b.presseure difference b. Prilocaine d.suxamethonium c a u s e s tachycardia in a patient .half life of pseudocholinesterase is: a.in pregnancy respiratory rate increases which hormone is responsible for that : a.estrogen b.renal excretion is l e s s b. po2 90 s p O 2 1 O O % b.you have given a d o s e of suxamethonium in a patient .which local anaesthetic c a u s e s methaemoglobinemia: a. a. < 5 0 % b.by decreasing av conduction c.density c.it blocks muscarinic receptors at postsynaptic terminal 78.more toxic than bupivacaine c.prolactin c.you are very cautious regarding second dose. po2 50 s p O 2 60% 89.3 hours c.what is the mechanism ? a..b.2 days 83. 82. po2 60 s p O 2 9 0 % c.thyroid d .in how m a n y percentage of population malignant hyperthermia c h a n c e s exist? a .. 6 0 % c.it d e c r e a s e s on increasing temperature 85.ace inhibitor acts in ccf by .radius 84.in huntingtons chorea succinyl choline is contraindicated .regarding ropivacaine what s true: a.reducing cardiac out put .it blocks nicotinic receptors at autonomic ganglia c.what is true about viscosity? a. chloride ion .Regarding Laudanosine.cell is in complete depolarization p h a s e in : a .bicarbonate .sodium in blood is electrically balanced by . tubocurare c. cisatracurium d. Q T interval c . g i v e rise to greater petrosal nerve b. produced by c i s than atracurium c.physiology.diffusion b. c n s depression 104. a c t i v e tranport d . Muscle relaxant that can be given to an asthmatic pt a.rest options don’t remember 99.it is called: a.nerve to submandibular gland c.All are the branches of V a g u s nerve E X C E P T a) Auricular b) Lacrimal c)Pharyngeal d) Recurrent Laryngeal 97.and H C O 3 was 26 a. s v c c. atracurium b.all true except (exact options don’t remember but I g u e s s they were like this ) a.end systolic volume d.one c a s e scenerio was given in which ph was 7 .type 2 respiratory failure and compensated respiratory a c i d o s i s d. What is the condition? a) Third degree heart block b) Mobitz T y p e 1 c) S i n u s arrhythmia d) Mobitz T y p e 2 94.maximum map is in a . c r o s s e s blood brain barrier d . l e s s c o n c . d o s e s n o t release histamine 91. . suxa 103. p C O 2 was 60.cis atrecurium is preferred over atricurium why? a .regarding intracranial part of facial nerve what is true: a . a c t i v e tranport 101.org/cgi/content/full/292/3/R1320) d) Mitral stenosis . s e c .90.metabolite of atra and c i s curium b . E C G of a patient s h o w s progressively increasing PR intervals followed by dropped beat.preganglionic sympathetic supply to sublingual gland is transmitted along which cranial nerve 92.distends intestine 93. 4 .metabolic alkalosis 95.nerve to tensor tympani 1 E C G changes can be seen in the following conditions E X C E P T a) C h a n g e in body position b) MI c) Sleep (http://ajpregu. q r s complex b . d u e to l e s s production of metabolites b . i v c b .type 1 respiratory failure and compensated respiratory a c i d o s i s c.organ perfusion 96.pulmonary artery d.facilitated diffusion c . S T s e g m e n t d . T wave 102..About C S F most appropriate statement is a) it is ultrafilterate of P l a s m a b) h a s more g l u c o s e than blood c) h a s cushioing effect d) absorbed in Choroid P l e x u s 98.regarding why nitrous not used in abdominal surgeries correct statement is: a.capillaries 100.glucose tranports across membrane due to its concentration difference.respiratory a c i d o s i s b.ventricular end diastolic volume c.best indicator of preload is: a.pulmonary capillary wedge pressue b. C (http://en. DNTKNWEXACTLY 5 Central v e n o u s pressure a) Inc.org/wiki/Neural_tube ) 3 SACRALIZATION a) Union of 1st sacral vertebra with 5th lumbar b) Union of 5th lumbar with 1st sacral c) F u s i o n of all sacral vertebra to form sacrum d) Flexion at sacrum e) … Ans: B http://wiki.com.e) Aortic regurgitation Ans: A: 2 AboutNEURALTUBE a) F o r m s primitive streak b) F o r m s notochord c) C n s develops from it d) F o r m s three germ layers e) … Ans.com/Q/What_is_sacralization_of_the_l-5 4 Stroke volume a) Cardiac output depends on it b) Heart rate determines stroke volume c) Increasesinhaemorrhage d) Independent of venous return e) … Ans.answers. in haemorrhage b) D e c in gram negative septicemia c) Dec in heart failure d) … e) … ans.pk/books?id=-V3k2MnDQcC&pg=PA275&dq=cvp+is+decreased+during+gram+negative+sepsis&hl=en&ei=LSEpTPu_NMaA lAeNmY2ACA&sa=X&oi=book_result&ct=result&resnum=9&ved=0CE0Q6AEwCA#v=onepage&q=cvp%20 is%20decreased%20during%20gram%20negative%20sepsis&f=false 6 What c h a n g e o c c u r s from lying to standing position? a) V e n o u s pressure inc b) Arterial pressure inc .wikipedia. b http://books.google. htm 7 A simple senario of hurshspring d i s e a s e 8 Which hepatitis dangerous in p r e g n a n c y a) A b) B c) C d) D e) E Ans.whatsdiagnosis DIC 10. who’s been deliverd in a remote village.cvphysiology. septicemia.org/wiki/Vitamin_D 12.wikipedia. D synthesis a) 1 hydroxylation o c u r s in k i d n e y b) 25 ocurs in kidney c) 1 o c u r r s in liver d) 25 in lung e) Both in s k i n Ans. presents with shock. A http://en. Most common cause of pulmonary embolism? a ) H e a r t failure b) Dvt c) Cancerlung d) Pneumonia e) … ans. B 11. A http://www. R eg ar di ngvit. e 9 A lady.com/Cardiac%20Function/CF017. . Treatment of erythroblastosis fetalis? a) E x c h a n g e transfusion with b +v e b) With b _ve c) Ab +ve d) Anti D e) … ans.Ans.c) Sweating d) C u t a n e o u s vasoconstriction e) …. bleeding from venae sites. Viridans c) Strep pneumonae d) S t a p h epidermiditis e) … ans. MostcommoncauseofSUBACUTEBACTERIALENDOCARDITIS a) Staph aureus b) Sterp. which 1 true etc 18 D i a g n o s i s of typhoid in 1st week A n s … blood culture 19. A lil senario of cervical rib i. A qs about C S F findings. b 20. a) Medial rectus b) Lateral rectus c) Inf oblique d) S u p rectus e) Inf rectus 14. C e ph al i c vein a) P r e s e n t in deltopectoral groove 16. c 15. . WHICHISNOTSUPLIE DBYOCUL OMOTORN. b) Content of carotid triangle c) P i e r c e s deep fascia behind sternocleidomastoid d) … e) … ans. A u r i c u l a r v.13. About external juglar vein a) F o r m e d by retromandibular a n d ant. e w e a k n e s s and paresthesia along ulnar nerv course 17. tube c) L o o p of henle d) Collecting duct e) …. Ans. There were 4_5 qs about aldosterone 24.pk/books?id=c41BJkHrniEC&pg=PA25&dq=omental+fat+necrosis&hl=en&ei=8i opTKykIaWlsQaB9JzEBA&sa=X&oi=book_result&ct=result&resnum=4&ved=0CDYQ6AEwAzgK#v=onepag e&q=omental%20fat%20necrosis&f=false 22. ljungdahlii c) Tetanustoxin d) C. Typeofomentalnecrosis? a) Caseous b) Gangrene c) Fat d) Coagulative e) Fibrinoid Ans.wikipedia. A http://books. d http://en.C a u s e of g a sg a n g r e n e (C. C.google. tube b) Distal c. C http://books.google. perferringens ws not givn) a) C. septicum e) … ans.org/wiki/Gas_gangrene 21. Stimulus for aldosterone release a) Hypernatremia b) Hypokalemia c) Hyperkalemia d) Alkalosis e) … ans.botulinum b) C.com.com. Most of bicarbonate absorption ocurs in a) Proxial c.pk/books?id=pekX0WUKzMoC&pg=PA414&dq=bicarbonate+reabsorption&hl= en&ei=oCspTPu0CIO0lQftobDBw&sa=X&oi=book_result&ct=result&resnum=1&ved=0CCYQ6AEwAA#v=onepage&q=bicarbonate%20r eabsorption&f=false 23. c . which ion r e p l a c e m e n t is most important? a) Ca b) Mg c) Na d) K e) Cl Ans. Aldosterone c a u s e s a) Hypercalcemia b) Hyperkalemia b) Acidosis c) Hypernatremia d) … ans.25. A p e r s o n suffering from dehydration. What to give to Inc vitamen in diet .wikipedia. Ans. tube c) Collecting duct d) Bowman capsule Ans c 28. D http://en.org/wiki/Circadian_rhythm 29. C 30. A D H acts on a) Loop of henle b) Distal c. e) Upraoptic n. F o r circardian rhythm optic nerve s e n d s fibers to a) Optic chiasma b) Medial geniculate body c) Lateral geniculate body d) Suprachiasmatic n. c 26. D e c aldosterone will lead to a) Hyponatremia 27. all the children there h a v e s o m e p e r m a n e n t disabilitythe p a r e n t s of s u c h children a r e suffering from a) Denial b) Anger c) Bargaining d) Depression e) Acceptance 34.a) Nuts b) Yougurt c) Egg d) Green vegetables e) Margarine (sumthin like it) Ans. at a children hospital. E n d o g e n o u s pigment in chronic hemolysis a) Hemosiderin b) Lipofuscin c) Anthracotic pigment d) … e) … Ans A 33. D 32. D 31 Natural anticoagulant a) Heparin b) Warfarin c ) Aspirin d) Plasminogen e) Fibrin Ans. In a ward. Square root of variance a ) Mean b) Variation c) Standard deviation d) Median e) Acuracy Ans C . berkeley. D r u g contraindicated in renal failure a) Amikacin b) Chloramphenicol c) Vancomycin d) Pencillin e) … ans. Difference BW hyperplasia and benign tumor a) Hyperplasia invades surrounding tissues .stat.edu/~stark/Java/Html/SampleDist. Which will not b present in a lung hamartoma (it ws senario of pulmonary hamartoma and qs ws a s k e d at the end of senario) a) Coin lesion on xray c h e s t b) Mostly asymptomatic c ) Mostly D i s c o v e r e d incidently d) Benign e) Cartilage in lesion area Ans E 40. a 38.35. What is therapeutic index of a drug? 37. Drug for hyperthyroidism in p r e g n a n c y a ) Methimazolepropylthyrouracil b) Ppu c) Radioactive iodine d) Total thyrodectomy e) L i g o l solution Ans B 39. b http://www.htm 36. Mean Inc with a) Inc in sample s i z e b) D e c in sample size c) I n cins t an d ar d deviation d) … e) … ans. b) c) d) e) E Benign tumor metastasize to tissues Benign tumor shows aggressive growth Hyperplasia shows pleomorphisim Benign tumor is surrounded by capsule Ans 41. After gastrectomy what can happen a ) Malabsoprtion b) Steatorhea c) Anemia d) … e) … ans c 46. Submucosal glands are present in a) Stomach b) Gall bladder c) Duodenum d) Small intestine e) Pancreas Ans C 45. . A senario was about sturge webber syndrome in which he gave neurological symptoms I.e seizures and convulsions then asked that what other symptom can be found in that child and ans ws GLUCOMA . Premalignant condition of oral cavity a) Erythroplakia b) Leukoplakia c) SCC d) Lichen planus e) … ans b 43. Point to note that they did not talk about PORT WINE STAIN (birth mark) on face or forehead A 44. Atrophy means a) Dec in cell size 42. Which hormone Inc gastric contraction? A) GASTRIN B) CCK C ) Secretin c) Gip Ans A 52.After gastrectomy B12 is not absorbed due to deficiency of a) Parietal cells b) Chief cells Ans A 47. Bile salts are absorbed where? a) Terminal ileum 50. Final common motor pathway a) Corticospinal tract b) Alpha motor neurons c) Upper motor neurons d) Cerebral cortex e) … ans b . Bile a c i d s are conjugated with which amino acid to form bile salts a) Taurine 51. G a s t r o p a r e s i s treatment (last part1 me b tha) a ) Metoclopramide 48. Absorption of long chain fatty a c i d s ? a ) Ileum 49. Rupture of post wall of duodenal ulcer will damage a) Gastroduodenal artery 53 Left renal vein relation to aorta a) Ant b) Post Ans A 54. . T 8 level and dy a s k d about wat will happen .. ans a 60.55. During systole a) Both ventricles contract simultaneously b) 5 0 % filling of ventricles occurs c) Blod flow inc in coronary arteries d) . There w a s a qs about injury at T 4 . e) . Regardi ngpulmonarywedgepressure a) Called so cz measured by a wedge shaped catheter b) Usually bw 10-15 mmhg c) Measure of rit atrial pressure d) … e) … ans b 59. c h o i c e s were not straight forward e . Glomerular pressure inc due to a) Afferent constriction . F a s t e s t conducting fibers in heart a) Purkinje 61. Stretch reflex maintains a) Muscle tone b) Balance c) Length d) … e) … ans c 57. g a s k i n s u m tract lesion etc dts y not remeberd 56. 58. T h e r e w a s q s i n which options were (forgot q s ) a) Pancinian corpucles b) Messiners corposccles c) Ruffinis end o r g a n s d) Golgi tendons e) …. b) Efferent constriction Ans b 62. A F B + v e a) Leprosy 67. A s e n a r i o w a s about dijoxin toxicity 63. Monocytes a) Most abundant cells in circulation b) Do not go out of vessels c) Not phagocytic d) … e) … 69. A pt with deformed nose. Treatment of acute asthma a) Terbutaline 64. Tuberculous meningitis a) Inc lymphocytes in c s f 70. Not a c a u s e of cardiogenic s h o c k a) Cardiactemponade b) Mi c) Arrythmias d) Hemorrhage e) Aortic dissection Ans d 66. 0 2 0 b) Neutrophils c) Protein >2g| dl d) . . Most imp point of exudative inflamation a) Sp gravity < 1 . e) … ans c 68. Isoniazid side effects can be prevented by a) Pyridoxine 65. Which is not covered by deep cervical f a s c i a a ) Thyroid b) Parotid c) Sternocleidomastoid d) Submandibular gland . B e l l y 78. B e l l y o f digastric.Bladder ca caused by a) Schistosomahaematobiam 71. a) Amount of air left in the lung after a max exhalation 74. ESTROGENOCPscancause a) Breast ca b) Ovary ca c) Endometrial ca Ans a 73. Residual vol. Vital capacity a ) After a m a x inspiration. Cardiacoutput a ) M e a s u r e d b y fick principle 76. ( R e m e m b r that s u p g l a n d s hv more constant position while I N F r variable in position. posteriorly b y post. Which tumor does not hav bone metastasis? a ) Thyroid b) Liver c) Lung d) Breast e) Kidney Ans b 72. Digastric triangle a ) B o u n d e d anteriorly ant.that’s y dy askd about s u p g l a n d s location. R eg ar di ngs u p parathyroidgland a) Located in c l o s e proximity to I N F thyroid artery and recurrent laryngeal nerve at the level of cricothyroid junction. m a x expired v o l u m e i s v c 75.its imp) 77. why a) Dnt remember choices. aduction a n d abduction 80.e) Sublingual Ans e 79. Pt having uppr respiratory tract INF. Diagnosis of pneumothorax? a) Xray chest 83. T h e r e s a l s o a s a c r a l micturation centre but it ws not in the choices 82. Approach to intercostal s p a c e for pleural effusion drainage a) Lower part of space (neurovascular bundle in upper part os S P A C E but lower part of RIB) 84. In nephron fluid which s u b s t a n c e c o n c is higher than p l a s m a a) Na b) Bicarbonate c) Glucose d) Urea e) Albumin Ans d 86. R e n a l p l a sm a flow is determined by a) PAH . sory B) C) D) 81. F o r surgery tibia should b aproached from medial side. then c h e s t pain that relieved by sitting up a) Pericarditis 85. extension. Micturation reflex centre is located in a) B r a i n s t e m (pontine micturation centre). Circumduction a) C o m b i n a t i o n of flexion. L 5 (l3-l4 a l s o correct but it ws not in choices.lymph. T i s s u e for H L A typing (there were two s a m e qs in e a c h paper) a) Buccal mucosa b) Leukocytes c) Skin d) B o n e marrowAns b 92. Caudaequina a) Collection of nerve roots and rootlets 94 Pilocarpine is nicotinic a) Agoist b) Antagonist 95 D i a g n o s i s of klinefeltrs syndrome . blood s upl y.87. WHICHVACCINEISNOTLIVE A) Measles B ) Mumps C) Rubella D) Yellow fevr E ) Tetanus Ans e 91. L P done a t a) L 4 . Dnt re mem be re xa c t l y 88. type of hypersensitivity reaction? a) 2 90. thanks A L L A H Dy did NT gv both c h o i c e s simultaneously :-) 93. diarrhea 1 0 d a y s after bone marrow transplant a) Graft vs host d i s e a s e 89 Erythroblastosis fetalis.n er ve sup. WHATISTRUEABOUTUPPERHALFOFANALCANAL A) Op ti o ns wr about epithelium. Jaundice. hepatitis. a) Barr body 96 W h i c h drug interers w i t h warfarrin a) Cemetidine 97 Adverse effect of chlorpromazine a) Extrapyrimidal 98 Mechanism of action of captopril a ) A c e inhibitor 99 Feature of ca a) Plomorphisim b) Nuclear ratio c) Hyperplasia d) Invasion Ans d 100 Most cancers hav which fillament in them a ) Kerratin b) Desmin c) Vimentin d) Neurofibrilary e) … ans a 101 Paraneoplastic syndrome is associated with a ) C a breast b) Adeno ca lung c) S m a l l cell ca lung d ) C a prostate Ans C 102 Hepatitis b I N F i s monitored b y a)Sur faceantigen . M 15gm. which test to perform a) HepAndBVIRUS b) Bilirubin c) Sgpt d) Albumin Ans c 104 Which hormone inhibits insulin secretion? a) Secretin b) Cck c) Somatostatin d) Glucagan e) Vip Ans c ( A s a general rule all git hormone stimulate insulin release and knwn as I N C R E T I N ) 105 MOSTIMPSTRESSHORMONE(itwasalsoinlastpart1) a) Epinephrine b) Insulin c) Growth d) Cortisol e) Acth Ans d 106 InC us hi ngsyndrome a) Neutrophils are Inc. M. nt exactly remembrd) a) T h e y h a v igA. g I g A 5gm. G in different ratios and % a g e s e. G 20gm b) IgM: igG S 1:20 109 H L A type in RA a)HLADR4 . lymphocytes and eiosiniphils are D e c 107 Asimplesenarioofgr ave sdisease 108 Whats pattern of antibodies in multiple myeloma (options were vry weird.b) Srfa ceantibody c) C o r e antibody d) E antibody 103 Hepatitis A inf. google. photosensitivity (these symptoms wr in the form of a simple senario).110 ABSOLOUTELYMPHOCYTOSISNOTPRESENTIN a) Sle b) Inf. Mononeucleosis c) Walders (sumthin like that) granulomatosis d) … e) … ans a (nt sure) 111 Pt with arithritis. W A T S diagnosis a) S L E 112 Platelets contraindicated in a) Splenomegaly b) Acute immune thrombocytopenia c) Coagulopathy d) Dic Ans b 113 Water m o v e s through a) Pores b) Membrane matrix c) Proteinchannels d) Fascilitated difusion Ans A(waterchannels) 114 Fascilitated diffusion a) P a s s i v e transport trough protein c h a n n e l s or carier protiens 115 Which factor strenthen pt doc relationship a) High professional skills b) High social skills c) Highly qualified d) Logical answers to qs e) A c t i v e listening Ans E http://docs. butterfly rash.com/viewer?a=v&q=cache:B- . 90RsYl_QIJ:www. is d a m a g e d a) Fascial b) Hypoglossal c) Submandibular d) Edengerwestphal e) … ans b .org.racgp. which N.au/afp/200512/200512robinson.pdf+active+listening+for+a+doctor&hl=en& gl=pk&pid=bl&srcid=ADGEEShL3aencNeNL6PcDL75akSth1myGNOsIBbWqXMOBxP5X7QKVmiu16qKkAOUhwMcNerrFuT2x-kVNZZz84Xr5Gy1K7AJJfrUhJoOWFb7PnEI52oZ3VXT9v3ZAKmA31w_EhEa0&sig=AHIEtbTDPrtyezt44aYcvJMnrIDzau8oZw 116 Which is more a g g r e s s i v e (also in last part1) a) Ba sal cell ca b) Squmaouscellca c) Melanoma Ans b 117 Nissel bodies are a) RER b) SER c) Mitichondria d) Golgi bodies Ans a 118 Greatest area of cerebral cortex is covered by a) Elbow b) Knee c) Thumb d) Shoulder e) Ankle Ans C 119 Deviation of t o n g u e on protrusion. d i a g n o s i s a) Aortic s t e n o s i s b) Mitral stenosis c) Aortic R.120 A pt with diastolic murmur in aortic area and colapsing pulse. which will b infected a) Spleen b) Jejunum c) T r ansver secolon d) D es c en di ng colon e) … ans d 125 . Division of MA Ans A 122 C s f absorbed by a) Arachnoid villi b) Choroid plexuses Ans A 123 Which i s pierced during L P A ) D u r a matter 124 Inf s p r e a d s retroperitonealy. d) Mitral R e) … ans c 121 SUB-DURALHEMATOMACAUSE A) SUPCEREBRALVEIN B) INFCEREBRALVEIN C) A N T DIVISION OF MA D) Post. b) Hemiazygous c) Rit gastric d) Lft gastric Ans D http://books. b) Optic nerve c) Optic tract d) Optic chiasma e ) Pretectal a r e a Ans E 129 Part of portal system draining e s o p h a g u s a) Azygous v.google.com. light reflex absent.Ant pituitary l o s s will result in D e c in s i z e of a) Z o n a glomerulosa b) Z o n a fasiculata c) Parafolicular cells of thyroid d) Adrenal medulla e) … ans b 126 Factor D e c wound healing a) Vit c deficiency 127 How thrombocytes play role in thrombosis a)Sor ryforgotnthec hoi ces 128 A c c o m o a t i o n reflex intact. D a m a g e to? a) E d e nge rwestphal n.pk/books?id=21e4wArL7hQC&pg=PA18&dq=portal+system+draining+esophag us+is&hl=en&ei=N7w0TNLEDKjonQefitGDBA&sa=X&oi=book_result&ct=result&resnum=1&ved=0CCkQ6 AEwAA#v=onepage&q&f=false 130 Bronchopulmonary s e g m e n t a) Anatomical and functional unit iof lung 131 . actualy dy askd that which factor shouldnotbthereinaGENERALHEALTHYDONOR) a) Age b) Sex c) Systemic disease 135 Which one is not a epithelium tumor a) Adenocarcinoma b) Sq ca c) Liposarcoma Ans C 136 Which receptor r involvd in acid production a) H1 b) H2 c) Acetylcholine d) … e) … ans b 137 Which is present in slow wave sleep .Temp set point is in a) Ant hypothalamus b) P o s t hypothalamus 132 Antibodies are produced by a) Plasma cells 133 Virulence of bacteria is associated with a) Dose b) Duration of exposure c) Toxin production d) B o d y resistence e) … ans c 134 For a blood donor what is not required (choices not well remembrd. 139 A child having d y s p n e a when lyng down a) Retrosternal goiter 140 Best buffer of body a ) Protein b) Hb c) Hco3 d) Phosphate Ans C 141 Protein utilization i s c h e k d b y a) Urinary N O 2 b) Blood N O 2 Ans A 142 A pt h a s fluid l o s s . It’s d u e to a) Aldosterone b) Adh c) Acth d) Cortisol Ans B 143 Ph 7 . 4 po2 65 pco2 33 hco3 19 .a) Dopa b) Acetylcholine c) Serotonin d) Norepinephrine e) … ans c and d 138 Hemibalismus due to damage to a) Subthalamic N. now h i s urine is concentrated. 3. d a m a g e to (qs nt remembrd well) a) S2. 3 c) L2. 4.s1 149 Hypospedias is due to defect in .4 b) S2.a) Metabolicaci d osis b) Metabolic alkalosis c) Compensatedrespalkalosis d) R e s p alkalosis 144 What h a p p e n s on mountain aclimitization? a ) Ventilation i n c 145 S y p h i l d i g n o s i s material taken from a) Blood b) Urine c) Semen d) Genital s o r e s (lesion) e) Saliva Ans D 146 Best indicator of serum iron stores a) S e ru miron b) S e r u m ferritin c) Hemosiderin d) TIBC Ans B 147 Why more oxygen g o e s to alveoli at a p e x than at b a s e ? a) Inc compliance b) Inc blood flow c) In v\q d) Inc pulmonary pressure Ans A 148 Urinary incontinence with overflow and e x c e s s i v e voiding. about its inc or dec in specific condition 154 Inc in systemic filling preesure c a u s e s a) Inc v e n o u s return 155 Beta-blockers do not c a u s e a ) I n c renin . To biceps d) E n d s in axilla Ans C 151 Upper part of anal canal is a ) L i n e d b y stratified s q epi b) Drained by s u p inguinal n o d e s c) Drained by inf rectal vein d) Sensitive to touch e) Sensitive to pain 152 Normal fetal heart rate a) 40…. c) Lat. To radial a. 60 b) 60…. 160 e) 160…. 200 Ans D 153 One qs was about CVP. 120 d) 120…. 100 c) 100….a ) Urogenital tubercle b) Urogenital fold c) Urachus Ans B 150 Cephalicvein a) B e g i n s in anatomical snuff box b) L i e s lat. 4 Ans A 159 In which parasite the respiratory symptoms predominate? a) Ascaris b) Cystecercosis c) … . n o complication.b) Vasoconstriction c) Bronchiodilation d) … e) … ans a 156 Lidocaine a ) Inc P R interval b) Dec A P c ) D e c automaticity Ans A 157 A d i a b e t i c pt w i t h BS 20mmole\l .cause a) Cerebellar lesion b) Upper motor neuron c) Lower motor neuron d) S2. 3. W h a t s h o l d be the treatment? a ) Insulin b) Insulin + sulphonylurea c) Sulphonylurea d) Biguanides e) Biguanides+sulphonylureaAns D 158 Pendulouskneejerk.its 1 s t visit to a doc. o b e s e . d) … e) … 160 D i a g n o s i s of H.influnzae meningitis a ) C s f culture b) B l o o d culture c) Inc lymphocyte in c s f d) … e) … an a 161 Antibody to TB bacteria is (dnt remember exact choices but they did a s k regarding antibody against T B ) a) Cell bound b) Membrane bound c) inplasma d) In saliva e) … 162 In which p h a s e os cell dividion the c h r o m o s o m e s are arranged in chromatids a) Prophase b) Metaphase c) Anaphase d) Telophase 163 Atrial contraction c o r e s p o n d s to a) C wave b) P wave c) Qrs 164 Dorsal roots supply which m u s s c l e s ? a) Internal intercostal b) Externat intercostal c) Innermost intercostal d) S o m e back musscle 165 Internal s pe rma ti c f as ci ais derivedfrom a) External oblique . 1 . skin is calm and cold due to a) Vasoconstriction b) Vasodilation 170 Most common manifestation of septic pt a) Tachycardia b) Hypotension c) IncGFRAns B 171 Premalignant condition a) Basalcellnevus b) Dysplastic nevus syndrome Ans B 172 Tumor involving lymph v e s s e l s a) Angiosarcoma b) CystichygromaAns . 5 L blood loss.b) Internal oblique c) Transvesalis fascia d) Transversus abdominus Ans C 166 Inputs to cerebellum a) From vestibular system 167 Resolving power of lens a) That e n a b l e s to s e e closely related subjetcs seperatly 168 Parasympathetic stimulation c a u s e s a) Ciliary m u s c l e contraction 169 R T A . It’s due to a) Compressionofopticnerve b) Damage to optic tract c) Damage to optic radiation d) Compression of upper part of optic chiasma e) Compression of oculomotor n. Ans D 174 What u find in primary hyperaldosteronism a) Inc k b) Deck c) Hypotension d) Inc renin e) Inc angiotensin 2 Ans B 175 Which take part in synthesis of aldosterone a) Angiotensin 1 b) Angiotensin 2 c) Cortisol d) … e) … 176 Inckcauses a) Inc aldosterone b) Inc ADH c ) D e c aldosterone d) D e c ADH 177 HYDROCHLORTHIAZIDECAUSES A) Inc k B) Inc ca C ) Inc mg D) Inc na .B 173 A female having very high levels of prolactin. suffering from homonymous hemianopia. 178 Hydrops fetalis o c c u r s to RH –ve mother and RH + v e father. which is pivot joint temporomandicular atlanto occpital Atlanto axial 3. Captopril causes hypokalemia .Corneal opacities r caused by ethambutol phenothiazides 5. H+ ion secretion in the kidney causes bicarbonate reabsobtion bicarbonate excretion 2. What type of hypersensitivity reaction occurred? a) 1 b) 2 c) 3 d) 4 e) 5 179 Blood supply of eye except c o n e s and rods a) Central artery of retina 180 T h e y gv senario of klinefeltr and a s k d what u will find most common a) Gynecomastia 181 Aldosterone c a u s e s a) Hyperkalemia b) Hyponatremia c) Hypercalcemia d) Hypokalemia e) Acidosis 1. Drug used for mountain sickness acetazolamide scopolamine 4. Esophago gastric junction competence is maintained by lying supine incresed intra abdominal pressure diaphragm paralysis use of metoclopramadie use of morphine 11. Ph =7.hyper kalemia 6. Which is P450 inducer isoniazid phenobarbitone cometidine ketoconazole 8.5 HCO3= 48 PCO2= 45 diagnosis is metabolic alkalosis respiratory alkalosis partially compensated metabolic alkalosis partially compensated respiratory alkalosis 7. Patient has acute blood loss the mechanism triggered imediately is baro receptor mechanism renin angiotensin system 12. Carotid body receptors respond to arterial PO2 arterail Pco2 13. Which causes decreased gastric motility Magnesium Sulphate Aluminium Hydroxide 9. which is increased in first responce . propylthyouracil acts by decreasing iodine uptake decreasing TSH levels decreasing thyro globulin levels 14. Serum gastrin levels are incresed by prolonged use of antacids PPI H2 blockers anticholinergics beta blockers 10. which crosses placenta Ig G Ig M Ig A Ig E 16. which has memory function Ig G Ig M Ig A Ig E 18. ejaculation is caused by sympathetic parasypathetic both 22. Alpha receptors stimulation causes pupilaary dilation increased heart rate increased contractility . which drug causes abnormal differentiation between red and green colours ethambutol amiodarone pyrazinamide 19. which is secreted in breast milk Ig G Ig A Ig M 17.Ig G Ig M Ig A 15. for best verbal communication skill one should hav good vacabulary competence in presentation fluent speech 20. which is not caused by parasympathelic stimulation increased GIT motility micturation miosis sweating 21. patient after ischemic attack has ventricular tachycardia. patients ecg shows increasing PR intervals and then a missed beat first degree block sinus arrythmia Mobitz type 1 block Mobitz type 2 block 3rd degree block 28.23. action of gluteus medius n minimus is abduction n medial rotation abduction n lateral rotation adduction n medial rotation adduction n lateral rotation 25. young patient has bruises on the body and no other significant history n examination unremarkable. Digoxin is drug of choice in Atrial flutter Atrial Fibrillation Ventricular Tachycardia Bradycardia 29. patinet cant raise arm and sensory loss on lateral surface of arm. apex beat is 4th left intercostal space mid clavicular line 8cm to the left of midline in 5th intercostal space 31. bromocriptine causes decresed prolaction levels by acting on dopamine receptors cholinergic receptors 27. fracture of surgical neck of humerus. damage to axillary nerve musculo cutaneous nerve 24. he has idiopathic thrombocytopenic purpura aplastic anemia . drug that must b used is lidocaine amiodarone verapamil 30. thymus is formed by 3rd pharangeal pouch 4th pharangeal pouch 26. which is not oncogenic Hep B virus Hep D virus Herpes Simplex virus EBV 39. Middle meingeal artery passes through foraman spinosum foraman lacerum foraman rotundum foraman ovale 40.Hemophilia 32. which investigation to be done in patient with hemophilia PT APTT bleeding time complete blood count 34. patient has hemoptysis and glomerulonephritis. P falciparum causes black water fever 36. diagnosis is good pasture syndrome wegeners granulomatosis 35. Hemophilia is X linked recessive Autosomal dominant Autosomal recessive 33. damage to middle meningeal artery causes hematoma formation between duramater n calvaria duramater n arachnoid mater arachnoid mater n piamater . C perferingens acts by producine lecithinase depleting ATP causing hypoxia 37. Gonococcus in easily identified in exudate specimen by gram staining Z N staining culture 38. germ cells r derived from ectoderm endoderm mesoderm 55. VII. which lobe has only 2 brocho pulmonary segments right middle right upper left upper left middle 57. which is thick filament actin myosin 59. patients right eye has moved upward n has his neck tilted to left to avoid diplopia. which is present in cavernus sinus abducent nerve 51. which cranial nerves r parasympathetic III. patient has homonymous hemianopia. X 52.50. lesion is at optic nerve optic tract retina optic chiasma 54. IX. foreign body in trachea goes in which bronchus right upper right middle right inferior left inferior 56. a sacromere is present between 2 Z lines . during iso volumetric contraction atrial pressure decreases semilunar valves r open coronary blood flow decreses first heart sound is produced second heart sound is produced 58. damage to superior oblique inferiour oblique 53. after Rh incompatabilty what is given to the mother to prevent it next time anti D antibodies 69. patient has blood group A+ which cannot b given to him O+ O- . in muscle cells calcium is released from endoplasmic reticulum sarco plasmic reticulum 61. after renal transplant which malignancy is common ??? 64 babinski sign is positive in pyramidal lesion cerebellar lesion 65.after normal saline infusion blood volume increases Na excretion decreases osmolality increases renin secretion increases 63. ventral spino thalamic tract ends at lumbar region mid thoracic end thoracic 66. commonest site of lumbar puncture is L4-L5 L2-L3 S1S2 67. in kidney select the best order of arteries renal > arcuate>interlobar>interlobular>efferent arteriole renal>interlobular>interlobar>arcuate>efferent arteriole renal>interlobar>interlobular>arcuate>afferent arteriole renal>arcuate>interlobular>interlobar>afferent arteriole 62. which is not punctured during lumbar puncture duramater ligamentum flavum longitudinal spinal ligament 68.H band A band 60. DR4 is associated with rheumatoid arthritis SLE scleroderma 77. medullary rays r present is cortex medulla 72. which causes dysphagia ankylosing spondylitis scleroderma dermatomyositis 79. which is essential amino acid phenyal alanine 74. patients had gastrectomy . which is present in SLE anti DNA antibodies anti jones antibodies anti smith antibodies 78. in Addisons disease what does not occur hyper kalemia hypokalemia 73. HLA.A-AB-70. after 3 months she presents with iron deficiciency anemia . which is teratogenic alcohol 75. iron deficincy anemia is hypochromic microcytic 80. which is associated with cancer methyl alcohol propyl alcohol 76. hamartoma is benign tumor malignat tumor 71. and Hb 8. turner syndrome has karyotype 45 XO 45 XX 45 XXy 45 XXX 89. which muscles divied submandicular gland in to super ficial n deep part omohyoid mylohyoid anterior belly of digastric .1 g/dl. what arches in front of the root of left lung Arch of aorta Azygus vein 84. RCA supplies right atrium only right atrium n right ventricle 87. how will the effect of warfarin immediatelty reversed vitamin K FFP protamine sulphate 86. tunica vaginalis is derived from transversalis muscle transversalis fascia peritoneum inter oblique muscle 88. she is suffering from iron deficiency anemia megaloblastic anemia 82. which is tributary of portal vein superior rectal inferior rectal 83. selective beta blockers do not cause broncho spasm 85. pregnant lady has MCV of 70 fl.pernicious anemia hemolytic anemia 81. 6 96.90. which is last mediator in septic shock IL-1 IL. he has damage to right fascial nerve left fascial nerve right trigeminal nerve left trigeminal nerve 91. Dose of Gentamicin is reduced in elderly due to reduced renal function . antigen antibody reactions r most reduced in Liver failure Low neutrophils 97. a pregnat lady from hilly area presents to. fetal scan at term shows no anomaly. patient has difficulty closin right eye and distorted facial appearance. Clavulanic acid destroys B lactamases causes decreased penicillin excretion 98. what is end product of glucose metabolism in the presence of oxygen lactic acid 1 molecule of pyruvate 2 molecule of pyruvate 94. which of the following can still b present in the newborn ASD VSD PDA pulmonary stenosis 93. which does not cause thenar muscle wasting carpel tunnel syndrome C8 neuritis cervical rib scalene muscle spasm cervical spondylosis 92. beetle chewing causes submucuous fibrosis leukoplakia keratosis 95. GFR can b best estimated by creatinine clearance serum urea levels serum creatinine levels 100. in congestive cardiac failure there is incresed pressure in venous system 109. diffusion through alveoli is directly proportion to thickness of membrane total cross sectional area 106. best test for diabetic nephropathy is serum urea levels serum creatinine levels urinary albumin 101.99. which does not has lymphoid follicles speen thymus lymphnodes payer patches 108. which enzyme could b most likely raised Alkaline Phosphatase LDH ALT 105. ventricular contraction causes which wave in JVP . which protein maintains red cell shape integrin secretin spectrin 102. patient has gall stones . 103 which of the following causes natriuresis stretch receptors in atria carotid bodies 104. Cyanosis is due to increased deoxy hemoglobin in blood decresed PO2 in blood. transpot of materials by carrier proteins is example of simple diffusion facilitated diffusion active transport 107. lacrimal duct opens in inferior meatus middle meatus 119. which is not part of limbic system vermis of cerebellum amygdala anterior thalamus hippo campus 112. behaviour sciences is branch of science which deals with behaviour of persons in different social . thiazide diuretics donot require potassium supplemets 114. site for venesection great sephanous short sephanous 111. cerebral cortex is concerned with voluntary movements of body 117.a c v 110. in asthma FEV1/FEV less than 65% FVC is reduced 120 6 years old girl has meningitis. lesion is right hemisection of spinal cord left hemisection of spinal cord 116. right gastric artery is branch of gastro duadenal right gastro epiploic 113. pateint has loss of proprioception on right n loss of pain n temperature on left. most common organis is streptocossus pneomonie Neiserria meningitidis Hemophilus Influenzae E Coli . DVT is causes by stasis of blood 118. psychological conditions 115. 010 122. Lambda chain is present in amyloidosis 124. diagnosis is TB leprosy sarcoidosis syphilus 123. exudate has protein more than 3g/dl specific gravity is less than 1. young boy has bilaterla parotid enlargement. lymph node biopsy shows non caseating granuloma and sputum AFB is positive. which is false causes disease only in humans favors aerobic conditions 128.121.about Mycobacterium TB . cholangio carcinoma is caused by C Sinensis paragonimus watermani taenia solium 126. fetal brain development is caused by Growth hormone thyroid hormone 130. gastric Acid increases most after taking . which is false about spores produced under nutrient rich conditions B Anthracis producs spores C tetani produes spores they r killed under 121 degree for 15 min in autoclave 129. which does not follow oro fecal route tenia solium trichuris trichuria entrobius vermicularis Ankylostroma duodenale 125. diagnosis bacterial parotitis mumps infectious mononuclosus 127. a young boy has distorted nose and cervical lymphadenopathy. carbohydrate protein fats 131. which hormone effects carbohydrate, protein and fat metabolism insulin Growth Hormone 132. which is not an epithelial tumor sq cell CA adeno carcinoma sarcoma 133. Bence jones protein are produced in multipla myeloma 134. which hormone effects both nor epinephrine n serotonin metabolism Mono amine oxidase 135. patient presents with epistaxis and bruising on body. best invertigation is complete blood count PT Aptt 136. ADH acts on Proximal tubule Distal Tubule Collecting Tubule 137. which is most slow growing malignancy of thyroid Follicular CA Medullary Papillary Anaplastic 138. Cerebellar lesion causes Ataxia 139. patient with cerebellar lesion cannot touch objects with his finger tip 140. PTH remains normal in renal insufficiency osteoporosis 141. Neonatal Rubella infection is screened by measuring which antibody? IgM IgG IgM and IgG also 6 -7 questions about aldosterone and blood pressure control Posted by Dr Sultan Ahmad 5 comments Reactions: GYNAE OBS JUNE 2009 Gyn Obs JUNE 2009. Recalled By Syeda. 1-Etiology of cleft palate. a-anticonvulsant bmultiple preg cmultifactorial d-inc maternal age 2-left cleft palat dt malformation of., a-palatine shelves bmaxillary prominence cmed nasal prominence dlat nasal prominence eincisive foramen 3-germ cells derived from., a-ectoderm bendoderm cmesoderm d-neural crest cell 4-karyotype of turner syndrom,, remembr) a-45XO ,,,,(smethng like dat exactly nt 5-which 1 contain granulosa cell,, a-graffian follicle b,,,...,.., 6-in primary infertility,,diagnosis of failure of ovulation by ?? a-wid lower abdominal pain,,..,, b-rise in basal body temp c-middle of menstual cycle,,, d-thick mucosa sec frm cx wch nt show fern pattern.... 7-pentamer immunoglobulin is??? a-IgA B-IgG c-IgM* d-IgD e-IgE 8-Regarding entameba histplytica.... a-it is a nematode b-it causes liver disease c-occurs in both trophozoite nd cyst stage d-it exist normally in intestine e-..,.,.,.,., 9-cornybacterium diphtheria..,., a-gm -ve bacilli b-club shaped* c-,,..,.,. 10-mode of cell injury by clostridium a-ATP depletion. b-.,.,.,. 11-shigella.. endotoxemia neurotoxin verotoxin enterotoxin exotoxin 12-corneal opacity caused by.. ethambutol quinine phenothiazine chloroquine ,,,,,??? .,,.,.,.,.., 13-drug inducer by p450 mechanism,.. cimitidine ketokonazole phenobarbitone ,., ,,,,,,,.... ,.,.,.,.,.,.,.,.,. 14-last mediator of endotoxic shock...,.., IL1 IL6 TNF ALPHA LIPOPOLYSACHARRIDE 15-yellow fever inf occurs ,.,.,..,.,.,., endemic sporadiac pandemic epidemic .,.,.,.,..,.,., 16-wall of inguinal canal Not formed by??????? ext oblique apponeurosis lacunar lig internal oblique conjoint tendon transversalis fascia 17-endocervical polyp in pregnancy,,, Physiological .,.,.,.,.,.,.,.,.,… 18-vaccination of mycobacterium T.B based on Type I hypersensitivity Type II Type III TYPE IV Cell mediated 19-after subdivided no of peoples into gps ,,sampling is taken a/c to age ….wat type of sampling it is Random sampling (other options nt remember, ,,,stratified etc ,,,,,nt sure) 20-ultrasound.. 4-8 weeks transabdominal u/s,, fetal heart sounds can detected Cresenteric line seen w/h cnfirm pregnancy,,,,(smething like dat ) 21-MRI Frequencies,,,,,,,,,,,, ,nd methaemoglobin.,,,,,,,,,,, (these sort of alien words were written) 22-bone scan ,,,,,, (name of sme disesases were written) 23-incidence,,,., No. of new cases in a population in a time given 24-pie chart 25-wch 1 is absent in muscles Glucose 6 po4ase Hexokinase 26-end product of glucose metabolism Lactic acid 1molecule of pyruvic acid 2 molecule of pyruvic acid 27-hormone ass with glucose ,fats ,and protein storage Insulin 28-fetal brain development Growth hormone Thyroid hormone 29-w/h of the following amino acid should be taken in diet Alanine Tyrosine Glycine’glutamine 30-folllowing phagocytic cell present in circulation? Monocyte Macrophage Lymphocyte Neutrophils 31-morphine can be given in Biliary cirrhosis Terminal ill patient of cancer Acute pancreatitis 32-functional layer of endometrium consists of Stratum basal Stratum compactum nd stratum spongiosum Stratum basal nd compactum 33-maldevelopment of uro-rectal septum ,, Rectovag fistula Rectouterine fistula Imperforate anus Rectoperineal fistula 34-structure lying b/w labium majus nd minus Urethral gland Bulbourethral gland Minor urethral gland 35-w/c 1 of following muscle not attached to tendon of perineal body Ischiocavernosus Ext urethral sphincter Bulbospongiosus Transverse perineal muscle 36-lymphoide nodule nt present in Thymus Spleen Lymphnode 37-Notochord forms Neural tube Cns 38-epithelium of ovary Simple cuboidal 39-pivot jt include Atlanto occipital jt Atlanto axial jt 40-sacroiliac jt Fibrous .,. Synovial.,., ,,.,.,..,.,.,.,. 41-hamartoma is.,.,.,.,.,., 42-an organ lined by stratified sq epi with inc cellularity, vascularity, nd elasticity,,tubular glands in lamina propria Vagina Esophagus Oropharynx Duodenum 43-post relation of left ureter Root of mesentry Iliac vessel 44-outer lat quadrant of mamry gland drain into Ant gp axillary lymph node 45-xerophthalmia dt Vit A def 46-anti-oxidant Vit E 47-w/h st having 3 layers of muscles Fundus of stomach Esophagus 48-fat embolism 49-regarding mammary tissues w/h is Not appropriate Pect major, seratus ant,lie in deep relations Can mov freely over retromammary space Peau’d orange caused by lymphatic obstruction.,.,.,.,.,.,.,.,. Puckering of skin dt subcut ductal infiltration.,.,.,.,.,.,.,.,. 50-lady died dt amniotic fluid embolism During delivery During c/s nd post partum After post partum During labour,delivery nd postpartum 51-pacinians corpuscles Tactile touch Fine touch Temp Pain Vibration(I think it was nt in option nt sure) 52-H+ exchange in proximal tubules with HCO3 reabsorption K secretion 53-pat wid severly vomiting came wid breathlessness,,,PH=7.5,,HCO3=INC Metabolic alkalosis Metabolic acidosis Compensatory metabolic alkalosis 54-uterovag prolapse w/h lig is injured Broad lig Round lig Transverse cx lig 55-alkaline po4ase rises in Prostatic ca BPH Pancreatic ca ,,,PCO2=55 56-in ca mammary gland w/h drug is given in post menopausal lady Tamoxifen Clomiphen .,.,.,.,.,.,.,.,. 57-in left shift O2 hbdissociation curve w/h 1 is increase Fetal hb 58-in thalassemia there is Poor hemoglobinization Dyserythropoiesis ,.,.,.,.,.,.,,.,.,.. 59-intravascular haemolysis diagnosed by Reticulocytosis Inc in hepatoglobulin binding Inc TIBC ,.,.,.,.,.,.,.,.,…….. 60-lady wid reddish cheeks, ,,arthritis,,,,, SLE 61-in metaplasia w/h is inappropriate Irreversible .,,,,,,,,,,,,,. 62-acute inflammation Transudate Exudate Inc protein content 63-w/h drug causing minimal GI side effects Paracetamol Aspirin Ibuprofen Ketorolac Diclofenac 64-in 6 yrs old child meningitis occurs dt w/h organism H-Infuenza Ecoli Niesseria 65-sepsis is diagnosed most appropriately by Blood culture Granulocyte <2000--->15000 H.R >100 HR>,.,.,…, 66-w/h is transmitted by mosquito Yellow fever ..,.,.,.,.,.,.,.,., 67-virus not associated with ca in human HSV HBV HDV HPV Papova virus(papilloma vitus +polyoma virus) 68-w/h is more in human milk as compare to cow’s Lactulose 69-gonococcus is best easily diagnosed by Gram staining Culture ZN stain 70-w/h is completely covered by pretracheal fascia Thyroid gland 71-is not an embryonic remanant Lat umbilical ligament 72-oxytocin nd vasopressin originate frm Hypothalamus Neyrohypophysin Autonomic ganglion 73-sucrose Is a diasaccride Metabolized into 2 glucose molecules Reducing or non reducing (nt remember) 74-w/h of the following dis not ass with carcinoma Lysosomal storage dis .,.,.,.,.,.,.,.,.,.,.,.,.,.,.,.,.,. 75-rough endoplasmic reticulum Protein synthesis 76-fragility of rbcs membrane Spectrin 77-behavioural sciences .definition 78-verbal communication skill Speak fluently Education nd research training Presentation Good vocabulary 79-Not a function of beta blockers -ve ionotropic -ve chronotropic .,.,.,.,.,.,,. .,.,.,.,.,.,.,.,.,. 80- alpha blockers 81-centrally acting HTNsive Clonidine Methyldopa .,.,.,.,.,.,.,.,.,. 82-w/h 1 decrease the muscle length Golgi tendon organ Muscle spindle .,,,,,,,,,,,,,,,,,,,,,,, 83-cerebellar lesion, Ataxia Rigidity Initiate muscle movement 84-pre-motor cortex lie in Frontal lobe 85-coccygeal part of spinal cord lie at the level of w/h vertebra C2 L2 S1 S2 86-lie in true pelvis nd crosses frm med to lat side of pelvis Sup rectal art Middle rectal art Inf rect art Internal iliac 87-inf fascia of urogenital diaghphram also called as Perineal membrane 88-female pelvis also called as Android Anthropoid Platypelloid Mesatipellic 89-at crowning w/h muscle is cut during episotomy Bulbospongiosis 90-lumbar puncture L2 L3 L3 L4 L4 L5 S1 S2 91-Lactation,,,,,, Hormones,,,wh stimulate 92-prolactin hormone Stimulated by,.,.,.,.,.,. 93-pregnancy is diagnosed by Gonadotrophin hormone 94-urogenital diagphram inserted on Ishchial tuberosity Ischial spine Sub pubic arch Ischial ramus 95-w/h muscle div submandibular gland into superficial nd deep part Mylohyoid Ant belly of diagastric 96-root of left lung Azygous vein arched at root of left lung. 97-teratogenic Alcohol Caffeiene 98-carcinogenic Methyl alcohol Benzidine 99-pat with blood gp A should nt given A+ A_ O AB 100-. Regarding kidney ,w/h sequence is correct renal art > arcuate>interlobar>interlobular>glomerular>efferent arteriole renal>interlobular>interlobar>arcuate>efferent arteriole>glomerular renal>glomerular>interlobar>interlobular>arcuate>afferent arteriole renal>arcuate>interlobular>glomerular>interlobar>afferent arteriole 101-in muscle ca+2 release frm sarcoplasmic reticulum T-tubule Sarcomere 102-sarcomere b/w 2 z line 103-parasympathetic nerves r 3, 7, 9, 10 104-w/c is present in de cavity of cavernous sinus Abducent nerv Trochlear Occulomotor 105-in haemophilia PT INC APTT INC Inc bleeding time 106-gestational age estimated frm.,.,.,.,. Abd circumference.,,. Biparital diameter.,.,.,. wks Crown rump length ,,, 8 weeks. 107-norepi + serotonin degraded by MAO 108-at last trimester w/c uterus is sensitive to w/c hormone Oxytocin 109-to prevent heat loss Shivering Vasoconstriction 110-inc K Aldosterone sec Posted by Dr Sultan Ahmad 4 comments Reactions: SURGERY June 2009 SURGERY JUNE 2009. Recalled By Safwan. Thankyou. (C) Means correct. 1. Gland of cloquet is located in: a. inguinal canal 2. Below is a carcinogen: a. methyl alcohol b. propyl alcohol c. benzidine (C) 3. Incidence is: a. number of new cases in a population (C) 4. Following is correct about femur: a. the head is in line of medial condyle b. the angle between the neck and shaft is average 125 degrees. (C) 5. Subclavian artery passes: a. above the 1st rib (C) b. below the 1st rib 6. Abdominal angina is caused due to: a. Superior mesenteric artery (C) b. inferior mesenteric artery c. celiac trunk d. abdominal aorta 7. Left Suprarenal vein drains into: a. inferior vena cava b. left renal vein (C) c. phrenic vein d. portal vein 8. Squamous cell carcinoma of urinary bladder is caused due to: a. shistosoma (C) 9. The species which causes carcinoma in urinary bladder is: a. shistosoma mansoni b. schitosoma hematobium (C) 10. Oxidative breakdown of glucose during glycolysis generates: a. 1 pyruvate b. 2 pyruvate (C) c. lactic acid 11. Referred pain to shoulder is via: a. C1, 2 b . C2, 3 c. C3, 4, 5 (C) d. C5, 6 12. Following is not palpable on per rectal examination: a. Ureter b. Prostate c. Seminal vesicle d. ?? e. ?? 13. Spinal cord ends at: a. S2 b. L1-2 (C) c. L3 d. L5 14. Femoral sheath is composed of: a. transversus abdominis fascia b. internal oblique fascia c. iliacus fascia (C) 15. Fracture at neck of fibula: a. injury to common peroneal nerve (C) b. superficial pernoeal nerve c. deep perneal nerve d. tibial nerve 16. Posterior displacement of knee is prevented by: a. Anterior cruciate ligament b. Posterior cruciate ligament (C) c. Tibial collateral ligament d. fibular collateral ligament 17. A patient had cut in the center of wrist during a suicide attempt, he has no abduction or adduction of fingers plus no abduction of the thumb, the injury is to: a. ulnar nerve b. median nerve c. radial nerve 18. Lymphatic supply of mammary glands is: a. parasternal lymph nodes b. abdominal lymph nodes c. axillary lymph nodes (C) 19. Defecation is because of: a. mass reflex b. myenteric reflex c. S2, 3 , 4 (C) d. Parasympathetic reflex 20. Prolonged coma is because of lesion of: a. both frontal lobes lesions (C) b. one frontal lobes lesion c. periaquaductal area lesion d. midbrain lesion 21. Retrograde amnesia: a. can be abolished via prefrontal lobotomy b. appears in amygdala lesions and propranolol therapy 22. Projection fibers of cerebral cortex travel through: a. internal capsule (C) 23. Regarding neural tube which one is correct: a. Involved in CNS development (C) b. Involved in meninges development 24. Primordial germ cells are of: a. ectodermal origin b. mesodermal (C) c. endodermal 25. Urorectal septum divides the cloaca into: a. anus and urinary bladder b. anorectal canal and urogenital sinus (C) 26. ADH and Oxytocin are released from: a. suprachaismatic b. preoptic c. adenohypophysis d. neurohypophysis (C) 27. Termination of subarachnoid space is at: a. L2 b . S2 (C) c. L5 d. S4 28. Following is true about middle meningeal artery: a. Passes through foramen spinosum (C) 29. Mass reflex is elicited by: a. decortication b. complete section of spinal cord c. lesions of cervical or thoracic spinal cord (C) 30. Medial lemniscus system mediates: a. Pain b. Temperature c. Tickling d. Fine touch (C) 31. What prevents muscles from over-stretching: a. Golgi spindles b. muscle spindles (C) 32. Hamartoma is a: a. neoplastic condition with no malignant potential b. non-neoplastic condition (C) c. completely benign d. malignant 33. In chronic hemolysis, following will deposit in tissues: a. bilirubin (C) b. biliverdin c. conjugated bilirubin d. lipofuscin e. iron 34. Premalignant lesion of mouth is: a. ?? b. ?? 35. Complication of betel nuts chewing is: a. haemorrhage ?? b. ulceration ?? ?? ?? ?? 36. Cleft palate is caused due to: a. Viral b. carbamazepine c. multifactorial (C) d. antiepileptics 37. A House surgeon is operating on posterior triangle, which structure is at risk: a. spinal portion of accessory nerve (C) b. cranial portion of accessory nerve 38. During surgery on submandibular glands, how will you protect labial branch of facial nerve: a. lift the deep fascia of hyoid bone b. nerve stimulation studies c. protecting it as it enters surgical field d. lifting the blood vessels 39. Enzyme for breakdown of serotonin and Norepinephrine is a. Catechol - o - methyltransferase b. Monoamine oxidase (C) 40. Inducer of drug metabolism is: a. ketoconazole b. phenobarbitone (C) 41. Lesions of cerebellum causes: a. failure of initiation b. resting tremor c. rigidity d. ataxia (C) 42. Essential amino acid is: a. Glycine b. glutamate c. tyrosine d. phenylalanine (C) 43. Slow growing tumour of thyroid is: a. papillary b. follicular c. adenoma 44. Following are features of thyroid tumour: a. ?? 45. Premalignant condition of colon carcinoma: a. inflammatory polyp b. hyperplastic polyp c. hamartomatous d. adenomatous (C) 46. Among the following statements about parotid gland, which is false: a. It has two lobes b. parotid duct arises from deep lobe c. retromandibular vein is related to its inferior pole d. ?? e. ?? 47. Which nerve lies in the cavernous sinus cavity: a. III, V b. IV, VI c. VI (C) d. IX 48. Muscles of facial expression are all of the following except: a. orbicularis oris b. orbicularis oculi c. buccinator d. platysma e. ?? (C) 49. Taste is sensed by all of the following except: a. fungiform b. filliform (C) c. circumvellate 50. Tympanic membrane forms: a. lateral wall of tympanic cavity 51. Thyroid gland is surrounded by which fascia: a. investing b. pretracheal (C) c. prevertebral d. deep 52. Azygous vein: ?? 53. Facilitated diffusion is: a. energy dependent b. carrier mediated (C) 54. The most significant prepotential is that of: a. atria b. ventricles c. SA node (C) 55. Best mechanism for temperature loss: a. Vasonconstriction b. thirst ?? ?? 56. Acute inflammation: a. transudate b. exudate (C) 57. alpha-adrenergic discharge leads to: a. cardioacceleration b. baroreceptor reflex 58. Which organ gets less than required oxygenation during exercise: a. Skeletal muscle (C) b. kidney c. skin d. brain e. heart 59. H+ ion secretion is in: a. hyperkalemia b. acidosis (C) c. alkalosis 60. H+ secretion is associated with: a. excretion of bicarbonate b. reabsortion of bicarbonate (C) 61. Central and peripheral regulation of respiration is via: a. PCO2 b. PO2 c. pH d. CO 62. A fisherman came to emergency department with pallor. The most probable cause for this is: a. Diphylobothrium (C) 63. Ileal resection leads to: a. Increased iron absorption b. Decreased amino acid absorption. 64. Aldosterone production is stimulated via: a. Heart failure b. Decreased perfusion of kidney c. Renin (C) d. Increased Na 65. Another use of thyroid hormone is: a. Weight loss b. Appetite c. Hair loss d. ?? e. ?? 66. Anabolic hormone is: a. Insulin (C) b. Glucagon 67. Adult Polycystic Disease is: a. Autosomal recessive b. X- linked recessive c. Autosomal dominant (C) d. X linked dominant e. Mitochondial 68. Lymphatic supply of ovary is: a. Internal iliac lymph nodes b. external iliac lymph nodes c. deep inguinal d. lumbar/paraaortic (C) e. sacral 69. Remnant of mullerian factor is: a. Duct of gartner b. Round ligament c. Appendix of testis (C) 70. Tuberculous ulcer has which type of edges: a. Everted b. Punched out c. Sliding d. Undermined (C) e. Elevated 71. Severe infection in a post-transplant patient is due to: a. Influenza virus b. Streptococcus c. Staphylococcus d. Cytomegalovirus (C) 72. Within 10 hours acute transplant rejection is due to: a. Type I b. Antibody-mediated (C) c. Type III d. Cytotoxic T cells e. Type V 73. Following is true about MAI: a. Widespread resistant to TB drugs b. Only causes lung disease c. Very common 74. Following is true about cryptorchidism: a. Failure of descent of testis from abdomen into scrotal sac. 75. Definition of Behaviour sciences is: a. Scientific study of human behaviour b. Scientific study involving epidemiology, biostatistics, ethics, psychology, sociology. (C) 76. Good communication skills are in a person who: a. Who does presentations (C) 77. RER function is: a. Protein synthesis (C) b. Transcription c. Post-translational modification 78. Following is true about sarcomere: a. Is formed between two intercalated discs b. Is formed between two Z-discs (C) 79. Calcium is released into muscles via: a. Mitochondrion b. RER c. Sarcoplasmic reticulum (C) 80. Lymphatic nodules are not present in: a. Thymus (C) b. Spleen c. Peyers patches d. Tonsils 81. In Pnemothorax following happens: a. Lung collapses and chest expands b. Lung collapses and chest indraws (C) c. Lung expands and chest expands d. lung expands and chest indraws e. lung expands and chest remains same 82. Vaccine of Bacillus-Calmette-Guerin is which type of vaccine: a. Formaline killed Toxoid vaccine b. Heat killed organism c. Live attenuated (C) 83. Vaccine of Bacillus-Calmette-Guerin induces which type of immunity: a. Active (C) b. Secondary immunity c. Passive 84. Which one is true about corynebacterium diphtheria: a. Produces powerful exotoxin (C) b. Produces powerful endotoxin 85. The most common cause of pneumonia in 6 months to 2 years age group is: a. Neisseria b. Streptococcs pneumonia c. E. Coli d. Haemophilus influenza e. Streptococcu pyogenes 86. Pseudo membranous colitis is caused by: a. Clostridium tetani b. Clostridium botulinum c. Clostridium perfringens d. Clostridium difficile 87. Clostridium perfringens causes injury via following mechanism: a. free radicals b. hypoxia (C) c. Invasion 88. Neisseria best diagnosed most easily via: a. Gram stain (C) b. Culture 89. Ocular opacity can be caused by: a. Ethambutol 90. Hydatid disease caused due to ingestion of: a. Cyst b. Eggs (C) 91. Patient is having MCV=78, MCH=28, it is: a. Normochromic normocytic anemia b. Normchromic microcytic c. hypochromic microcytic d. hypochromic normochromic 92. Patient is vomiting: ph = 7.25, PCO2=45 (35=45), HCO3=35. Cause is: a. metabolic acidosis b. metabolic alkalosis c. partially compensated metabolic alkalosis 93. Most common cause of Deep Venous Thrombosis is: a. Contraceptive pills b. Trauma c. Surgery d. Immobility (C) 94. Most common cause of hospital acquired pnuemonia is: a. Streptococcus b. Pnemococcus c. Pseudomonas (C) 95. Following is an important component of Counter current exchange mechanism: a. Vasa recta (C) b. Proximal tubule c. Distal tubule 96. Submucous glands are present in: a. Stomach b. Dudenum (C) c. Iluem d. Large intenstine 97. Patient is having bone pains, PO4 is decreased, Ca++ is raised, most probable cause is: a. Pagets disease b. Osteoporosis c. Osteopenia d. Hyperparathroidism (C) 98. Calcitonin works by: a. Increasing bone resorption. b. Decreasing bone resorption (C) XXY (C) 100. Posted by Dr Sultan Ahmad 3 comments Reactions: Saturday. X0 c. Afferent -> glomerulus -> efferent -> vasa recta Alongwith these there were few questions about premalignant conditions of mouth and a question about which is correct about cholesteatoma. stress hormone ACTH* Epinephrine Norepinephrine Growth hormone 2. September 12. XY b. 2009 RADIOLOGY JANUARY 2009. By 'Bolanian'. Sertoli cells (C) b. The vasculature of kidney is: a. Testosterone is produced by: a.99. wat is not in relation with pitutary gland Facial nerve* Abducent nerve Sphenoid sinus Trigeminal . True hermaphrodite genotype is: a. Correct answers are marked >> * << 1. Most imp. wat is not in relation with right kidney decending colon* ascending colon duodenum spleen 3. Leydig cells 101. See the june papers below the January papers. There was a Murad MCQ about pie chart. Nor-epinephrine & serotonin degration by MAO* COMT 7. tributery of external carotid vein Anterior jugular vein* Facial vein 5. what is secreted in breast milk Ig G* Ig A Ig M Ig D Ig E . wat is in between celeiac trunk and superior messentry artery Pancreas* Pancreas + jujenum Transverse colon + ilium 10. relation with left kidney Deudenum* Ascending colon Descending colon 9. wat is not in post. chemotaxic factor leukoterine B4* C3b C5 complex histamine 8.4. side effect of thophyllin nausea and vomiting* hypertension bradycardia seizures 6. . When r the ventricles filled with blood Diastasis* Atrial systole Rapid in flow 15... Spinal* Internal carotid Vertebral artery and ant cerebellar artery 13. hypotonia of the left limb. cause Atresia of CBD* Atresia of entire extra hepatic apparatus Atresia of hepatic duct Choledochal cyst 17.he is agressive in his mood. gall bladder looks ok in U/s. a new born presented with obstructive jaundice. where is the lesion . wat cannot pass placenta Ig G Ig A Ig M* Ig D Ig E 12..11. where is damage Mammillory bodies frontal lobe* temporal lobe parital lobe occipital lobe 16. he swangs to left side. A boy had behavior problem. ant and post spinal arteries arise from vertebral artery gives anterior spinal. A man with noticed weakness of left side of the body. from which foramen facial nerve enters temporal lobe internal accustic meatus* stylomastoid foramen foramen ovale foramen spinosum 14. loss of memory but resolve …. PICA gives post.. Esophagus constriction at Aortic arch* . supply to the dorsum of foot is by sephenous nerve* Sural nerve Common peroneal Deep peroneal 21. Gluteus Maximum* Gluteus medius Tensor fascia lata Quadricep femoris 22.Cerebral hemisphere Left cerebellum * superior c puduncle Red neucleus 18. patient with difficulty in standing from sitting postion... stucture compressing Esophagus Left bronchus (correct) Left Venticle Right Atrium Right ventricle Thymus 24. adrenalcortical insuficancy causes hypokelimia hyponatremis hypoglycemia* alkalosis 19. In resting stage actin is attached to tropomyocin troponin titin myosin 20. if there is damage to sciatic nerve.. wat is not in relation with left recurrent nerve SVC* Arch of aorta Lig arteriousum Trachea Esophagus 23. broncopulmonary segment wat is true anatomical. wat vein run with Post. wat is in proximal layer of carpal bone. drain in axillary ven* runs on radial side starts from palmar surface 28. fuctional n unit of the lung* contains lobar bronchus pulmonary veins run in intersegmental tissue 32. lunate triqutral & pisiform* 31. median nerve supply lateral 2 lumbricals* causes wrist drop 29. sephenous nerve wat is true drain into femoral 3.5cm inferior n lateral to pubic tubercle* . wat is true supplied by anterior division of internal iliac* separated from uterus by pouch of Douglas in males has seminal vesical above and vas deferens below 33. nerve damage Axillary nerve* Musculocutaneous Radial Ulnar 30. injury of neck of the humerus. interventricle artery great cardiac vein middle cardiac * small cardiac 26. Basalic vein . true Scaphoid.Left ventricle Rt atrium 25. urinary bladder. utrine cancer spread to labia majore via Round ligament* utrosacral ligament 27. diapharam contraction cause . diagnostic test for strptococuss is ASO titer Blood culture* Urine Dr 37. medial part of breast supply which node internal thoracic* lateral grp ant grp medial grp 42. injury to hippocampus Memory loss* 40.has only 1 tributary connected to short sphenous by perforators 34. down syndrome trisomy 21* 18 15 13 35.Which of the following tissue is most radiosensitive Skeletal muscle Cartilage* Bone 36.Erythropoietin is secreted from Yellow marrow Macula densa Preitubualr capillaries* PCT 41. 65% of Na and water reabsorbed from PCT* DCT Loop of Henle Collecting duct 38. . . sypathetic supply by alpha receptors dilator pupilary musscle of eye* brochoconstriction cardioacceleration 45.. secretioin of slivary gland* 46. wat is not supplied by internal thoracic artery Inferior portion of rectus muscle* Ant abd wall upto umbilicus Pericardium Thymus 49. A boy has edema due to protien loss of 3.. gastric emptying delayed by CCK* Secretin Gastrin Vagus 47. parasympathetic supply inc. thoracic duct passes with aorta * 44. Patient Gravida 4 with anemia and occult blood in stool Iron deficiency anemia * Sideroblastic anemia Thalassemia Megaloblastic anemia 50.increase in thoracic pressure increase in thoracic diameter 43.5g in urine. wat is the cause of edema Plasma colloid pressure is decreased* Increase hydrostatic pressure Increased capillary permeability 48. iron absorb in ferruos form* . Cerebral Anterior and pst. Trachea start from …. Cerebral 57.wat is true C6* C4 C5 Cricoid cartilage 53. premotor area . Of left coronary artery Posterior interventricular artery Main Left coronary artery Main right coronary artery 55. Bile salt absorb from Ileum* Proximal jejunum Distal jejunum doudenum 52. blood supply to Bundle of His. metastatic carcinoma least common site brain lung liver spleen(correct ) 56. arterial supply to primary visual area PCA* ACA MCA PICA . Circumflex artery a br. which artery supplies Anterior and middle cerebral (correct) Anterior Middle Middle and post. wat is true abt CSF secreted at rate of 500 ml/day* choroid villi are seen by naked eye aqueduct transmits CSF from 4th ventricle to spine 54.oxalate 51. Doctor patient relations and how good a doctor is known by his degrees his records his pleasant behavior and bed side manners* 64.20.25.58. wat the reflex gagtrogastric reflex gastro coic reflex (correct) 59. Right bronchus as compare to left is longer ant to pul artery right bronchus is more vertical than left* 62.30. Median of 20.40 25 20 27. A surgeon did a study on patients with heamorroids he included 100 pt in grpA and treated them surgicall whil kept the other 100pt on high fiber diet he interviewd them after 12mons and found that the grp with surgical treatment had a better response…… wat type of study is this? Case control Cohort Prospective Randomized control trial 63.35. if a patient develops a fatal disease it is best to hide it from him tell it to him and his family as soon as possible* 65.5* 30 60.25.Benedicts test is done for blood sugar urinary glucose urinary reducing sugars* .30. A boy defecate just after he takes meal. nerve supply to latismus dorsi thoracodorsal* subscapular long thoracic 61. sup parathyroid gland .66. first part of axillary artery ends at border of pectoralis major border of pec minor * border of teres major 69. If RCA is occluded distal to the origin of right marginal artery which part of the heart will be affected most? Rt atrium SAnode AVnode* Apex of the heart 71. Factor 9 def wat is the appropriate treatment? Cryoppt* .Esophagus is related to which structure whose dilatation can cause dysphagia l atrium* left vent SVC Rt atrium 70. A patient with multiple fractures and BP 90/60 pulse 60/min wat is the most appropriate initial treatment? IV fluids* Reduction os fractures 73.MHC complex is related to HLA typing* blood grp antigens same in identical twins 68.is related to sup thyroid artey* lies outside the pretracheal fasciae lies anterolatral to thyroid 67. Myleination of the brain wat is most inappropriate? Starts at birth Occurs hapharzadly Sensory neuros are myelinated first Occurs by oligodendrogliocytes 72. Highest mortality rate Hepatitis A Hepatitis B Hepatitis C Hepatitis D Hepatitis E 78. DIC most inappropriate low FDP* consumptive coagulopathy treated with heparin low platelets 75. Kleinfelters syndrome chromosomal deficiency autosomal dominant autosomal recessive extra X chromosome* extra Y chromosome 76. which organism causes meningitis secondary to pyogenic lung abscess? Strep . Fat embolism most unlikely urinary test is diagnostic* occurs due to trauma to fatty tissues caused bt acute pancreatitis 77.which organ has least chance of metastasis lung bone liver kidney spleen* 80. Diphtheria is transmitted by sexual contact respiratory * feco oral 79.FFP Whole blood WBC 74. influenzae 81. Most likely finding in CSF inc glucose in fungal meningitis dec glucose in viral meningitis lymphocytosis in aseptic meningitis dec glucose in pyogenic meningitis 88.Staph Pneuococcus h.which of these is not caused by bacteria histoplasmosis* 87. in autosomal recessive disease all siblings are affected both parents are affected only mother is affected all siblings have a chance of one into four of contracting the disease* 86. OCP increase the risk of breast cancer endometrial cancer thromboembolism* osteoporosis 83. most imp for wound contraction myoblasts . Osteoporosis is less in premenopausal women due to effect of estrogen* testosterone progesterone androgens 84. A trauma patient in shock with stab wound along left 10th rib which organ is damaged? Liver Spleen* Colon Duodenum 85. thromboembolism is most common in? leg* liver brain kidney 82. vesicles Prostatic urethra Ejaculatory duct* 92. About large gut descending colon has no haustra appendix has no tenia* all large gut is supplied by inf mesenteric artery ascending colon has a mesentry 93. Has thick submucousa Mucousa has extensive folds* 94. most probable abt gall bladder.fibroblasts myofibroblasts* epithelium collagen 89. smooth muscles are striated voluntary mostly arranged in circular and longitudinal layers * 90. Ductus deferens ends into Prostate u. hyperparathyroidism causes least likely hypertension dystrophic calcification* inc PTH 91. Which HBpathy presents with crises sickle cell trait sickle cell disease* thalassemia .Bladder S. a patient with known HBV infectio comes with HBSAg + HBe ab+ HBV DNA + HBC IGM ab+ Wat is the diagnosis? Acute dis * Chronic carrier Chronic active 95. Heparin is released from which cells. which cells produce antibodies plasma cells* T cells Bcells Platelets 102.PDGF is secreted from which part of platelets alpha granules dense bodies cytoplasm lysosomes 101. . 60 yr male with lymphandenopathy and low hb low platelets and increased wbc microscopy show large mature cells wats the diagnosis? CLL* Burkitts Follicular NHL 99. what Is true abt Hb consisits of alpha and gamma chains in adults imp buffer of H+ * not assoc with CO2 transport 100.Hb C 96. unconjugated bile is carried by which protein? Albumin * Pre albumin A globulin B globulin Fibrinogen 97.After gastrectomy patient will develop iron def anemia megaloblastic anemia due to folic acid def pernicious anemia* sideroblastic anemia 98. at the end of pregnancy uterus is most sensitive to which hormone? Estrogen Progesterone Prolactin Oxytocin* 110. Epitheloid cells in granulomas are formed by which cells macrophages lymphocytes langerhans cells* 104. wat is true abt pituitary ant pitutiary derived from neurons no portal vessels btw hypothalamus and post pituitary* 107. thyroid is invested in which fascia? Pretracheal* Deep cervical Prevertebral 105. Most abundant phagocytic cells in circulation basophils monocytes . D & C at 18 days after LMP endometrium will show which stage of uterine cycle? Follicular Ovulatory Secretory* Proliferative 109. wat is true abt thyroid consists of collecting ducts and acini epi changes shape according to state of activity.Mast cells* Eosinophils IgE 103. Satiety center is located in which part of hypothalamus anterolateral VPL VL Medial* 108.* Has parthyroids ant to it 106. macrophages neutrophils* 111. Pallegra is due to def of riboflavin thiamin e niacin * vit C 114. Trauma to the middle meningeal artey causes subdural hemorrhage epidural hemorrhage* SAH 115. wat is not true abt adrenal gland fetal gland is 1/3 size of the adult gland . patient with loss of fine touch proprioception and vibration injury to which part of spinal cord? Fasiculus gracilis Fasiculus cuneatus Post white column* Lateral white column 116. in females which structure lies btw ureter and peritoneum uterine artery* inf vesicle artery 117. most potent stimulus for release of aldosterone is inc K+ * inc Na+ inc glucose 113. which of these muscles is a part of the pelvic diaphragm deep transverse perinei puborectalis* bulbospongiosus ischeocavernosus 118. ICF & ECF differ in inc K+ in ICF 112. patient losses 1 litre of body fliud in 1 hour wat will happen first inc PR inc BP inc veno spasm* 123.tendon of which of these muscles is intracapsular bicep femoris plantaris popliteus* semitendinosus 121. which structure is most ant in popliteal fossa popliteal vein politeal artey * tibail nerve bicep femoris gastrocnemeus 120. a person in a room with optimum enviormental conditions how will the heat loss occur convections sweating breathing radiation and conduction* voiding 124. a bullet pierces the intercostals space which layer will it come after the intercostals muscles . major part of energy utilized during breathing is to overcome elastic recoil of lungs* resistance of chest wall large airway resistance small airway resistance 125.it is invested by renal fascia adrenal cortex is derived from mesoderm 119. which of these are the major resistance vessels arteries veins capillaries arterioles* venules 122. which of these is not a branch of internal iliac sup rectal* middle rectal inf vesicle sup vesicle 128. wat is seen in dysplasia pleomorphism inc NC ratio* inc mitosis 132. which vessel will be damaged if the phrenic nerve is cut musculophrenic int thoracic pericadiophrenic* 127.parietal pleura visceral pleura endothoracic fascia* pleural cavity 126. lymphatic of uterus do not drain into int iliac nodes ext iliac nodes sup ing nodes inf mesenteric nodes* 129. Due to inc ca Dec vit D PTHrP 131. pseudo hypoparathyroidism. metastatic tumors are identified by invasion of other structures* . a patient with hoarseness and lyrangeal nodule which is most unlikely atrophy * nodule hypertrophy hyperplasia 130. sarcomas have rapid growth inc vascularity* capsule benign 134. epi of a smoker will show inc ciliated epi mixture of st sq and pseudostratified cells* dec goblet cells 135. most unlikely abt glomerulus is has urinary space btw 2 layers is cup shaped is blind ended part of tubule has stratified epi* has podocytes 136. not part of the portal tract portal vein hepativ artery bile duct hepatic vein* 139. which vessel is involved and dilated in portal HTN portal vein* hepatic artey hepativ vien SMV 137. most unappropraite abt liver has dual blood supply hepatic artey gives only 35% of O2 portal vein has 70% O2 * 138.pleomorphism inc NC ratio inc mitosis 133. not a part of axillary nodes ant grp . medial grp apical grp central grp deep cervical* 140. in males uretric stones mostly lodge at which point just below kidney* at crossing of ext iilac vessels at pelvic brim at ischeal spine 142. if there is fracture of the acetabulum post and superiorly which bones r inv ileum and ischium * ileum and pubis isschium and pubis 143. Clavipectoral fascia completely covers pectoralis minor* forms suspensory lig of breast forms axillary tail 141. pain of the ovary is transmitted to the medial side of thigh thru which nerve femoral obturator* ilioinguinal 146. left ventricle does not contain supraventricular crest chordae tendinae aortic vestibule papillary muscles 147. at the level of second part of duodenum wat is unlikely aorta at the right of SVC* left renal vein 145. most constricted part of the male urethra? Ext meatus* Infandibukum Navicular fossa 144. epidural space contains venous plexus* contiues into skull at foramen magnum . Osmotic pressure depends on electrical equivalence conc gradient yenp no of particles mol size 154. Otic ganglion lies under foramen ovale* foramen spinosum maxillart nerve stylomastoid foramen 149. middle menigeal artery enters thru which foramen? Foramen spinosum* Foramen rotundum Foramen ovale 150.attached to dorsal coccyx upto S2 148. chemoreceptors respond to inc PCO2* . Superior petrosal sinus lies in falx cereberi fakx cerebelli tentorium cerebellli* diaphragma sella 151.which of these is not a basic tissue of the body epi nerve muscle blood CT 153. Great cerebral vein does not drain deep cerebral vein sup cerebral vein basal vein thalamic vein 152. Medicine paper (Jan 2009) . which vessel lies ant to IVC left renal vein left renal artery right renal vein right renal artery right testicular vein* 158. if inguinal canal of a female is opened wat does it contain round ligament and ilioinguinal nerve 159. Processus vaginalis covers testis only* covers ductus deferens forms ext spermatic fascia arises from parietal peritoneum arises from visceral peritoneum 156. Dorsal rami of nerves supply ext of trunk ext of limbs 157.dec PO2 inc Ph inc temp 155. lymphatic drainage of testis is into para aortic nodes sup ing nodes int iliac nodes ext iliac nodes Posted by Dr Sultan Ahmad 6 comments Reactions: MEDICINE JANUARY 2009 By 'Crack Smart'. what is the diagnosis? a. ACTH b. Chronic relapsing pancreatitis c. Norepinephrine Q#5.Major stress hormone is: a.Iron is stored in parenchyma as: a. Acute pancreatitis b. Hemosiderrin c.a 45 yr old male comes with the complaint of acute abdominal pain with migratory thrombophilibitis. Transferrin Q#3. Kidney e. Least site of metastasis is: a. D. C.M Q#2. Liver c. Lymph nodes Q#4. Epinephrine c. Ferritin b.Q#1. Spleen b.a pancrease (ans) d. Lung d. Benedict test is used for: . Intercostal membrane b.A lady with hepatitis B. Cleft hard and soft palate (ans.Penetrating injury in left 4th ics will damage which of the following structure? a. urinary ketone. Hbv Dna +ve. Immunized Q#7. urine glucose test Q#6.. Serology shows: HbsAg +. HbeAg +ve. renal glycosuria b.what does it show? a. Intercostal muscle c. Viral load is decreasing d. left ventricle e. Hormonal deficiency d. Laryngeal weakness Q#8. Cleft soft palate c. Chronic hepatitis B c. Nasolacrimal duct block e.A healthy baby comes apparently with the complaint that he does not take breast milk instead he prefers the bottle feed.. Anti HBc IgM +ve. What is the diagnosis? a.Anti HBe -ve.a. c. Acute Hepatitis B (ans) b. left lung d. Left atrium . most probably) b. Regarding polycystic kidney: a.In Hyperglycemia. Pt will develop: a.Non-myelinated nociceptor fibre directly synapses with: a. Insulin (ans) Q#10. Macrocytic Anemia Q#12. It is autosomal recessive c. Dorsal spinothalamic tract options sorry cant remember other . It is autosomal dominant b.After 3 months to gastrectomy. Megaloblastic anemia (ans) d. Glucagon d. Microcytic hyperchromic anemia c.Q#9. Cortisol c. Only male suffer Q#11. which 1 will increase: a. Microcytic hypochromic anemia b. Epinephrine b. Genetic mutation d. Hemolytic Anemia e. Only females suffer e. perferinges c.A 2 yr old child (premature). which cardiovascular anomaly will most likely develop: a.Which is the major circulating phagocyte? a. Mesoderm Q#14. Monocyte d.Somites are derived from a. Failure of closure of foramen ovale c. Mast cell c. Clostridium difficile (ans) b. PDA (ans) Q#16. Helicobacter pylori Q#15.Pseudomembranous colitis is caused by a.Q#13. Macrophage (ans) Q#17. Has increased Na+ . C. Coarctation of aorta b. Neutrophil b.ICF differs from ECF in: a. Has increased Ca+ Q#18.R . S. Has increased K+ (ans) c.b. Duodenum b.Antidote for Morphine: a. Glycine b.E.R (ans) b. ileum d. Jejunum c. Dopamine c. GABA Q#19. Nalaxone (ans) Q#22.which of the following is absent in Axon hillock a. Stomach Q20. R.E.Bile salts are normally absorbed from a.Neurotransmitter of corticostriatal fibres is a. Ach d. S1Q3T3: cant remember options for this Qz Q#21. c. Abductor pollices b. DCT .C6 Q#25. Cause inactivation of drug b.Dorsal rami of cervical nerves innervate a.Which part of kidney produces Erythropoiten: a.Upper end of esophagus is at the level of a. Stabilizers the shoulder Q#24. Microtubules d. Mitochondria Q#23. MACula densa b. Extensors of trunk d. Extensors of limbs c. Pct d. pertubular capillary (ans) c. Makes drug insoluble Q#23. Is type 1 reaction c.Glucronide conjugation of drugs a. Microfilaments e. Liver biopsy from a pt shows a lesion which is comprised of central collection of structureless material surrounded by epitheloid cells and lymphocytes. Glossopharngeal nerve b. swelling of cell Q#28. Hydropic change e. Caseous necrosis b. One large cell with multiple nuclei arranged close to membrane is also seen.Side effects of thiopental a. Formation of free Radicles c. Coagulation Necrosis c.The most likely change is: a. Damage to Golgi bodies b.Posterior belly of diagastric is supplied by: a. Liquefective necrosis Q#27. Seizure 29.e.Tissue damage by Ionization Radiation is due to: a. Hydropic change d.Loop of henle Q#26. Fatty change d. metaplasia of cell e. Nausea b. Facial nerve . IgD e.Which is highest in human milk? a. IgG Q#32. Middle cerebral Artery c. IgA b. IgD d. IgM b.Aneurysm of arch of aorta will most like compress . IgM c.c. IgE e.Which immunoglobulin has pentameric structure: a. Posterior cerebral artery Q33. IgE d. Trigeminal Q#30. Ant:Cerebral Artery b. IgG Q#31. IgA c.Blood supply of Visual area: a. papillary Q36. Superior mesentric Q35. Medullary c. Anaplastic b. Iodine deficiency b.Gastroduodenal artey arises from a. Left main bronchus c.Which does not lead to Pre-malignancy a. Esophagus b. Slow growing tumor of thyroid is: a. Melanosis . Follicular d. Splenic d. Celiac b. HBV d. Thoracic duct Q34. Hepatic c.a. Phrenic nerve d. HIV c. Which is not “ Major Basic tissue” . Increased serum Na+ c. Axillary b.Aldosterone will increase with: a.A 35 yr old female is protected against osteoporosis due to A. Histology of pitutary will show. Median c. decreased plasma volium d. a. Increased serum K+ b. Increased ACTH Q39. Musculocutaneous Q38. which nerve is damaged? a. Ulnar e.A 34yr old man can’t abduct his arm due to fracture of humerus. Increased Chromophobes b. Radial d.A 14 yr Boy comes with the complaint of his abnormal Height and secretion from mammary glands. Estrogen (ans) B. Progestron 41.Q37. Increased Acidophilic cells (most probably diz was the ans) Q40. Blood e.ENTAMOBEA HISTOLYTICA? A.I.EARLIEST SIGN OF ASPIRIN OVER DOSE? A.WHICH LOBE OF LUNG HAS 2 SEGMENT A.INVERTED C.a.T C. Muscle Posted by Dr Sultan Ahmad 4 comments Reactions: SURGERY JUNE 2009 SURGERY PAPER 24 JUNE 2009-07-11 By Dr Sana Memon.METABOLIC ALKLOSIS Q3.T..EVERETED B.PROTZOA Q5.INDIFINE D.LEFT LOWER LOBE E. Q1.RT LOWER LOBE C.ARRTHYMIA B.INDURATED .LIMITED TO G. Nerve (ans) c. Bone d.UPER LOBE B.TINNITUS B.RT.NAUSEA VOMITING Q4.DOES NOT HAVE CARRIER B.TONIC CLONIC SEZIRES C.MAIN SIDE EFFECT OF LIDOCAINE A.LEFT UPPER LOBE Q2.RT MIDDLE LOBE D. Epithelium b.B ULCER MARGIN? A.GASTRIC UPSET C. CLOSTRIDIUM DIFFIC? A.G SIX PHOSPATES DIF D.ACTIN B.OVARIAN ARTERY.HEP.SECTRION OF HARMONE INCREASE AS DARKNESS INCREASING? A.CAUSES LOCAL HYPOXIA B.CONTENT OF INGUINAL HERNIA? A.AUTOIMMUNE HEMOLYTIC ANEMIA B.Q6.CONVERSVATIVLY TREAT.CORTISOL B. C.CA Q13.ALPHA TOXIN D.SPECTRIN D.RBC CELL MEMBRANE? A.B B.LYMPH OF FUNDUS OF UTERUS.DIALYSING FLUID COMPOSTION SAME AS PLASMA AXCEPT WHICH ONE HIGH? A.GLUCOSE B. Q10.UREA C.MYOCIN C.MELATONINE D.K* D.INDIRECT PASSES ONLY SUPER.RING B.DIRECT LYING ON MEDIAL SIDE OF INF:EPIGASTIC ARTERY Q11.AEROBES C.THROMBOCYTOPENIA Q8.COMMON ORGAINSM INFECTION IN LIVER TRANSPLANT PT: A. D.SICKEL CELL ANEMIA C.DIRECT PASSES THROUGH HESSAL BECH TRIANGLE C.GROWTH HARMONE C.CYTOMEGALO VIRUS .INGUINAL HERNIA? A.FEMORAL ARETY Q12.INF:EPIGASTRIC ARTERY B.INDICATION OF SPLEENECTOMY? A.HEB C C.INSULIN Q7.HEMOGLOBIN Q9. I.BOY WITH INCREASE BLEEDING TIME AFTER CIRCUMCISION.HIS COUSIN ALSO HAVE SAME PROBLEM? A.CLOTTING TIME C.INC:BONE REABSORBATION Q20.D.EPITHOID CA Q19.APTT D.COMPENSATION AFTER MILD BLOOD LOSS? A.BLADER CA? A.ABSORTION OF CA INTESTINE C.SECOND HEART SONUD? A.BRUN PT.HYPERKALEMIA C.TRANSITIONAL CELL CA.P .RUN IN POST INTERVENTRICULAR GROOVE D.AFTER MULTIPULE BLOOD TRANSFUSION? A.INFLUENZA Q14.SUPPLY BOTH ARTIUM C. B.HYPOKALEMIA B.INCREASE BLOOD CA LEVEL B.Sq: CELL CA C. HYPOTENSION.CLOSURE OF SEMILUNAER VALVES Q18.D.INC:ABSORTION FROM RENAL TUBULES D.INC.HYPOCALCEMIA D.NORMAL B.INFECTION Q16.RAISED BUN Q21.TACYCARDIA B.BR:CIRCUMFELX Q15.RT:CORNOARY ARETY? A.CALCITONIN? A.FILLING OF VENTRICLE C.BT B.C B.DEC: COMP OF VEIN C.PT Q17.RISES FROM RT:POST AROTIC SINCUS B.TACYCARDIA? A.CLOUSER OF TRICUSPID VALVES B. RECTUM C.CIGGRATE SMOKING C.HYPOVOLEMIA B.IRREVERSIBLE B.SUBMUCOSAL FIBROSIS B.GASTRO ESOPHAGEAL JUNCTION D.SHISTSOMIA MANSONI B.PT:WITH GRANULOMATOUS LESION IN BLADDER DEVELOPED CA.MARASMUS B.AGGREVATE BY TEMPORAL LOBE TRUMA Q27.HEMATOBOIUM D.BREAST TRUMA Q24.COMON CAUSE THROMBPHILIBITIES D.FEMORAL SHEATH? .DEUDENUM B.FAT EMBOLSIM? A.CERVIAL JUNCTION Q28. EPITHILIUM Q30.SYPMTOMS AFTER 12 HRS B.INCORRECT? A.CHILD THIN.KASHIKOR C.DIYES Q26.METAPLASIA.SHIS.BRONCHIAL EPI.FRONTAL LOBECTOMY B.APATHY A.UNCONSIOUS Q23.CA.DEPIGMENTED.80% FATAL C.ILIUM Q29.Q22.DIARREHA Q25.APPENDIC EPIPLOIC PRESENT IN? A. C.IN ALL TYPE OF SHOKE? A.TACYCARDIA C.EPITHEIAL POLYP C.RETROGRADE AMNEISA? A.CAUSE? A.COMMON COMPLICATION OF BETAL NUTS? A.JEUJENIUM D. VIT D C.VIT E D.FASCIA TRANSVERSALIS AND ILISCAP Q31. PERONEAL NERVE B. Q34.HEART B. Q32.FASICA ILISCA B.SPINAL PORTION OF ACCESSRY NERVE.BLEEDING D.LOWER CORD OF BRACHIAL PLEXUSES E.AXILLARY NERVE D.CRANIAL PORTION OF ACCERY NERVE.SORE THOART C.MUSLES .COMON PERNEAL NERVE D.A. B.BRAIN C.LUNG E.SK.FASICA TRAVERSALIS C.COMMON SIGN OF OF AGRUNLOSYTOSIS CAUSES BY ANTIEPILEPTICS? A.SEPRATE RECTUM AND SIGMOID COLON D.VIT B Q36.CERVICAL LYMH NODE BIOPSY? A.NEUROBLASTOMA.CHOLESTATOMA B.70% OXYGEN EXTRACT IN RESTING CONDITION IN WHICH TISSUE? A.SEPRATE RECTUM AND URINARY BLDER B. C.PHERINIC NERVE.KIDNEY D.SEPRATE RECTUM AND URETHRA Q37.UNABLE DORSIFLEX AND EVERT FOOT? A.GIT UPSET Q33.SEPRATE RECTUM AND UROGENITAL SEPTUM C.SUPER.DEEP PERONEAL NERVE C.URORECTAL SEPTUM? A.WT:LOSS B.LOCALLY MALIGNANT TUMOR? A.VIT A B.TIBEAL NERVE Q35.DURING OPERATION OF POST.VIT C E.ESTABLISHED ANTIOXIDANT? A.PT: WITH ROAD TRAFFIC ACCEDIENT. E.SIDE OF CHEST? A.PT.XYY Q43.PYLORI D.ACTINOMYCOSIS C.ENT.H.XXY C.AREA POSTREMA Q42. IRON ABSORBATION Q41.XX/XY B.XXX D.DEVELOPED CHRONIC WOUND ABBCESS AND DRAING SINUS.DEC.IPSILATERAL LUNG COLLAPSE AND CONTRALATERAL CHEST WALL COLLAPSE.TRUE HERMOPHADISM? A.PARAORTIC LYMPH NODE .CONTAING YELLOW GRANULES? A.CHLESTROL C.LFT B.AFTER APENDICTOMY.WHAT NEXT DIAGNOSTIC TEST? A.GLUCOSE D.INC ABSORBATION OF FAT D.COMA(DEMAGE OF PART OF BRAIN) A.ABSORTION OF AMINO ACID B.NUCLEUS CERULOSIS B.NO CHANGE IN CHEST WALL D. C.RT:TESTIS LUMPH NODE DRAINGE? A.CONTRALATERAL LUNG COLLAPSE AND IPSELATERAL CHEST WALL COLLAPSE Q40.SUP:MEDIAL GROUP INGUINAL NODE C.HISTOLYTICA Q44.AST/ALT Q4930 YRS OLD PERSON AHS STAB WOUND ON RT.DEEP INGUINAL NODE B.DEC ABS:OF WATER C.RETICULAR FORMATION AREA C.PT: WITH THE S/S OF PANCREATITIS BUT NORMAL AMYLASE.NOCARDIA B.IPSILATERAL LUNG COLLPASE AND IPSILATERAL CHEST WALL SPRING OUT B.DEC.Q38.DURING OPERATION ILIUM REMOVE? A.COLI E. MICROCYTIC.DEC PLATELETS D.CORTISOL.GLUCAGON C.CHO AND LIPID DEPOSTION BY WHICH HARMONE? A.POPLITIUS B.FAT.FOLLICULAR CA Q53.DEC CONTENT OF HB% B.SEMIMEMBRANOUS D.CHORNIC BLOOD LOSS C. D.HENOCH.SEMITANDANNIOUS C.PELVIC BRIUM C.THROMBOCYTOSIS.COMON SITE OF RIB FRACTURE? A.ANEMIA B.HEMOGLOBIN C.HEAD OF RIB B.ANEMIA Q48.SHIL:PUPURA B.CYNOSIS? A.INCREASE BLEEDING TIME CAUSE BY?ACCEPT A.GROWTH HARMONE Q46.BODY OF RIB Q51.MEDULLARY CA C.ANGLE OF RIB C.HYPOCROMIC ANEMIA? A.PUJ JUNCTION B.Q45.CROSS THE COMON ILLIC ARTERY D.INSULIN B.URETER CONSTRICTION?INCORRECT A.SICKEL CELL ANEMIA D.TURNER SYNDROME? .IRON DEF.SARTORIUS Q49.PAPILLARY CA B.CIROHSIS C. Q52.HEMOLYTIC ANEMIA Q47.INC:CONCENTRATION OF DEOXY.SLOWEST GROWING TUMOR? A.RUN ON PSOS MUSCLE Q50.UNLOCKING MUSCLES? A. A.FAT C.CONTINUE AS AXILLARY VEIN Q.PROTIEN Q58.5CM Q54PELVIC DIAPHAGRM FORMED BY MUSCLES EXCEPT? A.PYriformus MUSCLES C.BIVIRDIN Q59.PT WITH HYPOVOLEMIC SHOKE.WHAT WILL BE DECREASE? A.CHO B.TREATED.RUN OF RADIAL SIDE OF FORARM B.CHRONIC HEMOLYSIS AND INTRACELLULAR PIGMENT ACCUMULATION? A.MUSCULOCUTANEOS .COCCYGUS B.HIGHT 4.MENTAL RETARDATION C.CLINDAMYCIN Q61.LONG HIGHT B.AXILLARY B.CARRING ANGLE REDUCED D.NEOMYCIN C.after 48 HRS OF STRAVATION.DRUG LEAST NEPHROTOXIC A.HEMOSIDRIN C.urine OUTPUT C.HEART RATE B.WHT WILL BE BREAKDOWN TO PROVIDE ENEGRY A.FORM ON THE PALMER SURFACE OF HAND C60.SYPHILIS B.HERPES C.GONOCOCCUS Q56.ILOCOCCGUS Q55.BASCILIC VEIN? A.RADIAL C.P Q57.BILIRUBIN B.B.ANT:DISLOCATION OF SHOULDER JOINT WHICH NERVE DEMAGE? A.GENAMYCIN B.STD CAUSES ULCER ACCPET? A. EPIDYDEMUS C.INFRASPINATUS B.SRTOLI CELL Q63.HEMIAZYGOS VIEN D.STYLOGLOSSIUS Q67.INTERNAL INTERCOSTAL MUSCLES B.8CM FROM MID LINE Q64.SKIN CA Q65.RENAL VIEN B.LEFT SUP:RENAL VEIN DRAIN IN WHICH VEIN? A.STYLOPHARAGEUS MUSCLES B.LOWER BRACHIAL PLUXES Q62.DIAPHGRAM Q69.B+ B.BLOOD GROUP CAUSES REACTION? A.AFLATOXIN B1 CAUSES CA? A.AZYGOS VIEN C.TESTOSTERONE PRODUCED BY ? A. B.TOWARDS RIGHT C.EXT:INTERCOSTAL MUSCLES C.APEX OF HEART? A.MUSCLES DIVIDE SUBMEDIBULAR GLAND? A.ACCESSARY AZYGOS VIEN .LYDING CELL B.O+ Q66.BLADDER CA C.PT:CANNT ABDUCT THE ARM UPTO 30 DEGREE? A.MAJOR INSPIRATORY MUSCLES? A.MYLOID C.AB-D.LEUKEMIA D.A+ C.DELTOID Q68.HEPATOCELLUAR CA B.MEDIAN E.D.3RTH INTERCOSTAL SPACE.SUPRA SPINATUS C. OF ULNER NERVE Q72. Q81.ILIOFEMORAL B.SUP:EPIGASRIC ARTERY B.DEEP BR.PT:LOSS OF ABDUCTION.ANEMIA C.MEMORY B.DISOLVMENT OF CLOT Q82.DILATE THE PUPIL C.HEPATIC ARTERY C.ARRTHEYMIA Q77.RAIDAL NERVE D.DISLODGMENT D.DELTA LIGAMENT/MEDIAL LIGAMENT Q79.MEDIAN NERVE B.Q70. C.FORMED LATERAL WALL OF TYMPANIC CAVITY B.VERBAL COMUNICATION? A.S/S? A.EMBOLISM BSECRTED BY .HEPARIN A.ULNER NERVE.BASOPHIL C.STRONGEST LIGAMENT? A.SUPPLY BY VAGUS AND TRIGEMINAL NERVE C.MALE WITH T4N1M1 SERVIVAL RATE LESS 50%.EMPTY BLADDER Q71. ADDUCTION OF FINGERBUT NO LOSS OF SENSATION? A.SPINCTER CONSTRICTION B.TYMPANIC MEMBRANE? A.SHORT GASTRIC ARTERY BR:OF A.FUNCTION OF LIMIC SYS AXCEPT? A.SPINAL REFLAX Q78.BY PRESENTATION B.AGGRESTION C.OPENING OF AUDITARY TUBE IN POST WALL.INVOLVE IN RESEARCH WORK .SPLENIC ARTERY Q80.PARASYMPATHATIC SYS CAUSES ALL OF FOLLWING AXCEPT? A.CACHXIA B.ISCHEO FEMORAL C. 1ST RIB B.FORMED BY 4TH PHARENGEAL ARCHES C.PIVOT JOINT? A.PICTORIAL CHART Q84.HOW YOU SHOW IN CHART A.BAR CHART B.BMR Q85.SARCOPLASMIC RETICULIUM B.LYSOSOME D.THYROXINE CAN BE USE OTHER THEN THYROID TREATMENT? A.PCO2 B.GROOVE OF SUBCLAVIN ARTERY? A.PO C.ATLANTOAXIAL JOINT B.OXYGEN CONCENTRATION D.PROJECTION FIBERS PRESENT IN? A.THYMUS? A.T-TUBULES Q89. C.RETICULAR FORMATION Q90.WEIGHT LOSS B.MANDIBULAR JOINT C.37%POST MENUPOSAL WOMEN TAKING HRT.ESSENTIONAL AMINO ACID .63 WOMEN TAKING NO MEDICATION.PIE CHART C.H+ Q91.INCREASE APPEPITTE.ELBOW JOINT Q88.MITOCHONDRIA C.REGRESS AFTER BIRTH B.CA REALSE FROM SKETAL MUSCLES FROM? A.PERIPHERAL AND CENTRAL CHEMO RECPTORS BOTH RESPONES TO? A.INTERNAL CAPSULES C.CLAVICLE Q87.3RD RIB D.PRODUCED T CELL Q86.2ND RIB C.CORPUS CALOSUM B.Q83. LUMBER PUNCTURE? A.55% ABSORED B.WATER ABSORBTION IN PROXIMAL CONVULATED TUBULES? A.CEREBRAL CORTEX A.ASECNDING LIMB IS PERMIALE FOR WATER C.L2 TO L3 B.L4 TO L5 C.DORSAL COLUMN MEDIAL LAMINUSCUS COTAIN? A.ACTIN C.GIVE 1 MOLECULES OF PYRUVATE B.ARGININE D.RECEIVE NO SENSORY INFORMATION B.ORGAN TRANSPLANT REJECTED IN 10 MIN A.S5 Q96.ESSENTIONAL FOR INITATION OF VOLUENTRY MOVEMENT Q94.NATURAL KILLER CELL .TYROSINE B.THICK FILAMENT? A.CELL MEDIATED IMUNITY B.ALANINE C.PLASMA CELL D.SUBARCHANID LYER END AT? A.POST.S2 C.WATER ABSORED THORUGH FASCILITED DIFFUSION Q95.CROUDE TOUCH C.PRESENT ONLY IN FRONTAL LOBE C.TEMP Q97.L2 B.GLYCOLYSIS? A.A.FINE TOUCH B.MYOSIN B.ACTOMYOSIN Q99.PAIN D.ILIC SPINE Q93.SUP.ANTIBODY C.PHENYALANINE Q92.2 MOLECULE OF GLUCOSE Q98. THORACIC DUCT C.DIAZEPAM C.4.ARCHES OF LEFT LUNG? A.PROPYTHIORACIL D.BARBITURATES B.10 B.CHLOROQUINE B.CRANIAL NERVES CARRINYING PARASYMPATHATIC FIBERS? A.7.ISONIAZID D.CARBON TETRACHLORIDE Q101.WHICH FOLLOWING CHEMICAL EXPOSURE CAUSES CA? A.DIAPHGRAM SUPPLY BY? A.CAVERNOUS SINUS COTAIN? A.TROCHLER NERVE B.3.WHICH DRUG INCREASE CYTO P450 SYS A.10 C.INTERNAL JUGULAR VEIN C.SULFA DRUGS C.METHYL ALCOHAL B.7.10 Q102.ABDUCENT NERVE Q107.4 Q105.3.ARCH OF AORTA Q104.CULTURE B.BENZENE C.C 2.INTERCOSTAL NERVE C.C.SERUM ANTIGEN .CARBAMAZIPINE Q103.KETOKANAZOLE Q108.7.8.Q100.4.AZYGOS VEIN B.DURING PREGNANCY HICH DRUG SAFEST IN HYPOTHYRIOD PT: A.9.9.BETA BLOCKERS B.WHICH DRUG CAUSES CORNEAL OPACITY A.3.STAINING C.5 B.3.ERTHROMYCIN Q106.EASIEST METHOD TO DIAGNOSED MENINGOCOCCUS? A. Which hepatitis has the most mortality a. DCT 5.UNABLE TO PERFORM VOL. hepA b.MALARIA B.BALCK WATER FEVER CAUSES BY? A. diffusion is inversely proportional to .QUATRUN MALARIA Q110.Q109.OXYTOCIN AND ADH ORIGENATED BY? A. hep C d. Gastric emptying inhibited by Ans. osmotic pressure depends on a. increase ACTH c.PLAS. increase molecular size b. 65% Na reabsorption is in a.HYPOTHALAMUS C.MOVEMENT B. electric charge 6. CCK 2. increase no of particles (ans) c. PCT (ans ) b. hep B c. acts as a buffer (answer ) c. Aldosterone secretion increases in response to a.STATIC TREMORS C. hep D ( ans) e.ANT PITUTARY Q111.INTENSTION TREMORS 1. regarding heamoglobin a doesnot carry CO2 b. Hep E 3.PLAS:FALCIPARUM C.POST PITUTARY B.CEREBLLAR DISORDER? A. contains 2 alpha and 2 gamma chains 7. hyperkalemia (ans) 4. hyponatremia b. infants tend to defecate immediately after they eat because of a. urine ketones (ans) c. loss of myenteric plexus (ans ) 10. prolactin . pernicious anemia (ans) 17. cause of achlasia is a. appendix a. Benedict test is used for a. muscle proteins (answer) 12.a.which hormone is responsible for fetal brain development a. gastroileal reflex b. total gasrectomy results in a. growth hormone b. which is not the part of portal tract a. gastrocolic reflex (ans) 9. what results after portal hypertension a. liver glycogen c. iron def anemia b. thichness of the membrane (ans ) 8. urine glucose b. muscle glycogen b. iron absorption increases in a. hepatic vein (ans) e. hepatic artery b. microcytic anemia c. serum glucose 15. what is the energy source after 48 hrs of starvation a. portal vein d. iron def anemia (ans) 18.women with gravida 4 hb 8 occult blood in stool cause a. increase portal diameter (ans ) 14. ferrous form (ans) 16. does not have tenia coli (ans ) 11. connective tissue c. bileduct 13. dystrophic calcification (ans) 21. leukotriene B4 b. local invasion 24. RA factor 27. hairy cell d. which investigation a. increase nuclear cytoplasmic ratio b.adrenal cortical def results in a.c. diagnostic criteria for soft tissue tumours a. diagnostic criteria for malignant tumours a.pseudohyper PTH is seen in a. brown atrophy b. thyroid hormone (ans) 19. C5 complex (ans) c. uric acid (ans) b. mitotic figure c. vit D def b. Lymphocytes normal appearance cause a. patient with decrease hb. decrease platelets but increase TLC with generalized lymphadenopathy. pleomorphism c. hyponatremia (ans) 20. heparin is released by which cells . diagnostic criteria for pre malignant condition a. histamine 28. hyperglycemia b. CLL 26. pleomorphism b. tuberculous lymphadenitis b. non Hodgkin lymphoma c.patient with CML develop joint pain. mitotic figures 23. hypokalemia c. most important chemotactic agent a. paraneoplastic PTH release from tumours ( I think it is the answer ) 22. increase vascularity 25. mitotic figures c. hyperPTH will be in all except a. pleomorphism b. proliferative d.sideeffect of theophylline a. spleen ( ans ) 31. lung c. premenstrual 35. estrogen ( ans) 34. protein synthesis b. bone b. lymphnode d. neutrophils 29. which hormone decrease cholesterol a. diagnostic D&C will show endometrium in which sage a. most radiosensitive tumour a. apnea 32. lung c. brain 30.least site of metastasis a. secretory c. insulin def will result in a. lymphnode e. nausea vomiting d. glycogen syntheseis c. liver d.a. hypertension b. platelets d. mast cells b. thromboembolism ( ans) 33. NE . brain b. progesterone b.women with lower abdominal pain on 18 day of LMP presented with secondary infertility . ovulatory b. ketogeneis in liver (ans) 36. cortisol (ans) b. which is a stress hormone a. seizures (ans) c. most common side effect of estrogen contraceptive a. basophils ( I think so it is the ans ) c. loss of ECF only c. alpha granules (ans) b. K+(ans) b. PDGF released from which organelles of platelets a. tumour c. jejunum 40. severe dehydration will result in a. ileum (ans) b. hyperplasia (ans) b. loss of ICF only b. continuous stratified sq epi b. FFP c. mitochondia b. HCO3 39. whole blood 41. nissl substance (ans) 46.heavy smoker with hoarseness vocal cord will show a.37. axon doesnot contain a. pseudo stratified epi with patches of stratified sq epi 44. podocytes b. Na c. produced by chroidplexus which is cauliflower in shape coverd by smooth muscle b. daily production is 500 ml per day(ans) 45. Bilesalts absorb from a. dense granules c.what is not present in bowman capsule a. cytoplasm . loss of total body water (ans) 38.factor 1X def what should be given a. most imp ICF electrolyte in excess a. habitual smoker the epithelium will be a. cryoprecipitate b. Cld. atrophy 42. stratified epithelium (ans) 43. regarding CSF a. cell membrane 47. 7th costal cartilage c. T10(ans) b. cardiac muscle is a specialized smooth muscle (ans) 48. ectoderm b.diapgragm doesnot arise from a.dorsal rami contribute to a.pellagra is due to a. striated b. sartorius 50.d.ileopsoas b. membranous urethra c. navicular fossa 54.pt has difficulty rising from sitting position but flexion of leg is normal. mesenchyma 49. extensors of trunk(ans) b. L1 56. niacin def (ans) 51.gluteusmaximus (ans) c.narrowest part of urethra a. thin filament is covered by a. extensors of leg 55.somites are formed from a. which musle is involved a. smooth muscle is a. tropomyosin ( ans) c.9th rib d.bones of carpel . flexors of neck c. MAO (ans ) 52. mesoderm (ans) c. external meatus (ans) b.which enzyme common to metabolism of both NE and serotonin a. titin b. troponin t 53. COMT b. is pierced by axillary artery c.artery which is adjacent to the phrenic nerve a.increase thoracic pr b. pericardiophrenic (ans) 59. thoracodorsal n (ans) 58.decrease abdominal pr c. rt atrium c.is continuous with prevertebral fasia . axillary nodes 65.contraction of diaphragm causes a. invests pectoral minor muscle (ans) b. internal thoracic nodes (ans) b. arch of aorta b. tumour of cervix will spread to labius majora via a.lt recurrent laryngeal n not associated with a. which vein of heart is located in posterior interventricular sulcus a middle cardiac vein(ans) b. anterior cardiac vein c. rt ventricle 62. proximal row contains scaphoid.triquetral. lunate. trachea and esophagus d. roung ligament of uterus (ans ) 66clavipectoral fascia a. internal thoracic b.lattismus dorsi nerve supply a. it is more vertical and wider than left lung(ans) 61. musculophrenic c. posterior nodes c.a. increase thoracic volume (ans) 64. left atrium (ans) b.enlargement of which part of the heart causes esophageal obs a.ligamentum arterisus c. sup vena cava (ans) 60.medial quadrant of breast lymph drainage a. great cardiac vein 63.foreign body enters into rt lung because a. pisiform(ans0 57. chances of siblings to be affected is one in four 73. interatrium wall c. popliteal artery (ans) 72. common peroneal n c. sup petrosal sinus is related to a. tail of pancrease 71. proximal portion of CBD is supplied by a. most anterior structure in popliteal fossa is a. tibial n d. structure not lying posterior to left kidney is a.cbd is divided in to following portions a. blockage of rt marginal artery will result in interruption of blood supply to a. diaphragmatic sella b. middle cerebral vein drains into a. tentorium cerebelli c.67. rt atrium (ans) 68.thoracic duct a. retroduodenal+infraduodenal+supraduodenal+intraduodenal(ans) 77. straight sinus 75. spleen c.structure precent b/w celiac trunk and sup mesenteric artery a. regarding autosomal recessive trait a. cystic artey (ans) b. pancrease and jejunum b. popliteal vein b. pancrease (ans) 70. left hepatic artery 76. falx cebri 74. lesseromentum is divided inti following portions . sella turtica d. passes thru aortic opening (ans) 69. rt hepatic artery c. cavernous sinus b. SA node b. duodenum (ans) b. take place with damage to fat tissue only (ans) b.25. relatives should be told first c. 35 81. regarding mekels diverticulum a.a. regarding behavioural sciences . knowledge 82. how is the professionalism of a doctor judged a. what is the commenest site of ureter stone obstruction in a standing male patient a. reputation c. neutrophils b. it is a remanant of embryonic yolk sac (ans) b. it is 60cm distal to ileocecal valve 79.in atherosclerosis following are seen a. two gps of patients given haemmorhoidal treatment and their effect seen with time a. prospective cohort study(ans) 84. lymphocytes 85fat embolism all are true except a.35 a. upper part of ureter 80. macrophages d. patient should know fist b. information should be hidden 83. plasmacells(ans) c.antibodies are produced by which cells a. 30 b. calculate the median in following data 20. foam cells (ans) 87. lateral wall of pelvis b. it is precent in 10% of population c.24. acute pancreatitis b. trauma to the breast 86. chronic recurrent pancreatitis c. pelvic brim (ans) c.30.5(ans) c. gastrohepatic+hepatoesophageal+hepatoduodenal+gastrodenal (ans) 78. 28. in order to disclose news regarding fatal disease a. punctuality b. pancreatic tumour (ans) .30. migratory thrombophlebitis is seen in a. In liver of a patient under the microscope a lesion was seen with central cheesy appearance surrounded by large cells with lymphocytes and fibroblast what type of necrosis is present Coagulative* Liquefative Fatty tuberculous 5.which structure crosses the aorta a. left ovarian vein Posted by Dr Sultan Ahmad 6 comments Reactions: OCTOBER 08 Questions.What is most lateral in cerebral hemisphere : Insula Pars triangularis * Pars interemedia Amygdala 2. ?subject Contributed by Dr Farhan Karim.Pre cancerous in aids: Cmv Ebv* Hsv Hpv 4. but forgot to tell about the subject in which he took the exam.Below the umblicus anterior abdomen is formed by: External intercostal and part of Internal oblique* Internal intercostals and part of transverse abdominis uscle External intercostals and transverse abdominis muscle Internal intercostals and transverse abdominis muscle 3.Which has no antigens AB* .Which harmone of the following stimulates thirst Adh* Aldosterone Renin oxytocin 6.left renal vein (ans) b rt renal vein c. 1.88. When r the ventricles most filled with blood ' Diastasis Rapid inflow Atrial systole Ventricular systole 10.Which of the following harmone stops ovulation during lactation Estrogen Progestrone Prolactin* Oxytocin 12Which of the follwing delays wound healing Vit c* vita Immobilization Vit E 13.OO+ A 7.In a thyroid surgery mass was excisied which microscopically showed amyloids post surgically which marker would be taken in account PTH Calcitonin* Ca Po4 8.feature of Downs Syndrome Cardiac anomolies Fallots Hypospadiasis Early death* .Which variant rises with rise in BP TPR* Pulse Venous tone Arterial tone 9.Example of wet gangrene Thromboembolism Bacterial endocarditis Pancreatitis 11. X linked dominant Pku Alkaptonuria Familial polyposis* Glycogen storage 16.Common feature in nitrates.Midarm circumference is the measure of Proteins* Fats Carbohydrates .Asthma has low Fev1* Fvc Vc Tlc 19.Pre carcinogenic is Berryliosis Asbestosis* Benzethe Pentamidine 20.Which of the following tissue is most radiosensitive Skeletal muscle Cartilage* Bone 17.14Turner syndrome has Chromosomal defect Xlinked Short stature* Tall stature 15.In dorsal column lesion which of the following will be lost Pain Temp Crude touch Itch Proprioception 18. Throphylline is Postural hypotension Rapid pulse Vasodlation Venodilation 21. isoprenaline . Hipocampal injury causes .Water reabsorption through ADH is done in Proximal Distal Collecting Loop of henle 26.In food poisoning death occur by Exotxin of samonella Vibro cholera * Shigella Endotoxin of Shigella 24.Minerals 22.The organ placed retroperitoneally is Spleen Transverse Bladder Ascending* Liver 26.Temporal Arteritis is diagnosis by Esr* Biopsy B/c WBC count 23.Urinary bladder in male is Completely covered by peritoneum Superiorly covers From rectum separated by fascia of Denon Villiers 27.Hydrocephalus occurs because of obstruction in I/v foramen* Sylvius Magendie Lushka 28.Long term granulomatous disease is Crohn Celiac Sarcoidosis Ulcerative colitis* 25. Chemotactic factors functions mostly in Diapedisis Margination ' Migration Acute inflammation 29.Blood spread of CA occurs 1st in Metaphysis Diaphysis Epiphyses Epiphyseal plate Cortex 32.He is suffering from Primary hypoparathyroidism Vit D deficiency 2ndry hypoparathyroidism 33. Paramesonephric duct remnant in male is Appendice testis Appendix Ovary 30.A boy presented in the ER with fracture with less seum calcium.Musle traversing the shoulder joint Crochobrachilis Sspinatus Ispinatus 31.Example of synchondrosis is Symphisis Involves hyaline crtlg b/w 2 bones Costal joint .What organ lesion causes hypothermia Preoptic Lateral hypothalamus Post hypothalamus * Pitutary 28.Sex rage Hunger Loss of Memory* Temperature disturbances 27. minimus Adducts.About antibodies Have varible2 heavy chains IgM is abundant 42.Juvenile polyps are also called Hamartomas Rectal polyps Internal haemorrhoids 37. laterally rotate Flexes .34.TGlycerides are absorbed in Chylomicrons Vldl Hdl Ldl 38.medially rotate Abducts.Erythropoietin is secreted from Yellow marrow Jg complex Macula densa Preitubualr capillaries Loop of henle 35.In gastrectomy loss of following occurs Hcl Intrinsic facor Fats Vit D 39. Gluteus medius .Median nerve supplies .Adrenal gland is separated from kidney by Peritoneal fascia Renal fascia Perinephric fat 41.laterally rotate Adducts . medially rotate Abducts. medially rotate 36.Hb is enclosed in RBC at stage of Normoblast Late normoblast Erythroblast 40. About 1st lumbrical Arises from 2nd meta carpal Is bipennate Supplied by median nerve 44.Palmar medial 1/3 Palmar and dorsal medialm1/3 Dorsal medial 1/3 Dorsal 2/3 lateral 43.Lateral part of breast supplied by Ant group Post group Lateral group Central group 46.Lymph drainage of lateral 2/3 of tongue occurs through Sup deep cervical nodes Inferior deep cervical nodes Submandibular nodes Submental nodes 48.The muscle of passive inspiration is Diaphragm External intercostal Internal intercostal Scalenes muscles 47.Posterior triangle in the neck has follwing boundries Ant ant border of SCM post omohyoid Post ant border of SCM and ant diagastric Ant post border of SCM and post ant border of trapezius 50.Lumbricals are supplied by Median neve deep br of median n Deep br of ulnar n 45.Left recurrent laryngeal nerve has Shorter route than right recurrent laryngeal nerve Branch of vagus nerve Supplies all the muscles of soft palate .Boil on the tip of the tongue will drain into following lymph nodes Submental submandibular Submental nd submandibular 49. Inferior rectal artery is a branch of Ext illiac Int illiac Sup epigastric Int pudendal artery Femoral artery .Right hepatic artery is branch of Hepatic artery gastroduodenal artery Ceoliac artery Splenic artery 52.Supplies all the muscles of larynx 51.In CRF all of following occurs except Anemia Hyperkalemia Hypocalcemia Hypoposphatemia Hyperuricemia 55.Subcapsularis sinus is present in Spleen Lymph nodes Payers patches Palatine tonsils 53.Patient after radiation vomited for 4hrs Cl is raised.Patientt has Babinski + lesion is in UMN LMN Dorsal column Ant column 57. he has Metabolic acidosis Respiratory alkalosis wirh ion gap Metabolic alkalosis 56.After illeal resection absorption of which of the following components does not take place Chylomicrons Bile Tglyceric acid Vit d Vit c 54. Structures which passes through the aortic opening is vagus and azygous vn Azygous vn Esophagus Phrenic nerve 63.58.Granuloma formation does not occur in TB Cat scratch fever Toxoplasmosis Pneumonia 60.Liver helps in immunity by Detoxfying noxious agents Kupffer cells Hepatocytes 65.Melanocytes originate from Neural crest Mesoderm Ectoderm 62.Peristalsis in the intestine inhibited by CCK Gastrin Secretin GIP Histamine 59.One of the following is not a feature of anaerobe Cough Tetanus Pus Granuloma formation Gangrene 61.IJV drain in Petrosal sinus Sagittal sinus Straight sinus Sigmoid sinus 64.Sour and hot is perceived by tongue through . In testis seminferous tubules show following under M/S Sertoli cells Spermatogonia Mixed cells Spermatocytes Sperms 68.Cholestatic cholangitis is caused by Colonercis selecans diphyllobothrum latum Schistosoma Entameoba 71.Von ebner cells Folate papillae Pain fibers Nociceptors 66.The mc burney point indicates the presence of tip of appendix.After appendicectomy what may be damaged Inguinal ligament Superior epigastric artery Illhypogastric nerve Spermatic cord 72.Following is true about appendix Appendix ha incomplete muscular coat Appendix not attached to mesentry Supplied by ceoliac artery .ADH is inhibited by Alcohol Angiotensin Insulin Renin 67.Constricting afferent ducts in kidney reduces Gfr Rpf Creatinine clearance Glucose claerance 69.A boy is malnutrioned and has edema due to protien loss his Plasma colloid pressure is decreased Increase hydrostatic pressure Blocked lymphatics 70. One of the following is not a branch of External carotid artery Lingual A Ophtalmic A Occipital A Maxillary A Suprficial temporal A 74.Medial side of hand is supplied by Median nerve Ulnrr nerver Radial nerve Axillary nerve 77.Patient wirh Gravida 4 Par4 Abortio 0 with anemia and occult blood in stool Fe deficiency anemia Pernicious anemia Anemia of chronic disease Sideroblastic anemia 79.73.Injury to the neck and head of fibula causes damage to Sural nerve Sciatic nerve Tibial nerve Common peroneal nerve 80.Apex of the femoral triangle Lies under inguinal ligament Contains small saphenous vein Contains nodes Is pointed downwards 76.Femoral artery can be palpated in Mid inguinal lateral to pubic tubercle Mid point of inguinal ligament Adductor canal 75.Nerve to ligamentum teres capitis is Femoral n .Vomiting center is persent in Hypothalamus Pitutary Pons Medulla Midbrain 78. Role of middle ear bones is Transmit sound Amplify sound Does not respond to low noise Are sesamoid bones .In tissue bleeding occurs because of Loss of coagulation factors Congenital disorders Endothelial damage dec FDF 86.Obturator n Medial femoral n Lat circumflex femoral n 81.On bronchoscopy bronchoscope will 1st enter Sup bronchus Apical bronchus Inf bronchus Middle apical bronchus 84.Artery to head of femur is Medial femoral circumflex a Lateral femoral circumflex a Obturator a Femoral a 82.Following is not a feature of DIC Dec FDF Dec BT Dec PT Dec thrombin time Dec platelets 87.Muscranic receptors are present in Postgaglionic parasympathetic Postgaglionic sympathetic Preganglionic parasympathetic Preganglionic sympathetic 83.Essential fatty acid is Linoleic Palmitic Citric Oxaloacetic 85. Direct inguinal hernia lies Lateral to inf epigastric a Medial ti sup epigastric a medial to inf epigastric a Medial to pubic tubercle 89. Hot souses on tongue. M. Kazakhstan) FCPS -1 SURGERY 15.D (Semipalatinsk. a) Vit B 12 4.D (Semipalatinsk. Which is not a cause of anaerobic? a) Granulation formation . Pallor.Most drug metabolism occur in Kidney Liver Spleen Intestine 91. taste sent co taste centre by a) Chemoreceptor 5. M. End artery Spleen Bone Brain Posted by Dr Sultan Ahmad 1 comments Reactions: SURGERY October 2008 Dr Qasim Hussain. OCT 2008 1. Kazakhstan) Dr. TLC 3500cm.Muhammad Irfan.Thrombosis is initiated by Leukotrienes Prostacyclin Arachidonic metabolites Thromboxane A2 90.88. Lithargic patient with difficulty of walking have Hb 5. Regulation of PO2 and breathing rate is controlled by a) Carotid bodies 3. A boy with lean body mass with fracture of arm osteopenia a) VIT D deficiency 2. In lymph nodes. Smell fibers from nose a) Does not reach Thalamus . Atypical Tuberculosis a) Does not responds to common Anti TB drugs 9.6. If large amount of Dextrose water is given to a patient a) Decrease ADH 12. Interstitial fluid is a) 1/3 of TBW 13) Not includes in enzymes of necrosis a) Protease b)Catalase c) Hyaluronic acid 14. in Liver Cirrhosis a) Increase PT 19. Right Gastric artery is a branch of a) Hepatic artery 7) GFR is increased by increase in resistance a) Efferent arterioles 8. Which blood group have no agglutinin? a) AB+ 10. seen a) Caseous necrosis 15. Essential fatty acid is a) Leniolenic acid 11. Iron deficiency anemia best diagnosed? a)Increase TIBC b) Serum ferritin 16. Bones of Ear a) Amplify sound waves 17) Immune mechanism of Liver lies in a) Kuffer cells 1 8 . SA node is located on a) Above the sulcus terminals 26. Helps in diagnosis of colonic carcinoma a) CEA 33. Death from food poisoning occur with a) C. Botulism 27. Which is a characteristic of Cancer? a) Pleomorphism---ANSWER b) Increase mitotic figure .9. Pretracheal fascia completely covers a) Thyroid 28. SA node is supplied by a) RCA 25. 7. Subcapsular afferent lymph is in a) Lymph nodes 24. Lymph nodes of lower lip drain into a) Submental and submandibular lymph nodes 22.10 30. Heat Loss Mechanism lies in a) Posterior hypothalamus 21. Insulin deficiency a) Ketogenesis in liver 23. What causes depolarization? a) Na+ influx 32. What is called juvenile polyp? a) Hamartomatous 34. Contusion to Lateral border of Fibula result in a) Common peroneal nerve damage 3 1 . Narrowest part of male urethra is a) External meatus 29.20. Which cranial nerves are parasympathetic? ANS: CN 3. A 20 year old boy with Headache and photobphobia With high grade fever. Wound heals poorly if absent a) VIT C in diet 41. 2. Neurogenic shock is a) Vasomotor injury 4 3 . possibly due to a) Increase level of prolactin in serum 46. LP shows in CSF neutrophils 40. XXY is a) Klinefilter syndrome 45. Patient with some pituitary tumor has infertility.000 Glucose 3. a) Decreased bile salts reabsorption 40. Ansa cervicalis is a) Hpoglossal nerve and C1. Biopsy will show a) Normal hepatic architecture 48. In Down syndrome seen a) Short stature 44. Increase of Platelets aggregation is by: a)Thromboxane A2 36.5mg a) Acute Bacterial meningitis 37. If resected 10inch ilium. Cause of lymphoid tumor in HIV patient is a) EBV 47. 3 38) Mitral valve lies between a) Left atrium and left ventricle 39. a patient developed HAV completely recovered.35. High level of creatinine is seen in a) ESRD 42. Neurohypophysis contains a) Pititicytes . Posterior to Urinary bladder is a) Fascia Donovillaris 53. Medially rotate and abduct the thigh is . Dorsal column damage leads to loss of a) Proprioception 56. Melanocytes are derived from a) Neural crest cells 50. In adults. What is buried in lateral sulcus? a) Insula 57. in bronchial Asthma. Malignant malaria is caused by a) Falciparam malaria 55. seen a) Eosinophilia 6 7 . Where is lesion? a) Broca's area 58. Erythropoietin is produced by a) peri tubular capillaries 5 1 . IgE is present on a) Basophils 62. the spinal cord ends in a) Below L1 54. ADH works on a) Collecting ducts 52. Spinal cord is supplied by a) Vertebral artery 59. A patient has some gastric problem a) Iron Deficiency anemia 63. If Gastric mucosal permeability is increased a) H+ comes out in lumen of stomach and damage mucosal barrier 64. If infection of abdomen spread to retroperitonium. A patient can utter few words. Complements are decreased in a) SLE 60. Most common cause of malignancy is a) Asbestosis 66. END arteries are seen in a) Spleen 77. Anterior Cardiac veins into a) Right Atrium 65. can reach to colon? a) Descending colon 61.49. Direct inguinal hernia in a) Medial to inferior epigestric artery\ 94. 1st heart sound is best listened on a) On Mid clavicular line. S3 is produced by a) Rapid filling of left ventricle 85. During fasting. Aneurysm of abdominal aorta compresses a) Thoracic duct 91. Structure not related to Right kidney is a) Descending colon 89) Temporal arteritis best diagnosed by a) Temporal artery biopsy 90. D deficiency 92. circumference of mid arm is decreased after one month due to a) Decreased protein of body 96. Patient with bronchial asthma has a) Decreased FEV1 88. the cause of hepatitis is a) CMV 81. mostly due to a) Vit. Achalasia is due to a) Absent ganglions in LES 95. tetany is seen. 5th ICS 84. Neurovascular bundle is in a) inner and innermost muscles 80. Myasthenia gravis improves on taking a) Anti cholinesterase drugs 98. Which structure passes through shoulder joint cavity? . END diastolic volume is a) 120ml 82. On attempt to measure the B. Myelination of peripheral nerves are by a) Schwann cells 97. Berry aneurysms are mostly found in a) cerebrum 86. in EAC. most probably due to a) CA breast 83. Cause of edema is a) Decreased colloidal osmotic pressure 87.P. Source of folic Acid is a) Vegetables 79. Hydrocephalas occurs blockage of a) Aqueduct salvias 99.a) Gluteus Medius and and Gluteus Minimus 78. Palpable anterior Auxiliary lymph nodes. Liver transplant. Remittent of paramesonephric duct in male is a) Appendix testis 93. young patient came with fracture of arm. a)tendon of long head of biceps 100. Most sensitive to radiotherapy a) Lymphocytes 103 What is branch of ICA? ANS: Ophthalmic artery 104. what will decrease? ANS: Venous compliance 119: Is a sign upper motor neuron lesion? ANS: Babinsky sign 120: If numerous epitheliod cells are surrounded by lymphocytes. which hormone? ANS: Thyroid hormone 109: Diarrhea improves on fasting is ANS: Osmotic 110: Source of energy to brain during fasting: ANS: Amino Acid 111: Which of the following is not antiseptic? ANS: Acetyl Salicylic Acid 112: Aging determined by bones? ANS: Angle of mandible is less obtuse 113: Lacrimal gland is supplied by? ANS: Ptyrigopalatine ganglion 114: During Episiotomy. its lateral part is pulled down by: ANS: Subclavius muscle 108. damaged ANS: Lavator Ani muscles 115: Broadmann's area 312 is also called ANS: somatosensory area 116: Surfactant is produced by? ANS: Pneumocyte II 117: Sigmoid sinus is continuous with ANS: Internal carotid artery 118: In shock. called ANS: Caseous necrosis (Granuloma) 121: Fracture of medial epicondyl of humerous. on hand lost sensations? ANS: medial 1/3 of hand palm. Squamous cell is cervix is an example of ANS: Metaplasia 105: Thoracic duct drain all except ANS: cerebrum 106: Increase blood in heart chamber is ANS: Isovolumatric relaxation 107: If Clavicle is fractured. Impaired metabolism of purine cause in urine a) Increased uric acid 101. Most important muscle of inspiration is a) Diaphragm 102. and dorsal 1/3 of hand . For uncoupled oxidative phosphorylation. 122: Winging of scapula is due to damage of? ANS: Serratus anterior muscle 123: Right adrenal gland is ANS: Pyramidal shape 124: Submucosal gland is in ? ANS: Duodenum 125: Which is true? ANS: Right brochus is longer 126: In bronchial asthma? ANS: increased breathing by stimulating beta –adrenergic 127: inferior rectal artery is a branch of ANS: internal pudendal artery 128: Is not a carcinogen? ANS: Cyclophosphamide 129: Microsomal metabolism is ANS: Liver 130: Half life is ANS: time to metabolism of half of drug 131: Most of the drug is metabolize in ANS: Liver 132 Diverticulosis occurs: ANS: sigmoid colon 133: Stimulation of RBC from bone marrow by? ANS: Erythropoietin 134: Occult blood in stool is in ANS: Iron deficiency anemia 135: Partial gastrectomy ANS: Pernicious anemia 136: Fat store is increased by ANS: insulin 137: In stress which hormone is released? ANS: Cortisol 138: Temperature regulator centre is in? ANS: Hypothalamus 139: In heart ANS: Right atrium lies anterior to left atrium 140: In hypovolumic shock ANS: Rapid intravenous fluids 141: Not seen in DIC ANS: Thrombocytosis 142: Best method of diagnosing ectopic pregnancy ANS: Laparoscopy 143: superior parathyroid is ANS: supplied by superior thyroid artery 144: The supra renal gland ANS: The supra renal gland is supplied artery which is direct branch of aorta . congenital omphaloseal? A) Not associated with any congenital Heart diseases B) Associated with VSD? c) Associated with ASD? D) B and C? 157. A patient developed type I anaphylactic reaction. transverse muscle 149: Lymph from posterior 1/3 drain into ANS: deep superior cervical lymph nodes 150) Negative Montoux test is seen in a) Steroid therapy b) Immunosuppressive Therapy 151) Permeability of a substance depends on ANSWER a) Total cross sectional area-b) Carrier c) Receptors 152. but if supported up to 30 degree. Treatment of choice? ANS: Adrenaline 153: Amputated leg is the result of ANS: Amniotic bands 154. can do.Polysithemia rubra vera associated with a) bone tumors----ANS b) lung diseased c)Living on Himalayan mountain 158. Ptyrigopalatine canal is supplied by wrong . Actin physically binds with A) Troponin B) Tropomyosin ANS C) Myosin 156. Rectus muscle Posterior fibers of internal oblique. Widal test is positive? Don’t Know A) 1: 80 for O antigen B) 1:180 for O antigen C ) 1: 100 for O antigen D ) 1:180 for O and H antigen E) 1:80 for H antigen 155. anterior fibers of internal oblique. What is? ANS: Supraspinatus muscle 148: incision between umbilicus and ribs passes ANS: external oblique.145: Gas gangrene is not caused by ANS: Bacterial endocarditis 146: permeability of a substance depends on ANS: Total crosses sectional area 147: A patient can't abduct arm. portal entry of infection by hematogenous route a) Through epiphysis-ok b) Metaphysis c)Diaphysis 167. Common side effect of Isoprenlene.I wrote This answer . tip of conical ceacum has appendix attached—ok? 165. InG is in Largent quantity 164. Appendicectomy can damage to Inferior epigestric artery iliohypogastric nerve I wrote This answer . Cholangiocarcinoma caused by a) S. Splenomegaly 163. Nitroglycerine. Don’t know exact b) Present in pinna of ears c) is fibrocartilage 168.a) Deep branch of petrosal nerve. mansonei b) S. Don’t know exact ilioinguinal nerve\ 166.ANS OK b) superficial branch of petrosal nerve c) maxillary nervr branch 159. Japonicum c) S. Muscrinic receptors respond to a) preganglionic parasympathetic . Primary cartilageneous joint a) Connect the to bones together. Colonarisa ----ok 161. Histamine. DIC c. What passes through ligament teres of head of femur a) obturator nerve branch---ok b)branch of femoral artery 162. In Osteomylitis of long bones. is. Auto antibodies to platelets ---ok b. Insertion of 1st lumbrical 160. The premalignant condition is a) Psoriasis b) Cervical erosion c) myelodisplastic syndrome ok d) leukoplakia 169. Contraindication for platelet a.Don’t know a) Hypertension b) increased bronchial constriction c)Lacrimation d) ? 170. At Mac burny point located Tip of appendix Base of appendix In infants. Autosomal Dominant is a) 176. Stratified cuboidial epithelium is present in a) Ducts of Salivary glands ok b) Distal convoluted tubules of nephron c) Lacrimal gland ducts 173. The flow rate of urine is if 2 ml / min . The concentration of a substance in the blood is 10mg and in in urine is 100mg. Regarding RBC a) buffer as accepter of oxygen b)Only mature forms contain Hemoglobin c) Adult hemoglobin is alpha and Gamma d)Not transport oxygen 178. Amyloidosis result in a) Adenocarcinoma stomach 177. What begins with Premalignant conditions? a)Adenocarcinoma stomach b) c) 172. 35 year old female Type I diabetic got fracture due to osteoporosis. Varicose veins a) Somewhat associated with smoking ok . ADH will decrease in a) Nausea b) Pain c) Anxiety d) Decrease serum osmolrity 180. The cause? ANS: Insulin a) insulin b) Estrogen 175. what is its renal clearance? HINT: use formula UV/ p ANS: 20 174.b) Postganglionic parasympathetic Sok 171. Does not cause Gangrene a) Bacterial endocarditis b)???? ANS 179. Cardiac Muscles are 189. A patient with sepsis is best diagnosed with a) Pulse > 100 b) Pulse > 120 c) Positive bacterial culture 183. Gross increase level of HCO3 a) Persistent vomiting b) CRH c) Lung fibrosis 185. Femoral artery is best palpated on? ANS: mid inguinal ligament .b) Only in lower limbs c due to defect in adventitia 181. Microtubules are part of a) Centrioles b) Cell membrane 187. intestinal motility is decreased by a) C C K b) Gastrin c) Trypsin d)lipase e) Insulin 184. BP is increased when there is a) Increased sympathetic output b)Increased total peripheral resistance 188. Primary brain vesicle is a) Mesencepholan b) Telencepholan c) Diencephalan 186. Primary ossification centre is a) Epiphysis b) Metaphysis c) Diaphysis 182. RIZWAN RIAZ 1. B cells+macrophages c. A. B+T cells e.malariae c. In type 4 Hypersensitivity mainly cells are a. P. In brochhocscopy. Ent histolytica B. E. 2: Malignant tertian malaria caused by a. Carotid cartilage is ANS: on base of thyroid 191. B. None.190. Fatty Acids are transferred from adipose to liver in the form of? a) Chylomicrons b) HDL c) LDL d) VLDL Posted by Dr Sultan Ahmad 2 comments Reactions: MEDICINE Oct 2008 15th OCT 2008(medicine & Allied) BY DR. P. Gastric Artery 4: Parasite transmission e no feco-oral route A.Vernicularis C. IGE+T cells d. Tenia solium . P. P.vivax 3: In duodenal perforation the artery involves. Gastro duodenal artery Lt. P.Gastric Artery Splenic Artery Rt.knowlasea d. C.falciparum b.ovale e. T cells+macrophages b. Fist structure to be visualized is: ANS: Right lower bronchus 193. D. 5: Antimalarial drug e can be used other than malaria A. Pyramidal in shape D. Quinine 6: About RT adrenal gland choose correct A. Organism is B-hemolytic. non_lactose 8: Melanocytes derived from A. Ectoderm C. Roth spots on 7th day C. All pts becomes carrier B. Lungs . Heart B. Basoquine C. Lymphocytes C. Kidney D. Arthemether B. Pharyngeal arch E. Has 3 arterial supply C. Liver C. Choroquine D. Myeloprolefrative 12: In Cushing decrease amount of A. Has same size in adults & infants B. Monocytes . Mefloquine E. Endoderm B. Cervical erosion 2. Lateral plate 9: Most of drugs are metabolize in A.10: ½ life of a drug Options not remember 11: Premalignant condition 1. Separate form kidney by perinephric fat 7: Typhoid fever A. Neural crest cells D. Neutrophils B. Basophiles D. Hepatic vein B. Rt lower lobe of lungs 16: Comatose patient are more prone to regurgitation A. Ascending colon C. Stomach full B. Pelvis kidney v. New born D. Horse shoe kidney 21: Drug with inhibit COX-1 as well as COX-2 A. Liver B. Non rotating kidney iv. Pons B. Crossed ectopic kidney ii. LMNL 14: Loss of accommodation reflex due to occulumotor nerve site of lesion A. Ribs D. Naproxen . Diclofenic B. Unilateral agenesis iii. Midbrain ? 15: Aspiration of gastric contents go in the A. UMNL B. headache Mydriasis associated with no option of meningitis Options not remember 20: An I/v urograph of old boy shows Excretion of dye normal at Rt side but absence of shadow of Lt kidney instead there is a small shadow above the bladder on Lt side Most likely i.13: Babinski sign -ve in A. Rt lateral position C. Lt lateral position 17: Vein that doesn’t directly drain in the inferior vena cava A. Descending colon 19: Fever. Sleep C. Medulla C. Lumber vein 18: Rt kidney doesn’t relate to A. 2nd heart sound C. Body fat <5 g/dl D. Brain C. Receptors destruction C. Increase ketogenesis by liver 27: Thirst increase due to A. Aspirin 22: Rx of salicylate poisoning which can increase the excretion of salicylate A. Lungs E. Resistance to insulin B. Eryropoietin D. Lingual tonsils C. Glucagons lipogenic B. Insulin lipolysis C.C. Lymph node E.C space in midclavicular line 29: AV valve closure is responsible to A. Kidney D. Diuretics D. Spleen 24: End arteries are in A. Epinephrine has lipolytic effect 26: IDDM main disturbance by causing A. Aldosterone C. 10% D/W 23: organ having subcapsularis & receives afferents A. 3rd heart sound . Heart 25: Fat stores in body A. Angiotensin 2 28: Sound of mitral valve is best heart at Left 5th I. ADH B. Dehydration state C. Thymus D. I/v NAHCO3 B. Palatine tonsils B. Spleen B. 1st heart sound B. RT auricle C. To increase the gastric acid secretion 38: I/v large amount of DEXTROX will inhibit A.R C. Due to decrease interstitial fluid osmotic pressure C. Upper part of Crista Terminals of R.A 36: Boy having edema due to renal A. Saturated F. 4th heart sound E. Increase C. Decrease C. Loss of vasomotor tone B. None of the above 30: During cardiac cycle ventricles are filled in A. Decrease capillary pressure 37: Histamine theophyline & Aminophylline have same action A. Decrease plasma colloid osmotic pressure B.D. Atrial systole B. CORTISOL B.O C. Palmitic acid B. Cardiac output 33: CCF pt developed tachycardia best management of RX OPTIONS NOT REMEMBER 34: Site of SA node A. Lower part of Crista Terminals of R. Venous Compliance B.O 32: In Hemorrhage after compensation with one of the following decrease A. Isovolumetric relaxation D. Len oleic C. ADH INSULIN . Rapid inflow 31: In neurogenic stock A. Sulcus terminalis D. H.A B.A 35: Which one is essential Fatty Acid A. Rapid ejection C. Squamous cell carcinoma C.C. Medulla C. Ant. Thrombin time E. AC. Esinophils B. Marfan syndrome B. Choroidal artery C. Basophiles . THYROID D. CH. Reticular Formation 45: Arterial supply of spinal cord A. APTT C. Duchene muscular dystrophy D. Vertebral artery B. Choroidal artery 46: Mast cells r released 4om A. OXYTOCIN 39: Which 1 of the following is aggressive in nature? A. Fibrin products 43: Vomiting centre located in A. Hypothalamus B.inflammation B. Plat. Basal cell carcinoma D. PT B. Pituat. Pons B. Midbrain D. Small cell carcinoma B. Glycogen storage disease C.count D. Cystic fibrosis 42: In factor 8 deficiency specific test is A.inflammtion 41: X-linked disorder A. Nevis cell 40: CHEMOTAXIS is a feature of A. Cerebellum 44: Arousal is controlled by A.gland C. Post. Lymphocytes 47: Regarding Glomerulotubular balance. Derived 4om 1st pharyngeal arch 49: Radiosensitive tumor A. Inc. Covered by pretracheal fascia C. Supplied by sup. PT B. Inc. Bone B. Inc. Thrombin . H. APTT C. Lymph node C. Dec. H. Located anterolateral 2 thyroid gland B.P E. Thyroid artery D.O C.R 54: Regarding Microscopic feature of CASEOUS Necrosis OPTINS NOT REMEMBER 55: Female pt. Urine O. Cartilage 50: Rh incompatibility related 2 Type 1 hypersensitivity reaction Type 2 hypersensitivity reaction Type 3 hypersensitivity reaction Type 4hypersensitivity reaction None of the above 51: Regarding IGG2 cells OPTIONS NOT REMEMBER 52: Regarding Th-2 cells OPTIONS NOT REMEMBER 53: Hypovolumic shock A. OPTIONS NOT REMEMBER 48: Regarding parathyroid gland. Hot peripheries B. Muscle D. PT & aPTT D. Choose correct 1.R D.C. C. A. on oral contraceptive & u want to give her oral anticoagulant u must chk A. Neutrophils D. Maximus B.d with c/o loss of sensation of RT. Dissecting aneurysm C.P C. Injury to A. Thumb during c-setion after giving epidural anesthesia. I. its due to. Little toe with difficulty in eversion of Rt. S2 59: Below the medial epicondyle of humerus passes A.COZ OF DEATH A. Temporal arteritis 62: Artery enters in cranium & becomes the principle artery of brain A. B. C7 E.E. Gravis B. Q. Berry aneurysm B. Ulnar nerve D. S1 E. Median nerve B. A. L4 C. C8 58:Diagnosed pt. Sartorius D.P. C5 C. Mys.Minimus C. Horner syndrome 61: Pt. Foot. G. Femorus 57:Pt. came after 1 day of RTA with c/o unable of standup after seated position. feel tingling sensation on rt.C. Long thoracic nerve E. Marfan aneurysm D. of sciatica cam in o. L3 B. Inc. Musculocutaneous nerve 60: Intermittent Diplopia is a classical sign of A. L5 D.Autopsy shows thinning of tunica media. C6 D. Probably muscle injury. C4 B. G. A. Gracilis E. Common Carotid .p. Fibrin products 56: Pt. expire due to sudden inc. Radial nerve C. Iron E. 10% D/W 67: Coz of FATAL DIARRHEA A. Psychosis C.B.Cholrea B. E. V. Folic acid . B6 D.Botilinium C. Resp. Choral hydrate C. flouxitin B. NIACIN BIOTIN B. Blood transfusion C. Maxillary Artery E. E-poi tin 69: Pellagra is due to def. Adrenaline B. Dialysis B. Vita. Amitriptyline C. Nausea & vomiting B. Pin point pupil D. Temporal 63: 1 of the following is not epileptogenic A. Vita. Internal Carotid D. Dopamine & Dobutamine C.Coli 68: Anemia of CRF is best treated with A. C C. Folic Acid D. of A. C. External Carotid C. I/V NaHCO3 D. Imipramine 64: Opoid Toxicity causes A. Triazolam D. Bromide B. depression 65: 1 of the following is not classified as non barbiturate hypnotic A. Chlormethimazole 66: In CARDIOGENIC SHOCK A. Ruffini C. HYPOTHALAMUS) C. Brocas area 75: Regarding lumber puncture which one is correct A. Post. Free nerve endings D. Left Bronchus crosses it C. Hippocampus . Cricopharangeus B. Pacinian B. rigidity. Cerebellum B. 4 C. Parkinson disease C. Sup. 5 D. Lat. 3 B. Thalamus E. Temporal gyrus D. Hypothalamus B. 9 72: Tactile two point discrimination by A. Angle of needle must be 45 degree E. UMNL D.70: Site of constriction of esophagus. hypokinesia A. Advice to patient stands up immediately after L. Between L4 & L5 B. Wernicke’s area B. Prefrontal gyrus C. Chose correct one A. CSF pressure in mm of Hg C. Spine fully extended D. LMNL 74: Motor aphasia due to lesion of A. 7 E. When enters in diaphragm D. Preoptic hypothalamus (NO OPTION OF ANT.P 76:Heat loose of body is controlled by A. At cardiac end 71: Single cranial nerve which doesn’t contain parasympathetic fibers A. Merkel disc 73: Patient with resting tremors. Hypothalamus D. Vibration 78: The first step occurs when light falls on retina A. TBM D. Encephalitis B. Supplies to middle ear D. Temperature D. Fear & rage B. FEV E. Diagnosis A.V C. Hyper sexuality C. Viral meningitis 82: Lateral sulcus of cerebellum A. Fine touch C. Hyperphagia 80: Regarding facial nerve which one is correct? A. I.L.R.C D. glucose 40 & mainly lymphocytes. Physiological dead space . O2-55 & CO2-25 cause of deranged ABG’S A. 11-Cisretinal to all Trans retinal 79: Lesion of mamillary body causes A.54. Salicylate poisoning 83: Asthma diagnosed by A. Bacterial meningitis E. Loose of Recent memory D. T. Pain B. Supplies to stapedius muscle 81: CSF having proteins 400 mg/dl.V B. Carbon mono oxide poisoning B. Purely sensory B. Insula B. Tidal volume 84: Diagnosis of Emphysema A. Give chorda tympani to posterior 1/3 of tongue C. Cryptococcus virus C. R.77: Damage to dorsal column tract causes ipsilateral loose of A. Hysterical hypoventilation C. Occipital lob 83: PH-7. External intercostals muscle 87: Regarding hemoglobin A.A pressure 92: Diagnosed case of thyrotoxicosis came to you with heart rate of 190b/mn. Lidocain C. Sternocladomastoid D. Best treatment is A. Beta-1 D. Nicotinic 89: Stress hormone of body called A. Contains iron in ferric form C. Alpha-1 B.B. Sarcomere B. V/Q-1 85: Muscle of quite inspiration A. Physiological shunt C. Scalenius posterior 86: Muscle of forceful inspiration A. Alpha-2 C. Tight junction E. Epinephrine NO OPTIONS OF CORTISOL 90: Heart muscle works as syncytium because of A. Beta-2 E.affinity D. Gap junction D. Digoxin B. Causes acidosis when add with H+ 88: Receptors mediates increase heat rate A.V pressure B. EDV of L. Diaphragm B. Verapamil . SER C. Intercalated disk 91: Preload depends on A. V/Q. ACTH B. V/Q-0 E. Intercostals muscle C. Adult contains alpha-2 & gama-2 B. EDV of R. O. Na+ -K+ . Propanalol 93: Saw tooth P waves in ECG associated with A.D. Interstitial 98: GFR can be calculated by A. DEC.D.2Cl co transportation in thin ascending L.F B.A) Blocks ACE enzyme 95: Cyanosis due to A. Acidosis B.A spironolactone OPTIONS NOT REMEMBER 102: A.O. S.creatinine C. Urea 99: S/Chloride maintained by kidney through A. Alcohol B. Atrial flutter B.C. B.H B. Hemorrhagic Shock C.U. Anemia hypoxia B.O. 24 hours urinary out put B.V block 94: Captoprill (M.F C.N D.H is inhibited by A. When deoxihaemoglobin is more then 5g/dl 96: Pulmonary vasculature resistance increase in A. I.H 100: INC.C. Na+-K+-2Cl co transportation in thick ascending L. Pain . Plasma D.O. Rennin C. A. Aldosterone causes commonly A. High altitude 97: 1/3 of total body water contains A. K+ retention 101: M. Sodium depletion D. E. Puituicytes 108: In autoimmune diseases of thyroid antibodies against A. Macula densa 104: Hormone which causes relaxation of all G. G. Sucrose E.coz of disturbance is A. Juxtaglomerular cells C. CCK E. Follicular cells D.P B. Cholinergic B. Gastrin C. Adrenergic C. Glucose B. V. Fructose C.I. Thyroglobulin C.D. Primary Hypothyroidism due to Hypothalamus 107: Neurphyposis contains A. Secretin D.I. TSH B. TSH & when TRH given TSH slightly rises. Exercise 103: Erythropoietin secreted from A. Peritubular cells B. Maltose 106: Pt. Lactose D. Secretory cells D.P 105: Which causes diarrhea when it reaches to large colon? A.I. with puffy eyes & edema with dec. C cells 109: Amniocentesis shows 47XXY .T muscles A. Beta H. Thyroid gland 115: Hydrocele is the aculummation of fluid A.E 113: Tumor marker of Thyroid carcinoma is A. Epidydimis 116: After gastrectomy pt. Alpha fetoprotein D. Cortisol D. Tunica vaginalis D. Vas deferens E.G 114: Which hormone maintains body temperature by oxidative phosphorylation A. Ankylosing spondilytis B. C. Klinefelter syndrome 110: In pregnancy cause of Hyper Parathyriodism A. . Turner syndrome B. Reiters Syndrome C. Scortal ligament C. Adrenogenital syndrome C. Pernicious anemia. Delayed bone loss after menopause B. Prim. Nor adrenaline C.A D. Becoz estrogen causes inhibition of bone resorption 111: Estrogen doesn’t have action A.A. Iron def. Adrenal B.E. Megaloblastic anemia due to folate C. came with c/o A. Hypoparathyroidism B. S. Inc.L. R.C. serum Cholesterol D. anemia B. Follicular growth C. Calcitonin C.A B. 112: HLA DR 27 associated with A. Gubernaculums Testis B. Pneumococcal Pneumonia 124: Pt. Thyroid 122: Juvenile Polyps are A. Diagnosed as a case of OBSTRUCTIVE JAUNDICE & now cam With c/o bleeding tendency. Colorectal Carcinoma C. came with distortion of nose.K due to DEC. Dubin Johnson Syndrome D. D. Which 1 of the following cause. AFB +ve. Hepatitis B 118: IN Tumor immunity liver has a function of A. Of VIT.E.Japanicum D. Testis D. Rotor syndrome C. S. 121: C. Clonorchis –Sinesis B.B C. Colonic adenocarcinoma 123: Granuloma formation is not associated with A. came with jaundice Hb-10g/dl. Sarcoidosis B.whats Ur diagnosis. To detoxify the toxic substances B. Echunococcus Granuloses 120: Old age pt. Hyperplasic polyps C.A is a tumor marker of A. T. A. Breast B.Retic 10%.whats Ur diagnosis . absorption from gut C.Bili-2. Globulin Synthesis 119: Cholangio-Carcinoma caused by A. A. lymph nodes palpable.U/S abdomen normal. Defi.117: Old age Pt. Protein synthesis C. Gilbert Syndrome E.S. S. Acute Hepatitis B. Due to bile stones B. Meckels diverticulum’s B. E. Hereditary non polpyposis D.8mg/dl.Hematobonium C. E. Sjogran Syndrome B. Cox 1 Cox 2 133: Which 1 of the following factor causes Delayed wound healing . Xerostomia. conjunctivitis sicca. D.M B.I. Have tendency to get infection of A. D. associated with A. Vesmin C. Desmin 126: Which 1 of the following is the diff. Thromboxane A2 B.V B.P D. H. Coal C. C. Throboembolism C. B.B Sarcoidosis Leprosy None of the above 125: Which 1 of is not a Tumor marker A. Chicken Pox 129: Diagnosis of H.V when A. Alpha 1 antitrypsin B. Tobacco B.I. Pneumocystitis carnii E.S.V pts. Less CD4 cells count B. Prostaglandins C. b/w malignant & benign tumor A. Metastasis 127: Workers Having Increase chances 2 develop Malignancy A. Less CD8 cells count 130: Joint pain.P. H. Asbestos worker 128: H. Scleroderma 131: Which 1 of the following is not associated with Dry gangrene? A. Bacterial Endocarditis 132: Platelets aggregation is due 2 A.A.V C. T.B. Uric Acid D.What will be with blood insulin and Glucogan levels at the time when he crosses the win line? 5.T B. Steroid intake 135: Blood group having no antigen to A & B A+ B+ AB+ O-O+ 136: About leukocyte adhesion factor responsible Integrin Selectin 138: In humans the End product of PURINE metabolism is A. A.Mandibular fracture occurs just brfore the mandibular foramen. Lactic Acid E. Blood flow Vit. Urea C. E deficiency 134: Diagnosed case of tuberculosis PPD or Tuberculin test found to be negative Because of one of the following factor A.1 gram of protein contains calories? a) 4 calories 3-7th lobe of lung is called a) medial basal 4-A marathion runner wins a race. Xanthin B. Immunosuppressive C. Pyruvate 1.Inc. C Deficiency Vit. a) loss of taste to anterior 2/3 of tongue b)Myelohoid muscle is unable to stabalizse mandible? .Occulocardiac reflex mediated by a)CN5 2.T. followin if true about it onset of action a) 5 min b)5 sec .2 15)Head of Humerous is supplied by a) Anterior circumflex artery b)Posterior circumflex artery c)Suprescapular artery d)Subscapular artery 16) Gene are composed of a) RNA b)DNA c)Extrons d)Introns 17) I/V dopamine is given to a patiend.there is a)decreased palatlets 9-what is inferior in perenium? a) pereniul membrane 10)spleen is a) mesodermal b)endodermal 11)according to recent recommendations. recurrent MI should be treated with? a) Anticoagulants b) Lipid lowering drugs c) Beta blockers 12)Stratified squamous epithelium is present in a )palatine tonsils b)pharynx c)Ovary 13) Low voltage QRS comples ios seen in a) MI b) Bundel branch block c)Hypertension d)IHD e)Pleuritis 14)Isthmus of thyroid is present at a)C1.c) Loss of sensation to lower teets? 6-Head of femer a) has anger of 125 with shaft b)posteriorly completely covered by capsule? c)epicondyls are in parallel line with shaft 7-Lymph nodules are present in a)Spleen b)Thymus c)Lymph nobes 8-In DIC . Following is true about its arm-brain circulation a)60 sec b)5min c)30min d)5 sec e)? 19) DVT is most common in a)Femoral vein b)popleteal vein c)Superficial vericosed veins of leg 20) Which of the following is not a tumor marker a)Acid phosphatase b)PAP c)Hcg d)AFP e)CEA 21) Pus contains a)Dead bacteria b)Dead neutrophils 22) The most common cause of fatty liver in our society is a) Alcohol b)Hepatitis B and C c Drugs 23) Carotid bodies respond to a) increase blood H+ b)Increase blood co2 24) In anterior Duodenal perforation .c) 60 sec 18)I/V sedation is given to a patient. the contents will go to a) Right iliac fossa b)Left Iliac fossa c)Anterior subhepatic space d)posterior subhepatic space e) small bursae 25) Metabolic alkalosis results from a)Ingestion of ammonium chloride b)ingestion of carbonic anhydrase inhibitors 26)A Diabetic patient fas urine ketones and urine glucose ++ the cause is ? a)Hyperglycemia b)Insulin difficiency . d) 12.27 Response to chemotherapy is because it causes a)Atrophy b)Apaptosis 28)Dorsal column damage results in Ataxia because a)loss of pathway from dorsal column to thalamus b)Loss of proprioception input to cerebellum c)Loss of proprioception to Thalamus d) Loss of proprioception to Hypothalamus 29)Which of following nerve passes dorsal to brain stem? a) 3 . with Hypercellular bone marrow/\ a)Due to drug he is using for anaemia b) Leukemia c) Iron deficiency anaemia d)G6PD deficiency 31) The sign of cerebellar disease is a)Adiadohakinesia b)Static tremor c)Hypertonia 31)Neurohypophysis drain secretions a) Adrenergic neurons b)cholinergic neurons c)free nerve endings 32)CSF a)has osmolarity <1040 b) pressure <10 mm of water c) protein >200 33) Stimulation of Alpha adrenergic neurons 34)End arteries are present at a)Brain b)Heart c)Bone 35) In Tempomandibular Joint Dislocation. c)5. e) 9 30-A patient has hb 6.000. b) 4. palatelet 450.What structure prevents the the joint to go back in cavity? a) Tubercles on articular surface of mandible b)Tempomandibular ligament c)Lateral Ptyregoid muscle d)Medial Ptyregoid muscle . What happent ? a) Proximal Radio ulnar joint dislocation b)Distal Radio ulnar joint dislocation c)Fracture of acromion 43) Regarding typhoid fever a) Asypmtomatic b)Carriers do not excreate causative agent in fees c) Asypmtomtic carriers should be isolated and treated 44) Incision is given on Rt Lumbar area. After how much time .increases frequency and chills and riggors.\ a) Blood culture b)urine culture c) Blood and urine culture 40)Following is not true? a) HCV positive carriers have not been reported 41) Pulse pressure is increased in a)Arteriols b)capalleries C)Aorta 42) A grand father was playing with his grandson with holding his wrist and rotation him in a circule. He is breathing 100% oxygen.36)Where the gangliom joining the superior petrosal nerve and inferior petrosal nerve is located? 37) A patient has hypercarboxyhemoglobin.extaperitoneal fat.peritinium 45) Carpus callosum .superficial fascia.his blood will have 100% oxygen? a) 1 min b)2 min c)5 sec d) 5 min 38) FRC is? a) TV + ERV b)IRV + ERV 39)Aldosterone stimulate a) Melatonin? b)? 40)A patient has fever 40c with flank pain .Suddenly the child starded crying and sit with holding his forarm in porn position. The layers incised will be a) Skin-superficial facia-deep facia.peritoneum b) skin.external oblique-internal oblique-transverse –Transverse fascia-exta peritoneal fat. Flank pain could have .Lactation does not occur because a) Increased Level of progesterone and Estrogen 54) Which of the following have Pulmonary cycle? a) Ascariasis 55)Causative agent of meningitis in 60 years old man is a) St pneumonia 56) Cause of death in pneumonia bue to shock is a)Gram –ve bacteria b)Pseudomonas c)st pneumonia 57) Diphtheria exotoxin hane powerful effect on a) Larynx b)Heart c)Nerve endings d)Kidneys 58) Aspiration of peanut will go in a) Right lower bronchus 59) A female with butterfly ace.joint pain.25 cholecalciferol in a)Hepatic failure b)Renal Failure 51) Which of the following is not present in posterior relations of kidney? a) colic flexure 52) Rt border of heart is formed by a)Rt atrium 53) In pregnancy . What is it? a) Prostate 50)25 cholecalciferol is not converted to 1. and its center contains two calcified bodien of pink colour.a) connects the two hemispheres 46)Most constricted part of male urethra is a) External meatus 47)Superior Thyroid glands a) Are located behind the Thyroid gland outside the facia b)Have blood supply from the superior thyroid arteries 48) Which of the following is true a) Piriformis attach to inferion trochanter b) Schiatic nerve passes through superior sciatic foramen 49) A structure lined by cuboidal epithelium. The condition is called A) Intussusceptions B) Hirschsprung’s disease 66) In what portion of nephrone.a) SLE 60) A young patient has difficulty in swallowing a) Sclerodermia 61) A patient with SLE have a) Possitive ANA b) Possitive anti SS 62) Incision on Supra pubic area caused heavy bleed. Na is absorbed totally neutrally active transport? a) PCT b)DCT c)ATT 76) Cell mediated immunity is mediated by a) T cells b)B cells 77) Virus does not cause a) Lipoma b) Kaposi sarcoma c)cervical cancer 78) Iodine is stored in thyroid follicles as a) Thyroglobulin b)DIT and MIT 79) A patient died due to colonic cancer. The cause could be a)Adenomatous polyps? 80) A patient has double ureters and double pelvis. The rectal examination showed no ganglion cells.On autopsy found thousands of polyps. cell swelling occurs due to a) Entry of water in cell fromj surrounding b) Entry of lipids 64)What is potent Antoxident ( Anti Aging) a)VIT E 65) A boy is excreting meconium from rectum and has severe constipation. It could be a) Damage to inferior epigestric artery? 63) In Hypoxia. It could be a) anomaly is due to premature separation of ureteric bud 81) Chylomicrons contains a) Fatty acids with vitamins 82) Transudate has a) Low Albumin . Patient's father also had colonic cancer. posterior wall is formed by a) Conjoint tendon b)Internal oblique muscles 100) What is part basal ganglia are a) Putamen ( 32 mistakes ) Paper 2 .83) common mediator of Basophils and mast cells have a)Histamine 84)In Skeletal muscle . What could be happen? a) Increased body volume? b) decreased intracellular osmolarity c)Increased extracellular osmolrity 94) Inhibition of heart Rate will be in a) Stimulation of parasympathetic nervous system 95) Aldosterone is stimulated by a) Increase of serum K 96) Follic Acid difficiency causes a)Megaloblastic erythropoises b) Foliate deficiency 97) Difference B/w Plasma and interstitial fluid is a) Protien 98) C02 is mainly carried by a) Hco3 99) In Inguinal hernia. and he took 2 L plan water. the patient a) iron deficiency anemia 93) A patient has sweating 2L . contraction occurs due to a) Sacromere b) Actin c)Myosin 88) In chronic inflammation. there is predominate a) Fibroblasts b)Macrophages c)neutrophils 89)Characterics of malignant cancer is a)Metastasis 90 )Characteristic of malignant cancer is a) pleomorphism 91) Pretraceal fascia a) completely encircle the Thyroid gland b)Completely encircles trachea 92) 6 months after the resection of terminal ilium. lined by stratified squamous epithelium.there might damage a) spinal part of accessory nerve b) CNS part of accessory nerve c) Vagus nerve 117)A patient in RTA came in EAC with hypovomumic shock.101) Blood supply of vertebral column is a)Vertibral artery 102)Suprascapular nerve arises from a) Thyrocerv ical trunk 103) Ist pharyngeal pouch forms a) Auditory tubes 104)Diapharm is supplied by a) c 345 105)Urachus is remnant of a) Allantois 106)A patient had surgery of breast and he is now unable to lift his arm. Gastroduodenal ligament 111) A patient can not abduct and adduct his fingers. a) Deep branch of ulnar nerve 112)Femoral canal is formed by what fascias? a) Fascia Lata + memberenous Layer b) c) 113) Organ . What is lost in feeces? a) HCO3 109)phasic fast adaptive structure is a) Mickle disk b)Carotid baroreceptors c)Rufini's carpuscles 110)The lesser omentum forms which structures? a) Hepatoduodenal ligament. Falciparum Ligament. What is damaged? a) serratus anterior muscle 107)Thurst is stimulated by a) ADH 108) A patient has dirrhoea and developed metabolic acidosis. rounded structure. a patient might develop a) Megaloblastic anaemia 116) A patient got injury in the posterior triangle of neck. First what to treat? a) I/V Fluids . Lost adduction of thumb also. following test is appropriate. a) TIBC b)serum ferritin 115)After gastrectomy.? Palatine tonsils 114)Regarding the Iron requirement of a pregnant woman. cells a) Matured in Thymus after birth b)Both T cells and B cells are derived by same precursors 133)Neurovascular bundle is located at ribs at a)inferior border of ribs 134)Which of the following does not predispose the cancer? a) Radiation b)Bacterial products c)virus d)Chemicals 135)Methicilline resistant staph are a) Usually sensitive to Vancomycin 136) Cardiac muscles can not be tetanized becosed a)They have long refractory period 137) Severe blood transfusion reactoion occurs if transfused all except a) A+ blood to 0+ . The causative agent is a) EBV 129)Regarding adrenal glands a) Right is Larger than left b)Supplied by numerous arteries .118) Clostridia cause a) Gas Gangren 119)Angiotensin 1 is converted to Angiotensin 2 in a) Lung capillary endothelium 120) First week diagnosis of Typhoid is a) Blood culture 121)A soldier posted at high altitude for many years came witg PVC > 40 a) Secondary polycythemis 122) Buerger's disease is strongly associated with a)Smoking 123)Acute appendicitis is associated with a) Neutrophelic leukocytosis 124) Medial arcuate Ligament encloses at its upper end a)Psoas muscle 125) QRS comples is due to a)Ventricular systole 126) The transitional epithelium have a) Couoidial epithelium lined by more larger and rounded cells? 127) A patient have right optic tract damage.and drained by single vein 130) Repture of middle meningeal artery cause hematoma in a)Between the two layers of dura mater 131)Middle esophageal constriction is by a)Arch of Aorta b)Left bronchus 132) T. He might developed a)Left homonymus hemianopsia 128)In Burkit Lymphome. Hemoglobin released will be attached with a) Heptoglobin b) Albumin 151) A patient with itching.b)A + blood to AB+ c) B+ to O+ d) B+ tp AB+ e)O +ve blood to OA+ 138)Xenograft is a) Transplant of tissues of different species 139) Lysosomes contains a) Acid Hydrolase 140) Rt Gastro epiploic artery is the branch of a) splenic artery 141) Right gastroduodenal artery is the branch of a) Hepatic artery 142 )An asthematic patient may have FEV1/FCV <75 143)Ligamentum arteriosum connect with a) Aorta with Left pulmonary artery 144)In atrial fibrillation.RR 20. hot dry skin. What blood test you will advise him after one week? a) Anti streptolysin O titer 150)A patient developed hemolytic anaemia after blood transfusion. pulse 120/min. therer is a) Pulsus deficit 145)A patient lost weight . DB 120/80 a) Hyperthyroidism 146) Alpha receptors stimulation may cause a) Lipolysis b)Gluconeogenesis c)glycogenolysis d) Glycogenesis 147)Trypsinogen in the duodenum will be activated by a) PH < 7 148) Pulmonary embolism most commonly involves a) Left pulmonary artery 149 A patient had streptococcal infection. fever with Antimitochondrial antibody +ve a) Primary billiary cirrhosis 152) PDA occurs in a) Prematurity 153) Fertilization occurs in a)Oviduct 154) Liquificative necrosis occurs in a) Brain 155) Temperature regulatory center is in . B2 c. Granuloma but no necrosis. B6 167)Valve less vessel is A) Aorta B) Pulmonary artery C) Coronary Sinus D) Pulmonary Trunk E) SVC 168) Within 1 hr of the Acute M. which of the following enzyme will be raised? A) CK-MB .a) Hypothalamus 156) Skin cancer occurs a) In Sun exposed area 157) Giant cells have a) Macrophages 158) Inflamatory mediator is a) C5a 159) Optic tract ends in a) Lateral geniculate body 160) Gastric motility is increased by a) Acetylcholine 161)Wound healing is delayed by a) infection 162)Achalasia is due to a)loss of Aurbach plexus 163)Patient has severe chest pain with ST elevation a) Disectening aorta b)Pleuritis c)Pneumothorax d)costochondral e)Pneumonias 164)Patient X-Ray shows hilar lymph nodes.I. B1 b. High output cardiac failure a. What is the diagnosis? A} Sarcoidosis B} Silicosis C} Tuberculosis 165) Strongest layer of small intestine is a)Circular b)longitudinal c)Mucosa d)Submucosa ok? 166). B) LDH C) Alkaline Phosphatase D) AST E) Troponin T 169) Replacement of cells of other normal site of body is called a) Metaplasia 170)Heart sound produced by rapid ventricular filling is A) 1st heart sound B) 2nd heart sound C) 3rd heart sound D) 4th heart sound 171)Corticosteroids decrease a) Neutrophils b)Lymphocytes c)Eiosinophils d)Monocytes e)Basophils 172) DVT occurs in females by use of OCPs 173) Visceral Pericardium is supplied by a) Phrenic Nerve b) Sympthatic Nerves c) Vagus neve d) Cardiac Plexus 174) Basal Cell Carcinoma involves a) Buccal Mucosa b) Hard Palate c) Soft Palate d) Lower Lip e) Oral Cavity 175) Primary spermatocyte divide by a)Mitosis b)Spermiosytosis c)Primary meiotic devision d)Sec Meiotic devision 176) The volume of distribution of drug is not influenced by a)Sex b)Age c)Heart failure d)Renal Failure . apoferritin Posted by Dr Sultan Ahmad 0 comments Reactions: .177)In Hypothyroidism a) Increased cholesterol level 178)craniophrangioma in the center of optic chiasmata may cause a) Bitemporal hemianopsia 179)Which one of the following most likely causes Increase in GFR? a)Constriction of afferent arteriole b)Constriction Of efferent arteriole 180) cardic output measured by thermodilution method is called a)fick law? 181.tachycardia E. precepillary sphincter c.increased urine output D.decrease cardiac output B.ferritin B.restlessness 184) Iron is stored in the form of A. venules 182)Thiamine deficiency causes A} Peripheral Neuropathy B} Pellagra C} Chelosis D} Dermatitis 183) Which of the following is not a feature of shock A. arterioles b. Blood flow regulation is mediated by: a. capillaries d.transferrin C.muslce weakness C. a….30 sec c….9.half life of dopamine is 5 min .T cells d….16 sec b….2008 Gynaecology n obstetrics By SHEHER BANO.60 sec 3...size at birth is equal to adult size 5.Renin 4. Min 9.1 kcal 11.IgG b….energy liberated on fat metabolism is a….8 sec c…. a…....2008 June 25.which of the following are features of type 2 hypersensitivity reaction a….heating at 151 degree C for ….B cells 8.thyroid isthmus is at level of a….3 k cal c….10 sec 2..wt is the most potent stimuli for aldosterone release. a….9.which statement regarding adrenals is wrong.2nd 3rd tracheal ring .4.energy liberated on complete metabolism of protein is a….wt is the circulation time b/w brachial n cerebral circulation. a….. how much time will it take to reach the steady state concentration..1 k cal b…. 1.Gyn Obs June 25..1 k cal 10.Ig M c….actions of glucagons are all except 6..actions of growth hormone r all except 7.ACTH b….wt is the requirement for complete sterilization a….5.15 sec b….. brain c…..patient feels pain on defecation nerve involved is a….almost 10 to 15 questions on NSAIDs and prostaglandins 19..laryngeotracheal bud appears during a…...spinal arteries c….rt atrium + rt ventricle 25.internal juglar arteries 13 end arteries are present in a….rt atrium b….pudendal nerve b….7th week 16..chemoatractant for neutrophil is.muscle cut on episiotomy is a…..muscular layer of ureter in lower third is 23.b….mesentry 15... .derivative of endoderm is a….5th 6th tracheal ring 12..vertebral arteries b…..derivatives of pharyngeal arches (2 to 3 questions) 24.wt makes right border of heart a….rt ventricle c….epthelial lining of ureter is 21.blood supply of spinal cord is a….4th week b…5th week c….inferior rectal nerve 18.3rd 4th tracheal ring c…..one of the following regarding levator ani is true 20.lymph node 14..superficial transverse perineal + bulbospongiosus 17.spleen b…. the site for direct inguinal hernia.capsule Posted by Dr Sultan Ahmad 0 comments Reactions: Medicine JUNE 2008 . are… 36.platelets b….plasma b….factors affected by vitamin K deficiency 27.ext iliac nodes 28.point of meeting of intrinsic n extrinsic pathway 29.RBCs c…..oxyhemoglobin dissociation curve (2 questions) 30. 35.26.structure palpable through lateral vaginal wall is 37.simple sqamous epithelium is present in all except 31..invasion b….neutrophils 38 difference b/w benign n malignant tumors is( 2 almost similar questions) a….metastasis c…....int n ext iliac nodes b….lymph c…..CSF 33.origin of ext oblique muscle is from….....autosomal dominant n recessive disorders (2 questions like name was give n was asked wt is wt) 34..protein content of CSF is 32.which has highest protein content a…..int iliac nodes c….lymphatic drainage of cervix is a….lymphocytes d….boundaries of inguinal triangle.cortisol causes decrease in a…. 1) If a person came to u with infertility and azospermia what would be the best test a) FSH+LH b) prolactin c) buccal smear d) LH e) testosterone 2) whats Hb H disease?? a) omission of 3 alpha chains of Hb b) presence of barts hemoglobin c) requires frequent blood transfusion 3) which one is not the part of brainstem? a) pons b) cerebral peduncles c) cerebellum d) superior colliculus e) mesencephalon 4) Radiation affects/causes injury to a) cell membrane b) nucleus c) cytoplasm d) mitochondria 5) cerebellar lesion causes a) adiadokinesia b) sensory loss c) motor weakness d) hypertonia 6) ovarian ca metastasizes early to a) lung b) peritoneum c) stomach d) opposite ovary 7) Isthmus of thyroid gland lies against the tracheal rings: a) 1st and 2nd b) 2nd and 3rd c) 3rd and 4th d) 4th and 5th e) 5th and 6th 8) Lymphoid nodules are not present in a) spleen b) thymus c) tonsil d) peyers patches 9) Most common Souce of pulmonary emboli a) femoral vein .Contributed By Dr Noman Butt. b) popliteal vein c) periprostatic veins d) periovarian veins e) superficial saphenous vein 10) A lady suffered an injury in a RTA and became unable of flex both her leg and thigh. The muscle most likely affected a) Biceps femoris b) Rectus femoris c) Semimebranosus d) sartorius e) semitendinosus 11) A child sufferd from repeated episodes of mild jaundice. Diagnosis after his labs showed unconjugated hyperbilirubinemia a) Dubin johnson syndrome b) Bile duct obstruction c) Gilbert syndrome d) Rotor syndrome 12) A child aspirated a peanut. In which lobar bronchus it must have lodged a) Left upper b) Right inferior c) rt middle d) rt superior e) left inferior 13) Left 7th bronchopulmonary segment is called a) lateral basal b) anteromedial basal c) lingual lobe d) apicoposterior e) posterior basal 14) The following does not have an associated valve a) superior vena cava b) inferior vena cava c) coronary sinus d) aorta e) pulmonary trunk . 2 weeks back she went to meet her parents in a remote village.fikelendenberg cells with many nuclei.. At autopsy.whats her diagnosis a) hepatitis A b) hep B c) hep c d) hep d e) hep e 21) Genes are a) intron b) exon c) DNA d) RNA e) ribosome . causative organism: a) Ebola virus b) CMV c) measles virus d) rubella virus e) pneumocystis carinii 18) The following disease most severely affects the esophageal phase of swallowing a) myasthenia gravis b) scleroderma c) poliomyelitis d) stroke 19) The normal esophagus is a) 10 inches in length b) lies entirely in the thoracic cavity c) consists entirely of skeletal muscle 20) A 26 yr old girl 28 week of gestation developed vomiting..bilirubin 20mg/dl.15) The following is not present in relation to Posterior relations of left kidney a) quadratus lumborum b) psaos c) colonic flexure d) diaphragm 16) Normal quiet expiration is brought about by contraction/recoil of a) diaphragm b) elastic tissue in thoracic and lung wall c) abdominal muscles d) sternocleidomastoid 17) A child died of severe pneumonia. the lung shoed Warthin.SGOT 272 and SGPT 220 . she has no history of any blood transfusion or past surgery. An injection of insulin given to her will raise her a) blood sugar level b) pH of blood c) urinary excretion of ketones 26) Which one of the Following is terratogenic a) alcohol b) coffee c) heroin d) Phenothiazine e) tobacco 27) Lamia propria of vagina is made up of a) collgen fibres b) reticular c) elastic 28) Which of the following is not forming double peritoneum layer a) broad ligament b) suspensory lig c) mesovarium d) mesosalpinx e 29) Richest in triglycerides a) chylomicrons b) LDL c) VLDL d) HDL .22) Lymphatic drainage of medial quadrant of breast a) axillary b) internal memory lymph node 23) Renal excretion of a drug a) depndnt on GFR b) 24) Menopause is associated with a) low estrogen and high FSH and high LH b) low estrogen and high FSH and low LH c) high estrogen and low FSH and LH d) low estrogen and normal FSH and LH 25) A 20 yr old girl with type 1 diabetes is brought to emergency in semi comatosed condition. a person has a) high insulin and low glucagon b) high glucagon and high insulin c) high glucagon and low insulin d) high cortisol 31) A pts GFR dropped from 100 to 20. If the serum creatinine was 1 with GFR 100ml/min. .5 b) 2.0 c) 5. the new serum creatinine would be a) 0. What will u do? a) volume replacement b) oral airway c) tracheostomy d) splinting of fractures e) 36) Which of the following drugs is proved to be beneficial in the treatment of Subdural haematoma? a) Atenolol b) nifedipine.30) At the end of a marathon race.0 d) 30 e) 20 32) Target tissue of Rickettsia?? a) endothelial cells b) nervous tissue c) macrophages d) muscle cells e) blood cells 33) Most effected organ by dental radiologya) brain b) thyroid gland c) lungs d) heart 34) Which nerve is most likely to damage during 3rd molar surgical removal? a) mandibular nerve b) inferior alveolar nerve c) lingual nerve d) baccal nerve 35) A patient presented to emergency in a semicomatosed condition after an RTA with multiple long bone fractures with BP 80/50 mmHg. resport indiactes hematoma in right parietal region. diagnosis is a) aids b) infectious mononucleosis 40) Changes in V1 & avf indicates a) anterior wall MI b) inferoir wall MI 41) A Patient presented with cysts in liver. examination shows .c) losartan d) nimodipine 37) Which of the following tumors in children is due to gene amplification? a) Neuroblastoma b) retinoblastoma c) Wilms tumor 38) Chronic diarrhea can be due to a) giardiasis b) crohn’s disease 39) A patient presented with sore throat & fever. cause is a) internal carotid artery b) basilar artery c) middle cerebral artery 45) Functional residual capacity a)can b measured by spirometry b) RV+ERV 46) A Diagnosed case of emphysema presents in emergency with dyspnea. blood picture shows atypical lymphocytes..cause can be a)teania solium b)teania echinococccus 42) 3rd heart sound is due to a) rapid ventricular filling b) AV valves closure 43) Following is responsible for CNS myelination a) shwan cell b) oligodendrocyte 44) Pt presented in emergency in unconcious sate.. decreased breath sounds & hyper -resonance.What is the diagnosis? a) Acute MI . spinal cord ends at level of a) L1 b) L3 c) L2 Perhaps it was middle meningeal artery in the above question 48) Upper end of esophagus is at the level of a) C6 b) C7 c) T1 49) Fluid having 0.5gm protein/dl a) CSF b) Lymph c) plasma d) serum 50) Severe blood transfusion reaction will occur a) A+ given to A-b) AB+ given to O+ 51) In achlasia cardia. there is a) Decreased resting tone of LES b) Increased no of ganglia in myenteric plexus c) Decreased no of ganglia in myenteric plexus 52) Renal clearnace a) depends on GFR b) decrease in infants 53) Cerebellum lesion can cause a) adiadokinesia b) static tremors c) hypertonia d) sensory loss in body 54) A patient presented in the ER with an acute onset of severe chest pain radiating towards the left arm and jaw with a BP of 130/80 mm of Hg. cardiac enzymes are normal.investgation to evaluate cause is a) chest x-ray b) ABG'S 47) In adults.ECG shows ST segment elevation. Cause a) ventilation-perfusion mismatch b) blockage of pulm capillary c) Fall in PO2 d) Fall in atm pressure e) ……. 59)A pt.. the treatment of choice will be a) MGSO4 b) methyldopa c) clonidine 61) Which of the following develop from endoderm ? a) b) gonads c) d) palatine tonsil 62) Cannon wave is associated with a) tricuspid regurgitation b) complete heart block 63) Which of the following property prevent heart muscles from tetanization? a) rythymicity b) prolong refractory period . 57) Increase level of erythropoetin is seen in a) lung carcinoma b) renal CA c) cerebellar tumor 58) A young 20yr old guy while climbing stairs had sudden sharp Rt sided chest pain.b) Ruptured Aortic Aneurysm 55) A patient presents with scanning speech & intention tremors.. on MAO inhibitor is given 1st dose of barbiturate will develop a) coma b) insomia c d 60) A lady in 3rd trimester comes with hypertension and edema . lesion is in a) cerebellum b) basal ganglia 56) There were 2 or 3 questions regarding acid-base balance but dont rememer the values. she has history of seizures . The condition is A} Gastroschisis B} omphalocele 2] Which of the following is not related to Meckel’s Diverticulum? A} is remnant of urachas B} lies 60 cm proximal to illeocecal Valve .ecg shows ST elevation .64) A pt. most likely to have (exact scenario not memorized) a) b) 67) Half life of a drug is 2 mins . how long it'll take to reach steady state concentration? a) 4 mins b) 9 mins c) 16 mins 68) The pt. the most likely diagnosis is a) MI b) pulmonary emboli c) retrosternal goiter 65) Jaw angle is innervated by a) b) c) 66) pH 7. comes to emergency with severe SOB for 4 hours.hco3 18 .3 . serum cardiac enzymes are not raised . is suffering from UTI . It is not covered with amnion. pt. the diagnosis is confirmed by a) urine culture b) blood culture c) urine + blood culture 69) The most potent stimulant for erythropoietin? a) b) hypoxia 70) The true statement about growth hormone is a) b) c) d ) its a polypeptide 1] A newborn child has abdominal swelling on the right side of the umbilical cord. pCO2 30 . 3] A boy is excreting meconium from rectum and has severe constipation. The rectal examination showed no ganglion cells. Which one of the following is not the developmental remant A} lateral umbilical ligament B} Medial umbilical ligament C} Median umbilical ligament D} ligamentum teres E} ligamentum venosum 8] The part of mouth derived from ectoderm is A} Epithelium of parotid salivary gland B} Epithelium of Tongue C} Floor f mouth D} E} 9] A male with Gynaecomastia. The karyotype involved is A} 45 XO B} 46 XX C} 47 XXX . diagnosed as Klienfelters syndrome. The condition is called A} Intussusceptions B} Hirschsprung’s disease 4] Regarding somites A} They develop on both sides of notochord. small testes. B} they develop from lateral plate mesoderm 5] Regarding diaphragmatic hernia the most likely is A} Absence of septum transversum B} Failure of pluiroperitoneal membrane 6] Which one of the following is not the derivative of 1st (mandible) pharyngeal arch? A} Incus B} Malleus C} Stapes D} E} 7] Embrologically. D} 47 XXy E} 47 XX 10] Somites are derived from A} Intermediate mesoderm B} Paraxial mesoderm C} Lateral plate mesoderm D} Ectoderm E} Endoderm 11] An I/V Urograph of 8 year old boy shows Excretion of dye normal I kidney but absence of shadow on left side. Which one of the following is most likely? A} Horse shoe kidney B} Constricted ectopic kidney C} Pelvis kidney D} Non rotating kidney E} Unilateral renal agenesis 12] Which one of the following is not formed from urogenital sinus? A} Bartholin’s gland B} Urethral glands C} Para urethral glands D} Seminal vesicle E} 13] Virus damage the cell by A} making the pores in cell membrane B} Forming free radicals C} Altering the formation of proteins D} Nuclear damage E} using energy for their own metabolism 14] The tissue damage by Ionization Radiation is due to A) Damage to Golgi bodies B} Formation of free radicals C} Hydropic degeneration D} Metaphase of cell E} Swelling of cells 15] All are features of IRREVERSIBLE cell injury EXCEPT A} Karyolysis B} Karyorhexis . Instead there is a small shadow just above the bladder on the left side. What is the diagnosis? A} Sarcoidosis B} Silicosis C} Tuberculosis D} E} 18] In women the most common form of necrosis after trauma is due to A} Trauma to fatty tissue B} C} D} E} 19] Dystrophic calcification is present in which of the following A} Acute pancreatitis B} Malaria C} Tuberculosis D} E} 20] Which one of the following is most likely feature of reversible injury? A} Cell swelling B} Karyorhexis C} Decreased glycogen D} Myelin figures E} 21] A person fell on road and got an abrasion on elbow the first event likely occur would be A} Vasoconstriction .C} Autolysis D} Shrinkage of mitochondria E} Appearance of myelin figure 16] Which one of the following is least likely involved in increasing Osteoporosis A} Increased wt: on bone B} Renal cell carcinoma C} Carcinoma prostate D} E} 17] Patient X-Ray shows hilar lymph nodes. Granuloma but no necrosis. A few days after he developed pain and .B} Platelet adhesion C} Platelet aggregation D} E} 22] Which one of the following facilitates phagocytosis in neutrophils? A} C5a B} C3a C} C3b D} C3 .C5 complex E} 23] Which one of the following is potent antioxidant? A} Vitamin A B} Vitamin C C} Vitamin E D} Transferrin E} 24] Which one of the following among Arachidonic acid metabolite is most likely function mediated in phagocytosis? A} Chemo taxis B} Diapedisis C} Transmigration D} E} 25] Which one of the following is potent COX-II INHIBITOR? A} Aspirin B} Celocoxib C} Indomethacin D} Meloxicam E} Piroxicam 26] In case of pulmonary embolism extensive infarction causes sudden death due to A} Air embolism B} Fat embolism C} Thromboembolism D} E} 27] A patient was operated for abdominal surgery. linked disorder D} E} 30] In a renal transplantation. but within 10 mins the organ failed. Which one of the following should be given immediately? Fresh frozen plasma B} 20 days old blood C} 3 days old blood D} E} 29] A boy was born with the condition of polycystic kidney disease. Which one of the following is most likely? A} endothelial injury B} hypercoagubility C} stasis and endothelial injury D} E} 28] A soldier has profuse bleeding. which one of the following is single most significant test to be done? A} ABO blood grouping of donor and recipient B} HLA typing and matching C} Donor and recipient should be close relatives D} E} 31] A organ was transplanted to the patient. Which one of the following is correct? A} Autosomal Dominant disorder B} Autosomal recessive disorder C} X.swelling in his right leg. Which one of the following is likely etiology? A} Antibody mediated B} T-Cell mediated C} Preformed antibodies D} E} 32] AIDS occur in the presence of . He was admitted in the hospital. A} Kaposi’s sarcoma B} C} D} E} 33] A 2 year old boy is suspected as HIV positive. which one of the following is the confirmatory test in the boy A} ELISA test B} Western Blot test C} PCR D} Increased levels of HIV in T lymphocytes E} 34] A 10 year old boy was operated for gastrectomy. which one of the following is seen? A} Anti thyroglobulin B} C} D} E} 37] To differentiate Chron’s disease from Ulcerative colitis. which one of the following is likely significant regarding Chron’s disease A} has increased frequency to cancer B} Involves both small and large intestine C} . which one of the following type of anemia is developed postoperatively A} Megaloblastic anemia B} Iron deficiency anemia C} Normocytic norm chromic anemia D} E} 35] Autoimmunity is not involved in which one of the following A} Pernicious anemia B} Grave’s disease C} Diabetes mellitus D} Hashimoto’s thyroiditis E} Thymoma 36] In case of hashimoto’s thyroiditis. Bromocriptine is given to treat it the drug causes A} Inhibition of prolaction from pituitary gland B} Decreased prolaction from hypothalamus C} Inhibition of prolaction from breast D} Stimulation of prolaction of anterior pituitary E} 40] An unconscious patient presented in ED does not respond to Naloxone. following is finding A} Mallory bodies B} C} D} E} 39] A 20 year old girl has developed Galactorrhea due to prolactinoma.D} E} 38] Regarding alcoholic hepatitis. Which of the following drug has been taken by this patient? A} Morphine B} Heroine C} Pethidine D} Phenobarbital E} 41] Strongest layer of small intestine is A} Mucosa B} Sub mucosa C} Circular layer D} Serosa E} Longitudinal layer 42] Regarding adrenal medulla which one of the following is true? A} Secretes epinephrine and Norepinepinephrine in 80/20 ratio B} C} D} E} 43] R.R most likely .E. A} Synthesizes protein B} Synthesizes lipids C} D} E} 44] Half life of insulin is A} 5 mins B} 90 mins C} 120 mins D} E} 45] Insulin A} Converts glycogen to glucose B} Stimulates protein synthesis C} Stimulates lipolysis D} E} 46] Which one of the following most likely contains abundant cholesterol? A} VLDL B} LDL C} Chylmicrons D} HDL E} 47] Which one of the following most likely causes Increase in GFR? A} Constriction of afferent arteriole B} Constriction Of efferent arteriole C} D} E} 48] Gastric emptying is delayed by A} Gastrin B} Secretin C} CCK D} . E} 49] Which one of the following pancreatic enzyme is most likely involved in fat absorption A} Amylase B} Lipase C} D} E} 50] Smooth muscle A} Contain sarcomere B} Ttubules are present C} Gap junctions are present D} Forms motor end plate E} 51] Steady pressure on skin is perceived by A} Pacinian’s corpuscle B} Ruffini’s corpuscle C} Meissener’s Corpuscle D} Golgi tendon organs E} 52] Which one of the following is related to pain? A} Serotonin B} Substance P C} D} E} 53] Free nerve endings have receptor for A} Light touch B} Pressure C} Pain D} Vibration E} 54] Which one of the following has highest pressure? A} Pulmonary artery B} Left atrium . which one of the following would be immediate mechanism to maintain B.P A} Baroreceptor reflex B} Chemoreceptor reflex C} Secretion of ADH D} Aldosterone E} Renin angiotensin mechanism 57] Water enters into the cell by A} Pinocytosis B} through pores C} Simple diffusion D} Facilitated diffusion E} Active transport 58] Insulin secretion is controlled by A} Glucagon B} increased glucose C} Decreased glucose D} E} 59] Systolic B.C} Right atrium D} SVC E} 55] Flow of current through junctions in the muscle A} Smooth muscle B} Skeletal muscle C} Cardiac muscle D} E} 56] A profusely bleeding patient develops hypotension.P is highest in A} Pulmonary arteries B} Cerebral arteries C} Renal arteries D} Brachial arteries E} 60] End diastolic volume depends on . A} Atrial contraction B} Distensibility of ventricles C} Duration of diastole D} Venous return E} 61] Mean electrical axis of heart is deviated to left if A} Heart is angulated on right side B} Pulmonary stenosis C} Right bundle branch block D} COPD E} 62] Pulse Oxymeter measurement gives error in the reading if A} Pt. is dark skin B} Increased conc: of Hb A C} Increased Conc: of Hb F D} Aortic stenosis E} 63] Secretion of potassium by distal collecting tubule decrease by A} Increase in potassium in diet B} Hyperaldosteronism C} Metabolic alkalosis D} Spironalactone intake E} Thiazide diuretic 64] Absorption of iron is markedly affected by A} Ingestion with meal B} Partial gastrectomy C} Trivalent form D} E} 65] Patient with pancreatitis has developed steatorrhea due to deficiency of A} Lipase B} Pepsin C} Amylase . extensive 1st pass effect b.26th MARCH' 08 ANESTHESIA 1. venodilator c. How will u assess the drug? a. Regarding Nitroglycerine.D} Renin E} 66] Absorptive surface of small intestine is A} 50 B} 100 C} 200 D} 350 E} 500 67] Daily dietary protein for 70 kg. urine drug conc 2. 2008 ANESTHESIA MARCH' 08 FCPS I . If a drug is taken orally. May 10. dec HR . plasma drug conc b. dec venous return d. blood drug conc c. all are true except a. Man is A} 12 gm B} 120gm C} 200 gm D} 150 gm E} 68] Source for metabolic fuel for liver in post absorptive state is A} Glucose B} Ketones C} Glycerol D} Fatty acids E} Posted by Dr Sultan Ahmad 0 comments Reactions: Saturday. altered permeability 10. equal no. alkalosis 7. dec Na* e. phosphodiesterase inhibitors 11. Inc K* b.e. Capillary permeability is inc by a. Enzyme induction occurs in: a. equal no. liver b. kidney c. K* d. aspirin b. plasma colloid osmotic pressure c. all parasymp are cholinergic b. Digoxin toxicity a. glucose b. Why the dose of theophylline in infant is smaller than in adults a. corticosteroids c. intersitial hydrostatic press d. effect inc by a. Which is true. Na* c. Cimetidine 4. of atoms 9. of molecules b. adrenals 6. 2 liters of 2 different gases of diff mol wt contain a. dec Ca** d. a. Urea 8. Secondary active transport occurs thru a. Warfarin. dec metabolism b. Inc Mg* c. hypertension b. all sympathetic are adrenergic . PG in inflammation dec by: a. heart d. dec plasma protein binding 5. t1/2 is 3-5 min 3. promotes each others actions a. atropine and amitryptyline 20. Thirst is dec by: a. inc HR b. ACh c. pt is hypotonic. baroreceptor efferent e. Somatostatin 14.c. Gastrin b. Secterin d. constipation 18. aldosterone c. Cause ? a. bronchodilator 16. dec HR . midriasis d. ACh c. inc sweating c. hypersensitivity b. Basal level of bile 15. alpha recptors effects ? a. atropine and reserpine b. Gastric acid is inc by a. amytriptyline and reserpine c. inc gastrin b. Ag II d. piloerector contraction e. ADH b. inc Hematocrit 19. After induction of anesthesia by 10 mg/kg thiopentone. Inc HR and force of contraction 17. all post-gang parasymp are cholinergic 12. Secterin d. ACh effect? a. Inc salivation b. Beta effect ? a. Gastric motility is inc by: a. lipogenesis c. Somatostatin 13. bleomycin 25. Pt with mild CCF. ACEi d. amiodarone 28. Nifedipine b. purkinje e. Drug responsible: a. descending loop of Henle b. furosemide c. Rt ventricle (correct) 27. streptomycin b. collecting duct 23. ascending loop of Henle c. digoxin b. 1 week later pt complains of severe pain at Rt big toe. nifedipine d. Nicardipine 22. corticosteroids d. SA node b. AV node c. amiodarone 24. which Ca** channel blocker is used: a.c. Rt atrium d. Cause: a. Terminals shd be in: a.Lungs inactivate Bradykinin by: a. propranolol 26.In hypertonic obs cardiomyopathy. distal convoluted tubule e. Drug to be given: a. Metastatic pleural effusion. enz involved in converting kallikrein to plasma protein . clindamycin c. captopril c. Treatment started. Ca** channel blocker c. proximal convoluted tubule d. Monotherapy for mild CCF: a. dec contractility 21.Pacemaker for complete heart block. thiazide b. ACEi b. Loop diuretics + thiazides enhance each others effect ( Dec NaCl abs) at: a. Infant with renal abnormality (renal artery stenosis). Lidocaine. MOA? a. inc HR c. no change 30. Class IA antiarrythmic drugs a. K* channel blocker 35. amiodarone 34. carbonic anhydrase . Amiodarone > MOA 32. inc contractility b.After loss of 1 litre of blood in 5 min: a. CO = VR. exercise 37.Drug that inc extracellular K* (moves K* out of the cell): a. H2CO3 c. dec atrial contraction d. inc end-diastolic vol 31. Lidocaine as antiarrrythmic: a. dec abnormal tissue conduction and no effect on normal tissue b. Carbonic anhydrase d. inc QRS f. aldosterone inc d. Sec K* in renal cells a.Verapamil as antiarrythmic ? 36.29. inc PR interval c. blocks Na* channels b. dec AV conduction e. Beta blocker c. diastolic press dec b. systolic press dec c. inc QT 33. quinidine c. Angiotensin b. pH e. Large hump on left of cardiac pacing mentions: a. metabolic alkalosis b. Lidocaine b. procainide d. Prothrombin c. Cause a. Hb 8. Ca++ dec. ARF b. MCV 58. Hb 8. only alter A delta and C fibers b. Consistent finding in hypoKalemia a. ureters attached to colon 39. PCT b. High output cardiac failure a.38. discoloration of skin. affects only pain and temp and no other sensory modalities 43. a. Capillary abn b. pH Inc. enlarged spleen. Thiamine b. 10 yrs old girl from Baluchistan c/o fatiguw. tumor cells . B6 d. ARF. No itching or pain. Folic acid 44.0. No comorbids. Vit D def c. S. iron transfusion 46. 90 yrs old lady with purplish large patch on Rt hand and arm. Metabolic alkalosis a. Ferritin 1000.inc (35). HCO3. deferroxamine b. DCT c. PTH 42. Otherwise Normal. Vit K dependent clotting factors 45. Reason? a. blood transfusion c.most effect. B2 c. Anesthetics effect at cerebral level and analgesics at spinal cord level c.0. LOH d. lethargy. Anesthetics differ from anelgesics in: a. Appropriate treatment? a. CD 41. renal failure 40. mild jaundice. Fat b. Death d/t embolism after accident a. folic acid deficiency d. prolong vomiting b. diarrhea c. an inactive metabolite of diazepam is: a. Inc ICP. tachycardia 54. air d. DOC a. HTN. Factor VIII b.c. tachycardia d. dec ACh release in bronchi b. 50-100 hrs c. Bradycardia b. 20-50 hrs b. inc secretion 49. TSH e. Mast cell stabilizer. vasodil c. Na chromoglycate 50. produced by ependymal cells of choroid plexus d. CSF a. hypoTN. bradycardia c. total 150 ml b. inc ICP. 36-200 hrs (correct) 51. Tamoxifen 52. inc ICP. CSF: . Post menopausal women with brease ca. a. production rate 20 ml/hr c. Cushing's triad: a. Prothrombin 48. factor III c. TRH 53. T1/2 of desmethyl diazepam. Sec released from thyroid gland entering circulation is: a. 100-200 hrs d. Thyroxine b. HTN. Inc ICP. effects on: a. free T4 d. dec ICP. hypoTN. tachycardia e. Free T3 c. Ibpratopium bromide given by nebulization: a. drained by lymphatics 55. thromboembolism 47. Warfarin. HTN. Pseudomembrane colitis linked to: a. dec specific gravity 56. erythromycin 64.a. PZA 62. except: a. Rifampin c. Dark urine after starting ATT. d. a. CRF d. inc Ca resorption from bones c. ant spinal lig b. Imp buffer in interstitial fluid a. post spinal lig c. Anemia 57. inc PO4 abs from renal tubules b. tetrahydrofolate in bacteria 60. H2CO3 b. PTH a. 1 week after starting of ATT. PO4 58. Needle inserted at paramedian vertebral canal pierces: a. Penicillin 59. Rifampin 61. cephalosporins d. INH b. Sulphonamides inhibit a. dec glucose c. Inc K* than plasma b. supraspinal lig d. ligamentum flavum 63. Regarding chemotherapy all are true. clindamycin b. often multi therapy is not curative . Ethambutol d. plasma pr. Cell wall synthesis inhib a. interspinal log e. pt c/o pain in big toe. Hb c. Reason a. penicillin c. Blood flow regulation a.5 3 . a. req daily dosing of multiple drugs for long time 65. not concerned to dose / conc adm c. Isoflurane b. enflurane c. cord > cerebellum 73. desfurane < isoflurane < enflurane < sevoflurane < halothane < methoxyflurane 0. Ether causes a. nonevident facial spasm 69.5 20 72. sp.9 b. onset of action 30-60 min b.66-0.b.68 67. Metabolism of halides to release fluroide levels in descending order: a. all unconscious pts must be screened by this drug c. Enflurane a. dec HR 66. cerebrum > sp. cord > cerebellum > cerebrum c. max safe dose 1 mg/day 66. inc ICP d. paradoxical tachycardia d. autonomic stimulation 68. MAC of Sevoflurane with 60% NO a. req special vaporizer c. Sevoflurane a. Flumazenil a. cerebrum > cerebellum > sp.2 2. Isoflurane a. Halothane 71. More pungent. cord b. N2O shd not be used in elderly with intestinal obs d/t: . EEG typical of seizures b. dec HR b. 0. MAC 1. desflurane at MAC <1 70.02 0. req use of normal vaporizer 67. long duration of action a. corticosteroids c. MAC related to: a. Bupivacaine 81. Pressure in the cylinder till the last drop evaporates? 76. Pressre of N2O cylinder is 750 psi. Prilocaine 80. Bupivacaine 78. does not undergo metab and is exc unchanged 77. MetHbemia a. anemia 83. longest acting L/A to be given a. retrolental fibroplasia b. tubocurare c. Cylinder half filled with liquid N2O. alveolar air conc. tubocurare . atelectasis c. risk of inc distension 74. serotonin 84. Pressure in the cylinder? 75. dec surfactant prod d. atracurium b. oil / gas solubility c. Hyperoxia. Fracture 5. Muscle relaxant that can be given to an asthmatic pt a. Myocardial toxicity to a L/A? a. renally exc. suxa 85. T1/2 N2O ? a. aspirin b. a. 6th rib. 82. cardiovascular collapse a. All true except 1. Muscle relaxant. CV depression e. Bl / gas solubility b.a. bupivacaine 79. PG released in inflammation are inhibited by: a. cisatracurium d. 1% phosphoenol c. radiation (UV light) d. CaO formation c. Flow meter a. formaldehyde 87. For disinfection hands shd be washed with a. pancuroniun d. following surgery of HIV pt: a. Laminar flow (MURAD QS) a. not affected by humidity d. 91.b. autoclaving is heating objects at 121 degC at 15 psi for 3 min c. When Rynolds no. Reason for heat generation when CO2 is passed thru soda lime a. Flowmeter stops working when a. dry heat e. The operating room shd b cleaned with the following drug. flow of O2 > N2O c. friction b/w soda and CO2 90. used towards expiration b. vancuronium c. resistance independent 92. breakage / disruption of control valve. velocity dec 93. formation of CaCO3 b. flow of N2O > O2 b. alcohol with chlorhexidine c. exceeds 3000 a. gellamine 86. repeated heating denatures polyvenyl tubings b. flow directly prop pressure b. phenol b. 10% gluteraldehyde b. Following is true regarding sterilization: a. density dec c. cannot be attached to the ventilator . H2O2 88. formaldehyd 89. kept vertical to keep the dial needle at zero c. 2% gluteraldehyde d. viscosity dec b. diffusion b. doppler effect 96. radiation d. the temp at which gas must be cooled to become liquid 97. radiation d. Relation of vol of a subs to its weight is: a. Conduction b.32 b. becomes zero with gain of energy 98. temp at which liquid converts to vapour / gas without the utilization of external heat b. surface vapour with highest energy is evaporated . capacitanc transducer can be used on body 95. gas is heated b. 5/9 *F x 32 c. convection c. Latent heat of vaporization: a. Subract 32 and multiply by 5/9 e. Regarding transducer all are true except a. conc 99. 9/5 *F. Max heat is lost by: a. osmosis c. evaporation 101. viscosity c. outside of the container is cooled c. Pt lying uncovered in operating room.94. Pt with ETT tube. density b. add 32 and multiply by 9/5 100. changes signal from 1 form of energy to another b. convection c. Conduction b. evaporation 102. Heat is lost by: a. Critical temp is: a. 5/9 *F + 32 d. To convert Celcius scale to Farenheit scale: a. As the gas flows thru the vaporizer a. Movement of molecules from high conc they layer/ surface: a. capnograph can also measure a. followed by slow frequency. CT scan b. 107. inc as the wire is stretched b. During diathermy when neutral plate is disturbed: ( Burn inc as current density inc. pt may get a burn even at ECG electrodes ( Burn can occur at all frequencies ) b. thermometer 104.. Strong magnetic instruments may not be used in a remote village d/t side effects. Angioplasty 110. proportional to current flow 111. depends it is operating at cutting or coagulation mode c. high altitude wave 108. hygrometer b. rise of baseline indicate failure of valve b. convert to beta when eyes open b. Humidity is measured by: a. rise of height indicate valve failure 106. O2 is measured by all techniques except mass spectrometry (MURAD QS) a. Resistance in series: a. Wheatstone bridge a. known as strain gauge used as pressure transducer c. alpha waves with eyes closed a. signals occur at 0. Pulse oxymetery a. Regarding Capnograph a. oxyHb 105. depends on current frequency 109. Instrument that can be used is: a.5 sec b. measures changes of resistsnce b. MRI c. but independent of freq ) a. no current flows when bridge is balanced 112. Apart frm measuring end tidal CO2 conc. PO2 c. reduced Hb d. convert to delta in REM c. CO2 saturation b. light emitted by filtered light source .103. covered by squamous ep b. has str. Umblical artery b. Umblical vein c. Which measure of central tendency is used most frequently in clinical practise a. Aorta 119. Length of larynx. Surgeon treats 200 pts of heamorrhoids. cohort study c. has 3 layers of muscles e. Treatment of strep pneumonia by sulphonamide and penicilline by random selection is called: a. shows relation of 2 paired means 116. Variance is related to: a. variability b. Wilcoxon rank sum test 115.control study b. range 114. 50th percentile e. double blind c. Superior vena cava d.113. median d. sq in upper 1/3 c. a. tripple blind 118. O2 carried most in fetal circ. has str. After 1 yr he reasses the pts and finds that surgically treated pts have better outcome. This is called: a. t-test a. case . in lower 1/3 d. Esophagus histology a. cricoid to T2 cartilage . He randomly divides the pts into surgically treated and pharmacologically treated groups. prospective cohort d. 15 cm average b. esophageal opening acting as anatomical sphincter 120. sq. a. randomized control trials 117. mean b. mode c. single blind b. square root of std deviation c. xiphisternum d. esophageal plexus d. internal membrane c. superior vena cava d. Inferior wall MI d. L1 vertebral body c. acts as anatomic sphincter to the stomach 122. left ventricle e. ECG shows changes in V 1 . azygous vein e. Structures entering thorasic inlet a. anterior wall MI b. It denotes: a.V4. Needle inserted at 5th intercostal space on the left of the sternum pierces: a.121. Inferior wall MI 123. 1st rib c. clavicle b. Pt complains of chest pain for more than 30 min . internal intercostal muscle b. anterolateral MI c. intercostal nerves 127. cardiac plexus c. 10th rib 126. phrenic nerve b. Lateral wall MI e. Fibrous pericardium and parietal layer of serous pericardium is supplied by ä. Groove for subclavian artery at a. Diaphragm is directly not attached to: a. Left phrenic nerve also passes thru this opening c. accessory nerve b. T10 vertebral body b. at left crus of diaphragm b. at level of T12 vert d. aorta c. Esophageal opening a. 6th rib 124. left atrium d. 2nd rib d. Left recurrent laryngeal nerve 125. right ventricle . Damage to pneumotaxic centre leads to: a. left subclavian 129. apnea 136. isovolumetric contraction 134. 1st branch of ascending aorta is: a. Which is a branch of posterior division of sacral plexus a. LCA from arch of aorta c. L4 vertebra d. S2 splitting occurs d/t a. delayed closure of pulmonic valve 131. L5 vertebra e. L3 vertebra c. diaphragm 135. tropomyocin . rapid ejection phase c. Aortic pressure tracing is highest at which phase of cardiac cycle: a.128. Muscle of quiet inspiration a. closure of aortic and pulmonary valves 130. S1 vertebra 132. irregular breathing e. isovolumetric relaxation b. apneusis b. RCA from anterior aortic sinus b. Which is untrue. common peroneal nerve d. 2nd heart sound is produced d/t : a. short expiratory phase d. nerve to quadratus femoris e. pudendal nerve b. posterior cutaneous nerve of thigh c. slow ejection phase d. perforating cutaneous nerve 133. myocin c. L2 vertebra b. actin b. Spinal cord ends at the level of lower border of: a. deep inspiration c. Thich filaments: a. medial collateral lig and anterior cruciate lig. actin and myocin e. 143.Breur reflex b. Na. 0.d.receptors of pulmonary capillaries c. At the end of expiration: a. H2CO3 .03 ms 142.3 ms e. intra-alveolar pressure is subatmospheric b. Ventilation perfusion mismatch in which Ventilation > Perfusion is called as: a. Bain Bridge reflex 145. Example of carrier mediated counter transport: a. Hering. Chloride shift 146.glucose transport 138. 30 ms c. intra-pleural pressure is subatmospheric 141. Haldane effect c. Depolarization of cell is maintained by: a. Knee joint is supplied by: a. Vagus forms the efferent pathway of: a. Hb b. Rt apical bronchus 140. Inhaled foreign body lodges in which part of lung a. Blow to the lateral surface of knee joint damages: a. Leftwatd shift of oxy-Hb curve is called: a. 0. J. 300 ms b. Most important buffer is interstitial fluid a. 3 ms d. L3-L4 144. troponin 137. shunt b. medial meniscus. Action potential thru large nerve fiber: a. Bohr effect b. Na influx 139. dead space 147. has more anions 153. Uncontrolled DM 152. has more mucous and pigments c. H* sec from nephron is inc by a. H2PO4 d. pressure d. acidosis b. Increased filtration thru capillary is fascilitated by: a. alkalosis 155. Ag II d. Medullary respiratory group of neurons part of ascending reticular system a.c. inc hydrostatic pressure of cap c. thrist b. inc oncotic pressure of ISF 157. slowly adapting pain 150. Coronary blood flow is maintained by: a. autoregulation 156. Pacinian corpuscles related to a. Proteins 148. Anion gap increases in: a. medulla 149. rapidly adapting pain e. ADH c. Inc plasma colloid osmotic pressure b. Bile from liver is different from that from GB in: a. True b. Plsma osmolarity e. vibration c. Damage to what part of brain will result in central cease of respiration a. General anesthesia influences . has more HCO3-b. touch b. Plasma vol 154. inc hydrostatic pressure of ISF d. false 151. Dry mouth increases all except a. leg foot area of motor and sensory cortex true / false 159. IL1 d. FRC = ? a. Hypoglycemia increases a. C5a c. C3a b.0. capillaries d. SSK b. precepillary sphincter c. Secretin 167. duration of PR interval wld be b/w 0. sensory neurons blocked 1st d. arterioles b. Middle cerebral artery supplies a. arterioles c.12 .a. Mediators of inflammation a. TNF alpha 161. If heart rate is 80 beats/min a. autonomic neurons blocked 1st 158.16 sec 164. VIP e. Gastrin d. veins 162. ERV + RV 166. Gastric acid is stimulated by: . pain neurons blocked 1st c. capillaries d. Lung function that cannot be measured by spirometry: a. Largest total cross-sectional and surface area is of: a. Somatostatin c. Residual vol 165. venules e. venules 163. Blood flow regulation is mediated by: a. Stretch reflex afferent 1a fibers true/false 160. motor is affected 1st b. artery b. Cortison c. Calcitonin 170. Gastrin b. cystic artery. A postmenopausal women suffered fracture d/t osteoporosis. Her Ca** is Inc. Physics. Oxytocin d.. portal vein B} Bile duct.. VIP 168. ADH b. Capacitance Posted by Dr Sultan Ahmad 1 comments Reactions: Medicine March 2008 PREPARED BY: DR:MUHAMMAD SHEERAZ AKHTAR SOOMRO 1] Which one of the following is COX-II INHIBITOR? A} Aspirin B} Ibuprofen C} Indomethacin D} Meloxicam E} Piroxicam 2] Which one of the following vitamin deficiency causes HOMOCYSTENURIA? A} Vitamin B1 B} Vitamin B2 C} Vitamin B6 D} Vitamin B12 E} Biotin 3] Which one of the following structures is present at free edge of LESSER OMENTUM? A} Bile duct. Prolactin e. PO4 is dec. Insulin c. Secterin d. hepatic artery. portal vein . Thyroid hormone 169. CRF e. Estrogen b. CCK c.a. PTH d. Reason? a. GIP e. Somatostatin dec sec of which hormone a. after few hours he developed red color urine. portal vein 4] A 16 year old boy was given CO-TRIMAXAZOLE for fever. likely abnormality is in A} Glucose-6-Phosphate deficiency B} Immune hemolysis C} Paroxysmal Nocturnal Haemoglobinuria D} Malaria E} Sickle cell disease 5] Thiamine deficiency causes A} Peripheral Neuropathy B} Pellagra C} Chelosis D} Dermatitis 6] Milk is notoriously deficient in A} Vitamin C B} Iron C} Riboflavin D} Pantothenic acid E} Vitamin A 7] Dry heat kills the bacteria by A} Coagulation of proteins B} Cell lysis C} Free radical formation D} Direct killing E} Oxidation 8] Virus damage the cell by A} Making the pores in cell membrane B} Forming free radicals C} Altering the formation of proteins D} Nuclear damage E} using energy for their own metabolism 9] Which one of the following lipoprotein has highest protein content? A} HDL B} LDL C} VLDL D} IDL E} CHYLMICRONS 10] All are related to PLATELET FUNCTION EXCEPT A} Capillary resilience test .C} Hepatic vein. hepatic artery. I.B} BT C} Clot retraction 11] ALLELLE is: A} Fragmented genes B} Non-identical genes C} Structural genes D} Regulatory gene E} Autosomal Dominant gene 12] D. Dd). DD). Which of the following blood group cannot be inherited by their children? A} A+ B} B+ C} AB+ D} ABE} O+ 15] The person has suffered the injury to spinal cord leading to HEMISECTION of the cord. the IPSILATERAL sign will be A} Loss of pain B} Loss of temperature C} Loss of sensation of touch and vibration D} Lower motor neuron paralysis E} Upper motor neuron paralysis 16] Valve less vessel is A} Aorta B } Pulmonary artery C} Coronary Sinus D} Pulmonary Trunk E} SVC . father is having blood group B+(Bo.C is stimulated by: A} Factor X B } Factor XII C} Thromboplastin D} Factor IX E} Fibrinogen 13] In parenchymal cells Iron is deposited in the form of A} Tranferrin B} Ferritin C} Heamatin D} Heamoglobin E} Lipifuscin 14] a mother is having blood group A+(Ao. She is prone to develop: A} Hyperglycemia B} Hypocalcaemia C} Hypophosphatemia D} Hyperviscosity E} Hypoglycemia 18] Turner syndrome A} XXY B} 45XO C} 46XY D} XXX E} 69XX 19] Regarding Von Willbrand disease A} Autosomal Dominant B} Autosomal Recessive C} Some family members have low level of Factor IX D} Some family members have abnormal level of Factor IX E} has positive family history 20] Which of the following condition is related to lymphatics? A} Angiosarcoma B} Rhabdomyosarcoma C} Cystic hygroma 21] The condition in which pulmonary trunk fails to divide leading to A} PDA B} Truncus arteriosus C} TOF 22] Basal Cell Carcinoma is located at A} Lower lip B} Upper lip C} Tongue D} Hard palate E} Soft palate 23] All are branches of Vagus Nerve except A} Auricular Nerve B} Lacrimal Nerve C} Recurrent Larengeal Nerve D} Internal Larengeal Nerve .17] A lady has undergone colostomy is now on the TPN. The likely cause is A Decreased GIT motility B} Deficiency of intrinsic factor C} Malabsorption 30] Stimulation to increase Serum Calcium is A} Hypo-phosphatemia B} Paratharmone C} Decreased ECF 31] Malnutrition causes all of the following EXCEPT A} Pitting Edema. B} Loss of subcutaneous fat. .24] Permanent Atrophy is caused by A} Loss of Innervation B} Hormonal Deficiency C} Immobilization D} Nutritional deficiency E} blockage of arterial supply 25] Which of the following is not Carcinogen? A} Alcohol B} Benzathracine C} Cyclophosphamide D} Benzidine E} Dimethylsulfate 26] The Initial step in APOPTOSIS is A} Inhibition of P53 genes B} Activation of BCL-2 C} Activation of Caspases D} pores formed by lymphocytes 27] Which of the following is content of Cavernous Sinus? A} Oculomotor B} Trochelear C} Abducent D} Trigeminal (V3) 28] Which part of kidney produces ERYTROPOITEN A} Macula Densa B} Peritubular Capillary C} PCT D} DCT E} LOH 29] A patient with signs and symptoms of megaloblasstic anemia. D} Skin pigmentation E} Thrombocytopenia 32] Liver biopsy from a patient shows a lesion which comprises of central collection of structure less material surrounded by epitheliod cells and lymphocytes.5%. He is expected to have all EXCEPT A} Hypocalcaemia B} Hypokalemia C} Hypothermia D} Left shift of Oxyheamoglobin dissociation curve E} Metabolic acidosis 35] The endothelial molecules ICAM-1 & VCAM-1 are responsible for A} Direct endothelial injury B} Endothelial cell contraction C} Junctional retraction D} Leukocyte adhesion E} Leukocyte margination 36] Eating raw or under cooked meat can cause infestation by A} Dracunculous Medinensis B} Echinecoccus Granulosis C} Liver Fluke D} Schistosomiasis E} Tape Worm 37] A pt: has sustained burn that is very painful with blisters. The most likely causative parasite is A} Ankylostomata Duodenale B} Ascris Lumbricoidis C} Diphylobothrium Latum D} Strangyloids Stercoralis E} Taenia Saginata 34] A 40 year old male after road traffic accident had received massive blood transfusion. Physical examination: Pallor.5 gm/dl. easy fatigue and palpitations. The most likely change in central area would be: A} Caseous necrosis B} Coagulation necrosis C} Fatty changes D} Hydropic changes E} Liquefective necrosis 33] A 28 year fisherman presented with slowly developing lethargy.C} Failure to thrive. Reticulocyte count: 0. Loss of touch sense in both feet and legs Hb: 7. Stool DR: Ova of Intestinal parasite. Platelet count: 110 x 10 9/L. One large cell with multiple nuclei arranged close to membrane is also seen. His diet comprises of fish and rice only. Healing of this burn will take place . Alkaline Phosphate 2300 IU/L 500IU/L. His LFT shows Direct Billirubin 11 mg/100 ml. The most likely cause is A} Drug induced jaundice B} Extra hepatic Cholestasis C} Autoimmune Hepatitis D} Alcoholic Hepatitis E} Intrahepatic Cholestasis 39] The tissue damage by Ionization Radiation is due to A} Damage to Golgi bodies B} Formation of free radicals C} Hydropic degeneration D} Metaplasia of cell E} Swelling of cells 40] Formation of free radicals cause by A} Ionization Radiation 41] Edema of renal origin is most likely due to Albuminuria & sodium retention B} Decreased water excretion C} Hypertension D} Renal stone E} Sodium retention by the kidney 42] Increase in Light Chain Immunoglobulins Is the characteristic feature of A} Endocrine type Amyloidosis B} Hereditary Amyloidosis C} Localized type Amyloidosis D} Reactive Amyloidosis E} Systemic type Amyloidosis 43] Chemotaxis is caused by A} C5a B} C5b C} Lymphokines D} Leucotriene B3 E} Opsonins .from which of the following layers of the skin A} Basal layer of epidermis B} Keratinized layer C} Layer of upper dermis D} Lower dermis E} Subcutaneous layer 38] A 40 year old male presented with jaundice and generalized itching. SGPT 75U/L. The blood condition is most likely due to A} Iron deficiency B} Decreased bone marrow activity C} Folic acid deficiency D} Increased placenta size E} Plasma Volume Expansion 47] The Sensations of Anterior Triangle of the neck is mediated through A} C1. Which of the following muscle is likely paralyzed A} Rectus femoris B} Sartorius C} Semitendinosus D} Vastas lateralis E} Popliteus . C3. C2. C3. He is most likely to have A} Anterior infarction B} Anterolateral infarction C} Anteroseptal infarction D} Inferior Infarction E} True Posterior infarction 46] A female at 30 wks of gestation has Hb: 9. C2 48] All are features of IRREVERSIBLE cell injury EXCEPT A} Karyolysis B} Karyorhexis C} Autolysis D} Shrinkage of mitochondria E} Appearance of myelin figure 49] A young athlete has difficulty in flexion of knee and extension of hip. C3s E} C1. C3 B} C1. C2. And has ST segment elevation in LEADS V1 to V4. C4 D} C2.0 g/dl with normocytic normochromic picture. C4 C} C2.44] Following are the actions of Archidonic acid metabolites in inflammation EXCEPT A} Chemotaxis B} Increased vascular permeability C} Thrombosis D} Vasoconstriction E} Vasodilatation 45] A 65 year old male pt: presented with chest pain for last 30 mins. 50] In the Aortic opening I the Diaphragm is constricted. He has history of taking some unknown drug. has likely taken: A} Phenobarbital B} Phenothiazine C} Morphine D} Diazepam E} Alcohol 55] Within 1 hr of the Acute M.I. NaHCO3 reverses the action of the drug. which of the following enzyme will be raised? A} CK-MB B} LDH C} Alkaline Phosphatase D} AST E} Troponin T 56] Which organelle is not present at axon hillock A} Microtubules . urine will leak into A} Deep Perineal Pouch B} Superficial Perineal Pouch C} Fascia lata D} Scrotum E} Ischiorectal fossa 52] The most common site of fertilization in humans is A} Ovary B} Uterus C} Fallopian tube D} Cervix E} Peritoneal cavity 53] The initial event in the transfer of RBC to site of inflammation A} Margination B} Emigration C} Pavementation D} Diapedesis E} Phagocytosis 54] A semiconscious patient is brought to the emergency room. which of the following structure will be compressed along with Aorta A} Azygous vein and Vagus nerve B} Thoracic duct and Vagus nerve C} Thoracic duct and Azygos vein D} Azygos vein and both phrenic nerves E} Inferior vena cava 51] When the bulbur urethra wil injure. Which drug the pt. Which investigation for follow up A} Lipid profile + Renal profile B} Renal profile + Hepatic profile C} Renal profile + Hepatic profile + Cardiac 64] If the whole movement of Abduction of arm is lost the likely damage is in the Brachial Plexus A} Upper trunk + Posterior cord B} Upper trunk + Medial cord C} Lower trunk + Posterior cord . of cell D} Decrease in size of cell 63] Diabetic patient using Roziglitazone and metformin. of cell C} Decrease in no.B} microfilaments C} Mitochondria D} RER E} SER 57] Which is not related to embolus? A} Liquid B} Gas C} Thrombus D} Amniotic fluid E} Fat 58] Which of the following muscle compensate for supinator? A} Biceps Brachi B} Brachialis C} Brachioradialis 59] A patient presented with anterior wall M.I Artery involved is A} LAD 60] Which of the following molecule easily crosses Blood Brain Barrier? A} CO2 B} H2O 61] Hypervitaminosis A cause A} Ataxia B} Anemia C} Scaly Dermatitis 62] Hypertrophy indicates A} Increase in size of cell B} Increase in no. D} Lower trunk + Medial cord 65] Cords of Brachial Plexus are named according to Relation with A} Axillary artery B} Subclavian artery 66] Myelination in CNS is done by A} Schwann cells B} Oligodendrocytes C} Astrocytes D} Microglia cells 67] Captopril A} can be given in large doses in hypertensive crisis 68] Which of the nucleus is NOT related to Facial Nerve? A} Main Dorsal Nucleus B} Nucleus of Tractus Solitarius C} Nucleus Ambiguas D} Sensory nucleus 69] Platelets A} Are formed from precursor cells in the bone marrow B} Has no nucleus 70] Primary auditory cortex is present in A} Occipital lobe B} Superior Temporal Gyrus C} Frontal lobe D} Prefrontal lobe 71] Patient is suffering from factor VIII deficiency A} Genetically Engineered Factor VIII is available 72] Most Common cause of Thyrotoxicosis A} Graves disease B} Solitary nodule C} Toxic nodular goiter D} Diffuse goiter 73] Which of the following is involved in maintaining RESTING MEMBRANE POTENTIAL? A} Outward Potassium current B} Outward Sodium current C} Inward Sodium current . Frontal lobe 5-The most commom type of salivary gland tumor is: Salivary glands.Vertebre.Endoderm 4-Memory centre is located in: Insula.Parotids.pituatary gland causes dec.Ectoderm. head tilting. and also difficulty coming downstairs the likely nerve damaged is A} Oculomoter nerve B} Trochlear nerve C} Trigeminal nerve D} Optic nerve E} Abducent nerve 76] Substance has to cross semi permeable membrane in simple diffusion A} Is not saturable B} Require carriers C} Require energy 77] Carcinoma prostate drain into A} Para-aortic nodes B} External iliac nodes C} Internal iliac nodes D} External and internal iliac nodes 78] Pulmonary capillary wedge pressure is so A} Catheter used is wedge shaped B} Is direct measurement of left atrial pressure C} Is direct measurement of Right atrial pressure 1-The muscle involved in unlocking of knee joint is: Poplitus.D} Inward Chlorine current E} Sodium/potassium ATPase Pump 74] Heart sound produced by rapid ventricular filling is A} 1st heart sound B} 2nd heart sound C} 3rd heart sound D} 4th heart sound E} 5th heart sound 75] A patient presented with vertical diplopic.Rectus femoris.Soleus 2-Adult derivative of notochord is: Nucleus pulposis.functioning of .Sclerotome.Parietal lobe.Vertebral canal 3-Vertebrae is derived from: Myotome.Temporal lobe.Plantaris.Sublingual 6-Distruction of Ant.Submandibular.Annulus fibrosis.Dermatome. Inf.Jaw.interosseous A.Lyrnx 24-Olfactory area is present in: Ant.Dec.Occipital lobe.Vit D.Axillary.Whole blood for 18 days 16-Maxillary sinus opens into: Sup.21st 21-Patient having dec levels of factorII.D.perforating material.bHCG.Glom.Ulnar A.Hypertrophied adrenals.Medulla.Cerbral.Nasal septum 17-Patient having increased levels of IgE.Fungal inf.8th.the ideal fluid replacement would be: Packed RBCs.Glom.Radial.Folate def.Hypertrophied thyroids 19-Excitation of post.Ant. 26-Feature of Rickets and Osteomalacia is: Inc.K 22-Young pt.IX.Stress. 15-Soldier comes with heavy bleeding.temporal gyrus.antithrombin III.Dec Vit.Dec power of muscles below the leson on same side.Lateral and medial cord 9-Nerve injured in Ant.cord.Hyper vit.Inferior meatus.Plts:1lac is suffering from: Iron def.Inc.Post.granulosa.most probably suffering from: Bacterial inf.Crystallines.Costochondral joint.Infections.Median.Post cord.Brachial A.5/ul.Cerbral A.Ant.Adrenal cortex 7-Least chances of renal stones is associated with: Hyperlipidemia.HyperPTH.Dec sense of pain and temprature below the lesion on same side.Whole blood for 3 days.meatus.Parietal lobe 25-Patient with injury to left 8th cervical segment of spinal cord will not show following sign: Dec sense of position.Dec.VII.deviation of tongue.Shaft.Mixed.Ca.sense of touch and vibartions.X.Small CA 14-Patients comes with Rt.CEA.0..disloacation of Shoulder joint is: Musculocutaneous..Viral inf.Vit B12 def.most prabable finding would be: Hypertrophied PTH gland.Parasitic inf.Extensor plantar on left side.TLC:3.Suprascapular 10-Damage to scaphoid bone causes injury to: Radial A.vibration below lesion on same side.Aplastic anaemia 23-Nucleus ambigus supplies to: Teeths.Inc PTH.bone density 27-Most common site of malignancy in pts suffering from nuclear outbreak .pituatary would cause: Sweeting.the artery commonly involved in brain is: PICA.AdenoCA.Hunger pangs 20-Foetal period starts after which week: 11th.having Hb:6..Tongue.12th.Spinal.AFP.Acid Phospatase 13-Most common histology found in lung tumors is: Squamous. 11-Common site of fracture at rib is: Angle.Medial cord.Shivering.Hyperurecimia 8-Nerve supply to extensors of arm is supplied by: Lat.Colloids.tubercle 12-Following is not a tumor marker: PLAP.Middle meatus.most likely have: Dec.fasiculata.16th.AICA.Sup.Hypertrophied pituatry. 18-Autospy done on the patient having CRF. Thalasemia.epigastric.Neurons.Openning at bladder 33-Common relation of ureter is: Ant.Invloving protien synthesis 29-Most sensitive cells to hypoxia are: RBCs.high reticulocyte count is having: Folate def.Damaging nuclear membrane.Bladder..Commom Iliac V.Ovaries.polycythemia 42-Following vein would be dilated in portal HTN: Inf.to gonadal vessels and ant.Cystic A and Left Hepatic. 38-Hypothyroid patient in on thyroxine.Appendeces epiplocea 45-Bile salts are reabsorbed from the: Duedenum.Renal vein.Myelofibrosis.Fallopian tubes.Sickle cell anaemia 47-Type of defect in Heredetary spherocytosis is: Enzymatic defect..TSH.best marker to monitor his thyroid status is: T3.Mitochondrial shrinkage 32-Ureter isnot constricted at: Pelvic brim.Fallopian tubes 35-Least common site for ectopic pregnancy would be at: Ovaries.cheif.to iliac A.External Iliac V.Megaloblastic.T3 and T4.T4 and TSH levels 39-Patient having hyperplastic bone marrow.Astrocytes 31-Feature not associated with irreversible cell injury is: Mylein figures.T3..WBCs.T4.Karyolysis.Hausstral folds.Schawan cells. 44-Fat tags attached to the walls of large intestine are known as: Taenia coli.anaemia.Rectum 46-Following is an autosomal disease is: Cystic fibrosis.. 34-Patient feels pain after she underwent surgery for fibroids.to iliac A.Pouch of douglus.Bones 28-Virus acts on cells by: Damaging cell membrane.Internal and external V. 37-Venous drainage of urinary bladder is into: Internal Iliac V.Prostatic urethera 50-Terminal part of CBD in relation to pancrease is: .Jejunum.Ileum.Primary polycythemia.Platelets 30-Myelin in CNS is formed by: Microglia.at PUJ.anaemia.Leaukemia 40-Stimulus for Eryrthropoiten secreation is: Hypoxia 41-Erytropoiten is always raised in: Polycythemia vera.parietal.Oligodendrocytes.Nephrons.Gonadal.VitB12 def..Hemolytic anaemia 49-Ejaculatory duct opens into: Ureter.Colon..the cells lost in the gastrectomy are: Mucous.Thyroid..Psoas muscle.Cervix 36-Gall bladder is supplied by: Cystic A and Left gastrcA.Goblet cells.Inc Platelets is probably suffering from: Aplastic anaemia.Left colic V.Karyorrehxis.to gonadal vessels and post.Ureteric crest.Structural defect 48-Patient with jaundice..Post.Iron def.Lung.Sec.structure injured would be: Ureters.Only Cystic A. 43-Patient after gastrectomy is on VitB12.Hereditary spherocytosis.Haematopoitic.Greater omentum.Breast. Cholesterol 56-Capsule of liver in known as: Disse. 62-Femoral sheath is formed by: Fascia transversalis.it would be: FAP.caseous necrosis.Gram negative inf.Fat.phosp.Pregnenolone.Tubular adenoma.Kidneys.the genotype would be: XX.HyperPTH.and Fasica iliaca.XYY 66-Organ having least chances of infarction: Lungs.lie anteriorly 51-Following would ause massive infarction and destruction leading to patient death: Thrombus.nerve involved is: Median.Beneath Endothoracic fascia .transversus.Air embolism.inc Alk.Adrenaline.Embeded into pancrease.Glissons 57-Most common site of thyroglossal cyst is: Suprahyoid.Investing fasica.Internal oblique.Gram positive inf.Transversus muslce 65-In Turner syndrome.+ve AFB sputum.XO.Infrahyoid.most likely suffering from: Pagets.Carotid fascia 70-Neurovascular bundle in chest wall lies between: External and Internal intercostal mucles.Prolactin.Internal oblique 63-Patient with bone pains having normal Ca.External oblique.Osteomalacia 64-External spermatic fasica is formed by: External oblique.with cervical lymphadenopaty.Vertebral fascia.Superior surface 60-Shock without having vasodilation is likely associated with: Burns.Embolus.with 13th week of pregnancy after having crush fractures in RSA dies after 3 days.Traumatic 68-Pt.Hyoid 58-Wasting of thenar eminence.rectus abdominus.Fasica trans.Metaplastic polyp 54-Tyrosine derivateve doesnt include: TSH.Dopamine 55-Precursor of steriod hormone is: Progesterone.Innermost and enothoracic fasica.loss of sensation over thumb and index finger.Inferolateral.Ulnar 59-The base of urinary bladder is made by: Post surface.internal oblique.Bone mets.lie posteriorly.Fatty thrombus.Hyper VitD.Axillary.Internaland Innermost layers.Tubulovillous.Fat embolism 53-Patient with old history of adenoCA of colonis operated for polypectomy.Spleen 67-Type of necrosis in brain infarction: Couglative.Liquefactive.Liver.lymphocyctic infiltartion 69-Thyroid gland moves with swallowing because it is enclosed in: Pretracheal fasica.Nor adrnaline.XXY.XY.Villous adenoma.Ant surface.Cremastric muscle.on histologic evaluation pathologist labelled it as benign growth with no chances into malignant transformation.sign to look for TB on microscopy is: Chronic inflammation.Heart.Thromboembolism 52-22yrs female pt.most likely cause of death is: Amniotic fluid embolism.Radial.Superantigen infection 61-Following doesnt form the layer of inguinal canal: External oblique.. . 76-Facor affecting collagen synthesis during healing is: Infections.Interssusption.ANF 85-Tumor compressing optic chiasma would cause: Binasal haemianopia.the venous blood returns to the heart by: Muscle pump in calves 74-Circulation in heart is maintained by: Symphthatic sys..Vit C.haemturia.Copper dust.Ectopic gastric tissue 79-During lactation.Intercostal N.Excercise.nodes..Aldosterone.it is due to: Volvulus.nodes 73-In exercise.Amenorrhea is due to: Inhibition of LH and FSH thru Prolactin.Dec..Internal iliac.Huma 1.Neoplasia.External iliac L.Vit C def.Anxiety BY KASHIF NADEEM NISHTAR HOSPITAL.Superficial inguinal.Vit B12 def. 83-Microscopic feature of polyarterites nodusa is: Granuloma..Which of the following does not form the wall of perineum .Inflammatory cells 82-Child with sore throat after 3 weeks developed oedema.MULTAN Posted by Dr Sultan Ahmad 4 comments Reactions: Saturday. November 3. 2007 OBST&GYNEA FCPS-1(OCT 23 07) Q & A’s.Foriegn body 77-Mesothelioma is associated with: Vinyl chloride.Ashh & Dr.IgA nephropathy.Bitemporal haemianopia.71-Fibrous pericardium and medicatinal pleura is supplied by: Vagus N.Phrenic N.Pre aortic.Complete blindness 86-Growth hormone is maximally rasies in: Sleeping.Bilateral haemianopia.Dec oestrogen 80-Presence of pancratic tissue in gastric mucosa is termed as: Hamartoma.prolactin.cause would be: Minimal change disease.Fibrinoid necrosis 84-Angiotension II exerts its effects by activating: Renin.Local mechanism 75-Fisherman is presented with gingival bleeding and ecchymosis.Hunger.Iron def..Focal segmental GS.Silica. 72-Man is suffering from testicular ca.contributed by Dr.Dysplasia 81-Main difference between primary and secondary intention wound healing is: Granulation tissue.the lympahtic drainage of testicle is into: Para aortic L.Metaplasaia.Angiotension I.Wound contraction.Carbon 78-Child having meckels diverticulum is having bleeding per rectum.Parasymphthetic sys.Choristoma.Autonomic N.he is suffering from: Folate def.post streptococcal GN.Asbestos. Lesser omentum A.retrouterine pouch E.intestinal obstruction D.internal iliac nodes B.A pt presented with pain on defecation after severe constipation.n presented in emergency in state of shock.she suddenly experianced severe spasmotic pain in her rt iliac fossa.which of the following structure is not palpable through vagina A.ureteric stone D.forms falciform ligament C.renal colic C.inguinal nerve 3.related to left kidney E.it has .medial group of horizontal superficail nodes E..sacrotuberous ligament E.rectum C.vesicouterine pouch 7.external iliac nodes C.ishcial tuberosity D.most probable diagnosis is A.Lymph drainage of perianal skin is from one of the following A.related to left adrenal gland D.vertical group of nodes 4.cervix B.coccyx B.A.pain in constipation occurs due to ? A.pudendal nerve D.perineal nerve E.acute appendicitis B.autonomic nerves B.pubic symphsis C.lateral group of horizontal superficail nodes D.connects with deudenum B.Internal pudendal artey C.related to rt kidney 5.A young girl of 25 yrs went with her husband on honey moon after one month of marriage.ectopic Pregnancy E.sacrospinous ligament 2.acute cholecystitis 6.The peritoneum covers the anterior abdominal wall which is continous with the perineum. left renal vein 12.Which of the following is not a feature of shock A.hypophosphatemia E.present at the base of broad ligament C.Pudendal nerves 11.restlessness 10.autonomic nerves E.muslce weakness C.Genital tubercle in females give rise to .increased urine output D.supply of sphincter urethra comes from A.pelvis splanchnic nerves C.extend above the fundus of uterus 13.superior mesenteric C.5mm when dilated E.decrease cardiac output B.acute renal failure B.perineal membrane D fascia E fascia 8.diuretic use D.the inferior fascial layer also called as A.wat is the cause A.moves laterally to open medialy in the ovaries B.If a known hypertensive pt took some diuretic n died in 6 hours.Rt ovarian vein drains the following A.is 6cm long in adults D.rt renal vein E.has an abd ostiium which is 0.What is most appropiate about uterine tube A.colle's fascia B.scarpe fascia C.inferior vena cava B.inferior mesenteric D.ilioinguinal nerver B.chronic renal failure C.fascia which has superior n inferior layers.sacral splanchnic nerves D.hypokalemia ? 9.tachycardia E. absence of pleuro-peritoneal membrane D.bulbourethral glands 14.urachal cyst is the remnant of A.allantois B.1st pharyngeal pouch D.mesoderm of 3rd brachial pouch 16.ectoderm of 2nd brachial pouch B.1st pharyngeal cleft B.endoderm of 3rd brachial pouch E.vestibular glands E.kidney C.mekel's diverticulum C D E 15.Diaphragmatic hernia occurs due to A.A.clitoris D.Erythropoises occurs in middle trimester mainly from A.labia majora B.ectoderm of 3rd brachial pouch D.2nd pharyngeal pouch E.labia minora C.3rd pharyngeal cleft 17.2nd pharyngeal cleft C.both liver n kidney . absence of arch? 18.spleen D. absence of central tendon E.this problem occured due to abnormality in one of the following A. absence of pleuro-pericardial membrane B.liver B.A pt presented with an opening in the ant wall of sternocledomastoid muscle with pussy discharge coming out of it.Thymus develops from A.endoderm of 2nd brachial pouch C. absence of septum transversum C.bone marrow E. progesterone B .Thirst is least stimulated by A.GnRH 24.3rd brachial cleft 20.prolactin D.19.superficial inguinal E. supplies S.decrease blood volume B.oxytocin E.2nd brachial arch C.angiotension 2 C. supplies left atrium 22.1st brachial arch B.progesterone C.Fat deposition n ductal development of breast is the function of which hormone A.enters the rt heart border after passing through sulcus on posterir surface C.arises from anterior aortic sinus B.prolactin C.estrogen B.salt depletion D.external iliac C.wat is the lymph drainage A.RT coronary artery A.3rd brachial pouch D.deep inguinal 21.Superior parathyroid gland develops from A.decrease plasma volume .hcG 23.dopamine D. supplies both ventricles D.estrogen E.A node E.4th brachial pouch E.both internal n external iliac D.internal iliac B.Inhibition of ovulation is the function of which hormone A.In cervical carcinoma. subcecal 27.tendon of obturator internus E.One slide of a specimen shows stratified squamous epithelium with vascular smooth muscle and elastic fibres most probably the specimen is A.paracolic B.round ligament of uterus 28. he is diagnosed to have acute appendicitis on flexing his rt thigh inward n medially he cries with pain.Nerve supply of rectum A.transverse cervical ligament D. Glycogen 26.Stab wound above the piriformis muscle will damage which structure A.A pt presented with rt iliac fossa pain.inferior gluteal nerves C.blood pressure 25.growth hormone B.E.broad ligament B.Most imp hormone involve in gluconeogenesis is A.pelvic D.inguinal nerves 29.retrocolic C.short spanchnic nerves E.greater splanchnic nerves C.hypogastric plexus B.wat is the type of appendicitis he has A. inferior mesenteric plexus 30.which of the following structure has both intra n extra pelvic extension A.sciatic nerve D.pelvis splanchnic nerves D.ovarian ligament C.insulin C.paracecal E.thyroxine D.superior gluteal nerves? B.cervix .cortisol E.round ligament of ovary E. viruses E.ankylosing spondiolitis C.Low serum complement seen in which condition A.Apoptosis is inhibited by A.C5a is potent anaphylactic C.C5a is a potent chemotactic agent D.anus 31.anti DS antibodies C.antimitochondrial antibodies .uterine tube C.what is the most appropiate answer A.IL-1 D.C3a B.SLE B. apoptosis related protein ?? E… 32.dermatomiositis 35.rash n joint pain.anti SS antibodies B.which is the daignostic in this case A.capsases activation B.Pt presented with photosensitivity.arachdonic acid metabolites 33.rheumatid arthritis D.C5a ? C.Most potent chemotactic factor is A.uterus E. Activation of P-53 D.RA D.multiple sclerosis E.C3b n C5a are anaphylactant E 34.vagina D.B. Bcl-2 inhibition C.C3a is opsonin B.anticentromere antibodies E. 41.her karyotype is? A.on examination she was having short blind vagina wid normal vulva. suppression of lactation B.If a pt is on long term corticosteroid therapy. ? 40. hirsutism C.decrease wound healing C. E.to maintain its number C.47XY C. 38.A normal looking Girl came to you with primary amenorrhea.Most dangerous Side effect of estrogen therapy is A.normal breasts.she wil most probably hav increased risk of A.ammonia C. scanty pubic n axillary hairs.osteoporosis E 37. a single turn of DNA is 2nm C.urea B. to increase number E 39.Which is most imp indicator of muscle protein loss A.Platelets r stored at room temprature because A.repeated infections B.47XXY .36.thrombophlebitis D.to avoid allergic reaction D.46XY B.uric acid E..Double stranded in which two strands are antiparallel to each other B.absent uterus.to optimise its function? B.osteoperosis n fractures D E.none of above D.wat is true about DNA A..Can exist as helix in single stranded form as well as in Double stranded form D.. phosphoric acid? D.Which hormone convert to its active form at target tissue with the help of 5 a reductase? A.ribose C.Digoxin B. VLDL D. LDL C..47XXY D. HDL E.Androgen E.dihydoepiandosterone D.45XO 45.Progesterone E.45XO B..Griseofulvin C.46XX E 46.proline B.Hydrolysis of DNA gives A.ultrasound .A pt came to you with primary amenorrhoea. wht is her karyotype? A.short stature diagnosed as Turner syndrome. IDL 48.Which of the following has highest cholesterol content A. 49.Cimetidine D. 47.D..Which drug do not cause gynaecomasia? A..46XY C.Which is the best option in identification of Turner's syndrome A.spironolactone 50. Chylomicron remnants B.. E..Estrogen C. Testosterone B.webbed neck. peritubular interstitium of kidney C.. E.barr bodies C. 51.Pyruvic acid is used as intermedite in which step of cell metabolism A.albumin is decreased C. 55..glucose to acetyl co A C.Ct scan E.globulin is decreased D.uric acid C.urea and water B.proline D.water and ammonia E..decreased plasma volume 56.B.arginine C..Epinephrine.macula densa of kidney D..tyrosine E.ESR is decreased with A.ammonia 54.decreased blood volume E. 52.liver cell B.karyotyping D.End product of purine metabolism is A.alanine B.Erythropoitin is secreted from A.glucose D..phenylalanine 53.carbohydrate to acetyl co A B.norepinephrine n dopamine are derivatives of which aminoacid A.acetic acid to acetyl co A D E..temprature B.RBC's are . involve viscera only .liver E.increase in size of cell B..congenital abnormality E.permaturity D.infection 60.cyclophosphamide C.kidney 58.cisplatin B.is reversible D.radiations E.drugs B..largest of all cells E.Most common Cause of metaplasia is A.blephamide E 59.Commonest Cause of PDA A.CNS C.Tamoxifen has effect on which system A.breast D.ultrasonography B.biconvex B.chronic irritation C..is premalignant ? E.toxin C.have glycolytic enzyme activity 57.have nucleus D.A.Which of the following drug is most notorious for causing renal toxicity A.vincristine D.CVS B.Wat is the differentiating point of hyperplasia fron hypertrophy A.increase in size n no of cell C. 61.infections D.nonfragile C. epithelia D.primary tumor of brain D.fibrous tissue E.62.mRNA has a codon B.subacute degenaration of spinal cord E.lymphoid tumors C.Wat is the differentiating feature of hyperplasia from benign tumor A.STD E.What is the most true among following A.thiamine D.deficiency of THAIMINE(B1)causes all except A.tRNA is largest E..biotin C...bone marrow B.mRNA has anticodon C.VITAMIN involve in collagen synthesis is A..cartilage C. E.pyridoxine .increase in no of cell B..defects in alcohlics D.rRNA is most abundant n involve as ribosomes on endoplasmic reticulum D.cardiac anomalies B.AIDS is associated with all of the following except A.leukemia 67.Dysplasia is seen mainly seen in A.kaposi sarcoma B.cytplasm n nucleus ratio is derranged D.muscle wasting C. 64.are the same as parental cell C.deep tissue 63.ascorbic acid B.tRNA involve in protein synthesis 65.cns abnormalities in alcoholics 66. vaginal wart D.the most probable daignosis will be A.12 B.proliferative E.fasilitated diffusion C.HSV is associated with A.CA cervix B. Which phase will she be in? A.CO2 is tranported in lungs through A.22 .ferritin B.niacin 68.channels 71.HPV 73.Neural tube defects occur due to deficiency of A.simple diffusion B.ovulatory C.polyp D.squamous cell CA of skin C.CA vulva C.iron B.Pt presented on 18th day of menstrual cycle her normal cycle is such dat she ovulates on day 14.apoferritin 70.fibroepithelial lesion B.transferrin C.If a pt has presented with the wart on lateral wall of introitus.secretory? 74.vulval papules? 72.active transport D.follicular B.carriar proteins E.Iron is stored in the form of A.pre mestural D.folic acid C 69.Optic groove appears on left side of forebrain on day A.E.squamous CA of vagina E. 42 E. What is the most appropriate answer A.standard deviation e.limb deformaties E 76.Two groups one with CHD and associated risk factors is compared to another group with CHD but no associated risk factors.biguanides D.mean? B.32 D.insulin C.cardiac anomalies? D.umblical hernia D.mode C. cohort C. 79.administration of testosterone E.sulphonyl urea B.omphalocele B.suphonylurea n biguanides E. case control B.52 75.drugs blocking the action of T4? B.direct suppresion of T4 C.A newborn with anterior abd wall swelling and umbilical cord attached to it.wat is the option for treatment of exophthalmos A.C.hypophysectomy D.PT has exophthalmos n his T3 n T4 are increased.intestinal malformation 77.partial parathyroidectomy 78.gastroschiaosis C.central 80.Statistical average applicable to the measurement of a population is A. What type of study is it? A.Germen measles causes which abnormality. prospective .wat is the condition called A...congenital deafness? C.congenital cataract B.Treatment of diabetes in pregnant lady is A.median d.omentocele E. E coli? B.cell membrane B. C.impairment of hearing B...Oxytocin is given to a pt in labour.30S E. E 87.klebsella E.dec prostaglandin recp on cervix D.tetani C. 86. retrospective E.. random 81.D.Cl.is caused by long term antibiotic use.DNA D.it has effect on: A.side effect of streptomycin A.is caused by fungal B.botulinum E.CL perfiranges D.Cl. 84.cell wall C.Cl.chlostridium 85. D.usually seen in preexisting lung diseases? C. D.Which of the following is opportunistic organism A.Psudomembranous colitis is caused by A.Beta lactum acts through A.dec prostaglandin recp on myometriun 82.50S 83.Most common cause of puerperal sepsis is .About aspargillosis which is not correct A. oxytocin receptors on cervix B.difficile B.shigella D. E.oxytocin receptor on myometrium and endometrium C.salmonella C. wat is the diagnostic finding in this case for T.Inutero infection is not caused by A.the organ not found is due to abnormal development of A.trichomonas D.wat is the cause A.well developed secondary sexual characterstics.bacteriodes B.mesonephric duct B.AFB B.A 25 yrs old multiparaous presented with lower abd pain.caseous necrosis C.endometrial cyst C. . he has granuloma on histopathology.lactobacillis E..A woman presented with vulvar itching n pinkish purulent discharge.dermoid cyst B.cytomegalovirus D.A pt with cervical lymphadenopathy.trichomanas vaginilis B.E coli C.A pt presented with primary amennorhea.present breast but absent uterus.genital tubercle D 93.chlamydia C.Another question associted with aspargilosis 89.mumps C.ovarian CA D.on histology it showed solid n cystic areas intermixed but on laproscopy it was semi solid with brown centre.gonorhea D.HIV E.gonorrhea 88.montoux test D.teratoma 92..paramesonephric duct C.syphlis B.A..wat is the most probable daignosis A.x-ray E 91.enodmetrial CA E.B A.rhinovirus 90.chalymadia E. pyosalpinx D.now he developed same pain again.factor 9 D.recovery C.acute salpingitis B.At which step intrinsic n extrinsic pathways converge A.chlamydia D.trichomonas E.What is the drug of choice for DVT in ist trimester of pregnancy A.Most common organism involve in post operative abdominal & gynaecological infection is A.Chronicity D.Infectivity B.eosinophils C.heparin C.presense of delta antigen shows( HDV) A.chronic salpingitis C.wat is the condition involved A.Deficiency of Vit K will effect all of the following except A.factor 5 C.mast cells 98.94.warfarin D.wat type of cell will present at this time of disease A.Measure of severity/fatality of the disease 95.neutophils B.fibrinogen E.hydrosalpinx 97.basophils E.bacteroides C.lymphocytes D.aspirin B.om laproscopy fallopian tubes were sticked together n ruptured during operation.activater 10 .mefanamic acid 100.prothrombin 101.A pt presented with lower abd pain n purulent white dischrge.E coli B.indomethacin E.Pt has a history of infection 4 weeks back.gomorrhea 96.factor 7 B. Acute phase E. glucose B.Volume … 108.Transfusion reaction will not occur in a pt if we transfuse the A. of Na n Ca B.group O with B 104.Which drug is used in the treatment of hirsutism A.E coli C.CaCalmodulin complex is present in smooth muscles .activated 12 D.Type 2 hypersensitivity reaction includes A.group AB with A D.activated 11 C.estrogen C.high K low Na C.group A with O B.Which of the following does not cross placenta easily A.high conc.cryptoterone acetate E.CO2 105.testosterone D.prothrombin E. Of K & Ca with low Na E.gonorrhea E.antiandrogens B.diacrone 106.diffrence of smooth n skeletal muscle A.B. high conc.activeted 9 102.Most common organism involved in infection caused by IUCD A.amino acid C.group A with AB C.Ig G D.bacteroide D.Ig M E.trichomonas 107.low Na n Ca high K D.ECF differ from ICF in A.actinomycosis B.erythroblastis fetalis all other options were wrong 103.group B with O E. karyotype is A.alkaline phosphate 112.Ca C.If carbohydrate is separated from proteins wat will be the fate A.47XY E.cystedine C. If a test is able to both diagnose and eliminate diseased and non-diseased wht is it called? A.homocystine D.Lysozome contains A.Which of the following does not have sulpha group A.What is not true about levator anni muscle A.69XXY 110.cysteine B.45XO C.46XX B.Accurate C.What is not true about ureter A. MCQ on Significance A.Specific B.In partial mole.47XX D.methionine E.threonine 113.can be palpated at pelvic brim E.enter the bladder on superio lateral surface of trigone C.109.aggregation n precipitation B.supplied by L1 n L2 B.increase uptake int muscle 111.puborectalis forms part of it 114.enter the pelvis infront of common iliac artery B.Sensitive D… .glucose B.eliminate fron kidney? C.not hav sphincteric valve at opening into bladder D. 116.Na D.?? 115. hydrolases E. From a population two groups are selected on age basis and then subgroups are assigned randomly for the study. Source of Estrogen & Progesterone in last trimester is a) Chorionic Villus b) Placenta c) Overy d) Adenohypophysis e) Corpus Leuteum 124.Q on compensated respiratory alkalosis 121. Following is Autosomal Dominant Disorder a) Phenylketonuria b) Hereditary Spherocytosis c) Hemophilia A d) Cystic Fibrosa 123.Crossmatch blood to mothers plama C.Give red cell concentrates crossmatched wid fetal blood 122.Regulation .wht type of sampling is it? A.Multistage sampling B. About K .it is a) Squamous Cell CA .A 125.Crossmath blood to mothers blood D.K. For Rh blood transfusion to the anemic baby A.simple random sampling 118. Metabolic Alkalosis is caused by a) Chronic Hypoventilation b) Pancreatic Fistula c) hyperglycemia d) ATN 119.Random stratified sampling D. following is correct a) K is 98% in the cells b) normally completely reabsorbed in Distal tubule c) increase water affects K-balance? d) decrease in D.117.Q on Metabolic acidosis 120.crossmatch blood to babys plasma B.give wbc depleted transfusion E.Stratified sampling C.On examination of a female.a plaque like lesion seen on Postero-superior aspect of Vagina. 6 Pulmonary Arterial pressure increases with..5 All of the following have ability of phagocytosis EXCEPT a) Microgila b) Lymphocytes c) Kuffer cells d) Neutrophils Q.4 MS is characterized by.3 Congenital cataract is associated with a) chiken pox b) chromosomal abnormality c) small pox d) Rubella Q. a) Lewy bodies b) Patchy demylination & white fiber gliosis c) axonal neuropathy d) Q. urine in initial stages c) increase in s..1 Paracetamol a) increases PT b) its overdose causes Nephrotoxicity c) is a poor anti-inflamatory d) is more stronger than codeine e) causes met-Hb..2 ATN is associated with.emia more frequently than Phanacetin Q. a) Hyperkalemia b) conc.. a) Essential HPTN b) Sympthatic stimulation c) Nitroglycerine ..urea but normal s.b) Adenocarcinoma c) Rhabdomyosarcoma d) wart? Q.creatinnine d) Q. .4 md/dl MCV.9 Tissue which has in-ability to replace dead cells a) Heart b) Kidney c) Liver d) Bone e) Bowl Mucosa Q.causing anaesthesia of an autonomic ganglion.12 Glucronide conjugation of drugs a) causes inactivation of drug b) is Type 1 reaction c) makes drug insoluble in water d) increases its effects Q.10 Non-essential amino acid includes a) Leucine b) Iso-leucine c) Methionine d) Tyrosine Q.> 120 fl. is a) sublingual glycerine trinitate b) intra. on investigation he is having Un-conjugated Bilirubinemia. Hb= 7.11 Metabolic Alkalosis is caued by a) Chronic Hypoventilation b) Pancreatic Fistula c) hyperglycemia d) ATN Q.7 in an Asthamtic female. the long needle goes far into greater palatine canal.13 A patient having generalized body weakness & numbness.venous nitroglycerine c) propranalol d) Q.8 During giving local anaestheia.d) Excercise e) Hypoxia Q.immeditae treatment of HPTN during intra-operative HPTN.as a result along with dryness of nasal mucosa also occurs a) decreased secretions of Parotid b) decreased secretion of Submandibular & sublingual glands c) Q. . .15 Cause of joint pain in Deep sea divers a) arterial embolism b) venous embolism c) DIC d) thromboembolism Q. a) Hemolysis b) Vit B 9 def.15 Cause of joint pain in Deep sea divers a) arterial embolism b) venous embolism c) DIC d) thromboembolism Q.in a fasting man the cause of concenterated urine is a) Skin b) Blood vessels c) Pupil d) Salivary glands e) Ciliary muscle Q.in a fasting man the cause of concenterated urine is a) ADH b) No Oral intake c) Aldosterone d) increased sweeting Q..the anemia is due to .17 A 35 yr old man is having bilateral ptosis of eye during evening since last few weeks. d) Hypersplenisim Q..the confirmatory test of the diagnosis will be a) Tensilon Test b) Ach Receptor Antibody Test c) blood culture d) CT scan Brain Q.16 In summer.16 In summer.18 if a High Dextrose water is infusion given to a person.14 Tissue with Dual autonomic supply but not reciprocal suuply is a) Skin b) Blood vessels c) Pupil d) Salivary glands e) Ciliary muscle Q.it inhibits . c) Vit B 12 def. 3 yrs ago. Now he notices a bulging swelling above the clavicle on sneezing & coughing. a) Injury to deep fascia of neck b) demage to supra-pleural membrane c) Un-united first Rib Fracture Q.22 A young man had sinjury to the neck with sum sharp object.24 Dorsal Rami of the cervical nerves innervates a) Abductor Pollices b) Extensors of Limbs c) Extensors of Trunk d) Stabilizers of Shoulder Q.S4 b) Pelvic splanchnic Nerves c) vagus nerve d) Q.S3.a) Cotisol b) ADH c) Aldosterone d) Insulin Q..23 Middle meningeal artery if ruptured while meinges are intact.20 which does not take part in inflamatory reactions a) Adrenaline b) Histamine c) 5-hydroxy tryptamine d) Dopamine Q.19 Parasympthatic supply of Sigmoid Colon is a) Sacral segments S2.the Haemorrhage will be a) Epidural b) Subdural c) Intracranial d) Q. 21 Parietal & Visceral Pericardium is supplied by a) Phrenic Nerve b) Sympthatic Nerves c) Vagus neve d) Cardiac Plexus Q..25 Muscles of Hand supplied by a) ventral rami of C-8 b) Ventral branches of T-1 c) Median Nerve . this swelling most probably due to. 20 mg/dl s. the nerve injured is a) Median Nerve b) Axillary nerve c) Radial Nerve d) Musculocutaneous Nerve Q.Uric Acid-.5 mg/dl s. after which He is unable to Flex his elbow & having loss of cutaneous sensations on lateral aspect of forearm.29 Pharmakokinetic interaction among drugs is examplified by : a) Shortening of action of Procaine by Adrenaline b) increase peripheral toxicity of Levodopa with Carbidopa c) increase toxicity of Methotrexate by Aspirin d) prevention of Nitroglycerine induced tachycardia by Propranolol e) blokade of acetylcholine receptors by atropine Q.26 A person has sharp knife injury to upper medial aspect of arm.0.7.30 & Fasting BSL--.Na -.30 A 30 years old 75 kg Diabetic man with recent diagnosis of Hypertension.HCO3 -.5 mmol s.7 mg/dl s.d) Ulnar Nerve e) Radial Nerve Q. On ECG : Left Ventricular Hypertrophy s.Urea -.103 mmol s. 27 A Transplant from identical twin is a) Allograft b) Heterograft c) Homograft d) Isograft e) Xenograft Q.Creatinine -.28 Therapeutic Index of a drug indicates its a) Effectivity b) Efficacy c) Potency d) Toxicity Q.2.23 mg/dl 24 Hr Urinary Proteins-.140 mmol s.80 mg/dl The cause is a) Essential HPTN b) Diabetic Nephropathy c) Pheochromocytoma d) Addison Disease e) Primary HyperAldosteronism .Cl --. K-. 34 Multipule sclerosis is characterized by...38 Kleinifilter syndrome a) contains single X chromosome b) less chances of Mental retardation .32 GFR is measured by clearance of a) Inulin b) Creatinine c) Urea d) Glucose Q.Q.creatinine is normal Q. a) Lewy Bodies in Neucleus b) Ptachy demylination with white fiber gliosis c) Axonal degeneration d) Q.Urea is increased & s.33 Sarcoidosis is diagnosed microscopically by a) Granuloma with Asteroid b) Non-caseating Granuloma c) Caseating Granuloma d) Macrophages & Giant cells Q.35 What is INAPPROPRIATE about Vagus Nerve. a) its longest Cranial Nerve b) passes in between IJV & ICA c) passes posterior to Carotid Sheath d) Gives Recurrent Laryngeal Nerve e) gives Superior Laryngeal Nerve Q.36 Thymus is a) Rich in Lymphocytes b) Present at birth but shorly regree after birth c) derived from 4th Pahryngeal Pouch d) is a part of Thyroid e) Lies posterior to Trachea Q.31 Acute Tubular Necrosis is characterized by a) Hyper-kalemia b) Initially concenterated urine c) Metabolic Alkalosis d) s.37 True statement about Down Syndrom is a) Trisomy 23 b) contains single X chromosome c) more chances with increasing maternal age Q.. 43 @ Leprotomy Retroperitoneal Abscess was found.41 Thoracic Duct.the leision lies in a) Facial Canal .the site woud be a) Stomach b) Sigmoid Colon c) Transverse Colon d) Ascending Colon e) Small intestine Q.44 Source of Estrogen & Progesterone in last trimester is a) Chorionic Villus b) Placenta c) Overy d) Adenohypophysis e) Corpus Leuteum Q.40 At 35 weeks of Gestation.45 A patient having Facial deviation along with loss of sensations on anterior 2/3 rd of tongue. a) recieves Lymph from both right & left sides b) lies in posterior mediastinum c) passes just right to the thoracic aorta d) extends from upper abdoment to neck e) has a dilatation called Cisterna Q.39 Following is Autosomal Dominant Disorder a) Phenylketonuria b) Hereditary Spherocytosis c) Hemophilia A d) Cystic Fibrosa Q. there is increased Alpha-Fetoprotein..its most probable cause is. a) Down syndrome b) Turner syndrome c) Spina Cystica d) Hydrocephalous e) Q...c) phenotypically female Q..42 Rate of passage of food in esophagus depends upon a) Gravitational Force b) Neuromuscular excitation different @ upper & lower oesophagus c) progressive peristalsis in esophagus d) Acidity Q.in sample taken from amniocentesis. 48 In a Pituitary Adenoma. married to a female having B+ (Bo.children can have all blood gp EXCEPT a) AB +ve .the Bitemporal Hemianopia occurs due to leison of a) Central part of Optic chiasma b) Optic Nerve c) Optic Tract d) peripheral Part of Optic chiasma e) Lateral Geniculate body Q.49 Muscle of Forced Expiration is a) Internal Intercostal b) External Intercostal c) Sternoceidomastoid d) Scalenus Anterior e) Scalenus Medius Q.50 Na Retension is associated with a) Dehydration b) increase ADH c) Decreased Aldosterone d) Heart Failure e) Hyperthyroidism Q.b) Internal auditary meatus c) Facial Nerve Nucleus d) Stylomastoid Foramen Q.51 Von-Willebrand Disease is a) Autosomal Dominant b) Autosomal Recessive c) in some family members asoociated with Factor IX deficiency d) in some family members associated with normal Factor IX Q.DD) Blood group.46 All are the branches of Vagus nerve EXCEPT a) Auricular b) Lacrimal c)Pharyngeal d) Recurrent Laryngeal Q.51 A male wd A+ (Ao.Dd ) blood group.47 Following nerve lies in the Cavernous Sinus a) Opthalmic b) Occulomotor c) Abducent d) Trigeminal Q. 54 Difference between Adenoma & Hyperplasia is a) increase in size of cell b) increase in No. d) PPI e)whole blood Q. a person has inability to stand from sitting position.55 In a CLD patient presenting with Haematemesis.53 When a branch of Pulmonary Artery is blocked by embolus. the nerve demaged is a) Infragluteal b) Supragluteal c) Schiatic d) Femoral e) Obturator Q.the preferred immediate treatment is a) Octeriotide b) Vit K c) formed Platelet Conc.58 About K .following Lung function increases a) Alveolar Co2 b) Alveolar O2 c) Pulmonary artery O2 d) Pulmonry artery Co2 e) Ventilation Perfusion Q.52 Vital Capacity a) measured directly by Spirometry b) is decreased in some Lung diseases c) is the sum of Tidal Volume+ERV+IRV Q.57 After a trauma.b) AB -ve c) A + ve d) B +ve e) O -ve Q.Regulation . following is correct a) K is 98% in the cells b) normally completely reabsorbed in Distal tubule c) increase water affects K-balance .56 Among most important initial steps in apoptosis a) Activation of Bcl-2 Receptors b) Activation of Caspases c) Water influx into the cell d) Q. of cells c) Capsule d) Q. 60 An unconsious man with pin-point pupils not responding to Naloxone.59 A man with persistant fits without regaining consciousness for 30 minutes.62 The Hormone which stimulates release of HCO3 from Pancreas without stimulating pancreatic enzyme secretions.61 Stomach gets its arterial supply from all EXCEPT a) Ceiliac artery b) Splenic Artery c) Superior Mesenteric Artery d) Hepatic Artery Q.is a) CCK b) Secretin c) Gastrin d) Pepsin e) Mucin Q. the immediate treatment is with a) Carbamazepine b) Valporate c) Lorazepan d) Q..d) decrease in D.the probable cause is a) Heroin b) Hasheesh c) Morphine d) Phenobarbitone Q.65 Which infection is not caused by DNA virus a) Chicken Pox b) Herpes Simplex c) Herpes Zoster d) Mumps e) Small Pox ..K Q. Q.64 Stenson's duct a) arises from posterior surface of Parotid b) pierces the Buccinator muscle c) opens in the oral cavity against ..63 Supra-Renal Gland arterial supply a) recieves all braches directly from Aorta b) recieves blood from 3 sources c) recieves branches from both renal arteries d) recieves blood from inferior phrenic artery Q. 67 About CSF most appropriate statement is a) it is ultrafilterate of Plasma b) has more glucose than blood c) has cushioing effect d) absorbed in Choroid Plexus Q.given Parenteral Vit B12.71 A female underwent Gastrectomy for Zollinger-Ellison Syndrome.it is a) Squamous Cell CA b) Adenocarcinoma c) Rhabdomyosarcoma d) wart Q. the inability to absorb B-12 is due to loss of. October 18.70 Basal Cell Carcinoma involves a) Buccal Mucosa b) Hard Palate c) Soft Palate d) Lower Lip e) Oral Cavity Q.Q.66 On examination of a female.a plaque like leision seen on Postero-superior aspect of Vagina.69 Metastatic Calcification occurs mostly in a) Kidney b) Bone c) Lung d) Intestine e) Pancreas Q.. 2007 Welcome friends! .@ apex of lung there is increase in a) Blood Flow b) Ventilation c) pO2 d) pCO2 Q.68 On standing from a sitting position. a) Parietal Cells b) Gastric Cells c) Cardiac Ceells Posted by Dr Sultan Ahmad 1 comments Reactions: Thursday.. I can say that after the 1st few days of exam. May I request you all who have benefited from this website in any way to kindly post your own recalled questions soon after the October exam (all specialities). October 14. p. one is not able to jog the memory and reproduce much. Posted by Dr Sultan Ahmad 2 comments Reactions: Sunday. Out of experience.com Jazakallah. Do remember me in your prayers as I am writing the oct exam aswell. Please leave comments at the end of the posts or at the extreme end of this page regarding feedback and suggestions.s. So. Now that the july 23 exam date is just around the corner I wish you all success and good luck! May we all shine and rise in all our endeavours. 2007 Gyn Obs July 07 By Dr Ayesha Bisharat GYN OBS Questions JULY 2007 Hi just got to know of dis site through cpsp community otherwise would have mailed u questions way back anyways great effore am mailing fcps part 1 obs gynae mar n jul 2007 question no 1:eversion of endocervical epithelium in pregnancy is due to: a)physiological change b)chronic cervicitis c)carcinoma in situ d)estrogen stimulation e)epidermadization of epithelium 2:primary oocyte surrounded by a non cellular layer: a)theca interna b)theca externa c)corona radiata d)cumulus oophurus e)zona pellucida . And send your recalled stuff to [email protected] o Allaikum everybody. kindly do make sure to send your recalled stems/q's so future candidates could benefit from them. she has now watery vaginal discharge.causative organism: a)bacteroides b)chlamydia c)herpes simplex d)gonococcus e)trichomonas vaginalis 7: a married woman has right sided tubulo ovarian abscess which was removed in a surgery had an iucd placed before.LH is increased: a)2 fold .just 2 days before ovulation.has painful red papules n ulcers on vagina and vulva has fever and tender inguinal lymph nodes.the late secretory phase of menstrual cycle is not characterised by: a)the spongy layer becomes thick b)corpus luteum secretes estrogen n progesterone c)the endometrial thickness is doubled d)endometrial cells have subnuclear vacuolations e)endometrial glands become tortuous 9.question no 3: simple columnar epithelium is the lining of: a)epithelium lining the ducts b)cornea c)testis d)uterine tubese) question no 4:48 years lady has carcinoma cervix that has involved the cervix and has involved the vagina but not its lower third and has not extended onto pelvic walls stage of ca: a)stage 1ab)stage 1bc)stage 11d)stage 111e)stage 4 question no 5:38 years lady has endometrial ca that has involved upto half the tickness of myometrium and ovaries are normal stage: a)stage 1a b)stage 1b c)stage 1c d)stage 11 e)stage 111 question no 6:20 yrs old girl has been raped.causative organism: a)chlamydia b)gardenella c)tuberculous d)bacteroides e)gonococcus 8. 72 hours 12.paracetamol: a)is more potent analgesic than codeine b)is a weak anti inflammatory agent c)overdosage can cause nephrotoxicity question no 15:anticoagulants are contraindicated in: a)cerebral infarction b)thrombocytopenia 16.24 hours c)12 -36 hours d)24 .examination revealed: a)multinodular goitre .which of the following investigation is least likely to benefit: a)clotting time b)FDPs c)prothrombin time d)platelet counte)serial fibrinogen levels 17.b)2 to 4 fold c)2 to 6 fold d)6 to 8 fold e)8 to 10 fold 10.20 yrs lady has endotoxic shock after septic abortion.30 yrs male has dyspnea on lying down.the bioavailability of a drug determines : a)efficacy b)efficency 14.48 hours e)24 .sperms stay active in female genital tract: a)12 hours b)12 .the volume of distribution of a drug is not influenced by: a)age b)sex c)cardiac failure d)renal failure 13.menopause is characterised by: a)atrophy of uterus b)decrease in ovarian size c)derease estrogen increased FSH AND LH d)decreased estrogen increased FSH decreased LH 11. JVP is differentiated from carotid pulse: a)does not change with abdominal pressure b)does not change with posture c)does not change with respiration d)pulsation varies with respiratione)prominent outward movement 20.the point of reference is: a)ischial tuberosity b)pubic arch c)pubic symphysis d)ischial spines e)sacral promontory 21.following is not a tumor marker: a)acid phosphatase b)beta hCG c)alpha fetoproteins d)CEA e)placental alkaline phosphatase 23.at LSCS the artery damaged in superficial fat in suprapubic incision: a)inferior epigaSTRIC artery b)superior epigastric artery c)internal iliac artery 22.and head at zero station.baby cephalic.a multigravida admitted with labour pains examination revealed os 6 cm dilated.b)retrosternal goitre c)colloid goitre d)cold nodule 18:cortisol: a)increases amino acids n glucose in blood b)is a glucocorticoid c)is found in circulation bound to proteins d)metabolised in liver e)formed from cholesterol 19.question o 23:beta hCG:a)it is a polypeptideb)it has action similar to FSHc)it prevents involution of corpus luteumd)e) 24:adrenal medulla will selectively secrete norepinephrine in the condition:a)exerciseb)sleepc)cold exposured)hemorrhagee) 25:cardiac output is decreased in:a)pregnancyb)standing or sitting from lying positionc)exercised)e) . due to injury and communication in: a)ureter n bladder b)ureter n vagina c)bladder n cervix d)bladder n vagina e)bladder n uterus question no 31:perineal body is injured.26:cardiac output is decreased in all except:a)sleepb)starvationc)sittingd)e) 27:epinephrine will selectively onstrict muscles in:a)arteriolesb)bronchiolesc)ciliary muscled)pupilse)intestinal walls 28:spores should be disinfected by:a)dry heat at 100cb)dry heat at 60cc)dry heat at 160cd)cidex solutione)moist heat 29:following is not a disinfectant: a)derivatives of salicylic acid b)alcohol c)soap d) e) question no 30: that she had urinary incontinence.muscle damaged is: a)bulbospongiosis n superficial perineal muscles b)superficial n deep perineal muscles c)external anal sphincter n deep perineal muscles d)ischiocavernosus n sphincter urethrae e)sphincter urethrae n deep perineal muscles 32:pudendal nerve: a)formed from ant primary rami of second third n fourth sacral spinal nerves b)crosses the ischial spine medial to internal pudendal artery c)enters the pudendal canal on the lateral side of ischiorectal fossa d)leaves the pelvis by passing through the upper part of greater sciatic foramen e)reenters the pelvis through lesser sciatic foramen 33:following does not form wall of inguinal canal: a)aponeurosis of external oblique b)internal obliquec)conjoint tendon d)lacunar ligament e)fascia transversalis 34:following does not pass through lesser sciatic foramen: a)inferior gluteal vessels b)tendon of obturator internus . c)nerve to obturator internus d)pudendal nervee)internal pudendal artery 35:after LSCS patient had peritoneal hemorrhage she was reopened bleeding most likely from: a)branch of ovarian artery b)branch of uterine artery c)internal iliac artery d)obturator artery e)external iliac artery no 36:left ovarian artery is a branch of: a)abdominal aorta b)left renal artery c)uterine artery d)left inferior mesenteric arterye)obturator artery question no 37:patient had laparotomy for ovarian mass mass was burried in ovarian fossa artery likely to be injured is:a)common iliac vesselsb)internal iliac vesselc)external iliac vesseld)ovarian arterye)obturator artery questoion no 38:insulin stimulates transport of glucose into: a)renal tubular cells b)neurons of cerebral cortex c)skeletal muscled)intestinal cells e)liver question no 39:the secretion of following is not increased in pregnancy: a)cortisol b)growth hormone c)thyroid hormone d)prolactine)PTH question no 40:a patient had hypophsectomy for pituitary tumor she has amenorrhea for 8 months ovulation induction can be done in her by: a)clomiphene citrate b)pulsatile hCG c)hMG d)hMG followed by pulsatile hCG question no 41:following does not occur in pregnancy: a)increase in fasting blood sugar b)increase in uric acid level c)increase in serum urea level d)decrease in serum albumine)decrease in serum proteinse)decrease in nitrogen levels question no 42:respiratory acidosis occurs in: . a)emphysema b)hyperventilation c)ingestion of excess amount of sodium bicarbonate d)starvatione) vomiting question no 43:CO2 is transported from the alveoli of lungs into blood in lungs via: a)diffusion b)active transport c)facilitated diffusion d)secondary active transporte)osmosis question no 44:O2-Hb dissociation curve delivering O2 in normal limits will have: a)PaO2 40mmHg SO2 60% b)PaO2 68mmHg SO2 78% c)PaO2 96mmHg SO2 90% d)PaO2 123mmHg SO2 98% e)PaO2 256mmHg SO2 99% question no 45:b-hCG has alpha n beta units,each unit has aminoacids:a)102 alpha 210 betab)100alpha n 160 betac)92 alpha n 156 betad)e) qyuestion no 46:mothers milk differs from cows milk in that it contains:a)calciumb)lactosec)proteinsd)vitamin D question no 47:pelvic outlet: a)can b increased in supine position b)begins at the level of ischial spines c)dimensions change by movements of coocyx d)contracture is more common than mid cavity probleme) question no 48:patient has endocrine disorder has metabolic alkalosis and hypokalemia,he has excess of:a)ACTHb)aldosteronec)cortisold)estrogene)progesterone question no 49:hormone that stores carbohydrates, lipids n proteins in cells is also known as hormone of abundance:a)growth hormoneb)cortisolc)thyroid hormoned)estrogene)progesterone question no 50:hyperglycemia is caused by:a)insulinb)glucagonc)estrogend)progesteronee)growth hormone question no 51:ketoacidosis is caused by: a)increased production of glucose b)decreased production of glucpoose c)decreased utilisation of carbohydrate in body d)insulin excess question no 52:insulin secretion is stopped by:a)glucagonb)decreased k levelsc)somatostatind)e) question no 53:hormone that stores carbohydrates,lipids n proteins in body is:a)growth hormoneb)thyroid hormonec)cortisold)insuline)PTH question no 54:8 yrold girl has vaginal bleeding she is taller than her age and examination revealed an abdominal mas arising from pelvis she is suffering from: a)constitutional; symptoms b)hypothalamic tumors c)hypothalamic mass d)granulosa cell tumor e)adrenal gland tumor question no 55:least positive titre for widal test isa)1:80b)1:160c)1:260d)1:320e)1:156 question no 56:all of the following can b stored at 4c except:a)shigellab)vibrio cholerac)d)e) question no 57:vaccine is available for all except:a)EBVb)typhusc)pneumococcusd)e) question no 58:patient has presented with fever n generalised lymphadenopathy, bone marrow exam normal,weight loss positive,he is suffering from:a)metastatic carcinomab)hogkins lymphomac)non hogkins lymphomad)CMLe) question no 59:patien is taking glucocorticoids for 10 days,she wont suffer from:a)moon faceb)hypotensionc)thin skind)e) question no 60:gestational age can best be estimated from:a)abdominal circumference at 36 wksb)biparietal dia at 28 wksc)crown rump length at 8 wksd)femur length at 32 wkse) question no 61:turners syndrome,most definitive diagnostic test:a)presence of barr bodyb)chromosomal analysis question no 62:14 yrs girl has primary amenorrhea,normal breast development,secondary sexual characteristics normal,short blind ending vagina,slight pubic hair,usg revealed absent uterus,underlying cause: a)congenital adrenal hyperplasia b)chromosomal defect c)failure f development of genital tubercle d)defect of paramesonephric duct question no 63:the lumen of uterine cavity is obliterated by fusion of: a)decidua basalis n spongy layer b)decidua basalis n decidua capsularis c)decidua capsularis n decidua parietalis d)decidua parietalis n decidua basalise) question no 64:a woman has been exposed to radiation in young life,now she has visited a gynaecologist after some years,organ most likely to b affected:a)fallopian tubesb)cervixc)ovaryd)endometriume)vagina question no 65:repeated blood transfusions (AB n Rh compatible) in an individual will lead to:a)haemochromatosis of liverb)hypocalcemiac)haemoglobinuriad)transfusion reactione) question no 66: Rh incompatibility occurs in a mother if she has:a)Rh negative fetusb)Rh positive fetusc)d)e) question no 67:the best way to prevent Rh isoimmunisation in a woman who has given birth to Rh positive fetus:a)blood transfusionb)platelet transfusionc)serumd)Rh immunoglobulinse) question no 68:a person brought in er had burns,he was admitted later on he developed fever and bleeding diathesis,he is suffering from:a)DICb)hepatorenal failurec)hepatic failure of prothrombin productiond)renal failuree) question no 69:following is not a cause of DIC:a)pre eclamsiab)amniotic fluid embolismc)cervical carcinomad)abruptio placentaee)PPH question no 70:following is not a cause of DIC:a)abruptio placentaeb)burnsc)malignant tumorsd)e) question no 71:multigravida P9+0 delivered a boy in a village 2 hours later she started bleeding profusely had oozing from gums she was brought to hospital her labs showed hb 6 TLC 15000/ul platelets 30000/ul she is suffering from:a)DICb)PPHc)clotting disorderd)bleeding diathesise)ITP question no 72:27 year old female had profuse vaginal bleeding 2 hrs after delivery her labs showed hb 6 TLC 15000/ul plt 30000/ul peripheral blood film showed burr bodies,she is suffering from: a)DIC b)PPH c)ITP d)von willebrand disease question no 73:vit k directly influences d reaction:a)conversion of fibrinogen to fibrinb)prothrombin to thrombinc)activation of factor 7d)formation of fibrinogen question no 74:intrinsic n extrinsic pathways converge upon:a)activation of factor 7ab)activation of factor 5ac)factor 12ad)prothrombine)tissue factor question no 75:bacteria that grow at 42c:a)vibrio cholerab)gonococcusc)shigellad)Ecolie)salmonella question no 76:following is not true about endocarditis:a)blood cultures for causative organisms r usually negativeb)is caused by streptococcus group of bacteriac)is more common in elderly groupd)valvular involvemente) question no 77:cause f subacute bacterial endocarditis:a)H.influenzaeb)Ecolic)streptococcus viridansd)e) question no 78:interferon:a)binds to DNAbbinds to RNAc)is a protein derivatived)inhibits protein synthesis n their phosphorylation question no 79:highest energy compound:a)ATPb)glucosec)hexose phosphated)starche)glycogen question no 80:the end product of carbohydrate metabolism:a)fructoseb)glucosec)galactosed)starche)pyruvate question no 81:L-dopa taken by a patient will decrease :a)estrogenb)progesteronec)prolactind)cortisole) question no 82:isoniazid:a)cant b administered with rifampicinb)is contraindicated in nephropathyc)pyridoxine is used prophylactically to prevent its neurotoxic effectsd)e) question no 83:hepatitis b following is not true:a)has an incubation period less than that of hep ab)caused by sexual contactc)caused by DNA virusd)e) question no 85:actinomyces does not cause disease in:a)boneb)lungc)brain question no 86:disseminated coccidiodomyces following is not true:a)infection will not provide life time immunityb)c)d)e) question no 87:middle aged man crossing the road stumbled n hit by a car he fell down had injury to neck of fibula of his left leg was taken to hosp examination revealed that he is unable to dorsiflex and evert his left foot the injury is to: a)common peronel nerve b)tibialis post c)superficial peroneal nerve d)deep peroneal nervee) question no 88:ECF:a)provides nutrients to cellsb)provides glucose to cellsc)bathes the cellsd)takes away electrolytese)takes away wastes question no 89:spermatogenesis is stimulated by:a)FSHb)testosteronec)LHd)FSH n testosteronee) question no 90:patient brought in shock in er successful fluid replacement will b indicated by:a)decrease in tachcardiab)increase in bpc)increase in urine outputd)e) question no 91:cardoiac cycle:a)60% of ventricular filling occurs in diastoleb)end diastolic left ventricular vol is 100 mlc)ejection fraction is 75%d)the notch of d pulse is due to atrial systolee) question no 92:a man has a pulse rate of 72/min impulse originates in:a)AV nodeb)SA nodec)left bundle branchd)purkinje fibrese) question no 93:muscles r derived from:a)mesodermb)ectodermc)endoderm question no 94:in our country d cause of cell inujury is:a)alcoholb)hypoxiac)ischemiad)ionising radiatione)malnutrition question no 95:in our country cause of fatty liver is:a)alcoholismb)amoebiasisc)due to hep b n cd)increase intake of fatsdecreased protein utilisation question no 97:in d female d neck of urinary bladder is located :a)in front of rectumb)below d urogenital diaphramc)in d urogenital diaphramd)above d urogenital diaphrame)behind symphsis pubis question no 98:the highest content of triglycerides is in:a)chylomicron remnantsb)HDLc)LDLd) IDLe)VLDL question no 99:following is not a phospholipid:a)cephalinb)lysolecithinc)lecithind)plasminogene)sphingolipid question no 100:a human being can survive without:a)calciumb)proteinsc)carbohydratesd)lipidse) question no 101:patient has pyuria but has negative urine culture for organisms this happens in all conditions except: a)acute post streptococcal glomerulonephritis b)polycystic kidneys c)urine stored at room temp for long d)alkaline urinee)vesical calculus question no 102:broad ligament a double layered fold of peritoneum ,ovary is attached to it by: a)mesoovarium b)suspensory ligament of ovary c)round ligament of ovary d)cardinal ligament Posted by Dr Sultan Ahmad 5 comments Reactions: SURGERY July 07 Contributed By Dr Ahsan Siraj. FCPS Part 1 SURGERY AND ALLIED 5TH JULY 2007 Note: Only correct answers are given as choice “A”. Some questions do contain other choices but you consider “A” the right choice. ANATOMY 01. A 10 year old boy with shoulder injury lost abduction of his shoulder up to 30 degree which muscle is lost: Supraspinatus Deltoid 02. A boy is brought to a hospital with injury at elbow joint lost extension of medial 4 fingers at MTP joint, abduction of thumb with intact sensation due to: Injury to Radial nerve at elbow 3. Which of the following muscle is not supplied by median nerve: Adductor polices. 4. Regarding palm: Superficial palmer arch is below palmer apeneurosis. 5. Which of the following muscle have dual nerve supply from median and ulner nerve: Flexor digitorum profundus. 6. Infection of the first pulp space lymph nodes will first drained. Epitrochlear lymph nodes. 7. Which of the following muscles is flexor at hip and extensor at knee: Sartorius. 8. In an injury to knee joint a man is unable to extend his knee the root value affected: L3-L4. 9. In the injury to neck of fibula the artery damaged: Ant. Tibial artery. Peroneal artery. 10. Regarding post. Triangle: Base is form by middle 3rd of clavicle. 11. Bifurcation of common carotid artery: Beneath the any. Border of sternocladomastoid at the sup. Border of thyroid cartilage. 12. Great saphenous vein has how many valves: 20. 13. Vertebral venous plexus: Has no communication with cerebral sinuses. Lies in epidural space. Is formed by two vertebral veins. 14. A 40 years old man feels pain in his gastrocnemeus muscle after he walks 100 meter which relieves when he stops walking is likely due to obstruction of: A. Post. Tibial artery. 15. Subclavian artery grooves over: A. 1st rib. 16. What is inappropriate about azygus vein. A. It enters the thoracic cavity thru esophageal opening. 17. Which of the following structure arches over root of left lung: A. Arch of aorta. 18. Weakest point of rib is: A. Angle of the rib. 19. A pathologist wants to pass dye to the liver he will reach thru: A. Ligamentum Teres. 20. Internal spermatic fascia is a continuation of: A. Transversalis fascia. 21. The branch of internal carotid artery which remains in true pelvis throughout its course: A. Middle rectal artery. 22. During a per rectal examination the examiner will not reveal: A. Ureter 23. Parasympathetic nerves supplying the urinary bladder are: A. Pelvic splanchnic nerves. 24. Nerve supply to the lower airways: A. T3-4 25. During an injury to the bulbar part of urethra the urine will extravasate to: A. Superficial perineal pouch. 26. Nerve supply to the distal scrotum is thru: Illioinguinal nerve. Illiohypogastric nerve. Genitofemoral nerve. 27. During surgery of right colon for carcinoma which structure would not be injured: A. Aorta. 28. Slow growing tumor of head of pancreas will compress: A. Common Bile Duct. 29. Tumor of the head of pancreas will compress: A. Common Bile Duct. EMBYROLOGY. 1. Regarding Decidua: A. Decidua basilis forms the maternal part of placenta. 2. Embryonic period is: A. 3-8 weeks. 3. Which of the following is a derivative of neuro ectoderm: A. Sphincter pupili muscle. 4. Derivative of Ectoderm include: A. Secretary Epithelium of parotid gland. 5. Derivative of Ectoderm include: A. Secretary Epithelium of parotid gland. 6. Regarding Allantois: …… 7. left umbilical vein will regress to: A. Ligamentum Teres. 8. Which of the following structure is not an Embryological remnant: A. Lateral umbilical ligament. 9. Regarding Mesothelium: A. Lines the body cavities. 10. Physiological hernia occurs in between: A. 6-10 weeks. 11. What will happen immediately after birth: Anatomical closure of Ductus arteriosus. Anatomical closure of Ductus venosus. Anatomical closure of foramen ovale. Obliteration of Left umbilical vein. Obliteration of Right umbilical vein. 12. Which of the following bone is derived from 2nd Pharyngeal arch: A. Styloid process. 13. Muscles innervated by hypoglossal nerves are derived from: A. Occipital Somites. 14. Primordial Germ cells are derived from: A. Yolk sac endoderm. NEUROANATOMY 1. In adults spinal cord ends at the intervertebral disc between: A. L1-2. 2. Injury to L1 vertebra will directly damage which of the following spinal structure: A. Conus Medullaris. 3. Which of the statement regarding Lateral Horn of spinal cord is inappropriate: A. It is present at the cervical portion of spinal cord. 4. Regarding Trapezoid Body: A. It is related to the auditory pathway. 5. Loss of Fine and skilled movements of the hand is due to damage of: A. Corticospinal Tract. 6. What is inappropriate regarding Glossopharyngeal nerve: A. It is entirely sensory. 7. Loss of temperature regulation is due to damage at: A. Anterior Hypothalamic Nucleus. 8. Temperature regulation centre is located in: A. Hypothalamus. 9. In spinal tap the needle which will go across: A. Epidural Space è Dura Matter è Sudural Space è Arachnoid Matter è Subarachanoid space. 10. Regarding spinal cord what is inappropriate: A. Its dura matter has two layers HISTOLOGY 1. Simple columner epithelium is present: A. Choroidal plexus of ventricles. 2. Cytoskeleton: A. Maintains the integrity of the cell. 3. Mast cell. ………. 4. Newly formed Elastic cartilage looks yellow & dense because of: A. Large no: of Elastin fibers. 5. Compact bone contains: A. Osteoblast cells in the lacuner spaces. 6. Which part of the respiratory tract contains mucous glands: A. Nasal cavity. 7. Skeletal muscles have: A. Multiple nuclei located at their periphery. 8. Surfaces which are prone to great amount of friction have: A. Straitifed Squamous Keratinizing Epithelium . PHYSIOLOGY 1. Pulmonary Wedge Pressure: A. Indirectly measures left atrial pressure, 2. Ventricular Depolarization on ECG strip is represented by: A. QRS Complex. 3. Ventricular preload is measured by: A. LVEDV. 4. Blood flow to the left ventricles is increase by: Acetylcholine infusion. Sympathetic stimulation 5. Bain Bridge reflux. …………… 6. IPSP is generated by opening of: A. Cl- channels. 7. During Depolarization: A. There is rapid influx of Na+. 8. Blood group antigen: A. Will not found in any other tissue than blood. B. Will be inherited by Autosomal recessive pattern 9. If father’s blood group is B+ve & mother’s blood group is AB+ve there child can not have which of the following blood group: A. O+ve. 10. ESR will increase with the decrease in: A. Albumin. 11. ADH will act on: A. Distal Tubules. 12. Renal absorption of glucose thru secondary transport with sodium occurs at: A. Proximal Tubules. 12. Simple Diffusion depends on all of the following factors except: A. Magnetic field. 13. What is the suitable I/V fluid for the patient of acidurea: Normal Saline. Dorrow`s solution. Ringer lactate. 5% dextrose 10% dextrose. 14. Which of the following causes Hyperkelemia: Zollinger Ellison syndrome. Cushing syndrome. Conn`s syndrome. Pyloric stenosis. 15. Which of the following hormone is called STRESS HORMONE: A. Cortisol. 16. Inhibitory factor released by hypothalamus against which of the following hormone: Prolactin Growth hormone. 17. Bile of the Liver differs from the GB bile because GB bile contains decrease amount of: A. Water. 18. Which of the following cell can never reproduce: Erythrocyte. Neuron. Skeletal muscle. Smooth muscle. 19. Which of the following is the NOT non dividing cell: A. Hepatocytes. 20. Heat loss mainly depends on: Temperature of the surrounding. Evaporation. 21. Cortisol decreases which of the following cell: A. Lymphocyte. 22. Resection of distal ileum will impair the absorption of: A. Bile Salts. 23. Regarding Bradykinin: It formation is activated by killkeran. 24. Regarding Interferon: ………. 25. Surfactant is released by: A. Type II pneumocytes. 26. Increase peripheral resistance is due to: A. Increase vasomotor tone. 27. Chloride shift means: A. Transfer of Cl- in erythrocytes in exchange of HCO3. 28. Cardiac muscles are prevented by tetanization due to its: Rythmicity Automaticity Conductivity Long refractory period. 29. Sustained rhythmic reflex tremors induce by sudden movements is: A. Clonus. 30. Most active form of thyroid hormone present in circulation is: A. T3. 31. Regarding conduction of visual pathways: ……….. 32. Regarding olfaction: A. Sharp odors have the quality of water & lipid solubility. 33. Unilateral Anosmia is due to: A. Affection of Nasal mucosa. GENERAL PATHOLOGY 1. Irreversible cell injury starts with: A. Rupture of the Lysosomal membrane. 2. Which of the following is not a mediator of acute inflammation: A. Dopamin. 3. ICAM & VCAM mediates: A. Leukocyte Adhesion. 4. Fever in inflammation is caused by: A. IL1 & TNF α. 5. A 36 years old women presented with acute abdomen. At laparoscopy most of the bowl loops were dark purple black. Her mesenteric veins were patent. The most probable underlying pathological process is: A. Wet Gangrene. 6. Superimposed infection on necrosis is called: A. Gangrenous Necrosis. 7. Fluid accumulates in acute inflammation contains: A. Proteins >3gm/dl. 8. Fat necrosis occurs in: A. Acute pancreatitis. 9. What is inappropriate about fat necrosis: A. Only caused by trauma to the fat tissue. 10. A 25 year old lady with 14 weeks pregnancy had a road traffic accident brought to the emergency department with a large open wound on thigh and femur fracture. After 2 days of accident she suddenly collapsed & died. The most probable pathological process involve in her death: A. Fat Embolism. 11. Regarding Fat Embolism: It is Fetal in >80% of cases. IT manifest within 12 hours. 12. A 30 year old women brought to emergency department with bleeding….. 13. A 16 years old boy deeply jaundiced presented with gum bleeding due to: A. Vitamin K deficiency. 14. A patient presented with enlarge lymph nodes, the diagnosis of Tuberculosis in this patient is confirmed by: A. Presence of caseous necrosis in lymph nodes. 15. An 18 year old girl came to family doctor complaining of primary amenorrhea, on examination she found to have well formed breasts, blind ended vagina with no uterus & ovary. Her probable Karyotype is: A. 46 XY. 16. Edema due to increase hydrostatic pressure is seen in: A. Congestive cardiac failure. 17. Which of the following is a benign tumor: A. Warthin`s tumor. 18. Which of the following is a premalignant condition: A. Leukoplakia. B. Erythroplakia. 19. Which of the following is a premalignant condition: Condyloma Metaplasia Endocervix. Cystic hyperplastic endometrium 20. Metaplasia does not occur in: Brain. Vitamin A deficiency. 21. The primary source of Creatinin is: Liver. RBC. Skeletal muscles. Cardiac muscles. Lungs. 22. Dystrophic Calcification is seen in all of the following except: Malarial parasite. Dead fetus. Hydatid Cyst. 23. Maximal tensile Strength of a wound is attained in. 3 Months. Over an unpredictable period of time. 1 Year. 6 days. 10 days. 24. Regarding autosomal recessive disorders: ……………… 25. Which of the following are not non-dividing cells. A. Hepatocytes. 26. In a patient there is atrophy of submandibular gland due to its duct obstruction. Atrophy of the gland is because of: A. Apoptosis. 27. The mechanism of injury of ionization radiation is: A. Free radical formation. 28. LASER act by: Cutting. Biochemical changes 29. A patient admitted to the hosp. diagnosed as case of gas gangrene dies due to: A. Toxic shock 30. Which of the following is the confirmatory test of AIDS: Western blot. ELISA. SPECIAL PATHOLOGY 1. Hepatic hemengioma is associated with: A. Vinyl Chloride. 2. Which of the disease is common in I/V drug abusers: A. Infective Endocarditis. 3. Typhoid carriers are: A. Usually asymptomatic. 4. Intestinal metaplasia due to reflux esophagitis leads to: A. Adenocarcinoma. 5. Embryological marker that reappears in circulation in Ca Colon is: A. CEA. 6. Which of the following is peculiar for crohn`s disease is: A. Perianal Lesions. 7. Anemia of pregnancy is due to: A. Increase plasma volume. 8. Major complication of severe burn: Formation of granulation tissue. Carcinoma formation. 9. What is Inappropriate about hyper parathyroidism: A. There is increase PO4 renal absorption. 10. A 25 year old lady complaining of palpitations, heat intolerance on examination her pulse is 112b/min, BP 120/80, R/R 20 br/min. She is most probably a case of: A. Hyperthyroidism. 11. Resection of Anterior lobe of pituitary will lead to: A. Decrease glucocorticoids. 12. A 6 year old boy is complaining of proximal muscular weakness was found to be Ca++ channel antibodies positive. The most probable diagnosis is: Lambert Eaten Syndrome. Mysthenia Gravis. 13. In pulmonary embolism, respiratory failure is due to: A. Ventilation / perfusion mismatch. 14. Left ventricular failure will lead to: A. Increase pulmonary arteriolar Pressure. 15. Immediate effect after injury to a vessel is: Vasoconstriction 16. Effects of hyper parathyroidism in Ca. lung is due to: Parathyroid like protein Parathyroid hormone 17. Spleenectomy will help in treating which type of anemia: A. Hereditary Spherocytosis. 18. Malignant Tumor that will not metastasize: A. Basal cell carcinoma. 19. Achlasia is due to: A. Absence of Myenteric plexus. 20. After a traffic accident a lady is brought to a hosp the IMMEDIATE step you do: A. Clear airway. 21. Ameobic infection reaches lungs via: Direct extension from liver. By aspiration of ova of Entameoba histolitica. By aspiration of trophozoits of Entameoba histolitica. Via Portal vein. 22. In Hypothyroidism there is increase in: A. Cholesterol 23. In thyroid disease the antibodies are directed against: A. Thyroglobin. 24. A 50 years old man is awakened in the middle of the night with acute pain & tenderness of the right knee. He has a low grade fever. His knee is hot, tender & swollen. Analysis of fluid from right knee shows: leukocytes 70,000/mm3 with 75% neutrophils, crystal analysis = negative birefringent, gram stain = negative. The most likely diagnosis is: A. Gout. B. Pseudo gout. C. Septic arthritis. PHARMACOLOGY 1. Which of the following blocks α and β receptors: A. Labetalol. 2. Ketamin is used as anesthetic in repeated dressings of burn patient because: A. It relieves pain as well. 3. Drug that decreases the tone of lower esophageal sphincter & increases gastric emptying: A. Metaclopromide. 4. Which of the following is suitable antihypertensive for asthma & IHD patient during surgery: I/V sodium nitroprusside. I/V nitroglycerin. 5. Which of the following is drug of choice for patient of status asthematicus: I/V aminophylin Oral steroid I/V salbutamol. 6. During surgery antibiotics should be given at: A. At the time of induction of anesthesia. 7. Heparin will inhibit: A. Clot propagation. B. Clot organization. 8. Acid suppression is done by blocking: A. H2 Receptor. 9. Drug of choice for acute pancreatitis. Pethidine. Morphine. Paracetamol 1 trapezious body relating to auditory pathway MLF dorsal column-medial leminiscus laterl column 2 which one of these cells is not a non dividing cell neurons nerve cells myocardiocytes hepatocytes 3 papillary muscle derived from ectoderm endoderm mesoderm neuroectoderm 4 LASERworksby cutting thermal crystalization 5 which of the followings is the benign tumor wilms tumor mesothlioma hamartoma 6 internal cremas tari cfas ci a derviedfrom fascia tranversalis fascia internal oblique fascia tranversus abdominis 7 Regarding bone regular lamellae irreagular lamellae horizontal c a n a l s oblique canal s 8 A D H works at distal c o n v u l a t e d tubule collecting tubule distal c o n v u l a t e d n collecting ttubule proximal convulated tubule 9 commonly rib fractures at angle neck shaft body 10 right umblical artery b e c o m e s left medial umblical l i g a m e n t right median umblical ligament left medial umblical l i g a m e n t median umblical ligament 1) What structure is located superio-posterior to pituatory fossa Mastoid air cells Ethmoid sinus Sphenoid sinus 2) Which Lung function Test is diagnostic of Asthma? FEV1 FVC FRC RLC 3) Bee sting, shock like condition of a child, flushed, what mechanism is involved Type 1 reaction Type 2 Etc 4) What sensory receptors sense VIBRATION? Pacinion corpuscles Ruffini Etc 5) Cause of anemia in Pregnancy Fall in Hb Decreased heme Volume Expansion. 6) Cut to the postero lateral area of FOREARM near the head of Radius, a nerve is cut, cant appose thumb, no sensory loss, difficulty (I guess extending) thumb, injury to Radial Median Ulner 7) Structure not passing through FLEXOR RETINACULUM of forearm Ulnar nerve Median Nerve Flexor policis longus Etc 8)Cause of SQUAMOUS METAPLASIA in a 38yrs old lady Multiparity HSV infection IUCD 9) Mild CCF symptoms, Monotherapy to be prescribed ACE-I Thiazide Frusimide Ca Channel blockers 10)Pregnancy Induced Hypertension, Rx? Methyldopa 11) Side Effects of ACE inhibitors Hyperkalemia HypoKalemia Hypercalemia Hypocalcemia 12)BARRETT’S Esophagus, whats the cause? Loss of Myenteric nerve plexus Failure of relaxation of LES 13) Which is not a branch of VAGUS Lacrimal Nerve 14) Volume of Distribution, Defination? 15) First pass Hepatic Metabolism, Basic concept. 16) Branches of BASILAR Artery 17) Vertebral Artery Branches. Like, which is a branch of it. (Circle of Willis was a hot thing this time, more than 3 Q) 18)Which of these form Circle of Willis Post communicating artery Vertebral A Basilar A 19) RENIN ANGIOTENSIN System Mechanism of Aldosterone release, the basic concept. 20)Body concerves body water by ADH release Cortisol release Aldosterone 21) METAPLASIA, Defination. 22) HYPERPLASIA Definition. 23) VOLUME DEPLETION, Which mechanism is activated Renin angio aldo system ADH Etc 24) Which drug has a n ATROPINE LIKE ACTION Physostigmine Scopolamine 25)Old obese Diabetic, newly diagnosed, choice of Rx? Biguanides Biguanides plus sulphonylurea Insulin 26) BARRETT ESOPHAGUS can lead to: Adenocarcinoma esophagus Adenocarcinoma Stomach Sq carcinoma Esophagus Stricture 27) Location of PSEUDO STRATIFIED SQUAMOUS Epithelium 28) STRATIFIED SQUAMOUS Epithelium, characters 29) Pseudo Stratified COLUMNER CILIATED Epithelium, Location? 30) Histological section of a lymphoid tissue, a cortex and a medulla seen, and some ?ducts visible, Identify. 31) Which cells cannot PHAGOCYTOSE Kuffners cells Mast cells Monocytes Polymorphs 32) SENSORY RECEPTERS in Joints 33) Complement system is activated by the interaction of ?some factor with A (cant remember the options) BC 33) Most Important use of ECF Excretion of CO2 from body Electrolyte exchange Cell nutrition 34) Reasons for susceptibility to infections in AIDS Decrease in CD4 count CD8 35) HAIRY LEUKOPLAKIA is seen in Fungal infection HIV Etc 36) Following PARTIAL HEPATECTOMY, remaining part can regenerate in 10 days 15-25 days 5 weeks 7 weeks 37) Which drug binds to COX-2 recepters and not to COX-1 Aspirin Indomethaci n Piroxicam Meloxicam Ibuprofen 38) What is transmitted through corpus callosum. 39) What is the result of a lesion at the OPTIC CHIASMA 40) What structure is Not related to OPTIC TRACT? Lateral Geniculate body Medial ~ 41) Area of brain affected in BITEMPORAL HEMIANOPIA 42) CSF pressure is increased when what structure is blocked : Internal Jugular vein Common carotid 43)Functions of CEREBELLUM 44) Venous drainage of the HEART Anterior cardiac sinus Inferior vena cava Cardiac vein 45) RIGHT HEART BORDER is formed by RA RA+SVC RA+RV 46) CIRCUMFLEX ARTERY supplies LA LV Apex LA L auricle 47) Whats correct regarding HEART? Atria and Ventricles contract simultaneously. Contraction starts from Left apex Left Atria is 3 times larger than Right atria. Left atria is 1st to contract Excitation passes from Atria to Ventricles directly. 48) What structures are present in the free border of the LESSER SAC Cause of abdominal angina, occlusion of: Superior Mesenteric A Inferior Mesenteric A 49) Lymph Nodes involved in Ca CERVIX are? 50) Ca Cervix is caused by: IUCD HSV HIV 51) Sympathetic stimulationof Beta recepters causes Bronchial dilatation 52) Mix lesion of loss of pain and Temperature of one side and Proprioception of the other side, whats the cause/where is the lesion. 53) In Cervical part of spinal cord, whats deficient? Grey matter White matter Anterior Horn Lateral horm Posterior horn. 54) In REM sleep, the following happens: Dreams Sleep Walking. 55) Lesion in the Broca’s area: Can’t write Can’t speak except in few words 56) Surfactant: Decreases compliance Decreases when alveolar size decreased Has a low turnover rate 57) An athlete at rest: Increased Stroke rate Increased cardiac output Increased Heart rate 58) H-K-ATPase pump is blocked by Omeprazole Cimetidine Bismuth 59) Murmur of MS best heard at Apex Tricuspid area Etc 60) In Hypertension, Renin release is mediated by Renal artery dilation Na in distal limb loop of Henle 61) Vasodilation is mediated by Histamine 62) Tidal Volume is a part of which capacity 63) Pneumoconiosis, whats the differenciating lesion Fibrosis Pleural plaques 64) Mesothelioma: Pleural Plaques 65)Asthma, which investigation is diagnostic: (BRS-Physio Page 136 to confirm) Decreased Fev1 Decreased FRC 66) Thellasemia, Where is the defect – in Heme or Globin 67) CCF, raised Ph, carbon dioxide 44mmhg, Hco3 raised. Whats the metabolic disorder. 68) Histology of nephron, which part is in the medulla Loop of Henle Collecting duct 69) Infusion of I/V dextrose, how will body respond Decrease in insulin release Increase in Glucagon Cortisol release Somatostatin release 70) Bone lesion in the elderly, bone matrix is being eroded, which hormone is responsible 71) CRF, Hypertension, which of these changes CANNOT happen: Raised Potassium Decreased Calcium Decreased Hb Decreased Po4 72) Release of ADH happens under what circumstances, (Basic concept) 73) Acid is released in stomach when Proteins are ingested Fat Carbohydrate 74) Crohn’s disease, differenciating lesion from UC: Perianal disease Etc 75) Appendicitis, on lab test major finding : Raised TLC Water levels in R Iliac fossa 76) Polypeptide hormone: Growth Hormone And 4 steriod hormones mentioned 77) Protein synthesis in secretory cells: RER SER Golgi A 78) Peroxisomes release: Hydrolysing enzymes 79) Hormone/s responsible for BREAST ATROPHY E P E+P 80) Dumb Question>> Which Hormone is responsible for respiration during the leutinisation phase FSH LH E (Do leave a comment if you think this Q makes any sense) P LH 81) Hashimoto’s Thyroiditis (Scenario given), whats the cause? 82) Pheocromocytoma, whats the treatment? Alpha blockers Beta blockers Both Alpha+ Beta. 83) Hypertension, Increased Na, Decreased K, whats the reason Ans: Hyperaldosteronism 84) Non Anionic Gap Metabolic Acidosis is seen in 85) Effects of Insulin on body fat deposits/metabolism 86) Gametogenesis 87) Modes of inheritance, ( 2+ questions were there) Like which is what. Autosomal Dominant was asked 88) Autosomal Recessive disorders are: 89) Wheat contains which vitamin/mineral 90) Child having diarrhea, has been on glutin free diet for 8-10 days. Whats the diagnosis Shigella Whipples Celiac disease 91) Young girl with emlarded clitoris Androgenital syndrome Down Etc 92) An old man hit by car bumper> #fibula, which nerve is affected. Ans: common paroneal 93) Knee fully extended, in a locked-in position, which nerve is affected. 94) Right foot in the air, Left pelvis sinks down, which nerve is affected? L Superior Gluteal Nerve (correct) R Superior Glut L Inferior Glut R Inferior Glut 95) About Ca Colon, Right sided present early Cause is increased fat intake, low fibre (correct) 96) Man, previously had a disorder in U bladder, now came for follow up and is found to have Ca, whats the cause: Nitrosamine Schistosoma 97) Lateral quadrant of breast drains into which group of Lymph node. 98) CA anterior 2/3 tongue, which group of lymph nodes are involved: L anterior superior submandibular R L Post Sup submandibular R post sup submandibular. 99) Chorda Tympani, course. Option: joins lingual nerve when exits skull. 100) About Lacrimal gland, Is located in lacrimal fossa on the lateral side Both glands drained by single duct. 101) Lacrimal duct opens at: Superior Inf Middle Spheno ethmoidal recess 102) In pure motor stroke, which area is involved 103) Functions of Corpus callosum, Globus pallidus, Putamen. 104) A patient on a psychiatric drug, side effects mentioned, which drug is responsible. Clorpromazine SSRI Etc 105) A man, around 35 brought to emergency, sweating, salivation, Heart rate 45, BP 60/40, whats the treatment. Atropine (correct) (Organo phosphate poisoning.) Lignocaine Digoxin 106) This area is normally resonant to percussion 2-4 Intercostal space on Right 2-4 Intercostal space on Left 5-6 I/c space on Right 5-6 I/c space on 107) Boy, age 10, history of trauma to chin 2 years ago presents with inability to open mouth fully, cause? Sub mucous fibrosis Misplaced Injection Ankylosis of TM joint The Q's below are courtesy of 'Dr Daffodil'. 108) Vitamin raised in disturbed homocystine metabolisma.B1.b.B6c.B12 109)Vit B 6 involve ina.oxidationb.reductionc.Carboxylation 110) End product of Purinea.uric acidb.urea 111) Miscellesa. contains more Triglyseridesb.absorb fats in duodenum 112) Chemotherapy causesa.degenerationb.apoptosis 113) Corpus callosum containsa.commisural fibersassociation fibers 114) Aphasia a.paralysis of muscles of palate/toungueb.due to damage of frontal cortex in dominant hemisphere 115) Tabes dorsalis causes bladder incontinence due to a.Atonic bladderb.automaticc.neuropathic 116) Dopamine regulates a.Prolactineb.Somatostatin 117) Neurotransmitter related to emotionsa.ACHb.Norepinephrinc.glutamted.Dopamine 118) Proencephalin is abnduntly found ina.Basal ?Gangliab.Cerebellumc.Thalmus 119) Regarding pul. circulationa. resistance is (dont remember % or ratio) less than systemic circulation 120) During Ventricular ejectiona. min pressure difference b/w Lt. Vent. and Aorta 121)Facial nerve/chorda tympani?( dont exactly remember)a. supply mucus membrane of int. earb.gives secretomotor fibers to Parotid gland 122)Disease of cervix inolves lymh nodesa.Int. illiacb.Int. n ext. illiacc.depp inguinald.Para aortic 123) prostate CA which lymph nodes be involveda.Int.illiacb.Para aorticc.Deep Inguinal 124) Radiotherapy effect the most a.ovaryb. Ut. tubesc.uterusd.cervix 125)CSF showing raised neutro,proteins ,low glucose .which drug to givea.ceftriaxoneb.rifampicin 126) Insensible lossoesa.200-400b.500-600c.800-1200127) Pri. opsonin in classic pathwaya.C5ab.C3ac.IgGd.C3b Posted by Dr Sultan Ahmad 3 comments Reactions: Surgery Q's July 2007 FCPS-1 1.mesothelioma is .present in all da body cavities .lined by simple cuboidal epi 2.surfactant is produces by .kidney .liver. .pneumocytes1 .pneumocytes2 3.allantois is .first formed blood vessels(sumthin like dat) .remains of vitelline duct(sumthin like dat) 4.activation of one of followin occurs in kidney .cholecalciferol .renin 5.a pt cums 2 u wid loss of abduction of right arm upto 30degrees....but ven supported upto 30 drgrees abduction is normal,which muscle is involved? .deltoid .supraspinatuis .infraspinatus .teres major 6.how many valves r present in great saphenous vein? .5 .10 .15 .20 .25 7.the ca head of pancreas may compress .common bileduct .duodenal artery .spenic artery 8.wot structure arches over left bronchus .aortic arch .azygous vein 9.the central regulation of temperature occurs in .hypothalamus .pons .medulla .cerebellum 10.there was another stem regarding da temperature regulation but it was asked peripheral temp regulation...n one of da option was skin in dat plus anterior hypothamus...i dun remember da xact stem! 11.fine voluntary skilled movements of upper xtremities r conducted by .cerebellum .medulla .pons .upper motor neuron .cortico spinal tract 12.there was another stem regardin fracture of bone n its complication...i dun xactly remember da clinical correlated stem...but one of da option was FAT EMBOLISM 13.which sensory part is involved in da flexion of knee joint? (sumthin like dat) .s1-s2 .s3-s4 .s4-s5 14.a pt is unable 2 flex metacarpels,inability 2 abduct fingers, and unable 2 flex wrist joint(i think)...which nerve is damaged? .ulnar nerve above da elbow joint .ulnar nerve below da elbow joint .radial nerve .median nerve 15.an unaffected stage in amoeba .amoebic cyst .amoebic larva .adult amoeba (sumthin lyk dat) 1.neutro penia and hepatosleenomagaly is present in . ducts of salivary glands 19.keratinized sqamous epi .stratified squamous epi .ligamentum venosus .non keratinzed sq epi .all those structures in contact wid air hav da following epi .medial longitudnal lig 17. If Carcinoma involves nipple of breast which lymph nodes r likely to involve first? a) Pectoral b) Apical c) Central d) inernal mammary .which structure has no embryonic bacground? .16.median longitudnal lig .falciparum malaria .pseudo startified sq epi 18.ligamentum teres . Eversion of cervix during pregnancy occur in response to: a) Estrogen b) Physiological change c) Ca Cervix d) Gonadotropins e) Cervicitis 2.anemia.simple cuboidal epi is present in .chagas disease Posted by Dr Sultan Ahmad 0 comments Reactions: Gynae Obs Paper July 2007 Part-1 Questions sent by Dr Samra. came in OPD With enlarged Ant. b) Femur length at 28th wk.on examination she was having short blind vagina wid normal vulva.absent uterus.Axillary lymph nodes and diagonosed a case of Ca Breast .her karyotype is? a) 46XY b) 47XY c) 47XXY 7.3. c) Biparietal diameter at 18th wk.short stature diagnosed as Turner syndrome. A pt came to you with primary amenorrhoea.normal breasts. reapeated again 5. which region is most likely involved? a) Upper Outer Quadrant b) Lower Inner Quadrant 4. scanty pubic n axillary hairs.Sperms life span in female genital tract is ? a) 24 to 48 hrs b) 24 to 72 hrs . d) Abdominal circumference at 8. Wht is the best way to detect exact gestational age of fetus? a) CRL at 8 wk. wht is her karyotype? a) 45XO b) 46XY c) 47XXY 6. A normal looking Girl came to you with primary amenorrhea. More or less same question as # 3. A Pt.webbed neck. In a motorbike accident neck of fibula fractured and lead to a loss of dorsiflexion n eversion of foot. Wht is most likely to b true about pudendal nerve? a) Enter the ischiorectal fossa thru its lateral wall b) Leave the pelvis thru upper part of greater schiatic notch c) Re enter the pelvis thru lesser schiatic notch .which nerve is likely to b damaged? a)Common Peroneal b)Deep peroneal c)Superficial Peroneal 14. In drug receptor relations which one is incorrect? a) Bradycardia --.Tubocurarine d) Bronchodilation --. a) b) c) d) Spermatogenesis is controlled by? Testosterone FSH+ Testosterone LH Estrogen 10.Propranolol b) Miosis --.Pilocarpine c) Skeletal Muscle Paralysis --. Which one of them do not pass through Lesser Sciatic Foramen? a)Tendon of obturator internus. b)Pudendal Nerve c)Internal Pudendal Vessels d)sup. & inf. Almost same as above 1 1 .9. gluteal vessels 13.Salbutamol 12. If Ca.Ureter is found at : a) the bifurcation of common iliac artery b) pelvic brim 17. uterine artery crosses the: a)Ureter b)Fallopian Tube c)Ligament of ovary 19. If perineal body is cut which muscle likely to b damaged? a) Bulbospongiosus + superficial Transverse Perieal b) Deep + Superficial Transverse Perineal 16. If ovaries are removed frm ovarian fossa. wht is most likely to b damaged? a) Internal iliac b) uterine artery c)Ovarian artery d) External iliac artery 18. Of uterus b) pubocervical ligament c) pectineal ligament 20)which structure do not participate in forming the walls of inguinal canal? a) Transversalis fascia b) Internal Oblique c) External Oblique . To reach the uterus.Cervix metastasize to Labia majora which one of these is most likely responsible for spread? a) Round lig.15. Hct 20 with same peripheral film wht is the cause? a) Physiological change b) Iron deficiency c) Folate Deficiency d) Vit B12 deficiency 24) Ovaries are attached to broad ligament by? a) Mesoovarium b) Mesosalpinx c) suspensory ligament 25) In females Neck of bladder ends : a) above the urogenital diaphragm b) below the urogenital diaphragm c) .d) Conjoint tendon e) Lacunar ligament 21) If after 1 hr of ceaserian section pt start b bleeding internally which one is most likely damaged? a) Uterine artery b) Internal iliac a. c) Ovarian artery d) uterine vein 22) If on lying down pt became dyspnoec wht is the cause? a) Retrosternal goiter b) Multinodular goiter c) 23) If a women at 10th week pregnancy hav Hb 12.9 .Hct 25 Normocytic Normochromic film but at 32 wk pregnancy her Hb is 10. 26) Most common cause of hosp. acquired infection is? a) Indwelling catheters b) I/V lines c) 27) Which method is used 2 kill spores of surgical instruments? a)dry heat at 160c for 1 hr b)dry heat at 160 c for 4 hr c)moist heat at 160c for1hr d)Gension violet for 4 hrs 28) Which one is not used as insecticidal? a) Derivatives of salicylic acid b) Phenol c) Sodium hypochlorite d) Soap 29) Most common cause of subacute bacterial endocarditis is: a) sterpt viridans b) sterpt epidermidis c) staph aureus 30) Food poisoning cause by staph. Is mainly due to: a) exotoxin b) endotoxin c) enterotoxin 31) Now a days the best way to diagnose viral infection is: a) ELISA b) by detecting virus from secretion or wound c) immunofloresence . 32) If pt hav persistent pyuria all can be the cause except: a) polycystic kidney disease b) long standing alkaline urine at room temperature c) Post Streptoccocus glomerulonephritis 33) wht is the least positive value for widal to diagnose typhoid? a) 1:80 b) 1:120 c)1:360 34) Two days before ovulation LH surge rises to a) 1 to 5 fold b) 4 to 6 fold c) 6 to 10 fold 35) villous surrounded by cyto n syncitiotrophoblast is: a) Primary villous b) secondary villous c) tertiary villous 36)Following is not a proven cause of hirsuitism? a) Androgen b) PCOs c) steven leventhal sundrome d) 37) CO2 is transported frm blood to alveoli by : a) Facilitated diffusion b) Active transport c) Diffusion 38) Active transport differs frm Passive due to: a) depend on osmosis . b) require carriers but no energy c) requires energy but no carriers d) require energy by phosphate anhydride bond 39) Mean Arterial pressure is a) difference b/w systolic and diastolic pressure b) Diastolic + 1/3 pulse pressure c) 40) Columnar epithelium is found i n : a) uterine tubes b) Thymus c) ovaries 41) Obese are at risk of all except: a) Hyepertension b) Diabetes c) Thyrotoxicosis d) IHD 42) Following will lead to Respiratory Acidosis? a) emphysema? b) heavy intake of NaHCO3 c) Vomiting 43) Hormone which causes an increase carbohydrate protein and lipid stores is? a) GH b) insulin c) cortisol d) Aldosterone e) thyroxine 44) same question . 45) insulin secretion is inhibited by: a) glucagon b) secretin c) 46) Acellular layer surrounding an ovum is : a) Zona pellucida b)Granulosa layer c) Theca interna d) Corona radiate 47) Primordial germ cells arises frm: a) Yolk sac 48) Highest energy containing compound is: a) Starch b) Glucose c)Hexose phosphate d) ATP 49) Which contain high triglyceride? a) Chylomicron remnants b) LDL c) VLDL d) HDL e) IDL 50) Which is not a phospholipid? a) Lecithin b) ISolecithin c) Cephalin d) Plasmalogen . LH c) Testosterone 56) Which hormone convert to its active form at target tissue with the help of 5 α reductase? a) Testosterone b) Estrogen 57) Pt came to you with eosinopenia.wht is most likely? a) Cushing disease b) Cushing Syndrome .polycythemia.51)Low PO2 Stimulate respiratory centre thru: a) Directly acting on medulla b) stimulating peripheral chemoreceptor c) acting on central chemoreceptor 52) If Rh -ve mother deliver an O +ve fetus wht is the best way to protect subsequent pregnancies? a) Immunoglobulin 53) wht is the landmark for biscop scoring: a) Ishial Spine b) Ishial Tuberosity 54) In a women of craniopharyngioma wht is the best opetion to induce ovulation? a) Clomiphen citrate b) Menopausal Gonadotropins 55) If you give L Dopa to the pt of Parkinsonism which hormone will decrease? a) Prolactin b) FSH.lymphocytopenia and raised ACTH level. wht is the most likely cause? a) Vesicovaginal Fistula b) Vesicouterine Fistula 64) Which one is the supporting ligament of uterus tht is . developed Urinary incontinence. is : a) Optimize the platelet function b) platelet number do not dec. cortsiol level? a) Hypotension b) Moon face c) Buffalo Torso d) Thin skin e) Hirsuitism 59) Main support of uterus is: a) Cardinal ligament 60) Rational for storage of platelet at room temp.c) Addisons disease 58)Wht change will not seen in a women having inc. 61) Women brought to you in ER in shock who had a 10 pint transfusion 10 yrs back due to PPH is most likely having: a) Sheehan syndrome 62) Therapeutic index indicate: a) Bioavailability b) Efficacy c) 63) If after 15 days of c-section due to Obstructed Labour pt. Photosensitivity.palpable on Per-rectal examination? a) Uterosacral ligament 65) If 35 yr old women developed Generalized Lymhadenopathy wht is the most likely cause: a) Hodgkin Lymphoma b) Nonhodgkin lymphoma c) Malignant disease d) CML e) ALL 66) If women accidently exposed to radiation 3 to 4 yrs back which organ is most likely effected? a) Cervix b) Fallopian tube c) Ovaries d) Vagina 67) Wht is the best method to check bone density? a) CT scan spine b) MRI spine c) Calcium Level d) D scan of spine 68) Which drug do not cause gynaecomasia? a) Digoxin b) Griseofulvin c) Cimetidine d) Androgen e) Estrogen 69) Pt developed Morning stiffness.rash which test will be positive? a) RA . T4 raised in pregnancy due to: a) Inc TBG b) Inc TSH c) Inc TRH 76) Which one is not a tumor marker? .b) ANA b) antimitochondrial 70) Wht is the end product of glucose breakdown ? a) Galactose b) Lactose c) Fructose d) Pyruvate 71) In a pre eclamptic pt which drug is used to lower BP before surgery? a) Hydralazine b) Amlodipine 72) Amniotic fluid embolism is related to: a) DIC 73) Which hormone is necessary for brain development of fetus? a) GH b) Thyroid c) Cortisol 74) If α feto protein is high wht is most likely? a) Anencephaly b) Spina bifida Occulta c) Down syndrome 75) T3 . a) b) c) d) e) HCG CEA α fetoprotein Placental Acid phosphatase(PLAP) Alkaline Phosphatase 77) Hysterosalpingography is more accurate than MRI i n : a) Adenomyosis b) blocked tubes 78) TORCH can lead to all EXCEPT: a) Pnuemonia b) Cataract c) Deafness d) Cardiac anomalies e) Blindness 79) DNA is: a) Double stranded in which two strands are antiparallel to each other b) all other wrong 80) Which antithyroid drug is given during Pregnancy? a) Propylthiouracil 81) Tx of raised sugar in pregnancy is : a) Insulin b) Sulfonylurea 82) Menopause is diagnosed by: a)Raised FSH.Raised LH.dec Estrogen b)dec FSH n LH 83) Tuboovarian abcess caused by IUCDs…wht is the most . likely causative agent? a) Actinomycosis b) Streptococcus 84) Which component is low in Cow`s milk as compared to mother`s milk? a) Lactose b) Fat c) Calcium d) Protein 85) Excess of Aldosterone will cause: a) Hypokalemia b) Hyperkalemia 86) Which method is most useful in identifying chromosomal abnormalities? a) Karyotyping b) U/S c) Bar bodies 87) DIC is related to all EXCEPT: a) Cancer b) Amniotic fluid embolism c) Sepsis d) RPOCs e) answer 88) In C-section insicion is given in subcutaneous fat suprapubically will likely to damage? a) Superior mesenteric artery b) Deep Circumflex iliac artery c) Superior epigastric artery d) inferior epigastric artery . e) Perforating arteries 89) Insulin increase absorbtion of glucose in? a) Hepatocytes b) Mucosa of small intestine c) Renal tubular cells 90) Glucose is absorbed in renal tubules through: a) Secondary Active transport b) Facilitated diffusion c) Diffusion 91) A pregnant women is on Phenytoin.wht is the least likely complication in fetus? a) Bone deformity b) Mental retardation c) Cleft palate 92) Anticoagulants are indicated in all EXCEPT: a) Thrombocytopenia b) Cerebral infarction c) Prolong bed rest d) DVT 93) A mother brought her 8 yr old daughter in OPD with complain of vaginal bleeding.she was taller acc.wht is the cause: a) Ovarian tumor b) Hyperpituitrism c) Hypothalamic disorder 94) Main function of Extracellular fluid is: . To her age with well developed breasts.she was otherwise normal. Rabeeka Khan (Expect repetition of questions) 1)which hormone is resposible 4 storage of carbohydrate.protein in body? a)cortisol b)growth hor c)thyroxine 2)which hormone is responsible 4 storage of fat CHO n protein in body n called hormone of abundance? a)costisol b)throxine c)grwoth hormone 3)which hormone is needed 4 brain development of fetous during intranatal life? a)growth horm .a) Transport electrolyte b) Remove waste products c) Provide nutrition 95) Which investigation is least likely done in a suspected case of DIC? a) Clotting time b) FDP c) D-Dimer d) Platelet count e) PT 96) Which drug is not used in Dysmenorrhea? a) Ibuprofen b) Mefenamic acid c) Aspirin d) COX inhibitor Questions sent By Dr.fats. which is most likely 2 show onset of menopause? a)anovulatory cycle b)atrophic uterus .b)thyroid etc 4)tubo ovarian abcess in a woman using iucd is caued by? a)actinomycosis b)streptococi etc 5)which component is low in cow milk as compare 2 mothr milk? a)lactose b)protien c)calcium d)fat 6)among following which is not antiseptic? a)soap b)salicylic acid c)sodium hypochlorite d)ethanol 7)4 tx of eclampsia which drug is used? a)hydralazine b)minoxidil etc 8)which method is used 2 kill spore? a)dry heat at 160c for 1 hr b)dry heat at 160 c for 4 hr c)moist heat at 160c for1hr 9)excess of aldosteron will cause? a)hypokalemia b)hyperkalemia etc 10)structure of 2ndry villus unclude? a)mesoderm cytotrophoblast syncytium b)capillaries cytotrophoblast syncytium etc 11)a 45 yrs old lady is post menopausal from last 9 months. her wt n ht for age were normal bt absence of uterus.etc 12)spermatogenesis is supported by? a)FSH n testosteron b)gonadotrophin etc 13)germinal epithelium of ovary contain? a)cuboidal epithelium c)columnar etc 14)columnar epithelium line which structure? a)thymus b)choroid plexus c)uterine tubes 15)which method is most helpful in identifying chromosomal abnormalities? a)karyotyping b)u\s etc 16)a normallooking female cam 4 primary amenorrhea on examination she was having short blind vagina wid normal vulva n scanty hair.she was othrwise a normal lookinggirl karyotype was xy a)46xy b)46xx etc 17)DIC is related 2 all of the following except? a)amniotic fluid embolism b)cancer c)gram-ve sepsis d)retained dead fetus e)etc(answer) 18)a women brought 2 hospital after having difficult delivery at home wid bleeding diathesis wat is the cause a)dic etc etc 19)which of the following is leat likely 2 b effected by actinomycosis . a)bone b)brain c)oral cavity 19)which is not the manifestation of child born wid toxoplasma infection? a)pneumonia b)cataract c)mental retardation etc 20)a woman having morning stiffness of hand n pericardial effusion which lab finding woud help in dx? a)anti nuclear DNA b)strptolysin c)rheumatic factor etc 21)which structure pass thro inguinal canal from uterus 2 labium majus in female? a)round ligament b)broad ligament etc 22)which ligament can b palpated on per rectal examination? a)uterosacral b)broad ligament c)round ligament etc 23)main support of uterus? a)broad ligament b)transverse cervical ligament etc 24)an incision given supra pubically in a female for LSCS would damage which artery? a)superior epigastric b)inferior epigastric etc 25)ovary is attached 2 back of braod ligament by? a)mesovarium etc etc . which is the least likely complication? a)bone deformity b)mental retardation c)cleft palate 32)anticoagulant are contraindicated in pt wid? a)thrombocytopenia etc etc 33)a pt showing eosiniphilia.lymphoctytopenia polycythemia his blood levels show increase ACTH level which of following is the cause? a)cushing syndrome b)sudhing disease c)addisons disease .26)which sturcture does not form walls of inguinal canal? a)lacunar b)external oblique apenerosis c)conjoint etc 27)insulin help in absorption of glucose through? a)musoca of small intestine b)renal tubules c)hepatocytes etc 28)glucose is absorbed in renal tubules thro? a)2ndry active transport b)facilitated diffusion etc 29)after lscs(c\section) a woman is bleeding intaperitonealy which artery can b damaged? a)uterine b)ovarian c)internal iliac etc 30)which organism require selective media 4 its diagnosis? a)vibrio cholera b)e coli c)campylo bacter etc 31)a pregnant woman on phenytoin. While treating this patient's gonorrhea infection. Stomach .etc 34)ca breast will effect which of the breast quadrant mostly? a)upper n outer b)medial c)inner n medial etc 35)a women having ca breast which group pf lypmh nodes is effected? a)anterior axillay etc 36)a mother brought her daughter wid vaginal bleeding she is 8 yrs old taller 4 her age wid well develop breast n odr wise normal which coud b the cause? a)constitutional b)hypothalamic etc 37)a man wid full burns in icu develop bleeding diathesis which coud b the cause? a)dic b)infection etc 38)a women brought 2 ER unconcious wid history of postpartum hemorrhage 10 yrs back which of the following is dx? a)sheehan syndrome b)low bp etc 39) Main functions of extracellular fluid is? a)transport electrolytes b)removal of waste material etc 40)therapeutic index show a)drug efficacy b)safety Q1. She is sexually active with one partner and uses condoms intermittently. Examination reveals some erythema of the cervix but is otherwise unremarkable. She developed this symptom 2 days ago. A urine culture is sent which comes back negative. Most of the dietary iron is absorbed in the A. A 16-year-old female comes to the physician because of an increased vaginal discharge. treatment must also be given for which of the following? Q2. Sexually transmitted disease testing is performed and the patient is found to have gonorrhea. She also complains of dysuria. 3rd lumbar Q6. Anti endothelial antibody C. 9th thoracic B. Iliohypogastric C. Deep inguinal node Q7. C. Femoral vein D. CCk causes it to contract D. Proteinuria C. 2nd lumbar E. Which is not a content of femoral sheath A. 1st lumbar D. Angioderma Q8. Which muscle extends the knee with hip extended A. The Most Common Side Effect Of Captopril Is A. Cough E. All of the following are examples of traction epiphysis except A. Rectus femoris D. 10th thoracic C. Referred pain from ureteric colic is felt in the groin due to the involvement of the following nerve: A. Femoral nerve B. Femoris B. Vastus lateralis Q9. E. The Gall bladder: A. Subcostal B.B. Genitofemoral . Concentrates the bile C. only B and C E. Condyles tibia Q10. Mastoid process B. Vastus medialis C. The spinal nerve supplying skin around umbilicus is? A. Tronchanters of femur D. D. Produces the bile B. Tubercles of humerus C. Upper part of the small intestines Lower part of the small intestines Upper part of the colon Lower part of the colon Q3. None of Above Q5. which is immunological marker of type 1 DM : A. Ilioinguinal D. Postural Hypotension B. Femoral artery C. B & C Q4. A. Anti saccharomyces antibody D. Acute Renal Failure D. Anti GAD B. Parietal C. Atrial pathways C. Colorectal cancer C. Zygomatic bones B. Lymphoma D. Bundle of His D. Tuberculosis Q13. All of the following are pneumatic bones except A. None of above Q12.Q11. Purkinje system E. Ethmoid D. Maxilla B. Atrial myxoma E. Hypernephroma B. Treacher Collins syndrome is characterized by underdevelopment of : A. Sphenoid bones D. SA node B. Mastoid . Which one of the following is least likely to cause a pyrexia of unknown origin ? A. Maxillary bones C. Conduction speed is slowest in the: A. The propagation of repolarization Q14. Furosemide C. Bronchioles C. Which of the following diuretic is used in Petit -Mal Epilepsy? A. 3rd of 12 th B. Terazosin C. Which crosses placenta A.Q15. 6th to 12 Q16. alpha1 antagonist C. Dystrophic calcification Q22. Muscles of thenar eminence D. alpha1 agonist B. 4th to 12 C. Ig M C. Ig E Q19. Bleeding tendency C. Yohimbine Q20. alpha2 agonist D. Lumbrical muscles to middle finger C. Trachea B. Spironolactone . Oesophagus Q18. If median nerve is injured at the wrist then loss of function of all of the foll. Prazosin B. Acetazolamide B. Clara Cells are found in : A. alpha2 antagonist Q21. Ig A B. Clonidine is : A. Adductor pollicis Q17. Mannitol E. All of the following are seen in Multiple myeloma except : A. Visual disturbance B. Selective alpha1-A blocker is : A. Tamsulosin D. will take place except: A. Chlorothiazide D. Proteinuria D. Alveoli D. Ig g D. 5th to 12th th D. Which of the following nerves is present in the posterior fossa th A. Lumbrical muscles to index finger B. The cricoid cartilage of the larynx C. T wave inversion E. Term euphoria refers to: A. which forms a lid over the airway when swallowing B. Haemophilia A . Cervical branch B.factor XI deficiency D. Zygomatic branch E. Elated mood Q27. Tall T wave Q24. Haemophilia C . The most common malignant tumour of kidney in adults is A.factor IX deficiency C. Posterior auricular branch Q26. Depressive mood C. Transitional cell carcinoma Q25. Leukemia D.vascular haemophilia. What is the most specific ECG finding for pericarditis? A. Lymphoma E.factor V deficiency E. PR depression C. Aggressive mood B. The "Adam's apple" in the throat is actually A. Pearl : A. von willebrand's disease Q28. Osteosarcoma Q29. Squamous cell carcinoma of the skin B. Patient with xeroderma pigmentosum is likely to develop ? A. Haemophilia B . The epiglottis. Squamous cell carcinoma of the lung C. Squamous cell carcinoma D. Renal cell carcinoma C. Buccal branch D. The thyroid cartilage of the larynx D. ST deprsion D. Which of the following nerve is the first extracranial branch of facial nerve? A.Q23.factor VIII deficiency B. The vocal cords of the larynx E. Parahaemophilia . Wilms tumor B. Marginal mandibular branch C. Wide spread ST elevasions B. Blunted mood D. Pseudohaemophilia . Several paired cartilages of the larynx . Thyrotropin releasing hormone (TRH) Q36. The bone which forms the shape of your face (wide or narrow) is your “cheekbone” or your ? A. Acromegaly B. Intraventricular septum Q34. Gigantism C. Factor VII C. Mandible D. Dwarfism D. except: A. Growth hormone releasing hormone C. Vomer Q32. Protein S E. Zygomatic bone C. Thyrotropin (TSH) E. All of Above Q33. Choroid & sclera of eye C. Vomer Q31. Cortisol B. SA node D. Inter arterial septum B. Tunica media of great vessels D. Factor VIII Q35. Sphenoid D. Excess growth hormone after epiphyseal growth causes: A. Parietal B. All the following are vitamin K–dependent coagulation factors except A. Olfactory epithelium .Q30. Frontal bone B. All are neural crest cell derivatives except A. Right coronary artery supplies all. In the adult skull each of the following bones exists in pairs except the: A. Apex of heart C. Factor X B. Zygomatic E. Oxytocin D. Vomer bone B. Which of the following is a glycoprotein hormone? A. Protein C D. Maxilla E. Temporal C. and Olanzapine E. Abnormal uterine bleeding B. Natal teeth are defined as: A. Staph. Lateral pterygoid C. Osteoblast D. Relation of tooth to ramus of mandible and 2nd molar B. Teeth erupting after 1 year of age E. Relative depth of 3rd molar in the bone C. Teeth erupting after 2 years of age. All of the above Q41. Amoxapine Q42. Zygomaticus Major Q39. Classification of impacted 3rd molars(mandibular molars) is based on: A. Legionella pneumophila B. Teeth erupting in the 2nd-3rd month C. Fenoldopam B. Multiple ovarian cyst . Aureus C. Streptococcus D. Pre eclampsia toxaemia C. Bromocriptine And Cabergoline C. Headache D. Clozapine. Droperidol. Mycoplasma E. Teeth present at time of birth B. Based on position of long axis of impacted 3rd molarin relation to long axis of second molar D. Osteoclast cells are derived from which cell type: A.Q37. Features of hyperthyroidism E. Temporalis B. Risperidone. Monocyte B. Viral pneumonia Q43. Metoclopramide. Medial pterygoid D. Q40. Osteocyte Q38. Osteoprogenitor C. Teeth erupting between 6months to 2years D. And Domperidone D. Which of the following are D2 agonists and thus are useful for hyperprolactinemia? A. Excessive contraction of following muscle causes dislocation of jaw: A. All are clinical feature associated with Gestational Trophoblastic Disease except: A. Community-acquired pneumonia (CAP) the most common cause is : A. Which one of the following diseases is most likely to be associated with scleritis? A. Crohn's disease C. She has no past cardiac history of note and has been generally fit and well recently. Severe pneumonia. Femoral vein D. Epispadias Q48. C6. Rheumatoid arthritis D. Lateral plantar B. E. Nerve involved in tarsal tunnel syndrome is : A. Pulmonary embolus. T2 Q47. Submandibular gland C. SLE E. Which is the most common congenital malformation of the male urethra A. Ankylosing spondylitis B. Ant tibial Q51. Necrobiosis lipoidica B. Acute myocardial infarction. Psoriasis D. T1. C. C6 B. C7 C. Pyoderma gangrenosum C. Which of the following is the most likely cause of her arrest? A. Post tibial D. Parotid gland Q50. The lateral boundary of femoral canal is formed by A. Lacunar ligament B. On examination he has an erythematous circular lesion that has a raised border. C8 D. Hypertrophic obstructive cardiomyopathy. A 40-year-old lady collapsed during an aerobics class and was brought to accident and emergency by ambulance in asystole. Pneumothorax Q46. Ulcerative colitis Q45. The small muscles of hand are supplied by spinal segments? A. Lacrimal gland B. D. Granuloma annulare Q49. Hypospadias D. Nasal glands D. B. A 23-year-old man with type-1 diabetes has noticed an unusual lesion on the dorsum of his left hand. C5. All are supplied by Facial Nerve Except: A. T1 E. Urethral stricture B. Posterior urethral valve C. C7. Femoral nerve . What diagnosis fits best with this clinical picture? A. Femoral ligament C.Q44. Medial plantar C. C8. . Systemic lupus erythematosus Q57. Hepatic artery C. Hepatomegaly & Hyperglycemia B. and painless nasopharyngeal ulcers. sparing the nasolabial folds. Spleenomegaly & Hyperglycemia Regarding histology of gastrointestinal tract. Radial nerve D. Goodpasture syndrome B. Salt retention D. Paneth cells contain eosiniphilic granules in apical cytoplasm Q60. The pronator quadratus has the same innervation of the following muscle: A. Aldosterone release Q56. Cystic artery Q59. Macroglossia & Hypoglycemia E. Suspecting an autoimmune disorder. H1N1 B.Q52. Which one of the following diseases is most likely responsible for this immunological reaction? A. H2N1 C. Median nerve Which virus is responsible for avian influenza? A. Thirst B. Hepatomegaly & Hypoglycemia C. Diapedesis Q55. H5N1 Q54. Long thoracic nerve B. Axillary nerve C. Celiac artery B. Which one is the initial event in transfer of WBC to the site of inflammation? A. Flexor digitorum profundus of middle finger Q58. Winging of scapula is due to paralysis of: A. A 25-year-old female presents to her doctor’s office with a rash over the malar eminences of her face. which is inappropriate: A. Pavementation D. The gastroduodenal artery is derived from A. a skin rash due to exposure to sunlight. Multiple sclerosis C. Myasthenia gravis D. Rheumatoid arthritis E. Flexor digitorum superficialis B. Q53. Vasoconstriction C. Hepatomegaly & Macrglossia D. Margination B. H3N1 D. Palmaris longus C. . her physician obtains an antinuclear antibody test of her blood and finds high titers of anti-double-stranded DNA antibodies. Short transient action of angiotensin II is: A. Emigration C. Flexor pollicis longus D. Splenic artery D. Most common clicnical finding/symptom encountered in glycogen storage diseases: A. H4N1 E. Patient is to get renal transplant. RBC B. Prostacyclins are secreted by: A. Stratum spinosum D. Mast cells Q64. Anti-phospholipid antibody syndrome B. Peyer’s patches are present in submucous layer of duodenum C.B. Buccal mucosa D. Stratum basale B. Stratum cornium C. Protein C resistance Q63. HLA testing is to be done. Most important pre-requisite for renal transplantation: A. Mixed lymphocyte assays . Fibroblasts E. Parietal cells of human stomach secretes intrinsic factor D. What should be send for testing: A.what cells will become the basic source of its regeneration? A. Renal tubules D. WBC C. Protein S def E. ABO compatibility B. On the limited area of epidermis as a result of trauma layers up to a growing are absent . Protein C def D. Vascular endothelial cells C. Most common acquired deficinecy causing thrombosis: A. Bone marrow Q65. Striated brush border is composed of micro villi Q61. Stratum lucidum Q62. HLA testing C. T cell count of recipient D. Anti thrombin III def C. Platelets B. Following is not a function of liver: A. Albumin synthesis C. Lipogenesis B. Gluconeogenesis .Q66. Ig synthesis E. Cholesterol synthesis D. Urination C. Pathogenicity of staphylococci is because of: A. Main mechanism for heat loss: A. Treponema C. Oral D. Gonococcus D. Lecithinase B. ECF C. SC E. Sister Q68. identical twin. Respiration B. Brother C. ICF D. A female pt with chronic renal failure has 5 ppl who are willing to serve as donor for renal transplant. Tetanus B. IM B. IV C. half brother. Stepbrother E. SL . ECF+ICF Q69. Sweating Q70. which is markedly decrease: A. Husband D. Conduction & radiation D. Coagulase C. Plasma B. All Q71. Most suitable donor for the pt would be: A. Total body fluid E. Bioavailability is for drugs administered: A. In sever dehydration. M protein D. sister and son.Q67. Humans are the terminal end for: A. Her husband. Cell wall Q72. Patient lying in operating room with room temperature of 21c and 80% humidity. Twin B. Shanghai fever---Pseudomonas C. Brazilian purpuric fever---Hemophilus aegyptius (pink eye) D. Colorado tick fever---Orbivirus H. Oroya fever---Bartonella bacilliformis (Carrion's d/s) F. Different types of fever A.Q73. Q fever---Coxiella burnetti G. Trench fever---Rochalimaea quintana (five day fever) . Havernhill fever---Streptobacillus monoliformis E. Pontiac fever---Legionella pneumophila B. Protamine sulphate Q77. Elastin Q76. Nerve root supplying anterior triangle of neck: A. Liver B. Laminin D. Hypothalamus E. C3-C4 D. Kidney C. IV CALCIUM D. Artery of Adamkiewicz B. Basilar artery C. Vertebral artery Q79. Atrium D. Internal carotid artery D. Biconcave shape of RBC is maintained by : A. Reversal of the anticoagulation effect of warfarin is accomplished most quickly by: A.Q74. This artery arises from which of the following? A. Dermatology Q75. high serum phosphate increase PTH. Band 3 & band 4 proteins B. C2 . Osteomalacia C. Angiotensinogen is produced in? A. Primary hyperprarathyroidism Q78. Adult with low serum calcium and ionic calcium. C2-C3 C. Chronic renal failure B. VIT KConcentrated factor VIII C. Peudohypoparathyroidism E. C4 E. FFP B. Posterior inferior cerebellar artery E. Hyperparathyroidism D. Spectrin & ankyrin C. C1 B. Parathyroid adenoma B. About 75% of the blood supply of the spinal cord is derived from the anterior spinal artery. normal alkaline phosphatase. Normal pth is seen in: A. Parathyroid hyperplasia C. Most likely: A. Osteoporosis D. Bone metastasis Q80. Damage to cervical sympathetic trunk will cause: A. All of above Q84. Pupillary dilatation B. Partial ptosis Q82. In traumatology hospital a patient is delivered with the wound of large lumbar muscle. Apoptosis D. Femoral nerve Q89. HPL C. When removal a tumour of pancreas is damaged vessel passes to the superior border of pancreas. Ilioinguinal C. Reperative regeneration Q86. manipulating in the region between stomach and liver. Insulin E. cells which are divided are visualized in a basal layer. Gastroepiploic right artery D. Where is its gate (hilum)? A. In histological specimen of bioptate of epidermis of skin of the healthy adult man . Medial D. HCG B. Genito-femoral D. The hormone only secreted during pregnancy? A. Thyroid hormone C. In supravesical fossa C. Radial B. Physiological regeneration B. Musculocutaneous Q83. Which vessel is damaged? A. Pancreaticoduodenal superior artery B. A patient has oblique inguinal hernia. A patient leaves an opportunity to extended leg in a knee-joint. Relaxin D. Pancreaticoduodenal inferior artery C. What process is provided by these cells? A. Laterally from the inferior epigastric artery B. Hormone that stores carbohydrates . because there is: . Medially from the inferior epigastric artery Q85. Ilioepigastric B. Which nerve is damaged in this case? A. a surgeon beware of to injure a hepatoduodenal ligament. Which nerve is damaged? A. and also absence of skinning sensitiveness on back surface of hand. Cortisol D. Adaptation C. Inc sweating C. Growth hormone B. A patient after trauma of right upper limb there is the parafunction of the extensors muscles.lipids n protein in body ? A.Q81. Dry mouth D. Ulnar C. PTH Q87. Splenic artery Q88. During the operation. In femoral fossa D. 25-Dihydroxycholecalciferol D. Avidin binds with: A. 25-Hydroxycholecalciferol B. Subclavian artery C. Which is the most active metabolite of vitamin D involved in calcium homeostasis ? A. B. hypotaxia motions. C. HLA DR2 C. HLA B5 B. A 45 years old patient. Vertebral artery B. 11 C. Which is this vessel? A. Riboflavin Q94. D. Femoral vein C. umblical hernia and upward slant of lateral epicanthus. Epigastric superficial vein B. 21 D. HLA DR4 Q91. Most likely trisomy of chromosome: A. Vit B12 B. proper hepatic artery. What is the most common human leucocyte-associated antigen (HLA) type in rheumatoid arthritis ? A. protruded tongue. the patient has the osteochondrosis of the cervical part of vertebral trunk and compressed vessel passes through the transverse openings of the cervical vertebrae. 24. complaints about dizziness. 1. 9 B. Vit D E. 7-Dehydrocholesterol C. hypotonia. Biotin C. Varicose expansions of which veins can be from the system of porto-caval anastomoses? A. stagerring during walking. A patient suffers from the cirrhosis of liver. Circumflex iliac deep vein Q93. portal vein Proper hepatic artery. common hepatic artery. hypermobility. 14 Q95. Subcostal vein D.25-dihydroxycholecalciferol E. Vit C D. Scenario of mental retardation. portal vein Common biliary duct.A. External carotid artery D. hepatic veins Q90. HLA DR3 D. Cholecalciferol Q92. Сommon biliary duct. hepatoduodenal artery Portal vein. Internal carotid artery . After being set under inspection. what cells are these? A. Posterior tibial artery D. Internal iliac artery Q97. Early phase of acute tubular necrosis C.they originate from the bone marrow and behave to the system of mononuclear phagocytes . In which one of the following . Anterior tibial artery . In a patient. Pudendal internal artery D. Was performed supravaginal amputation of uterus with is the removal of the uterine tubes. Hypocalcemia E. Ovarian branch of uterine artery C. Ovarіan artery B. the malignant tumour of abdominal part of eosophagus is diagnosed. Keratinocytes C. and vessel which supplies ovaries passes in it. Langerhan's cell Q98. Stem B. Langerhan's Q99. Secondary hyperaldosteronism Q101. Ovaries are not removed Suspensory ovary ligament is fully stored. Nephritic syndrome D. Keratinocyte C. will decompose it on particles and will present to other immunocompetent cells of skin? A. Data cells of skin are to 3% from all cells of epidermis . Pericardial lateral lymphatic nodes Q100. Paratracheal lymphatic nodes C. Cardic lymphatic annulus B. Which group of lymphatic nodes is regional for the indicated part of the eosophagus? A. Popliteal artery B. Fibular artery C. renal excretion of water most likely to be increased: A. Melanocyte D. what cells of epidermis will take an antigen the first. Merkel D. Prevertebral lymphatic nodes D. About оcclusion of which artery is it possible to think? A.Q96.in composition of epidermis are localized mainly in a stratum spinosum . Chronic renal failure B. A 40years old woman has fibromyoma of the uterus. Melanocyte E. Barrel B. Name it: A. A man on a manufacture got the lacerated wound of shoulder by ferruginous instrument. A patient has the ischemia of tissues below the knee joint that is accompanied by “remittent lameness”. Hand-to-face contact C. Eating contaminated food . Breathing viruses in air B. such as colds and influenza.Q102. Drinking infected water D. most commonly spread? A. How are infectious diseases. . Hepatitis B B. Primary cartilaginous B. Saddle synovial D. Middle meningeal artery is transmitted through: A. Zona spongiosa Q108. Zona compacta D. Secondary cartilaginous C. Which part of visual way for certain is damaged? A.Q103. Calcarine groove . Dose variability Q104. Foramen ovale C. Ball & socket synovial Q105. Foramen rotundum B. Thorax E. Optic chiasma B. Foramen lacerum Q109. Optic tract D. Zona pellucidum C. Optic nerve C. The therapeutic index of a drug is a measure of its: A. One of the following vaccination is absolutely contraindicated in pregnancy: A. Which of the following is the type of joints between stapes and incus? A. Efficacy D. Foramen spinosum D. Zona basalis B. Abdomen D. Upper limb B. Lower limb C. From which of the following layers the regeneration of endometrium take place: A. The most common site of venous disorders is A. Potency C. Yellow fever Q107. Rabies D. Head and neck Q106. At the inspection of patient absence of sight is discovered in the medial halves of sight areas of both eyes. Safety B. Cholera C. Inferior branch of oculomotor nerve C. A man. Serum & urine bilirubin E. Muscle. Most relevant investigation: A. Right vagus nerve D. 47 XXX D. Trochlear nerve D. 45 XXY Q115. biceps brachial muscle B. Biceps brachial muscle. The most common type of intussusception is: A. Ileocolic D. Deltoid muscle. Submucous layer Q116. Left trigeminal nerve Q112. Right glossopharyngeal nerve C. Which nerve is certainly damaged? A. Ileoileal B. Brachial muscle. Liver biopsy . infraspinatus muscle E.Q110. 45 XO C. Superior branch of oculomotor nerve B. Patient presented with yellow discoloration of sclera. What tunica of the organ react to this action? A. Coracobrachial muscle. The chromosomal anomaly in Klinefelter syndrome is: A. Bilirubin+ALT C. Right hypoglossal nerve B. 42 years old appealed to medical hospital on an occasion of sword-cut of the inferior part of the anterior surface of shoulder. biceps brachial muscle Q111. As a result of trauma of skull with the damage of the superior wall of right orbital cavity a victim threw away an opportunity to levitate the superior eyelid of right eye and to look up. The bleeding stop following a delivery is connected with the action of oxytocin on the wall of uterus . Colocolic C. Endometrium C. Perimetrium B. 47 XXY B. anconeus muscle C. Which of the adopted muscles are probably damaged in the patient? A. Viral serology D. Myometrium D. dark urine. A patient at pulling out lingua there is the rejection of the apex at the left. Abducent nerve Q113. Objectively: heavy bending of forearm. Ileo-ileocolic Q114. Motive innervation of which cranial nerve is broken in this case? A. loss of apetite and vomiting. Alkaline phosphatase B. supraspinatus muscle D. Hypersegmented neutrophils B. None Q118. tRNA D. Celoblastula Q121. peripheral blood smear will show A. Attachment tto diaphragm D. About blood group which is appropriate: A. Low platelets C. DNA B. Zygote cell division after blastula formation. mRNA C. ß-endorphins most abundantly are found in: A. O2 D. Attachment of hepatic veins to inferior vena cava C. Hypothalamus C. Intraabdominal pressure E. Co2 E. In megaloblastic anemia. Glucose C. Abdominal muscle tone B. Blastocyst B. Morula D. Discoblastula C. Are enzymes B. Thalamus B. None Q119. what type of blastula is specific of a human being? A. Protein Q122. Na B. Basal ganglia . Microcytes D. Example of secondary active transport: A. Liver is held in the upper part of abdominal cavity by: A. Brainstem D. Secreted in saliva D. Genetic diseases mostly have pathololgy involving: A.Q117. Proteins Q123. Called agglutinins C. Peritoneal ligaments Q120. Formation of free radicals Q129. Clonidine E. Patient on HTN meds presents with increase indirect bilirubin with positive coomb’s test (at diff temp –ve)? Drug most likely involved: A. Proprioceptors B. Renal tubular Q127. Cardiac muscle C. Anemia of maturation failure: A. Chronic disease . Splenomegaly D. 50% of generation not affected and do not transmit disease to other generation. Autosomal recessive C. Iron def anemia B. Glial cells D. Hepatocyte E. Features seein hemolytic anemia all except A. PERMANENT OR NON-REPLICATING CELLS ARE: A.Q124. Visual C. Autosomal dominant B. Pernicious anemia D. Vestibular Q130. A man who went for a ride on a roundabout had amplification of heart rate. Changes in nuclear membrane B. Formation of cytopplasmic pigment E. Bone marrow hyperplasia B. Alteration in protein synthesis D. Increased reticulocytes C. Normocytic C. Auditory D. Attack mitochondria C. sweating and naurea. Skeletal muscle B. Hydrochlorthiazide D. Methyldopa B. What receptors stimulation is it primarily connected with? A. Mitochondrial Q128. Conjugated hyperblirubinemia Q126. Viruses produce pathological effects by: A. Hydralazine C. Microcytic hypo E. X-linked D. Multifactorial E. Beta blocker Q125. Regarding adult polycystic kidney disease. most likely inheritance pattern: A. Double blind B. None Q137. In course of operation a surgeon found that internal organs were not damaged but the knife injured one of muscles of renal pelvis. This is an example of: A. To disclose about a recntly diagnosed fatal disease. Most approprate: A. T-wave on ecg represents: A. Receptor B. Following use amp as their mechanism in cell membrane: A. logical. Refractory period Q134. Hermaphroditism B. Single blind C. A gyneacologist testing efficacy of norfloxacin and amoxicilllin in UTI. Active listening B. Triple blind D. Erector muscle of spine B. Troponin T D. Ion channels D. GOOD DOCTOR-PATIENT MUST HAVE: A. Carrier protein C. Examination of a newborn boy’s genitalia revealed an urethral hiatus that opens on the undersite of his penis. Crisp. Abdominal internal oblique muscle . Tropomyosin B. Ideal decorated clinic C. evidence based accurate information to the patient and family according to demand Q133. Good social skills D. Great psoas muscle D. Tell patient but not family C. Tell family & patient as soon as known B. Cryptorhidism D. What muscle is it? A. Tell family but not patient D. Atrial repolarization D. Cohort Q136. Ventricular repolarization B. Epispadia C. A patient with a knife wound in the left lumbar part was delivered to the emergency hospital. Ventricular depolarization C. Hypospadias Q138. Logical answers to questions asked Q135. Cross-sectional study E.Q131. Abdominal external oblique muscle C. Troponin C C. What malformation is it? A. Which of the following binds to and inactivates myosin binding site on actin: A. Hormones Q132. grouped as A & B randomly allocated to patients receiving either one of them. Which of the following shifts the o2-hgb dissociation curve to right: A. Hypothalamus B. As and result of growth of tumour in the cavity of a ІІІ ventricle of cerebrum at a patient develops vegetative disorders as the parahypnosis. Alkalosis B. NO ( nitric oxide ) D. Drug which should be stopped? A. develops gout. Isoniazid B. Tegmen of midbrain D. Pons Q140. Rifampicin Q141. Peduncles of brain C.Q139. Pyrazinamide D. not saccharine diabetes. Fetal hemoglobin . Ethambutol C. Streptomysin E. Increase pH C. The irritation of nuclei of which area of cerebrum caused these symptoms? A. all types of exchange. thermoregulation. Patient receiving ATT. Zona spongiosa Q146. Myocardium C. Zona pellucidum C. Transverse colon C. 41% of formed elements in blood comprises of rbcs. knife wound of lumbar region. Neutrophils B. A patient presents with itching (and some other symptoms) on workup found to have intestinal worm infestation. Endocardium D. All of the above Q144. 41% of formed elements in blood are RBCs B. A hematocrit of 4i% means? A. Zona basalis B. 41% of blood is serum C. From which of the following layers the regeneration of endometrium take place: A. Pericardium B. Duodenum B. Zona spongiosa . Basophils Q148. 41% of rbcs contain hemoglobin Q147. In a victim. wbcs and platelets D. Left flexure of transverse colon D. Takes place damage of right kidney. From which of the following layers the regeneration of endometrium take place: A. Zona compacta D. Which layer of heart is responsible for the pimping action of heart? A. Monocytes E. His blood workup will most likely show increase: A. Eosinophils C. Descending part of colon Q145. Zona basalis B. Muscle fibers which form the pacemaker & conductive system of heart are associated with? A. Zona pellucidum C. Endocardium D. Which organs of retroperitoneal space can be damaged in this case? A.Q142. on the right side. Pericardium B. None Q143. Myocardium C. Lymphocytes D. Zona compacta D. Which one is the malignant tumour of bone ? A. Simple bone cyst D. Osteoma C.Q149. Osteosarcoma B. Giant cell tumour . Secretion of which hormone will be inhibited A. Internal sphincter of anal orifice Q151.Q150. Parietal cells of human stomach secretes intrinsic factor D. Renal failure E. Distribution of a drug in body is not affected by: A. Insulin D. Adh B. Deep transversal muscle of perineum D. Paneth cells contain eosiniphilic granules in apical cytoplasm B. Vit C def Q156. Metastasis D. low serum albumin and moderate anemia. Person receiving large amount of iv dextrose water. Age C. Regarding histology of gastrointestinal tract. Nor-epinephrine Q157. which is inappropriate: A. The longitudinal coat of muscularis externa is arranged in 3 bands in colon Q152. Seminoma Q153. enlarged liver. main neurotransmitter is: A. Sarcoma C. Aldosterone E. Dopamine D. In slow wave sleep. Kwashiorkar B. Disc lesion between L4 & L5 will lead to? . A 6 yrs old boy. Peyer’s patches are present in submucous layer of duodenum C. Sphincter muscle of urethra B. Hamartoma E. Adenoca D. Acetylcholine C. Beri beri D. Cortisol C. Invasion and infiltration C. Anaplasia Q155. Superficial transversal muscle of perineum C. Teratoma B. Most likely diagnosis? A. Most impotant distinguishing feature between benign and malignant neoplasm: A. Marasmus C. Tumor wth involvment of all germ cells? A. Striated brush border is composed of micro villi E. Which muscle has trauma? A. Serotonin B. apathy. Sciatico-cavernous muscle E. Sex B. GH Q154. After the traumatic defect of the perineum a victim has any incontinence of urine. Inc cell growth B. Cardiac failure D. Pregnancy Q158. peripheral edema. Metaplasia can occur in all the following except? A. Brain B. Exhibits seasonal pattern D. Ventricular depolarization C. Is constantly present in a given population group C. Myocardial infarction is most commonly occur in: A. Heart is underdevelopment C. D. Gut Q164. Reduce knee jerk Reduce ankle jerk Weakness of foot dorsiflexion Reduced sensation on the small toe Q159. Occurs clearly in excess of normal expectancy B. Chromosomal breakage C. True about Apoptosis are all except: A. Refractory period Q161.A. Is prevalent among animals Q160. Esophagus D. Microfilaments C. Endometrium E. Inflammation is present B. Atrial repolarization D. Mitral regurgitation B. All of the above . T-wave on ECG represents? A. C. Clumping of chromatin D. Brain is underdevelopment D. Nissl bodies B. Mitochondria Q162. B. Aortic regurgitation Q163. Ventricular repolarization B. ‘Endemic Disease’ means that a disease: A. Axon of a nerve terminal doesnot contain: A. Microtubules D. Cell shrinkage Q165. Liver C. Aortic stenosis C. Infants have greater risk of developing water intoxication in the first month of life because: A. Filtration mechenism of kidney is underdeveloped B. Mitral stenosis D. Hypochromic Q169. In megaloblastic anemia . Three to five months B.Q166. Vitamin D B. Superior colliculus D. Diencephalon doesnot include: A. Vitamin A C. Six to nine months D. Fout to six months C. Twelve months Q168. Which one among the following vitamins is necessary for blood clotting? A. None of Above Q167.it is removed by the process of ? A. How long is breast feeding recommended by the PAKISTAN Paediatrics Association . Elastin B. Normochromic B. Clot retraction D. Once a clot has served its purpose . Nine months D. Microcytic . Fibrillin C. Collagen Q171. Twelve months E. Hypochromic C. Myosin D. Actin E. Six months C. Thrombosis . Pulvinar C. Habenular nucleus B. Three months B. Hemolysis B. Macrocytic . RBCs are? A. Normochromic D. Fibrinolysis C. Which protein defect is responsible for Marfan's syndrome? A. PPA ? A. Mamillary body Q172. How long is breast feeding recommended by the American Academy of Paediatrics? A. Macrocytic . Microcytic . Vitamin C Q170. Vitamin K D. Platelet plug .Q173. Bolus D. The term for abnormal clot that is formed in the vessel? A. Embolism C. Thrombus B. Internal longitudinal Q180. Which of the following enzyme plays main functional role in regulation of cholesterol synthesis ? A. A young patient presenting with massive haematemesis was found to have splenomegaly. A patient has pain. Intercostal space III B. Duct of Wirsung C. Stenson's duct D. Deep inguinal B. edema and reddening of his skin in the anterosuperior area of his thigh and his foot’s toe. Juxta-articular-osteosclerosis B. Decarboxylase C. Third rib . Esophageal varices C. Bone erosions D. A patient high bound of heart at level second intercostal space. Second rib D. Wharton's duct Q179. HMG CoA synthase D. Duodenal ulcer B. Superficial inguinal D. At which level the high bound of heart is localized at healthy person? A. What lymph nodes of his lower extremity responded to the inflammatory process? A. Intercostal space II C. Gastric ulcer D. Mevalonate kinase B. Which one of the following is a recognized X-Ray feature of Rheumatoid Arthritis? A. Erosive mucosal disease Q176. Superficial longitudinal C. Blood pressure C. Peri-articular calcification Q177. Colour of baby B. Apgar score inclued all the following except: A. Parotid duct is also known as: A. In this case the most likely source of bleeding is: A. Activity Q175.Q174. Duct of Santorini B. Bamboo spine C. HMG CoA reductase Q178. Hepatorenal ligament C. Hepatoduodenal ligament B.Q181. Duodenorenal ligament . Gastrocolic ligament D. Conducting surgical interference on gall-bladder a doctor must define a general cystic duct. Which anatomical formation of peritoneum is located? A. Which part of urinary canal more frequent narrows in old age? A. None Q183. About pseudomonas all are true except A. Plateletes D. They are acidic precursors of brain amines C. Which of the structures transferred below suffered? A. Most common organism in burn patient Q186. Air conditioning B. WBC C. Infection is mostly due to endogenous source D. Strict aerobes B. RBC B. Leptospirosis all are true except? A. Gaseous exchange D. Membranaceous part Q184. Ciliary body Q185. Cornea B. They cross blood brain barrier B. Urethral glands C. External urethrae sphincter muscle D. Bronchial circulation is associated with? A. They are metabolites of various neurogenic amines . Can grow in disinfectant in hospital C. Drug absorption C. Reserve volume Q188. Which corpuscle is non-nucleated ? A. Prostatic part B. Anterior camera of eyeball (humor) C. Both a and c E. Dopa and 5-hydroxytryptophan are clinically imp because? A.Q182. A electricweld which carried the burn of eyeball is delivered into eyeing department. They acts as neuromodulators D. In old man has complicated act of urination. Incubation period is 10 days D. Faces and urine of rodent is the cause B. Lens D. Onset of igm in 5 days C. It is sensitive to penicillin Q187. Ca of Lung C.cough and fever. Pneumonia D.Q189. bilateral chest pain. Lung failure . Lung abcess B. Whats Your Diagnosis? A. A case is presented by breathlesness. Which one is encounterd as the major cause of chronic liver disease? A. Which of the following method is used to detect primary herpes? A. Hepatitis C D. L. Acute hepatitis B. Lecithene B. Ovaric Q194. About 50 mm Hg E.Test C. Million's reagent test . ELISA D. Fibrin C. The defect of development of which of membranes of testis is reason of hernia of scrotum? A. Stomach C. Vaginal membrane of testis C. About 30 mm Hg C. External spermatic fascia D. With inflammation of which organs does it follow to conduct a differential diagnostics? A. Protein associated with obesity? A.Test B. Leptin D.Q190. Cremasteric fascia B. About 40 mm Hg D. Routine cytology D. A patient with appendicitis appendix is located in retrotcecal position. About 20 mm Hg B. Kidneys B. Liver D. Culture with wright stain C. Flourocent stain for cytology Q196. Internal spermatic fascia Q193. What is the name of test for HIV Aids A. Hepatitis B C. Taurine Q195. About 150 mm Hg Q192. T. Culture with giemsa stain B. Hepatitis D Q191. What is the normal systolic pressure in pulmonary artery? A. Palpable fluctuance E. Drooling C. Dysphagia D. All of the following are commonly seen in Ludwig’s angina except A. Decreased neck motion B.Q197. Trismus . Musicians nerve is? A. Acute myelomonocytic leukemia Q200. Furosemide C. Caseation necrosis D. Ulner C. The type of acute mylogenous leukemia associated with a high incidence with of disseminated intravascular coagulation is? A.complement and plasma protein produce a smudgy eosinophilic deposition temed as? A. Ipratropium C. Acute megakaryocytic leukemia D. Osteochondroma C. Coaglative necrosis B.Q198. What is the cause? A. Fatty necrosis Q202. Captopril B. Aminophylline B. Osteogenic sarcoma B. Propranolol Q204. hesitancy. Benign prostate hypertrophy C. Median D. The most common malignant lesion of bone? A. Fibrinoid necrosis C. Acute promyelocytic leukemia C. urgency & frequency. A 36 year old known hypertensive female becomes pregnant. Which one of the following antihypertensive drugs will you most likely prescribe her? A. Narrowing of the external meatus D. Methyldopa E. Which one of the following anti-asthma drugs is given by inhaler? A. The necrotic tissue and deposite of immune complex . An 80 year old gentleman presented to ER with Hx of decreased urine stream. Losartan D. Musculoskeletal Q203. Acute erythroleukemia B. Radial B. Stricture B. Urethral stone Q199. Ewing's sarcoma D. Metastatic carcinoma Q201. Montelukast D. Theophylline . Cystic B. It is set that an infection got into the tympanic cavity through auditory tube which lies in: A. Right gastric C. On the left. A six year boy tells his mom that "there is an alligator under his bed". near base of xiphoid process D. Isolation of affect . from the margin of sternum. Tympanic canal C. C5 to T1 C. A patient with defect of the aortic valve. Gastro-duodenal D. Canaliculus of tympanic chordate Q206. C6 to T1 D. What part of bowels should be examined in order to discover the diverticulum in course of an operation? A. Jejunum D. Fantasy E. in the second intercostal space B. On 2 cm to the right. Tranference D.Q205. Caecum B. The disease began from inflammation of nasolarynx. Ileum C. On the right. Projection C. At the fifth intercostal space C. A child entered to the ETN department of clinical hospital with diagnosis: festering inflammation of the middle ear. Carotid canal D. Altruism B. The defence mechanism he is using is? A. C4 to T1 B. Common hepatic Q209. C3 to T1 Q210. Musculotubal canal B. A patient was admitted to the surgical department with suspected inflammation of Meckel’s diverticulum. Duodenum Q207. A patient had to remove gall-bladder on the occasion of bilious-stoning illness. Which point of thorax does a doctor hearcen the tones of this valve? A. What is the vertical extension of the thyroid in relation to the vertebrae: A. Which artery a surgeon must cross during the operation? A. near base of the xiphoid process Q208. To the tympanic cavity D.Where can the infection spread to unless the disease is managed in time? A. To the orbit B. Examination of a patient revealed an abscess of pterygopalatine fossa. To the subgaleal temporal space C.Q211. To the interpterygoid space . CMV nd Rubella D. A teenager was irradiated with high radiation dose that resulted in serious damages of lymphoid system. Acute diarrhea B. Osseous cuff Q213. Bueberry Muffin rash is seen in infants suffering from infection of A. E. Thyroid B. Toxoplasmosis B.Coli A. Adrenal C. Rubella C. Shigella C. Rotavirus D. patient related a hemorrhage within the superior temporal gyrus. A patient who sufferes from cancer of back of tongue has an intense bleeding as a result of affection of dorsal lingual artery by the tumour. Persistent diarrhea Q218. Facial artery B. Middle cerebral artery B. Basilar artery Q215. Metaphyseal plate C.Q212. In course of indirect histogenesis of tubular bone tissue a plate is formed between epiphysial and diaphyseal ossification centers that provides further lengthwise growth of bones . Dorsal lingual artery C. In All TORCH infections Q217. Syphilis E. Lingual artery Q214. In the blood supply area of which artery is it? A. Osteon D. What vessel should be ligated to stop bleeding? A. What structure is it? A. Liver D. Major cause of Acute diarrhea is A. Anterior communicating artery C. Coli B. Giardia lamblia E.o. Anterior cerebral artery D. Ascending pharyngeal artery D. Osseous plate B. Cryptosporidium . lysis of many lymphocytes. Restoration of normal hemogram is possible due to the functioning of the following gland: A. Neurological examination of a 65 y. Chronic diarrhea C. Episodes of diarrhea lasting more than 14 days and are caused by infections like E. Thymus Q216. Coracobrachial muscle C. At which level of the damaged nerve. Superior vena cava Q221. with the partial loss of function of bending of forearm. Regarding thalasema trait which is appropriate? A. Howell jolly bodies Q223. Which of the muscle function will be damage in this case? A. Serum ferritin levels are normal Q225. Peripheral findings in dietery iron deficiency include? A. Body of vesicae D. The system of which vein is damaged? A. After the output of nerve from canal Q220. Continuous D. rectum and subcutaneous veins of the anterior abdominal wall. High grade B. In the canal of the facial nerve below knee. A patient breaking humerus on the border of the middle and lower third. dryness in mouth (diminishing of saliva separates). Apex of vesicae C. MCV <50% of normal C. Ovalocytes B. Anemia is present in more than 75% of cases B. Triceps brachial muscle D. Which of the anatomic formations can serve as оrientate for finding of openings of ureter at the internal inspection of urinary bladder? A. after output of large petrosal nerve C. On the base of brain B. Inferior vena cava C. Intermittent . Brachial muscle B. In a patient the defeat of facial nerve is diagnosed. A 60 years old patient is found with expansion of veins of eosophagus. Low grade C. Anconeous muscle Q224.Q219. Urachus Q222. Bone deformities are seen in more than 50% of pts C. if a patient has such connection of clinical symptoms: paralysis of mimic muscles. It not decrease of the separates of tear A. violation of taste sensitiveness of the anterior part of lingua. Portal vein B. to output of the large petrosal nerve D. In the cannel of facial nerve. Vesical trigone B. Hemiazygos vein D. Nature of Malarial fever In children is? A. PCV decreases in? A.Q226. Dehydration B. Pregnancy . Polycythemia C. Liver cirrhosis D. Rabies . Macrocytic & hypochromic C. Abscesses Q230. Canine tick C. Feces C. Injectable polio E. Glossitis C. Microcytic & hypochromic Q231. Vector of viral encephalitis is the? A. Which of following is live attenuated vaccine A. In new born infant the commnest source of Tetanus is? A. T memory D. BCG D. Decreases C. B cells C. Dermacenter D. House flies Q229.Q227. Niacin deficiency causes? A. In pernicious anemia. Increases B. Effect of insulin on HMP shunt? A. Cholera C. None Q233. Cytotoxic cells are? A. Umblicus E. Pertussis B. Mast cell Q232. Macrocytic & normochromic B. None Q228. Pellagra B. Anaemia D. Soil B. Demodex B. T lymphocyte B. Eear piercing D. Stops D. RBCs are? A. Microcytic & normochromic D. Which headache is commonest among the following: A.Q234. Exertional . Migraine B. Cluster C. Idiopathic stabbing D. Q235. Decreases C. base of occipital bone D. Sarcoidosis C. Hyoepiglottie D. MCH C. Microcytic & Normochromic D. Extrinsic membrane of larynx are all except? A. Coagulative necrosis is seen in A. Base of occipital bone D. Mxilla B. MCHC D. In iron deficiency anemia. PCV E. RBCs are? A. Cranial base D. Microcytic & Hypochromic Q238. Cranial vault Q241. Gangrene Q239. Effect of insulin on the enzyme activity of phosphofructokinease? A. Cryptococcus infection D. The average volume of a single RBC? A. Pharyngeal tubercle of occipital bone B. The continuous band of muscle constituting buccinator mechanism is anchoraed at A. TB B. None Q237. the continuous band of muscle constituting buccinator mechanism is anchoraed at A. Which 1 of the following undergoes predominantly intramenmaneos ossification? A. Macrocytic & Hypochromic C. Cricothyroid B. Stops D. infranuchal line of occipital bone Q242. Macrocytic & Normochromic B. Cricotracheal Q240. pharyngeal tubercle of occipital bone B. Infranuchal line of occipital bone . Increases B. CI Q236. MCV B. supranuchlline of occipital bone C. Thyrohyod C. Supranuchlline of occipital bone C. Palate C. Secretin B. Epinephrin C. Pantoprozole D. Cranial base D. Especially in winter during exercise and exposure to bright light. Which one is not gastrointestinal medication? A. Cricotracheal Q245. Major potent inhibitor of insulin release is? A.Q243. Which one of the following drug is capable of blocking both Alpha-Beta Adrenoreceptors? A. Step 3. Calcitonin E. Melatonin B. Lactate C. Ranitidine B. Cranial vault Q244. Famotidine C. extrinsic membrane of larynx are all except? A. Which 1 of the following undergoes predominantly intamenmaneos ossification? A. Step 2. Cricothyroid B. Pindololpindolol D. Gastrin D. Mxilla B. Step 10. Serotonin D. pyruvate kinase rxn D. Palate C. Hyoepiglottie D. glucose phosphate isomerase rxn . Oxytocin Q246. Steps 1. hexokinase (or glucokinase) rxn C. Misoprostol E. Labetalol B. Artemisinin Q249. Metroprolol E. Malate Q248. Thyrohyod C. phosphofructokinase rxn B. the production of which substance is increased ? A. Which step of glycolysis is not irreversible? A. Pindolol C. Terazosin Q247. Tuberculosis . Leukocytosis D. ESR increases in? A. Sickle cell anemia C. Polycythemia B.Q250. Mantinance of blood viscosity C. Von Ebner's glands are glands found in? A. Mesenchyme C. WBC AND platelets D. WBC C. Submucosa of the oral mucosa C. Immunoglobulins present in saliva? A. Body temperature regulation . Platelets Q256. Which one is not a function of plasma proteins? A. Epimysium C. Around the mandibular ramus E. Thyroxine D. RBC B. Circumvallate papillae B. Ig E Q255. Vasopressin Q253. Ig D E. Fascia Q252. Anterior to the submandibular glands Q257. Ig M D. Parathyroid hormone B. Ig G B. Perimysium B. Examination of a patient has shown deficiency pulp development .Q251. Buffy coat is seen. Which hormone is responsible for decreased blood calcium level? A. Endomysium D. In the PCV measurement. Erythrocyte sedimentation B. What embryonic source has been affected? A. Mantinance of blood osmotic pressure D.which is formed by? A. Ig A C. Calcitonin C. Oral cavity epithelium Q254. Ectoderm B. Beneath the lower jaws D. Connective tissue sheath that covers each muscle fiber called? A. Endoderm D. 4 to 8 days .Q258. Average lifespan of RBC? A. 80 days B. 50 days D. 120 days C. Trauma B. Monocytes D. Shuttle system used in fatty acid oxidation is? A. Cytosol B. Mitochondrial matrix D. Vitamin A B. Peri-mitochondrial space C. A women has got allergic dermatitis of hands after after industrial contact with Chromium compounds . 1 and 2 both Q264. Vitamin B C. Which one is not regulatory enzyme of citric acid cycle? A. Mitochondrial matrix D. Citrate synthase B. Peri-mitochondrial space C. Mast cell B. α-ketoglutrate dehydrogenase D. Glucokinase Q261.Q259. Hypertension Q265. Where does glycolysis occur? A. The most common cause of Subarachnoid haemorrhage is: A.what cells of skin were mainly damaged by this disease? A. Malate-aspartate shuttle C. Vitamin D E. Vitamin C D. Which vitamin deficiency is not associated with anemia? A. Amyloid angiopathy D. Glycerol phosphate shuttle B. Inner mitochondrial membrane Q263. Isocitrate dehydrogenase C. Rupture of Berry aneurysm C. Plasma cell C. Vitamin E Q260. Neutrophils . Where does krebs cycle occur? A. Inner mitochondrial membrane Q262. Carnitine shuttle D. Cytosol B. Deficiency of vitamin B2 .Q266. Deficiency of vitamin B12 B. Deficiency of vitamin B6 C. Deficiency of vitamin B1 D. Pernicious anemia is caused by? A. Progesterone C. During first two months of intrauterine life the rbcs are produced by liver E. Five time more viscous than water B. What thing about blood is not true? A. Estrogen B. The function of what eye apparatus suffers? A. 2:1 D. Transport of oxygen C. Trophic B. The ratio of plasma proteins. Myeloproliferative disorder C. BLOOD plasma has more than 90% of water Q268. The ciliary body of a person is damaged . Defensive function D. Which one is not a function of RBC? A. Accommodation C. 5-Hydroxytryptamine B. The substance associated with sleep is? A. Blood ph 7. 4:1 Q273. Dopamine D.which consists of luteal cells . Blood also called fluid of life. Oxytocin D. Testosterone Q270.4 normally D. Heparin Q271. All Q269. Polycythemia vera associated with? A. 1:2 C. GABA C.growth and health C. Blood grouping B.Q267. Protective D. Buffering action . Carbon monoxide poisoning Q272. In ovary of a woman in a period after ovulation are detected the bodies of spherical form . albumin : globulin? A. Chronic emphysema D. 3:2 B. Congenital heart disease B.what do produce these cells? A. A person in an accident has lot of blood lose. Increase Decrease B.Q274. Increase. Decrease Increase C. pulse rate and blood pressure? A. No Change D. Decrease . Decrease . Increase . Increase E. Aortic Stenosis D. Both C & D Q277. Aortic Stenosis D. Decreases B. Stops D. Which is unlikly to b associated with carcinoma of bladder? A. Aortic Regurgitation Q278.Q275.Regurgitaion B. Macrophage C. Increase C. Mitral Stenosis C. Aortic Stenosis Q279. expression of A B H antigen C. Tricuspid valve disease D. plasma cells D. expression of T antigen Q282. Mitral Stenosis C. A condition which is characterised by triad of cardinal symptoms Angina. No change Q276. Varicella C.Regurgitaion B. what is the first line of defenceagainst infection? A. Chlamydia trachomatis D. Aortic Regurgitation Q281. Treponema pallidum B. Austin flint Murmur are seen in which condition A. monocytes . Mitral valve disease C. Mitral. Aortive valve disease B. Pulmonary Stenosis E. Breathlessness and Synocope A. If carbon dioxide increases in the blood then respiration rate? A. hcl elaboration B. Mitral. Mitral Regurgitaion B. Causative agent of syphilis? A. Neutrophill B. Graham steell Murmur are seen in which condition A. Associated with carcinoid syndrome A. Mitral Stenosis C. Diplococcus pneumonia Q280. 5 μm. 3 µm D. Which one is not α-adrenergic blocker drug? A. excudate cause can be differentiated from transudate by measuring A. 7. 7. 5 µm B. All of the above Q286. U. 5 µm .Q283.S E. Alkaptonuria B. B9 Q287. Protein con B. Paracentesis Q289. 1 µm . REM E. B6 D. Cystinuria C. Fabry's disease D. 30 µm. B2 C. Beta rhythm C. Delta waves D. Tolazoline C. B. Tyrosinemia . Beriberi disease is caused by deficiency of what? A.5 µm C. which of the following feature is characteristic of slow wave sleep? A. The synthesis of the red blood cells D. Darkening of urine on standing is associated with A. 9 µm Q288. Propranolol Q285. The functional role of vitamin B-12 is? A. B1 B. Atipamezole D. SAAG C. involved in the metabolism of every cell in the body. The diameter of blood capillaries and RBC is (in series)? A. Protein + SAAG D. Profound loss of muscle tone Q284. The vitamin B12 combines with the folate in the formation of DNA C. Phenoxybenzamine B. Dream B. In case of ascitis . Nasopharyngeal carcinoma C. Referred pain in the ear is commonly from: A.Q290. Maxillary carcinoma B. Carcinoma tongue D. Malignant otitis externa . Widal test B. Tourniquet Test Q295. Ig M D. Xanthelasma were noted around both eyes. Normal C3 C. Ig D Q296. Submental nodes B. Submandibular nodes C. She has a long history of Rheumatoid Arthritis. 11 and 16 C. Test for typhoid? A. Juguloomohyoid nodes Q297. T8 B. Decreased C4 D. 22 and 9 B. Schick test D. S2 E. Ig A B. On examination. Ig G C. Decreased C3 B. What is the most likely diagnosis? Q294. 5 and 21 D. T12 D. Aortic opening is at the level of: A. Jugulodigastric nodes D. Type II Cryoglobinemia is characterized by? A. T10 C. joint pains and pruritis.Q291. Mantoux test C. 8 and 14 . A 61-year-old old lady was referred with fatigue. chronic myelogenous leukemia (CML) is appears due to reciprocal translocation between chromosome A. At the time of birth presence of which immunoglobulin in neonate's serum indicate the intrauterine infection? A. Normal C4 Q292. Tip of tounge is drained into? A. S4 Q293. Night. Down syndrome B.this is? A.Q298. A child has trysomy of chromosome no.mental retardation . His chest X-Ray shows: CXR: opacification especially in apical segments What's your diagnosis? . Cough. malaise. sweating. Edwards syndrome Q299. 18. micropthalmia . weight loss and a contact history. micrognatha. Cat cry syndrome C. Patau's syndrome D. months of fever. A patient complains. Saddle joint Q304. IUCD Q306. Endocarditis B. Most useful contraceptive in a young unmarried female A. His electrolyte report shows hyponatremia. None of above Q308. Ellipsoid joint D. Hyponatremia with hypervolumia C. Clinical parameters are more important in assessing severity C.Q300. Carcinoma C. Glucoma Q302. None of these . Finally diagnosed as acase of CCF. OCP C. POP B. Hyponatremia with euvolumia D. Levodopa C. Hyponatremia with hypovolumia B. Beta adrenergic receptor blockade D. Administration of atropine C. Leukemia B. Pulmonary av fistula C. Sarcoma D.Name the type of hyponatremia in this patient? A. Vagotomy B. Aortic dissection Q303. tachycardia. Which one is not cancer A. and right lower quadrant abdominal tenderness and guarding. Tricuspid atresia D. Always causes rectal bleeding Q307. It never cause malena D. Regarding Iron poisoning. Plasma level always determines the severity B. Knee joint is a type of A. Condylar joint C. Intrinsic heart rate can be determined by: A. IV administration of atropine and atenolol Q301. Selegilin D. Hinge joint B. Anticholinergics B. A 78-year-old is noted to have fever and chills. select true statement? A. Diaphragm D. decreased mentation. Parkinsonism is treated by A. A 65 years old male patient presented with exertional dysnea and pedal edema. Which of the following is the most likely diagnosis? Q305. Digital clubbing is seen in all except A. Salisylate poisoning is self aggravated by? A. He needs future followup. urine analysis confirms proteinuria. RBC B.HTN Q310. Rheumatic heart disease C. Which of the following feature is least characteristic of IBS? A. WBC C. Nocturnal diahrrea Q315. What is your advise for follow up for proteinuria? A. 06-12 Months C. 18-24 Months . LVD Q311. narrow pulse pressure and systolic murmur at aortic area. 900 mg/L Q313. What is the likely cause of this leison? A. O/E a house officer reported there is low volumic pulse. Urinary alkalization in salisylate poisoning is indicated at the concentratiion of ? A. Blood plasma D. Platelets Q314. Bicuspid valves B. Metabolic alkalosis D. Most common cause of cor pulmonale is A. Day time diarrhea D. P. 12-18 Months D. 700 mg/L D. But culture is negative for any growth and there is no any factor that requires renal biopsy. Respiratory acidosis C. COPD C. 500mg/L C. Constipation C.Q309. Metabolic acidosis Q312. 1-06 Months B. Pneumonia D. Enzyme Carbonic anhydrase activity found in? A. Abdominal pain B. Asthma B. Respiratory alkalosis B. 300 mg/l B. William's syndrome D. A 65 years old male patiet suffering from SOB. A 25 years old male patient presented with proteinuria. Omentum B. Appendices epiploicae D. Teania coli .Q316. Peritoneum C. The policeman of the abdomen is A. B. A. Diarrhea C. B1 C. Co enzymes FMN And FAD are derived from vitamin A. Digenic D. One of These following is a feature of coeliac disease: A. Metabolic acidosis C. Positive antiparietal cell antibodies Q321. Normocytic anaemia. Vertebral venous plexus D. Hypercalcaemia. Respiratory alkalosis D. Hypertrichosis is A. B6 Q323. All are false Q318. Spinal cord C. Alzhiemers disease C. except: A. C. Vertebral artery B. Horse shoe kidney F. Hypoalbuminaemia. A B. Chest dircomfort with palpitation Q319. Sex influenced character Q320. AML D. Foramen magnum transmits all.Q317. Respiratory acidosis B. All are associated wid Down’s syndrome except A. Metabolic alkalosis Q322. B2 D. Synocope E. ASD B. X-linked C. Nausea & vomiting B. Profuse sweating D. D. Select the more common association in inferior myocardial infarction. Holoandric B. Hypothyroidsm E. Tetany more commonally develops in? A. Spinal branch of 10th nerve . Purulant C. Serofibrinous . Most common type of pericarditis in acute rheumatic Fever is A.Q324. Serous B. Fibrinous D. CT C. Which one is not anticoagulant ? A. Radiotherapy C. < 2 cm Q332. In symptomatic mitral stenosis. Nervous system Q328. Atypical pulmonary TB C. Dot haemorrhages Q331. H pylori eradication therapy B.Q325. Millary TB Q330. Diarrhea Q326. Purirtus B. Methotrexate . Which of the following is typically the earliest lesion to develop in diabetic retinopathy? A. What should be the size of mitral valve ? A. USG D. Treatment for low grade gastric MALToma? A. Procedure of choice for evaluation of an aneurysm is A. Raised alkaline Phosphate D. Musculoskeletal D. Sodium oxalate D. Fever C. GIT B. Reactivation of upper lobe TB B. Which form of TB is less common in a patient of HIV with CD 4 count of < 200 cells /mm? A. < 4 cm C. < 3 cm D. Jaundice C. Arteriography Q329. < 6 cm B. Sodium citrate B. Respiration E. Blot haemorrhages B. Extra pulmonary TB D. Most common system affected in Whipple's Disease? A. EDTA C. Chemotherapy D. MRI B. Most commomn Intial complaint PrimarY BIlarY Cirrhosis Is A. Surgery Q327. Plasmodium B. Rotavirus D. None of the above Q334. Chagas disease D. Coagulopathy . Trypanosoma C. Losartan+ramipril E. Application of CPR with 15:1 D. Flukes C. Schistosomiasis Q336. Bladder cancer is associated with? A. Application of Shock B.upended a doctor establishes the presence of air . Yersinia pestis B. Amlodipine+enalapril B. Trypanosoma Q338. Amlodipine+lisinopril Q340. Wucheria malae D. Cardiac C. Cervical tear C. A patient has twilight vision impairment. Which one is not antihypertensive combination? A. Commonest cause of Post partum hemorrhage is? A. Drancunculiuasis Q337. B-1 C. Elephantiasis is caused by? A. Vaginal lacerations B. During the roentgenologic inspection of stomach of a patient . Pyloric Q335. Oncoceriasis E. A Q341.which part of stomach is it in? A. Fundus D. In Non-shockable rhythm first step in the treatment is? A.Q333. B-12 B. Loasis B. Salmonella Q339. Mycobacterium D. Wucheria bancrofti C. Amlodipine+atenolol C. Cestodes B. Pathogen of sleeping sickness A. C D. Doxapram+pemoline D. Pathogen of plague (black death) disease? A.what vitamin deficit takes place in the organism? A. Salmonella C. of CPR with 30:2 C. Body B. Nephrogenic diabetes Q346. HPV 16 B. OC. cervix? A. Abnormal shapes of heart A. Pons B. Von Willebrand's disease D. Muscles of the body B. HPV 35 Q348. Money bag heart-pericardial effusion B. Ureterocele D. Hydronephrosis Q349. Which is the Autosomal Dominant Disorder? A. In IVP. which is most commonly associated with ca. Universal donor blood group A. Diabetes mellitus B. Diseases which result in Charcot’s joint (neuropathic joint) are: A. AB+ D. ABQ344. Snowman heart-TAPVC(supra cardiac) D. Alkaptonuria C. Adder head appearance is seen in A. Horse shoe kidney C. HPV 33 D. Cerebellum Q347. Muscles of the internal organs and of the extremities Q343. Polycystic kidney B. Medial group of nuclei of the anterior horns of the spinal cord are innervate: A. Galactosaemia B. Uterine Atony Q342. Leprosy D. O+ B. Muscles of the internal organs C. HPV 18 C. Muscles of the extremities D. Egg on side-uncorrected TGA . Thalamus C. Boot shaped heart (coer en sabot)-TOF C. Which of the following is the commonest location of hypertensive hemorrhage? A.D. Amyloidosis Q345. Putamen/external capsule D. Tabes dorsalis C. Trochlear D.Q350. Aphasia B. coordination and comprehension is called: A. It is not associated with streptococcal infection of sites other than the pharynx C. It develops during the acute phase of a group A beta-hemolytic streptococcal infection of the throat B. Q353. The inability to carry out a motor task on command given adequate strength. Meningitis B. Sepsis B. The major cause of death in patients with fulminant hepatic failure is: A. Alexia D. Optic B. Bronchiectasis D. skin and nervous system . Variceal hemorrhage C. Meibomian gland B. Abducens C. Aprosodia Q355. Cerebral edema Q352. Gland of Zeis C. Hypokalemia occurs in A. Apraxia C. It is far more common in underdeveloped countries than in the United States E. Cranial nerve with largest intra-cranial course is A. Osteomylitis E. Which one of the following is not true about the development of acute rheumatic fever? A. Upper esophagus B. Mid esophagus C. joints. Which of the following is the modified sweat gland present in the eyelid? A. Its lesions involve the heart. sensation. It usually occurs during the course of epidemics of streptococcal throat infections in crowded settings D. Lower esophagus D. Trigeminal Q354. . Most common site of tear in Mallory-Weiss syndrome is A. Gland of Moll D. Hepatitis C. All of the above Q356. Cardia of stomach Q351. Q357. LP B. Nerve conduction study . In case of lower back pain with signs of spinal cord compression which of the following is the best initial diagnostic tool: A. Spinal x-ray C. MRI of the spine D. Which of the following structure is not palpable through vagina A.the genotype would be: A. Feature not associated with irreversible cell injury is: A. Retrouterine pouch E. Karyolysis C. Ureteric stone D. Mitochondrial shrinkage . Rectus abdominus Q363. XX B. Carbon Q360. Cervix B. Copper dust E. Rectum C. Karyorrehxis D. Mylein figures B. XYY Q362.Q358. External oblique B. Dopamine Q364. Prolactin D. Transverses D. Vesicouterine pouch Q361. XY E. Mitral regurgitation Q359. Vinyl chloride B. Internal oblique C. Hypertension B. In Turner syndrome. The commonest cause for atrial fibrillation is? A. Asbestos D. Adrenaline B. XXY D. Mitral stenosis C. XO C. Nor adrenaline C. Following doesnt form the layer of inguinal canal: A. Silica C. Mesothelioma is associated with: A. Tyrosine derivateve doesnt include: A. Thalasemia D.Q365. Sickle cell anaemia . Hereditary spherocytosis C. Cystic fibrosis B. Following is an autosomal disease is: A. Lewy bodies: Parkinson disease C. May cause pressure on brachial plexus E.Q366. Transmits Aorta D. Ribs and costal cartilages not attached to it D. GFR = Kf (HPG + HPB . Stress C. Open at the top B. Sweeting B. May cause thoracic outlet syndrome B. Which is incorrect: A.OPG + OPB) D. Hirano bodies: Alzheimer disease E. Excitation of post. All are incorrect Q368. Hunger pangs Q367.OPB) E. Transmits vagus nerve Q369.HPB . The formula for GFR is: A.HPB + OPG .OPG . Thin plane of bone E. Regarding cervical rib. Regarding Xiphoid Process Incorrect Answer: A. Transmits thoracic duct B. Lipofuscin granules: Aging B. Negri bodies: Rabies D.OPG .pituatary would cause: A. Closed at the top and the bottom E. ossified at proximal end B. Contracted throughout its length Q372. GFR = Kf (HPG . GFR = Kf (HPG . Lies at the level of T12 C. ossified at 35-40 years age C. Shivering D.OPB) . May articulate with the first rib Q370. GFR = Kf (HPG . Open at the top and the bottom D. Oesophagus at rest is: A.OPG + OPB) C. GFR = Kf (HPG + HPB . Present in 5% population D. which is incorrect: A. Regarding Aortic opening. Pigments and inclusions: A.HPB . Neurofibrillary tangles: Alzheimer disease Q371. May have free anterior end C.OPB) B. Open at the bottom C. Sinus tachycardia Q379. K 4.0. Cobalt E. V wave C.5.0. 30% D. 5% B. and several other peptidases: A. Peak left ventricular (LV) volume corresponds with (or correlates best with): A. K 3. The atrial component of ventricular filling: A. Na 150. 80% Q375. Compensated for by increased respiratory rate D. Exchanged for Cl across the red cell membrane C. Ca C. Y descent Q374. Osm 260 D.Q373. P wave flattening D. Zinc D. lactic dehydrogenase. Hartmann’s solution contains no: A. A wave B.0. Mg D. Na B. Intracellular proteins Q376. ECG changes in hyperkalaemia include: A. Osm 320 Q377.0. X descent E. K 3. Na 130. Osm 260 B. Osm 320 E. K 3. ST depression B. The trace element that is an integral component of carbonic anhydrase. 10% C. Acute onset (4 hours) diabetes insipidus in an otherwise healthy person produces these biochemical changes A. Na 150. Lactate Q378. Magnesium B. 60 to 70% occurs intracellularly B. Buffering of a bicarbonate infusion: A. C wave D. Na 160. Copper . T wave inversion C. Manganese C. Osm 300 C. 50% E. Na 130. K 3. Bone marrow Q382. Pyrazinamide E. Which is not carcinogen? A. 08 . Buccal smear B. Phenothiazines D. 10 B. Submental Q384. Herps D.Q380.receptors EXCEPT: (they increase intracellular cAMP): A. All of the following histamine effects are mediated by H2. Blood E. If a malignant nodule present on the anterior 2/3rd of left side of the tongue. 25 neonates were born with tatenus out of which the mother of only 5 was Immunized for tatenus. Vasodilatation B. Right superior deep cervical E. WBC D. Quinedine Q385. 11 C. Ethambutol B. Left inferior deep cervical B. A pt of chronic renal failure needs renal transplant.whats da odd's ratio? A. Hep D C. Tachycardia Q381. EBV E. 07 D. Hep B B. Renal tissue C.for this reason HLA matching is required. Corneal opacity is caused by A.which of the following is the best site of sample for HLA matching? A. Out of 50 normal born babies mother of 35 received double immunization for tatenus during pregnancy. HP Q383. Gastric acid secretion D. Right inferior deep cervical D. The group of lymph nodes most likely to be involved first? A. Bronchoconstriction C. Chloroquine C. Left superior deep cervical C. The normal liver stores of vitamin K lasts for? A.Q386. 1 week C. 2 weeks B. 4 weeks D. 6 weeks E. 1 year . Which nerve is involved A. 2nd branchial arch D. Griseoflavin Q389. Inferior fascia of urogenital diaphragm is A. Chlamydia B. Gonorrhea D. Gynaecomastia is caused by all of the following except A. Ilioinguinal nerve D. Thymus originates from A. Syntocinon D. Ergometrine B. Oxytocin C. Digoxin B. Pudendal nerve B. Which of the following drug is contraindicated in heart disease & hypertension? A. Perineal membrane D. Androgen D. Shows reduced mobility C. Sacroiliac joint is most likely A.Q387. A patient with IUVD presents with Pelvic discharge. Genitofemoral nerve E. Scarpa’s fascia C. Colle’s fascia B. Is supplied by external iliac artery D. Is innervated by dorsal column of S1 & S2 Q393. E. Trichomonas . Estrogen E. Endoderm of 3rd branchial pouch C. on P/R examination there is a posterior midline anal fissure. A female presents with painful defecation.Coli C. Deep external pudendal nerve Q391. Cimetidine C. Perineal nerve C. Ectoderm of 3rd branchial cleft B. Endoderm of 2nd branchial pouch E. Synovial joint B. Camper’s fascia Q392. Most likely diagnosis is A. Endoderm of 4th branchial pouch Q390. Progesterones Q388. Sulfonamides E. Methotrexate C.Q394. 6.mercaptopurine B. Vincristine D. Trimethoprim . Leucovorin is used to prevent side effects of A. Epiphyses C. Dunches dystrophy C. Mebendazole Q396. Which collagen gives strength A. Leukoplakia D. Selective B blockers cause side effects except A. C. Perfringens C. Osteosarcoma shows abnormality in A. Urinary retention Q399. C. Tetani D. Type 3 D. Cystic fibrosis B. Increased uterine tone C. Metronidazole C. Diaphyses D. Ulceration Q401. Which drug was she prescribed A. Ciprofloxacin D. Botulinum Q398. C. Chewing betal nuts& pan can cause A. Pregnancy without intake of iron & multivitamin B. Penicillin G B. Hook worm infestation Q397. Type 2 C. C. Epicondyles B. Bronchspasm B. Difficle B. Beta Thalassemia D. develops bad taste.Q395. Hereditary spherocytosis Q402. Type1 B. A patient with watery discharge is given a drug after which pt. Type 4 Q400. headache & then develops white curdy discharge & itching. Autosomal dominant disease A. Ileum resection C. MCV is least affected by A. Corynaebacterium diphtheria E. Metaphyses . B. Keratitis. Submucosal fibrosis C. Pseudomembranous colitis is caused by A. Furosemide B. Most Important use of ECF A. Pancreas C. Sq cell ca B. They r killed under 121 degree for 15 min in autoclave Q410. Drug used in acute pulmonary edema also causes smooth muscle relaxation A. Electrolyte exchange C. Sarcoma Q408. Patient came in ER with severe chest pain of 4-6 hours associated with sweating & breathlessness wat investigation would u do A. Papillary D. Anthracis producs spores C. Verapamil D. Guanithidine Q405. Cell nutrition . Drug causes tacycardia in ordinary usual doses A. Adeno carcinoma C. Anaplastic Q409. CPK C. Excitation passes from atria to ventricles directly Q411. Which is not an epithelial tumor A. Thiazide diruretic C. Excretion of CO2 from body B. Left atria is 3 times larger than right atria D. Whats correct regarding heart? A. Acetazolamide Q406. Colon Q407. Which is false about spores produced under A. Atria and ventricles contract simultaneously B. Female pt developed pain in epigastrium especially after having fatty food which organ causes pain? A. Propranolo C. Prazocin E. Which is most slow growing malignancy of thyroid A. Medullary C.Q403. Gall bladder B. Diltiazam B. CK B. Myoglobin D. Left atria is 1st to contract E. Follicular ca B. Tetani produes spores D. Nutrient rich conditions B. Trop T Q404. LDH E. Contraction starts from left apex C. in dorsal column lesion which of the following will be lost: A. Itch E.5 ml/dl the most probably yours diagnosis is : A. A patient is anxiety . Hamartomas B. Cushing disease Q415. Cometidine D. Triglycerides are absorbed in : A. Which is p450 inducer A. Vasodilation D. Ketoconazole Q413. Paramesonephric duct remnant in male is: A. Rapid pulse C. Phenobarbitone C. Common feature in nitrates. Postural hypotension B.isoprenaline and throphylline is : A. Internal haemorrhoids . Ovary Q418. palpitation. Scopolamine C. Venodilation Q417. Hyperthyroidism C. his serum sodium is 146 ml/dl.wt loss. Isoniazid B. Pain B. Appendice testis B. VLDL C. Temp C. LDL Q419. Metclopramide Q414. Appendix C.Q412. HDL D. Crude touch D. Drug used for mountain sickness A. Addison disease B. Proprioception Q416. Juvenile polyps are also called : A.k 4. Rectal polyps C. Chylomicron B. Acetazolamide B. Q420. Insulin E. PTH .lipids n protein in body : A. Hormone that stores carbohydrates . Growth hormone B. Cortisol D. Thyroid hormone C. Increase reabsorption at PCT C. Metabolic alkalosis B.Q421. Ileum C. Oliguric renal failure C. Frequent adverse effect of cimetidine is : A. SLE Q424. Defect in PCT B. Inhibits hepatic metabolism/enzyme B. Jejunum D. Dec BT C. Colon E. Dec PT D. It decrease pco2 in alveoli Q427. Maximum absorption of short chain fatty acids produced by bacteria occurs at A. Decrease Tm of glucose D. Dec platelets Q422. Hyponatremia is the feature of A. Monocytes E. It increases Po2 of blood D.her bld sugar is 100mg/dl. It activates neural mechanism which remove acid from blood B. Dec FDF B. Increasing alveolar ventilation increases blood PH because A. Neutrophils B. His blood workup will most likely show increase: A. Following is not a feature of DIC: A. Anti-estrogenic effect D.it's due to A. A pregnant lady comes to you with glycoseuria. Eosinophils C. Basophils Q423. Hypertension E. Lymphocytes D. Duodenum B. Decrease in salt intake Q425. Dec thrombin time E. Stomach . A patient presents with itching (and some other symptoms) on workup found to have intestinal worm infestation. Agranulocytosis C. Make Hb stronger acid C. It increases muscle work of increased breathing generates more CO2 E. Defect in DCT Q426. Patient having 28th week gestation. what is cause of odema A. Hypoprotenemia E. Supraduodenal artery B. Pulmonary trunk D. Increase interstitial fluid B. Nissl bodies B. All of these supply the first 2 cm of the duodenum except : A. Septic shock C. Heme binds with which protein? A. Esophagus C. Mitochondria Q432. Increase colloid osmotic pressure D. Microfilaments C. Microtubules D. Trachea B. Increase hydrostatic pressure C.Q428. Toxemia of pregnancy Q430. Hemopexin D. Surfactant C.developed BP 180/100mmhg. Gastroduodenal artery D. Azygos vein . Superior pancreaticoduodenal artery Q434. Pneumocytes B. Hemorrhagic shock Q429. Macroglobulin Q433. Neonates are deficient in which cells A. Heptoglobin E. Hypovolemic shock D. G-ve bacteria produces shock A. Eosinophils Q431. Axon of a nerve terminal doesnot contain: A. Lymphocytes D. Which one of the following form the posterior relation of left atrium: A. Toxic shock B. Common hepatic artery C. Globulin C.and odema. Albumin B. Q435. Glucagon C. Insulin B. Somatostatin D. Delta cells of pancreas secretes : A. Pancreatic polypeptide . FSH C. Last mediater of endometrial shock? A. Lipopolysacharides Q440. IL2 B.WOF is most likely the culprit A. Corpus luteum is maintained by : A. Hydatid cyst Q439. Eosinophil B.coli B. LH B. Ant: group B. Dead fetus C. Fludrocortisone is effective in autonomic neuropathy E. Progesterone Q437. Insulin is effective in improving diabetic amyotrophy B. Antioxidants are known to improve peripheral neuropathy D.a female developed hospital acquired wound infection. Proteus E. E. Lateral part of breast supplied by A. Estrogen D.aureus C. In diabetic neuropathy A.Q436. Gabapentine is effective in painful neuropathy Q438. Central group . Neutrophil D. Basophil C. TNF alfa D. Lat: group D. Dystrophic calcification is seen in all of the following except: A. Strep. IL6 C.pneumonia D. Staph. Malarial parasite B. Amitryptiline is an effective drug in painful neuropathy C. Macrophage E. Rods Q442. Which cell contains rhodopsin pigment? A. After an elective cholecystectomy. Psuedomonas Q441. Post: group C. Interrelationship among pressure . Angiosarcoma B. 1:180 for O and H antigen E. Sup lingual segment of left upper lobe E. Anterior apical segment of right upper lobe B. Now he cnnot raise his arm above his shoulder & and also has sensory loss on lateral surface of arm. Dopamine exerts its effect when it reached a steady state . Sarcoma botyroides Q448. 9 min D. 15 min E. More than 20 min Q446. Increase PCO2 & dec of PO2 stimulates sympathetic activity Q445.if its half life is 2 min. A Pt has fracture of surgical neck of humerus. 4 min C. Stimulation of lower area of vasomoter center increases the caliber of artery C. Apical segment of right lower lobe C. Which of follwng nerves damaged? A. Widal test is positive? A. Ewing's sarcoma E. 2 min B. Ulnar N Q444. Neuroblastoma D. Radial N D. 1: 80 for O antigen B. Stimulation of sympathetic fiber inc blood flow in capillaries E. 1:80 for H antigen . Axillary N C. Wilm's tumor C. Inferior lingual segmnt of left upper lobe D. Musculocutaneous N B.Q443. Aspiration pneumonia after aspiration involves which part of lung? A. Stimulation of parasympathetic fiber reduces vascular resistance D. Which tumor is not seen in childhood? A. 1:180 for O antigen C.flow and resistance is called hemodynamics B.after how many minutes it will show its effect A. Regarding hemodynamics of blood flow all of the following are true except A. 1: 100 for O antigen D. Lateral segment of right middle lobe Q447. Van Hoorne's canal E. Left lymphatic duct B. Chyliferous duct D. It is also known as the A. In human Anatomay the thoracic duct of the lymphatic system is the largest lymphatic vessel in the body.Q449. Alimentary duct C. Pecquet duct . Atrial Fibrillation C. Digoxin is drug of choice in A. Esophago gastric junction competence is maintained by A. Carcinoid syndrome D.Q450. Mesoderm C. Trigeminal Q451. Which one of the following is the largest cranial nerve? A. Testicular teratoma Q452. Atrial flutter B. Incresed intra abdominal pressure C. Ectodem Q453. Ig G B. Bladder originate from A. Ig E Q456. Facial B. Abduction n medial rotation B. Which of the following would be expected with an alpha protein concentraion of 300mu/L? A. Use of morphine . Lying supine B. Ig M C. Hepatoma B. Diaphragm paralysis D. Vestibulocochlear D. Action of gluteus medius n minimus is A. Endoderm B. Adduction n lateral rotation Q455. Cirrhosis of the liver C. Ventricular Tachycardia D. Occulomotor C. Ig A D. Use of metoclopramadie E. Bradycardia Q454. Colonic carcinoma with hepatic metastasis E. Adduction n medial rotation D. Abduction n lateral rotation C. Which crosses placenta A. Beta blockers . H2 blockers D. PPI C. Antacids B.Q457. Serum gastrin levels are incresed by prolonged use of A. Anticholinergics E. its 1st visit to a doc. Damage to optic radiation D. Insulin B.200 Q462. Aldosterone causes A. Upper part of anal canal is A. What shold be the treatment? A. Drained by inf rectal vein D. Hyperkalemia B.120 D.no complication. After obstruction in both Superior and inferior vena cava. A diabetic pt with BS 20mmole/L . Sensitive to pain Q463. Biguanides E. Compression of optic nerve B. Compression of upper part of optic chiasma E. Drained by sup inguinal nodes C. Internal memory . Thoracoabdominal D. Compression of oculomotor n Q460. obese. 160. Hypokalemia E.160 E.Q458. Hemiazygos C. 40. It’s due to A. 60. Biguanides +sulphonylurea Q461. 120. Acidosis Q459. Sulphonylurea D. Normal fetal heart rate A. Damage to optic tract C.100 C. 100. suffering from homonymous hemianopia. Azygos B. alternative route for venous drianage will be? A. Hyponatremia C. Insulin + sulphonylurea C. Lined by stratified sq epithelium B. Hypercalcemia D.60 B. A female having very high levels of prolactin. Sensitive to touch E. Q464. Herpes .most common invading organism is A. An old lady with red denture sores. EBV B. Staphylococcus C. Streptococcus D. Chlamydia E. Tyrosine B. Type 3 hypersensitivity reaction Q469. A patient suffers from appendicitis. Arginine C. Aspartate D. Blood transfusion reaction D. Dopa B. In heart speed of conduction is maximum in A. Carotid sheath D. Threonine F. SA node E. Abnormal thermal sensations B. Interatrial pathway C. Parasitic infection C. Numbness D. Which is present in slow wave sleep A.most prominent examination finding of sensory system? A. Axillary sheath is formed by which part of cervical fascia A. Ventricles . AV node B.Q465. Peripheral blood shows leukocytosis. IL 1 and TNF alpha B. Reduced kinesthesia Q466. Glutamate E. Valine Q468. IL 6 E. Thalamic hemorrhage. Prevertebtal layer Q467. Pretracheal fascia C. Histones are rich in which amino acid A. What is the main cause of eosinophilia? A. Norepinephrine Q471. Serotonin D. Enkephalin Q470. Pain will be mediated by A. Serotonin D. Purkinji system D. Investing layer B.. Bradykinin and prostaglandin E C. Type 1 hypersensitivity reaction E. Acetylcholine C. Hyperaesthesia C. Drug allergy B. Increase in dietary intake of K C. Aldosterone increases B. Decrease in Na reabsorption by distal nephron E.Q472. Amount of K from kidney will decrease if A. Increase excretion of organic ions . Increse in distal tubular flow D.
Copyright © 2024 DOKUMEN.SITE Inc.